Sunteți pe pagina 1din 820

Objectives

To outline how hematopoietic subsets differ in


proliferative ability and differentiation state and
how this enables blood to function under steady
state and stressed conditions.
Hematopoiesis Overview
To describe how hematopoietic cytokines,
transcription factors and microRNAs control
Richard Steinman MD PhD normal blood development.
Steinman@pitt.edu
January 5, 2009 To give examples of how aberrant regulation of
cytokine signaling pathways, transcription
factors or microRNAs promotes leukemias or
myeloproliferative diseases

Blood as an ISP
Supplies oxygen, nutrients, molecular cues,
host defense to entire body (100 trillion
subscribers).
~9 pints circulate through 60,000 miles of vessels
each minute
Laminar flow, RBCs in middle, WBC and platelets
on outside to sense disrupted tissue, facilitate
diapedesis
Red blood cells squeeze through capillaries half
their width (expel nucleus prior to leaving marrow
to optimize fluidity, room for hemoglobin)

Sites of blood production


OXYGENATION
CLOTTING Figure 1-10
HOST DEFENSE
TISSUE REGENERATION

Innate
Immunity
(CFU-GEMM)

Acquired
Stem cell Immunity
(Common
lymphoid
progenitor)

1
Hematopoietic Stem Cells Stem Cell Transplants
Donors treated with G-CSF
This mobilizes stem cells to leave the
bone marrow for the circulation
Cells expressing the surface marker CD34+
are collected from peripheral blood
mononuclear cells using antibody columns
%live
CD34+ is enriched on stem and early
progenitor cells
CD34+ cells are injected into the patient
Settings: Bone marrow rescue post
1 2 3
chemoTx, immune disorders, sickle cell
transplant
anemia

cell type cell type % of WBC cells/mm3 range


WBC 100% 6500
lymphs ~35% 2500
seg neut ~60% 3500 1160-8300 b;
1700-8100 w
band neut 2% 120
Blood element homeostasis monocyte
baso
8%
2
480
120
200-950
0-200
eosin 3 180 1-450
RBC 5,000,000 3.7-5.2 E6
plts 300,000 133-333 E3

The number of circulating blood elements


is highly regulated

Classic Model billion cells per kg 0.51


0.14 1.95
Blood Ontogeny po
stm
ito
tic
(6.
2.7 5 da
ys)
myeloblast promyelocyte myelocyte

mitotic (7.5 days) 3.6

metamyelocyte

2.5

band neutrophil

neutrophil

Differentiation of Myeloblasts (myeloid precursor cells)


Source:
daley.med.harvard.edu Disorders include congenital, acquired neutropenia

2
Flt3,
Homeostatic neutrophil Cytokine support
of hematopoiesis
SCF,
TPO

regulation-a whole body affair IL7

IL3
GMCSF
Leptin
GCSF
Apoptosis of neutrophils IL7

GCSF
Epo
IL6 IL5 MCSF
GCSF
MP, DCs IL-23 T cells IL-17 Stromal cells

SPLEEN
MLNs MARROW
GUT
Based on Stark MA, Immunity 2005; apoptotic neutrophils: Raza et al. Arthritis Research &
Therapy 2006

Stress Hematopoiesis
Mechanisms regulating
Hypoxia: Epo increases erythroblast
survival blood production and their
Downregulates Fas-FasL in erythroblasts disruption in leukemia
Increases ischemic tolerance in tissues
Infection: GCSF mobilizes marrow
reserve

Flt3,
Cytokine support SCF,
TPO
of hematopoiesis
IL7

IL3
GMCSF
Leptin
GCSF
IL7

Epo
IL6 IL5 MCSF
GCSF

A proliferative cytokine (IL3) can inhibit the function of a


differentiative cytokine (GCSF)

3
SOCS
L

R e cep
ce
Re

JA
pt or
P

ST
or

AT
t
P

JA
K
ST
AT
P

P
P

P
SOCS Gene

NR K
SIE

PT
NUCLEUS

Cytokine receptors have a modular structure and their function is


determined by which proteins bind to them and where they bind

GCSF
Lung
GCSFR

Stat3

Liver
SOCS3

Genes Spinal
involved in cord,
neutrophil
function epidural
space
Croker, Immunity 2004
GCSF ACTIVATES SOCS3, WHICH TURNS OFF GCSF SIGNAL SOCS3 + SOCS3-

FLT3-ITD

SOCS

Question L
R e cep
ce
Re

JA
pt or

P
K

ST
or

AT
t

P
JA
K
ST

How do cancer-causing proteins imitate


AT

P
P

P
P

growth factors?
SOCS Gene
NR K

SIE
PT

BcrAbl, Src
oncoproteins

MUTATED (V617F) IN NUCLEUS


MYELOPROLIFERATIVE DISORDERS:
Essential thrombocythemia
Polycythemia Vera
Idiopathic myelofibrosis

4
Flk1,
Transcription SCL
AML-1, GATA2, megakaryocytes
c-myb, Hox 4
factors in bmi-1 eed, Hox 3, Hox erythrocytes
A10
hematopoiesis
Ikaros
neutrophils
monocytes
GATA1 GATA3 CFU-GEMM
Stem Cell
GATA1 Pu.1 B cell
Myb requires lineage
FOG1
GATA1
SCL
Cebp-a
GATA2
SCL
Pu.1
Pu.1 T cell
Gfi1
EKLF Rb Egr-1
lineage
=scl SCL translocated into T-cell
receptor locus. Is aberrantly
expressing turned on during T-cell
1.Cebp-a 1.GATA2
cell development.
2.GATA2 2.Cebp-a

Leukemia

Proliferation

LEUKEMIA MicroRNA overview


Communication
Goal:
Differentiation 22-bp noncoding RNAs
therapy of
leukemia

Movement Differentiation Up to 1000 in humans


Transcription
factors needed for
differentiation (eg
Regulate >100 transcripts each
RAR, Cebp-a are
mutated in
leukemias
Block translation or degrade RNAs

Apoptosis Regulate up to one-third of coding transcripts

MicroRNAs in
Hematopoiesis

SDF-1 Chemokine

CXCR4 Chemokine
receptor

MiR 146a microRNA MiR150


Myb MiR223 MiR424
MiR155
PLZF
transcription
factor

5
Tumor and tissue clustering
efficacy of microRNAs
QUESTIONS?

Lu, Nature, 2005

6
Steinman -1

Hematopoiesis
Richard Steinman, M.D., Ph.D.
January 5, 2009
steinman@pitt.edu
412 623 3237

Hematopoiesis

One can view the hematopoietic system from a range of metaphorical perspectives, all of which
merely hint at its intricacy and dynamism. It is a salad of distinct cells, layered into a richly
textured topography within the marrow. It is a frantic dance of erythrocytes squeezing into the
bloodstream leaving their nuclei behind, of ribbons of platelets streaming out of lumbering
megakaryocytes, of phagocytes tugged forward by their ruffling membranes in a hunger-crazed
can-can. It is a tower of babel in which progenitor cells listen attentively for their siren cytokines
within the percolating chemical song. Unyielding and xenophobic, it is intent on the destruction of
foreign material or damaged cells; yet it also revels in the emergence of dissimilar lineages from
indistinguishable stem cells. And the hematopoietic system is a volcano of production, spewing
billions of new neutrophils during the span of this presentation alone. In short, the hematopoietic
system is a coordinated network of communication and response, movement, growth and death
i.e., it is an organ system. It is also an internalized ocean, coursing through 60,000 miles of blood
vessels each minute, conveying nutrients and messages to the 100 trillion cells of the body.

Nutrient exchange occurs through interstitial fluid, of which ~90% returns to the capillary and
~10% enters lymph capillaries. After coursing through lymph nodes and other lymphoid tissues
this fluid is returned to the subclavian veins via the right lymphatic duct or thoracic duct.

Functions of the blood include innate immunity, adaptive immunity, oxygen delivery and clotting.
The mature blood cells that effect these functions must be maintained in proper numbers under
normal, unstressed conditions. In addition, the production of blood cells such as erythrocytes and
neutrophils must be rapidly altered in response specific environmental stresses (stress, or
emergency hematopoiesis).

Objectives of this session are:

To outline how hematopoietic subsets differ in proliferative ability and differentiation


state and how this enables blood to function under steady state and stressed conditions.

To describe how hematopoietic cytokines, transcription factors and microRNAs control


normal blood development.

To give examples of how aberrant regulation of cytokine signaling pathways, transcription factors
or microRNAs promotes leukemias or myeloproliferative diseases

Where does blood come from?


In adult humans, blood is located within the vasculature, and in the axial bone marrow. Cells
important in host defense also reside in lymphatic and splenic tissue (lymphocytes), and within
tissue (macrophages). Although hematopoiesis takes place in the axial marrow in adults, it is
Steinman-2

sited in all marrow in children, and in the liver and spleen prenatally or in some diseases
(agnogenic myeloid metaplasia). Bone marrow is specially construed to support the proliferation
and differentiation of hematopoietic cells. It consists of a honeycombed latticework of venous
sinuses. The endothelial cells lining the marrow sinuses are bounded by fibroblast-like stromal
cells which generate a extracellular matrix which provides a nurturing microenvironment for
hematopoiesis to proceed. The hematopoietic cells are wedged between the vascular sinuses,
with megacaryocytes and erythroblasts clustered against the sinuses. Granulocytic
differentiation occurs deeper within the hematopoietic space. The close contact between
hematopoietic cells and the stroma facilitate transmission of proliferative signals or diffusion of
locally-produced cytokines.

Before there was bone marrow, there was the yolk sac. Primitive hematopoiesis takes place in
the yolk sac and thereafter blood is produced in the spleen and liver prenatally.

Blood cells and blood vessels go together like memory and the hippocampus. In fact, some
evidence indicates that they are made together--that both derive from a primitive mesenchymal
cell called a hemangioblast. Both blood vessels and the earliest blood cells express a receptor
for the VEGF growth factor; without VEGF neither develop. One report indicates that early blood
cells produce an endothelial growth factor called angiopoietin to help seduce blood vessels to
grow around them.

Stem cells
Hematopoietic stem cells (HSCs) and early progenitor cells sustain a durable blood supply. The
renewal capability of HSCs is dramatic in that a single HSC can reconstitute durable
hematopoiesis in an irradiated mouse; this reconstitution can be transferred to secondary and
tertiary transplant recipients (see figure below). Pluripotent stem cells are rare (10-5/mononuclear
cells) and give rise to all blood lineages

The most primitive blood cells are similar to vascular cells. Recent experiments support
the view that there is a common precursor for blood vessels and blood cells--a
hemangioblast cell.

Stem cells have been found to have several characteristics. They are:
pluipotent
lack differentiation markers
small
quiescent

It is hard to find and study stem cells. Two approaches are used to purify them. A surface
marker called CD34 is present on most stem cells and is lost in more mature cells. Using a
technique called flow cytometry, mononuclear cells can be sorted and cells which express CD34
and do not express CD38 (a marker of more mature cells). This CD34+CD38- population is
enriched in cells which exhibit stem cell behavior. Another approach is to select hematopoietic
cells which express the receptor for VEGF. This population of cells is very primitive, and may
represent some of the first hematopoietic cells to develop from a hemangioblast. These cells are
extremely effective in transplantation in mouse models.

Stem cells can also be selected based on their ability to survive in the presence of growth factors
and the cytotoxic nucleotide analogue 5FU. In these culture conditions, any cells which begin to
proliferate will die. Because early stem cells are deep in dreams of quiescence, they will be
enriched in surviving cells.
Steinman -3

Two types of functional assays have been used to identify stem cells. One is their ability to give
rise to mature lineages for a prolonged period of time in in vitro culture. The other is the ability of
the cells stably to generate multiple hematopoietic lineages in irradiated or SCID
(immunodeficient) mice. A hematopoietic stem cell is defined most strictly in terms of its ability
serially to reconstitute hematopoiesis in such mice. A stem cell loses the ability to do this after it
has self-renewed several times.

%live

1 2 3
transplant

Early stage blood cells enriched for HSCs by selecting the mononuclear cells for CD34+
expression are used clinically in bone marrow transplantation. This is done in settings such as
bone marrow rescue post chemoTx, immune disorders, and hemoglobinopathies such as sickle
cell anemia or thalessemias.

Currently stem cells are obtained from the peripheral circulation of donors treated with
Granulocyte Colony-Stimulating Factor (G-CSF).
G-CSF mobilizes stem cells to leave the bone marrow for the circulation
Cells expressing the surface marker CD34+ are collected from peripheral blood
mononuclear cells using antibody columns. CD34+ is enriched on stem and early
progenitor cells. The CD34+ cells are injected into the patient

Recently, there has been an increased appreciation of the plasticity of stem cells. That is, there
are cells circulating in the bloodstream which have the capacity to give rise to neurons, muscle
(smooth, skeletal, cardiac), intestine, liver, skin, and lung tissue. This new understanding is giving
rise to a discipline termed regenerative medicine. While still its infancy, this field is likely to have
an enormous scientific and clinical impact

What are some of the cells in the blood

Mature blood cells can be divided into several categories: lymphoid,


monomyelocytic, erythrocytic, and megakaryocytic. In some cases, the terminally differentiated
cells in these lineages can still be stimulated to divide-e.g. memory T cells. The terminally
differentiated myeloid cells--granulocytes--do not divide. Other final products in the circulation-
platelets and erythrocytes are anucleate.
Steinman-4

From the standpoint of host defense, the blood system contains two different sort of defenses--
innate immunity (eg granulocytes), and adaptive immunity (eg lymphocytes).

It is important to think of what cells do and do not divide, because only dividing cells can give rise
to leukemias. One cannot have a leukemia of a neutrophil, for instance, because it is a non-
dividing (eg post-mitotic) cell.

A 70 kg individual has roughly 5 liters of blood, of which cells comprise 2.2 liters are cells and
serum comprises the rest. The total volume of leukocytes (1.6% of cells) is about 40 ml, and the
total volume of circulating platelets is about 7 ml.

The final number of different mature blood types in the blood are highly regulated. A
characteristic count could be:

cell type cell type % of WBC cells/mm3 range


WBC 100% 6500
lymphs ~35% 2500
seg neut ~60% 3500 1160-8300 b;
1700-8100 w
band neut 2% 120
monocyte 8% 480 200-950
baso 2 120 0-200
eosin 3 180 1-450
RBC 5,000,000 3.7-5.2 E6
plts 300,000 133-333 E3

There are 1000 times as many RBC as WBC in the circulation.


However the ratio of RBC to WBC production is only 10-100:1.
The higher ratio of RBC:WBC in the circulation results from the longer survival of RBC in the
circulation.

Classical Model of Hematopoiesis

A classical model of how component cells of the blood arise describes a lineage tree in which the
differentiation of immature blood cells becomes progressively restricted and is irreversibleonce
forks in the lineage tree have been passed. This model, based on the studies of Weissman and
the observation of the composition of splenic colonies post transplant in mice is useful and
probably represents mainstream occurences in hematopoiesis. Recently, however, it has been
recognized that blood cells are more plastic, and instances of myeloid cells deriving from
lymphoid progenitors or mature cells (eg dendritic cells) arising from many different lineage paths
Steinman -5

have been described. It is not yet clear to what extent or in what environmental settings such
nonclassical events occur. Pending further exploration, we present the classical model.

How is blood cell production is highly coordinated to maintain circulating cell numbers within
certain levels and to respond rapidly to conditions requiring extra cells?

A lineage diagram has been devised to outline how cells are increasingly restricted in the types of
progeny which they can give rise to as differentiation proceeds.

Progenitor cells and precursor cells.


Cycling stem cells renew and give rise to more mature multipotent progenitor cells, which are
more restricted in the offspring which they will generate. This is associated with tremendous
amplification in cell number.

Progenitor cells

These cells are multipotent


They respond best to multiple cytokines
Steinman-6

self-renewal macrophage

Mitotic Post-Mitotic
erythroid monocyte
monocytic
1 3 CFU-M promono- monocyte
cyte blood
2 blast
CFU- CFU-
GEMM GM 4
CFU-G
pluripotent myeloid promyelo- myelocyte meta- bands/
cyte myelocyte granulocytes
stem cell
marrow

proliferation
differentiation
1-SCF, FLT3L, TPO, GCSF, IL6, ?LEPTIN
2-IL-3, GMCSF, IL-6 3-M-CSF 4-G-CSF
This is a compartment which expands the number of cells dramatically. Do not self
renew.

These progenitor cells are named by the types of colonies which they give rise to:
CFU-LM-Pluripotent cell giving rise to lymphoid and GEMM progenitors.

CFU-GEMM-Multipotent cell giving rise to granulocyte, monocyte, erythroid and megakaryocyte


colonies.

CFU-GM--Gives rise to both granulocyte and monocyte colonies.

Progenitor cells grow in specialized areas of the bone marrow, separate from stem cells.
Cells interact with the marrow microenvironment; colocalization allows modulation of
receptor signals by small amounts of cytokines.

Precursor cells
Blast cells committed to unilinear differentiation. Most responsive to one or two
cytokines. Still replicate until near terminal differentiation. Do not self renew.

More mature, and more limited precursor cells are:


CFU-G, CFU-M, CFU-E and CFU-Baso, giving rise, respectively to granulocytes,
monocytes, eosinophils and basophils.

Progeny of these precursors increasingly acquire specific differentiation markers and


functions.

Example: Myelopoiesis--formation of granulocytes.


Steinman -7

Order: Myeloblast, Promyelocyte, Metamyelocyte, Myelocyte, Band, Neutrophil.


billion cells per kg 0.51
0.14 1.95

2.7
po
myeloblast promyelocyte myelocyte s tm
it o
tic
(6.
mitotic (7.5 days) 5d
3.6 ays
)
metamyelocyte

2.5

band neutrophil

neutrophil

Example of differentiation from the Precursor Stage onward: Myeloblast to Granulocyte.

The stages of neutrophil maturation have been divided based on histologic analysis into:
Myeloblast, Promyelocyte, Myelocyte, Metamyelocyte, Band and Mature Neutrophil.

- Increasing development of primary, secondary, tertiary granuoles. Most granule


contents serve antibacterial or phagocytic functions

-Increasing phagocytic function. Neutrophil is extremely well adapted to the bacterial killing.
They surround microorganisms with pseudopodia which fuse to form a phagosome into which
granuole contents are released.

Mitotic and post-mitotic pool

Note that cells develop in two stages. Expansion of cell number occurs as cells in the mitotic, or
proliferative pool replicate. Myeloblasts, promyelocytes and myelocytes undergo mitosis and
belong to this pool. It consists of 2.6 x 109 cells/kg. Myelocytes give rise to a large number of
differentiated cells which no longer divide, but continue to mature into terminally differentiated
cells. This is the post-mitotic (maturation or storage) pool, consisting of metamyelocytes,
bands and neutrophils. This pool contains 9 x 109 cells/kg.. It takes roughly 7 days for cells to
traverse from pluripotent cells through the proliferative pool and another 5-7 days to complete
terminal differentiation.

It has been estimated that there are five cell divisions between the myeloblast and myelocyte
stage, and three cell divisions at the myelocyte stage. It takes roughly two weeks to proceed from
myeloblasts to mature neutrophils. It takes 5 days from the myelocyte stage until cells enter the
blood.
Steinman-8

In the setting of infection or stress, maturation time may be shortened, divisions may be
skipped, and cells may be released into the bloodstream earlier. This process has been called
emergency granulopoiesis.

A key concept : The marrow contains a large storage pool of neutrophils which can be reserved
for release in a setting of stress, and that the exponential expansion of progenitor cells can be
augmented by G-CSF under stress conditions. Granulocytes are short-lived (hours to days) and
therefore must be constantly replenished.

Neutrophils and Host Defense

Most are in the marrow--reserve

Circulating neutrophils, half in vessel, half adherent to wall

Diapedese in response to FMLP, chemotaxins


Steady flow of neutrophils into superficial tissue, skin, mucosa, lungs needed to prevent
infection.

Bactericidal mechanisms

Degradative enzymes in granules

Oxidative killing

Superoxide generation by NADPH-dependent oxidase


Involves hexose monophosphate shunt, cytochrome b
Patients with Chronic Granulomatous Disease lack cytochrome
b, cannot generate superoxide, and develop repeated infections.

Regulation of Differentiation and Proliferation


We will briefly discuss three factors that control the direction and pace of hematopoietic
differentiation. These are cytokines (proteins that bind to specific receptors on hematopoietic
cells and signal their survival, demise, or differentiation); specific transcription factors that turn
genes on or off in order to channel cells down a particular differentiation pathway, and microRNAs
that regulate what mRNAs are translated or maintained in cells. Current evidence suggests that
disruption of any one of these pathways-cytokine signaling, transcription factors or microRNA
expressioncan lead to leukemia.

Cytokines
The direction in which differentiation proceeds is influenced, in part by chance, and in part by the
cytokines with which cells come in contact. These cytokines support the survival and/or
proliferation of certain subsets of cells in the differentiation pool:

The progression from early, undifferentiated cells to terminally differentiated cells of distinct
Steinman -9

lineages requires a precise balance between Differentiation; Proliferation; Apoptosis; and


Movement.. The balance between these functions is largely regulated by cytokines. A fraction of
a percent change in the level of apoptosis or proliferation relative to differentiation can lead over
time to substantial aberration in the number of blood elements, such as neutropenia (low
granulocyte count) or leukoproliferative disorders including leukemia.

Progenitor cells require a minimum of two cytokines to grow effectively; committed precursor cells
will grow in response to a single cytokine. In some cases, cytokines have different effects on the
same population: for instance Stem Cell Factor will increase the survival of stem cells but will not
promote their proliferation; whereas IL-3 will promote proliferation and decrease the repopulating
ability of stem cells.

What is the clinical utility of these cytokines? Several of these growth factors have come into
clinical usage, such as G-CSF to improve the rate of recovery of patient's neutrophil counts after
chemotherapy or marrow transplantation. In some cases it is used to enhance host defense after
severe infection. Importantly, G-CSF not only causes expansion of committed myeloid precursor
cells, it also increases the sensitivity of cycling stem cells to other cytokines and increases their
entry into the cell cycle. G-CSF for those reasons has been effective in speeding the recovery of
normal granulocyte counts in patients who are neutropenic after chemotherapy.

One model of hematopoiesis proposes that specific cytokines induce cells bearing receptors for
that cytokine to develop along a specific lineage. There is certainly an additional level of control
involved in lineage commitment of cells which bear multiple receptors and which dogpaddle in a
pool of available cytokines.
Steinman-10

Cytokines signal through specific receptors.


MODULAR NATURE OF RECEPTORS--for a number of receptors, proximal cytoplasmic
domain is sufficient for proliferation; distal sequences are required for differentiation.

MOST CYTOKINE RECEPTORS LACK INTRINSIC TYROSINE KINASE ACTIVITY. SIGNAL


THROUGH CYTOPLASMIC PARTNERS, recruiting shc/ras; shc/PI3K; jak/stat pathway.
Steinman -11

SOCS

L
Re cep
ce
Re

JA
pt

P
K

ST
or

AT
to
r

P
JA
K
ST
AT

P
P

P
SOCS Gene
NR K

SIE
PT

NUCLEUS

The Stat signaling pathway is important in host defense

In the setting of severe anemia, stress erythropoiesis is triggered by the generation of


erythropoietin in hypoxic kidney tissue (via hypoxia inducible factor). The erythropoietin binds to
its receptor (Epo receptor) in developing erythroblasts. This activates Jak2, which phosphoylates
the Epo receptor (in particular at tyrosine number 343), leading to Stat5 activation and
accelerated formation of red blood cells, as well as increased survival signaling in the
erythroblast.

How do cancer-causing oncoproteins hijack the pathways of normal hematopoietic growth


factors?
Steinman-12

Master genes--modulators of normal and leukemic hematopoiesis

Another way of modeling hematopoiesis studies the expression of specific genes which strongly
control the direction in which cells develop.

Cytokines and leukemias: Leukemia has been associated with overexpression of cytokines (such
as IL3 or its receptor) of overexpression of intermediaries in cytokine signaling pathways such as
Stat5, Stat3 or src. The latter case is more broadly associated with myeloid leukemias in which
proliferative or antiapoptotic signaling pathways are constitutively activated. Overexpression of a
cytokine alone generally does not lead to leukemia because cytokines generally activate a
negative feedback loop.

Transcription Factors
Distinct genes have also been identified which are essential for the transition between
hematopoietic stages. Disruption of these master genes in mouse embryos is usually fatal
because of disrupted hematopoiesis. Most of these genes code for transcription factors--some,
such as pu.1 and C/EBP, appear to control the expression of genes for myeloid growth factor
receptors. The figure below sketches some of these genes and the lineages they control.
Absence of these genes results in the loss of the lineage under that gene's control. Artificial
expression of high levels of one of these master gene products can force a multipotential
progenitor to differentiate down a pathway controlled by that gene product.
Steinman -13

Leukemia and master genes.

Even though these genes are required for normal hematopoiesis, expression of one of these
master genes at the wrong time and place in the hematopoietic tree often results in leukemia.
This aberrant expression sometimes results from chromosomal rearrangements which lead to the
expression of the master gene product at the wrong time, sometimes as part of a chimeric (fusion)
protein with the protein coded for on the other chromosome involved in the rearrangement.
Examples include hox genes, AML-1, tal/scl.

Even though leukemia may only occur in one lineage, the derangement usually occurs at the
stem cell level. In one example below, the leukemic stem cell first made lymphoid leukemic cells
and then myeloid leukemic cells. The master gene which was abnormal was called scl (for stem
cell leukemia). This gene was essential for stem cell development and erythrocyte development.
Because a chromosomal abnormality, it began to be expressed in T cell progenitors and this
resulted in a T cell leukemia.

Many leukemias involve genes critical for early hematopoiesis being expressed in more mature
progenitor or precursor cells in which they should normally be silent. In many cases this allows
proliferation to continue, while the normal differentiation of those cells is blocked.
Steinman-14

megakaryocytes
erythrocytes

neutrophils
monocytes
CFU-GEMM
Stem Cell
B cell
requires lineage
SCL
T cell
lineage
=scl SCL translocated into T-cell
receptor locus. Is aberrantly
expressing turned on during T-cell
cell development.

Leukemia

MicroRNAs

Over the past two years, it has become evident that small, 22-bp noncoding RNAs called
microRNAs play a major role in differentiation, development, and cancer.

It is estimated that each microRNA controls the translation of hundreds of mRNAs in the cell,
controlling expression 1/3 of the proteins in the cell. By controlling how and when a subset of
gene transcripts are made into protein, microRNAs can alter the genetic makeup of a cell and
therefore its differentiation state.

Recent studies indicate that one microRNA (miR-223) is essential for the development of
neutrophils; another (miR-181) is required for B-cell formation. In contrast, two microRNAs (miR -
221 and 222) prevent the development of red blood cells.

Improper expression of microRNAs occurs in cancers. The precise expression pattern of the
many microRNAs in a leukemic cell accurately predicts the specific type of leukemia that it is. In
some leukemias, such as B-CLL, the pattern of expression of microRNAs has been shown to
predict how aggressive the leukemia will be (Calin GA et al, NEJM, 2005)
Steinman -15

Conclusions
The homeostatic regulation of the blood system is highly regulated and depends on the
expression of cytokine (or chemokine) receptors, exposure to environmental cytokines (or
chemokines), activation of transcription factors that control lineage transitions and the expression
of microRNAs that stabilize cells so that they sustain their lineage identity. There is considerable
interaction between cytokines/receptors, transcription factors and microRNAs, leading to a
complex network of positive and negative interactions that can fine tune hematopoiesis. In
addition, homeostatic maintenance of blood elements requires massive replication that is carefully
coupled to the rate of differentiation and that also takes into consideration the differing turnover
rates of different cells, and whether stress conditions dictate an acceleration in the production of a
specific lineage. Both local and whole body negative feedback mechanisms between mature
blood cells and the immature cells that form them have been shown to keep this balance.

Disease arises from improper numbers of specific cells (eg neutropenias, proliferative disorders)
or improper reaction of cells to environmental cues (leukemias). Disorders in blood flow arising
from loss of function (sickle cells devoid of flexibility) or aberrant function (coagulopathies or
hemorrhage) also compromise end organs. The host defense function of myeloid cells requires
proper numbers, proper response to blood-born signals, proper migration and cellular activation
and proper deactivation to end inflammation.
Learning Objectives

Red Cell Development and Understand the common origin of the cellular
elements of blood
Function
Understand growth factors involved
Roy E. Smith, MD, MS Understand Epo regulation of erythropoiesis
January 2009 Understand anatomy and function of RBC
University of Pittsburgh Cancer Institute Understand differential diagnosis of anemia
smithre@upmc.edu Understand normocytic anemias

Erythropoiesis Sites of Blood Cell Production


Sites of production Following fertilization
blood cells originate in
Components required for production yolk sac
9stem cells Early in utero,
utero, liver and
9marrow microenvironment spleen are sites of
production
9growth factors Children - all bones
Morphology Adults - axial skeleton
and proximal
extremities

Clinical Caveat Origin of Blood Cells


Myelofibrosis Common origin of all
One of the myeloproliferative syndromes (MPS) blood cells is
hematopoietic stem cell
Disorder in which marrow fibroblasts proliferate (HSC)
Blood cell production shifts back to extramedullary Common
(outside marrow) space granulocyte/erythrocyte/
Process is termed myeloid metaplasia monocyte/megakaryocyte
(GEMM) precursor
Liver and spleen enlargement diverges from lymphoid
precursor

1
Stem Cell Microenvironment Clinical Caveat
HSC home
home to marrow Aplastic anemia is a disorder characterized by
Bind to adhesion pancytopenia (RBC, WBC, platelets are low)
molecules expressed on Can arise due to injury to marrow
accessory cells microenvironment or damage to HSC
Are stimulated to Can be treated by HSC transplant or by
proliferate by growth immunosuppression targeting T cells (may
factors on glycocalyx directly kill HSC or by production of
inhibitory cytokines)

Erythroid Progenitors Visible Progenitors


Progenitors characterized by Pronormoblast
in vitro growth: Basophilic normoblast
Burst Forming Unit (BFU)-E: high Polychromatophilic
proliferative rate; insensitive to normoblast
erythropoietin (Epo) Orthochromatic
normoblast
Colony Forming Unit (CFU)-E: lower
Reticulocyte
proliferative rate; sensitive to Epo

Growth Factors Erythropoietin


Early multi-
multi-lineage growth 34 kD glycoprotein
factors Produced by peritubular
interstitial cells within
SCF
kidney (90% production)
IL-
IL-3 and liver (10% production)
GM-
GM-CSF Sensitive to oxygen content
of blood
Late, lineage specific
growth factor
In situ hybridization of cells
Erythropoietin (Epo) expressing Epo mRNA

2
Regulation of Epo Production Action of Erythropoietin
Cells producing Epo are Epo both increases
sensitive to oxygen proliferation of RBC
delivery development and
Oxygen delivery accelerates maturation
depends on blood flow,
ambient oxygen, release Process takes ~7-
~7-8 days
of oxygen by with one pronormoblast
hemoglobin generating 16 RBC
Epo action begins with
CFU-
CFU-E

Clinical Caveat Summary of Changes

RBC generally survival 100-


100-120 days Cell and nucleus size
shrink
1% of RBC are replaced daily Hemoglobin content
Patients with hemolytic anemias (eg, Sickle increases
Cell Anemia)
Anemia) have shortened RBC survival Nucleus is extruded to
(~21 days) produce reticulocyte
Reticulocytes remain
Patients with sickle cell anemia can increase within marrow 2-
2-3 days
the rate of production 5-
5-7X to mature

Synthesis of Hemoglobin Heme Synthesis

Heme synthesis Iron is transported to


developing RBC by
Globin chain synthesis transferrin (Tf
(Tf))
Binds to Tf receptors
Assembly of globin chains - tetrameric and is internalized
Iron can be either stored
structure ferritin) or used for
(ferritin)
heme synthesis

3
Heme Synthesis Clinical Caveat
Synthesis occurs both in Sideroblastic anemia
mitochondria and in Disorder caused by
cytosol ineffective production of
Final step is heme
incorporation of Fe Results in accumulation of
within heme ring iron-
iron-laden mitochondria
(ferrochelatase)
ferrochelatase) Sometimes responsive to
Concomitant with heme pyridoxine (cofactor for
synthesis, globin chains initial steps in heme
are synthesized in synthesis)
cytosol

Synthesis of Globin Chains Synthesis of Globin Chains


Two globin loci on each Hemoglobin switching
chromosome 16 are used globin chains are used
during fetal growth
Non- globin loci on
Non-
Within 3-
3-6 months of birth,
chromosome 11 are used globin loci are shut down
Globin chains are paired and loci are used
22 Hb F (fetal) Disorders affecting globin
gene may present during
22 Hb A (adult) fetal phase
22 Hb A2 (minor adult) Disorders affecting globin
loci (sickle cell) present
after Hb switching occurs

Assembly of Hb Tetramer Normal Blood Smear


Globin chains are paired Red cells are biconcave
Uneven production discs 8-
8-10 microns in
(thalassemia) results in diameter
precipitation of globin Central pallor (hole) is
chains 1/3 cell diameter
Interaction of globin Increasing pallor
monocyte lymphocyte
chains affects binding of suggests decreased Hb
oxygen to Hb platelet content (hypochromia
(hypochromia))

4
Function of Hemoglobin Interchain Associations
Transport of oxygen Globin chains interact
Facilitated by association of chains with one another using
hydrogen bonds (Bohr
Oxygen saturation curve effect) or 2,3 BPG
Modulation of loading and unloading by These bonds affect
2,3-BPG, pH binding and release of
9hemoglobin affinity oxygen

Oxygen Saturation Curve Normal Function


Oxygen binds to Hb in a Alteration in pH, CO2
facilitary manner content, temperature,
Oxygen binds to one Hb affinity, 2,3 BPG
50% saturated globin chain which
can affect binding and
at 28 mm Hg facilitates binding to other
chains release of oxygen
Generates a sigmoid-
sigmoid-shaped
curve
Arterial blood is 100%
saturated while venous
blood is 75% saturated

Shifts in Oxygen
Clinical Caveat
Saturation Curve
Acidosis facilitates -thalassemia
Defective globin loci results in
unloading oxygen decreased or absent Hb A
(alkalosis converse) Major hemoglobin is then HbF
HbF has high affinity for oxygen
In chronic acidosis (eg, good for fetus not good for post-
post-
COPD) generation of natal state
Oxygen sensor interprets low
2,3 BPG is decreased oxygen delivery and increases
shifting curve to left Epo
Increased Epo leads to expansion
(compensates) of marrow to compensate for
tissue hypoxia

Hair On End marrow expansion

5
Red Cell Membrane Abnormalities of RBC Proteins
Integral membrane
proteins
band 3; glycophorins
Cytoskeletal proteins
actin; spectrin
Linker proteins
ankyrin; protein 4.1 Elliptocytosis Spherocytosis

Clinical Caveat Red Cell Metabolism


Patients with hereditary Relies only on
elliptocytosis or glycolysis
spherocytosis have
compensated hemolytic Generation of 2,3-
2,3-BPG
anemias via Luebering-
Luebering-Rapaport
Spleen is site of RBC Pathway
destruction Generation of NADPH
Early gall stones (pigment via pentose shunt
stones)
Screening test is osmotic
fragility

Bite Cells
G6PD
(drug-
(drug-induced hemolysis)
NADPH is needed for Precipitated globin
regeneration of chains are removed by
glutathione the spleen/liver
Glutathione protects macrophages
RBC from oxidizing Drugs: sulfonamides,
agents anti-
anti-malarials
Oxidizing agents (drugs, G6PD deficiency is
fava beans) precipitate prevalent across
globin chains against Mediterranean, Middle
cell membrane East, India-
India-Arabian
peninsulas

6
Complete Blood Count (CBC) Routine Information Obtained
Measured values
9 Red cell number (~5 X 106/mm3)
9 Hemoglobin (g/dL)
9 Mean cell volume (MCV)
Values calculated
9 Hematocrit
9 Mean corpuscular hemoglobin (MCH)
9 Mean corpuscular hemoglobin concentration (MCHC)
9 Red cell distribution width (RDW)

Red Cell Distribution Width


Clinical Caveat
(RDW)
Information routinely Iron deficiency anemia causes a reduction in the
provided on CBC mean red cell volume (MCV)
Frequency distribution Folate (or B12) deficiency causes an increase in the
of cell volume (either MCV
normal or high) A patient may have concomitant deficiencies of both
High RDW suggests Fe and folate resulting in a normal MCV
populations of varying RDW would be increased to reflect populations of
volumes (anisocytosis
(anisocytosis)) both small (Fe def) and large (folate) cells

Reticulocytes Reticulocyte Kinetics


Measure of effective Reticulocytes normally
erythropoiesis Polychromasia or Shift Cell
circulate for 24h before
Storage pool within marrow they become a mature
Released into circulation RBC
where they complete
maturation within 24h With bleeding or
Requires special stain to hemolysis, reticulocytes
Reticulocytes may be shifted
shifted into the
delineate them
Can be inferred by circulation due to stress
polychromasia
polychromasia or shift These shift cells
cells may
cell
cell require 2 days to mature

7
Classification of Anemia
Correct reported reticulocyte count for
degree of anemia (corrected retic count)

[Patient Hct/Ideal Hct for sex] X reported retic count

Correct for reticulocyte maturation time


(presence of shift cells) (retic index)
Corrected retic count/2 (days to mature)

2 To Hypoproliferation or
Anemias 2
Etiologies of Anemias
Maturation Abnormality
The majority of anemias
MCV
are due to
inappropriately low
<80 fL 80-100 fL >100 fL reticulocyte response to
Aplastic anemia
the degree of anemia
Iron Deficiency Pure red cell aplasia B12 or folate deficiency present
Thalassemia Chronic inflammation Liver disease Chronic inflammation
Lead toxicity Renal or endocrine Myelodysplasia and iron deficiency
Sideroblastic anemia disorder Increased reticulocytes account for most cases
Infiltrative disorders

Anemia of Inflammation Anemia of Chronic Disease


Seen with inflammatory Hemoglobin 8-10 g/dL
states
Usually normal MCV (may be <80fL)
Inflammatory cytokines
suppress erythropoiesis Iron studies show normal ferritin and low
Induce state of relative iron binding capacity (transferrin)
resistance to Epo Etiologies: chronic infections, connective
Decreased iron re-
re- tissue diseases, cancer
utilization

8
Epo Response to Anemia Clinical Caveat
An expected Epo response
line was generated by
bleeding volunteers until
Epo supplementation is used for a variety of causes
they developed iron With renal failure (low Epo generation)
deficiency Without renal failure (relative resistance to Epo)
By checking an Epo level
Inflammatory diseases: RA
and correlating it with
degree of anemia, the Malignancies: myeloma; solid tumors
appropriate (or AIDS
inappropriate) response to
anemia can be seen

Review Review
Sites of blood production-
production-yolk sac, liver Hemoglobin synthesis involves separate
spleen, distal and axial skeleton pathways for the Heme ring and globin
Blood development comes for HCS and is chains
depends on growth factors and
Hemoglobin switching
switching--
microenvironment
EPO stimulates red cell development and Transport of O2-
O2- chains, O2 sat curve,
is O2 dependent-
dependent- may be used to treat 2, 3BPG, pH, hemoglobin affinity
anemia in some situations

Review
Red cell membrane structure-
structure-integral,
cytoskeletal,
cytoskeletal, and linker proteins
Abnormalities of RBC proteins-
proteins-HS, HE,,
osmotic fragility
RBC metabolism-
metabolism-G6PD
CBC-
CBC-measured vs calculated values, RDW
Reticulocytes-
Reticulocytes- RI, RPI, absolute, initial
classification of anemia (hypo vs
hyperprolfierative)
hyperprolfierative)

9
NOV-IB-200B TUE OB:57 AM FAX NO, p, 02

CHAPTER
t

Hematopoiesis:
The Development
ofBlood Cells
Stephen G. EmOTiOn

INTRODUcnON
All the cello observed in the adult perlpheral blood derive 1I'0n\ the 'he bene
marrow, through an extraordinary process termed lumtalo,Poims. Thi~ prQC(~S5 rC$ul til
In the malntenanee ora variety of blood cells, with each cbsH possclR'itl,g' uniq\lc rca-
LUres and diSjunct lifespnru, Ourcurrent understanding of lhi~ process i~ ernbodled
In a theory called lbe "em r.ellmodel a/hematopoiesis. TI1I. model fonn' 'he foun-
dation Ior aU our present thinking sbout normal \':Iematopoiesis, pSltlologic hema M

tologte disease ".,es. and hematologic therapies.

Stem Coli Theory. Unlike OTher dslue, of rnescderma! origin, which a


rule have little turnover in their comthuC'nt cell popul~t~iol)';, the cellular compo-
nents of the blood are con'13nLly undergoing cell death and replacement by new
celt. (Fig. 1.\). Where.. red blood cell. drculate for approximately 4 months,
rlatele"l.. ~ ollly about I week, and ~rlln"locy,o re vi.ble for Ie" than 10 hours.
It i~ estimated that ,.'very cby I X 101 blood cells are lost to wearand It:31'. and flr(~
replaced with an equal number of new blood cells. 1'0 fulfllllh. continual need for
"Ii
replacement blood cells, hematopClleau O(;C\l", actively throughout lif<:o a conse-
q ucnce. the blood-f()rmlng UgUCI are among the mostmitotically actlvc, al tIn" with
'hell""troin'esdnal epithelium. testes. and epiderml. The recognition of 'bi' hll';h
turnover role for blood cell. h.. ted to the development nf the stern cell theory of
blood cell development and maintenance, or hCmatopoic9ill, In the: course of this
chapter, we will examine thi5 theory ~n neWilil, as well ;L~ itl implicatioJl.'\i for cttnlcal
practice,

1
NOV-IB-200B TUE OB:57 AM FAX NO. P. 03

CHAPTER

Hematopoiesis:
The Development
ofBlood Cells
Stephen G. Emenon

, ','

,,\t!:,
:~tr{ lNTR.OnlJCT.ION
';, :~ .l:r";'
. ',,,/, All the cells observed in tbe adult perlpberal blood derive from ih.. rhe bone
.' '<'~, marrow, thmugban c-xtraordloaryprocel' termed h_lopokm. Tbis pro~, resu]"
.....\'},".
in Ihe malntenanee ofa variety of blood cellwith ellen cia" possesslngunlque fc~

ji;~
lures and dlulnellife'pan,. Oqr current understanding of Ihi, procel' iR embodied
In a thcotycalled lb. stem cellmodel ofheIMlOpoic.ls. Thl. model fonn' 111." foun-
dation for all our present thblking about normal hClllatopole'I', pathologic hema-
tologic dise... sllltes. and hematologic tberapie.

Stem Cell Theory. Unlike other ti"ue. of mesodermal origin, whlch a. a


. ,~\t>: (ii rule have little turnover in their constituenr cell populations, the cellular compo-
ncnts or the blood arc con'<antly undergoing cell death and replacement by new
cells (Vig, H). Where"" red blood cells clrculate for al'prDximotely 41110nth,.
platelets lastonly about I week, and granulocytes are vl2.ble for les' than 10 hours.
1L Is estimated that every day I X 10" blood cells are [0'\ to wear and te... and arc
replaced wirb an equal number of nowblood tells. To fulfill the continual need for
replacement blood celb, hemaLopoiealll OCtUI'$ ~ctlvety througllOUt liCe. As l\ COO~t1M
quence, the blood.fol'tlll~g tissue. are among the m.,.t mitotically a"ive,along with
th~ptroinwstimd cpithel~um~ I.t.lte~1 a.nd epldermis, "l'he recognition of this high
urmover rate for blood cell, ha.' led to the development of the stem cell 'heory of
blood cell development and maiutenance, or helOalOpole,ls. In the course of thl'
chapter. we will .lt~mine thl> theory in de~lI. as well., lIS hnpllca,lon, for eUnical
1'\":;'l r .t pr:lCu<:e,
I f~ :1r: ~;
! ;....

I';I~' 1''','
~ ",\ j I

l,,:;~~ ll'
FAK NO, P. 04
NOV-18-2008 TUE 08:57 AM

~ Platolol

IlS
Il

Il-6

IL2.ll-4
(I~Ymu.)

Unlll_1I! Motlll'/l
PIOIIOIIIlO, cell clrcul.tlng cell

'leu" 1.1. Hitt,uchir.aJ modol orhcmO\topolcsiJ andlym~hopolftlli4 induc1illM' !mpt.l113lll c)'lldn~,

Hematopoietic Growlb Fac:torll. One of tile


most exciting advances In the
"udy of hematopoiesis In the 1..1 decade has been the In..,stig-dtlon of the role of
hematnpolcriospccldepoll'l'eptid.. in the control of the 1''''00'' of blood eoU dle-
f"renli.tion. The", hematopoiotic honnon.~, also knOWt\ II hematopoietic growU,
rOCIOI'$, appear to control CV<l11' step in d,e proce.. ot the development OfUL'VI blood
cells, Atlhc prescnt tlme, we knowprincipallY'1ll01lp otsuch hematopoietic growth
factorsthat are Slimolaterylohernaropoletic "em cell. and their prager'Y' However,
some inhibitoryfactors havebeen dioco~cred lbat appear to play an e'l0any impor-
tant role in the negative contrcl nflhl. process, Other hormones (lullare felt to have
pl'imarily nonhOlO3l.Opoielic funcdoOi are ol,n known to lI!feet the prodllction of
new blood cell,. In IbiB chapter wewlll describeconci,ely the role of bollI 'dmula-
I<lly and Inhibitory hematopoietic growth lltcLors.

Hemotopn1eti<: Microenvironment. The term """"Iopoi,#. ml......viron-


men: refers to the "'lromal" elementsof the organ. in which hematopoiesis occurs,
that iI. the cellular And noncellularelements that do not directly Rive rise 10 the
blood ccll, but <hal provide. three-<llmenslonat structural matrix In which the
hematopoietic "em coli. and their progenyprollferateIIIId differenUateunLiI their
ID,gr-04lion into [he blcodsrream, The role ofthc: hemaeopoletic mlcroenvironment
in the control of the developmenl of blood celli I, felt to be of p:or,uoounlimpor-
ranee In the: ptOCC$S ofhelhl\topoie~ili.nom thestromal ct'lL~ rmd Uleir secreted ma-
trixproteins~Ip~~r ro il1Ilucn~e the proceAS of hCllJatopoie,i." as profoundly WI the
soluble, secreted hCI'"atopo~tic growth CM.clors, A sum1l'Qf)' of current understand-
ing of the contribution oflhe hematopolelic microrn~lronmenl will be Included in
this chapter,
NOV-18-2008 TUE 08:57 AM FAX NO, p, 05

J lI:MATOPOIESJS IN TilE EMBRYO AND n3TUS


Role of the Yolk Sac. The (erotiled egg develop. the beginning. of blood
11,,"e while 'till in the (mhryonle".ge (Fig.1-2).The finW.p reward development
of tht~ blood tissue i:l believed to occur in Lhe: yolk. sac. where undifferentiated cells
called mescblasts arc found and are thought to hevo nligrl'\\(!(l there fmm lhe prim-
itive strc;;tk of the embryo. TIUl mcsoblasts arc highly miLOdcally active and wilt sub-
SC,lq\ltntly differcutlate into cell~ called pnmititJ8trythrolJlas~1 which are clearly related
to the mature blood cells of the adult, ns well 1\.\ Into cells called pri."tili1J~ mulolJIl!lial
r.rll'i that Rive ri\c to vascular channels In the yolk sac. WiLhin hours after miltr:tlion.
. ,',~ th. yolk sac rncsobl..ts have generated prlmluve erythrocyteslhroUllh a pro'o" of
:,,,~:~ c-elldivision and differentiation. Most of these cells are nucleated, but, a minority arc
nonnucleated, and all s)'othcsiz.c hemoglobin and thu:cJ lend a reddl8h color to the

Ohromosorne 1115 ChrolnQtOm. 11


5' ", ( S' 5' 0 Gy Ay I ~ 3'
""
-"~ ~ '~:l\ I

(,K, (m "''0
PoI1llnd Ggwor2,
"1':1 lJif2 ",B, ""1l2
~
) Gcwor1 A
i L.!:.. "" I
A Embtyo Fetus Adult
Sll1I of E .. 101
,'.
'.'"
tlvI!

--
_-------:_~-- ~(Jf~ r'U'lv\\

l
'Il
50-

4 -
Volk &au
"
"'/
Spilln
"
~ --
1. PfH,l e.. V:1(QIlVr/IIf")
2. t:allr II: ClI~ (OOMll'~~
:1,
,.
I'; lao,,
.~
3. DOOftfI~9d or Wii~ 0: ~ fP'lII1IIM)
If. DeoAI4aedor.bMnIa;" U(.,
15. tlPfflIlN1I' Cli1,(flOl';naI HQfI'"
e. Dwc:nII/Idd Dr.bltllt~ (III.), ~, (101gb HJ
'r, Pa~Lt*'l1l norrNa!'lIlI.Jl (H9Il1ll
~ 20 8, l'\Illri.l.nil of ~A(~:.i/lIHQl1~
9. NormRI: ",6,OtgD ~
:a
~ 10 ~
Y
8
6 12 18 :14 30 16 8 12 11 2<1 :lO 36 42 48
R Bltttl
~." ... '-2. (A)The gJoVln rr=t'U:: "'w:ten.en chromOllOInCi 16 and 11.11'1 t:lnbryonic, rC\al, and adult
life, dil'fc:rcnt seReJi are acllva.twor .upprem:d. 'l'b.: dU....~nt globin cta.r.inl arc "'f'tuhcllloCd indef~cn
ttently end t1lt:n combine witll ell.ch othtsr tQ produce die dUrncn t hemogtobin&, The.., ~Il.. lI'UlY haVll
two JCquen~ dlff~rln8' by whtlthe "lhcrc iJ" I. gluLa mlcacidor en amnint t~idu.~:u polollion 1!6 (Goy or
Ay. rl)ll~ctivdy). (t'mm JiolJbmnd A.V. ruWtJr.. IumUaJ1lt:m4l~1ID, ~rd ed, Ce.mbritlcf', M~\~.: l'lIa('li,.
wall SclcnuRe P!,Jb\i.l.~lnll'; 199$,) (B) Di.'Velopnll!'nMI w:q\lcnce of lfl(;Mt<m of hcmlt(Jpnle.h and
hrmOflo~ln 5)'nlhe~is. The !olsl;o; conn~t:~ ~tlbtru; tha~ aoc.:4llll under nomW and Mbnrmnill tirmm"
't.l(lre'l. (.Afteor 1\IQWI) MS, F,kJ "1\iIl NrMlaW?rAropoirJit /1'1 D~uu fInt.l NMlt'llar JJ"ll4t~. Ni.'W
York: Llven rrt'Q; 1998. from li-.uldl" ru, Stuucl TP. Lm: SF., tld." BbnJlPrin,iplu lI,.d p""ttU tJj N~*"
~. Phil,do/phh,,lb Llpp'",oo, 19%.)
p, 06
FAX NO.
NOV-18-2008 rUE 08:57 AM

CHAPTER 1: [IEMATOrOIl:5rs, TUE DEV':LOrMJoNr 0,' nl.O<lD CIIl.L.5

<h"lIps of101k sac cells where blood cell' Ionn. The t1utllp' ofhe~oglobinI1~rl~ells
arc vi'ible to the naked '')'0, hence the name kllJlld islands to describe these localrt.ed
"ea. of embryonic hem.topol",i, in lh. 10lk c. Megakaryocytes, !he cclll which
produce platelets, arc oIso found In the blood i,l"nd. and are p""umably derived
fro,,\ the me'oblaaL', Other meaoblasts appear to dlfCercutiate into a type of ceil
culled The bemccytoblast,
A second stall" oChcmalopoicsil in the yolk sac occurs in the ernb~ or,ome
mammals In humun embryo', L1li' second Imge I. present but I, not as Vlgorou....
in rabbies The mammal III whkh the ernbryogen i, of hlood eel" has been mo,t ex-
tcnslvely studied, In L1,e second ~Lage of yolk sac h.maLopoie,i" h.mocycobla,lj; dlf
fercllliate Into definitive erytllroblMU. which Illbse'luenlly synthesize hemoglobin
and Me called tkfinl!iv.or ""rmdary nCl1'lllOl>l411f, 'lite latter may 10'" Iltdr nuclei and
become deflnitive erythrocyies Vascular channel, form ill the blood ISlan~ ~~cl
t:'v,~nlllalJy C('I,1I1CCt Co fOI'Ul a nelwork of blood vcs.;cls. This network nf primitive
blood vessels earlier contains the primitive erythrobla..u and hemocytobbsfA, and
later, definilive erythrobla", and erythroc)'le" 1Iythe end of the third week or em-
bryonic development in lhe ..abblt, all the hematepoletlc acLivit}' oCthe blood 1.land.
10as subsided and tile proc,," ofhem:llopoic'i' h.. shifted to the liver.

Tho EmblJ'Onie Body MtlIienehymc, An addidcnal rolc in early.mhryuni e


heml\t<>poie.i, i, played byprimitive mesenchymal cells ill l.he body cavity h."lf, par-
ticularly in tile autcrior precardiac mesenchyme, Small numbers of mesenchymal
cell, of the body cavil)' develop into eryll"obL'lSIS, megakaryocytes, grnnuloeyte'.
and phagocylie celh analogous to their counterparts in the adult. Quantitatively, the
number of cells produced iJ small, and large accretion. of blood cell. simil.r to the
e
blood i,land, nf the yolk do Mt form in the body cavity mesenchyme. Stem cells
localized among these non-yolk sac hematopcietic ceUs likely playa major role ill
the subsequent gcn era tlcn of Ceral and poslno'ill hematopolcsla, although the rel.-
live conu'ihutlons of yolk sac and non-yolk sac prrmhive stem eells to later
hell'litopoiC/d~ i~
uncertain.

Emergence of tho Uver ... lhe Prindpal Site of Hematopoiesis Jq the


Embryo. Beginning at around the l2-mm ,(;Ige of the humar embryo (6 weub of
age), bematopelesis hcgln~ to shift to Ihecmbryonlc liver (Fig. 12).1'h. Iiver sccn
becomes the dominant 'lite of hemacopolesls and remain. active in hePlatopoit.sis
undl birth. As lh. endodermal eorrll of the liver prlmordium gt'Ow Into the septum
rransvcrsum, they encounter wAndl!!ring mesenchymal cells wItb the morphologic
appea ranee oflympho,yt.o. Those small, round.lymphoeytOid cells called 9'/uJcy-
/oi-l ,valld""nguU"ul"eqncndy arc trapped between the prlmordlal llver endnder-
llIoil cordnd the endothelial cells ofinll"0wtng <"'pillorles, They give rise til hemo-
cytoblast' similar to tho of the yolk ,",c. These h.mocytilbla... ,oon Corm foci of
hem:\topQiesi.ll simil:ar [0 the blood IsJan(hi ot the yolksac, wherein secondary ety-
dlroblas'., are formed In large numbers. Secondary .rytIlrobl.... sLlbsequendy di.
vtde end differerulate into cl.fil1i~ve erythrocyt lhrough the progre..lve ac.qu"i-
lion of hemoglobtn and 10"' of the cell nucleus. Allhough delinltive ery!hl'OcyIcI
tn.y be ,e.n in lho liver., 6 wee'" of age. the')' do not emerge into d,e c1teulation
in any jp'e-at number until Inllch 1il.ICf, Thus bythe fourth fetal monLll, me majolilY
of circulating erythrocytes in dIe emhl)' 'dre k:condOl.I')' <definitive) erythroC)'t.t:s,

i
MegakaryoeylCsa1.o appear 10 form from !he hemoqtobl... r. in the embryonic l\IId

.. ;. '
; i

~-------
NOV-IB-200B TUE OB:57 AM FAX NO, p, 07

. :.'

I- Ietal llver. Granulocytic cells arc found in ihc cmbrycnlr llverbut appC'\f tr,) dcvdc,Jp
not from the hemO(yLobhl-\t.\ but perh:.!pli dil~(:uy frouuhc lyl"llphoc.)T.Clicl wanrh:ring
"J
~ tell>; ,h~nl$~lves.

I The Embryonic Bone Marrowand Myclopoicl.is. Ilono for 1!\;IUIlTI ;11 t11~
embryo oCC\lTS at val}'h)g times for dltleren; bone-s. The carllcs: bones LO fcnu ,U'(~
the lon~ bones or [he appendicular skeleton. Initially. a l"artil:rlgJllU\l~ mode-l of
cadi bone is formed. The ccntrul core or the diaphysis of ttilcb lung b(1llC Jj\lb!-;~..
tll1CIltly bcr.omml osslfled,and soon ;\11 area of bone resorpdon develops, fl)lIowt:tJ
by the ingrowLlI of nw.enr.hymaJ cells f"om the p"rio>,,,,,m. TI,cso mesenchyma]
cells are accompanied by the lngrowlh ofcapillaries. The n1t:1UJIIr,hYlnlll ('t:dlj1 con-
tinue \,0 Increase in number by continued inlhuc.ofothcr m~M~ll('lIYJmll (:~~11!ia~ well
'" by <livi"Oll of those .lready wllhin the newly fonnln~ marrow ""Vlly. 1'h<'Y 111-'0
elaborate a noncellular grcuud substance, or matrix, Ihat fills rh,' clcveloplng' mar-
row cavity. Cellsmorphologicallysimilar to the hOIllOtylQhl.,l, or ,I", liverand yolk
sue develop from these earlymarrow mesenchymalcells.Aiin lhe yol"~" ,md liver,
these give rise to megakaryocyles and erythroid cells, a., well .., myeloid cells Iu-
clodlll~ neutrcphils. basophlls and eosinophil" 'I'he cmbrvonlc marrow differs
markedly from the earlier centers of hcmatopelcsis in that the generadon of
,nye)n;!! r.<1I. Is e.'peclally vigorou. and dominate> the embryonic marrow
hematopoietlc a<:Llvlty. The process of formation of the early mydoitt ('<:II~, or
mytlopoies;,fJ occurs firs.L in t.he central POrtlQI1 orthc marrowcavity?ontlS(J!"L'Hlb; cut-
"""eI eventually from there to Include LIlt entire marrow e.vlt)', Erythrnpolesls oc-
c"" 'lightly later In the embryonic marrow and is generally admixed wllh the pro-
cess of n,yeiopole,Ia.Small foci of erythropoiesiscan 11111\ he ,.,)" among the ",ony
maturing cell> of rnycloidlineage. AIter bird" hematopclcsis ceases in lhe liver,
and the bone marrow couunues to be the pdndpnl slte ofhcmatopoiesis for the re-
malnder oflife.

Hematopoiesis in the Spleon of the Embryo Mel Fetus, TIl, 1." ,u*,r
site of hematopolcsls to form in embryonic lifeI, the spleen, Nlhollgh the spleen it-
self fQrm" much earlier, circulating h(}mat(lpoi{~Lic progenir.or!l and precursors be-
gin to popuiate the .pleen, beginning around the fOIIl,.11 (cral month In humans,
Probably as the n',ult ofil; large blood volume, tho felal splee" lh," become a cen-
tcr of erythropoiesis unul the limo of blnh, when 'picnic erylhrnpoie,j, grarillally
ceases . .AlthOUgh some myclopoic~is occurs in the embryonic 1il1ld l~tdl ~rt(!(m, it. i~
relatively 1n<Iw>ificam In companson. Much IMer, during the fiflh month ge".- "e
\;On, the whi"pul!, of the spleen forms by dlffetcntiotlonofmo,cllchymal coli, lhat
have grouped around d,e _plonle artcrlolc s, Tho for morlon of the sptcnle Iympho-
cyu.'s appears to occur us ~ prMell.A completelyseparate Irom (h~ (lri~in of tH)'UU."O,"
polests in thl' organ,

OlherSllOl8 of HCUlatopolcsls in thoEmbryoand FeluR, TI,eembryonic


thymus develops as an outgrowth of the third branehial pouch, The thymic epi-
[helium Is invaded by wandering mesenchymal cells that: begin mulliplying r:-tpicUy
and dlfferel'lLLadng into Iymphocytei. During lhi~ proccH JilTl2\1l numbCl'a of cry-
j

th"oid and myeloidcells are formod in tho thymus.. but the pdmary proCCSli is t.h.at
l,"lt lymphClpoi~~is. The lymphocytes formed 'n thili organ will cOllstittllC '" di5liuI;t
clUJ of lymphoeytM whh a spad.....) fl\ncdon~ that of ccllul"r-mcdhILcu immunily.
(S~~ iacer,)
NOV-18-2008 rUE 08:58 AM FAX NO, p, 08

6
CllAPl'ER 1: HlMA,TOl'Olr:sIS; '/ill D!WLOl'lolfNr 0' lIl.ooo CW..I

The lymph node. develop as outpouehing. from the prUrlitiVC Iymphalic ves-
sel, lh.t become surrounded by accretions of mesenchymal colis. Subsequeatly,
theseseem 10 round lip and becomeSimilar inappcar""ce to lhelyrnphocyreo oflile
adll1l. Afewof the mesenchymal ceUs give rise to cell' of other lineage., such ... elY-
throcytes, granulocylCO, and megukaryocyl"S, but alls is a transitory phenomenon;
as in the thymu., the principal proee.. is lymphopoiesis.

SummlU')' of Embryonic and Fetal Hematopoiesis. In all the hematopol-


eticorgan. of the embryo and fetus, a simnarprocess takes place (l'lg. 1-2).Cirelllat
ing primitive ht'ltlalOpoletic stem coils lodge in a particular wSlle niche, by pro-
Cesses ,till not clearly understood; at these niche. they differentl"e into cells
recognizable as hematopoietic preeu,iiOr$. These embryonichematopoietic precllr-
'0'" appear to be capable nf ,nultilineage dllTerenliation, but at anyone ,ILO thepro-
e(>SS of hematopoie.is may be dominated by the Iorrnauen of a particular lineage,
presumably under the Influence of the local environment. The varlOI" 'lteo of ern-
bryonic hematopoic.i. seem to b. active onlyat specific time' during development.
They follow 0 pattern of programmed involution, e"cept for the bone morrow,
which continues.. the prindpal locationof hemetopclesis in the adult. Thell'ltlph
nodes, spleen, 'hrmus, and other lymphatic ,I"ue, continue '0 be active in lympho-
poiesis in the adult .. well.

HEMATOPOIESIS IN THE AD'ULT


Afterbirth, primitive pluripotenl "em celldevelopmenland myelopoiesis oc-
cur in rhe bone marrow, where.. lymphopoiesis occurs in the l.hyrnu., spleen, and
lymph nodes. tinder certain conditions of patholollic ........ myelopoie.r. reenacts
itsfetal expre.. ion in the ,pl<'Cn and liveras well, The major aite of hcmatopoletlc
llClivily ,hilt. gr..dually from the liver and 'pleen to the bone marrow cavill.,. of
nearly all bone. of the axtaland appendicularskeleton. The marrow acqulre'S a red-
di'lt color like tlta' of blood once hematopoieticaCtivity begins, ... necting the vig-
Orow production of erythrocytes that cemain hemoglohin. The bone marrow cav-
ity.e",.s prinurily .. asite for the production of nonlymphoid blood cell., where..s
Iymphopoiesi. in the adultoccursprimarily in thespleen, lymphnodes, thymus,and
SU'"",soel."'d Iymphelcl tl.Isue, Includinl\ the ton,il., adenoids, and Poyer'. patch",.
Thuswhen examined withthe lightmicroscope, Iho~rlllit marrOW will be seen to be
composed primarily of erythroid and myeloid precursor cells, 'ogether witb .ca~.
tceed megakaryocyte,. 'there iJ alsoa populadon of cells known as "stromal cells'
thXl arc crucialCor 'he maturationof the precursorcellsand release of the fully dif.
rerent~ted cell types into the circulation.

BONEMARRQW

The hone marrow provides anumber of domains or hematopoietic j'Hluc:tivc-


microenvircnmenuto allowfor the production of red blood cells,whiteblocd cells,
and platelets (Fig. 1-5). Stromal eetls such OS ",dclll.. cells and bamer cell. fOrm
locules CU" enclosun., aa '1\ physh:.:al eepressicn of Ihe fragile :rnlcroenvironment
which also includes endosteal (lining) cells, lymphoid ...lls. oslcoblasli, Olllc:ocI.,.",
macrophages, and theirsolublegrowth f.clO... (CYlOklne,). ''ltese create and main.
tain lhe "soU' Cor nunuring lhe "seed" hematopoietic .ten) cell. and their progeny.
NOV-18-2008 TUE 08:59 AM FAX NO. P. 16

or
,

14
CHAmR I; HEMKropous", THY, OMLOPMY,NT or 8T 00)) em.s

rumors, extra- medullary hcmaropoiesls, or abnormal material Inl1ltraring the


spleen, .. in amyloidosis oeltoragediseases such as Gaucher's disease and heman-
giomas or cysts al", cause 'picnic enlargcment. The ded,ioll tc surgically remove a
hypersplenie, enlarged, diseased. or bleeding spleen is not an easy one sinee
plenectomy renders the patientImmunologically handicapped for life.
The full range of eau.e. of f""cflolm! and anatomic hyposplenlsm Include.
congenital absence, .pleneetomy, myclonbro,i. and other myeloproUferarJve dlilor-
del'S. impaIred vascular .upply lO the spleen, immune or autoimmune disorders
such a, lupus or rheumatoid .rthrltls. celiac disease and inflammatoty bowel ,Ii.
ease, invasive tumo~, syslt:mic amyloidosUJ nephrotic syndromc, mastocytosis, and
the neonatalstate.
Peripheral blood eonsequences of hYpOlplenl,m Include transient throm1>ocy-
IO$'S ami the pr..ence of Howell:Jolly bodle. (nuclear remnants in red cells), aawell
as.umce pits and pooks resulllng ll'om lack ofsplenleconditioning of red r<lIi. Tar-
gel oelL~ and acanthocy'"arealso seen In theabsensc of thespleen (Table S.9). 01ear-
arlee of enc3psulatt:d bacteria isreduced, asmentioned earlier. R~pon$e to allliFnic
challenge, inolU/ling Some vacdn.., Is altered. Susceptibility to overwhelming para-
.itic infcctions such all babesia andmalaria ix a consequence of..plenia,

LYMPH NODES
L)'1ng in the CO""e oflymph.ticVl!sscla are J)'JIlph nodeswhich are small, oval.
or kidney.shaped structures lhatare 0.1-.2.5 em long. Theyare connected to lhe clr-
<Illadon by atrerentlyml'hatio "".,clt lhat run intothe grealorCllrvature ofI'll.node
and efferent lymphatic ve... ~ that exit from the hUurn (~1g. 1.7). Valve. In lym_
phatic ....eb ensure unldlrectionallyrnph 1ow, At the hilum are enlering arterie.
and eXiling vein.Afibrew oapsnle enclo.........h nede, and trabeculae extendInto
the parenchyma. SpeclaIlr.lJd mcohwork or filtration becU composed of retiC\llor
cell, and libcn receive T- and IHyrnphocytes from lhe r<!CirClilating lymphocytic
pool. T.lymp~otyl'" ~mpn.e thepellpl,eIaI.rca oflymph node. and are concen-
tratedIn the mtcrfolhcularzon. (bet.wecn prim a'Y and .ecolldaryfollidc areas) and

Primary S-o.11
Follicle

',I,;
.' ""'.
,. iiiii"'iiiaiallllil-= _
NOV-18-2008 TUE 08:59 AM FAX NO, p, 17

15
lftmafopoi"t, in tn, Adult
111 the paraccrucal area, T cells ill lymph node. are CD4-helpel' type. (80%) and
CDB-ouppre"or type. (20%), B-lymphoeYleS In the lymph node cortex are con-
mined wllhln primary and secondary lymphoid follicles. lntcrdigiraling ml<ular
cell.1 (veiled or dcndrltie cells) may be identical to the I.:mgerhans cells of the ep-
[thelium that travel to lymphaticti.'ue withnowly collected antigens.
Each lymph node then Js composed of an aggregate of R-lymphold follide.,
each foWele repr"-'l'1llil1g the expansion ufo smallnumber ofn-ceU clones. Grouped
around these Iolliclcs are seas ofT-lymphocyteS that funr.tlon both in concert with
the 8 cell:'! and on their OWIl. Specia.lized cell-interaction moleculesare expressed on
fhu T cell, Ihal directly surround rhe fo!liel.., which se"", as T-<:ell-B-cell adh..ion
molecules and help mediate T-B fnreractiou in Mdbady maturation and secretion.
Barrier cellsof IibroWesticorigin variously aggregate to create blood p'lhWlly'
and sequest.....u icn sites. Ma crophages work-in concertwid, battiercelt, to control in-
fectlous diseases and paniclpate in the immune response, The .ffere,n lymphatic
vc..els that contain lymph, antigens, Iymphocyles, and macrophage, conuuet with
subcapsular space, tymplt and il.' conrents then go to paracnrlicaland medullaryar-
e.., medullaryslnuscs. and Ilnallydl~rontlymph.tic v...ell. Arterial cirmlat;on do.
Ii""" T cell. from thyml" and B cells from bone marrow La lymph nodes. B and T
cell. enter the Interior of the lymph node while they are 011 their exit route in the
lymph node venules, whose high endothelial cellsrccoR"17.e and direct l)'l1lpho<yle,
Into the lymph node. The .\ruelural and cellular makeup o(lympb nodcs allows for
Interaction, ofantigen and lymphocytic cell.,which set, up optlmal actlvarlon of the
immune response.
Lymph node. may increase In ,i,e because ofa ...met)' of nann,I but perhaps
exaggerated immune responses a., well ...everal clearly pathologic statce, They in-
clude the following: (a) Increased blood flow and cellulaemakeup as part oftheim-
mune response; (b) frank infection. or inllammal10n of the lymph node it>elf (lym-
phadenitis): (e) macrophage or&lOrage cellengulfment of metabolic debris or end
prodUC.Ul in certain ~norfige diseases: (d) neoplastlc lnvolvemem byprimal)'or meta-
static Iym phoreticular or solid tumors,

THYMUS
The thYl11lJslics: in the anterior mediastinum. Softand bilobcd in character, it
i. 10-15II ot bil'lh,rtlpidlylncre..,lng 10 2G-10 g. and doc. not change In weightdl'i\-
mOlically thereafter, although the amount oflymphoid ussue sradl,ally decreases
wilh age and tlle gland become. fatlybut stlllimmunologically acnve. The thymu,
'.. develo~, in the eIghth weekof embryoulclifefrom branchial pouche~ 3 and 4 I I an
epithelial Organ populated by thymocyte, (T cell.) that derive from bone m.rrow
prothymocyt... 'I'cell marke... ChOI-aClCrize,tate, of1'-<:ell development in the till"
mus.'I11e gland forms lobules because of eapsul...epradcnThere 1& a cortex and
medulla In each lobule (.ig. l-ll).The cortex contain. 95% of thymic lymphoeyws.
~ but supporting epithelial cells are abo present (epithelial retlcular cells). Prothy-
I mccytcsenter thjmlc parenchyma high in the cortex and move deeper towanllhe
\i conlcomedollary junction, maturing as rhey travel. They interact with .lrumal cell.
I (epithelial redcular cells, reticular cells, barrler cello. an~ macrophages) Ihat in.
\ SU"'c' the develapinll T cells La permanentlydistingu!'" .elffrom non.clf.
~
},
The medulla contain. 5% of rhymic lymphocytes. These are lIlatuce T cells.
Most cellular clements here acepolyhe<lrnl epithelial cells,These cell. may assume
r an Irregular, circumferential, lamollated appearance with central nccrosls, calcl-
I
NOV-18-2008 rUE 09:00 AM FAX NO, r. 18

... ~

I.

16
cHAPrE1t 11 HE!olATOl'Ol1'~I.' Tlit DEVELOPM~NT or111.000 GEr.!.\

Epill1ttUal
bordor
Siplum
T "tern CIII,
Mlllorophage

Epllhdal- <::::::]J
rO~Gultl.' coilS l~iI-- Sapl.1
Ilrtory
Carlall.

:W----"'Epi'hfll.1
border

Blood.
Ihymic
{on<t
bo"'o, Epllh.

~I
Oula,
modullo

Thym!c:---
COrpuaoll
(Hllulitll'~
r;orpueelol _l
-1-
Innor
m.dulla
_1-
'''UN 1.... Thymic lobuli: scbemauc dLaprn. CnrtL:"1 outer tllCcll.llla, Inner medulla. andcellufar
eompenenu areshewn. (Seetex.t for dtl:u1. tlllympho~ llrnyAod flow.) (flc:<Irawrl rfO,U Wcoj~tl, L, ed.
CrU ttrtd T.,)rH 1Jifl~: A TI~'lIll44 f{ 1f~~J::I, 00) ed. :BW.timor(!:, Munich: Urban and Sch""'I''ti't;nbcl'y:
l~BA.)

Ilcatlon, and ")'11 fonnarlon. Such strucrurcs are called thynlic or Hal'aU', corpus-
cles and represent the end .tage of thymir.-medulla~pithelia! differentiation, At
the corticolnedullaryjuncuon or In the medulla, T-lymphocyte. enter vti ns or lyro-
pho.lics and circulate to tho spleen, and then to the recirculating lymphocyte pool.
Approximatcly9G% oflymphocytell die within the thymic cortex, so that only 5% are
releaeed Immunologically competent cells (Fi~. 1-9).
In summaI}'. thymic T..,.II de\<elopment occurs pl'edomlnanlly during child,
hood and adolescence. After the second decade the thym... Islargely In\'Olulcd, al-
though some thymic activitysUIl occurs in adulthood. Ct!1'tain aspcets of ea,.1y T-cell
development appear \0 occur outside the !hymu.. .. _II, p.rh>l~ in l)'ll\ph nodes,
but the details oflhis procc ....re uncertain, 01\(.0 thymocyle' develop inlo mature
T oolls,these ccll. circulate throughout the body, Much of their Immune function
occurs In lymph 1I0dtA, where they Induce B 00110 to mature and develop into anti-
NOV-18-2008 rUE 09:00 AM FAX NO. P. 19

17

Thymic
cOrplJlcl1
(H_'.
Oorpu>c1e)

Oondnne
Col

Co... Medull.

151.cell. - - . {C04
CD4t -
t
{CIl4'
coa' ~ T'tRajl
coa=< - - -...
{Cll4f
TH
TC:Rt TC:Rajl
(lew) (hIgh)
(Common
pt'CICUrJor
(CD4 to
CDl1-
P,e-T Cel) T'tlll>j! ---Tc:
(hfgh)
Fu_1 T Lymph_.
HoIIle' (TH), Cytoiyl~ erel

Fieur. 1..9 . SlnlCLurc of the ~hymLlft. Janm:ltul'f thymocyte. mluut,p from the tOneK lOWllrlo1 rl'l~
lI.edulllu)' rcoglIJlt, Tel ctfJ moleculd arc MpK:t(!d. (TCJ.y8 cells are ....-a.) CO,tl.,llflrt'lttaigllation; Tell.,
Tcelln:cc:pttu;fJI!i,rlp Inoll."cules otl'CR; kHIJ, hfgtl<tonote Icvtlll of tl{jexpreMlon, (Rodf'lh'vlllwmNicbohl
~, K'fr" 1:r ~ltUe(Ur .. rind(l,lftcdQn of other l}'mphfljl1 Ci:t4\Wll. In: Beutler . Lichtml1ft MAo CollerBS,
ICiP1I~ 1], (,"(11. ~llf"lfdHPA~tlJwf:Y, 5dl ect. Nt.'w\'ork: McGnw-HiII: J'a95. 4.6,)

bony-<ecr<!ung plasma cefls, whereas other aspects of T cell Immune function occur
in nonlymphoid tiI,u. ,Ites,
TIle lh)'1l1wl. 'hw ....pon.lbl. for the induction ofT..,.. ll maruratlon and de-
velopment of the ability ofT cells to distinguish self from non ...I, The gland Invo-
lutes with Ilormalllging, Sir..... and diase, It may hypertrophy, induced. by tri-
iodothyronine (TS) prolactin and groWlb hormone, Rarely, following
chemotherapy for systemic mallgll.ncy. the chemically and hormonally enppresscd
gland m.y reboend and assume, worrisome but benlgn la.'!:er 'i1.e. Tho gland lraelf
m.y be Involved wilb thymoma.o-LUmors th.t have important lmmunologic seque-
lae and may be wod'ted wilb such diaea,e, lIS lny.l5lbenia gravis or pure red blood
cell apl..la (Chapler 7). Oc"",lonally, Ih)'1l1omu m.y be malignant and IOC1l1y ln-
v..lve: Lhey rarely give fl,e '0 distant metastatic diJoalc,
NOV-18-2008 rUE 09:00 AM FAX NO. p, 20

18

TIll?, STEM CEll. MODEl, OF HEMATOPOJlo:SIS


The cell rhat gives I'isc to all other types of blood cells. the stern cell. i, 60 rare
that it has never been morphologic.uy identified with certainly (Fig. I-I). Rather.
the extstcnce of Item cells i, inCerred by functional way' thou demonstrate the abtl-
ily of'inglc cells to generate multiple hem.topoiedc linellge,. 111\1' stem cell. arc
curr..ody deflned not by their appearance but by their function. Stem cell. arc
~nown 10 he extremely rare, although quantifying them;' kllnewhat impreclse he-
cause of diffcrent degrees of rigor "I'plled to the definition of what consdtutes a
stem cell. The: mostgenerous cstlmate i$ Lhat stem cellsoccur ill human bone 11'Ia.r~
rOwwith a frequency of 1 per 1 million nucleated bone marrow <ell,; more conser-
vative estlmale' place this ligore a' I per 10 milliOl\.
The tremendous production rate ofhematopoielic cello require. tbM roughly
every other day the bone marrow produce as many cells as it contains, To ",ainWn
til'" rate lhr"eghoullife. tile bone marrow must po5o'" cell, lil.t can generate vast
numbers of mature celli continuously (i.e., wilhout!"sing the .bility to do '0). This
self-renewal abiliWIs critical to the concept of the item cell. Atpresent there are two
theorie.about how this mlgh, occur, According to the fira'. every stern cell divi,ion
Is asymmeldc, producing one undiffercmiated stem cell and one more clifleremi-
"'ed cell that is commined to produce mature blond cells. In tile second theory.
each Stem cell division produceseitherf.WO additional stern cellsor twomoremature
cells, The stem cell 1'0011. thus maintained not by precise Mymmetric divillioll'
within .ach 6te", cell bur by balance bel""",n the number oCs~m cell .lIvMon.
yielding more stem cen, and dlvisions yielding more mature cells.
At t\10 point the stem cell leaves the scltrencwing pool to populate the dUTer
entiating pool, it ,till appear, morphologically". primitive bl... cell and relaln.
the capacity to produce cell. of all lincag... With each suhlequent division, tile
daughter ftrOgenitar UUf become more a.nd more restricted in their comminncnt tn
the prcduedon of.pecilie blood cell lineages. That i s, if one isolale. progenilorcclla
and permits them to propagate and differentiate, lhey will generate collecdons of
cel" that are of only one or a few lineage The more differenti.ted the progenitor
celt Ihe fewer lineages arc produced and the smaller tho number of cella produced,
These concepts, which have been supported byseveral decades of in vivo and in vitn>
experiments, havc now defined tho hierarchical stem cell model of hemalOpole.is
(see Fig. 1-1).

Tbe Hem..topoletic MIaoenvirown""t. Iflhey art malntlllned In a simply


nutritive environment, stern cell. willdie without differenti.tlng or dividing, 'fo su?,
port the proee.. of hem.topoledc 8C\~renewal and diLTerentiAllon, stem celli and
their progeny m\Jllt be malnta!ned in the clORe proximity of acnbcmatcpoierlc me s-
enchytna] cell called stmmJJl..u.. Til... cells-whlch are composed of II heteroge-
nco". group of flbroblasrs, endothelial cella, osteoblast<, and adipocytcs-tlne ti,e
eud,,"ealsunace. In the bone marrow CAvity. They appear 10 supply two closely re-
lated requirements for the bemarcpeleuc cc1b--$olublc hematopoietic growth fa.c..
to"" and membrane-bound auacbmeut molecules, '
The hcmaicpcietle grcwih faclors (HOt"), or eolonystimulatil'g f"elolB {CSi''J.
arc a cl... "f glycoprotein hormones that obligately rg\.late the division and <lit-
ferenUadon ofhcmatopcleuc cells. These bormones aro required for .urvi9al, pro-
Ilfc",uQIl, dllFerentiati<m, and function ofall the bematopoledc cells. Altbougl. ini-
tially discovered ...pontaneol1sly secreted produc", ofT-cell tumors, it is clear lllat

..
"
'
,
NOV-IB-200B rUE 09:00 AM FAX NO, p, 21

19

these horn~()I~e!i tire normally the products of bone miUTOW stromal cells as well aA
T-lyrnphucytc.:f and monocytes.
CSFs are produced ina rwo-tlered process, Firat, small amourus of ccrtaln C'iF~
(interlenkin n [Uc6], granulccyto-macrcphage colo"y ;timul"tinR Iactor [GM-
CSF1, stern cell factor [SCF]) and Ilt-S (Flt..lO Iigand.,e produced con"ilo'ivcly by
bone marrow stromal cells,probably In rcspcmc to sdmulatlon by plasmaprorelns.
The production of these ('S1,1, is responsible for basal hcmatcpoicsu, which muin-
"'ins blood counts in Il,e normal "lOge. (rig. 1-1).
CSf secretion ill l!roat!J' Increased above the basal levels in response lei infec-
ticn. Bacterial and viral products activate rnonocytcs, which then secrete interlcukin
I IIL.l), tumor necrosis factor alpha (l'NY-a), and granulocyte colony SlimulaLinK
factor (G-CSF) , a.s well as th.lr own macrupbnl:" colony .timul'ting [dolor (MCSF),
These pronuc" in turn sdrnulate additional CSF secretlon. IL!. tOKether with all r.i-
genicsri mulaeien of 'pecUic receptors,acdvare T cells to secrete GM..csF and lrucr-
leukin 5 (IL.s). fL.I anel 'INF.a each ,dmul,te fibroblasts and endothelial cells in
the bone marrow stromal mkroerwiromnent to increase endoLllcli.t1 cell secretion
of 11.-6 and CM-esF, and 10 secrete large quanrtdes of c.CSY. These hematopoietic
growlh factol'$ (eytokilleo) thereby directly increase the numbel~ ofcirc"UlalinK ncu-
trophlls, monocytes, andpll:U1ma cellsandactivate these cells when they mature. The
generauon oCeaeh .pec1ficlincagc of mature blood cell,;" regulated in thismanner
byo .pedRe 'et cfhematopoktlc gt'owth facton. Although the scl' of hern.atopolotle
groWlh fU.Ct.ul1 thatinduce spc!:ciflc mature blood cell 'lUmeL~ overlap, each i!l: ehar..
acteristicallyeli,tinn.

Errt'uopole.i., The tina! 'lag" of erythroid dilferentiation are '"Kul.ted


largely by erythropoietin (EPO) glycoproteIn produced in re'ponoe to ,i,,,why-
poxi. ill tho Celllillverancl adult kldney. Ofth. IB 0,"0 cell,Iivision. that rake place
during the time .. stem cell generates a mature red blood cell, erythropoietin
otrongry Induces the final B-IO divi.ions. The transcription of the erythropolclin
gene in renal perl tubular endothelial cell. and hepatobl..tll i, regulated byoxygOI1-
sensitive transcription factors that up-regulate gene expression with de!clining 0 41
delivery. Overproduction ofetythropoielinl observed in lomCCaJles of renal cell car-
cfnoma and hepatoma. leads directly to erythroid polycythemia (Chaprer S).
The precedIng cell di\is;ions. which give rise to erythropoieun-scnsidve ery-
throid prog.nir.or cells, are largelyindependent of ery1.hropoieliu. These prolifera-
tion and maturation eveuu are instead Induced hy granulocjtc-macrcphage colony
,timulallng factor (GM-CSY) and .\em cell factor (SCr), bath of which 'J'<: pro-
ducecilocaUy within the marrow microcnvironmcm by bone marrow Stromal ccUs,
In addition, these .teP'! Can be specifically amplified by the secredcn offL.3 by oct!.
vated T-lymphocyte s.

Graaulopoleals. Much like erythroid differentiation, ,helin.l "age' in neu-


tmpoicdl and monopoiesls are induced by granulccjte colony stimulotdng mtor
(G-CSF) and macrophage COIOII)l sLimuL~tinK fiu:tor (MGSF),res peeti\'Cly. The ealiy
divislonl~ which direct multipotential progenitors lO become committed to indlvid-
ual Iin~nge are regul~'ed by the syncrgl.otlc Int.racllon. ofGM.csF, SCF, and IW.
AI described earlier, a1l/lough there u a conatant. level of b...1secretion of CSF se-
credcn by bonemarrow stromal flbrobl:ulrJI that Hac: the marrow endosteal surfaces,
the secretion of CM-CSF and G-CSFI. d ramaticaUy up-regulated in the presence of
InO..nmation In rcsponae to the secretion of 11,.1 and TNF-a by monocjies.
NOV-18-2008 rUE 09:01 AM FAX NO. P. 22

20
CHAPTER 1: HEMATOrOI'ST.~: TIlE n .VEL01'ld~Nr 0.1I1.00D Curs

For the production of ecalncphils, lnterloukin " (nrn), .. well .., to a lessor
extent, 11,,1 and GM-CSF I,lay major inductive roles, lla>ophlisand mast cells are dl-
reclly.timulated bySCF and IW. In both at theseinstances, Ih. initialalTcrelll.ig-
nnls that trigger the release Df'tIlese cytokines i< not yet well understood.

Mep!<aryopoie.is. The earliest .rage. In the development of mcgakary-


ocyde progenitor cell. appear 10"LIO b. Induced by thrombopoicun ('1'P0), in con-
junction will\ aeF. 1'1'0 (toeU also induce. terrntnnt maturation of megakaryocyte.
and pL'lelet budding. In contrast, Interleujdn J I (IL-ll) Inay playa majorrole in
pl atelee budding, with comparatively minor influence. on megnkaryocyLO genera-
don and development per se. (For further dllenion of IhiB topic sec Chaptcr 5.)

n Lymphopoiesis. As with the myeloid lineages, tiledevelopmentorB cells


b"lJlrul with ibe dlll'cn:mJ.tion of pluripo10l\tstem cello imo undlfferentiated but
committed Ikoll progellilOl'I. The initial 'la~e.1n the proliferationand differenu.-
uon of thcac Bcell pmgcnitol'l are Induced byinterleuktn 7 (11.-7) anti 5Cl', Once
recognizable (lre-Bcells and B cell. arc generated, further di/ieretluaUon and divi-
sions a... Induced by stlmulation through the Immunoglobulin an ligen receptor,
through the Fe receptor, and through stimulation by soluble [nrerleukln 4 (IlA)
and [Ui, Once andbcdies producing plasma cells are genemtcd, addllional prolir-
erauon and antibody secretion arc stimulated byll..f, and GM-CSF.

T-lymphopoiosis. Once pre-T cells undergo the complex proce",e. orneg-


,uive and posluve selecuon in the thymus that seneratc IIclf:. and not\-Rc1r-discriJn~
lnatlon, the re.ultlng mature T cons are 'ubjeclto antigen- and eytol.lnc~nduced
actlvarton and expansion, 5timulalion with an autil{ttl plus fnterleukln 2 (1L-2), or
with an antigen plu. costlmuladon with accessory macrophage! or dendritic cell.
thAt."pres. B7-I or B72, I.ad,; to the direct activation oCboth C[H~' .,ld CD8+ T
cell.

THF. Cl.INICAL USE OF HEMATOPOmTlC


GROWTH FACTORS
Since the lim human hematopoietic growth facton were cloned and isolat.cd
8 years ago, precllnlcal end clinical trl.l. have rapidly led to their introduction into
the clinic tor routine and experimental usc. At the present rime, eryduopoictin, C~
OSF, anu CM-CSF have All been approved for human usc by the Food .nd DrllgAd-
mlnistratlon, and IIr3, SCFand 1L-6, TPO, and Flt3 ligand have oollUIl d\. fj",
phases ofbuman tc.ting.

Erythropoietin.. The anemia of renal failure is the prlmary condi~ion th.uis


directly responsive 00 treatment with RPO, Appropriate doses given.three dIlle'
weeklylead to prompt retlculocyto.u, a.long as the J>1'tientH have .ufJicientlron. Co-
hue, and Bl~ stores and have no other source of ongoing innammation that would
suppresserylhropoic.i1. The majorsidc cffeClofmch treauncnt-o!' hyperlCJUion, es-
peciallyit the hemoglohln levelru too hlg1\, 'I1\e Introduction ofllPO thulilpy ,,,,,
made thou ncb of renal dialysis patienu nontransfusion dependent and hasgreatly
improved the quality <>f theirIt"",. .
Other anemi.. in the ,etunlj of such chronic dls..... as cancer, AIDS, and
I

j
NOV-18-2008 rUE 09:01 AM FAX NO. p, 23

:;,1'
,.;
.

- 21

!ess,.r
rheumatologte diseases present a. second group of conditions tlnttalso can respond
re dI. " to EPO. HOWCVl..T, the doses required are greater than for patienr. wIth renal failure,
It "g-,: and even at high d()5eJI the responsesare variable. In general, the higher the base-
line EPO level circulatingill the padent's plasma, dlf~ It:llli the chance or response to
EPO therapy,
k:uy. A third, h'Towlng area oapplicmion ofF-PO thCI"RPY is \.0 a,,'ologo,,-' dnnaUon of
too red blood cell> prior to elective 8\lrgt'l'Y. By admInistering EPO under a controlled
<ytrJ setting, hematologist. can .timulate 0 mild erythrocyto'ls (high red bleed cell
.II! in count), which alloWll removal ofhlond without causing severe anemia-In thisway It
nera- '\ I. po..ible to store ...era! unlll of red blood cells prior to anyelective slllogel}', thus
5,} eliminating the need for and rlsk of allogeneic red cell ua",fu,ion. The only limj-
tauons on rhls therapy are the organizational abllide. of hematologls ~nd blood
cdJ$ banks. Given the large numbcr ofblocd ('mduet. required tor urgent and emergent
t but surgerle al10jieneic douation and blood lnln>f'u.,ioll will always he essentlal. llow-
nlla- ever, for the iuc:rea,;ing number of e1;CUVC condiliona requlring Me support. au-
Jnce tologous, RPO.. rlmulated donation will be relied upon marc o!\en.
diyj..
ator, Granulocyte Colony SLlmulal.ing Factor. wtien the periphcral ueutrophil
LA) coun t I, depressed, IncreMod levcbl of circulating granulocyte colony stimulating
'olif factor (G-CSF) can be detected by sensitive a".yo such lIS El.lSA. However. the
amount of G-CSF produced i' subcptlmal to ,'llmulate rapid granulopoiesis. With
,. additional ()"(,-~F, mpplled cs 3 pharmaceutical product, the neutrophil count will
m'g- j recover more rapidly al1d to a higher level The first majorlelung in whirh C',..('llF
rim- therapy 11... been applied hM been In chemotherapy-induced neutropenia. Delivcry
Ired of GC',sF subcutancoualy, beginning ~ppro~lmalCly I week following chcmcther-
l, or '. apy, helps Iessen the length of chemotherapy-induced leukopenic periods. Cnn-
cell' trolled Studies have now shown that such treatment helps to prevent infecli('ll'l.8 and
+T hospltallsadons. The onlymajorside eITert ob<ervrd withG-CSjlls bone paln,which
occurs in 10.-15% of podont> and can be c.sily controlled with onolgc,ics. A <<\11-
denary 1I0te; G-CSF therApy use, which hasaUowed phy<ici""s to USe higher elm.,
of chemotherapy (greater dose Inr,el\llty), hal 110' yelled 10 total elimination of dis-
ease and significalltly greater .urvlvuL
Pharmacologic dose' of G-CSlt are .1'0 effective in ral.lng the neutrophil
ltcd count in several cases orchronicneurropene, such as congenhal neu troptnia (Kost-
!nta ,\ , mann's disease}, idiopathic neutropenia, and irnmune-medlatcd ncuuopenlas,
.o j: such a. 1 ClymphoproUfcfQtiv. dis ease. In these cases, however, the doses or ('rCSF
Arl ,'I required can be .ubstanriallyhigher, and the response Or' not aft uniform. In addi-
first tion, recent !ludies suggestthat as manyas 15% of eOllgenilll11yncutr"l'ellic pattents
.'( treated witll ('rGS!' will go on to develop secondary acute rnyeloid leukemia.
:{:
Whetller the development oj'leukemlai. causedby the ,timulatinn of the abnormal
marrow <ella with the high lL"VCu of G-C~F is unknown, but this Is certainly 11 cause
for concern.

Cranulocyte-Mac:rophage Colony SLlmulaUng Factor. Like (;.(;Sl'. (lTlIn


ulocyte-macrophage colony"imulodng faclor (GM-CSF) Increases the neutrophil
count ,no tJivo, However. it hal a illb.nanli+Uy broader range of actMly, and iL inA
creaaes The monocyte count and eosinophil count. Some etudlcs have al"o reported
increases in retlculocytes and platelets, Given itsbroad spectrum ofac:livit:y, GM-esJr
h.. been upprovcd by the !'DAfOl' acceleranon of recovery or hcmatopolcric func-
NOV-IB-200B TUE 09:01 AM FAX NO. P. 24

22
CHAPTEtll: IlRM,\TOPOIUL<: l'H~ n,.vY.lmMltN"r OF BLoor, Crus

tlon following bone marrow transplanraricn.In thleulng GMCSF clearly .horlen.


the time (0 recovery, resulting In lloClbly decreased morbidity ofUlt: Lran'pllllll<1don
procedure,
In additlon, GM-CSFalmo.t ce,lltllllyhas the ..me salutary effecl f\$ r~cs~' In
preventing and tr ting chemotherapy-induced nadirs, Although it j. pQlllble tha.
II< specuum of .ide cffe<.ts (chills, fevers, thlrd.. paclng of plasma) might make Its
we in this setting more problematic than r....esF. at .hl. time there ha. been no .ide-
by-side comparison of GM-CSF with G-CSF in IllY .etting.
One recent application of GM-CSF and G-CSF that deserves pattlcwar com-
ment i. that of progenllor cell mobilization. In the first cllnlcallrial. of GMeSF. It
W>$ found that although the ,Ien.lt)' of bone marrow progenitor cell. tole slightly.
the demit)' of circuln'ing progenitor cell. rose dramatically, often hy a fiICtor of
GO-1M over h.... eline. Later. voir similar riata were obtained for G-CSF (and the
same i. probably true fer lUI. EPO, and SGF).l\ascd upon those data, bone marrow
transplant physicians have round thal,hcy have needed fewer periph eralblood cub
lccnons (leukopborcsts) to obtain iransplan; donor stem cell. (Sec Ch'pter 8.)
(These harvested stem cell. are used subsequently lo .upport bone marrow reen-
grafuncnt following ~lerapy that has ablated bone marrow in an auempt to rid the
body ofalll'lCoplastic cell.) Although manyque.t1ons remain regarding the cause
and the meaning of tl,is growth factor effect.... wellas the role of progenitor cell
mobilization in transplanlation, the application of hematepoletic g'I'Owth factors to
"hemotherapy" in u1m'plantation medicine rat exciting posslbllitles for the
decade ahead.

SELECTED READING
Bann"''''r L. Haomolympheid .y""m. I,: Pete, L. William', ed, o...y~ A...."'y. Rdln-
burgh:Churchill !.iYi.gslOne; 1995;1400-1442.
A t""id~ ,.rn"", mid IllwlnJ/A! ,Idio. in. """'~/.IIIOW ,am, ,.Its
booA. ,u1f>risl'lK
nnd ,Inri" .., il .wcr/b" tIutjlm,'I..........." ."d pl",i./Qgy ./ ~ _ , ,pr.m,
dtfJlh

Ifmph ,..<II, tmd Ii<lJmlLl.


Bonm~ MAl Gallo AP. Ruggiero M !!Il al: F.ff'eClJ of reeomblaant human GCSF on neu-
tropenia in patlon,' Wl,h cenll",ital agranuleGY""1s. NE"fIJ M" 19~;S:lO: I ~74,
T/iit .lIldy, _/ungwilhF".hlradi" 0/. (I 987).ntI/WImv.nili< ".1 (1991) ck""",,"'''''
ital tfflr..., 'J EPO, Ges}'.ntI GM.csJo'jn ./ini<aI_I<1I~nWrD. 17Ify P1t1lJW 1M
~",II..
filSl ,v,'dom", 1Il0l ph~<ltialll c.n, ., providing p",,,..."'1ogic dol.. / ,,"'_ IImuIlojIoI-
"it A."",,,... itnprotIe /NSJun, wt/I-b""r.
llowdlor Aj,ed. The Spkaa: S/nl<:Iu,", "'n,,"," nndCU.I'III Siplfi<""". NewYnrk: Chop'
man and 11011 Medical, Van Noltrollld Rheinhold; 1990.
",pre"""Wo
C.. "",in, 'J>Mn, ",!",ls '1'P~" . _ , tmd fJh"w/OfO
Rmerlon SO, Sie(l' CA. Wang FA. et '1\1. PUrULCation of fCl:i.1 hematopoietic progeniturS
and demoustratlon ut re,om.blnl\n~ I'DUllipoccntJal cQlony .timulillting activity. J
Clin [",,,,,, 1985;76:1286.
7'111I[11" P.rlfl_ ./ A_.n /umaIi>fJOI,'i<""" ctIJ.s oM~.
&<hb.chJW. r.gri.JC. JJowning Mil e' 01: Cerreeucn ef the anemia of end IIllge I.. nlll
di.euc with recombInant h"mallc/}'lhropoletill. N EnglJMill19B7,516:7S.
Th/.r nutlt} 1'''Il'';~,JI"./J/a010/. (1989) ""iN... _Ul,".1 (1991), d<t_IM w ,lilt-
IctJI rf!1("9 .JEPO. G-CSF. 01Id t;M.('.Sfo'/n ,linl,.I_Wogy.onco!nr:J. TitIsI til,.. IIlid-
It, JnllrJilfI 'Iu ftr>' ..Idma ..., phy<ic/<ml ..n im,..... p.il-n, wllu..htf '" /,mvitll"f
piwmQtQ/ogl, /J<l.Iu II/nt,,",.1 /rM"''''opsI<ti. h""""'.....
Ford (,..E, HamerwnJL, B~net DWH, l.outttjJ'. Cytologicalld~t1Iicati(mc'*huicn
chimera,. Hal"" 1961;177,452.
NOV-IB-200B rUE 09:01 AM FAX NO, p, 25

23

r
"'hil ,f~udJ (l;rkt "/lilt J%1) g(J'~ tht! jil:tJ"*","nmlul wUII'li.(.ilor 4Jjtll.itm:lt'clu'cClIsltnl (tit PlUlrbl
oj JJtlrln;r)pqkm, tlu mtull.'l tIlt" Ir.g, burl dntt:Wptd rmd liP/will f/T.llff,Jkl ptJ.f~ 15'j"fJn
GuuaSCI Sartor CI. (',Oll.~chalk I.R,et al. Done- rn..arrow ~tror~li.\l1ibfOhll1';I.~ secrete 1[...6
and CM-CSf in the absence of innamm<ltury :ltinHllatIQu. /JllJful 19\.>2,80 :
1190-1198.
This J/1I.dy kmrm.slmfts IIv.(llTlIlllc:eim& flJlinJ 'lUI JigtJi,fl('(I/u qU(JnlWf,~' /)/lI,(, (~ml t:M-C.f\I' Vy
ncmn(ll human fJ(l7l~ mrmmu stramalfilnolJfa.srs.
Zhiyncs .8t', EnlllT,,('ml~nl oflyxnph nodeslind spleen. II\: Is"elh~d,~~r Kj,tot ul., (Jd,~, liar-
1jS(ln~ Prlflr.itJU.~ tJf lnrl:rrlat Medicin" 15th ed. New York: MtC;mw-J-llIl; Hl91:
'2:1-329.
It 1Uu:mC~ a.,~d auth.m;rmiJII! tluruY.liun 0/dittmJlrS oJ lyw/II' 'NHJrJ "J.d $f,4fl.
HenryPIl, Longo 01,.Enlar;c:lI1claurIYfl"lph node. and ~(jlccn.ln: F:.l\ICi AS, "l \11 . crh,
f/(Jrris<m's Prineip/4 oj lnlCf'R41 M~rl',i.,lt:. Hlh ,~d. New York: twil:(:"tw-Hilli
19!1l1:~4S-&!i1.
Ao .p-I....dak appron</I I, j>oIi"". wil',
~,p~""""'paUIj arid fjl/tllltm'l("ry.
Hub:erH. Blehf], Antill [', 6[ I'll. Qmml.lll;and pr(lli(el':H1vo Il:dl (,l'dt-li: howlineages ~(:lIM
erase 0"11 dfvcOliLy and mainminfldeliLy. /l,'trtf CH11 mollu,r 108S:134: 21.\.
1'ht c!(U.,id dl.ttriplifm o!rJu ~ltm,td rlI:m,'t'fJ rtl/Jli,."Jirm fFWItef, ttli/h 111 i1(Jjlficflti1ml Iff WI"
Jllal M"wopi>iaif (U wtll Itt JjJJU sr1u JU(,k as U1lktm1Q (md htlflt': ,nrrmnri/rrilll'''',
},r.rnUlchka IF, ka\llct ell, MUlUfo'ln Re.. Developmental pOlt.:nLiftl no(1 dynmnit behav-
ior of hematopoietic Item ,dl~. ('~U 1986.45: 917.
At jf1..mneding ~iell miring the prJ.fsihjlilj wu
stem 'flls mtJ:hf wi""k tln anti all tlmruglumt (m
intli:rJid1,al's U/tlirllt,
N'~lnunoUtlsJ. SlnccrJW, Buckner CO, et al, Lcng-rerm follow LIp of pu!l'I\lJ.; who rc-
eetvedrcccrnblnant huml1n v..
nulc:u:yle--macrophliLUC colony !lthnutating ["ClOT ar
tcr ilU,ologous bone mllrrow It'an.phml:ll~nn (or l)'mphC'ld m:illguMcy. DO',",," M/JT-
row rra"'pfanl 1~91;7:i9.
nlis study, afrmg wLlh F,schbtuh n al. (1987) 8nd Btmil14 d at (19B9), dt.momlrn/''tt I lUI rJjtl~
~t ifli:at:y 0/ EI)O, G-CSf. ontJ C,M.Q'P'in dinimll"tfl(t.fofogy-onr.aloK/' 1'fw) lJr,trstltdir.,J
pro'lIrd, 11MJUst tvitUn,., tha' ph1:t'id<Jm can imfmM PalilnlllJ,Ullint!ry In'fI11/dmg jl!ltil'"'
"'"""' dot" o{"lUU1a! """~ '""""'"".
Ni,ho[li WS, K1Plll1 TJ. Suueuuu lind fu.ncl1on of other Iyt/lphotd [1';'~I~e~.Ii\: &1.1l1cr F:,
Lich[mnl'! MA. COUcr .as, KlpprTj,eda. Willi.:vlu It'fMfokJrj. 5th ed. New York:Me-
C...w-HlIl; I99E:"~.
A <i"", "'jlla"a/ion ,{ ~oplwid lisrut p"J.n.lugy.
Sill! RH. Hypo.plcn"m. l~: Pochc:dly C, SUI, Ill!, s'hwa," AD, ed. IJiMm!", of dy
SfJIRn: PI>I/Phph'},w"IDond Ma"agrmml NcwYork: M.",.I Dokker; J98~ :!l'l-I ii.
f1,lpfulloW,J 'Ip"'' '
"'P071<m"'" "" jOunJI i. ,hi,1uI.
S~C)ckm.all.JA. Splenarrarsnly: dhlgnowLic OveMew. 101 PochcrUy C. Silh RH, Schwd.rt:J:
AD, em. DWwdm~/ Ill, S/Jlilua: Path(Jp"~'iD1tJgy (/1M MaJl~l, New York: M-..t eel
Ocklu....; 1999:217-238.
A pod fJlJrrvUrA! oj (mU~K il/lpltnOrNwJy Clntllwt.c, 10 ditl{tWJtf1I1~1/t,
Till JE 1 McCullo,h [A, utrc:tt measLlremcnl of the ril.dbdOI1 IC11:\iliYity Qf normal
moUSet b01U~ ftk.'\rrow cdb. RadiICIR"J 1~(jliI1:21~.
Th" dud, ""d f."j (1911/) 114"' lit, F,I ajl"j""nlol..ntimt<f.,,'" hlm",',i<al fl.", ",a,."dn
,{ Itnultopoie<ll. ,lie mod<! Ihol A" w.o,
,1Iv.wped and.JlluUI."", W "",, J' , .. ,t.
TsalS. Sicff CA, et a1. Strumal celllU,",d'll.lCd c.rythropoiesis. BkJo4 1986;67: 111'A.
A br'.'iJi" Pafle<"""""",,,'hroJd uI4 dIv.w;mg on fi"""'ID.''''id ""d -09';'
TMflrd mui)! dmtmsitaling W InUmdU dna d)'"foU'ilic fJll~'i4k1gir. 4uuciawmof ~l1Irnid
Gdh""""'W,
MU 1lIfIA dtN .rfnlJn41 mfCl'lU'nWOmftnH. i,. rrrJ lImA
Wflklt L\'I'. C.U "net Tww BI'oIiJJtj: A TtdbooII: D/ Hi.st"JfJ1a, 61.h ed, !bltlmon::: Ulban "TId
Schwarunbefi; 1988.
":It'
Ap o/Q IIdWd "df1,lIrJtftlJwlIdJ.'td mndtr0lcJl alld Itmt"bi(;lu(:i, il ~fvlrll'dJy writ/en IJntl
f4uirhty illustrated ~!i.h diaetrl"u ~nd flluJlumitr'fJflVtj'Jhs lh/JI mllt:mrl l'Un,intfrtnt frrasfl,
Complete Blood Count (CBC)

Macrocytic Anemias Measured values


9 Red cell number (~5 X 106/mm3)

2009 9 Hemoglobin (g/dL)


9 Mean cell volume (MCV)
Values calculated
Roy E. Smith MD, MS 9 Hematocrit
Division of Hematology/Oncology 9 Mean corpuscular hemoglobin (MCH)
9 Mean corpuscular hemoglobin concentration (MCHC)
smithre@upmc.edu 9 Red cell distribution width (RDW)

Red Cell Distribution Width


(RDW) MCV
Information routinely
provided on CBC
<80 fL 80-100 fL >100 fL
Frequency distribution
of cell volume (either
normal or high)
High RDW suggests
populations of varying Iron Deficiency Aplastic anemia B12 or folate deficiency
(anisocytosis))
volumes (anisocytosis Thalassemia Pure red cell aplasia Liver disease
Chronic Inflammation Chronic inflammation Myelodysplasia
Lead toxicity Renal or endocrine disorder Increased reticulocytes
Sideroblastic anemia Infiltrative disorders

Learning Objectives
Causes of Macrocytosis
Know the differential diagnosis of
macrocytic anemias Hematologic Diseases
Understand the absorption, transport, Folate or B12 def
storage and utilization of cobalamin (B12) Myelodysplasia: Refractory Anemia (RA)
Understand the absorption, transport and Ringed Sideroblasts (RARS)
utilization of folic acid Increased reticulocytes in response to
Know the causes of cobalamin and folate bleeding or hemolytic anemia
deficiency

1
Macrocytosis
Causes of Macrocytosis
300 pts with MCV > 100 fL
Non-
Non-hematologic Disorders Drugs - 37%
Alcohol consumption Alcohol with liver disease - 13%
Liver disease Alcohol without liver disease 13%
Hypothyroidism Reticulocytosis 8%
B12 or folate deficiency 6%
Drugs (cytotoxic, anti-
anti-folate
(trimethoprim), hydroxyurea) Primary marrow disorder 6%

Am J Med Sci 2000; 319: 343-


343-352.

Macrocytosis Macrocytic Anemias


MCV>110 Problem is decreased nuclear division leads to
larger cells or abnormal membranes
B12 or folate deficiency 74%
Differential Diagnosis
Reticulocytosis 33%
9 Deficiency of B12 or folate (nuclear division)
Primary marrow disorder 33% 9 Myelodysplastic syndrome (nuclear division)
Drugs 31% 9 Hypothyroidism (nuclear division)
Alcohol w liver disease 30% 9 Liver disease (membrane lipid problem)

Alcohol w/o liver disease 3% 9 Reticulocytosis (larger cells increasing MCV)


Am J Med Sci 2000; 319: 343-
343-352.

B12 (cobalamin) Deficiency Paul Erhlich


Carl Weigert (his cousin)
Animal proteins contain cobalamin developed tissue staining
method using aniline dyes
Vegans are the only group in whom dietary
As med student working in
insufficiency should be considered a corner of Waldeyer
Waldeyers lab
Erhlich applied method to
Cobalamin requirement are low and stores staining blood smears
within liver can take years to deplete Ich probe
probe I am
messing about.

Lancet 2000; 111: 407.

2
George Whipple George Minot
Used stains to discriminate Like Whipple was also
blood cells interested in diet and
anemia
Used food supplements to Took detailed history from
correct anemia in dogs patients with anemia and
induced by venesection found that they often
lacked meat
Found that
With William Murphy,
supplementation of diet began feeding patients
with liver was most various diets and followed
effective reticulocyte counts
Found that liver was best
With Whipple and Murphy
won Nobel in 1934

William Castle Cobalamin Absorption


Fed patients with
pernicious anemia (PA) Dietary cobalamin is bound by R proteins (saliva;
beef muscle w/o response gastric juice) and by intrinsic factor (IF)
Fed patients with PA
gastric contents of normal Gastric acid cleaves R proteins but not IF from
subject who had ingested
beef and saw response cobalamin
Fed patients with PA
extract of gastric acid + Cobalamin/IF is absorption at terminal ileum
beef ex vivo and saw
response Cobalamin transported in blood by transcobalamin
Concluded that there was
something in gastric juice II (TCII)
(intrinsic factor)
factor) that was
needed for absorption

3
Functions of Cobalamin
Participates in the generation of methionine
from homocysteine
Participates in the generation of succinyl Co
A from methyl malonyl Co A
Both substrates increase in cobalamin
deficiency
Both products are involved in generation of
lipids involved in myelination

Causes of Cobalamin Deficiency Gastric Atrophy


Defective release of cobalamin from food ..\
..\My Documents\
Documents\My Pictures\
Pictures\3880769f9.jpg
acid)
(decreased gastric acid)
Pernicious anemia - decreased IF
Post-
Post-gastrectomy
Intestinal organisms - blind loops or
pseudo-
pseudo-obstruction; fish tapeworm
Ileal diseases - sprue, TB, lymphoma
Nitrous oxide - inhibits methionine synthase

4
Cobalamin Deficiency B12 or folate deficiency
Signs and Symptoms
Macrocytes
Hematologic - macrocytes (RBC),
hypersegmentation of neutrophils (>6
lobes), marrow megaloblasts
Neurologic - paresthesias, ataxia, dementia

Hypersegmented PMN

Intramedullary Hemolysis Schillings Test


(ineffective erythropoiesis)
Formerly done to determine site of
Reticulocytopenia
abnormality (PA vs gut)
Evidence for hemolysis
Megaloblastic changes in intestinal mucosa
elevated LDH limited reliability
low haptoglobin Availability is decreasing
indirect bilirubinemia

Antibody Testing PA
Methylmalonic Acid (MMA)
85% positive for anti-
anti-parietal cell antibody
Unlike serum B12 levels or Schilling
Schillings
(sensitive but not specific) test, MMA levels in serum or urine reflect
tissue B12 activity
Because B12 is protein bound,
50% positive for anti-
anti-intrinsic factor measurement of free B12 is problematic
antibody (specific but not sensitive) While both folate and B12 deficiency
elevate homocysteine levels, only the
latter increases the MMA level

5
Diagnosis of B12 Deficiency Cobalamin Deficiency
Treatment
Monthly injection of cobalamin (old)

Oral supplementation is probably adequate since daily


requirement is 1 microgam

1% absorption of 1 mg ingested by passive diffusion would


be enough (remember the liver experiments)

Treatment of B12 Deficiency Folate Deficiency


Oral initial therapy 1,000-
1,000-2,000 mcg daily Present in green vegetables, fruits, beans
maintenance therapy 1,000 mcg daily
Parenteral 1,000 mcg daily or qod for 2 wks Requirements are 10-
10-20X higher than
maintenance therapy 1,000 mcg q1- q1-3 cobalamin
months
Stores are depleted within months of
Relative costs:
costs: tablets are $4.00; IM is $0.11
but the additional costs of office visit makes cessation on intake
it $60-
$60-100/visit

Folate Deficiency
Polyglutamated folates are broken down into
monoglutamates on intestinal mucosa by
conjugases
Absorbed into intestinal cells and converted
to N5-methyl tetrahydrofolate which is
transported in blood stream
Involved in generation of methionine and
thymidine

6
Folate Deficiency
Signs and Symptoms

Hematologic - same as cobalamin


Neurologic - no neurologic manifestations
of folate deficiency

Folate Deficiency Folate Deficiency


Laboratory studies Causes
Inadequate intake - alcoholism
Serum folate - labile; fluctuates with recent Increased demand pregnancy, psoriasis
Malabsorption - sprue
meals
Drugs:
Drugs: phenytoin (affects absorption),
Red cell folate - more constant due to methotrexate (inhibits dihydrofolate
reductase)
glutamination within cell

7
Sprue (gluten enteropathy) Celiac Disease

Normal intestinal villi

Sprue

Macrocytic Anemias Myelodysplasia (MDS)


Problem is decreased nuclear division leads Macrocytic anemia with low reticulocyte count
to larger cells or abnormal membranes >50% of patients are >70 years old
Differential Diagnosis Symptoms relate to cytopenias:
9 Deficiency of B12 or folate (nuclear division) red blood cells fatigue
9 Myelodysplastic syndrome (nuclear division) white blood cells infections
9 Hypothyroidism (nuclear division) platelets bleeding/bruising
9 Liver disease (membrane lipid problem) Pathogenesis is unclear but marrow shows
9 Reticulocytosis (larger cells increasing
ineffective differentiation of cells and may relate to
MCV) increased apoptosis of differentiated cells

Review Review
Decreased nuclear division leads to larger cells Cobalamin may be stored for years.
Macrocytosis is determined by both visual Folate is found in vegetables
inspection and the MCV
pgFolates are reduced to mgFolate by
The MCV can be used to catagorize the different
causes of Macrocytosis conjugase in intestinal mucosa
Cobalamin is found in animal proteins Folate stores may be depleted in months
Cobalamin is bound to R protein and IF Cobalamin def. MMA and
Cobalamin-
Cobalamin-IF is absorbed at the terminal ileum Homocsyteine
Folate def. Homocysteine only

8
Review
Discussed the treatment of cobalamin
deficiency (oral vs parenteral B12
replacement)
Discussed the treatment of Folate deficiency
Introduced the concept of MDS

9
Macrocytic Anemias

Folic acid and vitamin B12 deficiency are primary drofolate. This situation creates a "methylfolate trap"
causes of macrocytic anemia in adults. Both vitamins and reduced availability of tetrahydrofolate to support
are essential for normal DNA synthesis, and high DNA synthesis.
turnover tissues such as marrow are especially sensitive
to any deficiency state. The marrow becomes mega-
loblastic; marrow precursors appear much larger than
Absorption & Distribution of Vitamin B12
normal and are unable to complete cell division. This Pathways of absorption and distribution of vitamin
results in ineffective erythropoiesis, release of macro- B n to tissues are shown in Figure 8-2. Food B12 is ini-
cytic red blood cells into circulation, and worsening tially bound to a salivary binding protein (an R-pro-
anemia. The severity of the anemia and the degree of tein) until it reaches the small bowel, where pancreatic
macrocytosis depends on severity and duration of the proteases release the vitamin for subsequent binding to
deficient state. the glycoprotein, intrinsic factor. The Bu-intrinsic fac-
Prevalence of folic acid deficiency depends on the tor (cobalamin-IF) complex then binds to a receptor
frequency of diseases associated with a decreased di- on ileal mucosal cells and is transported across the gut
etary intake of folic acid, malabsorption, or an in- wall to circulation. In the absence of intrinsic factor,
creased requirement. Alcoholism is a common cause of vitamin B12 absorption virtually ceases. Several transco-
folic acid deficiency in Western societies because of the balamin proteins in circulation are capable of binding
poor dietary habits of the alcoholic and alcohol's inter- free B 12 However, transcobalamin II is the principal
ference with folate metabolism. In developing coun- transport protein for deliveryof B12 to tissues and liver.
tries, tropical and nontropical sprue are more common Daily turnover of vitamin B12 reflects tissue require-
etiologies. Vitamin B12 deficiency can result from a di- ments and size of body stores and can rangefrom as lit-
etary deficiency, an autoimmune process directed at in- tle as 0.5 to as much as 8 ~lg per day, From 1 to 10 mg
trinsic factor, or anyone of a number of gastrointestinal of vitamin B 12 accumulate in liver stores in a normal
disorders that lead to vitamin Bl2 malabsorption. adult on an adequate diet,

NORMAL FOLIC ACID Absorption & Distribution of Folic Acid


& VITAMIN 8 12 METABOLISM
Dietary folic acid follows a similar pathway (Figure
Major metabolic pathways offolic acid and vitamin B 12 8-3), An essential step in absorption is hydrolysis of fo-
are illustrated in Figure 8-1. These two vitamins are late polyglurarnates present in food to rnethyltetrahy-
closely linked in the support of DNA synthesis. Within drofolate monoglutamate. This process depends on a
the cell, vitamin B12 is present in two forms. As de- carboxypeptidase located on the mucosal cell mem-
oxyadenosyl B12' it supports conversion of l-rnethyl- brane and a dihydrofolate reductase enzyme in mucosal
malonyl-CoA to succinyl-CoA. It also accepts a methyl cells. Most absorption occurs in the proximal portion
group from methyltetrahydrofolate to support synthesis of the small intestine. The rnethyltetrahydrofolate is
of methionine. The transfer of a methyl group from then rapidly transported to tissues to enter the intracel-
methyltetrahydrcfolate provides the tetrahydrofolate nec- lular metabolic cycle required for purine and pyrimi-
essary for synthesis of various folate coenzymes needed dine metabolism and DNA synthesis. Although there
for purine and glycine synthesis and for conversion of are proteins in plasma that bind folate, their primary
deoxyuridylate to thymidylatefor DNA synthesis. affinity appears to be for nonmethylared congeners that
A lack of either vitamin interferes with DNA syn- are not essential for transport to tissues.
thesis. When methyltetrahydrofolate is in short supply, Both methylated and non methylated congeners of
tetrahydrofolate cannot be generated to support other folate are absorbed by the liver, where they are stored as
folate coenzymes. When vitamin B I2 is lacking, there is merhylretrahydrofolatc polyglutamate. Depending on
no acceptor for the methyl group from merhylterrahy- the level of folate in the diet, the liver can contain sev-

95
96 I CHAPTER 8

DIET
I

FoodB12
B12
stores
l-lOmg

"ERe"
t
/,3-8 j.lg
J Liver

TCII.B12~

Tissues ,.
B12
( + 1ethYlfolate

Methionine

Figure8-2. Vitamin8 12 absorption and transport to tissues. Dietary vitamin 8 12 is sequentially bound to R-pro-
tein and intrinsic factor in preparation for binding to receptors on ileal mucosal cells. It is then transported on TC II to
liver and tissues.
MACROCYTIC ANEMIAS I 97

Food folate

Folate stores
+CH3/glutamates
... Liver
I
I
I
I
Binding
~=-+-.. CH3H4Pte Glul -; proteins

Tissues

H4 P t e Glur
~
DNA synthesis

Figure 8-3. Folic acid absorption and transport. Folate polyg Iutamates in food are hydrolyzed, reduced, and
methylatedto form methyltetrahydrofolate monoglutamate.This isthen transported to tissueswhere it participates
actively as a methyl donor and as substrate for both purine and pyrimidine metabolism and DNA synthesis. Liver fo-
late stores playan important role in providing a constant supply offolate through the enterohepatic cycle for reab-
sorption.

eral milligrams of folate stores. The liver also plays an from meat and meat by-products; vegetables are essen-
essential role in providing a constant supply of folate to tially free of the vitamin. Usually; the daily requirement
tissues. It maintains an active transport of methylte- of 3-5 /-lg is easily supplied by a Western diet. Only
trahydrofolate into bile for reabsorption by the gut. strict vegetarians are at risk for a true dietary deficiency
This enterohepatic cycle of folate is important for state.
maintaining folate hemostasis. Any interference with
the ability of the liver to store and release folate into CLINICAL FEATURES
bile or with the reabsorption of folate returned to the
gut can rapidly disrupt folate supply to tissues. Symptoms and signs of a severevitamin Bl2 deficiency
include those of marked anemia and neuropathy. Most
often, patients complain of the gradual onset of fatigue,
FOLATE & VITAMIN 8 12 NUTRITION
exercise intolerance, and progressive cardiacdecompen-
Most foods, especially leafy vegetables, are rich in fo- sation, The latter is associated with a severe anemia (a
late. However, excessive cooking and food processing hematocrit less than 15-20%). Vitamin BI2 deficiency
can significantly reduce folate content, so that an un- also has an impact on the central nervous system. Pa-
fortified Western diet may provide as little as 50-100 tients develop a demyelinating lesion of the neurons of
).lg per day. This amount barely meets the minimum the spinal column and cerebral cortex. This condition
daily requirement of an adult and may be insufficient results in paresthesias of the hands and feet, unsteadi-
during pregnancy or for patients with high rates of cell ness of gait, and eventually memory loss and personal-
turnover (hemolytic anemias). Chronic alcoholics, ity changes. The most distinctive characteristics of the
whose principal source of calories is derived from their neuropathy are the.~Esal column a~d corticospinal
alcohol intake, are at great risk of developing folate de- tract lesions (subacute combined degeneration of the
ficiency. spinal cord), which result in a loss of vibration and po-
Dietary supply of vitamin B 12 is derived largely sition sense and the development of an unsteady gait
98 I CHAPTER 8

and a positive Romberg test. Neuropathy may be pres- Table 8-1. Laboratory studies in the diagnosis
ent without anemia. It is important therefore to con- of macrocytic anemias.
sider vitamin Bn deficiency as a possible cause of a
peripheral neuropathy, dementia, or a psychiatric dis- Screening tests
order. Complete blood count/reticulocyte index
Other symptoms and signs of vitamin B12 deficiency Marrow aspirate
include complaints of a sore mouth and loss of taste, Ironstudies-SI,TIBC, serum ferritin level
in addition to atrophy of the mucosa of the tongue. Multilobed polymorphonuclear leukocyte count
The tongue appears srqq()th, red.and shiny on physical Serum/urine metbylmalonlc acid and homocysteine levels
examination. In addition; patients who develop perni- VitaminB 12 deficiency
cious anemia often have vitiligo and are prematurely Serum cobalamin (vitamin BIZ) level
gray. Several disorders of the gastrointestinal tract can Transcobalarnln protein levels
result in vitamin B12 deficiency. The most common Serum antiparietal cell and anti-intrinsic factorantibody
causes in the United States includ(Lper.t!i.c;io~saI1.eD1i? assays
associated with antibodies to parietal cells, intrinsic fac- Schilling test
~ tor, or the cobalamin-IF complex; gastrectomy,gastric DU suppression test
. bypass, or bowel resection; bacterial overgrowth of the Folic acid deficiency
small Intestiner and pancreatic insufficiency. Serum andred blood cell folate levels
<, Patients with folic'add deficiency-anemia often go
undiagnosed, especially alcoholics who have a very
poor diet and maintain blood alcohol levels above 100
rug/dL for extended periods. At this level of alcohol in- vitamins to tissues and enjoy the greatest popularity
take, the enrerohepatic cycle of folate supply to the in- as screening tests to identify clinically important
testine and tissues is impaired, setting the stage for a fo- deficiency states. Other tests, including serum methyl-
late deficiency anemia. However, unless anemia is malonic acid and homocysteine levels, and measure-
severe, the patient is relatively insensitive to its symp- ments of plasma transcobalamin protein and trans-
toms when compared to the other problems associated cobalamin saturation can help when the diagnosis is
with alcoholism. Diagnosis is also made difficult by unclear, Measurements of anti-intrinsic factor antibod-
rapid return of intracellular folate metabolism to nor- ies and the absorption of radiolabeled vitamin B12
mal once alcohol ingestion ceases. Within hours of al- (Schilling test) are used to determine the pathophysiol-
cohol withdrawal and res;;1P-Pti on of a normal diet, the ogy of a malabsorption defect.
serum folate returns t~orinal}nd the megaloblastic
defect begins to resolve,!'4'h~rerore, clinicians must be A. SERUM AND RED BLOOD CELL FOLATE LEVELS:
highly suspicious of the possibility of folate deficiency The serum folate level is exquisitely sensitive to the in-
in alcoholics and perform the workup while the patient take of dietary folate. Normal serum folate levels range
is still inebriated. from 5 to 30 ng/mL, but can be even higher with the
Folate deficiency during pregnancy is associated intake of folate rich foods. A minimal level of at least 4
with a high incidence of fetal developmental abnormal- ng/mL is required to sustain normal DNA synthesis. As
ities, especially neural tube defects. Peripheral neu- a rule, stores of folate in the liver will via the entero-
ropathies and neuropsychiatric disorders (dementia, hepatic cycle sustain the serum folate level during peri-
psychosis, and depression) in adult patients who are fo- ods of poor folate intake. Stores are limited, however,
late deficient and have normal vitamin Bl2 levels may and can be <J~Jl!eted.JYtr.hinafs:y,,~~,~I~s oLc!iet~ty..re"
also be observed. Subacute degeneration of the cord, miEtion, rnalabsoijicion, or chronic alcohol ingestion.
usually associated with vitamin Bt 2 defciency, has been The serum folate level will also quicldy return to nor-
reported in the occasional patient with folate defi- mal when alcohol is withdrawn andlor food intake re-
ciency. The difference in incidence may have more to sumes or when a folate supplement is provided. In this
do with the duration of the deficiency state than the vi- circumstance, it is still possible to detect a folate-defi-
tamin involved. cient state by assaying red blood cell folate, which re-
flects the state of folatestrppty at the time the cells were
Laboratory Studies first produced. However, intracellular folate is stored as
folate polygluramare, which must first be hydrolyzed to
Accurate diagnosis of folic acid or vitamin E I2 defi- the monoglutarnate prior to assay. This preparation
ciency requires several laboratory studies (Table 8-1). step is difficult to standardize, making the measure-
Serum folate and serum cobalamin levels provide the ment less reliable. In addition, the red blood cell folate
most sensitive measure of ongoing supply of these lev~L~dll""Q!l!xJKal:>normal if the patient has been fo-
MACROCYTIC ANEMIAS 99

late deficient for some.period.and.bas.not received a chance of a metabolic abnormality suggestive of a


~r~l1sfusiOn.AfaiselY low level is also seen in 50-60% subclinical deficiency state. From 3.0% to 35% ?[
of patients with primary vitamin B12 deficiency. apparently normal individuals WIth cobalam1l1
levels in this range will have anabnonm~l methyl-
B.SERUM COBALAMIN (VITAMIN Bu) Level: malonic acid levelwithout hematologic abner-
The normal range for serum cobalamin levels is most mafities:-" -
often stated to be from 200 to 500 pgl mL (ng/L). The Misleading cobalamin levels are seen in certain
traditional lower limit of normal of 200 pg/mL has clinical settings. Falsely low measurements have
been challenged, however, based on the ability to detect been reported with normal pregnancy, oral con-
subclinical cobalamin deficiency with the methyl- traceptive use, multiple myeloma, and ~rans.cobal
malonic acid and homocysteine assays. It is more accu- amin I deficiency. Patients who are folic aC.ld-d~
rate to state the probability of a metabolic abnormality ficient or are taking large doses of ascorbic acid
according to the serum cobalamin level, as follows: can have low serum cobalamin levels because of
interference with vitamin B 12 absorption and me-
Patients presenting with hematological findings tabolism. Conversely, normal or elevated cobal-
typical of a vitamin B12 deficient macrocytic! amin levelsare seen in patients with inborn errors
megaloblastic anemia haveserum cobalamin levels of cobalamin metabolism, isolated transcobal-
below 200 pg/mL 95% to 97% of the time. In ad- amin II deficiency, and high levels of plasma
dition, the more severe and protracted the anemia rranscobalamin I and III secondary to severe liver
the more likelythe level will fall below 100 pglmL. disease or a myeloproliferative disorder.
From the opposite viewpoint, 60% to 80% of in-
dividuals with serum cobalamin levels below 200 C. METHYLMALONIC ACID AND HOMOCYSTEINE LEVELS:
pg/mL will have anemia or metabolic evidence of Methylmalonyl-CoA mutase and methionine synthase
vitamin B l2 deficiency (abnormal methylmalonic are both cobalamin-dependent enzymes (Figure 8-4).
acid and homocysteine levels). Merhyltetrahydrofolate is required as a methyl donor to
When the serum cobalamin level is between 200 cobalamin in the methionine synthase conversion of
and 350 pg/mL, there is still a considerable homocysteine to methionine. Therefore, low cobalamin

1M M Mutase I
Methylmalonyl CoA . . . . Succinyl eoA

t
ADOCBL

I
COBALAMIN
5 - methyl THF
I Methionine
\MTHFRI( Cob(II)alamin

5, 10 methylene I
THF IMSR I
'-THF
METHYLCOBALAMIN
t Homocysteine

Figure 8-4. Key enzymes in the cobalamin metabolic pathways. Methionine synthase (MS) catalyzes the syn-
thesisof methionine from homocysteine using the methyl group transferred from methyltetrahydrofolate via methyl
cobalamin.Methylenetetrahydrofolate reductase (MTHFR) catalyzes the reduction of methylenetetrahydrofolate to
methyltetrahydrofolate, the obligate methyl donor In the formation of methylcobalamin.The methionine synthase
reductase (MSR) enzyme maintains coba lamin in its active form. Methylmalonyl-CoA mutase (MM Mutase) with
adenosylcobalamin (ADOCBL) as cofactor catalyzes the conversion of methylmalonyl-CoA to succinyl-CoA.
I 00 CHAPTER 8

levels result in elevations of both methylmalonic acid The level of holo'TC II may be the most sensitive
and homocysteine, whereas folate deficiency is accom- measure of early negative vitamin B 12 balance, although
panied only by an increase in serum homocysteine. The this is far from proved. The original assay was a techni-
normal range for serum methylmalonic acid is 73-271 cally difficult one and, therefore, not available in the
nM/L (19-76 ng/mL), whereas the normal serum ho- majority of clinical laboratories, while commercially
mocysteine ranges from 5.4 to 16.2 nM/L. Serum available immunoassays have yet to be approved for
methyl malonic acid and total homocysteine levels are routine clinical use. Normal individuals have holoTC
elevated in 95-97% of vitamin B12- deficient mega- II levels greater than 50 pg/mL, whereas negative vita-
loblastic anemia patients (the same sensitivity as the min B12 balance has been associated with values below
serum cobalamin measurement). Methylmalonic acid 40 pg/mL.
levels are increased 2-100 times the upper limit of nor-
E. TESTS OF VITAMIN-DEFICIENT ERYJHROPOIESIS:
mal, and homocysteine elevations are 2-20 times nor-
mal. Although homocysteine elevations occur with From a morphologic standpoint, 'th~ earliest sign of vi-
both cobalamin and folic acid deficiency, less than 2% tamin B12 or folic acid deficiency is the appearance in
of folate deficient patients will have an elevation of circulation of hypersegmented polymorphonuclear
serum methylmalonic acid. Folate deficient patients leukocytes, which are leukocytes with five or more seg-
typically have elevations of homocysteine of 2-1 0 times mented lobes. In part, sensitivity of the leukocyte to vi-
normal. tamin deficiency reflects the more rapid turnover of
The assay for serum methylmalo nic acid is complex this cellular population. Red blood cell morphology
and expensive. Moreover, it is not available in most changes come on more slowly. At first, a few
routine clinical laboratories, making it less valuable in ,1acrooya.locytes can be detected on the peripheral film;
the immediate diagnosis of a deficiency state. Elevated rater on, as the' anemia worsens, the mean cell volume.
values also are seen in patients with renal insufficiency (MCV) becomes elevated.
and severe methylmalonyl mutase deficiency. Homo- Patients with severe anemia, macrocytosis, and a
cysteine levels are easier to measure, rise early in and megaloblastic marrow will manifest severe ineffective
correlate with the severity of the cobalamin deficiency erythropoiesis (Table 8-2). This condition is character-
state, and only normalize with cobalamin therapy. ized by marked poikilocytosis on the peripheral blood
However, they are far less specific. Folate and vitamin film, an increase in the serum lactic dehydrogenase
B6 deficiency are both associated with increases in (LDH) level, and an increase in the serum iron level to
serum homocysteine levels. In fact, slight elevations in the point of full saturation of the total iron-binding ca-
serum homocysteine are common in otherwise normal pacity (TIBC). There is also an obvious mismatch be-
populations and appear to respond to dietary folate tween the increased ratio of erythroid to granulocytic
supplementation. Other causes of serum homocysteine
elevation include renal insufficiency, folate enzyme
polymorphisms, alcohol abuse, certain drugs (eg, folate
and vitamin Boinhibitors-methotrexate, isoniazid), Table 8-2. Erythropoietic profile of severe
hypothyroidism, and inborn errors of homocysteine macrocytic anemia.
metabolism. Delays in sample processing also can
falsely elevated levels. Anticoagulated plasma should be Anemia Severe (Hgb < 8-10 g/dL)
used for assay and the blood sample must be cen- ~---- --------- ---.---.-._- --------- -. - ----.-- .. ------- ---.- - --.-- ----- - ------.. - --.-
trifuged within the hour of collection. MeV (fL) 110-140

D. SERUM VITAMIN 812 Binding Proteins: ~~~_~~~p'~.?_I~~~ . ~~_~~~~y'~~~~~~~~_~_:~_~?_~~~ _


Transcobalamin II (TC II) is the primary plasma-bind- Reticulocyte index <1
-----------------_.--------.------------------_.-----------_.-.-----------.------ ...
ing protein respons~ble for the transport of vitamin Bl2 Marrow
to-tissues. Cobalamin absorbed from food or given par- E/G ratio > 1:1
enterally is avidly bound ro TC II in gastrointestinal Morphology Megaloblastic
mucosal cells and serum. The cobalamin-TC II com- ------------ --- .- .. -----.- .....- - - ------------. ------ - ._ ... - - - -. - ----- - - --- - --- -
-~._-

plex gains access to cells by binding to a specific mem- Serum iron/TIBC Increased/normal
brane receptor that is then internalized and delivered to %Saturation >50
a lysozyme for digestion and release of free cobalamin.
The turnover of cobalamin-TC II complex (holo-TC Marrow iron stores Increased
II) in plasma is very rapid, in con trast to the slower Serum ferritin Increased
turnover of cobalamin and cobalamin analogues bound
to transcobalamin 1. Bilirubin/LDH M
M_~_~ ~M_MM. M_. ._M Increased/increased
~ __ . ._. ~ _
MACROCYTIC ANEMIAS I 101

precursors (E/G ratio) and reduced production of new factor complex, the Schilling test is repeated with the
red blood cells, as measured by the reticulocyte produc- simultaneous ingestion of 60 mg of purified, hog in-
tion index. trinsic factor. This should correct malabsorption sec-
In the research setting, the deoxyuridine (D U) sup- ondary to intrinsic factor deficiency but not the malab-
pression test can be used to directly test the DNA syn- sorption owing to small-bowel disease. To avoid a
thesis of marrow precursors. This method measures the false-positive result because of a vitamin deficiency in-
ability of the cells to convert radio labeled deoxyuridine duced malabsorption, the Schilling test should be per-
monophosphate to thymidylate. Because this reaction formed some days or weeks after vitamin therapy is ini-
requires adequate levels of both vitamin B12 and folic tiated.
acid, the DU suppression test can be made even more Accuracy of the Schilling test requires cooperation
specific by testing the effect of adding each vitamin to by the patient. Inadequate flushing with nonradioac-
the reaction mixture. An abnormal DU suppression tive vitamin B12 or an incomplete urine collection will
test that is corrected by the addition of cobalamin but lead to false-positive results. In addition, if the intrinsic
not methyltetrahydrofolate is a sensitive measure of factor preparation used in the second part of the
early vitamin B12 deficiency. However, this sophisti- Schilling test is not fully active, confusion can arise as
cated radioisotope technique is not available in most to the cause of the vitamin B)2 malabsorption. Finally,
clinicallaboratories. use of crystalline cyanocobalamin in the test will not
detect those patients who malabsorb food cobalamin
F. ANTI-INTRINSIC FACTOR ANTIBODY
secondary to hypochlorhydria or achlorhydria. Acid
AND SERUM GASTRIN:
gastric juice containing pepsin is essential to normal ab-
Autoantibodies against parietal cells, intrinsic factor, sorption of food cobalamin.
and cobalamin-intrinsic factor complex can be detected
in the sera of most patients with so-called "pernicious
anemia" the most common clinical form of vitamin B12 DIAGNOSIS
deficient megaloblastic anemia ..Antipazieral cell anti~ As with iron deficiency, diagnosis of vitamin B]2 01'
body appears to be the most sensitive for atrophic gas- folic acid deficiency relies heavily on laboratory tests.
'tritisleading to vitamin B12 malabsorption but is not as The concept of stages of deficiency including store de-
specific or reliable. Antibodies against intrinsic factor pletion, abnormal DNA synthesis without anemia
and cobalamin intrinsic factor complex are seen in (subclinical deficiency), and a fully developed defi-
60-70% of pernicious anemia patients and have a ciency state with a macrocytic anemiaalso applies to vi-
specificity of close to 100%. Serum gastrin levels, at the tamin Bl2 and folic acid deficiency. However, unlike
same time, should be normal; a lack of serum gastrin iron deficiency, available laboratory methods do not
casts doubt on the diagnosis. provide as clear-cut a definition of eachstate.
G. SCHILLING TEST:
Vitamin 8 12 & Folic Acid Store Depletion
The Schilling test can be used to help define the nature
of a vitamin B)2 malabsorption defect. Unfortunately, Accurate measurement of liver stores of either vitamin
the test is a difficult one and has fallen out of favor. B12 or folic acid would require a direct assayof liver tis-
Many hospitals are no longer set up to offer the test sue. There are no indirect measures of the quantity of
which involves the use of tracer isotope, radiolabeled stores of these vitamins similar to the serum ferritin
cyanocobalamin. The test involves oral administration level in iron metabolism. An exception to this state-
of 0.5 uc (0.5-2.0 ug) of radiolabeled cyanocobalamin ment is the use of the h,QloTc:JI)IOy,el (the amount of
while the patient is fasting, followed by measurements cobalamin bound to' TC iI)~S'al1 iiidicator of negative
of appearance of radioactivity in serum and urine. For vite!El~~13ubalance. When vitamiilB 12 intake is inade-
the urine test of absorption, 1 mg of nonradioactive quate and stores are progressively depleted, the holoTC
cyanocobalamin must be given intramuscularly (1M) 2 II level falls below 40 pg/mL, indicating supply is in-
hours after the ingestion of the isotope to saturate the sufficient to match the rate of vitamin B12 clearance
serum-binding proteins and flush the radioactive cobal- from the TC II protein. However, this correlation is not
amin into the urine. A 24-hour complete urine collec- fully proved and, if even true, still does not provide a
tion is essential. Normal subjects should excrete 7% 01' quantitative measure of total body store depletion.
more of the ingested isotope in the first 24 hours after Once vitamin stores are exhausted, serum cobal-
the flushing dose. amin and folic acid levels measured in the fasting state
To determine whether vitamin B I2 malabsorption is will indicate inadequate tissue supply. Serum cabal-
related to a lack of intrinsic factor or an interference ~illJ<tl1sJOleyds below 200-350 pg/mL and the
with intestinal absorption of the cobalamin-intrinsic serum folate to less than 4 ng/mL. This decrease re-
102 I CHAPTER 8

fleets the fact that liver stores must be present to sup- Moreover, the entire issue may be moot if and when di-
port normal serum vitamin levels in the fasting state. etary supplementation with vitamin B I2 is adopted.
To make this interpretation with any certainty, how-
ever, other causes of false elevations or depressions of Vitamin 8 12 & Folic Add-
the vitamin levels must be excluded. Deficiency Anemia
A full-blown macrocytic anemia secondary to vitamin
Subclinical Cobalamin Deficiency B]2or folate deficiency is relatively easy to diagnose (see
The observation that otherwise normal individuals can Table 8-2). With moderate to severe anemia, the
have elevated methylrnalonic acid and homocysteine 'I; MCV increases to values in excess of 110 fL and the
levels, which return to normal with vitamin B I2 ther- 'I: peripheral blood film shows a marked distortion of
apy, has led to the concept of "subclinical cobalamin ::morphology with macroovalocytes, aniso- and poikilo-
deficiency." The clinical implications of this finding cytosis, and multilobed polymorphonuclear leukocytes
have not been well defined. Some patients with sub- (see Color Plates 7 and 8). As the anemia worsens, the
clinical deficiency may go on to develop a neuropathy hematocrit can fall to levels below 15-20% (hemoglo-
without hematologic changes, suggesting a significant bin 6 g/dL or less), and both leukopenia and thrombo-
metabolic defect. However, this is a small fraction of cytopenia can develop.
the large number of such individuals who can be iden- Distortions of marrow morphology are equally im-
tified in screening programs. Up to 35% of individuals pressive. The marrow is strikingly megaloblastic with a
:wirh}e:rum"COhalarni!Llevels between 200 and 350 shift in the E/G ratio to 1:1 or greater. Most of the ery-
pg/mL can be shown to have elevated methylmalonic throid marrow proliferation is limited to the earliest
acid and homocysteine levels ",ith?ut anemia or neu- precursors (ie, the basophilic and polychromatophilic
ropathy. Moreover, in studies of elderly populations, up normoblasts). This is, in fact, a demonstration of the
to 15% of subjects can have evidence of a metabolic de- ineffective erythropoiesis that accompanies megaloblas-
fect, even while cobalamin levels are greater than 350 tosis. Erythroid precursors,are arrested in S phase, can-
pg/mL. not go through cell division, and die:withill marrow. , .
The implications of classifying individuals with This is reflected by a mismatch between tne E/G ratio'
serum cobalamins between 200 and 350 pg/mL and el- of 1:1 (greater than three tiroes normal) and a reticulo-
evations of serum methylmalonic acid as clinically ab- ~y!epr.:9duction index ofTess thari'\. Other laboratory
normal and, therefore, in need of a full diagnostic eval- signs of the ineffectiveness of erythropoiesis include ele-
uation for vitamin B12 malabsorption, are staggering. vations in serum LDH level and an increase in the
Because one third or more elderly Americans have serum iron to full saturation of the TIBC. There may
serum cobalamin levels below 350 pg/mL and a third also be a modest increase in the indirect bilirubin level.
of these also have minor elevations of their serum Serum cobalamin levels should confirm the diagno-
methylmalonic acid, close to 5 million elderly persons sis (Table 8-3). The patient with a pure vitamin BI2 de-
would be eligible for evaluation. Not only is this be- ficiency and a well-developed macrocytic anemia will
yond availableresources, but it also ignores the fact that almost certainly have a serum cobalamin level,below
the clinical benefit of finding, evaluating, and treating 200 pg/mL, with more 'severe anemia patients falling
subclinical cobalamin deficiency has yet to be shown. below 100 pg/mL. At the same time, the serum folate

Table 8-3. Test of vitamin B12 and folic acid deficiency.

_~~ __ ~_~ ._w .. .__._... . Vitamin 812 Deficiency . . Folic Acid Deficiency
. _
Serum 812
(normal> 200 pg/mL) < 100 >200
Serum folate level
(normal> 4 ng/mL) >4 <4
Serum methyl malonic acid
(normal < 270 nM/L) 2-100xnormal Normal
Serum homocysteine
(normal < 16 nM/L) 2-20x normal 2-10 x normal
MACROCYTIC ANEMIAS { 103

level will be normal or increased. If the cobalamin level from a bleeding episode. In this situation, the only clue
is low, measurements of serum methylmalonic acid and to a prior deficiency in folic acid will be the occasional
homocysteine are not needed; they only add expense. rnacroovalocyte on the peripheral blood film and a
Patients with severe liver disease or myeloproliferative lower than normal red blood cell folate level.
disorders who have very high levels of transcobalamin Approach to diagnosis is also influenced by the
I/III are exceptions to this rule. severity of the anemia. If the patient requires immedi-
The diagnosis of a macrocytic anemia secondary to ate treatment, it is essential that all necessary laboratory
folic acid deficiency can be more difficult. This diagno- studies be drawn prib1:\to transfusion or vitamin ad-
sis is especially true for the alcoholic patient where ministration. The tests should include a CBC, reticulo-
there is a close relationship between the patient's diet, cyte count, marrow aspirate for morphology and assess-
level of alcohol ingestion, and serum folate level. Alco- ment of iron stores, serum cobalamin and folate levels,
hol has a dramatic and rapid effect on the serum folate and serum iron, TIBC, and ferritin levels.This battery
level. At bloodalc(}hoLLevelsQfJQo..I11;/~L or higher, of tests should make it possible to confirm the defect in
release o'FloIarefiom to
hepatic stores recycle through cell maturation, distinguish vitamin B t2 from folic acid
the enterohepatic pathway for tissue supply is impaired. deficiency, and provide an assessment of iron stores.
In the patient who has little or no dietary intake of fo- The latter is especially important in planning therapy.
late, this results in a rapid and sustained fall in the Patients with small-bowel malabsorption can present
serum folate level to values below 4 ng/mL Thus, an with a single deficiency of vitamin BJ2> folic acid, or
acute alcoholic can show low serum folate levels within iron or any combination of the three. Moreover, it is
a matter of a few days of a drinking binge, even though not uncommon that therapy with vitamin B12 and folic
liver folate stores are not exhausted. acid, alone or in combination, will uncover an iron de-
This phenomenon is easily detected if a serum folate ficiency state.
level is drawn while the patient is still inebriated. How- In patients with less-marked anemias, vitamin B 12
ever, a delay of a day or two following withdrawal of al- and folic acid therapy can be used diagnostically. This
cohol and/or placing the patient on a normal diet will therapy is most applicable for patients with vitamin B 12
result if), a.rise-in the serum folate to normal levels. De- deficiency owing to an autoimmune process (perni-
lays of several days or a week can result in an even more cious anemia), where folic acid and iron deficiency are
confusing picture. By this time, the patient's abnormal unlikely. To perform a thera.eeutic trial, the patient is
marrow morphology will have disappeared. Moreover, given 1-10 )lg vitamin~ p-arenterally each day for
the anemic patient will mount a reticulocyte response 10-14 days. The response to therapy can be measured
suggesting either a hemolytic anemia or a recovery in several ways. As shown in Figure 8-5, the LDH level

Rx

LDH

Figure 8-5 Therapeutic response to
vitamin 8 12 or folic acid. The first sign of
an effective response to one or the other
vitamin is a fall in serum iron and LDH lev-
els.This reflects a correction of the pa-
tient's ineffective erythropoiesls.On or
about day 3, the reticulocyte index in-
creases.peaklnq by the eighth day. The
subsequent level of red blood cell produc-
tion and rate of hematocrit rise will reflect
the severity of the patient's anemia, ade-
quacy of iron supply, and the presence of o 7 14 21
other complicating illness. Days
104 I CHAPTER 8

and elevated serum iron


2-3 days as the patient's erythropoiesis becomes effec-
levi'f:;L~tPidlyoverthe,fJ.l:sJ: More severe macrocytosis with red cell poikilocyto-
sis and megaloblastic marrow morphology is seen not
tive. This decrease is followed on days 3-5 with an in- only in vitamin-deficienct patients but also with
crease in the reticulocyte count. Recovery of the hernat- myelodysplasia and in patients receiving chemothera-
'ocdt is much slower and takes several weeks. Although peutic agents such as hydroxyurea and methotrexate.
a response to small amounts of vitamin B' 2 strongly Just as with the vitamin deficiency states; these patients
suggests a vitamin B l2 deficiency state, it is not absolute have a true nuclear maturation defect. Their marrow
proof; serum vitamin levels are still important to con- morphology is abnormal and they exhibit varying de-
firm the diagnosis. grees of ineffective erythropoiesis. Abnormalities in
white blood cell and platelet components of the blood
count are also common.
DifFERENTIAL DIAGNOSIS
Macrocytosis also can be an artifact of the auto-
The differential diagnosis of a vitamin deficiency state mated cell counter technique. Patients with cold agglu-
involves not just identifying whether it is vitamin B 12 tinins or marked elevations of their white blood cell
or folic acid deficiency but also the cause of the vitamin count may be reported to have an MCV in excess of
deficiency. There is also the task of distinguishing the 110 fL. In the first case, it is related to clumping of red
macrocytic anemia associated with vitamin B 12 and blood cells in the counter, whereas with marked leuko-
folic acid deficiency from the macrocytosis observed in cytosis white blood cells are being counted as red blood
patients with dysplastic anemias, liver disease, hemoly- cells. Modest increases in the MeV can be seen in pa-
sis, and exposure to chemotherapeutic agents. tients with hyperglycemia. This condition is an artifact
of the dilution step when the cells are prepared for
Macrocytosis counting. Because of the high intracellular glucose con-
tent, cells swell rapidly when they are placed in the
Macrocytosis, an MCV greater than 100 fL, is seen in isosmotic diluent.
several clinical settings (Table 8-4). In patients with a
,hSH1c>!rhagi<;: or Iiemolyric al1:~mia, where there is a high Causes of Vitam in 8 12 Deficiency
"level of stimulation of the marrow by erythropoietin,
the release of rnarrow-rericulocytes with an MCV in ex- Evaluation of a patient with a vitamin B 12 deficiency
,cess()f 140 iL will increase the MCV. The higher the anemia demands consideration of several possible
reticulocyte count, the greater the increase in MCV. Pa- causes (Table 8-5). Dietary history may suggest poor
tients with reticulocyte counts greater than 20% can intake of vitamin B 12 However, only the strictest of
have MCVs in excess of 110-120 fL, and the red blood vegetarians and breast fed infants born to vegetarian
cell-volume histogram will clearly show the double mothers are at risk for developing a deficiency state. In
population of yery Iarge rericulocytes and normal ma- the case of the adult patient with anemia, an impair-
ture red blood' cells. Miicn less dramatic, uniform ment in vitamin B 12 absorption, either intrinsic factor
macrocytosis, MCVs of 95-105 fL, is observed in pa- deficiency or a ciefc::ct; in small-bowel absorption, is al-
tients with hyp() thyro idism and liver disease, In the lat- most certainly present. In children, congenital defects
ter case, it 'maY"' represent an accumulation of excess in transport and intracellular metabolism must be con-
membrane secondary to disturbed cholesterol metabo- sidered. Transport defects include defective intrinsic
lism-and is usually accompanied by the appearance of factor synthesis, impaired intestinal transport (Imers-
target cells in circulation. lund-Grasbeck syndrome) and TC II deficiency. Except

Table 8-4. Causes of macrocytosis.

~~_~._~~ ~ .__.. ~._.w .. . . . Mev.(fl) Morphology .


Normal 90 Normocytic
.-._-------.--_ ..... ------.--.---_ ... ---------.----------------------------.----------------------------------------_.-------------------------------------------.-_ ..
Reticulocytosis 90-110
- - - .... - - - - - - - - - - - . - - - - - - - - - . - - - - - - . - - - - - - - - - - - - - - - - - - - - __ w ~ ~
Polychromasia (shift reticulocytes)
w w w w ~ _

Liver disease
------~ .. ~ w ~w __ ~ w
95-110 w
Uniform macrocytosis, targeting
w w w w w_w _

Megaloblastic anemia
______ w ~ ~
100-130
- - - - __
Macroovalocytosis, marked poikilocytosis
w ~ . _ .. w_w _

Cell counter artifact 100-130 Red bloodcellagglutinationor marked leukocytosis


-------_ . -_._--------~-----._-------------_ .. _-~----------------------------~-------------------------------- - - - - - - - - - - - - - - - - - - - - - - - - - - - - .._--------- .._--------------

\
,;..._ - -
MACROCYTIC ANEMIAS I lOS

Table 8-5. Causes of vitamin B12 deficiency. logic and endoscopic studies. Patients with nontropical
sprue may be diagnosed by biopsy of the small bowel.
Congenital defects Presence of one of these conditions obviously sets the
Intrinsic factor deficiency stage for vitamin B12 malabsorption. To confirm this, a
TC [I deficiency Schilling test first without and then with intrinsic fac-
lrnerslund-Grasbeck disease tor can be performed. Any of the defects in vitamin B12
Juvenile pernicious anemia transport or absorption secondary to small-bowel dis-
ease will show abnormal absorption of crystalline BIZ in
Poorvitamin B12 intake (vegans)
Malabsorption both parts of the study. Intrinsic factor does not correct
Intrinsic factordeficiency the absorption defect.
Autoimmune-antiparietal cell/anti-intrinsic factor The patient with intrinsic factor deficiency can be
antibody (pernicious anemia) diagnosed using a combination of laboratory studies
Gastric surgery and the Schilling test. The patient presenting with a
Pancreatic insufficiency macrocytic anemia, a serum cobalamin level less than
Small intestine absorption defect 200 pg/mL, and apositiveanti-intrinsic factof.anti-
Crohn's disease body is almost certainlyan exaniple of vitamin B Izde~
Sprue -Hdency secondary to an intrinsic factor defect.This is a
Lymphoma relatively common diagnosis that is most frequently de-
Diverticulosis or blind loopwith bacterial overgrowth tected in adult patients in their later years (classic per-
Fish tapeworm nicious anemia). Intrinsic factordeftciency is seen in
Ileal resection children who are born with an intrinsic factor produc-
Zollinger-Ellison syndrome tion defect. Adults who have gastrectomies where a ma-
AIDS jor portion of the fundus of the stomach is removed or
Vitamin B12 destruction a gastric bypass operation will predictably develop vita-
Nitrous oxide exposure min B 12 deficiency secondary to intrinsic factor lack,
usually within 2-4 years if they do not receive par-
enteral vitamin B 12 on a regular basis. Some patients
with low serum cobalamin levels can be shown to have
for the TC II deficient patients where normal levels of abnormal food-BIZ absorption.
TC I are present, these conditions will have low serum
cobalamin levels. Diagnosis ofTC II deficiency requires
Causes of Folic Add Deficiency
assay ofTC II total protein. Inborn errors of lntracellu-
lar metabolism include abnormalities of the methionine The causes of folic acid deficiency are listed in Table
synthase, methionine synthase reductase, and methyl- 8-6. Poor dietary intake coupled with alcohol inges-
malonyl mutase enzymes (see Figure 8-4). Patients tion is an obvious cause of folic acid deficiency in the
with an impaired methionine synthase reaction present adult population. Obviously, the history and physical
with megaloblastic anemia, a normal serum cobalamin examination are important in identifying the alcoholism
level, and an elevated serum homocysteine. and the level of dietary intake. Accurate diagnosis of al-
The impairment of vitamin B 12 metabolism by ni- cohol-related folic acid deficiencyrequires an immediate
trous oxide oxidation of the cobalt atom of the vitamin workup of the patient, at a time when the impact of al-
and inhibition of methionine synthase is also suggested cohol on folate metabolism is still operative.
by the patient's clinical history. The resulting defect in There are clinical settings where even a normal diet
methionine and S-adenosylmethionine synthesis can will not maintain folate requirements, Patients with
have a major impact on both hematopoiesis and the high levels of cell turnover (eg, hemolytic anemias,
central nervous system. It occurs following exposure to Lpsoriasis, andexfoliative dermatitis) can develop folic
high levels of nitrous oxide anesthesia for several hours acid deficiency even while on a normal diet. This is also
or with repeated low-level exposures over a long time in true for llorIIlalPE\:gUlYlcY;.,Prophylactic supplementa-
at-risk individuals, such as dentists. tion with "folic acid is reco~mended for these situa-
A full evaluation of vitamin Bl2 absorption in- tions.
cludes a careful review of the clinical history and an As with vitamin B12 deficiency, disease of the small
evaluation of ~mall-boV{~1 anatClIllY and function. Pa- intestines can result in folic acid malabsorption. Pa-
tients with ileitis, ileal resection, small-bowel dysfunc- tients with sprue (especially tropical sprue), bacterial
tion secondary to diverticulitis and bacterial over- overgrowth, and short bowel syndrome are at risk for
growth, fistula formation, or sprue may be recognized developing a macrocytic anemia due to folic acid defi-
from the clinical presentation with the help of radio- ciency. In the case of tropical sprue, folate deficiency is
I 06 I CHAPTER 8

Table 8-6. Causes of folic acid deficiency.

Poorfolic acid intake Increased requirement


Dietary lack/alcoholism Hemolytic anemias
Goat's milk anemia Exfoliative dermatitis/psoriasis
-.~.~:~~.~:~~~~~~~~!~~.~- .._
.. ._..__._._. __ ~~~.~_~~_~_:L. __.._ _._._.__ _ _ __._..__ _._ __
Malabsorption Metabolic inhibition
Sprue (nontropical andtropical) Dihydrofolate reductase inhibitors (methotrexate, trimethoprim, etc)
Lymphoma Alcohol
Small-bowel resection Vitamin Cdeficiency
Crohn's disease
Anticonvulsant drugs
.._-------------_ .
~--~--~---~----~---_ _.--------------.--~---_ .. _-- .. -- .. _-----_._----_ .. _---------------------- - - - ---_..---.------_ -----_ - -- ..----- _----.._-
Loss/destruction
Hemodialysis
Tropical sprue
_ .. -_ .. _---- .. - - - _""OO .. __ .. _ ...... _ .. ._ .... .. _ .. .. .. _ .. _. __ .... __ ..... __ ..... ... _ .. __ .. __ " __ ...... _ ...... _

thought to play an integral role in the disease process. in several multivitamin preparations sold as nutritional
Treatment with folate is therapeutic for this condition. supplements. The selection of preparation, dose, and
Sev.eral drugs have an antifolate actio~. Me;~otrex-
;:t~~.trla!!l~erene-, and sulfamethoxazole withtrirnetho-
trea.tment ~ch.edule .must be appr.opriate to the cli~ical
settmg. Misdiagnosis can result In a treatment failure
"'*
pruD. (Bactrim) are competitive inhibitors of methyl fo- and worsening of the patient's clinical condition. This
late and DNA metabolism. Methotrexate given in fact is of special importance in the patient with vitamin
therapeutic amounts is the strongest of these agents and B12 deficiency because a delay in therapy or mistreat-
with long-term therapy can be expected to produce - I ment with folic acid may result in irreversible neuro-
macrocytosis and a pancyropenia'AnEisCll1vulsants,,"', logic damage. Therefore, both the nature and cause of
(phenytoin, primidone, and phenobailral) appear to 'the deficiency state must be well defined as a founda-
be weak inhibitors of folic acid absorption. These drugs tion for planning management.
can cause macrocytosis and anemia when given in high
A. SEVERE, LIFE-THREATENING ANEMIA:
doses to children.
As regards congenital defects, severe methylenete- The treatment of a patient with a severe macrocytic
trahydrofolate reductase deficiency is the most com- anemia will need to proceed prior to completing the
mon of the inborn errors of folate metabolism. Patients diagnostic workup. Once appropriate laboratory stud-
present as children or adults with neurologic abnormal- ies, including serum cobalamin, folate, rnerhylmalonic
ities, developmental delay, and thrombotic disease, acid, and homocysteine levels, are drawn, the patient
probably secondary to their high levels of homo.-. should receive full therapeutic doses of both vitamin
c:ystc;Xne. Because thymidylate synthesis remains un- B12 and folic acid, using the parenteral route to avoid
affected, they do not have a megaloblastic anemia. the issue of malabsorption. Fumlel'Iriote,this treatment
Merhylenererrahydrofolate reductase polymorphisms should be continued with daily administration of both
resulting in 30% to 60% reductions in enzyme activity vitamins until a response is evident or the diagnosis is
are also quite common. They ate cited as a primary cleat. If the patient is unstable from a cardiovascular
cause of mild hyperhomocysteinemia in population standpoint, he or she should receive a transfusion with
studies and, therefore, as risk factors for cardiovascular packed red blood cells. This must be done with cau-
disease, neural rube defects, and colon cancer. Because tion, however, since patients with vitamin Bl2 defi-
serum homocysteine levels in these individuals can be ciency are usually older and have marginal cardiovascu-
normalized with folate supplementation, this has led to lar compensation. Any sudden increase _ in. blood
even more enthusiasm for dietary fortification. volume as with a rapidred.blood cell transfusion call
indiice a fatal arrhythmia or acute congestive heart fail-
ure. Therefore, packed ted blood cells should be in-
THERAPY fused slowlyand carefullywith concomitant use of a di-
General Guidelines uretic or simultaneous phlebotomy to avoid increasing
the blood volume.
Folic acid and vitamin B12 are available in their purified Treatment with both vitamin B12 and folic acid does
forms for oral and parenteral use and are incorporated not interfere with the subsequent evaluation of the pa-
MACROCYTIC ANEMIAS I 107

tient for malabsorption. Radiologic and functional pregnant women should receive supplementation with
studies of the small bowel can still be performed once a multivitamin that contains folic acid.
the anemia is corrected. This includes the use of the
Schilling test to distinguish intrinsic factor deficiency Vitamin 8 12 Preparations
from an abnormality in small intestinal absorption,
Vitamin B 12 is available in its pure form, cyanocobal-
B.ASYMPTOMATIC DEFICIENCY STATES: amin, in concentrations of 30, 100, and 1000 ~lg/mL
Whenever possible, therapy should be targeted to the for 1M or deep subcutaneous injection. The dose ad-
specific deficiency state. In the patient with a mild to ministered will depend on the management plan. A
moderate macrocytic anemia, this will mean a delay dose of 1-10 Ilg 1M daily is typically used in therapeu-
until a full hematologic workup can be performed and tic trials where the purpose is to confirm vitamin BJ2
the serum cobalamin and folate levels ate measured. If deficiency or distinguish vitamin BIZ deficiency from
the patient presents with neurologic disease and no folic acid deficiency. When a diagnosis of vitamin BIZ is
anemia, measurements of serum methylmaloriic acid established, the patient with a seyere, macrocytic ane-
and homocysteine can suggest the diagnosis. It is these mia should be treated with at least .100 Ilg of
settings where a therapeutic trial with a specific vitamin cyanocobalamin daily for at least 2 weeks. This regimen
can also help confirm the deficiency state. For the vita- will guarantee a maximum therapeutic response and
min BIz-deficient patient, a therapeutic trial can be encourage rebuilding of liver vitamin B12 stores. Subse-
performed using 1-10 Ilg of vitamin BIZ given 1M quently, patients should receive 100-1000 ug of
daily for 10 days while monitoring changes in the cyanocobalamin once a month for the rest of their
methylmalonic acid level, or when anemia is present, lives. This situation will maintain their balance unless
the serum iron, LDH, reticulocyte count, and over sev- there is a high level of cell turnover. If this is true,
eral weeks, the hematocrit and MCV: Similarly, a thera- twice-monthly injections are advised. Patients. with a
peutic trial can be performed using 50-100 ug offolic vitamin BIZ deficient neuropathy should be treated
acid given parenterally on a daily schedule for 10-14 more aggressively with weekly or biweekly injections
days to confirm an isolated folic acid-deficiency state. for several months to encourage maximum recovery.
This low level of folic acid will not run the risk of ei- Administration of more than 100 Ilg of cyanocobal-
ther inducing a hematologic response or worsening the amin in any single injection exceeds the binding ca-
neurologic signs in a vitamin BJ2-deflcient patient. pacity oCTC II and results in a rapid clearance of most
Also, it will not correct the high serum methylmalonic of the excess vitamin B 12 into urine. Therefore, there is
acid level of vitamin B 12 deficiency. no advantage to using a dose of 1000 ug of cyanocobal-
As a practical point, therapeutic trials with small amin. At the same time, cyanocobalamin is very cheap
amounts of either vitamin B1Z or folic acid are difficult and the higher dose is harmless.
to perform and often hard to interpret because other Several vitamin preparations that contain vitamin
disease states can interfere with the response. If the pa- B I 2 are marketed as nutritional supplements. Some of
tient is iron deficient or has an inflammatory illness, these contain not only cyanocobalamin but also intrin-
the lack of iron ubmate and suppression of erythro- sic factor concentrate prepared from animal stomachs.
poietin will dampell the reticulocyte and hematocrit re- Theoretically, the latter should provide an effective oral
sponses. In addition, a therapeutic trial with low doses medication for the treatment of patients with poor ab-
of vitamin BJ2will delay the performance of a Schilling sorption secondary to intrinsic factor deficiency. How-
test since the performance of this test includes adminis- ever, its effectiveness cannot be assumed. Patients can
tering a flushing dose of 1000 Ilg of nonisotopic vita- develop refractoriness to these preparations perhaps be-
min BJ2. In effect, the flushing dose is therapeutic. cause of the appearance of an antibody against the ani-
mal protein. Oral maintenance therapy is possible us-
C.VITAMIN PROPHYLAXIS: ing very high doses of vitamin B12 on the rationale that
There are clinical situations where vitamin B12 or folic a small amount of it will leak across the intestine. Al-
acid therapy should be given prophylactically. These though this does work, patients will need to be moni-
situations include conditions where the rate of use of tored closely for any clinical evidence of recurrence of
either vitamin exceeds the dietary supply, and disease their deficiency. The cost of this increased level of ob-
states where malabsorption can be anticipated. For ex- servation can cancel any advantage of oral over par-
ample, patients who have had a gastrectomy will ex- enteral therapy.
hibitvirarnin B 12 malabsorption and should, therefore, Multivitamin preparations containing small amounts
receive 10ng~terl1l"vitamin prophylaxis. Not'mal preg- of vitamin B 12 are marketed as over-the-counter med-
nancy isariexarnple 'of an imbalance between folate ications. Some of these contain as much as 80 ug of
needs for fetal growth and folate supply in the diet. All cyanocobalamin combined with 500-1000 ug of folic
108 I CHAPTER 8

acid, ascorbic acid, and iron. They are very useful as doses of 1-5 mg regardless of the cause of the defi-
nutritional supplements in patients who are on an in- ciency state.
adequate diet or have an increased requirement. Prophylactic administration of folic acid in pa-
tients with poor diets or high levels of cell turnover
will vary according to the individual clinical situation.
folic Add Preparations Multivitamin preparations containing as much as
Pharmaceutical preparations of folic acid include 1 mg of folic acid are used in pregnancy and as a sup-
pteroylglutamic acid folate congener (folic acid, fol- plement for nursing mothers because as much as 50
vire) , and 5-formyltetrahydrofolate congener (folinic ~g of folate is secreted each day in the breast milk.
acid, leucovorin, cirrovorum factor). Both are formu- Patients with hemolytic anemias or exfoliative der-
lated for either oral or parenteral administration. Folic matitis are usually given a higher dose (l or 2 mg of
acid tablets (folvite) contain 0.1, 0.4, 0.8, or 1 mg of folic acid orally each day). Like vitamin B12' the safety
pteroylglutamic acid, whereas folic acid for injection range of folic acid is very large and no side effects are
contains 5 mg/mL. Pteroylglutamic acid must first be seen even with doses exceeding 20 mg per day. How-
reduced and methylated by either intestinal mucosal ever, there have been reports of an increase in the fre-
cells or hepatocytes before it participates in cellular quency of seizures in children who receive large
DNA metabolism. amounts of folic acid to prevent the antimetabolite ef-
Folinic acid tablets (leucovorin) contain 5, 10, 15, fect of anriepileptic medications.
or 25 mg of 5-formyltetrahydrofolate. The principal The long-term management of any patient with a
application of folinic acid is to circumvent the inhibi- vitamin B1r or folic acid-deficiency state requires peri-
tion of dihydrofolare reductase by the chemotherapeu- odic reevaluation to guarantee therapeutic effectiveness.
tic agent, methotrexate. Leucovorin will also correct the Since the deficiencies are most often the result of an
defect in thymidylate production both in patients who underlying defect in absorption, patients may need to
lack rnethylterrahydrofolate and those who are vitamin maintain their vitamin therapy for the rest of their lives
B12 deficient, It should be avoided when treating vita- or at least until the absorption defect has been cured.
min B12- deficient patients because it can correct the There is also a constant risk that multiple vitamin and
hematologic abnormality while allowing the neurologic mineral deficiencies will occur. This risk is most typical
findings to progress. of the patient with sprue or widespread intestinal dis-
The approach to folic acid therapy will vary accord- ease from other causes where the patient is at risk for
ing to the management plan. If a therapeutic trial is at- folic acid, vitamin BJ2> and iron deficiency Less com-
tempted, the patient should be given a daily parenteral monly, vitamin C deficiency can playa role. The pa-
injection of 50-100 Ilg of folic acid (folvite). This tient with scurvy can exhibit a macrocytic/megaloblas-
treatment will require a meticulous dilution of the 5 tic anemia secondary to a defect in intracellular folate
mg/mL commercial preparation in order to avoid a metabolism resulting from low cellular vitamin C lev-
high dose that might give a partial response in the vita- els.
min B12-def1cient patient. It is also important to recog- Periodic evaluations of patients receiving mainte-
nize the difficulties associated with a therapeutic trial of nance vitamin therapy should include a careful history
folic acid. Patients with alcohol-induced defects in fo- and physical to look for reappearance or progression of
late metabolism often demonstrate a spontaneous re- the patient's neuropathy, a complete blood count, iron
covery of their hematologic abnormality with alcohol studies, and measurements of serum cobalamin and fo-
abstention. This reflects the reversal of the alcohol-in- late levels.
duced defect in liver folate-store metabolism. Patients
with folic acid deficiency can also demonstrate a com-
bined defect in vitamin B[2 and iron metabolism, In BIBLIOGRAPHY
this situation, a therapeutic trial with a single vitamin
Allen RJ:-! er al: 0 iagnosis of cobalamin deficiency: 1. Usefulness of
will not give an interpretable response.
serum merhylrnalonlc acid and total homocysteine concen-
Patients with severe macrocytic anemias are best trations, Am] HematoI1990;34:90.
treated with a combination of 100 ug of cyanocobal- Carmel R: Current concepts in cobalamin deficiency, Annu Rev
amin 1M and either oral or parenteral doses of 1-5 mg Med 2000;51:357.
of folic acid. Generally; oral doses of folic acid are ade- Carmel R: Prevalence of undiagnosed pernicious anemia in the eld-
quately absorbed, even in patients with known defects erly.Arch Intern Med 1996;156:1097,
in small intestinal absorption. It is a water-soluble vita- Canuel R, James S]: Alcohol abuse: An important cause of hyper-
min that can cross the mucosal barrier by passive diffu- homocysteinemia, Nurr Rev 2002;60:215,
sion when given in large doses. Therefore, it is common Chana-in I er al: Cobalamin and folate: Recent developments, J
practice to treat patients with oral folic acid in daily Clin PathoI1992;45:277.
MACROCYTIC ANEMIAS 109

Clarke R er al: Screening for viramin B-12 and folic acid deficiency sensitivities of serum cobalamin, methylmalonic acid and ho-
in older persons. Am] Clin Nutr 2003;66:750. mocysteine concentrations. Am] HematoI1990;34:99.
Cooper BA, Rosenblatt DS: Inherited defects of vitamin B 12 me- Rosenblatt D, Fenton W: Inherited disorders of folate and cobal-
tabolism.Ann Rev Nurr 1987;7:291. amin transport and metabolism. In: Scriver CR et al: The
Green R, Miller ]W: Folate deficiency beyond megaloblastic ane- Metabolic Bases of Inherited Disease, 8th ed. McGraw-Hill,
mia: Hyperhornocysteinemia and other manifestations of 2001.
dysfunctional folate status. Sernin Hematol 1999;36:47. Stabler SP er aI: Clinical spectrum and diagnosis of cobalamin defi-
Hillman RS: Hematopoietic agents: Growth factors, minerals and ciency. Blood 1990;76:871.
vitamins. In: The Pharmacological Basis of Therapeutics, 10th Toh BH, vanDriel IR, Gleeson PA: Mechanisms of disease: Perni-
ed. McGra.w-Hill, 2001. cious anemia. N Engl] Med 1997;337:1441.
LederleFA: Ora.l cobalamin for pernicious anemia: Medicine's best Wald OS et al: Homocysteine and cardiovascular disease: Evidence
kept secret?]AMA 1991;265:94. on causality from a meta-analysis. BM] 2002;325:1.
Lindenbaum] et al: Diagnosis of cobalamin deficiency: II. Relative
MCV

Microcytic Anemias <80 fL 80-100 fL >100 fL

Joseph E. Kiss, MD
Division of Hematology/Oncology Iron Deficiency Aplastic anemia B12 or folate deficiency
and Thalassemia Pure red cell aplasia Liver disease
Chronic Inflammation
The Institute for Transfusion Medicine Lead toxicity
Chronic inflammation Myelodysplasia
Renal or endocrine disorder Increased reticulocytes
Sideroblastic anemia Infiltrative disorders

A1

Microcytic Anemias
Iron
Common problem: hemoglobin synthesis Economy
In Man
Differential diagnosis
Iron deficiency (lack of iron) Normal Total
Body Fe
Thalassemia (lack of globin chains) Content:
3 - 4 Gm
Anemia of chronic inflammation (inability to re-
re-
use iron)
Sideroblastic anemia (inability to use iron)
*
Lead toxicity (inhibition of ferrochelatase)
ferrochelatase)
* RARE

Iron Deficiency Anemia


1

All animal tissues contain some iron so dietary


insufficiency is rare
2
Iron is absorbed within proximal small intestine
Stomach acid is necessary to convert ferric iron
to ferrous form
Absorption facilitated by divalent metal
transporter (DMT 1)

3 reserve
supply

1
Slide 4

A1 Fe def is the most common deficiency state in man, affecting 10-20% of the world population
Iron balance meticulously regulated to mainatin enough for cell metabolism yet avoid toxic SE
(oxidative- reactive O2)
Iron distribution/economy in the body
3 compartments are depicted:
largest - RBCs
other cells
storage iron
Normally absorption & losses in equilibrium
Administrator, 12/31/2008
Relative Dietary Deficiency Iron Absorption

Two periods of time


when relative dietary
iron deficiency can
arise:
Adolescence with Fpn=Ferroportin
growth spurt
Pregnancy Hematology 2006;2006:505-516
Ferroportin
Copyright 2006 American Society of Hematology. Copyright restrictions may apply.

Iron Deficiency Anemia Iron Deficiency Anemia


Iron is transported by intestinal enterocytes into
the blood stream by ferroportin;
ferroportin; hepcidin is a Symptoms and Signs
newly recognized regulatory protein that regulates
ferroportin activity
Pica:
Pica: craving to eat unusual material (chewing
Iron is transported in plasma bound to transferrin ice, eating clay or starch)
Fe-
Fe-transferrin binds to cells expressing transferrin Plummer-
Plummer-Vinson syndrome:
syndrome: esophageal webs
receptors,
receptors, is internalized and can either be used for
erythropoiesis, synthesis into other iron containing
Angular cheilosis
proteins (myoglobin
(myoglobin,, catalases,
catalases, cytochromes)
cytochromes) or Koilonychia:
Koilonychia: spooning of finger nails
stored
Apoferritin is a storage protein that can store iron
molecules (ferritin
(ferritin))

PICA Plummer-Vinson Syndrome

Esophageal
Web

2
Angular Cheilosis Spoon Nails (koilonychia)

Atrophic Tongue Iron deficiency anemia

Central pallor of red


cells is normally 1/3
the diameter
In iron deficiency, less
hemoglobin is made so
central pallor
increases.

Causes of Iron Deficiency Iron Deficiency Anemia


Laboratory Studies
Need exceeds usual absorption:
absorption: infant, Serum iron - labile; diurnal variability
adolescents, pregnancy, response to Total iron binding capacity (TIBC
(TIBC;; transferrin) -
erythropoietin (Epo) changes with factors affecting protein production
Increased loss:
loss: acute or chronic blood loss, (inflammation)
menses % saturation - Fe/TIBC; normal range 20- 20-50%
Decreased intake, absorption or use:
use: inadequate Serum ferritin - storage iron; increases with
diet (rare), malabsorption, acute or chronic inflammation; <10 ug/L
ug/L specific for Fe-
Fe-deficiency
inflammation Serum soluble transferrin receptor (sTfR) shed from
cell membrane if Fe-
Fe-deficient or erythroid
proliferation increased
Assessment of bone marrow stores absent in Fe
deficiency

3
Ferritin Study
in Blood Donors 2002-
2002-03
Hb and donation history Ferritin and donation history

155.0 160.0
150.0 140.0
Fer
145.0 riti 120.0
140.0 n 100.0
Hb ng/ 80.0
135.0
ml
130.0 60.0
125.0 40.0
120.0 20.0
115.0 0.0
0 1 2 3 4 0 1 2 3 4
Donations in prior 12 months Donations in prior 12 months

'Male
Female under 50
Females over 50

Iron Deficiency Anemia Iron Deficiency Anemia


Treatment Two-
Two-step diagnosis
Recognize that the anemia is due to iron
deficiency
Oral iron - 325 mg ferrous sulfate 3X daily; GI side
Determine why
effects (nausea, constipation) common
9 most commonly in menstruating females is due
Parenteral (IV or IM) iron previously use of iron to blood lost with menses
9 most commonly in males and post-
post-menopausal
dextran was associated with risk of anaphylaxis;
females due to GI blood loss
now use iron sucrose or ferric gluconate

4
Differential Diagnosis Thalassemia
Iron deficiency (lack of iron)
Thalassemia (lack of globin chains) Definition
Anemia of chronic inflammation (inability to re-
re-
use iron) Quantitative decrease in
Sideroblastic anemia (inability to use iron) globin chain synthesis
Lead toxicity (inhibition of ferrochelatase)
ferrochelatase)

Distribution of Thalassemias

Here too

-Thalassemia Diagnosis of -Thalassemia

Mechanism gene deletion By elimination of iron deficiency and -


thalassemia in a patient of the appropriate
genetic background (often including family
studies)
Globin synthesis ratios
Molecular studies for common deletions

5
-Thalassemia

Mechanism point mutations


affecting RNA generation or splicing

-Thalassemia Blood Smear - Thalassemia

Severity of disease predicated on Nucleated RBC Targets


inheritance of one or two defective genes
and also function of those genes
-thal no normal beta chains so no Hb A
0

+-thal some function of defective beta


globin loci; therefore, some Hb A present

Clinical Manifestations of Thal

Beta Thalassemia:
Thalassemia:
Consequences of
unbalanced
production of
globin chains

6
Clinical Manifestations of Thal Management of Thalassemia
Homozygous Beta-thalassemia

3 gene deletion -thalassemia transfusions;


case reports of BMT
4 gene deletion -thalassemia incompatible
with life
0-thal - BMT is curative or long term
transfusions if no match
+-thal may be milder with occasional
transfusions
Hydrops fetalis (Hb Barts)
4 gene del a-thalassemia

Differential Diagnosis
Hemoglobin Electrophoresis
Iron deficiency (lack of iron)
Thalassemia (lack of globin chains)
Routine hemoglobin
electrophoresis is done Anemia of chronic inflammation (inability to re-
re-
on cellulose acetate in use iron)
alkali pH Sideroblastic anemia (inability to use iron)
More commonly done
by HPLC
Lead toxicity (inhibition of ferrochelatase)
ferrochelatase)

Anemia of Chronic Inflammation


Iron within macrophages cannot be re-
re-
utilized
Pro-
Pro-inflammatory cytokines stimulate
hepatic secretion of hepcidin,
hepcidin, which blocks
release of iron
Proteins that compete with transferrin for
iron are made by macrophages (lactoferrin)
Characterized by a low TIBC
Among the most common anemias

7
Hepcidin and Anemia of
Differential Diagnosis
Inflammation
Iron deficiency (lack of iron)
Interleukin-
Interleukin-6 induces
hepatic synthesis of Thalassemia (lack of globin chains)
hepcidin Anemia of chronic inflammation (inability to re-
re-
Hepcidin blocks both use iron)
absorption of iron from
Sideroblastic anemia (inability to use iron)
GI tract and release of
iron from RE stores Lead toxicity (inhibition of ferrochelatase and
ALA dehydratase )

Heme Synthesis Sideroblastic Anemias


Iron being inserted into heme ring remains
within mitochondria where the latter is
synthesized
One of myelodysplastic syndromes (MDS)

ALA-Synthase
Congenital forms of sideroblastic anemia may
Enzymatic Step
respond to B6 (pyridoxine) but acquired forms
usually don
dont

Sideroblastic Anemia
Acquired Sideroblastic Anemia
MDS
Drugs
Iron-laden
Mitochondria: 9 Alcohol-
Alcohol- inhibition of B6 action or dietary def
Ringed 9 Isoniazid-
Isoniazid-prob inhibits ALA Synthase-
Synthase-2
sideroblasts
(RBC isoenzyme)
isoenzyme)

8
Differential Diagnosis Inhibition of Heme
Synthesis By Lead
Iron deficiency (lack of iron)
Thalassemia (lack of globin chains)
Anemia of chronic inflammation (inability to re-
re-
use iron)
Sideroblastic anemia (inability to use iron)
Lead toxicity (inhibition of ferrochelatase and
ALA dehydratase)
dehydratase)

Lead Ingestion
Manifestations of Lead Toxicity
Lead inhibits two enzymes involved in heme synthesis
ALA dehydratase (early)
Ferrochelatase (late - inhibits enzyme that inserts iron
molecule into heme ring )
Increase in heme precursor (free erythrocyte
protoporphyrin)
On exam hyperpigmented line on gingiva (lead line)
On blood smear basophilic stippling of red cells
Lead line Basophilic
Stippling

9
MICROCYTIC ANEMIAS Joseph E. Kiss, MD
Hematology 2009

Microcytic Anemias
Joseph E. Kiss, MD
MICROCYTIC ANEMIAS Joseph E. Kiss, MD
Hematology 2009

Learning Objectives

1. The student should understand the differential diagnosis of microcytic


anemias.
2. The student should comprehend the absorption, transport, storage and
utilization of iron.
3. The student should be familiar with anemia associated with
inflammation.
4. The student should know the causes of iron deficiency.
5. The student should know the molecular mechanisms resulting in
and thalassemia.
Microcytic Anemias

Introduction: In this lecture, anemias that are associated with alterations in the size
of erythrocytes produced will be discussed. Microcytic anemias (i.e., anemias with mean
corpuscular volume (MCV) <80 fL) represent abnormalities of hemoglobin production.
Conversely, macrocytic anemias (MCV>100 fL) are due to diminished nuclear
maturation resulting in generation of larger erythrocytes (macrocytes).

Anemia

check reticulocyte count


correct count for degree of anemia
(corrected retic count; use HCT)

reticulocyte count 2% reticulocyte count 3%


(hypoproliferative)

blood loss
hemolysis
check MCV

MCV <80 MCV 80-100 MCV >100


(microcytic) (normocytic) (macrocytic)
MICROCYTIC ANEMIAS Joseph E. Kiss, MD
Hematology 2009

Microcytic Anemias (MCV<80 fL):

1. Iron deficiency anemia (IDA):

a) Iron absorption, transport and distribution: The average American diet contains 7
mg of iron per 1000 calories and is derived from any animal protein. Hence, only
strict vegetarians eating no animal protein have dietary insufficiency as the cause of
iron deficiency anemia. Infants, children and adolescents may become deficient due
to rapid growth spurts that outstrip their dietary intake. A normal male has 50 mg/kg
of iron stored within his body whereas a normal female has 35 mg/kg. This disparity
is secondary to a higher red blood cell mass in males (containing more iron) and
greater iron losses in females. Iron loss in males is negligible (0.9 mg/day) and
occurs as cells are shed from the skin or into the gut or urinary tract. Women lose 30-
90 ml of blood during menstruation so their average daily iron loss is 1.3 mg/day.
Iron is absorbed largely within the proximal small intestine but is facilitated by
stomach acid that converts ferric iron (Fe+3) to the ferrous (Fe+2) form. Ferric iron is
reduced to ferrous iron by an enzyme (ferrireductase) on the intestinal brush border.
Absorption and transfer of iron across the intestinal cell membrane is facilitated by a
divalent metal transporter protein (DMT-1) and iron is then transported by a
serum protein (transferrin). Males absorb 6% of their dietary iron while female
absorb 12%. Transferrin loaded with two iron molecules binds to cells expressing
transferrin receptors and may be utilized for erythropoiesis or may be stored. Iron
is stored within tissue macrophages (liver, spleen, marrow) or within hepatocytes as
ferritin.

b) Clinical manifestations of iron deficiency anemia: Signs and symptoms of IDA


include those associated with anemia (tachycardia, tachypnea, pallor) but there are
also unique clinical features of iron deficiency anemia. Cheilosis (angular fissures at
the corners of the mouth) may be observed. Fingernails may form a spoon termed
koilonychia. Patients may complain about dysphagia due to the development of
esophageal webs (Plummer-Vinson syndrome). Finally, a unique symptom seen in
patients with IDA is pica, the compulsive eating of clay, corn starch or ice
(pagophagia).

c) Causes of iron deficiency: The diagnosis and treatment of IDA is a two-part


process: the first is recognition of IDA, per se; the second part is determination of
the cause of the IDA. General causes include:
increased demand: rapid growth, pregnancy, response to erythropoietin therapy
increased loss: acute or chronic blood loss; menses; blood donation; phlebotomy
as treatment for polycythemia vera
decreased intake, absorption, or use: inadequate diet (rare); malabsorption
(disease: sprue, Crohns disease; surgery: post-gastrectomy); acute or chronic
inflammation
MICROCYTIC ANEMIAS Joseph E. Kiss, MD
Hematology 2009
It is imperative that gastrointestinal bleeding be ruled out since this is the most
common cause of IDA in males and post-menopausal females. Stool hemoccult cards
should be tested yearly and non-menstruating individuals with IDA should be considered
for a GI evaluation to eliminate peptic ulcer disease, polyps or cancer as etiologies for the
IDA.

d) Laboratory findings: The laboratory diagnosis of IDA begins by examining the CBC.
Microcytic anemia should be present and a corrected reticulocyte count is <2%. The
usual biochemical panel requested includes a serum iron and a total iron binding
capacity (TIBC). The former is the least reliable test since it has a diurnal
fluctuation. The latter is an assessment of the transport protein (transferrin). In
IDA, the serum iron is low and the TIBC is high reflecting unsaturated carrier
protein. The normal saturation of transferrin (Fe/TIBC) is ~33% with values <7%
being characteristic of IDA. A measurement of storage iron (ferritin) is also
commonly performed. In IDA, stores are low; however, ferritin is elevated during
inflammation due to release from storage sites (eg, liver) making this test somewhat
less reliable. Another test that is not commonly available is a serum soluble
transferrin receptor (sTfR) which is elevated in IDA because of shedding into
blood. Finally, the gold standard concerning the presence or absence of storage iron
is an assessment of a sample of bone marrow stained with Prussian blue.
Examination of bone marrow material to assess iron stores is not routinely done due
to the discomfort of the procedure.

e) Treatment: Iron deficiency is treated with oral iron for ~3 months and is
continued until the TIBC saturation is normal (33%). Increasingly, iron therapy is being
given parenterally. This may occur due to gastrectomy or small bowel disease that affect
the absorption of oral iron.

2. Thalassemias Represent a primary reduction in synthesis of one of the globin chains.


In each of the thalassemias an excess and unbalanced amount of globin chains exist due
to the defective production of the other globin. The excess globin chains are insoluble
and bind tightly to the red cell membrane oxidizing the membrane. This leads to
intramedullary hemolysis (destruction of red cells within the marrow) due to ineffective
production. To compensate for the ineffective erythropoiesis, erythropoietin levels rise
leading to marrow expansion and the bony abnormalities characteristic of children with
thalassemia.
-thalassemia: In -thalassemia, which may be the most prevalent genetic disorder
of mankind, the synthesis of globin is diminished. The most common genetic
abnormality involving -globin gene is deletion of one or several of the globin gene
loci causing thalassemia.
1 gene deletion (silent carrier): Asymptomatic. No anemia or microcytosis. Seen
in persons from Africa, Mediterranean region, India-Arabian peninsulas, and SE Asia.
Twenty-five per cent of African Americans have one gene deletion.
2 gene deletion (-thalassemia trait): Mild decrease in MCV and anemia. Can be
due to inheritance of a defective chromosome 16 lacking both gene loci (- -) (cis-
deletion) or inheritance of two alleles each missing one gene locus (+ -) (trans deletion).
MICROCYTIC ANEMIAS Joseph E. Kiss, MD
Hematology 2009
Former is common in subjects from SE Asia while latter more common in persons form
Africa. Two gene deletion is seen in 6% of African Americans . In menstruating females
with -thalassemia trait, it is important to rule out IDA as the cause of microcytic anemia
to avoid excessive iron administration. Knowledge about -thalassemia trait is also
important for genetic counseling.
3 gene deletion [Hb H (4)]: Hb H is produced by the deletion of 3 of the 4 genes.
Hb H is very unstable due to the deposition of unbalanced globin chains within the red
blood cells. Clinically patients have a moderate to severe anemia, jaundice, and
splenomegaly. The peripheral blood smear reveals microcytosis, hypochromia, and
targets. Hb H is relatively common in SE Asia. This is due to the frequent inheritance of
chromosomes containing cis-deletions of globin genes, i.e., - -/ . Children produced
from the union of these individuals with one-gene deletion, silent carriers of -
thalassemia (- / ) could yield a child with - -/- resulting in Hb H. While the
inheritance of -thalassemia is also prevalent among African-Americans, two gene
deletions (-thalassemia traits) are inherited in the trans position (- /- ). Thus Hb H,
and similarly hydrops fetalis (4, Hb Barts, (- -/- -), is rarely seen outside of natives of or
immigrants from SE Asia.
MICROCYTIC ANEMIAS Joseph E. Kiss, MD
Hematology 2009

Thalassemias Involving -globin Genes (adapted from Fairbanks)


________________________________________________________________________

-thal design. MCV anemia HbH Other Features


________________________________________________________________________
1 gene del silent slight none none -------
dec

2 gene del trait slight min none ~5% Hb Barts


dec

3 gene del HbH mod mild- 10-15% hemolysis


dec severe

4 gene del Hb Bart ---- --- ----- hydrops fetalis

-thalassemia: As opposed to -thalassemia in which the cause is gene deletion, the


mechanism of -thalassemia is mutation in 5 or 3 regulatory regions or in regions
involved in mRNA splicing.

0-thalassemia (-thalassemia major, Cooleys anemia) arises when an


individual inherits defective globin genes from each parent. This results in absent
production of globin chains. Cooley's anemia, represents the total absence of
production of Hb A secondary to lack of the appropriate production of globin.
Electrophoresis reveals that the dominant hemoglobin produced is Hb F with an
increased, but still minor, proportion of Hb A2 (~5%). Individuals affected with 0-
thalassemia do not become symptomatic until hemoglobin switching has occurred.
Thereafter, patients are severely anemic and develop hepatosplenomegaly and congestive
heart failure.

+-thalassemia occurs if the mutation causes a decreased but present production


of Hb A. Normal hemoglobin may account for 10-20% of the total amount of
hemoglobin with the remainder being primarily HbF. Due to the continued production of
some normal hemoglobin, the clinical phenotype (i.e., the degree of anemia or transfusion
requirement) of an individual with +-thalassemia may be minimal (thalassemia minor)
or moderately symptomatic (thalassemia intermedia).

The diagnosis of -thalassemia is made by hemoglobin electrophoresis. In 0-


thalassemia no normal Hb A is produced. Hb A2 (22) is elevated reflecting the
dysfunction of the globin regions but the dominant hemoglobin seen is Hb F. The
diagnosis of -thalassemia is made by exclusion of IDA (iron studies) and -thalassemia
(hemoglobin electrophoresis). Molecular studies looking for common deletions and
family studies are also of use in diagnosing patients with -thalassemia.
Transfusion support is the mainstay of therapy in patients with symptomatic
thalassemia. Transfusions prolong survival but lead to iron-overload that requires
chelation therapy. Bone marrow transplantation is curative in patients with 0-
MICROCYTIC ANEMIAS Joseph E. Kiss, MD
Hematology 2009
thalassemia when offered prior to the onset of cardiopulmonary or hepatic disease.
Prenatal diagnosis of 0-thalassemia followed by election termination of pregnancy has
markedly decreased the incidence of 0-thalassemia in Sardinia.

3. Anemia of chronic inflammation Microcytic anemia observed in diseases


associated with chronic inflammation (eg, cirrhosis, arthritis) is due to decreased
utilization of stored iron. Iron is present within macrophages but is unavailable for
erythropoiesis due to an inflammatory block. The latter is due to suppression of
erythropoiesis by TNF- or IL-1, pro-inflammatory cytokines released during
inflammation, and by the local secretion by marrow macrophages of proteins that
compete for iron binding (lactoferrin and hepcidin). Anemia associated with
inflammation can be differentiated from IDA by decreased iron binding capacity
reflecting diminished production of transferrin.

Test Iron deficiency Inflammation

serum iron <30 <50


TIBC >360 <300
% saturation <10 10-20
ferritin (g/L) <15 30-200

4. Sideroblastic Anemia Sideroblastic anemia is one of the subsets of the


myelodysplastic syndromes (refractory anemia with ringed sideroblasts, RARS). In this
disorder iron-laden mitochondria encircle the red cell nucleus in the developing
erythroblasts to form ringed sideroblasts. A congenital form of sideroblastic anemias
may respond to pyridoxine (B6) but sideroblastic anemias seen in adults reflects
dyserythropoiesis (abnormal red cell production).

5. Lead toxicity Microcytosis secondary to lead ingestion is due to inhibition of


ferrochelatase, the enzyme responsible for insertion of iron into the heme ring. This
results in hypochromia and microcytosis similar to iron deficiency. It is typified by
basophilic stippling of red cells; however, stippling can also be seen in thalassemia and
myelodysplasia. The diagnosis of lead ingestion depends on finding an elevated blood
lead level.
Learning Objectives
Understand the normal pathways for red cell
Hemolytic Anemias I destruction

2009 Know the various methods to classify


hemolytic anemias
Roy E. Smith, MD, MS Understand the causes of hemolysis that are
extrinsic (acquired) to normal red cells
Division of Hematology/Oncology
smithre@upmc.edu

Red Cell Survival Recycling of Materials


Red cells typically Tissue macrophages (RE
cells)(CD36 receptor) within the
survive for 100-
100-120 days liver, spleen and marrow ingest
During this time they the RBC
Iron is stored ( 2-
2-4 gm) or re-
re-
continually circulate circulated via transferrin (3mg)
through sinusoidal Amino acids of the globin
endothelium within chains are recovered
the spleen and liver Heme ring is metabolized to
bilirubin with release of CO
With senescence the Bilirubin is transported to the
red cells are eliminated liver where it is conjugated to
glucuronide and excreted into
from circulation bile.

Frequency of Various
When To Think About Hemolysis
Causes of Anemia
Unexplained jaundice
especially unconjuated bilirubinemia

Unexplained drop in hemoglobin


remember senescent RBC destruction is accelerated
during infection

Elevated reticulocyte count


without evidence of blood loss

1
Tests for Hemolysis Tests for Hemolysis
LDH:
LDH: increased Haptoglobin:
Haptoglobin: decreased
Ubiquitous intracellular enzyme
2 globulin produced by liver
often measured in 103

no need to subtype isoforms binds free hemoglobin


Total and indirect bilirubin:
bilirubin: increased cleared in liver with excess filtered in
primarily unconjugated (indirect) form
kidney
Normal I/D ratio maintained
level of unconjugated bilirubin is rarely >4-
>4-5 mg/dL levels high with inflammation and low with
end-
end-stage liver disease

Tests for Hemolysis Reticulocytes


Decreased Haptoglobin and Increased Retic ct - measure
LDH: of RBC production
Retics mature in 4
95% Sensitive (5% False negatives) and days
95% Specific (5% false positive). Normally mature in
circulation within
24h of release
Marrow Pool of
Less mature retics

Reticulocyte Life Span


Shift Cells
and Time in Circulation
Reticulocytes are like
bands
Are retained within
the marrow for 2-
2-3
days maturing
With bleeding or
hemolysis, retics
leave marrow early
(shifted out)

2
Tests for Hemolysis Retic Ct. (other methods)
Count:
Reticulocyte Count:
increased Uncorrected-
Uncorrected- % Retics ID IDed by Special
at UPMC FC gives an stain of PBS. (eg
(eg.. Hct 35, Retic 10%)
absolute number
Corrected (aka. Reticulocyte Index)-
Index)- %
normal is ~0.05 (1% or 5 Retics x( Hct pt/Hct nl)nl) = 10% x 35/45 =
x 106 RBC)
7.8%
If RBC 3x106/mm3 and
Reticulocyte Production Index (RPI)-
(RPI)-Based
retic ct is 10%, abs retic
ct would be 0.3 (6 X inc)
Upon Shift Cells. eg.
eg. RI/circ. RLS) =
6.6%/1.5 = 5.2

Various Ways To Differentiate Intravascular Hemolysis


Destruction is in vessels
Free Hb is bound to
haptoglobin and removed
Site of destruction:
destruction: by the liver or spills over
Intravascular into the kidneys
Marked by:
Extravascular
Hb
Hbemia
Hereditary vs Acquired HB
HBuria
Hemosidinuria
Methemalbumin

Intravascular Hemolysis Intravascular Hemolysis

Hemoglobinuria

Urinary hemosiderin

3
Intravascular Hemolysis Extravascular Hemolysis
Transfusion reactions Extravascular
Hemolysis occurs
G6PD deficiency
within the RE Syst.
Paroxysmal nocturnal hemoglobinuria Less dramatic:
Sepsis [C. perfringens; malaria (blackwater
(blackwater Hb Drop
fever); bartonellosis] Jaundice
Mechanical heart valves Splenomegaly
Snake bites

Another Way to Determine


Extra-vascular Hemolysis
Cause of Hemolysis
Bacterial (mycoplasma
(mycoplasma)) or viral (EBV)
infections
Hemolysis
Drug-
Drug-induced
(High Retic Anemia)
Autoimmune hemolysis
Hemoglobinopathies Acquired or Extrinsic to RBC Hereditary or Intrinsic to RBC

Membrane defects
Environmental disorders (DIC, TTP, HUS)

Extrinsic Immunologically Mediated


(Auto-
(Auto-immune Hemolytic Anemia
AIHA)
Warm-
Warm-antibody mediated [IgG]
Extrinsic Causes
Cold-
Cold-antibody mediated [IgM (via
Immune Mediated Non-Immune Mediated
complement)]
IgG (Warm) IgM (Cold)

4
Test for AIHA Test for AIHA
Coombs Test
9 Rabbit anti-
anti-human Direct Coombs:
Coombs: Detects
IgG or anti-
anti- presence of IgG or
complement added to complement on
patient
patients RBCs patient
patients RBC
9 Rabbit reagent cross-
cross- Indirect Coombs:
links human IgG or screens for presence of
complement Ab in serum using
9 Agglutination of cells donor RBCs
from cross-
cross-linking

Scheme for Serological Investigation Scheme for Serological Investigation


of AIHA of AIHA
Polyspecific DAT (+) Polyspecific DAT (-
(-)
Monospecific DAT (anti-
(anti-IgG vs anti-
anti-3Cd) Due to IgA or IgM;
IgM; low affinity Abs; or few IgGs
Check for Ab specificity on rbc surface
Determine Bld Grp,
Grp, Rh D and Kell type. More sensitive techniques need to be used to

Check for free serum Ab (aggl vs lysis at 37C; ID


detect Ab.
Elution
antibody with rbc panel)
Polybrene
If transfused in last 30 days, check rbc eluates
Enzyme pretreatment
for alloAb
Absorbtion

IgG-mediated Hemolysis
Warm Antibody (IgG)-Mediated
Morphology:
Morphology: spherocytes
Mechanism:
Mechanism: Ab coats Idiopathic
RBC
Ab-
Post-
Post-infectious (viral)
Ab-coated RBC traverses
spleen Lymphproliferative
M FcRs Recognize Ab-
Ab- Lymphoma
coated RBCs
CLL
Portions of RBC
membrane are removed Auto-
Auto-immune Diseases
RBC surface area
Drugs: PCNs,
PCNs, quinidine

5
Treatment of Warm
Drug-induced AIHA
Ab-
Ab-Mediated Hemolysis
Drug may bind to
RBC inducing an Remove Site of Destruction splenectomy
Ab (PCN)
Block mechanism of destruction-
destruction- steroids;
Drug may bind to
serum protein IVIG
(hapten) and the Suppress Production of antibody steroids,
complex attaches to rituxan, chemotherapy
RBC (quinidine)
Treat Underlying Cause stop drug; treat
Drug may induce
neo-
neo-epitope on primary disorder
RBC (methyldopa)

Cold Antibody Mediated


IgM-mediated Hemolysis
Hemolysis
Morphology: red cell
Idiopathic
agglutination
Mechanism: IgMs are Post-
Post-infectious
pentamers,
pentamers, fix C
C EBV (anti-
(anti-I antigen on RBC)
Each Fab (variable Mycoplasma (anti-
(anti-i antigen on RBC)
Lymphoproliferative (NHL, CLL)
region) of the antibody
Auto-
Auto-immune diseases
can bind an RBC

Treatment of Cold
Extrinsic Non-Immunologic
Ab-
Ab-Mediated Hemolysis
Morphology:
Morphology: schistocytes
Warmth patient, room, IV fluids, blood
Mechanism:
Mechanism:
Plasmapheresis IgM mediated is totally Shearing of RBC
intravascular Fibrin consumption within
small blood vessels
Suppression antibody production Valves
chemotherapy, rituxan Bones in feet
Steroids and splenectomy play less of a
role

6
Causes of Non-
Non-immunologically
Mediated Hemolysis DIC
Microangiopathic Hemolytic Anemias
Hallmark is intravascular coagulation
(MAHA) and end-
end-organ dysfunction
Disseminated intravascular coagulation Cardinal feature is consumption of
(DIC) fibrinogen and platelets
Thrombotic thrombocytopenic purpura Causes:
(TTP) or hemolytic uremic syndrome Sepsis
(HUS) Obstetrical problems
Vasculitis (eclampsia, autoimmune) Malignancies
March hemoglobinuria/Marathoners

Treatment of DIC TTP/HUS


Treat the underlying cause TTP:
TTP: clinical diagnosis involving a
pentad of signs/symptoms
Support what needs to be supported:
MAHA
Platelets
Thrombocytopenia
Fibrinogen:
Fibrinogen: Cryoprecipitate is Something renal
enriched in VIII, vWF and Something CNS
fibrinogen Fever
9 1 unit cryo raises fibrinogen 5 mg HUS:
HUS: first three only

Mechanism of TTP/HUS
Treatment of TTP/HUS
vWF is stored within granules in platelets and
endothelial cells Plasmapheresis with plasma infusion
Activation of these cells releases vWF which Steroids
promotes platelet adhesion to matrix proteins
and platelet aggregation Follow LDH and platelet count re. when
vWF is cleaved by serum protease (ADAMST
to withdraw pheresis
13) 50% TTP may have chronic course
Absence of protease or antibody-
antibody-inhibition of HUS more often has an identifiable
protease may causes chronic, relapsing course cause (drugs, infections) than TTP

7
Review Review (continued)

RBC Survival /Destruction Types of Hemolysis


Frequency of Hemolysis Warm vs Cold vs Drug-
Drug-Induced
Think of Hemolysis (Bili,
Bili, Hb.,
Hb.,Retics)
Retics) Coombs Test (Direct vs Indirect)(+ vs -)

Hemolysis Tests ((LDH, Hapto,


Hapto, Hb-
Hb- MAHAs (DIC vs TTP vs HUS vs
emia,
emia, -uria,
uria, methemalbumin ) Vasculitis vs Other)
Reticulocyte Ct. (RI, RPI, Absolute)

8
Learning Objectives
Know the general schema for the differential
Hemolytic Anemias II
diagnosis of hemolytic anemias (HA)
Know the disorders that affect the cell membrane
2009 of RBC including the cell morphology, lab tests to
make the diagnosis and the clinical course of the
disorder
Roy E Smith, MD, MS Know the mechanism involved in hemolysis due to
Division of Hematology/Oncology G6PD deficiency
Know the common hemoglobinoptahies associated
smithre@upmc.edu with hemolysis

Hemolytic Anemias
Intrinsic/Hereditary Causes
Problem is Extrinsic Problem is Intrinsic
to RBC to RBC Surface* Metabolism Hemoglobin

* PNH is an acquired cause

Red Cell Membrane Surface Abnormalities


Integral membrane
proteins
band 3; glycophorins
Cytoskeletal proteins
actin; spectrin
Linker proteins
ankyrin; protein 4.1

1
Hereditary Spherocytosis Hereditary Elliptocytosis
Morphology:
Morphology: Ovalocytes
Morphology:
Morphology: Spherocytes Mech: or spectrin
Mech:
dimer formation abn.,
Mechanism:.
Mechanism:. Ankyrin, defective spectrin-
spectrin-
spectrin, Band 3 or ankyrin associations or
protein 4.2 (proteins protein 4.1 abns.
involved with vertical [proteins involved with
interactions) abn. horizontal interactions]
AD Inheritance

Test for HS or HE HS or HE
Osmotic Fragility Clinical Manifestations:
RBCs are placed in Osmotic Fragility Test Chronic compensated hemolytic anemia
progressively more
hypotonic solutions Site of RBC destruction: Spleen
Normal RBCs:
RBCs: lysis at Early gall stones (pigment stones)
a defined range of May develop red cell aplasia due to folate
hypotonicity deficiency or parvovirus infection
HS or HE RBCs:
RBCs: lysis
Treatment:
Treatment: Splenectomy
earlier

Paroxysmal Nocturnal
PNH
Hemoglobinuria (PNH)*
Morphology:
Morphology: none unique CD55 and CD59 prevent C-mediated lysis of cells
Mechanism:
Mechanism: clonal Clinical features of PNH: episodes or
disorder where some cells intravascular hemolysis (hemoglobinuria),
are unable to produce thrombosis, development
predisposition to venous thrombosis,
phosphatidylinositol of aplastic anemia and leukemia
glycan (PIG)
PIG) linkages Diagnosis:
Diagnosis: flow cytometry for CD55 and CD59
Absence of CD55 and Treatment:
Treatment: historically steroids; new inhibitor of
CD59 C5 (eculizumab)
eculizumab) decreases rate of hemolysis and
thrombosis

2
G6PD Deficiency
X-linked enzyme
deficiency
Intrinsic/Hereditary Causes Absence of enzyme
needed to regenerate
Surface* Metabolism Hemoglobin redox intermediary to
recover glutathione

G6PD Deficiency G6PD Deficiency


Morphology:
Morphology: Bite cells
Mechanism:
Mechanism: oxidizing
agent causes
precipitation of Hb
which attaches to inner
surface of membrane;
as cell passes through
the spleen, the
precipitated Hb is
removed by the RE
cells

G6PD Deficiency G6PD Deficiency


Clinical severity:
severity: mediterranean>
mediterranean> oriental >
Clinical manifestations:
manifestations: episodes of african
hemolysis induced by infection, Diagnosis:
Diagnosis: measurement of enzyme level;
medications, fava beans need to assess enzyme activity after acute
event since reticulocytes have higher
Worldwide 400,000,000 affected
enzyme level
400 variants
Treatment:
Treatment: avoidance of precipitating
causes

3
Pyruvate Kinase Deficiency Table 6-2 Causes of acute hemoglobin decline in patients with sickle cell disease

Second most common


enzyme deficiency
Causes chronic, non-
non-
spherocytic hemolytic
anemia
AR Inheritance
Not infrequent in Amish
populations
Clinical:
Clinical: splenomegaly,
splenomegaly,
pigment gallstones, Buchanan, G. R. et al. ASH-SAP 2007;2007:102-142

jaundice
Diagnosis:
Diagnosis: assess enzyme
activity

Copyright 2007 American Society of Hematology. Copyright restrictions may apply.

Figure 6-1 Organization of the gene clusters that encode human hemoglobin and the
composition of specific hemoglobin tetramers during development

Intrinsic/Hereditary Causes

Surface* Metabolism Hemoglobin

Buchanan, G. R. et al. ASH-SAP 2007;2007:102-142

Copyright 2007 American Society of Hematology. Copyright restrictions may apply.

Figure 6-3 The genetics of {alpha} thalassemia

Hemoglobinopathies

Hb S

Hb C

Hb E (thalassemic
(thalassemic hemoglobinopathy)
hemoglobinopathy)

Unstable hemoglobins
Buchanan, G. R. et al. ASH-SAP 2007;2007:102-142

Copyright 2007 American Society of Hematology. Copyright restrictions may apply.

4
Frequency of Abnormal Hemoglobins
in African-
African-Americans

Frequency of Abn Hbs in African-


African- Freq of Abn Hbs in African
Americans Americans
Heterozygous Prevalence
Alpha Thal 24,000 Heterozygous Conditions

SA 8,600 30,000
AC 2,400 25,000
20,000
Beta Thal minor 1400
15,000
HPFH trait 100 10,000
Others 300 5,000
0
Alpha SA CA Beta HPFH Others
Thal Thal trait

Freq of Abn Hbs in African Freq of Abn Hbs in African


Americans Americans
Doubly Heterozygous Conditions Homozygous Conditions

140 6,000
120 5,000
100 4,000
80
3,000
60
40 2,000
20 1,000
0 0
SC S-beta C-beta S- C- Others Alpha Thal SS CC Beta Thal
Thal Thal HPFH HPFH m maj

5
Frequency of Abnormalities in
Frequency of Hb E
SE Asians

Causes of Hb
Hbpathies (#variants)
Causes of Hemoglobinopathies Abn Total
Single AA 439 137 242 17 43
subs
Two AA 7 0 7 0 0
subs
Del 14 1 13 0 0

Elongation 12 6 6 0 0

Fusions 9 0 9 8 1

Totals 481 144 277 25 44

Functional Alterations

6
Distribution of Hb Disorders

Hemoglobin S Hemoglobin S

Sickle Cell Disease


Homozygous S: S: Sickle cell anemia; 70-
70-
75% pts
Double heterozygous situations:
Hb SC: 10-10-15% pts
S/
S/ +/-
+/- thalassemia: 10% pts
Rarely others

7
Pain
60
Rates in Sickle Syndromes
50

40 r=0
0<r<1
30
1</r< 3
20 3=<r<6
6=<r
10

0
SS SC SB+ SB 0

Onset of Chronic Organ Failure


Onset of Chronic Organ Failure SS N=785 SC N=231

Event % Med. Age % Med.Age


CVA 8.8 13 3 47
CRF 4.1 25 2.2 49.8
CLD 4.2 24.2 0.4 32.4
Leg Ulcer 10.2 24.5 1.7 24.3
Osteoneco 9.4 23.1 9.1 31.7
sis
Retinopah 5.6 25.8 19.9 27.7
y
Priapism 6.7 22.3 1.9 23.3

Frequency and Onset of Chronic


Organ failure by Category
60 Survival SS v SC
50

40
SS %
SC %
30
SS Age
Sc Age
20

10

0 CVA CRF CLD Leg Ulcer Osteonecrois Retinopathy Priapism

8
Types of Crises Natural History of SCD
Aplastic Crisis (reticulocytopenia)
reticulocytopenia) Frequency of vaso-
vaso-occlusive crises
Folate deficiency Onset of end-
end-organ failure
Parvovirus Survival
Sequestration Crisis Prognostic factors for survival
Spleen (children) Hb F

Liver (adults) Leukocyte count

Vaso-
Vaso - occlusive (painful) crisis accounts Frequency of painful attacks
for 85% of all admissions

Pathophysiological Characteristics of Sickle Cell Anemia


and the Effect of Hydroxyurea

9
Distribution of Hb Disorders
Sickle Trait (SA)
Normal life expectancy
Rare sickling occurrences and splenic
infarction
Asymptomatic hematuria and
hyposthenuria
Increased risk of sudden death
Increased risk of medullary carcinoma of
the kidney

Hemoglobin C Hemoglobin C
(6)
Point mutation at same site as S ( Targets
In this situation glu lys
Mutation arose in west Africa - therefore
predominantly in African-
African-Americans
2.4% are CA (trait)
crystal
Homozygous:
Homozygous: mild hemolytic anemia
with targets and crystals
Traits:
Traits: targets without anemia

Hemoglobin Electrophoresis Unstable Hemoglobins

Discovered by chronic compensated


hemolysis, intermittent jaundice, early
cholelithiasis
Hemolysis may occur following exposure
to drugs (sulfonamides, oxidants) or
during febrile illness

10
Substitutions Causing Unstable Hemoglobins
Unstable Hbs Heinz Bodies

Special stain to define


precipitated
hemoglobin

Review
Laboratory Testing Classification
of (HA) (Ext vs Int)
Int)
Surface Abnormalities: HS (vertical
Isopropanol precipitation:
proteins); HE (horizontal proteins);
precipitation: denatures PNH( absent CD55, 59)
unstable Hbs Osmotic Fragility Test (HS/HE lyse
early)
Heat denaturation:
denaturation: similar mechanism G6PD and PK Deficiency
Abnormal Hbs (S, C, E,Thallassemias)
E,Thallassemias)

Review (continued)
Discussed ISCs and Hb F
Sickle Crisis, End Organ Damage and
Survival
Hydroxyurea and BMT
Hb electrophoresis (Hb
(Hb A, A2, C, S,
and F levels)
Unstable Hbs:
Hbs: Isopropanol and Heat
stability tests, Heinz body stain

11
American Journal of Hematology 69:258271 (2002)

Autoimmune Hemolytic Anemia


Bradley C. Gehrs1 and Richard C. Friedberg2*
1
Department of Pathology, University of Alabama at Birmingham, Birmingham, Alabama
2
Department of Pathology, Baystate Medical Center, Springeld, Massachusetts

Red blood cell (RBC) autoantibodies are a relatively uncommon cause of anemia.
However, autoimmune hemolytic anemia (AIHA) must be considered in the differential
diagnosis of hemolytic anemias, especially if the patient has a concomitant lympho-
proliferative disorder, autoimmune disease, or viral or mycoplasmal infection. Classi-
cations of AIHA include warm AIHA, cold agglutinin syndrome, paroxysmal cold
hemoglobinuria, mixed-type AIHA, and drug-induced AIHA. Characteristics of the au-
toantibodies are responsible for the various clinical entities. As a result, diagnosis is
based on the clinical presentation and a serologic work-up. For each classication of
AIHA, this review discusses the demographics, etiology, clinical presentation, laboratory
evaluation, and treatment options. Am. J. Hematol. 69:258271, 2002. 2002 Wiley-Liss, Inc.
Key words: autoimmune hemolytic anemia; red blood cell autoantibodies

INTRODUCTION IHA. Drug-induced antibodies can recognize either


intrinsic RBC antigens or RBC-bound drug. Anti-
Immune hemolytic anemia (IHA) is the clinical bodies that react with intrinsic RBC antigens are se-
condition in which IgG and/or IgM antibodies bind rologically indistinguishable from autoantibodies. In
to RBC surface antigens and initiate RBC destruction contrast, antibodies that react against RBC-bound
via the complement system and the reticuloendothe- drug require the drug for hemolysis.
lial system. IHA is classied as either autoimmune, The pathogenesis of IHA ultimately overlaps for
alloimmune, or drug-induced based on the antigenic these three classications. The degree of hemolysis
stimulus responsible for the immune response. depends on characteristics of the bound antibody
Autoimmune hemolytic anemia (AIHA) is charac- (e.g., quantity, specicity, thermal amplitude, ability
terized by the production of antibodies directed to x complement, ability to bind tissue macrophag-
against self RBCs. Since the autoantibodies usually es) as well as the target antigen (density, expression,
are directed against high-incidence antigens, they of- patient age). IgG antibodies are relatively poor acti-
ten exhibit reactivity against allogeneic RBCs as well. vators of the classical complement pathway, but they
AIHA is a fairly uncommon disorder, with estimates (in particular IgG1 and IgG3 antibodies) are recog-
of the incidence at 1 3 cases per 100,000 per year nized readily by Fc receptors on various phagocytic
[1,2]. In contrast, alloimmune hemolytic anemia re- cells [6 8]. Therefore, IgG-sensitized RBCs generally
quires exposure to allogeneic RBCs, and the resulting are eliminated by phagocytes of the reticuloendothe-
alloantibodies show no reactivity toward autologous lial system. Since reticuloendothelial cells also have
RBCs. Sources of allogeneic RBC exposure include
pregnancy, blood product transfusion, and trans-
plantation. The principal manifestations of RBC al- *Correspondence to: Richard C. Friedberg, MD, PhD, Chairman,
loimmunization are hemolytic transfusion reactions Department of Pathology, Baystate Medical Center, 759 Chestnut
and hemolytic disease of the newborn (HDN). The Street, Springeld, Massachusetts 01199.
incidence of acute hemolytic transfusion reactions
Received for publication 15 October 2001; Accepted 15 November
has been estimated to be 0.003 0.008%, while
2001.
0.05 0.07% of transfused patients develop a clinically
recognized delayed hemolytic transfusion reaction Published online in Wiley InterScience (www.interscience.wiley.
[3 5]. Drug-induced IHA is the nal classication of com). DOI: 10.1002/ajh.10062

2002 Wiley-Liss, Inc.


Concise Review: Autoimmune Hemolytic Anemia 259

receptors for complement factors C3b and iC3b, these Table I. Classication of Autoimmune Hemolytic Anemia
complement components, if present, can potentiate Warm autoimmune hemolytic anemia
the extravascular hemolysis [9]. On the other hand, Idiopathic
IgM-sensitized RBCs generally are associated with a Secondary (lymphoproliferative disorders, autoimmune disorders)
combination of intravascular and extravascular Cold autoimmune hemolytic anemia
Cold agglutinin syndrome
hemolysis. Intravascular hemolysis occurs because Idiopathic
IgM antibodies, unlike IgG antibodies, readily acti- Secondary
vate the classical complement pathway and produce Acute transient (infections)
cytolysis. However, due to the presence of regulatory Chronic (lymphoproliferative disorders)
RBC proteins such as decay accelerating factor Paroxysmal cold hemoglobinuria
Idiopathic
(DAF, CD55) and membrane inhibitor of reactive Secondary
lysis (MIRL, CD59), overwhelming complement Acute transient (infections other than syphilis)
activation usually is required to produce clinically Chronic (syphilis)
evident intravascular hemolysis, e.g., as seen with Mixed-type autoimmune hemolytic anemia
ABO-incompatible blood transfusions. More com- Idiopathic
Secondary (lymphoproliferative disorders, autoimmune
monly, IgM-sensitized RBCs undergo extravascular disorders)
hemolysis. While reticuloendothelial cells do not have Drug-induced immune hemolytic anemia
receptors for the Fc fragment of IgM antibodies with Autoimmune type
comparable activity to the receptors directed against Drug adsorption type
the Fc fragment of IgG, they do have receptors for the Neoantigen type
abundant RBC-bound C3b and iC3b resulting from
complement activation. Whereas the spleen is the
principal site of IgG-associated extravascular hemol- tions, immunodeciency disorders, and tumors
ysis, Kuper cells in the liver are the principal eec- [1,10 13]. Lymphoproliferative disorders account for
tors of IgM-associated extravascular hemolysis. approximately half of the cases of both secondary
The remainder of this article concentrates on warm and cold AIHA. Autoimmune disorders are the
AIHA. Initial focus is placed on the basic classica- next leading cause of secondary warm AIHA,
tion scheme as well as the shared laboratory ndings whereas infections are the next leading cause of sec-
and theoretical etiologies. Later, more detailed ondary cold AIHA. Idiopathic disease is more com-
background, laboratory ndings, and treatment op- mon in women and has a peak incidence in the fourth
tions are presented for each distinct clinical subtype and fth decades of life. The demographics of sec-
of AIHA. These subclassications include warm ondary disease correspond with those of the various
AIHA, cold agglutinin syndrome, paroxysmal cold underlying illnesses.
hemoglobinuria, mixed-type AIHA, and drug-in-
duced AIHA.
Laboratory EvaluationSerology
Two criteria must be met to diagnose AIHA: se-
AUTOIMMUNE HEMOLYTIC ANEMIA rologic evidence of an autoantibody and clinical or
Classications laboratory evidence of hemolysis. Serologic evidence
Cases of AIHA generally are classied according to of an autoantibody is provided by positive autocon-
the characteristic temperature reactivity of the RBC trol and direct antiglobulin test (DAT, direct Coo-
autoantibody (Table I). Warm autoantibodies react mbs test) results and subsequent identication of an
most strongly near 37C and exhibit decreased anity autoantibody in the RBC eluate and possibly the se-
at lower temperatures. Cold autoantibodies, on the rum. Serum reactivity with autologous RBCs gener-
other hand, bind to RBCs most strongly near 0 4C ally indicates the presence of an autoantibody, but it
and typically show little anity at physiologic tem- does not exclude the presence of an alloantibody. In
perature. Occasionally, patients have a combination addition, if the patient has been transfused recently,
of warm and cold autoantibodies. Cases of AIHA can alloantibodies bound to the transfused cells can pro-
be subdivided further on the basis of etiology. By duce a mixed-eld positive autocontrol. While the
denition, idiopathic or primary AIHA shows no autocontrol measures in vitro binding of antibodies to
apparent association with an underlying disorder. RBCs, the DAT indicates whether there is in vivo
Depending on the patient population studied, a sus- RBC binding of either IgG or C3d. The DAT in-
pected secondary cause of AIHA has been determined volves mixing the individuals anticoagulated RBCs
in 20 80% of reported series; these include lympho- with polyspecic anti-human globulin (AHG) with
proliferative disorders, autoimmune disorders, infec- anti-IgG and anti-C3d activities; if positive, the
260 Concise Review: Gehrs and Friedberg

sample can be tested separately (``split'') with reagents conrms that it is an alloantibody; in the absence of a
specic for anti-IgG or anti-C3d. A positive DAT is recent transfusion, a positive result suggests that it is
non-specic for an RBC autoantibody. Additional an autoantibody. Autoadsorption can also be used to
primary causes of a positive DAT include an acute or cross-match donor RBC units for patients with warm
delayed hemolytic transfusion reaction, hemolytic autoantibodies (only for patients who have not been
disease of the newborn, transplantation, drug-in- transfused recently).
duced antibodies, and administration of various In addition to the customary serum antibody
therapeutics including intravenous immunoglobulin identication, an RBC eluate should be tested for
(IVIG), Rh immune globulin (RhIg), antilymphocyte antibodies if the positive DAT shows anti-IgG reac-
globulin, and antithymocyte globulin. A positive tivity. IgM autoantibodies generally detach from the
DAT also may arise secondary to sickle cell disease, RBC surface in vivo, so they are not detected directly
b-thalassemia, renal disease, multiple myeloma, in the DAT and are not present in the eluate. Elution
autoimmune diseases, AIDS, and other diseases with may also concentrate any existing bound antibodies
elevated serum globulin [14,15]. and thus improve sensitivity. Autoantibodies are
Given the broad dierential diagnosis for a positive present in both the eluate and the serum in approxi-
DAT, supplemental serologic testing is necessary to mately 80% of the cases of AIHA [16]. Together with
ascertain the cause. The antibody screen detects both the DAT, results from serum and RBC eluate anti-
unbound autoantibodies and alloantibodies. Anti- body studies help distinguish the forms of AIHA
body bound to RBCs can be eluted o the cell surface (Table II).
and subsequently identied in the eluate. Both the
RBC eluate and the serum sample can be examined for
soluble autoantibodies and/or alloantibodies using Laboratory EvaluationHemolysis
diagnostic RBC panels and indirect antiglobulin test- The diagnosis of AIHA also requires manifestations
ing. Drug-dependent antibodies, on the other hand, of hemolytic anemia. From 0.007% to 0.1% of healthy
may not be detected unless the drug is added for in blood donors and from 0.3% to 8% of hospital pa-
vitro testing. Generally autoantibodies react with all tients have a positive DAT without clinical evidence of
panel RBCs (i.e., panreactive), whereas alloantibodies IHA [17 20]. Hemolysis in AIHA can be either ex-
exhibit antigen specicity, only reacting with specic travascular or intravascular. Typically, intravascular
antigen positive cells. Other techniques may be nec- hemolysis has a rapid and aggressive presentation,
essary to distinguish between the two types. For ex- whereas extravascular hemolysis is milder. A CBC
ample, an alloantibody to a high-incidence antigen in with peripheral smear, bilirubin, LDH (in particular
a post-transfusion setting can mimic an autoantibody isoenzyme 1), haptoglobin, and urine hemoglobin are
with a positive DAT (mixed-eld) and reactions with the basic tests used to evaluate and dierentiate in-
all panel cells. In addition, autoantibodies may exhibit travascular and extravascular hemolysis.
apparent specicity. Autoadsorption and antigenic
phenotyping can help dierentiate autoantibodies and
alloantibodies, especially if the patient has not been Autoantibody Etiology
transfused recently. Autoadsorption uses autologous The etiology of most RBC autoantibodies is not
RBCs to adsorb autoantibodies prior to repeating well understood. Given the association between
serum testing for alloantibodies. If an antibody ex- AIHA and other autoimmune disorders, generalized
hibits specicity, then demonstration that autologous immune system dysfunction likely plays a role. The
RBCs are negative for the corresponding antigen relationship between AIHA and lymphoproliferative

Table II. Serologic Characteristics of Autoimmune Hemolytic Anemias*

Ig type DAT RBC eluate Specicity

Warm AIHA IgG (IgA or IgM) IgG and/or C3 IgG Panreactive;


(Rarely Rh  others)
CAS IgM C3 Nonreactive I > i >> Pr
PCH IgG C3 Nonreactive P
Mixed-type AIHA IgG, IgM IgG + C3 IgG Panreactive +
unclear > I > others
Drug-induced AIHA IgG IgG C3 IgG Often Rh-related

*Ig, immunoglobulin; DAT, direct antiglobulin test; RBC, red blood cell; AIHA, autoimmune hemolytic anemia; CAS, cold agglutinin syn-
drome; PCH, paroxysmal cold hemoglobinuria.
Concise Review: Autoimmune Hemolytic Anemia 261

disorders and other neoplasms likewise suggests gen- hepatomegaly, and lymphadenopathy accompany the
eralized dysfunction of immune surveillance. The anemia. This more acute, potentially life-threatening
immune system has many control points that keep a presentation is usually associated with viral infec-
balance between the need to tolerate self-antigens and tions, especially in children. In cases of secondary
the need to respond appropriately to foreign antigens. disease, the symptoms of AIHA may precede the
Immune self-tolerance occurs centrally with develop- recognition of the underlying illness by months to
ing lymphocyte precursors via clonal deletion or years, but ultimately the symptoms of the underly-
clonal anergy, and peripherally with mature T and B ing disorder dominate. When presentation includes
cells via down-regulation of the immune response. massive splenomegaly or lymphadenopathy, an un-
Disruption of any of these processes may be a po- derlying lymphoproliferative disorder should be con-
tential cause of autoimmune disease. Less generalized sidered. Symptoms can be precipitated by trauma,
autoimmune processes also exist. For example, surgery, infection, pregnancy, and psychological
autoimmune disease can arise from the response to a stress. Pregnancy also carries a 5-fold risk of devel-
foreign antigen if the foreign antigen shows sucient oping autoantibodies compared to an age-matched
homology with a self-antigen. Another theoretical control population [32]. Some infants born in the
source of autoantibodies is a malignant B cell clone, setting of maternal AIHA are unaected, but others
but RBC autoantibodies generally are polyclonal. On suer transient hemolysis due to passively acquired
the basis of studies with other autoimmune disorders, autoantibody [33 35].
such as ankylosing spondylitis and multiple sclerosis,
both genetic and environmental factors likely play a
role in the production of RBC autoantibodies. Laboratory Evaluation
The laboratory evaluation shows a hemoglobin and
hematocrit that varies widely, from normal in the
WARM AUTOIMMUNE HEMOLYTIC ANEMIA compensating patient with indolent hemolysis to se-
Background verely decreased in the patient with fulminant hem-
Warm autoantibodies are responsible for 48 70% olysis. The MCV usually is elevated reecting
of AIHA cases [13,21]. Lymphoproliferative disorders reticulocytosis, but reticulocytopenia can exist early
such as CLL, Hodgkins disease, non-Hodgkins lym- in the disorder or secondary to autoimmune hemol-
phoma, and Waldenstroms macroglobinemia are the ysis of the reticulocytes or an inadequate bone mar-
leading causes of secondary cases. Rare cases have row response [36]. The WBC count typically exhibits
been associated with multiple myeloma [22]. Other a mild leukocytosis representing predominantly neu-
secondary causes include autoimmune disorders (e.g., trophils, but neutropenia may also be seen. The
SLE, rheumatoid arthritis, scleroderma, and ulcerative platelet count is normal to low. Sometimes there is
colitis), non-lymphoid neoplasms (e.g., ovarian concurrent leukopenia and/or thrombocytopenia due
dermoid cysts, teratomas, Kaposis sarcoma, and to an immune-mediated process, such as Evans syn-
carcinomas), immunodeciency disorders (e.g., AIDS, drome [37]. The peripheral blood smear generally
dysglobulinemia, and hypogammaglobulinemia), and displays polychromasia and macrocytosis from the
childhood viral illnesses. Due to these secondary reticulocytosis as well as nucleated RBCs. Erythro-
causes, the incidence of warm AIHA increases starting phagocytosis and microspherocytes also are indicative
around 40 years of age. Children have a peak incidence of autoimmune hemolysis. The bone marrow typically
in the rst 4 years of life. There is an approximate 2:1 reveals erythroid hyperplasia, and increased lympho-
female predilection, at least partially due to the asso- cytes or plasma cells suggest the presence of an un-
ciation with autoimmune disease, and no racial pre- derlying lymphoproliferative disorder or collagen
dilection is evident. Occasional familial cases have vascular disease. Hemolysis also causes other labo-
been reported, and there is conicting data concerning ratory ndings. Mild to moderate indirect bilirubin-
association with HLA-A1, B7, and B8 [23 30]. emia and an increased LDH are common. In cases of
Warm AIHA has a highly variable clinical presen- fulminant hemolysis, the released hemoglobin quickly
tation. Typically patients insidiously develop anemic depletes haptoglobin and produces hemoglobinemia.
symptoms such as weakness, dizziness, fatigue, and Because haptoglobin is an acute phase reactant, its
dyspnea on exertion; other less specic symptoms level may be normal or even increased if the hemolysis
include fever, bleeding, coughing, abdominal pain, is mild and there is adequate hepatic function. Mild
and weight loss [31]. Cases with insidious onset gen- cases increase urobilinogen, and severe cases create
erally follow a chronic waxing and waning course. In hemoglobinuria and urine hemosiderin.
patients with fulminant hemolysis, jaundice, pallor, As previously mentioned, by denition warm au-
edema, dark urine (hemoglobinuria), splenomegaly, toantibodies react more strongly at 37C than at
262 Concise Review: Gehrs and Friedberg

lower temperatures. They generally are polyclonal. complete remission and can discontinue steroids, but
Studies have showed that over 95% of warm AIHA more typically, patients require a maintenance dose
cases have a positive DAT. Consistent with the high [65]. If the patient has no initial response to steroids,
prevalence of IgG among the warm autoantibodies, then the next line of therapy includes splenectomy
the DAT positive cases include 20 66% with only and cytotoxic drugs. Because of side eects, these
IgG detected on the RBC surface, 24 63% with both additional therapies are reserved for patients who still
IgG and C3 on the surface, and 7 14% with only C3 require high-dose steroids after several months of
on the surface [13,21,38]. The vast majority of the IgG treatment.
autoantibodies are in the IgG1 subclass; the IgG3 Splenectomy is usually the second line treatment in
subclass is the next most common, but it is found surgical candidates who fail glucocorticoid therapy.
alone in less than 7% of warm AIHA patients [39,40]. Removal of the spleen theoretically has a 2-fold eect.
The small percentage of negative DAT cases may First, because IgG antibodies predominantly mediate
represent patients with either IgG autoantibodies in warm AIHA, it removes the primary site of extra-
quantities lower than the detectable threshold for the vascular hemolysis. Less importantly, the spleen is a
DAT, IgA autoantibodies, or IgM autoantibodies site of antibody production. Studies indicate that
[41 46]. splenectomy has a response rate of approximately
The characteristics that distinguish a pathologic 60 75%, but many of these patients require mainte-
warm autoantibody from a clinically harmless one are nance with lower doses of steroids, and other patients
not clearly delineated. As a result, there is no diag- relapse months to years later [21,67 69]. The proce-
nostic test to determine whether a warm autoantibody dure has a low morbidity and mortality rate. Due to
produces in vivo RBC destruction. The presence of the infection risk from encapsulated bacteria, sple-
bound IgG1 and especially IgG3 autoantibodies nectomized patients should be vaccinated for pneu-
[6 8,47,48], the presence of concomitantly bound mococcus and meningococcus.
IgA and/or IgM autoantibodies [49], and the quantity Cytotoxic drugs are another treatment option for
of bound RBC autoantibodies and the resulting patients who have failed glucocorticoids and/or
strength of the DAT [50 53] all play a role in the risk splenectomy. Studies suggest that the response rate in
of hemolysis. Other variables aecting hemolytic po- that population is approximately 40 60% [66,68,69].
tential include the quantity of bound complement, Cyclophosphamide is the agent most commonly used,
characteristics of the RBC antigens, the anity of the although azathioprine also is employed. One potential
autoantibodies for those antigens, the number of IgG regimen involves starting the patient with a cytotoxic
Fc and complement receptors on the macrophages, drug and a steroid, tapering the steroid over 3 months
and the functional status of the mononuclear phag- if clinically responsive and tapering the cytotoxic drug
ocytic system [55 57]. Warm autoantibodies are after 6 months of treatment [65]. The possibility of
typically panagglutinins, reacting with all cells on the bone marrow suppression from these agents necessi-
diagnostic antibody panel. Recent studies employing tates careful WBC monitoring. These agents also can
immunoblotting have variably implicated Rh anti- produce secondary malignancies, hemorrhagic cysti-
gens, membrane protein band 4.1, protein band 3, tis, alopecia, and sterility. Cyclosporine A also has
and glycophorin A as universal RBC targets [57 59]. reportedly produced a response in some patients [70].
When apparent, specicity is usually directed against Recently, a case of warm AIHA refractory to steroids
Rh antigens such as e, E, or C. Other reported spec- and the commonly used chemotherapeutics has been
icities include A, B, K, Jka, Fyb, M, N, S, LW, U, reported to respond to rituximab (anti-CD20 mon-
Wrb, Ena, and Ge [16,60 64]. oclonal antibody) treatment [71].
Other therapies have been tried with variable suc-
cess. Plasmapheresis has shown a limited response,
Treatment but in the case of fulminant hemolysis it may be
Therapy generally depends on the severity of the transiently benecial until drug therapy takes eect.
hemolysis, though folic acid supplementation is rec- Some reports suggest that IVIG is benecial, other
ommended for all. If the bone marrow can compen- reports show no benet [72 78]. Danazol has also
sate, then the patient can continue to be monitored. been noted to be eective for some patients [79,80].
However, once anemia develops, glucocorticoids are Another treatment providing anecdotal success is
the rst-line treatment. Patients generally show some vincristine-loaded platelets; selective macrophage in-
improvement within a week, and 70 80% improve jury is the suspected mechanism of its activity [81,82].
within 3 weeks. In responders, the steroid dose can be Although RBC transfusion is not contraindicated
tapered gradually as long as clinical improvement is for AIHA, it is generally limited to cases of life-
maintained. Among new cases, 15 20% will achieve threatening anemia or a high risk of cardiac or cere-
Concise Review: Autoimmune Hemolytic Anemia 263

brovascular ischemic events. Panagglutinating warm episodes of acute hemolysis with hemoglobinemia and
autoantibodies complicate the serologic work-up on hemoglobinuria are more common in the winter
the transfusion service. A panreactive autoantibody months. Because blood is more susceptible to the
can mask an existing alloantibody by making all do- eects of the environmental temperature in the
nor units appear cross-match incompatible, regardless extremities, patients also present with acrocyanosis
of whether an alloantibody is present. If the patient during exacerbations. Some patients experience Ray-
has not been recently transfused and does not have a nauds phenomenon, and rarely the RBC agglutina-
high titer autoantibody, autoadsorption techniques tion becomes signicant enough to produce vascular
can eliminate the confounding autoantibody and re- occlusions with resulting necrosis. These additional
duce this risk. When transfusion is necessary, the least complications may be potentiated by concurrent
incompatible units should be issued, and the infusion cryoglobinemia [88 90].
should be slow and carefully monitored. Donor RBCs The clinical presentation of the transient form of
are destroyed at the same rate as autologous RBCs CAS usually corresponds with the immune response
[83] unless the autoantibodies exhibit specicity, in to an infectious agent. As such, the symptoms appear
which case antigen-negative units should be trans- 2 3 weeks after the infection starts (corresponding to
fused if available. Transfusions may induce further a peaking cold agglutinin titer), and they resolve
autoantibody production [84]. spontaneously an additional 2 3 weeks later. Typi-
cally, the patients present with pallor and jaundice,
but severe cases can result in signicant hemoglo-
COLD AGGLUTININ SYNDROME binuria and transient renal failure. Acrocyanosis can
Background result from exposure to cold temperatures.
Cold-reactive autoantibodies cause two distinct
clinical entities: cold agglutinin syndrome (CAS, cold
hemagglutinin disease) and paroxysmal cold hemo- Laboratory Evaluation
globinuria (PCH). CAS represents approximately Often the laboratory evaluation provides the rst
16 32% of AIHA cases [13,21]. Primary CAS gen- evidence of CAS. Clumping from the cold agglutinins
erally aects older adults, with a peak incidence at complicates both the peripheral blood smear and the
approximately 70 years of age [85]. There appears to calculation of the cell counts and red cell indices. RBC
be a slight female predilection. Infection and lym- clumping artifactually increases the MCV while de-
phoproliferative disorders are the predominant causes creasing the apparent RBC count. Dissolution of the
of secondary cases. The typical case of an infectious clumping upon warming indicates the presence of a
etiology involves mycoplasmal pneumonia or infec- cold agglutinin rather than rouleaux formation or -
tious mononucleosis in an adolescent or young adult. brin clumping. In the absence of an acute exacerba-
Other infectious agents associated with CAS include tion, hemoglobin and hematocrit are mildly decreased.
adenovirus, CMV, inuenza viruses, varicella zoster The reticulocyte count is mildly elevated unless bone
virus, human immunodeciency virus, Escherichia marrow failure is present. The WBC and platelet
coli, Listeria monocytogenes, and Treponema pallidum counts are usually normal. The peripheral smear may
[86]. While infectious etiologies may produce tran- show agglutination, polychromasia, anisocytosis,
sient CAS, lymphoproliferative disorders (e.g., CLL, poikilocytosis, and occasionally spherocytosis. Other
lymphomas, and Waldenstroms macroglobulinemia) common laboratory ndings secondary to the hem-
typically produce a more chronic course. Approxi- olysis include mildly elevated indirect bilirubinemia
mately 40% of these patients with a B cell clone have a and LDH. As with any cause of intravascular hem-
karyotype of trisomy 3 and/or trisomy 12. These olysis, severe exacerbations produce decreased ha-
karyotypes also are seen in some cases of idiopathic ptoglobin, hemoglobinemia, and hemoglobinuria.
CAS. Other neoplasms have been reported in associ- As a group, patients with CAS have more homo-
ation with CAS, including pulmonary squamous cell geneous DAT results than patients with warm AIHA.
carcinoma, metastatic colonic adenocarcinoma, met- Since the pathophysiology of CAS typically involves
astatic adrenal adenocarcinoma, basal cell carcinoma, IgM autoantibodies and complement, patients almost
and a mixed parotid tumor [87]. exclusively have a positive DAT with anti-C3 and
Patients with primary CAS and CAS secondary to polyspecic reagents and a negative result with anti-
a lymphoproliferative disorder commonly have a IgG. The IgM autoantibodies dissociate from the
mild, chronic hemolytic anemia producing pallor and RBCs subsequent to C3 binding, so they generally are
fatigue. The baseline disease is often stable, with any not detected in vitro. In rare cases, cold autoanti-
progression being insidious. However, a cold envi- bodies are IgG or IgA [21,64]. The cold autoanti-
ronment may exacerbate the condition. Therefore, bodies in idiopathic CAS and CAS secondary to
264 Concise Review: Gehrs and Friedberg

lymphoproliferative disorders typically are IgM j commonly is observed with primary CAS and CAS
monoclonal antibodies, while they are polyclonal IgM secondary to lymphoproliferative disorders and my-
j or k in CAS secondary to infection. coplasmal pneumonia. While the majority of patients
By denition cold autoantibodies react more infected with Mycoplasma pneumoniae transiently
strongly at 0 4C than at higher temperatures. Be- produce anti-I agglutinins, most are not clinically
cause cold agglutinins can be detected in most healthy signicant. On the other hand, anti-i agglutinins are
individuals if testing is performed at 0 4C, the ma- typically associated with infectious mononucleosis,
jority of cold autoantibodies clearly are benign. and they also have been identied during CMV in-
Pathologic cold autoantibodies are characterized by a fections and with lymphomas [93]. As with mycopl-
large thermal amplitude or a high titer, with thermal asmal pneumonia, the majority of anti-i cold
amplitude as the better predictor of hemolysis [91]. autoantibodies secondary to infectious mononucleo-
Thermal amplitude refers to the temperature range at sis are benign. Studies have noted that the sera of
which in vitro agglutination is detectable. Activity at 8 69% of infectious mononucleosis patients have
37C always corresponds with a clinically signicant anti-i, but only 0.1 3% exhibit hemolysis [94 98].
cold autoantibody. In addition, pathologic cold au- Rarely do patients with infectious mononucleosis and
toantibodies generally have a titer of greater than angioimmunoblastic lymphadenopathy develop CAS
1:1,000 at 0 4C. Primary CAS and CAS secondary from an IgG anti-i autoantibody [99]. These patients
to lymphoproliferative disorders usually exhibit also produce IgM anti-IgG antibodies that ultimately
higher titers than CAS secondary to infection. initiate the agglutination and hemolysis.
The pathophysiology of CAS is highly temperature- Occasionally cold autoantibodies are identied
dependent. The thermal amplitude determines whether with specicity against the Pr antigen. Anti-Pr ac-
and where initiation occurs. Hemolysis is initiated tivity should be suspected when a cold agglutinin re-
when cold autoantibodies bind to RBCs in the cooler acts equally strongly with both cord and adult RBCs,
peripheral circulation. Since the temperature in the and disappearance of the agglutination with papain
distal extremities ranges from 28 to 31C, cold au- pretreatment further suggests anti-Pr specicity. Anti-
toantibodies with a signicant titer at 30C or more Pr cold agglutinins often are pathologic due to a high
can cause chronic hemolysis. Cold agglutinins with a titer and a high thermal amplitude [100]. Other re-
lower thermal range require excessively cold environ- ported specicities include Gd, Sa, Lud, Fl, Vo, M, N,
ments to bind RBCs. Subsequent to binding RBCs, D, and P [63,64,101].
IgM autoantibodies x C1 and initiate the classical
complement cascade. Warmer temperatures of the
central circulation then facilitate hemolysis. First, the Treatment
higher temperatures maximize complement xation Treatment for CAS is dependent on its etiology and
and activation. In addition, the higher temperatures severity. With primary CAS most patients only have
cause dissociation of the cold agglutinins, which allows mild anemia. Therefore, avoidance of cold exposure is
them to bind additional RBCs back in the colder pe- the primary therapy. For some patients this necessi-
ripheral circulation and repeat the cycle. If the com- tates moving to a warmer climate. Folic acid supple-
plement cascade progresses to the membrane attack mentation also is recommended for these patients. In
complex, intravascular hemolysis results. Otherwise, cases with severe hemolysis, immunosuppression with
bound C3b leads to extravascular hemolysis. chlorambucil or cyclophosphamide may be benecial
Cold autoantibodies commonly show specicity [102]. a-Interferon also reportedly has produced sig-
against the Ii blood group system, with approximately nicant responses [103]. While steroids are eective for
90% directed against the I antigen and most of the WAIHA, they are rarely helpful for CAS. Steroids
remaining ones directed against the i antigen [21]. The only have been noted to be benecial for patients with
I/i antigens are closely related, high-frequency car- either low titer or high thermal amplitude IgM cold
bohydrates that also are related to the ABO antigens. agglutinins or with IgG cold agglutinins. Likewise,
The RBC surface densities of I/i are inversely pro- since extravascular hemolysis in CAS generally occurs
portional to one another: neonatal RBCs almost ex- in the liver, splenectomy has only beneted those pa-
clusively express large amounts of i antigen on their tients with IgG cold agglutinins [104]. Plasmapheresis
surface, but antigen switching occurs during infancy, can provide a temporary improvement in cases of se-
so that I antigen is predominant by 18 months of age. vere hemolysis, and it may be used prophylactically
As a result, antibody panels with adult RBCs will for surgeries requiring cold exposure. However, be-
detect anti-I agglutinins, and cord RBCs are needed cause plasmapheresis has no eect on the underlying
to detect anti-i agglutinins. Only rare adults never production of cold agglutinins, it should be combined
express I antigen on their RBCs [92]. Anti-I specicity with concomitant immunosuppressive therapy.
Concise Review: Autoimmune Hemolytic Anemia 265

Similarly, treatment of the hemolysis often is not mumps, EBV, CMV, varicella zoster virus, adenovi-
necessary for secondary CAS. However, when sig- rus, inuenza A, M. pneumoniae, Haemophilus inu-
nicant hemolysis occurs, treatment of secondary enzae, and E. coli. No racial or gender predilection is
CAS is directed at the underlying cause. Thus, for evident.
CAS resulting from a lymphoproliferative disorder Clinically, PCH is characterized by an acute attack
or another malignancy, treatment typically encom- of high fever, chills, back and/or leg pain, and ab-
passes chemotherapeutic agents. In the context of dominal cramping. Other symptoms may include
CD20-positive chronic lymphoproliferative disorders headache, nausea, vomiting, and diarrhea. Hemo-
producing monoclonal IgM, cold AIHA has been globinuria typically occurs, producing dark red to
successfully treated with rituximab [105,106]. For CAS black urine. A preceding exposure to cold tempera-
secondary to non-viral infections, antibiotics can be tures may be identied retrospectively. Cold urticaria,
helpful. Because infection-associated CAS is a tran- tingling of the extremities, and Raynauds phenome-
sient disorder, immunosuppressives are rarely indi- non have been reported in cases precipitated by cold
cated. In cases with severe hemolysis, plasmapheresis exposure. Constitutional symptoms generally subside
may be necessary. Supportive measures also can in- within a few hours. However, the hemolysis can be
clude transfusion and hydration to maintain adequate severe and even life-threatening and can require up to
renal blood ow. several weeks to recover. The chronic forms of PCH
RBC transfusions should be limited for two rea- are characterized by recurrent episodes of hemolysis
sons. First, as mentioned previously, cold agglutinins precipitated by cold exposure.
cause serologic diculties during the blood bank
work-up. Thus, the blood bank may have to release
least incompatible RBC units that have a higher risk Laboratory Evaluation
of containing an undetected alloantibody. Second, Hematologic ndings include hematocrit and he-
transfused units can potentiate the hemolysis. Most moglobin that may be severely decreased. Reticulo-
cold autoantibodies are directed against the I antigen, cytopenia is common in the acute phase, but
and I antigen-negative donor units are extremely rare. ultimately reticulocytosis develops. Similarly, initial
In addition, if low complement levels limit hemolysis, leukopenia is followed in the recovery phase by a
then exogenous donor plasma complement can ex- normal to high WBC count. The platelet count gen-
acerbate the hemolysis. As a result of these factors, erally is unremarkable. Examination of the peripheral
transfusions typically are reserved for patients with blood smear can show agglutination, polychroma-
signicant underlying cardiovascular or cerebrovas- sia, nucleated RBCs, anisocytosis, poikilocytosis,
cular disease. If necessary, washing RBC units will spherocytes, and erythrophagocytosis. Additional
reduce the exogenous complement load. The risk of evidence of intra- and extravascular hemolysis is
additional, transfusion-related hemolysis can be re- provided by increased LDH, indirect bilirubinemia,
duced by using an in-line blood warmer at 37C and decreased haptoglobin, hemoglobinemia, and hemo-
by keeping the patient warm [107]. globinuria. The urine also contains methemoglobin.
Since PCH can produce acute renal failure, elevated
BUN and creatinine may be present.
PAROXYSMAL COLD HEMOGLOBINURIA PCH is caused by a biphasic IgG autoantibody
Background (Donath Landsteiner antibody) that xes comple-
Paroxysmal cold hemoglobinuria (PCH) is a rela- ment at low temperatures but ultimately dissociates at
tively uncommon form of AIHA, with studies noting higher temperatures. As a result, the DAT is positive
an incidence of 2 10% among cases of hemolytic with anti-C3, but it is generally negative with anti-IgG
anemia [21,108,109]. Both idiopathic and secondary unless performed at colder temperatures. Biphasic
forms of the disease exist. Both idiopathic PCH and IgG autoantibodies bind RBCs eciently at 0 4C
PCH secondary to late-stage or congenital syphilis are and subsequently x complement C1 at that temper-
chronic processes that have become increasingly un- ature. However, the other complement components
common. Most present-day cases of PCH have an bind more eciently and cause lysis at temperatures
acute transient pathology secondary to infection. nearer normal body temperature. While the maximal
These acute cases predominantly aect children, and thermal amplitude of the autoantibody typically does
they represent one of the leading causes of acute he- not extend above 20C in vitro with acute transient
molytic anemia in children [109,110]. While most PCH, for unknown reasons hemolysis can still occur
cases involve an antecedent upper respiratory infec- in the absence of cold exposure. Occasionally Do-
tion, the causative agent often is not identied. In- nath Landsteiner antibodies appear to be mono-
fectious agents associated with PCH include measles, phasic because their thermal amplitude extends to
266 Concise Review: Gehrs and Friedberg

body temperature. Donath Landsteiner antibodies Laboratory Evaluation


are potent, so even small titers can produce hemoly- The laboratory work-up shows that the DAT is
sis. As with cold agglutinins, biphasic IgG autoanti- positive for both IgG and C3. As with the separate
bodies often exhibit specicity. Usually they are diseases, the mixed-type AIHA produces diculties
directed against the P antigen, which is found on the with the antibody screen and the cross-match. The
RBCs of most individuals [111]. RBC eluate typically indicates panreactive warm IgG
The Donath Landsteiner test was developed to autoantibody. The cold autoantibody usually exhibits
detect the biphasic IgG autoantibody responsible for specicity against the I antigen, but reactivity against
PCH. It involves in vitro testing for hemolysis with a i has been reported [114 116]. Donor units have to be
patients serum, washed group O cells expressing the released as cross-match least-incompatible due to the
P antigen, and fresh normal serum (complement autoantibodies.
source) [112]. One group of samples is incubated only
at 0 4C, another group is incubated only at 37C,
and the third group is incubated rst at 0 4C for 30 Treatment
min and then at 37C for 60 min. The diagnosis of
Mixed-type AIHA appears to respond to treatment
PCH is indicated when hemolysis only is observed in
in a similar manner as warm AIHA. Patients gener-
samples with patients serum which have been incu-
ally respond to steroids, and immunosuppressive
bated rst at 0 4C for 30 min and then at 37C for
agents and splenectomy have been employed suc-
60 min.
cessfully as well. Associated diseases, if present, also
need to be treated to optimize recovery.

Treatment
Since most cases of PCH are self-limited, treatment DRUG-INDUCED AUTOIMMUNE
is usually symptomatic, including keeping the patient HEMOLYTIC ANEMIA
warm. While needed blood transfusions would ideally Background
consist of P antigen-negative RBCs, the rarity of Drugs can produce hemolysis by both immune and
these units normally precludes their use. P antigen- non-immune mechanisms. Historically, a-methyldopa
positive RBCs typically are benecial when a blood and high-dose penicillin were responsible for the
warmer is employed [113]. Although glucocorticoids majority of cases of drug-induced IHA. Studies
have not been shown to be eective routinely, they published in the early 1980s, when these two agents
often are given to severely anemic children. Splenec- were more commonly used, indicated that 12 18% of
tomy typically is not helpful. In cases of life-threat- AIHA cases were drug-induced [13,21]. While the
ening PCH, plasmapheresis can temporarily dampen incidence of drug-induced IHA has likely decreased
the hemolysis. Syphilis should be ruled out in patients since then, second- and third-generation cephalospo-
with chronic PCH. If syphilis is present, treatment rins, especially cefotetan and ceftriaxone, have been
generally eliminates the concurrent hemolysis. associated increasingly with cases of drug-induced
IHA [117,118]. Rarely, these cases of cephalosporin-
induced IHA are fatal.
MIXED-TYPE AUTOIMMUNE HEMOLYTIC ANEMIA Drug-induced IHA results from several types of
Background interactions between a drug, antibodies, and RBC
Some patients with warm AIHA also possess a cold membrane components. The three major mechanisms
agglutinin. While the majority of these cold aggluti- include induction of autoantibodies, neoantigen (im-
nins are not clinically signicant, occasionally they mune complex) formation, and drug adsorption onto
have a sucient thermal amplitude (greater than the RBCs. IHA secondary to neoantigen formation
30C) or high titer (greater than 1:1,000 at 0 4C) to occurs when a drug binds weakly to a normal RBC
indicate CAS. Similar to the separate entities, mixed- component, and the immune system perceives the
type AIHA can be either idiopathic or secondary to drug+RBC component complex or the conforma-
lymphoproliferative disorders or SLE. Patients usu- tionally altered RBC component as foreign. IHA due
ally have a chronic course interrupted by severe ex- to the drug adsorption mechanism results when an-
acerbations, which can result in a hemoglobin level tibodies largely directed against the drug interact with
below 5.0 g/dL. These exacerbations do not appear RBCs to which the drug has strongly bound. These
to be associated with cold exposure, and they do mechanisms are not mutually exclusive. For example,
not result in acrocyanosis or Raynauds phenome- while cephalosporins generally are associated with
non [114]. IHA due to the neoantigen formation and/or drug
Concise Review: Autoimmune Hemolytic Anemia 267

adsorption mechanisms, they also can induce au- evident hemolysis, a positive DAT alone is not an
toantibody formation [118 120]. indication to discontinue drug usage. If the hemolysis
is clinically signicant, use of the suspected drug
should be discontinued, especially when alternative
Laboratory Evaluation therapies exist. Drug-induced AIHA usually resolves
The three mechanisms can be distinguished on the within several days of discontinuing the medication
basis of serologic reactions of the serum and the el- but occasionally requires months to fully resolve
uate (Table III). Drug-induced IHA secondary to [132]. In cases with severe hemolysis, corticosteroids
neoantigen formation or drug adsorption has a pos- may aid in recovery. RBC transfusions also can be
itive DAT and can be serologically distinguished from given, but the transfused RBCs will be hemolyzed at a
the autoantibody mechanism because of the require- similar rate as the endogenous RBCs in the typical
ment for exogenous drug to detect the antibody. In case where the donor unit of RBCs is crossmatch
contrast, with drug-induced AIHA, the serum and incompatible.
eluate antibodies react with RBC panels even if drug
is absent. Drug-induced AIHA is serologically indis-
tinguishable from warm AIHA; a presumptive diag- SUMMARY
nosis can be made only if the patient responds to Diagnosis of AIHA requires both serologic evi-
withdrawal of the drug. a-Methyldopa is the proto- dence of an autoantibody and clinical and labora-
typical drug operating by the induction of autoanti- tory evidence of hemolysis. Cases of AIHA can be
bodies, producing a positive DAT in 11 36% of classied on the basis of the characteristic tempera-
patients (dose-dependent) within 3 6 months of ini- ture reactivity of the RBC autoantibody. The
tiation [121,122]. The DAT is positive due to bound autoantibodies associated with warm AIHA and
IgG, and occasionally complement may also be pre- drug-induced AIHA react most strongly near body
sent. These autoantibodies are typically panreactive, temperature, while CAS and PCH are characterized
though documented specicities include c, e, Wrb, by autoantibodies that react most strongly near
Jka, and U [16,123 125]. As mentioned previously, 0 4C. Rarely patients develop a mixed-type AIHA
second- and third-generation cephalosporins occa- with both warm and cold autoantibodies. Cases of
sionally induce RBC autoantibodies [118]. Other AIHA also can be classied on the basis of etiology
drugs that reportedly can induce autoantibodies in- as idiopathic or secondary. While lymphoprolifera-
clude levodopa, mefenamic acid, procainamide, and tive disorders are the most common secondary
diclofenac [127 130]. cause, autoimmune disorders sometimes produce
warm AIHA, and infections occasionally lead to
cold AIHA. When clinically indicated, treatment of
Treatment warm AIHA and mixed-type AIHA includes gluco-
Despite the high incidence of a positive DAT as- corticoids, splenectomy, and immunosuppressive
sociated with a-methyldopa usage, the responsible agents. If drug-induced AIHA is suspected, the ini-
autoantibody results in hemolytic anemia in less than tial treatment is discontinuation of the drug. The
1% of patients [131]. Therefore, without clinically primary treatment for cold AIHA is warming the

Table III. Serologic Characteristics of Drug-Induced Hemolytic Anemias*

AutoAb formation Drug adsorption Neoantigen formation

DAT
Polyspecic + + +
IgG + + Usually )
C3 Usually ) Usually ) +
Serum Ab
Routine ) )
Soluble drug ) +
Drug-treated RBC's + +
RBC eluate Ab
Routine + ) )
Soluble drug + ) )
Drug-treated RBC's + + )

*Ab, antibody; DAT, direct antiglobulin test; RBC, red blood cell.
268 Concise Review: Gehrs and Friedberg

patient. Glucocorticoids and splenectomy have lim- 17. Garratty G. The signicance of IgG on the red cell surface.
ited benet in patients with CAS and PCH. RBC Transfus Med Rev 1987;1:47 57.
transfusions are not absolutely contraindicated, 18. Lau P, Haesler WE, Wurzel HA. Positive direct antiglobulin
though they should be limited to patients with life- reaction in a patient population. Am J Clin Pathol 1976;65:
368 375.
threatening anemia or signicant cardiovascular or
19. Okuno T, Germino F, Newman B. Clinical signicance of
cerebrovascular disease due to the increased risk of
autologous control. Am Soc Clin Pathol 1984 (abstr).
possessing undetected alloantibodies and the risk of
20. Judd WJ, Barnes BA, Steiner EA, Oberman HA, Averill DB,
exacerbating the hemolysis. Butch SH. The evaluation of a positive direct antiglobulin test
(autocontrol) in pretransfusion testing revisited. Transfusion
1986;26:220 224.
REFERENCES 21. Petz LD, Garratty G. Acquired immune hemolytic anemias. New
York: Churchill Livingstone; 1980.
1. Pirofsky B. Autoimmunization and the autoimmune hemolytic 22. Vaiopoulos G, Kyriakou D, Papadaki H, Fessas P, Eliopoulos
anemias. Baltimore: Williams & Wilkins; 1969. GD. Multiple myeloma associated with autoimmune hemolytic
2. Bottiger LE, Westerholm B. Acquired haemolytic anaemia. Acta anemia. Haematologica 1994;79:262 264.
Med Scand 1973;193:223 226. 23. Fialkow PJ, Fudenberg H, Epstein WV. Acquired antibody he-
3. Linden JV, Paul B, Dressler KP. A report of 104 transfusion molytic anemia and familial aberrations in gamma globulins. Am
errors in New York State. Transfusion 1992;32:601 606. J Med 1964;36:188 199.
4. Heddle NM, Soutar RL, OHoski PL, Singer J, McBride JA, Ali 24. Dodds CE. Familial auto-immune hemolytic anemia. Arch Intern
MA, Kelton JG. A prospective study to determine the frequency Med 1965;116:273 276.
and clinical signicance of alloimmunization post-transfusion. Br
25. Cordova MS, Baez-Villasenor J, Mendez JJ, Campos E. Acquired
J Hematol 1995;91:1000 1005.
hemolytic anemia with positive antiglobulin (Coombs test) in
5. Vamvakas EC, Pineda AA, Reisner R, Santrach PJ, Moore SB. mother and daughter. Arch Intern Med 1966;117:692 695.
The dierentiation of delayed hemolytic and serologic transfusion
26. Pirofsky B. Hereditary aspects of autoimmune hemolytic anemia:
reactions: incidence and predictors of hemolysis. Transfusion
a retrospective analysis. Vox Sang 1968;14:334 347.
1995;35:26 32.
27. Da Costa JAG, White AG, Parker AC, Grigor GB. Increased
6. Abramson N, Gelfand EW, Jandl JH, Rosen FS. The interaction
incidence of HL-A 1 and 8 in patients showing IgG or comple-
between human monocytes and red cells: specicity for IgG
ment coating on their red cells. J Clin Pathol 1974;27:353 355.
subclasses and IgG fragments. J Exp Med 1970;132:1207 1215.
28. Clauvel JP, Marcelli-Barge A, Gautier Coggia I, Poirier JC,
7. Anderson CL, Looney RJ. Human leukocyte IgG Fc receptors.
Benajam A, Dausset J. HL-A antigens and idiopathic autoim-
Immunol Today 1986;7:264.
mune hemolytic anemias. Transplant Proc 1974;6:447 448.
8. Ravetch J, Kinet J-P. Fc receptors. Annu Rev Immunol
1991;9:457 492. 29. Abdel-Khalik A, Paton L, White AG, Urbaniak SJ. Human
leucocyte antigens A, B, C and DRW in idiopathic warm
9. Ehlenberger AG, Nussenzweig V. The role of membrane recep- autoimmune haemolytic anaemia. Br Med J 1980;280:760 761.
tors for C3b and C3d in phagocytosis. J Exp Med 1977;145:
357 371. 30. Toolis F, Parker AC, White A, Urbaniak S. Familial autoim-
mune haemolytic anaemia. Br Med J 1977;1:1392.
10. Evans RS, Weiser RS. The serology of autoimmune hemolytic
disease: observations on forty-one patients. Arch Intern Med 31. Pirofsky B. Clinical aspects of autoimmune hemolytic anemia.
1957;100:371 399. Semin Hematol 1976;13:251 265.

11. Dausset J, Colombani J. The serology and the prognosis of 128 32. Sokol RJ, Hewitt S, Stamps B. Erythrocyte autoantibodies,
cases of autoimmune hemolytic anemia. Blood 1959;14: autoimmune haemolysis and pregnancy. Vox Sang 1982;43:
1280 1301. 169 176.

12. Bell CA, Zwicker H, Sacks HJ. Autoimmune hemolytic anemia: 33. Starksen NF, Bell WE, Kickler TS. Unexplained hemolytic
routine serologic evaluation in a general hospital population. Am anemia associated with pregnancy. Am J Obstet Gynec 1983;146:
J Clin Pathol 1973;60:903 911. 617 622.

13. Sokol RJ, Hewitt S, Stamps BK. Autoimmune haemolysis: an 18- 34. Burt RL, Prichard RW. Acquired hemolytic anaemia in preg-
year study of 865 cases referred to a regional transfusion centre. nancy: report of a case. Obstet Gynecol 1957;10:444 450.
Br Med J 1981;282:2023 2027. 35. Yam P, Wilkinson L, Petz LD, Garratty G. Studies on hemolytic
14. Toy PT, Chin CA, Reid ME, Burns MA. Factors associated with anemia in pregnancy with evidence for autoimmunization in a
positive direct antiglobulin tests in pretransfusion patients: a case patient with a negative direct antiglobulin (Coombs) test. Am J
control study. Vox Sang 1985;49:215 220. Hematol 1980;8:23 29.

15. Clark JA, Tanley PC, Wallas CH. Evaluation of patients with 36. Liesveld JL, Rowe JM, Lichtman MA. Variability of the ery-
positive direct antiglobulin tests and nonreactive eluates discov- thropoietic response in autoimmune hemolytic anemia: analysis
ered during pretransfusion testing. Immunohematology 1992;8: of 189 cases. Blood 1987;69:820 826.
9 12. 37. Evans RS, Takahashi K, Duane RT, Payne R, Liu C-K. Primary
16. Issitt PD, Pavone BG, Goldnger D, Zwikder H, Issitt CH, Tessel thrombocytopenic purpura and acquired hemolytic anaemia.
JA, Kroovand SW, Bell CA. Anti Wrb and other autoantibodies Evidence for a common etiology. Arch Intern Med 1951;87:48 65.
responsible for positive direct antiglobulin tests in 150 individu- 38. Chaplin H Jr. Clinical usefulness of specic antiglobulin reagents
als. Br J Haematol 1976;34:5 18. in autoimmune hemolytic anemias. Prog Hematol 1973;7:25 49.
Concise Review: Autoimmune Hemolytic Anemia 269
39. Engelfriet CP, Overbeeke MA, von dem Borne AE. Autoimmune 57. Wakui H, Imai H, Kobayashi R, Itoh H, Notoya T, Yoshida K,
hemolytic anemia. Semin Hematol 1992;29:3 12. Nakamoto Y, Miura AB. Autoantibody against erythrocyte
40. Garratty G. Factors aecting the pathogenicity of red cell auto- protein 4.1 in a patient with autoimmune hemolytic anemia.
and alloantibodies. In: Nance SJ, editor. Immune destruction of Blood 1988;72:408 412.
red blood cells. Arlington, VA: American Association of Blood 58. Victoria EJ, Pierce SW, Branks MJ, Masouredis SP. IgG red
Banks; 1989. p 109. blood cell autoantibodies in autoimmune hemolytic anemia bind
to epitope on red blood cell membrane band 3 glycoprotein. J
41. Sokol RJ, Hewitt S, Booker DJ, Stamps R. Small quantities of
Lab Clin Med 1990;115:74 88.
erythrocyte-bound immunoglobulins and autoimmune haemoly-
sis. J Clin Pathol 1987;40:254 257. 59. Leddy FP, Falany JL, Kissel GE, Passador ST, Rosenfeld SI.
Erythrocyte membrane proteins reactive with human (warm-re-
42. Schmitz N, Djibey I, Kretschmer V, Mahn I, Mueller-Eckhardt
acting) anti-red cell autoantibodies. J Clin Invest 1993;91:
C. Assessment of red cell autoantibodies in autoimmune hae-
1672 1680.
molytic anaemia of warm type by a radioactive anti-IgG test. Vox
Sang 1981;41:224 230. 60. Celano MJ, Levine P. Anti-LW specicity in autoimmune ac-
quired hemolytic anemia. Transfusion 1967;7:265 268.
43. Yam P, Petz LD, Spath P. Detection of IgG sensitization of red
cells with 125I staphylococcal protein A. Am J Hematol 61. Marsh WL, Reid ME, Scott EP. Autoantibodies of U blood
1982;12:337 346. group specicity in autoimmune haemolytic anaemia. Br J Hae-
matol 1972;22:625 629.
44. Sturgeon P, Smith LE, Chun HMT, Hurvitz CH, Garratty G,
Goldnger D. Autoimmune hemolytic anemia associated exclu- 62. Marsh WL, Oyen E, Alicea M, Linter M, Horton S. Autoimmune
sively with IgA of Rh specicity. Transfusion 1979;19:324 328. hemolytic anemia and the Kell blood groups. Am J Hematol
1979;7:155 162.
45. Reusser P, Osterwalder B, Burri H, Speck B. Autoimmune he-
molytic anemia associated with IgAdiagnostic and therapeutic 63. Garratty G. Target antigens for red-cell-bound autoantibodies.
aspects in a case with long-term follow-up. Acta Haematol In: Nance SJ, editor. Clinical and basic science aspects of im-
1987;77:53 56. munohematology. Arlington, VA: American Association of
Blood Banks; 1991. p 33.
46. Salama A, Mueller-Eckhardt M. Autoimmune haemolytic anae-
mia in childhood associated with non-complement binding IgM 64. Dacie JV. The autoimmune haemolytic anaemias. 3rd edition.
autoantibodies. Br J Haematol 1987;65:67 71. Edinburgh: Churchill Livingstone; 1992.
47. LoBuglio AF, Cotran RS, Jandl JH. Red cells coated with im- 65. Murphy S, LoBuglio AF. Drug therapy of autoimmune hemo-
munoglobulin G: binding and sphering by mononuclear cells in lytic anemia. Semin Hematol 1976;13:323 334.
man. Science 1967;158:1582 1585. 66. Pirofsky B, Bardana EJ Jr. Autoimmune hemolytic anemia. II.
48. Augener W, Grey HM, Cooper NR, Muller-Eberhard HJ. The Therapeutic aspects. Ser Haematol 1974;7:376 385.
reaction of monomeric and aggregated immunoglobulins with 67. Bowdler AS, The role of the spleen and splenectomy in autoim-
C1. Immunochemistry 1971;8:1011 1020. mune hemolytic disease. Semin Hematol 1976;13:335 348.
49. Sokol RJ, Hewitt S, Booker DJ, Bailey A. Red cell autoanti- 68. Zupanska B, Sylwestrowicz T, Pawelski S. The results of pro-
bodies, multiple immunoglobulin classes, and autoimmune hem- longed treatment of autoimmune haemolytic anaemia. Haemat-
olysis. Transfusion 1990;30:714 717. ologia 1981;14:425 433.
50. Lalezari P. Serologic prole in autoimmune hemolytic disease: 69. Gibson J. Autoimmune hemolytic anemia: current concepts. Aust
pathophysiologic and clinical interpretations. Semin Hematol NZ J Med 1988;18:625 637.
1976;13:291 310. 70. Hershko C, Sonnenblick M, Ashkenazi J. Control of steroid-
51. Van der Meulen FW, de Bruin HG, Goosen PC, Bruynes EC, resistant autoimmune haemolytic anaemia by cyclosporine. Br J
Joustra-Maas CJ, Telkamp HG, von dem Borne AE, Engelfriet Haematol 1990;76:436 437.
CP. Quantitative aspects of the destruction of red cells sensitized 71. Ahrens N, Kingreen D, Seltsam A, Salama A. Treatment of re-
with IgG1 autoantibodies: an application of ow cytouorome- fractory autoimmune haemolytic anaemia with anti-CD20 (rit-
try. Br J Haematol 1980;46:47 56. uximab). Br J Haematol 2001;114:244 245.
52. Garratty G, Nance SJ. Correlation between in vivo hemolysis and 72. Anderson O, Taaning E, Rosenkvist J, Moller NE, Mogensen
the amount of red cell-bound IgG measured by ow cytometry. HH. Autoimmune haemolytic anaemia treated with multiple
Transfusion 1990;30:617 621. transfusion, immunosuppressive therapy, plasma exchange, and
53. Dubarry M, Charron C, Habibi B, Bretagne Y, Lambin P. desferrioxamine. Acta Paediatr Scand 1984;73:145 148.
Quantitation of immunoglobulin classes and subclasses of au- 73. Kutti J, Wadenvik H, Safai-Kutti S, Bjorkander J, Hanson LA,
toantibodies bound to red cells in patients with and without Westberg G, Johnsen SA, Larsson B. Successful treatment of
hemolysis. Transfusion 1993;33:466 471. refractory autoimmune haemolytic anaemia by plasmapheresis.
54. Kay NE, Douglas SD. Monocyte erythrocyte interaction in vi- Scand J Haematol 1984;32:149 152.
tro in immune hemolytic anemias. Blood 1977;50:889 897. 74. Macintyre EA, Linch DC, Macey MG, Newland AC. Successful
55. Gallagher MT, Branch DR, Mison A, Petz LD. Evaluation of response to intravenous immunoglobulin in autoimmune hae-
reticuloendothelial function in autoimmune hemolytic anemia molytic anaemia. Br J Haematol 1985;60:387 388.
using an in vitro assay of monocyte macrophage interaction 75. Bussel JB, Cunningham-Rundles C, Abraham C. Intravenous
with erythrocytes. Exp Hematol 1983;11:82 89. treatment of autoimmune hemolytic anemia with very high dose
56. Chaplin H. Red cell-bound immunoglobulin as a predictor of gammaglobulin. Vox Sang 1986;51:264 269.
severity of hemolysis in patients with autoimmune hemolytic 76. Flores G, Cunningham-Rundles C, Newland AC, Bussel JB.
anemia [editorial]. Transfusion 1990;30:576 577. Ecacy of intravenous immunoglobulin in the treatment of
270 Concise Review: Gehrs and Friedberg
autoimmune hemolytic anemia: results in 73 patients. Am J He- 98. Dacie JV, Worlledge SM. Autoimmune hemolytic anemias. In:
matol 1993;44:237 242. Brown EB, Moore CV, editors. Progress in hematology, Vol VI.
77. Salama A, Mahn I, Neuzner J, Graubner M, Mueller-Eckhardt New York: Grune and Stratton; 1969. p 82 120.
C. IgG therapy in autoimmune haemolytic anaemia of warm 99. Capra JD, Dowling P, Cook S, Kunkel HG. An incomplete cold-
type. Blut 1984;48:391 392. reactive G antibody with i specicity in infectious mononucleosis.
78. Weinblatt ME. Treatment of immune hemolytic anemia with Vox Sang 1969;16:10 17.
gammaglobulin [letter]. J Pediatr 1987;110:817. 100. Roelcke D. Cold agglutination. Transfus Med Rev 1989;3:
79. Ahn YS. Ecacy of danazol in hematologic disorders. Acta 140 166.
Haematol 1990;84:122 129. 101. von dem Borne AE, Mol JJ, Joustra-Maas N, Pegels JG,
80. Pignon J-M, Poirson E, Rochant H. Danazol in autoimmune Langenhuijsen MM, Engelfriet CP. Autoimmune haemolytic
haemolytic anaemia. Br J Haematol 1993;83:343 345. anaemia with monoclonal IgM (j) anti-P cold autohaemolysins.
Br J Haematol 1982;50:345 350.
81. Gertz MA, Petitt RM, Pineda AA, Wick MR, Burgstaler EA.
Vinblastine-loaded platelets for autoimmune hemolytic anemia. 102. Evans RS, Baxter E, Gilliland BC. Chronic hemolytic anemia due
Ann Intern Med 1981;95:325 326. to cold agglutinins: a 20-year history of benign gammopathy with
response to chlorambucil. Blood 1973;42:463 470.
82. Ahn YS, Harrington WJ, Byrnes JJ, Pall L, McCrainie J.
Treatment of autoimmune hemolytic anemia with Vinca-loaded 103. OConnor B, Cliord JS, Lawrence WD, Logue GL. Alpha-in-
platelets. JAMA 1983;249:2189 2194. terferon for severe cold agglutinin disease. Ann Intern Med
1989;111:255 256.
83. Plapp F, Beck ML. Transfusion support in the management
of immune hemolytic disorders. Clin Haematol 1984;13: 104. Silberstein LE, Berkman EM, Schreiber AD. Cold hemagglutinin
167 183. disease associated with IgG cold-reactive antibody. Ann Intern
Med 1987;106:238 242.
84. Ness PM, Shirley RS, Thoman SK, Buck SA. The dierentiation
of delayed serologic and delayed hemolytic transfusion reactions: 105. Lee EJ, Kueck B. Rituxan in the treatment of cold agglutinin
incidence, long-term serologic ndings, and clinical signicance. disease. Blood 1998;92:3490 3491.
Transfusion 1990;30:688 693. 106. Bauduer F. Rituximab: a very ecient therapy in cold agglutinins
85. Schubothe H. The cold hemagglutinin disease. Semin Hematol and refractory autoimmune haemolytic anaemia associated with
1966;3:27 47. CD20-positive, low-grade non-Hodgkins lymphoma. Br J Hae-
matol 2001;112:1085 1086.
86. Thomas AT. Autoimmune Hemolytic Anemias. In: Lee GR,
Foerster J, Lukens J, Paraskevas F, Greer JP, Rodgers GM, 107. Roseneld RE, Jagathambal. Transfusion therapy for autoim-
editors. Wintrobes clinical hematology. 10th edition. Baltimore: mune hemolytic anemia. Semin Hematol 1976;13:318 319.
Williams & Wilkins; 1999. 108. Van Loghem JJ, van der Hart M, Dorfmeier H. Serological
87. Wortman J, Rosse W, Logue G. Cold agglutinin autoimmune studies in acquired hemolytic anemia. Sixth International Con-
hemolytic anemia in nonhematologic malignancies. Am J He- gress of the International Society of Hematology. New York:
matol 1979;6:275 283. Grune and Stratton; 1958.
88. Mitchell AB, Pergrum GD, Gill AM. Cold agglutinin disease 109. Heddle NM. Acute paroxysmal cold hemoglobinuria. Transfus
with Raynauds phenomenon. Proc R Soc Med 1974;67: Med Rev 1989;3:219 229.
113 115. 110. Gottsche B, Salama A, Mueller-Eckhardt C. Donath-Landsteiner
89. Shelley WB, Shelley ED. Acrocyanosis of cold agglutinin disease autoimmune hemolytic anemia in children. A study of 22 cases.
successfully treated with antibiotics. Cutis 1984;33:556 557. Vox Sang 1990;58:281 286.
111. Levine P, Celano MJ, Falkowski F. The specicity of the antibody
90. Roelcke D. Cold agglutination. Trans Med Rev 1989;3:140 166.
in paroxysmal cold hemoglobinuria. Transfusion 1963;3:278 280.
91. Rosse WF, Adams JP. The variability of hemolysis in the cold
112. AABB Technical Manual. 13th edition. Bethesda, MD: AABB;
agglutinin syndrome. Blood 1980;56:409 416.
1999.
92. Issitt PD. I blood group system and its relation to other blood
113. Sokol RJ, Hewitt S, Stamps BK. Autoimmune haemolysis asso-
group systems. J Med Lab Tech 1967;24:90 97.
ciated with Donath Landsteiner antibodies. Acta Haematol
93. Horwitz CA, Skradski K, Reece E, Lewis FB, Schwartz B, Kelty 1982;68:268 277.
R, Polesky H. Haemolytic anaemia in previously healthy adult
114. Shulman IA, Branch DR, Nelson JM, Thompson JC, Saxena S,
patients with CMV infections: report of two cases and an eval-
Petz LD. Autoimmune hemolytic anemia with both cold and
uation of subclinical haemolysis in CMV mononucleosis. Scand J
warm autoantibodies. J Am Med Assoc 1985;253:1746 1748.
Haematol 1984;33:35 42.
115. Sokol RJ, Hewitt S, Stamps BK. Autoimmune haemolysis. Mixed
94. Jenkins WJ, Koster HG, Marsh WL, Carter RL. Infectious warm and cold antibody type. Acta Haematol 1983;69:266 274.
mononucleosis: an unsuspected source of anti-i. Br J Haematol
1965;11:480 483. 116. Kajii E, Miura Y, Ikemoto S. Characterization of autoantibodies
in mixed-type autoimmune hemolytic anemia. Vox Sang
95. Hossaini AA. Anti-i infectious mononucleosis. Am J Clin Pathol 1991;60:45 52.
1970;53:198 203.
117. Shammo JM, Calhoun B, Mauer AM, Homan PC, Baron JM,
96. Horwitz CA, Moulds J, Henle W, Henle G, Polesky H, Balfour Baron BW. First two cases of immune hemolytic anemia associ-
HH, Schwartz B, Ho T. Cold agglutinins in infectious mono- ated with ceftizoxime. Transfusion 1999:39:838 844.
nucleosis and heterophil-antibody-negative mononucleosis-like
118. Arndt PA, Leger RM, Garratty G. Serology of antibodies to
syndromes. Blood 1977;50:195 200.
second- and third-generation cephalosporins associated with im-
97. Hoagland RJ. Infectious mononucleosis. New York: Grune and mune hemolytic anemia and/or positive direct antiglobulin tests.
Stratton; 1967. Transfusion 1999;39:1239 1246.
Concise Review: Autoimmune Hemolytic Anemia 271
119. Shulman IA, Arndt PA, McGehee W, Garratty G. Cefotaxime- 126. Kessey EC, Pierce S, Beck ML, Bayer WL. a-Methyldopa-in-
induced immune hemolytic anemia due to antibodies reacting in duced hemolytic anemia involving autoantibody with U speci-
vitro by more than one mechanism. Transfusion 1990;30:263 266. city. Transfusion 1973;13:360 (abstr).
120. Chenoweth CE, Judd WJ, Steiner EA, Kauman, CA. Cefotetan- 127. Lindstrom FD, Lieden G, Engstrom MS. Dose-related levodopa-
induced immune hemolytic anemia. Clin Infect Dis 1992;15: induced haemolytic anaemia. Ann Intern Med 1977;86:298 300.
863 865. 128. Scott GL, Myles AB, Bacon PA. Autoimmune haemolytic
121. Carstairs KC, Breckenridge A, Dollery CT, Worlledge SM. In- anaemia and mefenamic acid therapy. Br J Med 1968;3:
cidence of a positive direct Coombs test in patients on a-meth- 534 535.
yldopa. Lancet 1966;2:133 134. 129. Kleinman S, Nelson R, Smith L, Goldnger D. Positive direct
122. Worlledge SM, Carstairs KC, Dacie JV. Autoimmune haemolytic antiglobulin tests and immune hemolytic anemia in patients re-
anaemia associated with a-methyldopa therapy. Lancet ceiving procainamide. N Engl J Med 1984;311:809 812.
1966;2:135 139. 130. Salama A, Kroll H, Wittmann G, Mueller-Eckhardt C. Diclofe-
123. LoBuglio AF, Jandl JH. The nature of the a-methyldopa red-cell nac-induced immune haemolytic anaemia: simultaneous occur-
antibody. N Engl J Med 1967;276:658 665. rence of red blood cell autoantibodies and drug-dependent
124. Bakemeier RF, Leddy JP. Erythrocyte autoantibody associated antibodies. Br J Haematol 1996;95:640 644.
with a-methyldopa: heterogeneity of structure and specicity. 131. Worlledge SM. Immune drug-induced hemolytic anemias. Semin
Blood 1968;32:1 14. Hematol 1969;6:181 200.
125. Patten E, Beck CE, Scholl C, Stroope RA, Wukasch C. Auto- 132. Ewing DJ, Hughes CJ, Wardle DF. Methyldopa-induced auto-
immune hemolytic anemia with anti-Jka specicity in a patient immune haemolytic anaemiaa report of two further cases.
taking Aldomet. Transfusion 1977;17:517 520. Guys Hosp Rep 1968;117:111 118.
PLATELET
PLATELET
DEVELOPMENT AND
DEVELOPMENT AND
FUNCTION
FUNCTION
Joseph E. Kiss, M. D.
Division of
Hematology/Oncology

Thrombocytopenia in the THE CENTRAL ROLE OF


Intensive Care Unit PLATELETS IN HEMOSTASIS

VESSEL
VESSEL
WALL
WALL
Adhesion

PLATELETS
PLATELETS
Procoagulant Aggregation
Activity

COAGULATION
COAGULATION PLATELETS
PLATELETS
Vanderschueren: Crit Care Med, Volume 28(6). 2000:1871

The granular appearance of the platelets helps to distinguish


them from artifact in the peripheral smear

Maslak, P. ASH Image Bank 2005;2005:101353

Copyright 2005 American Society of Hematology. Copyright restrictions may apply.

1
Platelets are seen breaking off from the cytoplasm of a
megakaryocyte Thrombopoietin Binding to
I125
a Megakaryocyte

Maslak, P. ASH Image Bank 2003;2003:100873

Copyright 2003 American Society of Hematology. Copyright restrictions may apply.

THROMBOPOIETIN (TPO)
THROMBOPOIETIN (TPO) z TPO level is regulated by platelet + megakaryocyte
mass
z Regulates megakaryocytopoiesis and thrombopoiesis TPO expression is constitutive
(proliferation and maturation); alternate name - Platelets/Megas bind, internalize, and degrade TPO
Megakaryocyte Growth and Differentiation Feedback loop: platelets / TPO / platelets /
Factor(MGDF)
z Cloned in 1994 using the c-Mpl cytokine receptor TPO
(itself identified in 1990) to isolate and purify the z TPO increases the number, size, and ploidy of
factor megakaryocytes
z Structure: 332 amino acids w/ two domains z TPO increases basal platelet levels 4-6X and
NH3-terminal: EPO-like - contains c-Mpl binding promotes earlier recovery from a thrombocytopenic
site nadir
COOH-terminal: glycosylated end - increases circ. z Novel TPO-mimetic drugs newly licensed
t1/2 (romiplostim, eltrombopag)

2
PLATELET GRANULE COMPOSITION
PLATELET GRANULE COMPOSITION
Dense Bodies
Alpha Granules
z von Willebrand z Coagulation Factor V z Adenosine diphosphate(ADP)
Factor z Platelet Factor 4
z Fibrinogen z Growth factors: z Adenosine triphosphate(ATP)
z Thrombospondin Platelet-derived
z Fibronectin Growth Factor z Ca++ ions

z Serotonin
And More!

PLATELET ACTIVATION
Vesicles (micro-
ADP,Ca particles)
Receptors:

IIb-IIIa
Agonists
Ib-IX

Subendothel-
ial matrix
Ia-IIa
PRIMARY HEMOSTASIS
Vessel

PLATELET MEMBRANE
GLYCOPROTEIN RECEPTORS

Receptor Ligand Functional Amino


activity acid
sequence
INTEGRIN 21 Collagen Adhesion DGEA
(gp Ia/IIa)
51 Fibronectin Adhesion RGD
(gpIc/IIa)
IIb3 Fibrinogen Aggregation KQAGDV
(gpIIb/IIIa) vWF RGD
NON gp Ib vWF Adhesion Not short
INTEGRIN sequence

KQAGDV: Lys-Gln-Ala-Gly-D Asp-Val


RGD: Arg-Gly-D Asp

3
GENERAL MECHANISM OF SECOND MESSENGERS (I):PROSTAGLANDIN
PLATELET ACTIVATION PATHWAY IN PLATELETS
Thrombin
Epinephrine
TxA2, ADP, etc Membrane Phospholipids
Phospholipase A2
Agonist
Arachidonic Acid
Cyclo-oxygenase Aspirin Block
Receptor G
Effectors (Phospholipases A & C) Cyclic Endoperoxides
A
A
G
G Second Messengers
G Proteins C
TxA2 Thromboxane A2
DAG
IP3
Ca++

SECOND MESSENGERS (II):PHOSPHOINOSITIDE


HYDROLYSIS PATHWAY IN PLATELETS PRIMARY PLATELET PLUG

Membrane
Phosphoinositide
Phospholipase C

Diacylglycerol (DAG) Inosine triphosphate (IP3)


membrane cytosol
Protein Kinases Dense Tubular System

Fibrinogen receptor Ca++ mobilization


expression
Granule secretion Microparticles

CLINICAL ASSESSMENT OF CLINICAL ASSESSMENT OF


PLATELET DISORDERS PLATELET DISORDERS

z History z Physical Findings


Bleeding, esp.dental or other surgery, Skin--petechiae, purpura, ecchymosis
pregnancy Oral bleeding
Drugs Epistaxis
Family history Gastrointestinal, genitourinary
bleeding

HALLMARK: Mucocutaneous bleeding

4
a

Large ecchymotic area over the thigh following minor trauma CLINICAL ASSESSMENT OF
PLATELET DISORDERS

z Laboratory Evaluation
Complete blood count & platelet count
Examine blood smear
Bleeding time (Closure time)
Platelet aggregation studies
*ADP *Arachidonic acid
*Collagen *Ristocetin
Lazarchick, J. ASH Image Bank 2001;2001:100177

Copyright 2001 American Society of Hematology. Copyright restrictions may apply.

Closure Time: An automated platelet function analyzer uses whole blood


flow through a capillary device to simulate primary hemostasis

Maslak, P. ASH Image Bank 2005;2005:101281

Copyright 2005 American Society of Hematology. Copyright restrictions may apply.

5
Platelet Aggregometry Platelet Aggregometry

6
PLATELET DEV/FXN Joseph E. Kiss, MD
Hematology 2009

Platelet Development and Function

Joseph E. Kiss, MD
PLATELET DEV/FXN Joseph E. Kiss, MD
Hematology 2009

Platelet Development and Function

Lecture Objectives

1. The student should be able to describe the structure of platelets


and related platelet function.
2. The student should understand the production of platelets from
megakaryocytes and the unique role of thrombopoietin in the
regulation of platelet levels.
3. The student should comprehend the interplay of platelets, plasma
cofactors, and the blood vessel wall in the physiology of primary
hemostasis.
4. The student should describe the tests used to evaluate platelet
function in the clinical laboratory.

Platelet Development and Function


Abnormalities of platelet number or function are very common in clinical medicine. Likewise,
physicians often prescribe antiplatelet medications in order to block involvement of platelets in
pathological disease processes. This lecture provides the framework for understanding normal
and abnormal platelet function. The first part will acquaint you with the formation of platelets
and the regulation of platelet production. Next, we will examine the physiology of platelet
activation and tests that are used to evaluate platelet function. This is followed by a lecture on
qualitative platelet abnormalities and quantitative platelet disorders. The text is meant to be
used in conjunction with the slide materials.

Platelet Development and Structure

Platelets are formed in the bone marrow by budding from megakaryocytes and circulate as 2-4
disks with a life span of 8-12 days. A number of different cytokines are involved, including
stem cell factor (SCF), IL3, IL6, IL 11, and thrombopoietin (TPO). TPO has been identified as
the most important lineage-specific growth and differentiation factor regulating platelet
production. The average concentration of platelets in the blood is ~250,000 per l with a range
of between 150,000 and 400,000 per l. Approximately one-third of the total platelet mass
temporarily resides in the spleen.

2
PLATELET DEV/FXN Joseph E. Kiss, MD
Hematology 2009

Ultrastructural studies of platelets reveal a cell membrane surface with multiple invaginations,
forming an open canalicular system. A cytoskeleton, composed of actin-like proteins and
microtubules, helps maintain the discoid shape of the circulating platelet. Platelets are filled with
secretory granules. Three different compartments are seen: alpha granules, which contain
high molecular weight proteins, dense granules containing low molecular weight substances,
and lysosomes containing acid hydrolases. The granule contents are actively secreted by
platelets during their activation, a process that is known as the "release reaction". Of note, von
Willebrand's factor, fibrinogen, thrombospondin, and fibronectin act as adhesive ligands
which bind platelets to each other and to the subendothelial matrix. Coagulation Factor V
binds to the platelet surface following its release wherein it becomes a receptor for activated
Factor X, which, along with calcium ions forms the prothrombinase complex (critical for
amplification of the coagulation pathway). Platelet Factor 4 (PF4) is also released. PF4 is an
anti-heparin that binds to heparin-like polysaccharides coating endothelial cells that line the
inner aspect of blood vessels. Alpha granules contain several types of growth factors. Platelet
derived growth factor (PDGF) is a fibroblast/smooth muscle mitogen that functions
physiologically in wound healing, and pathologically in atherosclerosis. Adenine nucleotides,
calcium and serotonin are released from dense bodies. These substances participate in the
recruitment of additional platelets into the hemostatic plug.

Physiology of Platelet Activation

From a teleologic standpoint, platelets are a critical component of host defense, having evolved
as a front line mechanism to keep blood within the vascular system after vascular injury.
Platelets interact with the vessel wall, a process known as adhesion. They also interact in
complex fashion with each other ("aggregation"), and with the coagulation mechanism itself
("procoagulant activity").

Circulating platelets rapidly adhere to elements of the sub-endothelial matrix after a blood vessel
is injured and the normal endothelial cell barrier is disrupted. Two receptors located on the
surface of the platelet mediate adhesion: Glycoprotein (gp) Ib-IX, which binds von Willebrand's
factor, and gp Ia-IIa, which binds collagen. Once activation has occurred, another platelet
receptor, gp IIb-IIIa, undergoes a conformational change which allows it to bind the large
molecular weight ligands, which are present in plasma as well as being released from platelets.
The most important of these is fibrinogen, which can bind to other activated platelets in a
process known as aggregation. A short amino acid sequence (motif) responsible for binding to
the receptor is present, eg, fibrinogen has two Arg-Gly-D Asp sequences on each end of the
molecule that mediate binding. Whether by contact of platelets with elements of the sub-
endothelial matrix, or in response to stimulation by soluble agonists such as ADP, thrombin, or
collagen, the platelet becomes transformed from a compact disk to a sphere with pseudopods.
There is activation of phospholipases, which activate protein kinases (leading to protein
phosphorylation, release of intracellular Ca++, and secretion of platelet granules), and release
of arachidonic acid from plasma membrane phospholipids. This results in the formation of
thromboxane A2, which amplifies platelet activation reactions.

3
PLATELET DEV/FXN Joseph E. Kiss, MD
Hematology 2009

The release of platelet granule contents mediates and reinforces platelet aggregation and
adhesion as part of a positive feed-back loop which recruits additional platelets into the growing
platelet plug. Finally, membrane phospholipids are rearranged and tiny vesicles are shed.
These vesicles, or micro-particles, provide the phospholipid which comprises the prothrombin
activator complex consisting of phospholipid, calcium, factor Va and factor Xa. This
procoagulant function accelerates the conversion of prothrombin to thrombin to complete the
coagulation sequence.

In recent years it has become apparent that platelets may also recruit leukocytes to sites of
vessel wall injury by exposing a glycoprotein, P-selectin, which is redistributed to the platelet
surface when alpha granules fuse with the cell membrane. Leukocytes, which express the
mucin PSGL-1 (P-selectin Glycoprotein Ligand-1) ligand for P-selectin, may be seen
tethering and rolling on platelets and endothelium using this cytoadhesive mechanism.
Leukocytes are an important source of Tissue Factor for promoting thrombus formation.
Likewise, platelets are recruited and participate in inflammatory reactions with leukocytes.

In summary, platelet activation is a complex process in which adhesion, aggregation, and


secretion act to form a platelet plug capable of rapidly stopping blood loss from an injured
vessel. The net result of platelet procoagulant activity is the formation of a fibrin meshwork
(resulting from the action of thrombin on fibrinogen), which stabilizes the plug. The importance
of each of these functions is illustrated by the fact that clinical disorders exist in which one or
another of these steps is defective.

Evaluation of Platelet Function

The clinical evaluation of platelet disorders consists of the history, physical findings, and
laboratory evaluation. As in other disorders of coagulation, experiences with dental procedures,
other surgery, and pregnancy can give useful clues to a bleeding tendency. A thorough drug
history, as well as a family history are indispensable. Physical findings of patients with platelet
defects are characterized by evidence of mucocutaneous bleeding. This is in contrast to
patients with other types of coagulation disorders, in which more "deep seated" bleeding occurs.
Skin findings include petechiae, which are pinpoint or slightly larger hemorrhages primarily
occurring in the dependent portions of the body, ecchymoses (bruises), or a combination of the
two, i.e., purpura. Oral bleeding can also be seen, as well as epistaxis. Laboratory evaluation
begins with a complete blood count and platelet count, since platelet disorders frequently are
accompanied by other hematologic findings. The blood smear should be examined for clues
which point to a platelet destructive process or a process which can result in underproduction of
blood platelets.

A bleeding time and platelet aggregation studies are useful only in patients whose platelet count
is not decreased, in order to evaluate for qualitative platelet abnormalities. The bleeding time
is a measure of platelet-vessel wall interaction. It primarily reflects the capability of platelets to
adhere and to aggregate. Due to practical difficulties, the in vivo bleeding time test has been

4
PLATELET DEV/FXN Joseph E. Kiss, MD
Hematology 2009

largely replaced with an in vitro closure time test. The basic principle of the assay is to
determine the time for epinephrine or ADP to activate platelets and plug a small diameter
collagen-coated channel (simulating a blood vessel). Platelet aggregation studies may also be
useful to find the underlying platelet defect. Several agonists which induce platelet aggregation
are added to a cuvette in which platelets are stirred. A spectrophotometer measures the
transmitted light through the platelet rich plasma. As platelets aggregate, more light is detected
by the sensor, which creates an inflection in the paper tracing. Both general and specific
platelet defects can be studied with this approach.

5
Phagocytes
Development and
Granulocytes Monocytes
Function of Granulocytes
Eosinophils Basophils Neutrophils
Gurkamal Chatta,
Chatta, MD
Associate Professor of Medicine
University of Pittsburgh Cancer Institute
chattags@upmc.edu

Outline Granulocytes
Review of production of neutrophils Neutrophils
3-5 lobed nucleus
Review granulocyte pools Cytoplasm contains
Discuss leukocyte migration granules that degrade
pathogens (lysozyme,
Review the function of neutrophils cathepsin), generate
oxidants (peroxidase)
Short mention about eosinophils and basophils and cause tissue damage
(elastase, collagenase)

Granulocytes Granulocytes
Eosinophils Basophils
Bilobed granulocyte with Segmented nucleus
azurophilic granules Cytoplasm filled with
Able to recirculate large, dark granules
Longer lived than Granules rich in heparin
neutrophils and histamine
Possess same capacity for
Surface rich in receptors
phagocytosis and oxidative
for IgE
metabolism as neutrophils
Precursor cell for tissue
mast cells

1
Monocyte Leukocyte Numbers
Mononucleated cell with Total Leukocytes 4.0-11.0 X 109/L
4.0-
horse-
horse-shoe nucleus Neutrophils 2.5-7.7 X 109/L
2.5-
Cytoplasm contains Lymphocytes 1.5-3.5 X 109/L
1.5-
granules similar as Monocytes 0.2-0.8 X 109/L
0.2-
neutrophils Eosinophils 0.04-0.4 X 109/L
0.04-
Cytoplasm often Basophils 0.001-0.1 X 109/L
0.001-
vacuolated
Percentages: PMNs ~70%, Lym 20%, Mono 5-
5-10%

Neutrophil Production

Initial cytokines/growth
factors are multi-
multi-lineage
Lineage-
Lineage-specific growth
factors lead to
development of
neutrophils, eosinophils
Metcalf D. IMMUNITY; June 2007
or monocytes

Rossi et al. Murine Hematopoiesis

Clinical Caveat Pools of Neutrophils


G-CSF GM-
GM-CSF
Short acting form One form
Filgrastim (Neupogen
(Neupogen)) Sarmograstim (Leukine)
Leukine)
Long-
Long-acting form Uses:
Uses:
Pegfilgrastim (Neulasta
(Neulasta)) Chemo-
Chemo-induced
Uses:
Uses: neutropenia (AML et al)
Chemo-
Chemo-induced Stem cell mobilization
neutropenia (AML et al) Bone marrow failure or Mitotic Pool Post-mitotic
Pool
Stem cell mobilization engraftment delay
Chronic neutropenia

2
Mitotic Pool - Myeloblast Mitotic Pool - Promyelocyte

Large, round nuclei Larger than myeloblast


with fine, granular Similar nuclear features
chromatin Cardinal feature is presence
of primary granules
Prominent nucleolus Contain microbicidal
Scant, agranular proteins, eg,
cytoplasm myeloperoxidase, lysozyme,
defensins

Mitotic Pool - Myelocyte Post-


Post-Mitotic Pool - Metamyelocyte
Last progenitor that
undergoes mitosis
Nuclear chromatin begins to Cytoplasm remains
condense unchanged
Nucleoli are sparse or Nucleus begin to indent
absent
Chromatin is coarse and
Secondary granules form
(lactoferrin, B12 binding
clumped
proteins, NADPH oxidase,
2 integrins/CR3

Transit through the mitotic pool takes 7 days

Post-Mitotic Pool - Bands Post-Mitotic Pool - PMNs


Final product of
maturation
Similar to
metamyelocytes but 2-5 nuclear
smaller lobes/segments
Horseshoe nucleus and >5 segments suggestive
segmentation begins of megaloblastic anemia
due to deficiency of B12
or folate

Transit through the post-mitotic pool takes 7 days

3
Clinical Caveat Summary of Changes
Pelger-
Pelger-Huet Anomaly
Bi-
Bi-lobed neutrophil
May be congenital
Markers used to
which is benign define AML in
Acquired is associated presentation of acute
with acute myelogenous leukemias
leukemia or
myelodysplastic
syndromes (MDS)

Leukocyte Migration Pools of Neutrophils


Bone Marrow Component
Stem cell compartment, mitotic and post-
post-
mitotic pools
Account for 90% of neutrophils
Increased mobilization of cells from this
compartment following cytokines (IL-
(IL-1, TNF-
TNF-
, G-
G-CSF), complement activation, endotoxin,
administration of glucocorticoids

Pools of Neutrophils Circulating Pool


Circulating Pool Marginating Pool Circulating Pool
Neutrophils circulate for 6-
6-7 hours and then migrate Cells that are lightly 50% of cells
into tissues where they live another 1-
1-4 days adherent to vascular Circulate freely and
As opposed to macrophages and eosinophils, endothelium constitute the true
neutrophils do not re-
re-circulate 50% of neutrophils circulating pool
Senescent neutrophils in circulation are removed by Can shift to circulating
the spleen/liver pool with stress,
Neutrophils in tissue die via apoptosis and are epinephrine
ingested by tissue macrophages

4
Neutrophil Pools

Neutrophil Migration - Adhesion Cascade Adhesion Cascade


Rolling/Tethering Firm Adhesion
Mediated by Mediated by
interactions of selectins interactions of leukocyte
and their ligands integrins and their
L-selectin ligand following
Initially P-selectin leukocyte activation
(cytokines, chemokines,
E-selectin with EC others)
activation
LFA-1:ICAM-1 or -2
Mac-1: ICAM-1

Adhesion Cascade Summary of Events

Diapedesis/Migration
Mediated by the
interaction of Integrins
and PECAM-1

5
Neutrophil Function Neutrophil Function

Chemotaxis Phagocytosis
Optimized by opsonization of bacteria
Directed migration (vs chemokinesis which is increased,
Neutrophils have an array of Fc receptors that
undirected motion)
bind Ig
Caused by local release of chemokines (CXC family), by
CD16 and CD32 normally expressed
complement components (C3a and C5a), or the interaction of
CD64 induced on PMN by IFN
adhesion molecules with ligands on the damaged tissue
All bind IgG1 and IgG3

Neutrophil Function
Phagocytosis
Also optimized by complement coating of
bacteria
Neutrophils have complement receptors (CR1
and CR3) that bind C3b and C3bi respectively
CR3 also a cell adhesion molecule (Mac-
(Mac-1)

Neutrophil Function Neutrophil Function


Killing Killing
With activation of the neutrophil a burst of Myeloperoxidase generates hypochlorous acid
oxygen consumption and activation of the that halogenates organism
hexose monophosphate shunt occurs Defensins - cysteine-
cysteine-rich peptides with broad
anti-
anti-microbial activity
Leads to assembly of a NADPH oxidase that
catalyzes the generation of superoxide radicals Lactoferrin bacteriostatic by depriving the
organism of iron
and H2O2
Nitric oxide
Lysosomal enzymes that lower pH

6
Eosinophil function Basophil function

Play a major role in


immediate-
immediate-type
hypersensitivity reactions
Granule release results in
urticaria, rhinitis, asthma,
anaphylaxis

Learning Objectives

Understand the quantitative disorders of


Disorders of granulocytes (neutrophils, eosinophils,
Granulocytes basophils)

Understand the qualitative disorders of


neutrophils

Neutrophils Neutropenia
With low neutrophil count patients present
with aphthous ulcers, gingivitis or periodontal
disease.
Quantitative Disorders
Neutropenia Qualitative Disorders Skin, respiratory tract, and bone infections are
Neutrophilia
common
Rarely overwhelming sepsis or meningitis
Lymphadenopathy is common.

Hereditary Acquired Hereditary Acquired

7
Congenital Causes Congenital Causes
Cyclic Neutropenia
Benign/Familial Neutropenia
Due to a mutation in elastase gene Organisms:
Organisms:
Normal values for total WBC and ANC variety including C perfringens
are lower in persons with ancestors from Clinical:
Clinical: aphthous ulcers, gingivitis,
Africa or the Middle East stomatitis, cellulitis
Due to increased marginating pool Recurrent, severe neutropenia lasting 3-
3-6 days
No history of infections every 21-
21-28 days.

Congenital Causes Acquired Neutropenia


Chronic Idiopathic Neutropenia Three Mechanisms
Absence of cyclic periodicity Decreased production
Non-
Non-racial/ethnic Drug induced
No offending medication Hematologic disorders
Marrow infiltration (myelophthisis)
No associated autoimmune disease
Nutritional deficiencies (B12 or folate)
More common in females
Increase destruction immune or hapten mechanism
Thought to be due to decreased myelopoiesis
Peripheral pooling sepsis, biomaterials
Treat with G-
G-CSF if recurrent infections marginating pool; splenic sequestration

Drug-induced Neutropenia Drug-induced Neutropenia


Anti-
Anti-inflammatory (phenylbutazone) Differential Diagnosis of Acquired Neutropenia
Antibiotics (chloramphenicol, sulfa, imipenem) 1. Drugs
Anti-
Anti-convulsants (phenytoin, carbamazepine)
2. Drugs
Anti-
Anti-thyroid (carbimazole)
Hypoglycemics (tolbutamine
(tolbutamine)) 3. Drugs
Phenothiazines (chlorpromazine) 4. Other things
Anti-
Anti-psychotics (clozapine)
Misc: gold, penicillamine)

8
Acquired Causes of Neutropenia Neutrophilia
Most common causes is iatrogenic Increased production
Glucocorticoids
Decreased production due to drugs Infection
Chemotherapy MPS

Immunosuppressants Increased release from the marrow


Glucocorticoids
Antibiotics
Infection/inflammation
Increased destruction Defective margination
Autoimmune mechanism Drugs (epinephrine)
Stress
Drugs as hapten
LAD I and II

Neutrophilia Clinical Caveat


Infections Leukemoid Reaction
Inflammation or tissue necrosis Persistent elevation in neutrophil counts
Neoplasms (MPD) Often ranges to 30,000-
30,000-50,000
Concern is leukemia
Acute hemorrhage/hemolysis
Reflects infection/inflammation
Drugs (steroids, lithium) Polyclonal expansion of normal granulocyte
Asplenia precursors
Leukocyte alkaline phosphatase (LAP) score is
elevated

Neutrophils Qualitative Disorders/Adhesion


Congenital
Leukocyte adhesion deficiency I (LAD
(LAD I)
I)
Quantitative Disorders
Mutation in the 2 subunit of leukocyte
Neutropenia
Neutrophilia
Qualitative Disorders integrins
Organisms:
Organisms: gram neg or pos; fungus
Sites:
Sites: none characteristic

Hereditary Acquired Hereditary Acquired

9
Qualitative Disorders/Adhesion Qualitative Disorders/Adhesion
Congenital Acquired
LAD II Medications:
Lack of fucosylation of selectin ligands Aspirin
Organisms:
Organisms: gram neg or pos; fungus Colchicine

Sites:
Sites: none characteristic Glucocorticoids

Associated with growth and mental retardation

Qualitative Disorders/Chemotaxis Chediak-Higashi Syndrome


Congenital: Chediak-
Chediak-Higashi syndrome
Severe autosomal recessive anomaly
Mutation in LYST gene that is involved in All cells with lysosomes
protein transport have large cytoplasmic
granules
Results in lack of fusion of lysosome with Affects nerves and hair as
phagosome well
Organisms:
Organisms: staph and strep
Partial albinism, neuropathy

Qualitative Disorders/Chemotaxis Qualitative Disorders/Killing


Acquired Congenital
Medications: Chediak-
Chediak-Higashi
Colchicine Chronic Granulomatous Disease
Glucocorticoids Myeloperoxidase deficiency
NSAIDs Congenital causes of agammaglobulinemia or
IL-
IL-2 hypogammaglobulinemia
Congenital causes of complement deficiency

10
Qualitative Disorders/Killing Qualitative Disorders/Killing
CGD Myeloperoxidase Deficiency
Failure to generate Most common congenital abnormality of
NADPH oxidase leads
PMNs
to recurrent infections
Organisms: catalase + Autosomal recessive inheritance
Abscess of lung, liver, Defect in MPO gene on 17q
brain and bone Not usually associated with infections
(NADPH oxidase, lysozymes, etc)
Associated with disseminated candida in DM

Eosinophilia Hypereosinophilic Syndrome


Neoplasms (Hodgkin
(Hodgkins) Persistent elevation of eosinophils (50,000-
(50,000-
Allergic reactions/Asthma 100,000/ml)
Addison
ddisons Affects end organs (heart, lungs, kidneys, CNS,
GI tract)
Collagen vascular diseases
Some secondary to over-
over-production of IL-
IL-5
Parasites
Some also secondary to mutated PDGF receptor
with tyrosine kinase
Worms, wheezes and strange diseases
diseases

Monocytosis Basophilia
Chronic bacterial infections (TB, brucellosis) Seldom seen
Connective tissue diseases (SLE, temporal Associated with myeloproliferative syndromes
arteritis, RA) (P vera, CML)
Protozoan infections Reactive basophilia occurs in small pox and
Chronic neutropenia chicken pox)
Hematologic malignancies: (Hodgkins, AML)
MDS (chronic myelomonocytic leukemia,
CMML)

11
Granulocyte Development and Gurkamal Chatta, MD
Granulocyte Disorders Hematology 2009

GRANULOCYTE DEVELOPMENT & FUNCTION


Learning Objectives
1. Understand the various stages of granulocyte production and the growth factors
that affect their development.

2. Understand the various pools of granulocytes and the redistribution of


granulocytes within these pools.

3. Understand the function of neutrophils.

4. Understand the function of eosinophils and causes of eosinophilia.

5. Understand the function of basophils.

1. Production: Myeloid progeny develop from the common hematopoietic stem cell
which gives rise to the first myeloid stem cell: colony forming unit granulocyte,
erythrocyte, monocyte, megakaryocyte (CFU-GEMM), that is not morphologically
identifiable but expresses CD34 and HLA-DR on the cell surface. The major growth
factors at this point in granulocyte development affect multiple lineages [stem cell factor,
IL-3, IL-6, flt3 ligand, GM-CSF] but lead to the formation of CFU-GM. This progenitor
then separates into granulocyte or monocyte precursors under the influence of their
lineage specific growth factors, i.e., granulocyte (G)-CSF and monocyte (M)-CSF,
respectively.
The granulocyte series enters into six stages of development based on cell size,
nuclear volume, chromatin density, prominence of nucleoli, granule number and
cytoplasmic staining. In general, cell maturation involves decreasing cell volume,
increasing chromatin condensation, loss of nuclear nucleoli, appearance of cytoplasmic
granules and changes in cytoplasmic basophilia. In addition to these morphologic
changes, precursors are also characterized as being in a mitotic/proliferating pool or a
post-mitotic/maturation pool. Each pool lasts ~7 days so it takes 10-14 days to
generate a mature neutrophil.

A. Proliferating/Mitotic Pool:

Myeloblast: This cell is the youngest myeloid precursor that is identifiable by


light microscopy. It is 15-20 microns in diameter with a large, round nucleus
that has finely granular chromatin. Nucleoli are present. Cytoplasmic
granules are absent and cytoplasm is scanty.
Granulocyte Development and Gurkamal Chatta, MD
Granulocyte Disorders Hematology 2009
Promyelocyte: The promyelocyte is larger than a myeloblast but has similar
nuclear features (fine chromatin, nucleoli). The cardinal feature of a
promyelocyte is the presence of many violet (azurophilic) primary granules.
The latter contain microbicidal proteins [myeloperoxidase, lysozyme,
bactericidal-permeability inducing factor, defensins] and acid hydrolases.
Defensins are cysteine-rich polypeptides with broad anti-microbial activity
against bacteria, fungi and enveloped viruses.

Myelocyte: The myelocyte is the last myeloid precursor that is capable of


mitosis. Three cell divisions occur at this stage with each division resulting in
smaller cells. During this stage of development, the chromatin becomes more
coarse and is clumped. Nucleoli are sparse or absent. Secondary granules
are formed at this stage the contents of which include lactoferrin (competes
for iron to prevent use by the bacteria), B12-binding protein (the reason that
the B12 binding capacity is elevated in P vera and other myeloproliferative
syndromes), membrane components of the NADPH oxidase that is required
for H2O2 production, and receptors for chemoattractants and adhesion-
promoting factors (2 integrins).

B. Post-mitotic/Maturation Pool:

Metamyelocyte: During the final phases of maturation the cytoplasm


resembles that of a mature neutrophils. However, nuclear changes continue to
occur. The metamyelocytes have a nucleus that is indented with chromatin
that is coarse and clumped.

Band forms: Bands are smaller in size than the metamyelocytes and have a
non-segmented and horseshoe-shaped nucleus. Bands are also called the stab
forms or juvenile forms.

Polymorphonuclear neutrophils (PMN): The final product of development


is a mature PMN or poly with 2-5 nuclear segments. Excessive
segmentation (>5) is seen with B12 and folate deficiency. Conversely,
hyposegmentation can occur in the Pelger-Huet anomaly that results in bi-
lobed nucleus. This anomaly can be either congenital (benign) or acquired
(AML or myelodysplastic syndrome) where pseudo-Pelger cells can be
observed.

2. Circulation: There are two major pools of myeloid cells:

A. Bone marrow component. This component consists of the stem cell


compartment and the mitotic and maturation pools. Ninety per cent of neutrophils are
within the bone marrow compartment with only 2-3% of the cells circulating and the
remainder in tissues. Increased mobilization from the marrow of cells in the maturation
pool can occur following the generation of cytokines (IL-1, TNF-, G-CSF), complement
Granulocyte Development and Gurkamal Chatta, MD
Granulocyte Disorders Hematology 2009
activation (a leukocyte-mobilizing factor derived from the third component of
complement (C3e) or C5a), endotoxin or the administration of glucocorticoids.
Presence of metamyelocytes or myelocytes in the peripheral circulation is termed
a left shift of the neutrophil differential. This is often seen in sepsis where additional
granulocyte abnormalities, such as toxic granulations or Dohle bodies may be seen.
Toxic granulations are coarse, purple cytoplasmic granules seen in PMNs and are thought
to be prominent primary granules. Dohle bodies are pale blue staining cytoplasmic
inclusions in PMNs and are remnants of endoplasmic reticulum. They are usually seen in
sepsis with toxic granulations.

B. Circulating component: Neutrophils circulate for 6-7 hours and then migrate
into tissues where they survive another 1-4 days. As opposed to macrophages and
eosinophils, PMNs do not return to the circulation once they have entered the tissues.
Circulating, senescent PMNs are removed by macrophages in liver or spleen.
Neutrophils within tissue undergo apoptosis and are eventually engulfed by tissue
macrophages.

The circulating pool itself has two dynamic compartments each of which contains
half of the circulating pool.: Some of the cells attach lightly to the vascular endothelium
and constituent a marginating pool while the other 50% circulate freely and make up the
true circulating pool.. However, the populations are under dynamic interchange and
stress or administration of epinephrine can shift the marginating pool into circulation.

3. Circulating PMN interactions with endothelial cells:

Recruitment of neutrophils at sites of infection/inflammation requires several steps


that have been described as an adhesion cascade. To accomplish these steps, molecules
expressed on PMNs must interact with counter-receptors on endothelial cells. A series of
molecules have been found that contribute to these interactions and these proteins have
been grouped into families.

A. Selectins: Selectins are characterized by an N-terminal lectin domain that binds


carbohydrate moieties expressed by their counter-receptors. Leukocyte (L-) selectin is
expressed on PMNs and following neutrophil activation, L-selectin is shed.
Granulocyte Development and Gurkamal Chatta, MD
Granulocyte Disorders Hematology 2009
There are two selectins expressed on endothelial cells. Endothelial (E-) selectin
expression is induced on endothelial cells following activation by cytokines (TNF-, IL-
1) or LPS. In contrast, Platelet (P-) selectin, which is also expressed on endothelial cells,
is synthesized and stored in granules within platelets or endothelial cells. With activation
of these cells (thrombin, histamine), P-selectin is rapidly expressed on the cell surface.
B. Selectin counter-receptors: Ligands for selectins are structures modified by
sialylated and fucosylated sugars. These ligands include Platelet Selectin Glycoprotein
Ligand -1 (PSGL-1) which is expressed on PMN and monocytes. The importance of
carbohydrate modification in the generation of ligands for selectins has been
demonstrated in patients with Leukocyte Adhesion Deficiency II (LAD II).
C. Integrins: These are proteins that are grouped in families by a constant beta
chain (2) and variable alpha chains. The 2 family of integrins are expressed solely on
leukocytes and are comprised of four members, two of which are involved in PMN
adhesion and migration. Lymphocyte Function Associated protein-1 (LFA-1) is created
by the association of one chain (CD11a) with CD18, the common chain of this family
of integrins, while the association of a second chain (CD11b) with CD18 creates Mac-
1. LFA-1 is expressed on all leukocytes while Mac-1 expression largely is restricted to
PMN and monocytes. The importance of the 2 family of integrins is reflected in LAD-I.
Individuals with this disorder cannot generate functional 2 integrins due to abnormalities
of the common chain. Patients with LAD-I have an almost complete block in PMN
adhesion and migration and are subject to severe, life-threatening infections.

D. Immunoglobulin-like family: Four adhesion molecules expressed on


endothelial cells belong to a family of molecules that are structurally similar to
immunoglobulins. Two of these molecules [Intercellular Adhesion Molecules (ICAMs)-
1, -2 are ligands for integrins expressed in PMNs. ICAM-1 and ICAM-2 are
constitutively expressed on vascular endothelium. Surface expression of ICAM-1,
however, is up-regulated by endothelial activation (TNF-, IL-1, LPS). Both of these
endothelial adhesion molecules are ligands for LFA-1 while Mac-1 binds only to ICAM-
1. A third endothelial adhesion molecule within this family (Platelet Endothelial
Adhesion Molecule (PECAM)-1) is the ligand for PECAM-1 expressed on PMNs.
PECAM-1 is constitutively expressed and is not subject to up-regulation. Following
endothelial activation, however, it migrates to inter-endothelial borders to facilitate
transendothelial migration.
Granulocyte Development and Gurkamal Chatta, MD
Granulocyte Disorders Hematology 2009

3. Adhesion Cascade:
Leukocytes normally circulate within the vasculature with minimal interaction
with endothelial cells. Following local injury, factors that activate PMNs (complement
fragments, PAF, leukotrienes, chemokines) and endothelial cells (thrombin, histamine,
cytokines) are generated altering adhesive interactions between PMNs and endothelium.
Neutrophils initially roll along the endothelium due to interactions of selectins and their
ligands. This phenomenon serves to juxtapose PMNs near the endothelium where they
can be activated by factors within the region. A classic example of this are chemokines.
Chemokines bind to glycosaminoglycans (e.g., heparan sulfate) on the luminal surface of
the endothelial at the site providing a regional localization of chemokines for PMN
activation. Following the binding of chemokines with their receptors, PMN integrins
undergo a confirmational change that increases the avidity of the integrin for its
endothelial ligand (i.e., an on-off switch). The interaction of PMN integrins and their
ligands provides firm adhesion that prevents further rolling and maintains the PMN at the
site despite continued shear forces created by the circulation. The final step of PMN
migration involves sequential release and adhesion of integrins as the PMN crawls
(diapedeses) to the endothelial junction where transendothelial migration occurs. This
process is facilitated by the interaction of PECAM-1.

4. Neutrophil function:

The normal functions of PMNs can be divided into three phases:

a) chemotaxis (directed migration): The PMN is attracted to the bacteria or the site
of inflammation by chemotactic substances (often CXC chemokines such as IL-8)
released by the damages tissues, by complement components (C3a, C5a) and also by the
interaction of the adhesion molecules with ligands on the damaged tissues.

b) phagocytosis: Foreign materials or damaged cells are phagocytosed. Recognition


of a foreign particle is aided by opsonization with immunoglobulin or complement
because PMN have Fc and complement receptors. The major immunoglobulin opsonins
are IgG1 and IgG3 that bind to CD16 (FcRIII) and CD32 (FcRII) expressed on PMNs.
In addition, a third Fc receptor, CD64 (FcRI) is induced on PMN by -IFN. This is a
high affinity receptor for IgG1 and IgG3. Neutrophils are express complement receptors
Granulocyte Development and Gurkamal Chatta, MD
Granulocyte Disorders Hematology 2009
(CR) 1 and 3 on the surface. CR1 binds to C3b covalently bound to the bacteria during
complement activation. CR3 recognized a proteolytic breakdown product of C3b (C3bi).
CR3 is one of the PMN integrins involved in adhesion of endothelial cells also.

c) killing and digestion: With phagocytosis, a burst of oxygen consumption and


activation of the hexose-monophosphate shunt occurs. A membrane-associated NADPH
oxidase, consisting of membrane and cytosolic components, is assembled and catalyzes
the reduction of oxygen to superoxide anion that is then converted to H2O2 and other
toxic oxygen products. Hydrogen peroxide and chloride with myeloperoxidase generates
hypochlorous acid (bleach), hypochlorite, and chlorine. These products oxidize and
halogenate microorganisms. In addition to oxygen-dependent killing, strongly cationic
proteins (defensins) and probably nitric oxide also participate in microbial killing. An
additional factor in PMN granules, lactoferrin, is bacteriostatic by depriving bacteria of
iron. Finally, the non-oxidative microbicidal mechanisms also involves a fall in pH
within the phagocytic vacuoles into which lysosomal enzymes are released.

5. Eosinophils: Share similar morphology and many lysosomal constituents with


PMNs. They have phagocytic capacity and oxidative metabolism. Eosinophils are much
longer lived than PMNs and have the ability to recirculate. During most infections
eosinophils are not important. They play a major role in parasitic infections. They are
also important in hypersensitivity reactions such as asthma and cutaneous allergic
reactions.

A. Eosinophilic Granule Content

Primary Granules:
Charcot-Leyden crystal - cleaves fatty acids from lysophospholipids)

Secondary Granules:
Major basic protein: binds to acidic lipids and disrupts membranes
Eosinophic cationic protein: forms transmembrane pores

B. Eosinophilia is defined as presence of >500 eosinophils/microliter of blood.


Causes include:
allergic reactions
collagen vascular diseases
malignancies, Jobs syndrome (hyper IgE)
CGD

The most dramatic hypereosinophilic syndromes is idiopathic hypereosinophilic


syndrome where absolute eosinophilia may range from 50,000-100,000/microliter. This
degree of persistent eosinophilia may results in organ damage affecting the heart, CNS,
kidneys, lungs, GI tract and skin. This disorder may be due to clonal proliferation of T
lymphocytes secreting IL-5, an eosinophil growth factor. Some cases of idiopathic
Granulocyte Development and Gurkamal Chatta, MD
Granulocyte Disorders Hematology 2009
hypereosinophilia are also caused by a mutation within the PDGF receptor causing
dysregulation of a tyrosine kinase. Patients with the latter mutation may response to
imitanib (Gleevec).

6. Basophils: least common of all of the granulocyte subsets.

A. Basophil Granule Content: heparin and histamine

B. Basophilia: usual cause is a myeloproliferative syndrome such as CML or


Polycythemia vera.
Granulocyte Development and Gurkamal Chatta, MD
Granulocyte Disorders Hematology 2009

GRANULOCYTE DISORDERS
Learning Objectives

1. Understand the algorithm in determining disorders of neutrophils.

2. Know the mechanisms of neutrophilia and neutropenia.

3. Know the mechanisms of qualitative disorders of neutrophils.

Disorders of neutrophils: In general, disorders of PMNs, due either to quantitative


decrease in normal PMNs or to a qualitative abnormality of function, present with similar
clinical manifestations. Patients present with aphthous ulcers of the mucous membranes,
gingivitis, or periodontal disease. Skin, ear, upper and lower respiratory tract and bone
infections are common while sepsis and meningitis are rare. Lymphadenopathy and
hepatosplenomegaly are common.

A. Quantitative Problems with Neutrophils

1. Neutropenia: susceptibility to infections occurs when the absolute neutrophil


count (ANC) falls below 1,000. ANC is calculated by multiplying the sum of the
per cent [PMN + bands] by the total white blood cell count. For example, if the
white cell differential is reported to have 80% PMNs and 10% bands and the total
white cell count is 1.5 X 109/L, the ANC would be 1,350 [90% X 1.5]. When the
ANC count is <500, control of endogenous bacterial flora at barrier sites (e.g.,
mouth, gut, lungs) is impaired. When the ANC is <200 no inflammation arises.
Neutropenia can be caused by any of three mechanisms (or combinations of these
mechanisms.

Decreased production: most commonly drug-induced. Medications that alter


PMN production include chemotherapy (alkylating agents, anti-metabolites) and
non-cytotoxic drugs [antibiotics, phenothiazines, anticonvulsants, anti-psychotics,
anti-inflammatory drugs, anti-thyroid]. Hematologic diseases such as idiopathic
neutropenia, cyclic neutropenia, Chediak-Higashi syndrome, aplastic anemia.
Marrow infiltration by fibrosis or tumors (myelophthisis) also is be a cause as
are nutritional deficiencies (B12 or folate) and infections (TB, typhoid).

Peripheral destruction: Anti-neutrophil antibodies splenic/lung trapping of


PMNs may occur in association with autoimmune diseases like RA and SLE. In
addition, Feltys syndrome (RA, splenomegaly, neutropenia and leg ulcers) is a
cause where the splenomegaly leads to sequestration of PMNs as well as possible
antibody-mediated destruction. Drugs (e.g., methyl dopa or phenylbutazone)
may act as haptens to induce antibody formation.
Granulocyte Development and Gurkamal Chatta, MD
Granulocyte Disorders Hematology 2009
Peripheral Pooling: Increased in the size of the marginating pool may arise
during overwhelming sepsis. Contact with biomaterials (eg, hemodialysis,
cardiopulmonary bypass) also causes neutropenia.

2. Congenital causes of neutropenia

Severe congenital neutropenia (Kostmann syndrome):


defect: unknown
pathogenetic organisms: Staphylococcus aureus, Burkholderia aeruginosa
sites affected : cellulitis, perirectal abscess, stomatitis, meningitis
clinical presentation: developmental arrest of myeloid cells at
promyelocyte stage; increased risk of AML and MDS

Cyclic neutropenia:
defect: due to mutations in elastase gene that affects the catalytic site of
the enzyme. This can lead to failure of inhibitors to bind to and inactivate
the enzyme. With failure to inhibit the endogenous enzyme, the myeloid
precursors are destroyed.
pathogenetic organisms: variety of bacterial including C. perfringens
sites affected: aphthous ulcers, gingivitis, stomatitis, cellulitis
clinical presentation: recurrent, severe neutropenia that lasts for 3-6 days
of every 21-28 days cycle; death from overwhelming infection occurs in
~10%.

NB: many individuals from Africa and the Middle East have normal ranges that
are lower than the general population. This is of no clinical significance.

3. Acquired forms of neutropenia: Most common forms are iatrogenic


resulting from the use of cytotoxic or immunosuppressant therapies for treatment
of malignancies or control of autoimmune disorders. In general neutropenia is
caused by decreased production. Antibiotics: chloramphenicol, trimethoprim-
sulfamethoxazole, flucytosine, vidarabine and zidovadine. A second mechanism
is induction of an antibody response where the drug serves as a hapten and
sensitizes PMN or precursors to immune-mediated peripheral destruction.
Antibiotics associate with this mechanism include: sulfa, penicillins, and
cephalosporins Drug-induced neutropenia usually resolves in 7-10 days.
Autoimmune neutropenia, where the PMNs themselves serve as antigen target, is
another acquired neutropenia that results in increased destruction.

4. Neutrophilia: Three main mechanisms are involved:

increased production: glucocorticoids, infection, inflammation,


myeloproliferative syndromes (P vera, CML, ET)
increased release from marrow: glucocorticoids, infection, inflammation
Granulocyte Development and Gurkamal Chatta, MD
Granulocyte Disorders Hematology 2009
defective margination: drugs (epinephrine glucocorticoids, NSAID),
stress/exercise; LAD 1 (lack of 2 leukocyte integrins) and 2 (inability to
produced selectin ligands).

NB: Acute infection increases both production and marrow release. Leukemoid
reactions, defined as a persistent neutrophilia with counts >30,000-50,000, may occur
and the question of leukemia arise. These are polyclonal expansions of normal PMNs
and are not an acute leukemia

B. Qualitative Disorders of neutrophils

1. Congenital

Defects in adhesion/aggregation:

Leukocyte adhesion deficiency I (LAD I):


defect: mutation in 2 subunit of leukocyte integrins
pathogenetic organisms: gram negative bacteria; S. aureus,
candida and aspergillus
sites affected: no specific site is characteristic
clinical presentation: recurrent infections of the skin, soft tissues,
respiratory and gastrointestinal tracts; periodontal disease

LAD II:
defect: lack of fucosylation of adhesion proteins
pathogenetic organisms: gram negative bacteria; S. aureus,
candida and aspergillus
sites affected: no specific site is characteristic
clinical presentation: recurrent infections of the skin, soft tissues,
respiratory and gastrointestinal tracts; periodontal disease; growth
retardation and neurologic deficits.

2. Acquired: drug induced [aspirin, colchicines, glucocorticoids]

Defects in chemokinesis/chemotaxis:

Congenital:

Chediak-Higashi syndrome: severe, autosomal recessive anomaly.


defect: mutation in LYST gene that encodes a cytoplasmic protein
involved in protein transport. All cells containing lysosomes
have giant granules resulting from the abnormal fusion of
Granulocyte Development and Gurkamal Chatta, MD
Granulocyte Disorders Hematology 2009
primary granules with secondary granules. This results in a
delay in fusion of granules with phagosome.
pathogenetic organisms: S aureus, beta hemolytic strep
sites affected: no specific site is characteristic
clinical presentation: partial ocular and cutaneous albinism,
peripheral neuropathy, recurrent bacterial infectious, easy
bruising mild mental retardation, severe periodontal disease.

Acquired: drug induced [glucorticoids, colchicines, NSAID, IL-2]

Defects in killing:

Congenital:

Chediak-Higashi syndrome (see above)

Chronic granulomatous disease: rare (1:200,000) condition that


results in defective PMN oxidative metabolism. 70% inherited as
X-linked recessive. Mutations in the four proteins that assemble at
the plasma membrane account for all CGD. Two proteins (91kd
and 22kd) form a heterodimers cytochrome b-558 in the plasma
membrane. Two other proteins (47 kd, and 67 kd) interact with the
cytochrome after cell activation to form NADPH oxidase which is
required to form H2O2. The disease is characterized by increased
number of infections due to catalase-producing organisms which
destroy their own H2O2. that could have been used as a substrate
for NADPH oxidase. Patients often have extensive inflammatory
reactions. The condition is detected by either nitroblue tetrazolium
(NBT) dye test or the dihydrorhodamine (DHR) oxidation test.
These tests are based on the ability of products of oxidative
metabolism to alter the oxidation states of reporter molecules so
that they can be detected microscopically (NBT) or by flow
cytometry (DHR); Prophylactic antibiotics (TMP/Sulfa) and anti-
fungals markedly diminish the frequency of life-threatening
infections. Recombinant interferon- which non-specifically
stimulates the phagocyte system reduces the frequency and
severity of infections.

defect: mutation gp91phox (X-linked form); mutations in p47phox,


p67phox or p22phox in autosomal recessive forms; leads to inability
in assemble of NADPH oxidase required to generate oxygen
radicals.
pathogenetic organisms: catalase positive organisms (S aureus, B
cepacia, aspergillus species, nocardia species, Serratia marcescens
sites affected: no specific site is characteristic
Granulocyte Development and Gurkamal Chatta, MD
Granulocyte Disorders Hematology 2009
clinical presentation: abscess formation in the lungs, liver, brain,
and bone; soft tissue infections; gastrointestinal and urogenital
obstruction from granulomas.

Myeloperoxidase (MPO) deficiency: most common congenital


disorder of PMNs (1:2000). Inherited as AR. MPO deficiency is
not associated with clinically compromised defenses because of
other defense systems (NADPH oxidase, NO). MPO catalyzes the
formation of hypochlorous acid (bleach) from hydrogen peroxide
and chloride ion; hypochlorous acid in then converted to chlorine;
defect: mutations in MPO gene on 17q
pathogenetic organisms: not usually associated with clinical
disease
clinical presentation: associated with disseminated candidiasis in
patients with diabetes.
Transfusion Therapy
Basic blood group serology
Transfusion Therapy Pre-transfusion testing
Blood component therapy
Darrell J. Triulzi, M.D. Autologous/directed donations
Director, Division of Transfusion Medicine
Department of Pathology Transfusion reactions
University of Pittsburgh Transfusion transmitted infections
January 2009

ABO Blood Group System


Blood Membrane Plasma
Transfusion Therapy Group Phenotype Genotype Antibody
A (40%) A substance* A gene anti-B
B (10%) B substance** B gene anti-A
Basic Blood Group Serology
AB (5%) A&B substance A&B gene none
O (45%) No A or B No A or B anti-A, anti-B
substance gene
*n-acetylgalactosamine (GalNac) **galactose

ABO Structure on Red Cells ABO Antibodies


Naturally occurring
Usually IgM, efficiently bind
complement
Cause acute hemolytic transfusion
reactions
Can mediate rejection of transplanted
kidney, heart or lung

1
Rhesus (RhD) Blood Group RhD Antibodies
Occurs in Rh negative individuals exposed by
Integral red cell membrane spanning transfusion or pregnancy to Rh positive red
protein coded for by the RHD gene cells.
Present in 85% of individuals (Rh Antibodies are IgG.
positive), absent in 15% of individuals Capable of causing extravascular hemolysis
(Rh negative) and hemolytic disease of the newborn. (Rh
negative mother of an Rh positive fetus can
Most immunogenic red cell antigen
make anti-D which crosses the placenta and
outside of the ABO system hemolyzes fetal red cells).

Pretransfusion Testing Antibody Screen


TYPE determine ABO and Rh type of patient
red cells. +
incubation
SCREEN perform antibody screen for Sensitized Reagent RBC
Reagent
unexpected red cell antibodies in patient serum. Patient serum
red cell
Also called indirect antiglobulin test.
CROSSMATCH perform compatibility test
between patient serum and donor red cells. +
DIRECT ANTIGLOBULIN TEST (Coombs
test) tests if patient red cells are coated with IgG Sensitized Reagent RBC Coombs
or C3b, diagnostic of immune hemolysis. reagent
Agglutination

Antibody Screen Crossmatch


Method of detecting unexpected red cell
+
antibodies. Present in 1-5% of patients. incubation
Detects both IgG and IgM antibodies. Patient serum
Donor Sensitized Donor RBC
IgG antibodies usually clinically significant red cell
and are directed most commonly at the Rh,
Kell, Kidd,Duffy ,Ss blood group antigens. +
IgM antibodies usually NOT clinically
significant. Sensitized Donor RBC Coombs
reagent
Agglutination

2
Direct Antiglobulin Test
Crossmatch (Coombs Test)
Test to establish compatibility of patient Coombs reagent
serum with donor red cells. Anti-human IgG

Full 30 minute incubation crossmatch


required in patients with unexpected red +
cell antibodies (pos antibody screen).
Abbreviated (5 min) or computer Sensitized Patient RBC
crossmatch allowed if antibody screen in Anti-human C3d Agglutination
negative.
= C3d

Ordering Blood Components


Type and screen.
Transfusion Therapy Patient ABO, Rh type and antibody screen performed.
Indicated for patients in whom transfusion may be
needed but is not expected.
Type and crossmatch.
Blood Component Therapy ABO, Rh, Ab screen and crossmatches performed.
Indicated when transfusion is expected.
Uncrossmatched.
Blood is released before testing is complete.
Indicated for urgent transfusion.

Blood Products Whole Blood


Whole Blood
500 ml
Packed RBCs Platelet-rich Plasma 200 ml RBCs
250 ml plasma
Plasma Platelets 50 ml anticoagulant

Cryoppt- Cryoprecipitate Plasma Derivatives


reduced Storage:
Plasma
Albumin IVIg 35 days @ 1-6 C

3
Packed Red Blood Cells Indications for PRBCs
250 ml
To increase oxygen-carrying capacity in pts with
200 ml RBCs
symptomatic anemia
50 ml plasma &
anticoagulant Hgb<7g/dl (Hct<21%).
Storage: BP stable & no signif CV disease.
42 days (ADSOL)
Hgb 7-10 g/dl (Hct 21-30%).
Dose:
Tx based on severity of underlying CV/pulm disease.
1 unit Hgb 1 gm/dl
Hct 3%
Acute blood loss with BP instability or sx
anemia.
EBL 15-20% blood vol (750-1000ml).

PRBC Transfusion
Whole Blood Platelets
Emerging evidence indicates that even 50 ml each (approx.)
critically ill patients do not benefit from >5.5 x 1010 platelets/bag
liberal transfusion strategies eg. Hb 9-10g/dl Storage: 5 days at room temp,
Transfusion thresholds should be lowered to constant agitation
7-8g/dl for most patients Adults (>50 kg) - pool of 5
The optimal threshold for patients with Peds 1unit/10kg body wt
acute myocardial ischemia is not known
Expected increment:
plt ct 30 50K

Indications for Platelet


Transfusion Apheresis Platelets
To control or prevent bleeding due to 200-400 ml
deficiencies of platelet number or function >3.0 x 1011 plts/bag
Plt ct <10-20k/L prophylaxis (Equivalent to 5-6 WBplts)
Plt ct <50k/L bleeding or Storage: 5 days @ room
undergoing invasive procedure temp, constant agitation
Massive transfusion & bleeding Dose: 1 apheresis plt
product per transfusion
Bleeding with evidence of
platelet dysfunction

4
Indications for Apheresis
Platelets Fresh Frozen Plasma
200-250 ml
To control or prevent bleeding in patients Contains normal levels of all
refractory to whole blood plts (HLA- coagulation factors
matched or cross-match compatible Storage: 1 year frozen, 24 hrs
apheresis platelets) @ 1-6 C
Dose: 10-20 ml/kg (3-6 units)
To reduce donor exposures in patients to correct a significant
receiving a limited number of transfusions coagulopathy
Otherwise, same as for whole blood plts (Each unit will raise plasma factor levels by 3-5%)

Indications for FFP Indications for FFP (Cont.)


To control or prevent bleeding in patients with
a documented clotting factor deficiency Emergency reversal of
Warfarin - immediate
Active bleeding/invasive procedure with: PT Oral or IV Vit K reversal 6-8
> 3 secs (INR > 1.5) hours
Massive transfusion & bleeding w/ lab TTP - Plasmapheresis
proven coagulopathy
Congenital deficiency of II, V, X, XI, XII, or
XIII

Cryoprecipitate Indications for Cryoprecipitate


15-20 ml
Congenital or acquired hypofibrinogenemia
Contains:
Fibrinogen (150-250 mg) Uremic bleeding diathesis unresponsive to
vWF, factor VIII (80-100 units) DDAVP
Factor XIII, Fibronectin
Storage: 1 yr frozen, 6 hrs @ RT post thaw.
No longer used for von Willebrand Disease or
hemophilia due to availability of factor
Dose: pool of 6 units. Each unit will raise
fibrinogen level in an adult by 5-10 mg/dl. concentrates

5
Autologous PRBCs
Transfusion of Peri-operative:
Transfusion Therapy patients own blood Hemodilution collect
in OR with saline
Pre-op: replacement
Weekly donations Intra-operative
Autologous and Directed Donation Iron therapy salvage recover
Hct>33% suctioned blood
5-6 weeks storage Post-op:
Falling out of favor Wound drains

Transfusion Risks:
Allogeneic Vs Autologous
Indications for autologous PRBC transfusion?
Complication Allogeneic Autologous Transfusion Therapy
Viral Transmission Yes No
Immunologic Reaction Yes No
Clerical Error Yes Yes
Transfusion Reactions
Bacterial Contamination Yes Yes

Volume Overload Yes Yes

Same as for any PRBC transfusion.

What Should I Do? Acute Hemolytic TR


1. Stop the transfusion! Symptoms & Signs:
2. Keep IV open with 0.9% NaCL. Fever & Chills most
common initial sx. Hemolyzed
3. Product tags & pt ID.
Back pain, chest serum

4. Notify blood bank. pain, nausea,


If transfusion is terminated: flushing, dyspnea,
5. Collect & send blood & urine samples to bleeding (DIC),
blood bank (1 red & 1 lavender top tube). hemoglobinuria,
6. Send unit, tags & admin set to blood bank. acute renal failure.

6
Acute Hemolytic TR Acute Hemolytic TR
Etiology:
Management:
Transfusion of incompatible
1. Steps 1-6.
blood, 1:25,000 transfusions
2. Aggressive fluid (0.9 NaCl)
Usually due to misidentification
replacement for Tx of shock.
of patient or specimen
Most life-threatening rxns due
to ABO mismatch A O 3. Lasix/mannitol to increase renal
10-20% mortality, >50% ARF blood flow/urine output.

Febrile, Non-hemolytic TR Febrile, Non-hemolytic TR


Symptoms & signs Etiology:
Fever (temp rise at Recipient Abs to Cytokines in
least 1 C or 2 F) transfused WBCs transfused product
Chills
Dyspnea
Tachycardia
Fever-chill reaction Flushing

Febrile, Non-hemolytic TR Allergic Reactions


Management: Symptoms & signs:
Steps 1-6. Cutaneous hypersensitivity
Antipyretics for Urticaria & local Erythema
fever. Pruritis (no fever)
Plasma
Plasma
Protein
Etiology: (Allergen)
Protein

IV Meperidine for
Tx

Allergic reaction to
shaking chills
transfused plasma
Prevention: Premed with antipyretics and
proteins
use of leukoreduced blood components

7
Allergic TR Bacterial Sepsis
Management: Symptoms:
Stop transfusion. Fever Hypotension may progress
Administer antihistamine. Chills rapidly to endotoxic shock
If sx subside, may restart slowly
& observe for recurrence. Etiology:
For severe reactions: Stop and Bacterial
follow Steps 1-6. contamination
Prevention: of the unit
Antihistamine premed, Steroids if severe

TRALI Transfusion Related


Bacterial Sepsis Acute Lung Injury
Symptoms & signs:
Prevention:
Respiratory distress
Careful donor
screening Severe hypoxemia
Thorough cleansing Fever & chills
of phlebotomy site Hypotension
Bacterial detection Non-Cardiogenic pulmonary edema.
or culture of units Symptoms begin during or within 6 hours of
transfusion.

TRALI TRALI
Etiology: Management:
Anti-WBC antibodies (HLA or granulocyte) in Steps 1-6.
the transfused blood component Oxygen
React with recipients WBCs Mechanical
WBC aggregates occlude ventilation
pulmonary circulation. Intensive care
Cytokine release increased unit support
vascular permeability/edema
Most patients recover within 4 days.
& local inflammation.

8
Estimated Risks of Viral
Transmission
How safe is the blood supply?
Virus 1996 2001 2004
HIV 1:493,000 1:1,326,000 1:1,738,599

Hepatitis C 1:103,000 1:237,000 1:1,575,733

Hepatitis B 1:63,000 1:137,000 1:166,938

HTLV I, II 1:641,000 1:641,000 1:2,437,296

Note: CBB collects approximately 175,000 donors per year.


Dodd RY et al Transfusion 2002;42:975-9.

9
PRINCIPLES OF BLOOD TRANSFUSION THERAPY

Darrell J. Triulzi MD Institute for Transfusion Medicine


University of Pittsburgh School of Medicine

Blood transfusions have become an integral part of medical care in a wide variety of disciplines. The
following outline and lecture will summarize the important aspects of transfusion therapy.

I. BASIC BLOOD GROUP SEROLOGY

II. PRETRANSFUSION TESTING

III. COMPONENT THERAPY

IV. AUTOLOGOUS AND DIRECTED DONATION

V. TRANSFUSION REACTIONS

VI. TRANSFUSION TRANSMITTED DISEASES

Objectives

1. Understand the serology of the ABO and Rh system and how they apply to
blood component selection.

2. Become familiar with the methods and indications for performing direct (Coombs test) and

indirect antiglobulin (antibody screening and crossmatch) testing.

3. Understand the limitations and indications for type and screen, type and

crossmatch, and uncrossmatched blood.

4. Become familiar with the content and indications for each blood component.

5. Recognize the types and etiology of acute reactions associated with transfusion.

6. Describe how blood is screened for transfusion transmitted infections and the

risks associated with screened blood.

1
I. BASIC BLOOD GROUP SEROLOGY

A.
ABO system -the ABO group of an individual is determined by the phenotypic
expression of carbohydrate antigens on the surface of the red cell membrane.
Expression of" A" and "B" substance is controlled by separate genes in a
codominant fashion. Naturally occurring ABO antibody directed at the ABO
antigen(s) lacking in that individual develops by 4 months of age.

Membrane Plasma
Blood group phenotype Genotype antibodies
A A substance A gene anti-B
B B substance B gene anti-A
AB A and B substance A and B gene no ABO antibody
O no A or B substance lacks A or B gene anti-A and anti-B

In the general population, the normal distribution of ABO groups is as follows: 45%
group O, 40% group A, 10% group B, and 5% group AB.

B. Rh system - the D antigen is the most important of the Rh (rhesus) antigens and is
second only to the ABO antigens in clinical significance. It is a protein antigen
integral to the red cell membrane of 85% of the general population who are
referred to as Rh "positive". Individuals who lack this antigen (15% of the
population) are referred to as Rh "negative" and are capable of making IgG anti-D
when immunized by transfusion or pregnancy. Anti-D is the most common cause
of severe hemolytic disease of the newborn. This can occur when an Rh negative
mother has an Rh positive fetus. Fetal red cells can enter the maternal
circulation and stimulate production of anti-D. The maternally produced anti-D is
an IgG antibody that can cross the placenta causing hemolysis in the fetus.
Pregnant Rh negative mothers should receive Rh immune globulin (passive anti-
D) to prevent maternal immunization.

C. Other Blood Group Antigens - there are numerous other antigens within the
Rh system as well as over 20 other red blood cell antigen systems that have
been described. Over 95% of clinically significant red cell antibodies seen

2
are directed at one of six red cell antigen systems. These included ABO, Rh (D is
the most important), Kell, Kidd, Duffy, and Ss. The clinically significant antibodies to
antigens other than ABO are generally IgG and are capable of causing hemolytic
transfusion reactions and hemolytic disease of the newborn.

II. PRETRANSFUSION TESTING

There are multiple measures taken to ensure that a blood transfusion is as safe as possible.
These multiple layers of safety include the following:
Donor measures:
- Donor medical history and physical exam
- Check donor deferral registry
- Tests on donor blood for ABO group, Rh type, and serum antibodies.
- Infectious disease testing of donor blood for hepatitis B, hepatitis C, HIV1 &2,
HTLVI /II, West Nile virus, cytomegalovirus, bacteria, T. cruzi (Chagas) and syphilis.

Recipient measures:
- Positive identification of the recipient and the recipient blood sample.
- Review of the recipient transfusion history.
- Determining ABO group and Rh type on the recipient.
- Antibody Screen: test of recipient serum/plasma for non ABO (ie Rh, Kell, Kidd,
Duffy,Ss) unexpected antibodies (indirect Coombs test).

Indirect Antiglobulin test: Antibody screen

Sensitization

Agglutination

Unexpected red cell antibodies are generally found in patients with a history of
transfusion or pregnancy. Potentially hemolytic antibodies are found in 1-5 % of
all patients and 5 -15% of chronically transfused patients.

3
- Crossmatch: Test of recipient serum or plasma against donor red cells
1 ) Abbreviated crossmatch: Only used in patients with negative antibody screens
- Electronic crossmatch for patients with previous ABO group on file, or
- 5 minute incubation crossmatch to confirm ABO compatibility

2) Complete crossmatch - Required when the patient antibody screen is


positive. 30 minute incubation crossmatch to confirm compatibility of IgG
antibodies in addition to ABO.

Indirect antiglobulin test; Crossmatch

Coombs
reagent Agglutination

Special Testing

A Direct Antiglobulin Tests (DAT) = Direct Coombs Test

. Detects antibody (IgG) or complement (C3d) bound to patient RBC.


. Should be ordered when hemolysis is suspected from any cause.
. The DAT is also routinely done as part of a transfusion reaction
investigation to diagnose immune hemolysis.

Direct Antiglobulin (Coombs) Test

)
Agglutination

B. Specific non-ABO Antibody Identification

. Performed when the antibody screen is positive to identify the


specificity of the unknown antibody.
. A panel of reagent red cells of known antigen phenotype are incubated
with patient serum.

4
. Some antibodies (e.g., Rh, Kell, Kidd, Duffy, Ss) can cause hemolysis. These
are generally IgG antibodies that react best at body temperature (37C).
Antigen negative blood is required for transfusion.

. Some antibodies are clinically insignificant. They are generally IgM and react
best at room temperature or below and not at body temperature (e.g. Lewis,
I, P, M). Exception: ABO antibodies are generally IgM and can cause
hemolysis because they efficiently bind and activate complement.

III. COMPONENT THERAPY

Ordering Blood Products

"Type and Screen" -The patient's ABO group, Rh type and antibody screen are
performed. No units are crossmatched. Type specific blood can be crossmatched
and available in 10-15 minutes if no antibodies are present in the patient's serum.
Type and Screen should be used for patients in whom transfusion may be needed
but is not expected.

"Type and Cross" -The patients ABO group, Rh type and antibody screen are
performed. A specified number of units are crossmatched and reserved for that
patient for 72 hours. The number of units crossmatched depends on the surgical
procedure and clinical situation. A Surgical Blood Ordering Guideline has been
established with the surgeons which lists recommended pretransfusion testing for
each procedure, (i.e. Cholecystectomy -Type and Screen, Coronary Bypass -Type
and Cross 2 units RBC).

"Uncrossmatched" -Group O RBC can be issued immediately for a patient with life
threatening hemorrhage when time does not permit completion of the ABO and Rh
testing. When the latter can be done, type specific uncrossmatched blood can be
issued. The requesting physician must authorize the use of uncrossmatched RBC
because there are slightly increased risks for the patient. Uncross matched blood is
safe >95% of the time.

5
Description and Indications of Blood Products

A.
Whole Blood (WB)
-Each unit has a total volume of approximately 500 ml (200 ml RBC, 250 ml
plasma and 50 ml of anticoagulant preservative).
-The hematocrit is approximately 40% with a shelf life of 35 days when stored at
1-6C. Platelets and WBC are nonfunctional. Stored WB is deficient in some
coagulation factors.
Indications:

- Rarely available. Each unit made into components.


No advantage over using RBC and volume expanders.

B.
Red Blood Cells (Packed)
-Each unit contains the RBC from one unit of whole blood (200 ml of RBC) and
120 ml of preservative anticoagulant.(ADSOL additive solution)
-The hematocrit is 55-65% with a shelf life of 42 days when stored at 1-6C.
Platelets and WBC are nonfunctional. There is a small amount of plasma (40 ml)
in a packed RBC.
Indications:
Symptomatic anemia to increase oxygen carrying capacity. One unit will
increase the hemoglobin 1 g/dl and the hematocrit 3% in an average adult.
Hct < 21 %: Anemia in this range is often symptomatic and associated with
compensatory changes even in normal individuals.
Hct 21-30%: Clinical judgment of the need for RBC is based on symptoms,
the amount of bleeding, cardiopulmonary status, duration of anemia, etc.
Most patients do not need to be transfused unless Hct is <25%, exception:
acute myocardial ischemia or infarction.
Hct > 30%: RBC transfusion rarely if ever indicated unless there is continuing
rapid hemorrhage or acute myocardial ischemia or infarction.

6
C. Platelets
Whole blood platelets
-Each unit is derived from one unit of whole blood and contains a minimum of 5.5 x
10e10 platelets and 60 ml of plasma with hemostatic levels of all coagulation factors.
-Each unit will raise the platelet count 5,000-10,000/ul in an average adult.
- Recommended adult dose is 5 units. For pediatric patients a dose of 1 unit/10kg body
weight is recommended.
Apheresis platelets
- each unit comes from a single donation taking 90 minutes on an apheresis
instrument. Contains the same number of platelets as a 5 unit pool of whole blood
platelets in 200-400 ml of plasma.
- recommended adult dose is one apheresis product per transfusion episode
- HLA matched platelets are derived from a single donor by apheresis. They are
indicated in patients who do not respond to whole blood pooled platelets due to HLA
antibodies.
Storage of platelets
-Platelets are stored continuously agitated at 22-26C for up to five days.

Indications
Platelet count <10,000/ul: Prophylactic use of platelets is indicated to prevent
spontaneous bleeding.

Platelet count 10,000 -50,000/ul: Platelets indicated for hemostasis in patients who
are bleeding or undergoing invasive procedures.

Platelet count 50,000 -100,000/ul: Platelets may be indicated in bleeding patients


who have compromise of other aspects of hemostasis, i.e. coagulopathy or who
have bleeding at critical sites, i.e. retina, CNS, thoracic cavity.

Platelet count> 100,000/ul: Platelet transfusion not indicated unless there is


evidence of dysfunctional platelets causing a prolonged closure time, i.e.
aspirin, uremia, cardiopulmonary bypass.

7
D. Fresh Frozen Plasma (FFP)
-Each unit is derived from one unit of whole blood and has a volume of 220 ml and
can be stored one year at -18C. All coagulation factors are at normal levels. There
are no viable RBC, white cells, or platelets. Each unit of FFP provides 3-5% of the
normal plasma coagulation factor levels in an average adult.
- Recommended dose is 10-20 ml/kg body weight.

Indications
-coagulopathy defined by a prothrombin time (PT) INR>1.5 or partial
thromboplastin time (PTT) 8-10 sec above normal in a patient with active bleeding,
or surgery or an invasive procedure.
-Not indicated as a volume expander or for protein supplementation.
E. Cryoprecipitate
-Each unit has a volume of 10-15 ml and is prepared from one unit of FFP. -
Cryoprecipitate is rich in fibrinogen, Factor VIII, von Willebrand factor, and
fibronectin. It does not contain appreciable levels of other coagulation factors.
-Dosed as a pool of 6 units. Each unit will raise the fibrinogen level by 5-10 mg/dl in
an adult.

Indications:
-Fibrinogen deficiency <100 mg/dl in a patient with active bleeding, or surgery or an
invasive procedure
-Corrects the platelet defect in uremia in some patients.
-Not indicated for hemophilia A or von Willebrand disease: virus free, factor
concentrates are preferred.

F. Albumin and Plasma Protein Fraction


-Colloid volume expanders.
-Albumin preparations contain 95% albumin and PPF 85% albumin
-Available in 5% (5 gm/dl) and 25% (25 gm/dl) concentrations
-Do not transmit viral disease due to heat treatment.

Indications
-Short term intravascular volume expansion when albumin levels are low, i.e.
liver disease, plasmapheresis, cardiopulmonary bypass, hemodialysis.

8
IV. AUTOLOGOUS BLOOD AND DIRECTED DONATION

Autologous Blood

Preoperative Autologous Donation (PAD)

PAD use is declining as there are limited indications for patients to donate blood for
themselves prior to elective surgery and patients are rendered anemic making it
more likely that they will require transfusion. Often (50% of the time) these units
are thrown out. Units of blood are usually collected at weekly intervals up to one
week before surgery, and the red cells can be stored for up to six weeks. The
patient is given iron supplements to help maintain their hematocrit over the
required 33% level. The main contraindications to autologous donations are
bacteremia or severe cardiopulmonary disease. Autologous blood cannot be used
for other patients.

Perioperative Autologous Blood Collection

Use is increasing as an alternative to PAD. In some types of surgery, shed blood


from the operative field can be salvaged by suction, filtered, washed, and reinfused
back to the patient. There are also salvage systems to collect blood shed
postoperatively (e.g., chest tubes). The sites of collection for these methods should
be sterile to avoid disseminating bacteria or tumor cells. Another perioperative
technique is intraoperative hemodilution, in which blood is drawn off in the OR
immediately before surgery, replaced with other fluids to maintain the intravascular
volume, and then reinfused for bleeding if needed.

Directed Donations

When a patient chooses his/her blood donor this is called a directed donation.
There is no evidence that directed donors are safer than volunteer blood donors.
However, directed donors are useful to ensure the availability of blood for the
patients surgery and to supplement the general blood supply if not needed by the
patient.

9
V. TRANSFUSION REACTIONS

Acute Reactions STOP TRANSFUSION, send blood bag and new specimen to blood bank with completed transfusion reaction report form.
Lab will check for evidence of immune hemolysis and recommend any changes needed in blood therapy.

Type Rate Cause Treatment Prevention

Acute hemolytic 1:25,000 (RBCs) Usually wrong blood given Diuresis, DIC therapy Proper patient-blood ID

Delayed hemolytic 1:2,500 (RBCs) Past RBC antibodies Compatible blood RBCs compatible with prior
antibodies

Febrile 1% HLA antibodies to Antipyretics Leukoreduced blood


donor WBCs

Allergic 1% Antibodies to donor Antihistamines Premedication with anti-


plasma proteins histamines and/or steroids.Wash
RBCs if recurrent and severe

Transfusion Related Acute 1:5,000 Donor antibodies to patient Pulmonary support Minimize plasma transfusion.
lung injury (TRALI) WBCs Avoid use of plasma from
female donors
Transfusion Associated Rare Donor T-lymphocyte No effective therapy. Irradiate cellular blood
Graft-vs-host (TAGVHD) immune reactions components for at-risk patients
Eg stem cell transplant
Septic (bacterial) 1:10-20,000 platelets Bacteria enter unit during Antibiotic therapy Proper arm preparation.
1:100-500,000 red cells donation Bacterial screening of platelets

Anaphylactic 1:150,000 Antibodies to IgA or other Epinephrine Avoid plasma exposure in IgA-
plasma contents deficient patients

Post-transfusion Rare Platelet antibodies, usually IV immune globulin or Autologous or PLA1-negative


Al
purpura anti-PL plasmapheresis blood if past episode

Page 10
VI. Transfusion Transmitted Infectious Diseases

Agent Donor Screen Risk Per Screened Unit Comments

Hepatitis C virus Anti-HCV, <1:1,600,000* HCV accounts for 95% of cases of


anti-HBc post-transfusions hepatitis (non-A, non-
HCV nucleic acid test B). HCV nucleic acid screening began
in 1999.

Hepatitis B virus HbsAg, anti-HBc 1:166,000 Infrequent cause of post-tx hepatitis.

Human Immunodeficiency anti-HIV-1,2 1:2,100,000* HIV nucleic acid testing began in 1999
virus Type 1 HIVp24 testing
HIV nucleic acid test
Human Immunodeficiency anti-HIV-2 <1:10,000,000 Very rare in US blood donors
virus Type 2

Human T-cell lymphotrophic virus 1 anti-HTLV-1 1:2,437,000 A few cases of myelopathy reported.
No cases of T cell leukemia/lumphoma.

Cytomegalovirus anti-CMV <1:100 Clinically significant primarily in


immunosuppressed patients.

Bacteria Bacterial testing of platelets 1:50,000 risk of a clinical More common with platelets due to
reaction room temperature storage.

Syphilis RPR Virtually zero One case in US in 40 years

Malaria, Chagas Donor travel history Rare 3-5 cases in US annually

West Nile Virus WNV nucleic acid test Rare Risk limited to mosquito season. Rare
transmission with testing.
* Nucleic acid testing

Dodd RY et al. Current prevalence and incidence of infectious disease markers and estimated window period risk
in the American Red Cross blood donor population. Transfusion 2002:42:975-979.

Page 11
BIBLIOGRAPHY

1. American Association of Banks Technical Manual


16th edition, 2008.

2. American Association of Blood Banks Standards 25th


edition, 2008.

3. Rossis Principles of Transfusion Medicine. 3rd Edition 2002


Lippincott, Williams and wilkins.

10/08 DJT
CONGENITAL PLATELET
FUNCTION DEFECTS
PLATELET
PLATELET
DISORDERS
DISORDERS z Membrane Defects
Glanzmanns thrombasthenia (deficency of
IIbIIIa surface receptor - aggregation
defect)
Bernard-Soulier Syndrome (deficiency of Ib
surface receptor adhesion defect)
Joseph E. Kiss, M. D.
z Intracellular Defects
Division of Storage pool(dense body) deficiency
Hematology/Oncology Alpha granule deficiency (gray platelet
syndrome)

ACQUIRED PLATELET
ANTIPLATELET DRUGS
FUNCTION DEFECTS
z Drug-induced
zAspirin
Aspirin(acetylsalicylic acid)
Other nonsteroidal antiinflammatory zThienopyridine derivatives (ADP-
drugs receptor antagonists)
Penicillins, cephalosporins Ticlopidine

z Uremia Clopidogrel
zGlycoprotein IIb/IIIa receptor
z Cardiopulmonary bypass surgery inhibitors

Effect of ASA
(650 mg)
On Bleeding
Note: NSAIDS Time
Irreversibly acetylates reversibly inhib-
platelet cyclooxygenase it cyclooxygenase
Small doses (81 mg)
are effective, but
relatively weak
effect

1
TICLOPIDINE AND CLOPIDOGREL INTEGRIN b
b3 RS
RS
z Drugs used to interfere with the binding of
z Thienopyridine inhibitors of the low affinity type II
purinergic ADP receptor (other agonists also affected)
fibrinogen to aIIbb3 (IIbIIIa
(IIbIIIa)) receptor Rx in
z Prolong bleeding time in a dose- and time-dependent manner, unstable angina and angioplasty/stenting
to a greater extent than aspirin procedures
Maximal prolongation of the bleeding time takes 4 to 7 Abciximab (ReoPro) - a humanmurine chimeric Fab
days fragment derived from a monoclonal antibody
Bleeding times take as long as 10 days to normalize after
discontinuation; PDR recommends d/c 5 days before Eptifibatide (Integrelin) - a cyclic heptapeptide containing
surgery a Lys-Gly-Asp (RGD) sequence that resembles a region
z Reduce incidence of within the carboxy terminus of the fibrinogen chain that
mediates binding of fibrinogen to the receptor. Reversible
(a) recurrent stroke, transient ischemic attacks, vascular inhibition.
death or myocardial infarction in patients with unstable
angina Tirofiban (Aggrastat) - a small molecule peptidomimetic
(b) coronary (and peripheral arterial?) stent thrombosis designed to mimic the Arg-Gly-Asp (RGD) sequence
within fibrinogen, VWF, and other ligands. Reversible
inhibition.

THROMBOCYTOPENIA: CLINICAL
CORRELATES

CONDITION PLATELET COUNT


(Per ul)
Thrombocytopenia <150,000

Prolonged bleeding time <50-100,000

Increased surgical hemorrhage <50,000

Spontaneous hemorrhage <20,000

RELATIONSHIP BETWEEN PLATELET COUNT THROMBOCYTOPENIA


& PRIMARY HEMOSTASIS MECHANISMS

z Decreased Production
z Increased Destruction
z Splenic Sequestration
z Dilutional

2
THROMBOCYTOPENIA DUE TO THROMBOCYTOPENIA DUE TO
DECREASED PRODUCTION INCREASED DESTRUCTION

z Aplastic anemia & related hematologic z Immunologic z Non-immunologic


disorders ITP (Immune DIC(Disseminated
Thrombocytopenic Intravascular
z Chemical/physical toxins(chemotherapy, Coagulation)
Purpura
radiation) TTP (Thrombotic Sepsis

z Marrow infiltration(leukemia,fibrosis) Thrombocytopenic


Purpura)
z Infection(viral,bacterial) HIT (Heparin-Induced
Thrombocytopenia)

HALLMARK: Megakaryocytes decreased &/or


abnormal appearance Hallmark: Normal/Increased megakaryocytes in bone marrow

Bone marrow aspirate showing megakaryocytic hyperplasia with clustering of Peripheral smear in a patient with ITP showing an almost total absence of
the megakaryocytes around a spicule platelets

Lazarchick, J. ASH Image Bank 2001;2001:100177 Lazarchick, J. ASH Image Bank 2001;2001:100177

Copyright 2001 American Society of Hematology. Copyright restrictions may apply. Copyright 2001 American Society of Hematology. Copyright restrictions may apply.

CLINICAL PICTURE OF ACUTE


AND CHRONIC ITP

CHARACTERISTICS ACUTE CHRONIC

Age <8 yr old 20-40 yrs


Sex Predilection None F/M 3:1
Viral Prodrome Common Unusual
Onset Abrupt Insidious
Platelet Count <20,000/ul 20-80,000/ul
Splenomegaly Absent Absent
Duration 2-6 weeks Years
Spontaneous 90% Uncommon
Remissions

3
MECHANISM OF PLATELET ITP Treatment
DESTRUCTION IN ITP
Observation if not bleeding and
IgG-coated thrombocytopenia not severe
Platelet Block reticuloendothelial (RE) clearance of
platelets
Glucocorticoids (prednisone 1 mg/kg/da)
IVIG
Splenectomy

IgG antiplatelet antibody Block antibody production


Immunosuppressive agents (eg, anti-CD20 monoclonal
Fc receptor Splenic macrophage
ab Rituximab)

Enhance platelet production TPO agonists

Thrombotic Thrombocytopenic Peripheral smear showing microangiopathic hemolytic features with


numerous RBC fragments (helmet cells/schistocytes)
Purpura (TTP)

z Systemic microvascular platelet aggregation


and deposition of platelet-vWF in
microcirculation, esp. brain, kidney, GI tract,
and heart
z 10-20% mortality
z Females predisposed (2:1)
z Primary (most common idiopathic =
autoimmune inhibitor to ADAMTS-13; rare
congenital = ADAMTS-13 deficiency) and
secondary causes
Lazarchick, J. ASH Image Bank 2001;2001:100174

Copyright 2001 American Society of Hematology. Copyright restrictions may apply.

TTP
ENDOTHELIUM vWF
vWF
Platelet
Moake JL HospPract 1999; 34:53

Platelet Platelet Platelet


Platelet
Platelet Platelet Platelet
Platelet
(ADAMTS-13)
Platelet Platelet
Platelet vWF
Platelet

TTP

ENDOTHELIUM

4
Heart in TTP: Extensive involvement of small TTP: Immunohistochemistry for
muscular arteries in the myocardium factor VIII/vWF (Figure D)

Hosler, et al. Arch Pathol Lab MedVol 127, July 2003 Hosler, et al. Arch Pathol Lab MedVol 127, July 2003

Classic Pentad
Pentad Clinical Features of
TTP TTP Treatment Give ADAM(TS13)!

z Thrombocytopenia z Plasma exchange replenishes missing


ALWAYS
z Microangiopathic hemolytic anemia ADAMTS-13 (vWF cleaving protease)
(shistocytes on peripheral blood smear, and removes antibodies to ADAMTS-13
increased LDH) z Immunosuppressive agents
z Fever corticosteroids, anti- CD20 moab
z Fluctuating CNS changes: TIA, confusion, z Plt transfusions not used because may
VARIABLE
seizures, coma, etc. increase microvascular plt-vWF
z Renal involvement: Proteinuria,
Proteinuria, hematuria,
hematuria, thrombosis
renal insufficiency

Heparin-Induced Heparin-Induced Thrombocytopenia


Thrombocytopenia (HIT) (HIT): Pathophysiology

z Prevalence: up to 1% to 3% of heparin- IgG antibody


PF4 Heparin

treated patients Formation of


immune complexes

Consequences: ~30% of untreated HIT


(PF4-heparin-IgG)
z
patients are at risk for developing life- or Formation of
PF4-heparin

limb-threatening thromboembolic
complexes
Microparticle
release

complications due to venous and/or EC injury

arterial thrombosis Platelet PF4


release

z Management: immediate cessation of


heparin; strongly consider use of non-
Platelet
activation*

heparin alternative anticoagulant Fc receptor


Heparin-like
molecules

Blood vessel
ECs in vessel wall

5
Heparin-Induced Thrombocytopenia (HIT):
Diagnostic Overview

z Consider HIT when:


Platelet count falls within 4-14 days of heparin
exposure (sooner if previously exposed):
>50% from baseline and/or
150,000/mm
New thrombotic or thromboembolic event occurs

z Do not delay treatment; perform


confirmatory laboratory assay (heparin-PF4
antibody ELISA) Lazarchick, J. ASH Image Bank 2001;2001:100172

Copyright 2001 American Society of Hematology. Copyright restrictions may apply.

THROMBOCYTOSIS
z Reactive--Platelets <1,000,000/ul
Inflammatory, infectious conditions
Rx underlying cause

z Myeloproliferative syndromes--Platelets
>1,000,000/ul
Potential for bleeding and/or thrombosis
JAK-2 mutation present in 40-50% patients
Rx - lower platelet count, ASA for ischemic events

z Rebound--Transient increase following


splenectomy
Reversible, Rx usually not needed

6
PLATELET DISD Joseph E. Kiss, MD
Hematology 2009

Platelet Disorders

Joseph E. Kiss, M.D.


PLATELET DISD Joseph E. Kiss, MD
Hematology 2009

Platelet Disorders

Learning Objectives

1. The student should know how congenital and


acquired conditions or drugs inhibit the function of
platelets
2. The student should comprehend the basic
mechanisms that result in thrombocytopenia
3. The student should understand the
pathophysiologic basis, clinical presentation, and
treatment of three common causes of
thrombocytopenia
4. The student should be familiar with the main
causes of thrombocytosis

Platelet Disorders

This lecture will cover both qualitative and quantitative platelet disorders,
especially those that illustrate key pathophysiologic principles and/or are clinically
important.

Qualitative Platelet Disorders

Congenital Defects of Platelet Function

Platelet function defects fall into two categories: congenital and acquired.
Congenital platelet function defects can further be classified according to whether
they affect the membrane receptors or are intracellular. For example,
Glanzmann's thrombasthenia is the result of a defect in gp IIb-IIIa, the receptor
for fibrinogen. As a result, fibrinogen cannot link platelets to each other, resulting
in the complete absence of platelet aggregation. Bernard-Soulier syndrome
results from a defect in gp Ib-IX, preventing binding of von Willebrand factor to
PLATELET DISD Joseph E. Kiss, MD
Hematology 2009

the platelet and an adhesion defect. Familial intracellular defects which have
been described consist of storage pool (dense body) deficiency and alpha
granule deficiency or so called gray platelet syndrome. All of these disorders
result in bleeding tendencies of variable severity, and are characterized by a
prolongation of the bleeding time or closure time. They are also characterized by
more or less distinct abnormalities seen on platelet aggregation tracings.

Aquired Defects of Platelet Function

Acquired platelet function defects are quite common in clinical medicine. A


number of drugs interfere with platelet function, however, only those with the
most pronounced effects will be discussed. Aspirin, or acetylsalicylic acid,
irreversibly acetylates platelet cyclo-oxygenase resulting in a blockade of
thromboxine A2 synthesis. Tiny doses, as little as 80 mg, can completely block
this enzyme in the cohort of platelets induced at the time of ingestion. This
persists for the lifespan of those platelets which are acetylated. Excessive
bleeding during surgical procedures may result. For this reason, a history of
aspirin ingestion is routinely taken prior to surgery. Aspirin is also used for
vascular disease prophylaxis (myocardial infarction and thrombotic
cerebrovascular accidents). Other nonsteroidal antiinflammatory drugs, such
ibuprofen, and naprosyn, also inhibit cyclo-oxygenase. However, these drugs
inhibit cyclo-oxygenase reversibly; their effect subsides rapidly after
discontinuation of the drug. Certain penicillins, such as ticarcillin, in high
doses can result in clinically significant platelet dysfunction sometimes resulting
in clinical bleeding and a prolonged bleeding time. Finally, two groups of drugs -
thienopyridine derivatives and anti-IIb-IIIa receptor blockers - have been
developed which have potent therapeutic antiplatelet effects.

Another common acquired platelet function abnormality is that induced by renal


failure. While the severity of renal failure does not correlate very well with
bleeding time abnormality, dialysis has been noted to normalize the prolonged
bleeding time in this situation. Abnormalities of platelet aggregation and of
platelet procoagulant activity may be found. Finally, passage of platelets through
an extracorporeal circuit such as during cardio-pulmonary bypass surgery, also
produces platelet dysfunction and dilutional thrombocytopenia due to the
crystalloid priming solutions used for cardiac bypass. Among other
abnormalities, the platelets become activated and partially release their granule
contents. As a result of this, the bleeding time becomes prolonged and clinical
bleeding can be problematic.

Quantitative Platelet Disorders

Depending upon the underlying pathophysiology, thrombocytopenia may be


associated with either bleeding (due to simple platelet deficiency) or thrombosis
(platelet/coagulation system activation).
PLATELET DISD Joseph E. Kiss, MD
Hematology 2009

Thrombocytopenia Mechanisms

Thrombocytopenia is defined as a platelet count of <150,000/l. As a general


frame of reference, conditions with platelet counts between 150,000 and 100,000
are not associated with a risk of bleeding. Between 100,000 and 50,000, the
bleeding time may become prolonged. Thrombocytopenia below 50,000 is
associated with an increased risk of surgical hemorrhage, and spontaneous
hemorrhages may occur when the platelet count is below 10-20,000.

The two primary mechanisms responsible for thrombocytopenia are decreased


production of platelets from megakaryocyte precursors, or increased destruction
of platelets caused by pathophysiologic processes which destroy or consume
platelets. It is also important to keep in mind that although the spleen sequesters
1/3 of the total circulating platelet mass, pathologic conditions resulting in
splenomegaly can result in significant thrombocytopenia, with sequestration of
80% or more of the total circulating platelet mass. Finally, patients undergoing
surgical procedures with massive fluid-volume resuscitation and total blood
volume replacement frequently develop a thrombocytopenia which is dilutional in
nature. This occurs to a limited extent during cardiopulmonary bypass surgery
and to a greater extent during liver transplantation surgery.

Causes of decreased platelet production include the following: 1) primary


disorders of the bone marrow such as aplastic anemia and related hematologic
disorders in which there is impaired thrombopoiesis, 2) Chemical or
physical/toxic injury to the bone marrow such as that seen with radiation
treatment or chemotherapy, 3) marrow infiltration such as that associated with
leukemia, cancer, and marrow fibrosis, and 4) infection. In all of these disorders
the characteristic finding on bone marrow examination is that megakaryocytes
are either decreased or are abnormal in appearance. Thrombocytopenia due to
increased platelet destruction could be due to nonimmunologic or immunologic
causes. Examples of nonimmune platelet destruction include DIC (thrombin is a
potent platelet agonist), and sepsis (endotoxin aggregates platelets). Disorders
in which platelet destruction occurs characteristically have ample numbers of
megakaryocytes in the bone marrow. This so called "megakaryocytic
hyperplasia" is a consequence of the bone marrow trying to compensate for
peripheral destruction of platelets.

Clinical Disorders

IMMUNE THROMBOCYTOPENIC PURPURA (ITP)

The major immunologic disorder of blood platelets is known as ITP. It is a


common platelet disorder generally affecting younger individuals. However, ITP
can occur even in the elderly. Clinically two forms are recognized. Acute ITP is
generally a disease of youngsters less than 8 years old. It occurs equally as
PLATELET DISD Joseph E. Kiss, MD
Hematology 2009

common in males as females and is preceded by a viral illness. It's onset is


abrupt, causing easy bruising, bleeding from nasal and oral mucous membranes,
and occasionally even more serious bleeding manifestations. Fortunately, the
disorder is self limited, resolving completely in 90% of patients within 2-6 weeks
of onset. It's underlying pathogenesis is believed to be associated with the
formation of antigen-antibody immune complexes which bind to platelets in an
"innocent bystander" reaction.

Chronic ITP typically presents in young adults between the ages of 20 to 40. It
affects females three times more often than males and has an insidious onset
with history of easy bruising and bleeding manifestations occurring over several
months prior to medical presentation. In contrast to the childhood form, the
disease does not remit spontaneously. Generally, corticosteroids are used for
initial management. Splenectomy is needed in the majority for long-term control.
The efficacy of steroids is reported to be 90% and for splenectomy responses are
seen in 75-80% of patients. Intravenous gammaglobulin is also very effective.

In ITP, anti-platelet autoantibodies bind to platelet surface antigens. The


antibody coated platelets are recognized by splenic macrophages which have Fc
receptors. Binding of IgG coated platelets to the Fc receptors occurs, followed
by phagocytosis and destruction. Complement potentiates the autoantibody
induced clearance of the patient's platelets. Therapies mentioned above
primarily affect the clearance mechanism. For example, steroids impair
phagocytosis of splenic macrophages; IV gammaglobulin acts by binding to Fc
receptors located on splenic macrophages, preventing immune adherence of IgG
coated platelets. The mechanism of benefit from splenectomy should be fairly
obvious. There is also evidence of blunted platelet production in patients with
ITP. This is believed to be due to antibody-mediated suppression of
thrombopoiesis. Recently, a new class of TPO-agonist drugs have been
developed which increase platelet counts in ITP patients by stimulating platelet
production from megakaryocytes.

THROMBOTIC THROMBOCYTOPENIC PURPURA (TTP)

TTP/HUS is a life-threatening disorder that represents both a diagnostic and


management challenge. TTP is characterized by the presence of
thrombocytopenia in association with microangiopathic hemolytic anemia, both
resulting from microvascular platelet/von Willebrand factor thrombi. Small vessel
occlusion in various organs, notably the brain and kidney, is responsible for the
clinical manifestations which include fluctuating neurologic abnormalities and a
variable degree of renal insufficiency. On clinical grounds, cases with more of a
neurologic presentation have been termed TTP, while those with more
pronounced renal involvement are often referred to as Hemolytic-Uremic
Syndrome (HUS). However, clinical overlap is common and some authors use
the combined term TTP/HUS. TTP may occur on an idiopathic basis or in
association with pregnancy, autoimmune conditions, infections, malignancy,
PLATELET DISD Joseph E. Kiss, MD
Hematology 2009

stem-cell transplantation, and with exposure to certain medications. Treatment


using plasma exchange has reduced mortality in idiopathic TTP from more than
90% to less than 20%.

Pathophysiology
Abnormal proteolysis of von Willebrand factor and the presence of circulating
ultra-large molecular weight multimers (ULvWF) found in patients with TTP have
been related to a deficiency of a plasma metalloprotease that processes vWF
(ADAMTS-13 - A Disintegrin And Metalloprotease with Thrombospondin
type I motifs). Endothelial injury plays a critical role in perpetuating the disorder.
Under physiologic conditions vWF is released from endothelial cells and platelets
as ULvWF (M. W. approximately 1 to 1.5 x 106 daltons). Sheer conditions
present in flowing blood cause unfolding of the molecule leading to its
accessibility to ADAMTS-13, which clips the protein into smaller molecular weight
multimers. The higher molecular weight forms have greater adhesive properties,
and hence, a greater propensity to promote platelet-platelet and platelet-
subendothelial interactions. The pathological hallmark consists of
microvascular occlusion of terminal arteries and capillaries. The lesions contain
predominantly platelets and von Willebrand factor. This is in contrast to the
situation in disseminated intravascular coagulation (DIC) where fibrin deposition
is prominent. Mutations in the ADAMTS-13 gene have been found to cause
familial (chronic/relapsing) TTP. In the more common idiopathic TTP, deficiency
of ADAMTS-13 has been associated with IgG inhibitors in most of the cases. At
this time, TTP remains a clinical diagnosis: the use of ADAMTS-13
measurements in making the clinical decision to initiate plasma exchange in a
patient with presumed TTP/HUS is considered investigational.

Clinical Manifestations
The classic pentad findings, consisting of thrombocytopenia,
microangiopathic hemolytic anema, fever, neurologic changes, and renal
dysfunction, are seen in only a minority of patients. A high clinical index of
suspicion is indicated because delays in recognition may adversely affect the
outcome. Acceptable criteria for a provisional diagnosis include
thrombocytopenia and microangiopathic hemolytic anemia in the absence of an
alternative cause. Microangiopathic hemolysis is suggested by the presence of
fragmented RBCs (shistocytes) on the blood smear in conjunction with elevated
LDH levels. Neurological symptoms include episodes of focal weakness, visual
disturbances, reduced mentation/decreased consciousness, headache, seizure,
and coma. The evanescence of some of the early symptoms probably reflects
reversibility of the microthrombi, which lodge in small vessels. Abdominal pain
resulting from intestinal and/or pancreatic ischemia may also occur along with
nausea, vomiting, and ileus. Even in cases without severe azotemia, renal
involvement may be evident, including proteinuria and hematuria.

Management
Daily plasma exchange using an apheresis machine has become the standard
of care for the management of TTP. In addition to replacing missing ADAMTS-
PLATELET DISD Joseph E. Kiss, MD
Hematology 2009

13, plasma exchange removes antibodies to ADAMTS-13, which have been


associated with idiopathic TTP. Adjuvant pharmacological therapy of TTP
involves using immunosuppressive medications, primarily corticosteroids and
more recently, the anti-CD20 monoclonal antibody - rituximab.

HEPARIN-INDUCED THROMBOCYTOPENIA (HIT)

Much has been learned regarding the etiology, pathogenesis, and management
of heparin-induced thrombocytopenia (HIT) over the last ten years. Two forms of
HIT are recognized. So-called HIT, Type I is a benign entity, characterized by
mild thrombocytopenia (platelets rarely <100,000) occurring soon after
administration of intravenous heparin, and gradual recovery of the platelet count
despite continued therapy. In contrast, Type II HIT is an immunologically-
mediated disorder associated with moderate thrombocytopenia (platelet count
<150,000 or 50% drop from baseline levels) occurring between 4-14 days after
initiation of heparin therapy, and often complicated by thromboembolic episodes.
The frequency of HIT in several studies is reported between 1% and 3% using
unfractionated heparin in therapeutic doses. Approximately one third of these
cases are associated with clinically-detected venous or arterial thrombosis.
Although the incidence is lower, HIT can also occur using prophylactic
unfractionated heparin and intravenous flushes. The incidence is dramatically
lower in studies using low molecular weight heparin. Arterial thromboses occur
in approximately 20% of all thrombotic episodes. These are seen frequently in
conjunction with vascular or cardiac procedures, suggesting that endothelial
damage may be an important predisposing factor. The target antigen in HIT is
the heparin/Platelet Factor 4 (PF4) complex that induces antibody formation in
susceptible individuals, forming immune complexes. The immune complexes
bind to platelet Fc receptors, leading to platelet activation. The antibodies can
also bind to heparan sulfate on the surface of endothelial cells, leading to
endothelial cell injury and tissue factor generation. Platelet activation and tissue
factor production lead to increased thrombin generation and a prothrombotic
state. The anti-heparin-PF4 antibody assay has a reported sensitivity of over
90%. The management of patients with HIT requires consideration of alternative
anticoagulation therapy in light of the high risk of thrombosis within 30 days after
diagnosis. Agents that directly inhibit thrombin activation are recommended.
Currently, two agents are licensed for this purpose: recombinant hirudin
(Lepirudin) and argatroban (Novastan). These agents have the advantage of
rapid action after intravenous administration.

Thrombocytosis

Finally, thrombocytosis, or an elevated platelet count, may be seen. In some


patients, platelets behave as a phase reactant in response to various
inflammatory stimuli. Examples include bacterial abcesses, osteomyelitis, and
rheumatoid arthritis. Platelets generally do not exceed 1,000,000/l. On the
PLATELET DISD Joseph E. Kiss, MD
Hematology 2009

other hand, certain myeloproliferative syndromes may be associated with very


high platelet counts (>1,000,000). In these patients, bleeding or thrombotic
manifestations may ensue which requires lowering the platelet count to safer
levels. Finally, rebound thrombocytosis may occur following splenectomy. This
is self limited and generally peaks at less than 1,000,000/l.
Workshop objectives
Evaluate four cases in order to

Enhance the understanding of anemia classification


Workshop: Hematopathology I based on pathophysiologic mechanisms

Anemias Reinforce the approach to the work-


work-up for anemia

Division of Hematopathology Learn the role of peripheral blood film findings as an


aid to establishing a differential diagnosis of anemia.
Department of Pathology
Expand the understanding of specific laboratory tests
UPMCHS used for evaluation of four cases

Case I
Classifications of Anemia The patient is a previously well 6-month-old
African American female who has become
Morphologic classification lethargic with loss of appetite, fever and cough.
Macrocytic anemia
Normocytic anemia Physical examination reveals a well-developed
Microcytic hypochromic anemia female infant.

Pathophysiologic classification The exam is notable for the following:


Blood loss Scleral icterus.
Accelerated red cell destruction Decreased breath sounds over right lower lobe and
crepitant rales in basal area.
(i.e. hemolytic anemias)
A palpable spleen tip 1 cm below left costal margin.
Decreased red cell production

Patient Normal range Reticulocytes: 20%


(6 months 2 years) Absolute retic. count: 540 x 109/L
WBC 7.0 x 109/L (6.0-17.5)
RBC 2.71 x 1012/L (3.70 - 17.5)
HGB 8.0 g/dL (10.5 - 13.5)
Hct 24 % (33.0 39.0)
MCV 80 fl (70.0 - 86.0) What does this tell us about the cause of
RDW 22 % (11.5 - 15.0)
PLT 550 x 109/L (150 - 450) anemia in this patient?

Differential:
% Absolute Absolute Normal Range
(6 months - 2 years)
What would be the morphologic correlate
Neutrophils 50% 3.50 x 109/L (1.00 - 8.50)
Lymphs 45% 3.15 x 109/L (4.00 - 13.50) on the peripheral blood film?
Monos 3% 0.21 x 109/L (0.05 - 1.10)
Eos 2% 0.14 x 109/L (0.05 - 0.70)

1
Polychromatophilia Enumeration of reticulocytes
Polychromatophilic
Manual
red cells are the least Residual RNA /organelles
mature circulating red are stained
Reticulocytes manually
cells counted on a glass slide

Automated counts by flow


Not all reticulocytes cytometry
are visualized using Fluorescent dye binds RNA
Wright-Giemsa stain Dye excited by laser light;
then reticulocytes
enumerated based on their
fluorescent emission

What general laboratory tests are useful


to detect a hemolytic anemia? While you are waiting for any
other additional tests (e.g.
Lactate dehydrogenase High in our
patient Coombs test) to come back, you
Haptoglobin Low in our patient notice that the CBC report comes
Serum bilirubin High in our patient back with some abnormalities
noted for red cell morphology.
Very severe intravascular hemolysis can also
result in hemoglobinemia, hemosiderinuria, and So you go down to view the slide yourself.
The laboratory technologist helps you to
hemoglobinuria. focus on the correct area of the slide
More specific laboratory tests are then indicated

Peripheral blood film preparation review: Peripheral blood film preparation:


Wedge technique Wedge technique

2
Peripheral blood film preparation:
Wedge technique
Direction of spread

Point of application Feathered end


of blood droplet is thin
Area of optimal
Area too thick
thickness for
examination

Sickle Cell Anemia


What is the genetic mutation?

Is this disorder rare?

What is the difference between a


hemoglobinopathy and a thalassemic
syndrome?

Hb Electrophoresis, (-) (+)


Specific laboratory testing in Alkaline (pH 8.6)
sickle cell disease
Since HbS, HbD,
HGB electrophoresis and HbG all migrate
to the same
High performance liquid position, what
should the lab do
chromatography (HPLC) next?

Prenatal analysis of fetal DNA

3
Hb Electrophoresis, (-) (+)
Hemoglobin analysis by HPLC
Acid (pH 6.2)

Is this patient
homozgous or
heterozygous for
Hb S?

Could there be another


explanation for the
same findings on
hemoglobin
electrophoresis?

Schrier, S. ASH Image Bank 2001;2001:100248


Copyright 2001 American Society of Hematology. Copyright restrictions may apply.

Gram stain of sputum sample


Sickle Cell Anemia
Septicemia and meninigitis by What would be the expected morbidity
encapsulated organisms are the and mortality?
most common causes of death in
children with sickle cell anemia. What curative therapy is available for
this disorder?

Which hemolytic anemias due to intrinsic red cell


What are other examples of hemolytic anemias due to
abnormalities are associated with these poikilocytes?
intrinsic red cell abnormalities?

Hereditary: Acquired:
Other hemoglobinopathies Paroxysmal nocturnal
Thalassemias hemoglobinuria
Hereditary spherocytosis or
hereditary elliptocytosis
G6PD deficiency
Enzymopathies (pyruvate
kinase deficiency)
Unstable hemoglobins

Which of these would be associated with


poikilocytes that have diagnostic importance?

4
A CBC was ordered as a part of his initial workup.
Case 2
http://www.hematology.org/education/teach_case/greenberg/
Patient Normal range

The patient is a 72-year-old man with chronic WBC 100 x 109/L (3.8-10.6)
RBC 2.7 x 1012/L (4.1 5.6)
lymphocytic leukemia (CLL), diagnosed 3 years HGB 8.2 g/dL (12.9-16.9)
ago. He has received no therapy. He has Hct 25.7 % (38.0-48.8)
MCV 96 fl (83 - 97)
developed shortness of breath and lethargy over RDW 17.0 % (11.8 - 15.2)
3 months. PLT 216 x 109/L (156-369)

Physical exam reveals a male who appears his Differential:


stated age. % Absolute Absolute Normal Range

Neutrophils 3 3x 109/L (2.2-7.7)


His physical exam is notable for pallor, Lymphs 96 96 x 109/L (0.8-3.6)
lymphadenopathy, and a palpable spleen tip. Monos 1 1 x 109/L (0.3-0.9)

Which of the following might be responsible


for the new onset of anemia?
Development of autoimmune hemolytic
anemia
GI bleeding
Iron deficiency
Renal failure

What would you expect with the reticulocyte


response for these circumstances?

5
Reticulocytes: 8.6 (0.4-2.4)
Absolute retic. count 0.23 x 1012/L (0.018-0.16)

Calculate the reticulocyte production index for


this patient.

What does this tell us about the cause of


anemia in this patient?

Example of peripheral blood from a


different patient with anemia due to a similar
etiology, but without CLL

What general laboratory tests are useful


to detect a hemolytic anemia? Direct Antiglobulin Test
(Coombs test)
Lactate dehydrogenase 320 IU/L (normal
upper limit 170)
Results for our patient:
Haptoglobin Not ordered, might be normal or The DAT is positive with anti-IgG AHG.
low in our patient. Why? The DAT is negative with anti-complement
AHG.
Serum bilirubin Not ordered, likely at least
mild to moderately elevated in our patient.
An eluate study reveals a panagglutinating
More specific laboratory tests are indicated and IgG antibody
would have been ordered concurrently.

The direct antiglobulin test (DAT) and indirect antiglobulin test (IAT)
This case is due to autoimmune
hemolytic anemia (AIHA) that is
warm antibody mediated

Warm-Reacting Autoantibodies (80-90% of


AIHA)
Autoantibody maximally active at body
temperature (37C)
RBCs typically coated with IgG, +/-
complement proteins

Zarandona, J. M. et al. CMAJ 2006;174:305-307

Copyright 2006 CMA Media Inc. or its licensors

6
Follow-up Pathophysiologic classification of
anemia
Patient treated with corticosteroids with
excellent response.
Blood loss
After a year, his CLL transformed to a diffuse
large cell lymphoma. Accelerated red cell destruction /
hemolytic anemias
He had a transient response to chemotherapy (e.g. cases 1 & 2)
(CHOP), but then progressed over 3 months
and died of disease.
Decreased red cell production

schistocytes
What are other examples of hemolytic anemias
due to abnormalities extrinsic to the red cells?
Immune-mediated
Autoimmune
Transfusion reactions or maternal-fetal
incompatibility
Trauma
Microangiopathic hemolytic anemia
Mechanical
Infection ( e.g. malaria)
Hypersplenism

Which of these is associated with a


poikilocyte that has great diagnostic importance?

Case 3
The patient is a 61-year-old white male
Non-specific who, over the last 6 months, has had night
poikilocyte sweats, fatigue, myalgias, loss of appetite
and 20-pound weight loss. Episodic
bilateral blurring of vision.

On physical exam he is well-appearing, but


thin. Of note, he has slight tenderness in
temporal regions. Exam reveals some
limitation on musculoskeletal examination
schistocytes due to pain.

7
A CBC was ordered as a part of his initial workup.
Reticulocytes: 0.5% (0.4-2.4)
Patient Normal range
Absolute retic. count: 0.019 x 1012/L (0.018-0.158)
WBC 7.3 x 109/L (3.8-10.6)
RBC 3.8 x 1012/L (4.13-5.57)
HGB 9.8 g/dL (12.9-16.9)
Hct 29.1 % (38.0-48.8)
MCV 76.1 fl (82.6-97.4)
RDW 15.6 % (11.8-15.2) Calculate the reticulocyte production index for
PLT 493 x 109/L (156-369) this patient.

What does this tell us about the cause of


Differential: anemia in this patient?
% Absolute Absolute Normal Range

Neutrophils 73 % 5.4 x 109/L (2.2-7.7)


Lymphs 16 % 1.1 x 109/L (0.8-3.6)
Monos 8% 0.6 x 109/L (0.3-0.9)
Eo 2% 0.1 x 109/L (0.0-0.4)
Baso 1% 0.0 x 109/L (0.0-0.1)

Microcytic anemias
Iron deficiency anemia

Thalassemias

Anemia of chronic disease


Normocytic 80%
Microcytic 20%

Sideroblastic anemias
Normal Patient
Patient

Additional testing

Ferritin 300 ng/mL (10-282)


Serum iron 30 ug/dL (65-165)
TIBC 250 ug/dL (250-420)
Transferrin saturation 12 % (25-50)

ESR 84 mm/hr (0-20)


CRP 17.4 mg/dL (0-0.74)

Normal iron stores and normal sideroblast for comparison

8
Anemia of inflammation
(a.k.a. anemia of chronic disease)

Is this a common contributing factor for anemia?

Review the key pathophysiology.

What laboratory features will differ from iron


deficiency anemia?

What is the therapy?


Increased iron stores

Anemia of Inflammation:
Normal internal iron cycle Dysregulation of Iron Homeostasis
Erythrocytes Erythrocytes
(Blood) (Blood)

Bone marrow Ineffective Bone marrow TNF-


Phagocytes (erythroblasts) Phagocytes
(erythroblasts) erythropoiesis IL-1
IL-6

Plasma Plasma
(transferrin iron) (transferrin iron)

Duodenal Stores (Ferritin Duodenal Stores (Ferritin


& Hemosiderin)
Absorption Absorption & Hemosiderin)

Iron Anemia of
deficiency inflammation Patient follow-up
anemia
MCV Low Normal or low
RDW Increased Normal (typically) Diagnosis: Giant cell arteritis and
Serum iron Low (typically) polymyalgia rheumatica.
TIBC High Low or normal
Ferritin Low (typically) High (typically) The patient has been treated with
Transferrin Low Normal or low prednisone.
saturation Markers of inflammation have normalized and
Stainable Absent Increased (typically) his symptoms improved dramatically.
iron in The anemia resolved.
resolved
marrow

9
Case 4 A CBC was ordered as a part of his initial workup.

Patient Normal range


The patient is a 52-year-old male with
history of alcoholism and poor diet. He WBC 3.8 x 109/L (3.8-10.6)
RBC 0.70 x 1012/L (4.13-5.57)
has developed fatigue and shortness of HGB 3.1 g/dL (12.9-16.9)
breath over 2-3 months. Hct 9.1 % (38.0-48.8)
MCV 130 fl (82.6-97.4)
RDW 25 % (11.8-15.2)
PLT 54 x 109/L (156-369)
Physical exam reveals a male who
appears his stated age.
Differential:
% Absolute Absolute Normal Range

His physical exam is notable for a beefy Neutrophils 50 % 1.9 x109/L (2.2-7.7)
1.1 x 109/L
red smooth tongue. Lymphs
Monos
30 %
15 % 0.6 x 109/L
(0.8-3.6)
(0.3-0.9)
Eo 3% 0.1 x 109/L (0.0-0.4)
Baso 2% 0.1 x 109/L (0.0-0.1)

Reticulocytes: 0.5% (0.4-2.4) Macrocytic Anemias


Absolute retic. count: 0.019 x 1012/L (0.018-0.158) Megaloblastic
Vitamin B12 or folate deficiency
Myelodysplastic syndromes
What does this tell us about the cause
Drug therapy (e.g. methotrexate)
of anemia in this patient?
Non-megaloblastic
Alcohol abuse
Liver disease
Hypothyroidism
Drugs
Reticulocytosis (2 to hemolysis or hemorrhage)

1000 X 1000 X

10
1000 X Normal patient
erythropoiesis

1000 X

1000 X 1000 X Normal granulocytopoiesis Patient

Patient bone
marrow aspirate

Band
1000 X

40 X

Megaloblastic anemia
Why is it important to distinguish anemia due
to folate deficiency from cobalamin
deficiency?

What laboratory features will differentiate


anemia due to folate deficiency from
cobalamin deficiency?

What is the pathophysiology of megaloblastic


anemia?

11
Why is it important to distinguish folate
B12 deficiency Folate deficiency
deficiency from cobalamin deficiency?

Cobalamin deficiency may lead to neurologic Serum folate Normal/ Low/normal


dysfunction, possibly irreversible high
Red cell folate Low/ Low
normal
Folic acid supplementation will correct
hematologic abnormalities and mask Serum cobalamin Low/ Normal/low
vitamin B12 deficiency. normal
Methylmalonic acid High Normal

If there is any doubt, cobalamin


supplementation should be administered Serum homocysteine High High
along with folate supplementation.

Relationship between folate,


folate cobalamin and Additional testing
DNA synthesis
Serum B12 200 (211-911 pg/mL)

Serum folate 1.9 (>5.4 ng/mL)


Needed for
DNA
synthesis Will accumulate with either RBC folate 200 (293-809 ng/mL)
folate or cobalamin deficiency

Homocysteine 115 (5-15 umol/L)


(serum)
Methylmalonic acid 200 (73-271 nM/L)
(serum)
LDH 6410 (<170 IU/L)

Required as an intermediate for Total bilirubin 2.9 (0.3-1.5 mg/dL)


demethylation of 5-Methyl THF
Main form transported in plasma

Pathophysiologic classification Anemias due to diminished erythropoiesis


of anemia
Decreased RBC precursors

Blood loss Decreased stimulus for erythropoiesis


E.g. Anemia of chronic disease
Stem cell defects
E.g. Aplastic anemia
Accelerated red cell destruction / Marrow replacement
E.g. Malignancy or storage disease
hemolytic anemias
Ineffective erythropoiesis

Decreased red cell production Megaloblastic anemia (B12 and folate deficiency)
Iron deficiency
(e.g. cases 3 & 4) Myelodysplastic syndromes (MDS)

12
Hematopathology I: Anemias Hematopathology I: Anemias
Key teaching points Key teaching points
The pathophysiologic classification of anemia
promotes a sound approach to diagnosis of a wide True
True schistocytes herald microangiopathic hemolytic
variety of causes for anemia. anemia.
Possibly, a medical emergency.
The reticulocyte count is useful initial discriminator
between two fundamental causes of anemia. Understanding iron metabolism facilitates the distinction
of anemia of chronic disease from iron deficiency anemia.
Sickle cell disease is a common hemoglobinopathy
affecting persons of African descent that has substantial Proper distinction between folate deficiency and cobalamin
morbidity and mortality. deficiency has critical implications.
Folic acid therapy corrects anemia due to cobalamin
deficiency, but neurologic effects will progress
The diagnosis of autoimmune hemolytic anemia is
based on clinical presentation and a serologic work-up.

13
TOPIC: CASE CONFERENCE I - ANEMIAS
(SMALL GROUP)

DATE: January 8, 2009

TIME: 8:00-10:00 AM

Student Groups Small Group Rooms Facilitators

Group 65&66 Rooms 502 & 503 Patricia Kropf, MD

Group 67&68 Rooms 504 & 505 Margaret Ragni, MD, MPH

Groups 69&70 Rooms 506 & 507 Frank Bontempo, MD

Groups 71&72 Rooms 508 & 509 Joseph Kiss, MD

Groups 73&74 Rooms 510 & 511 Robert Redner, MD

Groups 75&76 Rooms 512 & 513 Suzanne Lentzsch, MD, PhD

Groups 77&78 Rooms 514 & 515 Roy Smith, MD

Groups 79&80 Rooms 516 & 517 Peter Shaw, MD

1
Case # 1
A 36 year old male presents to the Emergency Department complaining of progressive
weakness and fatigue. The patient states that the onset of fatigue began six months
previously. He denies any chest pain with exertion. He denies any prior history of
pulmonary problems or anemia.

His past medical history is remarkable for a gun shot wound to the abdomen six years
previously. Because of the intra-abdominal organ damage, several segments of his
small and large intestine were resected. He is presently taking no medication and has
no allergies. Aside from the gun shot wound his past medical history is otherwise
unremarkable.

He admits to a heavy alcohol consumption in the past drinking a case of beer daily but
is drinking only six beers per day now. He states that his diet includes meats and
vegetables and denies a history of chronic diarrhea. The patients family history is non-
contributory.

In the review of symptoms, he denies any jaundice, melena or hematochezia. There


has been no exposure to toxins. Finally, he denies any paresthesias or neurologic
symptoms.

Physical examination: P: 120 and regular; BP 90/40 without orthostatic change; RR:
18; Temp: 36.8C. HEENT: mild scleral icterus and pale conjunctivae; smooth
appearing tongue. Cardiopulmonary: clear lung fields; RRR with a tachycardia but no
murmurs or gallops. Abdomen: non-tender, liver edge is noted 3 cm below the right
costal margin with a span of 10 cm by percussion; no spleen tip is palpable; there is a
midline scar extending from the xyphoid process to the pubic region; stool is negative
for blood Extremities: several ecchymoses are noted in the lower extremities.
Neurologic: alert and oriented X3; cerebellar testing is normal; motor strengths are 5/5
symmetrically; sensory exam reveals diminished vibratory sensation in the lower
extremities; deep tendon reflexes are brisk bilaterally in the lower extremities; cranial
nerves are intact.

Laboratory testing reveals the following:

Result Normal
Hemoglobin (Hb) 3 g/dL 12-14 g/dL
Hematocrit (HCT) 10% 36-42%
Mean Corpuscular Volume (MCV) 140fL 80-100fL
RDW 20% 12-15%
White blood cells 2.5 3.5-10 x 109/L
Neutrophils 66% 44-77%
Lymphocytes 25% 13-44%
Monocytes 7% 4-13%
Eosinophils 1% 0-6%

2
Basophils 1% 0-1%
Platelets 50 150-300 X 109/L

Reticulocyte count: 3% 1-2%


Serum B12: 25 200-900 pg/ml
Serum folate: >20 >5.4 ng/ml
Red cell folate 700 300-800 ng/ml
LDH 2000 <170 IU/L
Total bilirubin 3.5 0.3-1.5 mg/dL
Indirect bilirubin 3.0 0-0.3 mg/dL
Haptoglobin <9 16-200 mg/dL

Review of the peripheral blood smear showed red cells: oval macrocytes, granulocytes:
hypersegmentation of the nuclei

Questions for discussion:

Question # 1: How would you characterize his anemia?


Question # 2: What are the causes of macrocytic anemia?
Question # 3: Why did it take six years for the patient to become
deficient in B12?
Question # 4: What are the causes of B12 deficiency?
Question # 5: What are the causes and work-up of
thrombocytopenia?
Question # 6: What are the causes of pancytopenia?

3
Case # 2

A 15 year old girl is brought to the Emergency Department at Childrens Hospital due to
the new onset of bruising.
The patient had been in excellent health all of her life until one week before presentation
to the ED when she developed nausea and frequent diarrhea within 24 hours of eating
at a fast food establishment. Several of her friends also had become ill after the meal.
Two days later she began to notice increased bruising when she bumped into furniture.
She has had intermittent bleeding from her gums while brushing her teeth. Concurrent
with these symptoms she noted some fatigue with exertion.
Past medical history is negative. She is a healthy adolescent female who runs cross
country for her high school. She denied use of alcohol or tobacco products. She eats a
normal American diet consisting of meats and vegetables.

On physical examination her vital signs were remarkable for a blood pressure of 160/90
and a pulse of 120. Remarkable findings of exam included:

SHEENT: multiple ecchymoses and petechiae on her arms and legs; scleral
icterus and pale conjunctivae.
Cardiopulmonary: clear lungs; tachycardia with a regular rhythm
Abdomen: no hepatosplenomegaly, tenderness or masses; stool is negative for
occult blood.

Initial laboratory findings included the following:

Normal
Hemoglobin (Hb) 6 g/dL 12-15 g/dL
Hematocrit (HCT) 18% 34-43%
MCV 110fL 80-100fL
RDW 25% 12-15%
Platelets 50 150-300 X 109/L

Normal

Prothrombin time (PT) 11 sec 10-12 sec


Activated partial thromboplastin time (aPTT) 32 sec 26-37 sec

Review of her peripheral blood smear showed red cell fragments (schistocytes) and
polychromasia. Additional labs ordered revealed an increased serum creatinine and
blood urea nitrogen (BUN). Urinalysis demonstrated hematuria and red blood cells.

4
Questions for discussion

Question # 1: What additional laboratory testing are needed to confirm


hemolysis and to determine the cause ?

Question # 2: What is your differential diagnosis ?

Question # 3: What is the treatment for this patient ?

5
Comments Case # 1:
Pt anemia is : macrocytic (MCV >100), hypoproliferative [reported reticulocyte count 3%,
corrected reticulocyte count [his HCT (10)/ideal HCT for males (45) = 0.22 X 3% = corrected
reticulocyte count of 0.67%] anemia.

Causes of macrocytic anemia?


Folate deficiency, B12 deficiency
Reticulocytosis in response to bleeding or hemolysis, reticulocytes, which have a higher
MCV than mature RBC, may produce an increase in MCV (generally <110 fL)
Liver disease with severe liver disease, abnormalities in membrane lipids generate
macro-ovalocytes; MCV is generally <120fL
Myelodysplasia (MDS) in MDS nuclear division is affected resulting in mild
macrocytosis (MCV <110fL); in contrast to folate or cobalamin deficiency
hypolobulated (not hypersegmented) neutrophils are observed.

Patients may have macrocytosis independent of an anemia. This is commonly seen in patients on
medications that affect nuclear division (anti-retrovirals or chemotherapy agents such as
hydroxyurea) or patients with liver disease (due to alteration of cholesterol and phospholipid
content of the RBC membrane).

There was an insidious (i.e., pernicious) onset of his symptoms of fatigue over the preceding 6
months suggesting a gradual evolution of his anemia. This would tend to rule out acute blood
loss or hemolysis. Most significantly he had been shot previously and sections of his intestines
were resected, a major site of B12 absorption. B12 is stored in the liver and also undergoes
enterohepatic circulation (re-circulates between the intestines and liver). The minimal daily
requirement is low (5 micrograms) and there is 2-5 milligrams stored within the liver. On
average it takes >5 years of eating a diet lacking B12 to become deficient. The gun shot wound
six years previously followed by the resection of his terminal ileum suggests the correct time
course.

Causes of B12 deficiency include


defective release of B12 from food Gastric acid is necessary to separate dietary B12 from
proteins secreted by saliva. In older individuals or subjects taking drugs that decrease stomach
acid (eg, histamine receptor antagonists or proton pump inhibitors) may be unable to free dietary
B12 for binding to IF.
pernicious anemia Lack of parietal cells, either from atrophic gastritis or secondary to
autoimmune causes, produces a deficiency of IF. Autoimmune causes include the presence of
anti-parietal cell (90%) or anti-IF (50%) antibodies in the serum. This is seen in younger
subjects with PA and is often seen in conjunction with other autoimmune disorders (Graves
disease, Hashimotos thyroiditis).
postgastrectomy lack of IF
intestinal organisms Blind intestinal loops or diseases associated with pseudo-obstruction
(diabetes, amyloid, scleroderma) contribute to bacterial overgrowth which competes for dietary

6
B12. In Scandinavia, a fish tapeworm (Diphyllobotrium latum) competes (favorite board
question).
ileal abnormalities Abnormalities at the terminal ileum (sprue, non-tropical sprue, lymphoma,
TB, surgical resection) alter B12 absorption.

Thrombocytopenia can be caused by several different mechanisms:


Decreased production lack of platelet precursors (amegakaryocytic thrombocytopenia,
aplastic anemia); lack of elements needed for production (B12 or folate)
Increased destruction or consumption: immune thrombocytopenia, non-immune
thrombocytopenia (e.g., thrombotic thrombocytopenic purpura, hemolytic/uremic
syndrome, DIC), bleeding
Sequestration: splenomegaly

In addition, thrombocytopenia can be an artifact induced by clumping of platelets when


blood is drawn into tubes containing EDTA (pseudothrombocytopenia) or can result from
massive (~10 units) transfusion of RBCs (dilutional thrombocytopenia). In our case it was due
to lack of production.

Pancytopenia can be caused by:

Problems with stem cells: eg, Aplastic anemia: deficiency of stem cells due to an
intrinsic defect within the stem cells (e.g., Fanconi syndrome) or due to an extrinsic
assault on the stem cells (drugs, radiation, immunologic attack).
Problems with the marrow microenvironment: Myelophthisis: infiltration of the marrow
by fibrosis, tumors (leukemia, lymphoma, solid tumors), granulomas, storage diseases.

7
Comments Case # 2:
Mechanisms of hemolysis.

There are several systems used to delineate causes of hemolysis. One system relies on
categorizing diseases by the site of destruction (intravascular vs extravascular). However,
intravascular causes are generally uncommon [transfusion reactions, paroxysmal nocturnal
hemoglobinuria, (PNH)]. One system that is easy to remember divides causes into those that are
extrinsic and affect normal RBCs and those that are intrinsic to defects within the RBCs, per se.
Extrinsic causes can be divided again into two groups: those that are immunologically
mediated and those that are non-immunologically mediated.
An evaluation of the peripheral blood smear is important for discerning the cause of the
hemolytic anemia. If microspherocytes are observed in an individual with an acquired
hemolytic anemia, a warm antibody (IgG-mediated) directed against an RBC antigen is the
likely cause. Spherocytes are formed as portions of the RBC surface that have IgG attached are
removed by the macrophages within the spleen. If red cell agglutination is observed on the
smear, a cold antibody (IgM-mediated) is the etiology. IgM antibodies are pentamers and each
variable region is capable of binding a RBC; this leads to agglutination. If fragmented RBCs
(schistocytes) are observed, a non-immunologic mechanism is responsible for the hemolysis.
Schistocytes are formed by shearing of the RBCs on fibrin strands within the vasculature, on
mechanical cardiac valves or between the bones of the feet (march hemoglobinuria in soldiers).
The test to confirm an immunologic cause of hemolysis is a Coombs Test. This test uses
a secondary antibody (goat/rabbit) that reacts with human immunoglobulin or complement on
the patients RBCs (direct Coombs). Conversely, screening the patients serum for the presence
of antibodies that bind to donor RBC (indirect Coombs) can be done in situation of remote
hemolysis (e.g., delayed transfusion reactions).

Non-immunologically mediated, extrinsic causes is the category in which schistocytes


are observed in the blood smear. Conditions associated with the generation of a
microangiopathic (ie, fragmented RBCs) anemia include:
Disseminated intravascular coagulation (DIC)
Hemolytic/uremic syndrome (HUS)
Thrombotic thrombocytopenic purpura (TTP)
Hemolysis, elevated liver functions and low platelets (HELLP syndrome)
Vasculitis
Valves
Trauma

Disorders classified as being intrinsic to the RBC are largely congenital, although
paroxysmal nocturnal hemoglobinuria (PNH) is acquired. This category of hemolytic anemias
can be subdivided into disorders involving the cell membrane, the metabolic pathways and
hemoglobin.

8
Disorders affecting the red cell membrane:
Hereditary spherocytosis (HS) and hereditary elliptocytosis (HE)
Paroxysmal nocturnal hemoglobinuria (PNH)

Disorders within the metabolic pathways:


Glucose-6-phosphate dehydrogenase deficiency (G6PD)
Pyruvate kinase deficiency

Disorders of hemoglobin:
Qualitative disorders of hemoglobin: The most frequent mutations of hemoglobin in
America are those that produce Hb S (sickle cell) and Hb C.
Quantitative disorders of hemoglobin: alpha and beta thalassemias.

Laboratory testing used to confirm hemolysis and to determine the causes.

In addition to obtaining a reticulocyte count, other labs commonly requested to begin the
evaluation of possible hemolysis include: haptoglobin, LDH and bilirubin (total and indirect)
levels. Haptoglobin is an 2 globulin synthesized by the liver. The function of haptoglobin is to
bind free hemoglobin in the blood. The haptoglobin/hemoglobin complex is cleared by the liver.
However, if this clearance mechanism is saturated, the haptoglobin/hemoglobin complex may be
filtered by the kidneys. Haptoglobin is an acute phase reactant; hence, haptoglobin will increase
during inflammation and conversely will be decreased in patients with end stage liver disease.
Lactic dehydrogenase (LDH) is a ubiquitous enzyme present in all cells and has five isoforms.
While LDH-2 specifically is elevated during hemolysis, a routine, total LDH is the test ordered
to assay hemolysis. Breakdown of the heme ring leads to the generation of bilirubin which is
conjugated with glucuronides within the liver. With hemolysis the total bilirubin is elevated and
the conjugation mechanism is often overwhelmed. This leads to an increase in the proportion of
unconjugated (indirect) bilirubin.
An examination of the peripheral blood smear is imperative in the evaluation of anemia and
specifically hemolytic anemias. If spherocytes or red cell agglutination is observed, a Coombs
test should be ordered. If fragmented red cells (schistocytes) are seen, a microangiopathic
hemolytic anemia may be present. Examining the smear may also demonstrate target cells or
sickled cells suggesting a hemoglobinopathy.

Thrombotic thrombocytopenic purpura (TTP) is defined clinically by the abnormalities


caused by systemic thrombotic microangiopathy: thrombocytopenia and microangiopathic
hemolytic anemia. Additional clinical features may include neurologic abnormalities, renal
failure, and gastrointestinal symptoms.
Hemolytic-uremic syndrome (HUS) is another clinical presentation of thrombotic
microangiopathy. Like TTP, HUS is manifested by thrombocytopenia and microangiopathic
hemolytic anemia with the additional abnormality of renal failure. Although it is commonly
stated that HUS is manifested primarily by renal failure whereas TTP is manifested primarily by
neurologic abnormalities, these two syndromes cannot be distinguished clinically because many
patients have both renal failure and severe neurologic abnormalities, or neither. Often these
conditions are referred to by the comprehensive term, TTP-HUS

9
The diagnosis of TTP is based on the observation of thrombocytopenia and microangiopathic
hemolytic anemia without another clinically apparent cause. In rare instances, even these two
cardinal features may not be present, as for example, patients with previously diagnosed episodes
of TTP who subsequently have acute neurologic symptoms without thrombocytopenia or anemia
and are documented to have severe ADAMTS13 deficiency. Classically, TTP was diagnosed by
the pentad of clinical features: thrombocytopenia, microangiopathic hemolytic anemia,
neurologic abnormalities, renal insufficiency, and fever. However, in the current era, urgency of
diagnosis is required to initiate effective treatment. Therefore, only thrombocytopenia and
microangiopathic hemolytic anemia without another clinically apparent cause are sufficient to
establish the diagnosis. The differential diagnosis includes all conditions associated with the
clinical features of TTP

Systemic infections
Systemic fungal infections. Aspergillosis and other angioinvasive fungi can cause all
clinical features of thrombotic microangiopathy
Viral infections. Disseminated CMV infection can cause all clinical features of
thrombotic microangiopathy. HIV infection can also mimic TTP, typically related to
additional opportunistic infections
Rickettsial infections. For example, Rocky mountain spotted fever
Bacterial sepsis, especially bacterial meningitis

Systemic malignancy
Disseminated micrometastatic malignancies may mimic all clinical features of TTP without
evidence by imaging studies. Although disseminated intravascular coagulation can occur in
patients with disseminated malignancy, systemic small vessel metastases causing obstruction
and thrombosis can occur without evidence of DIC. A syndrome mimicking TTP may occur
with breast cancer, pancreatic cancer, gastric cancer, and non--small cell lung cancer
Complications of pregnancy
Severe preeclampsia and the HELLP syndrome may mimic all clinical features of TTP

Malignant hypertension
Severe hypertension may cause all clinical features of TTP including thrombocytopenia,
severe microangiopathic hemolysis, renal failure, and acute central nervous system
abnormalities

Autoimmune disorders
Patients with acute systemic symptoms related to systemic lupus erythematosus,
antiphospholipid antibody syndrome, acute systemic sclerosis, polyarteritis nodosa, and other
autoimmune disorders can have all clinical features of TTP

Plasma exchange treatment is the key element for management of TTP. It is the one treatment
with documented effectiveness by a randomized controlled clinical trial that compared plasma
exchange with plasma infusion. Plasma exchange is urgently indicated in all patients with a
clinical diagnosis of TTP. Although there are no data that clearly support the efficacy of

10
glucocorticoids, they are commonly given in addition to plasma exchange since acquired TTP is
commonly thought to have an autoimmune etiology. Many other immunosuppressive agents
have been used in the treatment of TTP, such as cyclophosphamide and vincristine. These are
used in patients with disease refractory to plasma exchange and glucocorticoids. Recently,
rituximab has been frequently used to induce durable remissions in patients with severe and
prolonged courses involving multiple exacerbations and relapses

ADDITIONAL INFO:

Know the initial algorithm used to determine the causes of anemia.

The classification of anemias relies on the assessment of the reticulocyte response. Patients with
anemia and high reticulocyte responses (>3%) have the capacity to respond to the cause of the
anemia which is usually either blood loss or hemolysis. Subjects with anemia and a suboptimal
reticulocyte response (<2%) have a hypoproliferative response to their anemia. For the latter
category, checking the MCV then divides the causes into microcytic anemias (eg, IDA,
thalassemia), macrocytic anemias (eg, B12 or folate deficiency), and normocytic anemias (eg,
renal disease, chronic inflammation).

Anemia

check retic count

<2% >3%

Check MCV

Hemolysis Bleeding

<80fL 80-100 fL >100 fL


Microcytic Normocytic Macrocytic

11
Understand the role of reticulocytes in the differential diagnosis of anemia.

Reticulocytes are the penultimate stage in red cell development. They are anucleated cells that
are stored within the marrow for 2-3 days prior to being released into the blood stream where
they mature for another day. Reticulocytes are larger than mature RBCs and continue to
synthesize hemoglobin using polyribosomes. On routine blood smear staining they are large
cells with a bluish coloration (polychromasia).
In patients with anemia, requesting a reticulocyte count is essential to discern the
compensation (appropriate or inappropriate) for the degree of anemia. A normal reticulocyte
count in a non-anemic patient is 1-2%; hence, the average survival of RBCs is 100-120 days.
The reticulocyte count can also be obtained by using a automated cell counter where the absolute
number of reticulocytes among 10,000 RBCs counted can be obtained (normal is 1% of 5 X 106
RBC/mm3 or ~50,000 (0.05 X 106 reticulocytes/mm3).

If the reticulocyte count is performed manually, the initial value is adjusted for the degree of
anemia. This adjustment is made due to the number of red cells/HPF that the technician is
counting (less in someone who is anemic). The reported hematocrit of a patient (e.g., 20%) is
divided by the ideal hematocrit for that individual (female = 40%) to generate a fraction (0.5)
used to multiple the reported reticulocyte count (3% X 0.5 = 1.5%). This is a corrected
reticulocyte count and demonstrates that the patients response to her anemia is inappropriately
low.
A second correction of the reported reticulocyte count is often done when the manual method
is used. In situations of severe anemia, the reticulocytes may be pushed (shifted) into circulation
earlier than normal causing the final stages of maturation to occur in circulation. These cells are
larger and more blue and are called shift cells. Since these cells may continue to mature for two
days (vs the normal 1d) in circulation, the corrected reticulocyte count is divided by 2 if shift
cells are observed in the routine smear. This adjustment in the reported reticulocyte count
generates the reticulocyte index which is used to classify anemias (>3% appropriate; <2%
inappropriate/hypoproliferative).

Know the common red cell morphologies.

Abnormalities of Red Cell Shape (Poikilocytosis)

Acanthocytes (Spur cells): 2-10 pointed projections of various lengths and irregular spacing.
Found in: abetalipoproteinemia, alcoholic liver disease, post-splenectomy, malabsorption states.

Echinocytes (Burr cells): 10-30 pointed projections evenly distributed around the cell. Found
in: commonly see in uremia but also present in pyruvate kinase deficiency.

Elliptocytes: narrow, elongated red cells resembling a cigar or rod. Found in: hereditary
elliptocytosis, thalassemia, iron deficiency, myelophthistic anemias.

Schistocytes: fragmented red cells. Found in: microangiopathic hemolytic anemias (TTP/HUS;
DIC), prosthetic heart valves, march hemoglobinuria.

12
Sickle cells: crescent-shaped red cells with at least one end that is pointed Found in: sickle
disorders (sickle cell anemia (SS), Hb SC).

Spherocytes: compact and round cells without central pallor and darkly stained. Found in:
hereditary spherocytosis, warm antibody-mediated immune hemolytic anemia.

Target cells: contains central bulls eye surrounded by a clear ring and an outer red ring. Found
in: obstructive liver disease, hemoglobinopathies (S, C), thalassemia, post-splenectomy, lecithin
cholesterol acetyl transferase (LCAT) deficiency, iron deficiency.

Tear drop cells: resembles a tear with one elongated end. Found in: myelofibrosis with myeloid
metaplasia; myelophthistic anemias; thalassemias

Red cell inclusion bodies

Basophilic stippling: multiple, uniform dark dots distributed evenly throughout the cell.
Represent altered RNA.

Heinz body: invisible on routine staining (Wrights stain). With supravital stain (new methylene
blue or brilliant cresyl blue show up as dark dots. Represents denatured, precipitated hemoglobin
in unstable hemoglobinopathies or thalassemias.

Howell-Jolly body: single or occasionally double purple round bodies that represent nuclear
remnants.

13
LECTURES:
I. Overview of Coagulation VII/TF XII OVERVIEW OF COAGULATION
II. Hereditary and Acquired Disorders of Coagulation
XI
OBJECTIVES:
IX
1. To understand the mechanisms of primary and secondary hemostasis
2. To recognize clinical and laboratory aspects of coagulopathies
VIII (v)

X _______________________________________
KEY WORDS: Hemostasis, coagulation factors, platelets, vessel wall
V v v
SUGGESTED READING: v
II ___________________________ vvvvvv_____
1. Furie B, Furie BC. Mechanisms of thrombus formation. NEJM 2008; 359: 938-49.
2. Mannucci PM, Tuddenheim EGD. The Hemophilias from royal genes to gene
I Ia (Fibrin)
therapy. NEJM 2001: 344: 1773-9.
Von Willebrand factor = v
3. Nichols WL, et al. von Willebrand disease: evidence-based diagnosis and Platelet =
management guidelines, NHLBI Expert Panel Report. Haemophilia 2008; 14: 171-232.
Fibrin clot =
4. Ragni MV, Lozier JN. Clinical Aspects and Therapy of Hemophilia, Chap. 125, in Hematology:
Basic Principles and Practice, Hoffman R et al eds., 5th edition, Churchill Livingstone, 2008.
5. James A, Ragni M, Picozzi V. ASH Special Educational Symposium: Bleeding Disorders in
Women: another public health crisis for hematology. Hematology 2006; 171-95.

NORMAL HEMOSTASIS

System of interactions between:

I. Overview of Coagulation 1. BLOOD VESSEL WALL


2. PLATELETS
3. COAGULATION FACTORS

Assures integrity of circulatory system after vessel injury

NORMAL AND PATHOLOGIC NORMAL AND PATHOLOGIC


CLOT FORMATION CLOT FORMATION

_______________________________________ _______________________________________

_______________________________________ _______________________________________

Vessel Wall Vessel Wall


Endothelial Cells
NORMAL AND PATHOLOGIC NORMAL AND PATHOLOGIC
CLOT FORMATION CLOT FORMATION

_______________________________________ _______________________________________

_______________________________________ ___________________________ / / _____

Vessel Wall
Endothelial Cells Vessel Injury

NORMAL AND PATHOLOGIC NORMAL AND PATHOLOGIC


CLOT FORMATION CLOT FORMATION

_______________________________________ _______________________________________

v v v v
v v
___________________________ / / _____ ___________________________ vvvvvv_____

Von Willebrand factor = v Von Willebrand factor = v

NORMAL AND PATHOLOGIC VII/TF NORMAL AND PATHOLOGIC


CLOT FORMATION CLOT FORMATION

_______________________________________ _______________________________________

v v v v
v v
___________________________ vvvvvv_____ ___________________________ vvvvvv_____

Von Willebrand factor = v Von Willebrand factor = v


platelet = platelet =
VII/TF NORMAL AND PATHOLOGIC VII/TF NORMAL AND PATHOLOGIC
CLOT FORMATION CLOT FORMATION

_______________________________________ X _______________________________________

v v v v
v v
___________________________ vvvvvv_____ ___________________________ vvvvvv_____

Von Willebrand factor = v Von Willebrand factor = v


platelet = platelet =

VII/TF NORMAL AND PATHOLOGIC VII/TF XII NORMAL AND PATHOLOGIC


CLOT FORMATION CLOT FORMATION
XI

IX

VIII (v)

X _______________________________________ X _______________________________________

V v v V v v
v v
II ___________________________ vvvvvv_____ II ___________________________ vvvvvv_____

I Ia (Fibrin) I Ia (Fibrin)
Von Willebrand factor = v Von Willebrand factor = v
platelet = platelet =

VII/TF XII NORMAL AND PATHOLOGIC VII/TF XII NORMAL AND PATHOLOGIC
CLOT FORMATION CLOT FORMATION
XI XI

IX IX

VIII (v) VIII (v)

X _______________________________________ X _______________________________________

V v v V v v
v v
II ___________________________ vvvvvv_____ II ___________________________ vvvvvv_____

I Ia (Fibrin) I Ia (Fibrin)
Von Willebrand factor = v Von Willebrand factor = v
platelet = platelet =
fibrin clot =
VII/TF XII BLEEDING DISORDERS VII/TF XII BLEEDING DISORDERS
1. Hemophilia 1. Hemophilia
XI XI 2. Von Willebrand Disease

IX IX

VIII (v) VIII (VWF)

X _______________________________________ X _______________________________________

V v v V v v
v v
II ___________________________ vvvvvv_____ II ___________________________ vvvvvv_____

I Ia (Fibrin) I Ia (Fibrin)
Von Willebrand factor = v Von Willebrand factor = v
platelet = platelet =
fibrin clot = fibrin clot =

VII/TF XII THROMBOPHILIA: VII/TF XII THROMBOPHILIA:


1. Deficiency of Clot Inhibitors 1. Deficiency of Clot Inhibitors
XI XI 2. Factor Mutations

IX IX

VIII (v) VIII (v)


Protein C, S Protein C, S
X _______________________________________ X _______________________________________

V v v V
Factor V Leiden
v v
v v
II AT-III ___________________________ vvvvvv_____ II
Prothrombin
20210
AT-III ___________________________ vvvvvv_____

I Ia (Fibrin) I Ia (Fibrin)
Von Willebrand factor = v Von Willebrand factor = v
platelet = platelet =
Clot Inhibitors

I. NORMAL HEMOSTASIS
VII/TF XII THROMBOPHILIA: A. Blood Vessel Wall
1. Deficiency of Clot Inhibitors B. Platelets
XI 2. Factor Mutations C. Coagulation System
3. Vessel Damage Inducers
IX
II. DISORDERS OF PRIMARY HEMOSTASIS
A. Von Willebrand Disease
VIII (v) B. Thrombocytopenia
Protein C, S Homocysteine LAC C. Disorders of Platelet Function
X _______________________________________
III. DISORDERS OF SECONDARY HEMOSTASIS
V
Factor V Leiden
v v A. Hemophilia
v B. Vitamin K Deficiency
II
Prothrombin
20210
AT-III ___________________________ vvvvvv_____ C. Liver Disease
D. Disseminated Intravascular Coagulation (DIC)
I Ia (Fibrin) E. Inhibitors
Von Willebrand factor = v
platelet = IV. DISORDERS OF THROMBOSIS
A. Factor V Leiden
B. Prothrombin 20210
C. Homocysteine C677T, Hyperhomocysteinemia
D. Protein C, S, Antithrombin III Deficiency
Vessel Wall

1. Reflex Vasoconstriction

2. Release of Tissue Factor

3. Contact Activation

4. Platelet Adhesion

INJURY

VASCULAR ENDOTHELIAL DAMAGE

CONTACT TISSUE FACTOR


ACTIVATION FACTOR VII
FACTORS XII, XI ACTIVATION

COMMON PATHWAY
FACTOR X

FIBRIN CLOT

Platelet Plug Formation

1. Platelet Adhesion..Denuded endothelium, VWF

2. Platelet AggregationADP, PF-4, serotonin

3. Platelet Agglutination..Release Reaction, Ca++, VWF


REGULATION OF PLATELET PLUG FORMATION

PLATELET VESSEL WALL

AA AA COAGULATION SYSTEM

PG PG Coagulation Cascade - Zymogen, Serine Protease


ENDOPEROXIDES ENDOPEROXIDES
Fibrin Clot Stabilization - FXIII
THROMBOXANE PROSTACYCLIN Fibrin Clot Retraction - Thrombasthenin
SYNTHETASE SYNTHETASE

THROMBOXANE A2 PROSTACYCLIN
(TXA2) (PGI2)

FUNCTIONS OF TXA2 FUNCTIONS OF PGI2

1. Stimulates platelet aggregation 1. Inhibits platelet aggregation


2. Vasoconstriction 2. Vasodilation

THE COAGULATION SYSTEM

INTRINSIC SYSTEM EXTRINSIC SYSTEM


Surface Contact Tissue Factor
XII XIIa VII VIIa

XI XIa
PF-3
IX IXa Ca++
PF-3
Ca++
VIII

X Xa
PF-3
Ca++
V

II IIa (Thrombin)

I Ia (Fibrin)
XIII XIIIa

COAGULATION SCREENING TESTS

1. Prothrombin Time (PT) Measures the extrinsic pathway


PRIMARY HEMOSTASIS SECONDARY HEMOSTASIS
Prolonged PT FVII deficiency
Site: Mucosal bleeding Joint, muscle, body cavity bleeding
2. Activated Partial
Onset: Early Late Thromboplastin Time (APTT) Measures the intrinsic pathway
Severity: Mild Severe Prolonged APTT FXII, XI, IX, VIII deficiency
Disorders: VWD, Platelet Disorders Hemophilia, Vitamin K Deficiency, Prolonged PT, APTT FX, F, II, I deficiency
Liver Disease, DIC
3. Thrombin Time (TT) Measures conversion of I to Ia (fibrin)

4. Reptilase Time (RT) Measures conversion of I to Ia


Prolonged TT Abnormal fibrinogen, presence of heparin
Prolonged RT Abnormal fibrinogen
.
5. Closure Time (with EPI, ADP) Measures platelet number/function
Prolonged CEPI, CADP Decreased or dysfunctional platelets
VII/TF XII CLOT FORMATION SCREENING TESTS
PT - Extrinsic Pathway
Extrinsic XI Intrinsic APTT - Intrinsic Pathway DIAGNOSIS OF BLEEDING DISORDERS
Pathway Pathway PT, APTT - Combined Pathway
PT APTT
IX Closure Time Platelet Function
TT, RT - Fibrin Formation
VIII (v) History Screening Tests
Closure
Time
Bleeding History PT
X
CEPI, CADP
Family History APTT
V v v
Combined
v
Medications Closure Time
Pathway
II PT, APTT vvvvvv Platelet Count
I Ia (Fibrin)
Von Willebrand factor = v

Fibrin
Formation
TT, RT

REVISED HYPOTHESIS OF COAGULATION


THROMBIN: KEY MULTIFUNCTIONAL ENZYME
TF

VII VIIa
IX IXa THROMBIN

VIII VIIIa INITIATING

X Xa
V Va I Ia PROTEIN C APC
II IIa XIII XIIIa VIII VIIIa
I Ia
PLATELET V Va
SUSTAINING AGGREGATION

COAGULATION SCHEMA

INJURY
CLOT INHIBITORS
VASOCONSTRICTION DAMAGED TISSUE FACTOR
VESSEL

COLLAGEN 1. ANTITHROMBIN-III. IIa, Xa, IXa, XIa


ADP

INTRINSIC PLATELET PLUG EXTRINSIC 2. PROTEIN C... Va, VIIIa


PATHWAY PATHWAY

COMMON 3. ALPHA-2-MACROGLOBULIN IIa. Xa


PATHWAY

II IIa 4. ALPHA-1-ANTITRYPSIN.. Xa

FIBRINOGEN FIBRIN

XIII XIIIa
FIBRINOLYTIC MECHANISM

INTRINSIC ACTIVATOR EXTRINSIC ACTIVATOR


FXIIa t-PA
STREPTOKINASE UROKINASE

PLASMINOGEN PLASMIN

FIBRIN FSP

FIBRINOGEN FSP
TOPIC: OVERVIEW OF COAGULATION

INSTRUCTOR: MARGARET V. RAGNI, M.D., M.P.H.

DATE: JANUARY 8, 2009

TIME: 10:00 AM

VII/TF XII OVERVIEW OF COAGULATION

XI

IX

VIII (v)

X _______________________________________

V v v
v
II _________________________vvvvvv_____

I Ia (Fibrin)
Von Willebrand factor = v
platelet =
fibrin clot =
COAGULATION

Normal hemostasis is the system of interactions between (1) blood vessel wall, (2)
platelets, and (3) coagulation factors which assures the integrity of the circulatory system after
vessel injury. First, we shall consider normal hemostasis, the events that occur between the
initial vessel injury and the formation of the final fibrin clot, which prevent man from bleeding
or clotting to death following injury. Then, we shall consider the clinical disorders involving
deficiencies or defects of specific coagulation factors, the congenital and acquired
coagulopathies; and the clinical disorders involving deficiencies or defects of coagulation factors
or factor inhibitors which lead to congenital and acquired thrombotic disorders. Separate lectures
will address quantitative and qualitative platelet disorders and the thrombotic disorders.

OUTLINE

I. NORMAL HEMOSTASIS
A. Blood Vessel Wall
B. Platelets
C. Coagulation System

II. DISORDERS OF PRIMARY HEMOSTASIS


A. Von Willebrand Disease
B. Thrombocytopenia
C. Disorders of Platelet Function

III. DISORDERS OF SECONDARY HEMOSTASIS


A. Hemophilia
B. Vitamin K Deficiency
C. Liver Disease
D. DIC
E. Inhibitors

IV. DISORDERS OF THROMBOSIS


A. Factor V Leiden
B. Prothrombin 20210
C. Hyperhomocysteinemia
D. Protein C, S, Antithrombin-III Deficiency

I. NORMAL HEMOSTASIS

A series of complex, simultaneously occurring events are initiated when vessel injury
occurs. These events involve the three main components of coagulation, blood vessel wall,
platelets, and coagulation factors. We shall consider each component separately and then
together.
A. Vessel Wall
Injury to the vessel wall produces four consequences: (1) reflex vasoconstriction of the
vessel occurs when blood is exposed to collagen and vascular basement membrane from the
damaged vessel wall. This serves to reduce blood flow to the area and to limit blood loss; (2)
release of tissue factor, a thromboplastic material from damaged endothelium, which activates
the coagulation system. This occurs through activation of factor VII which initiates the extrinsic
pathway of coagulation; (3) denuded vascular endothelium provides a surface for contact
activation of the coagulation system. This occurs through activation of factors XI and XII which
initiates the intrinsic pathway of coagulation; and (4) the denuded vascular endothelium and
exposed collagen fibrils attract platelet to stick to the vessel wall surface, or platelet adhesion.

Vessel Wall:
1. Reflex Vasoconstriction
2. Release of Tissue Factor
3. Contact Activation
4. Platelet Adhesion

INJURY

VASCULAR ENDOTHELIAL DAMAGE

CONTACT ACTIVATION TISSUE FACTOR


FACTORS XII, XI FACTOR VII

COMMON PATHWAY
FACTOR X

FIBRIN CLOT

A schematic diagram of the events occurring at the vessel wall during coagulation
illustrates the simultaneously occurring activation of the intrinsic pathway through contact
activation of FXII, and activation of the extrinsic pathway through activation of tissue factor
(TF).

B. Platelets
Platelets adhere to the damaged endothelium with the help of Von Willebrand factor
(VWF). VWF is a glycoprotein stored in endothelial cells that serves as a sort of glue to secure
platelets to the endothelial surface, promoting platelet adhesion. When activated, platelets
release adenosine diphosphate (ADP) which causes platelets to aggregate, or stick to each other.
Early in the reaction, platelet aggregation is reversible, and distinct platelet boundaries and
intracellular elements are visible by electron microscopy. As traces of thrombin (coagulation
factor IIa) begin to form, platelet granules release more ADP, along with PF-4 (platelet factor-4)
and serotonin: this reaction is referred to as the release reaction. Serotonin is a potent smooth
muscle vasoconstrictor that serves to promote and support ongoing vasoconstriction of the
injured vessel. Once the release reaction occurs, platelet aggregates are converted to irreversible
agglutinates that plug the injured vessels allowing primary, although only temporary, hemostasis.
Both VWF and Ca++ are essential for agglutination.
PLATELET PLUG FORMATION

Platelet Adhesion.Denuded endothelium, VWF


Platelet AggregationADP, PF-4, serotonin
Platelet Agglutination.Release Reaction, Ca++, VWF

The process by which platelet agglutination and platelet plug formation occur is termed
primary hemostasis, indicating its rapid but temporary nature. As platelets agglutinate, PF-3 (a
platelet phospholipid membrane component) is released into the local circulation where it
provides a surface on which coagulation reactions occur, promoting formation of more thrombin.
With time, the agglutinated platelet plug or white thrombus is extended by fibrin and the
enmeshment of other blood elements to form the final hemostatic plug.
Disorders of primary hemostasis are characterized by mucosal bleeding, primarily in the
oropharynx, gastrointestinal tract, and genitourinary tract. Bleeding occurs early and is usually
mild. Disorders of primary hemostasis include Von Willebrand disease (VWD) and quantitative
and qualitative disorders of platelets. When platelets are decreased and/or function poorly, or
when VWF is lacking, a normal platelet plug cannot be formed and bleeding symptoms occur.

REGULATION OF PLATELET PLUG FORMATION

PLATELET VESSEL WALL

AA AA

PG PG
ENDOPEROXIDES ENDOPEROXIDES

THROMBOXANE PROSTACYCLIN
SYNTHETASE SYNTHETASE

THROMBOXANE A2 PROSTACYCLIN
(TXA2) (PGI2)

FUNCTIONS OF TXA2 FUNCTIONS OF PGI2


1. Stimulates platelet aggregation 1. Inhibits platelet aggregation
2. Vasoconstriction 2. Vasodilation

The regulation of platelet plug formation is by the metabolites of the 20-carbon


arachidonic acid, specifically thromboxane A2 (TXA2) and prostacyclin (PGI2). TXA2, formed
inside the platelet, promotes platelet aggregation and vasoconstriction. PGI2, on the other hand,
is formed within the vessel wall and causes vasodilation, thereby controlling and inhibiting
platelet aggregation and limiting platelet plug formation.

C. Coagulation System
At the same time that events are triggered in the vessel wall and platelets, the coagulation
factors which are in an inactive or precursor state, become successively activated in a cascade
or waterfall type pattern, resulting in fibrin formation at the wound site. This fibrin clot then
stabilizes through interaction with factor XIII or fibrin stabilizing factor. Finally, the clot
retracts or contracts to half its volume by interaction with platelets, specifically the
actinomyosin-like platelet protein, thrombasthenin.

COAGULATION SYSTEM
Coagulation Cascade - Zymogen, Serine Protease
Fibrin Clot Stabilization - FXIII
Fibrin Clot Retraction - Thrombasthenin

Fibrin formation is checked by specific clot inhibitors that prevent excessive clot
formation (below). The trigger mechanisms of the coagulation cascade are not fully understood
but are believed to involve contact activation of the intrinsic pathway through factors XI and XII
by the damaged vascular endothelial surface, tissue factor activation of the extrinsic pathway
through factor VII, and then combined activation of the common pathway through factor X.

THE COAGULATION SYSTEM

INTRINSIC SYSTEM EXTRINSIC SYSTEM


Surface Contact Tissue Factor

XII XIIa VII VIIa

XI XIa

IX IXa PF-3
PF-3 Ca++
Ca++
VIII

X Xa
PF-3
Ca++
V

II IIa (Thrombin)

I Ia (Fibrin)
XIII XIIIa
The coagulation system involves a series of linked reaction that occur in sequence, in
which a parent enzyme (zymogen) is converted to an activated enzyme (serine protease). The
latter catalyzes the next zymogen-protease pair in the sequence until fibrin is formed. Both the
vessel wall (through contact activation and release of tissue factor) and platelets (through PF-3)
are involved in these reactions.

Fibrin formation via the coagulation cascade mechanism is called secondary hemostasis. It
serves to strengthen and stabilize the platelet plug formed in primary hemostasis. Through
interaction with fibrin stabilizing factor (coagulation factor XIII), the final fibrin clot becomes
stronger, tighter, and more durable.

PRIMARY HEMOSTASIS SECONDARY HEMOSTASIS

Site: Mucosal bleeding Joint, muscle, body cavity bleeding


Onset: Early Late
Severity: Mild Severe
Disorders: VWD, Platelet Disorders Hemophilia, Vitamin K Deficiency, Liver
Disease DIC

Disorders of secondary hemostasis, in contrast to disorders of primary hemostasis, are


characterized by joint, muscle, and body cavity bleeding. Bleeding is usually severe and occurs
late, as a primary platelet plug has formed, although without fibrin, is temporary. Disorders of
secondary hemostasis include hemophilia, coagulation factor deficiencies, e.g. factor XI or VII
deficiency, Vitamin K deficiency, liver disease, and disseminated intravascular coagulation,
DIC.

COAGULATION SCREENING TESTS

1. Prothrombin Time (PT) Measures the extrinsic pathway


Prolonged PT FVII deficiency

2. Activated Partial Thromboplastin Time (APTT) Measures the intrinsic pathway


Prolonged APTT FXII, XI, IX, VIII deficiency
Prolonged PT, APTT FX, F, II, I deficiency

3. Thrombin Time (TT) Measures conversion of I to Ia (fibrin)


4. Reptilase Time (RT) Measures conversion of I to Ia
Prolonged TT Abnormal fibrinogen, presence of heparin
Prolonged RT Abnormal fibrinogen
.
5. Closure Time (with EPI, ADP) Measures platelet number/function
Prolonged CEPI, CADP Decreased or dysfunctional platelets

In the intrinsic system, clot formation is measured by the APTT or activated partial
thromboplastin time, and is initiated by adsorption of F XII onto collagen of the denuded, injured
vessel wall. Subsequently, activated factor XII activates F XI; FXIa activates F IX. When F IXa
is formed, F VIII, together with Ca++ and phospholipid (PF-3), activates F X and the common
pathway. Then F Xa allows conversion of prothrombin (F II) to thrombin (F IIa). In the extrinsic
pathway, clot formation is measured by the PT or prothrombin time, and is triggered by release
of tissue thromboplastin (tissue factor), and F VII is converted to its activated form, F VIIa. This
F
VIIa then activates F X. This is termed the common pathway, as it involves both pathways,
VII/TF XII CLOT FORMATION SCREENING TESTS
PT - Extrinsic Pathway
XI APTT - Intrinsic Pathway
Extrinsic Intrinsic PT, APTT - Combined Pathway
Pathway Pathway
PT IX APTT Closure Time Platelet Function
TT, RT - Fibrin Formation
VIII (v)

Closure
X Time
CEPI, CADP

V v v
Combined
Pathway v
PT, APTT
II vvvvvv

I Ia (Fibrin)
Von Willebrand factor = v

Fibrin
Formation
TT, RT

intrinsic and extrinsic. In order to screen for a coagulation defect in the intrinsic and extrinsic
systems, the PT and APTT should be obtained. To further determine what deficiency might be
present, specific coagulation tests may be performed. A prolonged PT and normal APTT point
to a defect in the extrinsic system. A normal PT and prolonged APTT suggest a defect in the
intrinsic system. Prolongation of both PT and APTT suggests a defect in the combined pathway.
To monitor normal fibrin formation, additional tests may be obtained, specifically the thrombin
time (TT) and reptilase time (RT). When heparin, a potent anti-thrombin is present, the
thrombin time is prolonged, while the reptilase time is normal. When a dysfibrinogen, an
abnormal fibrinogen is present, both the thrombin time and the reptilase time are prolonged.

DIAGNOSIS OF BLEEDING DISORDERS

History Screening Tests


Bleeding History PT
Family History APTT
Medications Closure Time
Platelet Count

Defects in platelet number and function are evaluated by the platelet count and closure
time (previously bleeding time), while defects in the coagulation cascade are evaluated by the PT
and APTT. If the PT, APTT, and closure time are normal, a coagulation problem may still be
present. For example, as many as half of those affected by Von Willebrand disease may have a
normal APTT and closure time. This finding underscores the importance of obtaining a complete
bleeding history, which should include the severity and frequency of bleeding, the age at which
bleeding symptoms began, requirement for blood transfusion, anemia and iron supplementation,
and a family history of bleeding among relatives, taking care to notice differences in severity
among family members and whether both females and males are affected.

REVISED HYPOTHESIS OF COAGULATION

TF

VII VIIa
IX IXa

VIII VIIIa INITIATING

X Xa
V Va
II IIa
SUSTAINING I Ia

The initiation of coagulation and subsequent activation of coagulation factors involves


the extrinsic pathway, specifically the tissue factor/factor VII complex. In the current
understanding of coagulation, tissue factor (TF) and factor VII form a complex, the TF/F VIIa
complex, which triggers activation of F X initially; however, the continued activation of
coagulation requires activation of F IX and F VIII, which activates F X sufficiently to sustain
coagulation, resulting in the final fibrin clot. Several findings support this hypothesis:
1) contact activation plays no role in this schema, consistent with the lack of bleeding in
individuals with F XII and half those with F XI deficiency; 2) F IX and F VIII are necessary for
continued and lasting coagulation and clot formation, consistent with the delayed bleeding (days,
weeks) typical of patients with hemophilia (F VIII or F IX deficiency); and 3) a lipid clot
inhibitor known as TFPI (tissue factor pathway inhibitor) prevents the tissue factor/ F VIIa
complex from continued activation of coagulation, thus preventing one from clotting to death.
THROMBIN: KEY MULTIFUNCTIONAL ENZYME

THROMBIN

I Ia PROT C APC
XIII XIIIa VIII VIIIa
PLATELET V Va
AGGREGATION

Thrombin (F IIa) is a key multifunctional enzyme that serves to convert fibrinogen (F I)


to fibrin (F Ia) and to activate factor XIII (fibrin stabilizing factor) and coagulation factors V and
VIII. Thrombin is also a potent platelet aggregating agent, promoting clot formation at the site
of injury. In addition to its procoagulant role, thrombin also activates anticoagulation by
activation of protein C, the major clot inhibitor, to its active form, activated protein C (APC).
This illustrates the closely tied procoagulant and anticoagulant mechanisms, that is, clot-making
and clot-breaking mechanisms, which assure that adequate but not excessive clot formation
occurs.
In summary, the three separate systems, vessel wall, platelets, and coagulation proteins,
interact to form the final fibrin clot. Thus, platelet plug and fibrin clot formation involve three
separate but interactive systems, the vessel wall, platelets, and coagulation proteins, which
together accomplish the formation of a clot and prevention of bleeding.
COAGULATION SCHEMA

INJURY

VASOCONSTRICTION DAMAGED TISSUE FACTOR


VESSEL

COLLAGEN
ADP

INTRINSIC PLATELET PLUG EXTRINSIC


PATHWAY PATHWAY

COMMON
PATHWAY

II IIa

FIBRINOGEN FIBRIN

XIII XIIIa

Finally, there is a system of checks and balances to prevent excess fibrin clot formation
from occurring. Two major checking mechanisms are important in clot inhibition, and these
include (1) clot inhibitors and (2) the fibrinolytic system. Perhaps, the most important clot
inhibitor is the -2-globulin known as antithrombin III (AT-III). This protein neutralizes
thrombin (factor IIa) and activated factors X, IX, and XI, thereby opposing fibrin formation. A
second clot inhibitor, protein C, which is a Vitamin K-dependent protein, inactivates activated
factors V and VIII. The clinical significance of these inhibitors is the well-recognized
occurrence
of thrombotic tendencies, deep venous thrombosis (DVT), in individuals with congenital
deficiency of these inhibitors, e.g. antithrombin III deficiency or protein C deficiency.

CLOT INHIBITORS

1. ANTITHROMBIN-IIIIIa, Xa, IXa, XIa


2. PROTEIN C..Va, VIIIa
3. ALPHA-2-MACROGLOBULIN.IIa, Xa
4. ALPHA-1-ANTITRYPSIN..Xa

Infants with complete protein C deficiency develop microvascular thrombosis of the skin,
purpura fulminans, which can be reversed by infusion of protein C. Protein C, anti-thrombin III,
and tissue factor pathway inhibitor (TFPI) (which inhibits VIIa-TF complex), also play
important roles in regulation of coagulation and the inflammatory response in sepsis. A recently
published study in individuals with sepsis, systemic inflammation, and organ failure showed that
those treated with APC had reduced levels of the proinflammatory cytokine, TNF-, and
improved survival. Studies of APC and of TFPI in sepsis patients are ongoing and will improve
our understanding of the interrelated roles of coagulation, inflammation, and fibrinolysis.

In addition, individuals with congenital mutations in cleavage sites in coagulation


proteins (that is mutations at the site a clot inhibitor normally cleaves a coagulation protein) have
an increased risk for venous thrombosis. The lack of a protein C cleavage site in the coagulation
factor V protein, known as the F V Leiden mutation, which occurs in 5-7% of the population,
prevents break down of the activated clotting factor, F Va, after a clot forms and results in a 5-10
fold increased risk of thrombosis. The lack of a cleavage site in F II, a mutation known as
prothrombin 20210, prevalent in about 2% of the population, results in an increased risk for
thrombosis. The latter mutation appears to increase risk for both arterial and venous thrombosis,
whereas the former increases risk only for venous thrombosis. The reason for this difference is
not understood. Patients with F V Leiden require fewer transfusions with surgery.

FIBRINOLYTIC MECHANISM

INTRINSIC ACTIVATOR EXTRINSIC ACTIVATOR


F XIIA t-PA
STREPTOKINASE UROKINASE

PLASMINOGEN PLASMIN

FIBRIN FSP

FIBRINOGEN FSP

The second checking system is the fibrinolytic mechanism. Once a thrombus is formed,
the fibrinolytic system provides for its dissolution. Specifically, this involves local conversion
of the inactive enzyme, plasminogen to the activated enzyme, plasmin. This reaction is triggered
by 1) intrinsic activators within plasma, including factor XIIa or the drug streptokinase, or by 2)
extrinsic activators extrinsic to plasma, including tissue plasminogen activator (t-PA) which is
released when a thrombus is formed, or the synthetic drug t-PA, or the drug urokinase. Once
activated by intrinsic or extrinsic activators, plasminogen is cleaved to its active form, plasmin.
Plasmin cleaves fibrin and fibrinogen into soluble fragments called fibrin split (degradation)
products (FSP), the clot is lysed and vascular patency is restored. This reaction occurs only
locally at the site of the thrombus, because plasminogen is steadily adsorbed to local fibrin.
LECTURES:
I. Overview of Coagulation
II. Hereditary and Acquired Disorders of Coagulation

OBJECTIVES:
1. To understand the mechanisms of primary and secondary hemostasis
2. To recognize clinical and laboratory aspects of coagulopathies

II. Hereditary & Acquired Disorders


KEY WORDS: Hemostasis, coagulation factors, platelets, vessel wall
of Coagulation
SUGGESTED READING:
1. Furie B, Furie BC. Mechanisms of thrombus formation. NEJM 2008; 359: 938-49.
2. Mannucci PM, Tuddenheim EGD. The Hemophilias from royal genes to gene
therapy. NEJM 2001: 344: 1773-9.
3. Nichols WL, et al. von Willebrand disease: evidence-based diagnosis and
management guidelines, NHLBI Expert Panel Report. Haemophilia 2008; 14: 171-232.
4. Ragni MV, Lozier JN. Clinical Aspects and Therapy of Hemophilia, Chap. 125, in Hematology:
Basic Principles and Practice, Hoffman R et al eds., 5th edition, Churchill Livingstone, 2008.
5. James A, Ragni M, Picozzi V. ASH Special Educational Symposium: Bleeding Disorders in
Women: another public health crisis for hematology. Hematology 2006; 171-95.

Primary Hemostasis Secondary Hemostasis

II-1. Disorders of Primary Hemostasis


Defects: Platelet Plug Formation Fibrin Clot Formation

Disorders: Von Willebrand Disease Hemophilia A, B


Platelet Disorders Vitamin K Deficiency
Quantitative Liver Disease
Qualitative DIC

Bleeding Sx: Mild, early, mucosal Mod, severe, late, body cavity

VII/TF XII Von Willebrand Disease


XI
In Vascular Injury
IX

VIII (v)

V v v
v
II vvvvv

I Ia (Fibrin)
Von Willebrand factor = v
Platelet =
Fibrin clot =
Von Willebrand Disease
VWF & Menstrual Cycle
z Monthly sloughing endometrial lining Type 1 VWD Type 2 VWD Type 3 VWD

z Vessel Injury 65-70% 30-35% 5%

z Vasoconstriction of endometrial vessels Autosomal dominant Autosomal dominant Autosomal recessive


Defect: Low VWF Defect: Abnl VWF Defect: Absent VWF
z Platelet plug formation
Clinically mild Clinically moderate Clinically severe
z Fibrin clot formation Menorrhagia Epistaxis, bruise Epistaxis, bruise
Dx: Delayed Dx: Early Dx: Early
Rx: DDAVP Rx: VWF concentrate Rx: VWF concentrate

Clinical Bleeding History Laboratory Diagnosis


z Age at first bleeding symptoms
z Frequency, severity Von Willebrand Disease Hemophilia
z Requirement for transfusion Test
FVIII:C Low Low
z Spontaneous vs. traumatic bleed
RCoF Low Normal
z Postoperative bleeding
VWF:Ag Low Normal
z Family members affected, sex-linked
Closure Time Prolonged Normal
z Medication history

APTT CLOTTING ASSAY

0.1 ml plasma + 0.1 ml APTT reagent + 0.1 CaCl2 Clotting Time

Clinical Condition APTT FVIII

Normal 35 seconds 0.85 U/ml


VWD 45 seconds 0.40 U/ml
CLOSURE TIME

---------------------------------------------------------- C-EPI
----------------------------------/------------------------ C-EPI

----------------------------------------------------------- C-ADP
----------------------------------/------------------------ C-ADP

Blood

----------------------------------------------------------- C-EPI
::::::::
------------------------------::::/:::--------------------- C-EPI

----------------------------------------------------------- C-ADP
::::::::
------------------------------::::/:::--------------------- C-ADP

DISORDERS OF SECONDARY HEMOSTASIS

Congenital Coagulation Deficiency Acquired Coagulation Deficiency


II-2. Disorders of Secondary Hemostasis Rare Common
Involves single coagulation factor Involves multiple coagulation factors
Severe bleeding Mild to moderate bleeding
Chronic disability Minor, no disability
Specialized care, treatment No specialized care, treatment
CONGENITAL PLASMA COAGULOPATHIES
HEMOPHILIA
FACTOR NAME INCIDENCE BLEEDING INHERITANCE GENETICS
TENDENCY
z X-Linked Bleeding Disorder
Ia Fibrinogen 1:1,000,000 ++++ AR 4q23-32
z Deficiency of Factor VIII, IX
hypo 0/+ AD,AR 4q23-32
dys 0/+ AD,AR 4q23-32 z Bleeding
II Thrombin < 1:1,000,000 +++ AR 11p11-q12 Joints Hemarthroses
V 1:1,000,000 ++++ AR 1q21-q25 Muscles Hematomas
VII 1:500,000 +++ AR 13q34 Infancy Circumcision Bleeding
VIII Anti-Hemophilic 1:10,000 + to ++++ X Xq28
z Severity (FVIII, IX level)
VWF Von Willebrand 1:100 + to ++ AD,AR 12pter-p12
Severe < 0.01 U/ml - Spontaneous, traumatic bleeds
IX Christmas 1:30,000 + to ++++ X Xq27
Moderate 0.010.04 U/ml - Traumatic bleeds
X Stuart 1:500,000 + to ++++ AR 13q34
Mild > 0.05 U/ml - Rare, traumatic bleeds
XI < 1:500,000 0/+ AD 4q32-q35
XII Hageman ? 0 AD 5q33-qter z Clotting Factor Treatment
XIII-A < 1:1,000,000 ++++ AR 6p24-p25
Reactive, not Preventive
XIII-S < 1:1,000,000 ++++ AR 1q31-q32 Costly
Complications
More disabling than the disease (HIV, AIDS, Hepatitis, Inhibitor)

VII/TF XII HEMOPHILIA


BLEEDING SYMPTOMS IN HEMOPHILIA
XI

Hemarthrosis Hematuria
IX
Hematomas Retroperitoneal Hematoma
VIII (v) Epistaxis Compartment Syndrome
Oral Bleeding Psoas Muscle Hematoma
X Circumcision Bleeding Intracranial Hemorrhage
Postoperative Bleeding
V v v
v
II vvvvvv

I Ia (Fibrin)
Von Willebrand factor = v
Platelet =
Fibrin clot =

Treatment of Congenital Factor Deficiencies


HEMOPHILIA: IDEAL TARGET FOR GENE THERAPY
Factor Deficiency Treatment Comment
I, XIII Cryoprecipitate No viral inactivation
z Hemophilia is not a lethal disease.
II, V, VII, X, XI Fresh frozen plasma No viral inactivation
z Treatment is suboptimal.
Retested plasma
z Complications of current therapy continue: infectious, orthopedic. Recombinant Factor VIIa Costly, frequent dosing

z The factor level to be achieved is not crucial. VIII Recombinant Factor VIII
z The site of production of factor is not crucial, as long as secreted into circulation. Mild: DDAVP Tachyphylaxis, flushing

z The duration of treatment is not crucial. VWF Mild: DDAVP Tachyphylaxis, flushing
Other: VWF Concentrate Lipid-enveloped viruses

IX Recombinant Factor IX

XII None No bleeding


VII/TF XII VITAMIN K DEFICIENCY
Vitamin K Deficiency XI

z Fat soluble vitamin absorbed in ileum IX

VIII (v)
z Required for synthesis of clotting proteins
II, VII, IX, X, protein C, protein S
X

z Cofactor: glutamic acid -carboxyglutamic acid V v v


v
z Site of action of oral anticoagulant (coumadin) II vvvvvv

I Ia (Fibrin)
Von Willebrand factor = v
Platelet =
Fibrin clot =

Vitamin K Metabolism Causes of Vitamin K Deficiency

VITAMIN K
1. Dietary Deficiency
Green leafy vegetables: daily reqt 100-200 g/day
Glutamic acid -Carboxyglutamic Acid
2. Hemorrhagic Disease of the Newborn
Factor II IIa Day 2-7: bleeding in skin, mucosa
Third trimester deficiency, infant lack of gut bacteria
Factor VII VIIa
Factor IX IXa 3. Lack of Synthesis in Gastrointestinal Tract
Factor X Xa Antibiotic Therapy
Protein C APC
Protein S APS
4. Poor Gastrointestinal Absorption
Lack of bile salts Obstructive jaundice
Rapid Transit Diarrhea, sprue

5. Inhibitor of Synthesis
Coumadin
Pesticides

Vitamin K Dependent Factors: t

Vitamin K Deficiency
-
- z Diagnosis
- II PT Early deficiency: F VII t = 4-6 hours
PT, APTT Late deficiency: F II t = 72 hours
- X
- Protein S z Treatment
- Depends on clinical setting, bleeding severity
Vitamin K: 2.5 mg po, SQ to 10-15 mg IV
- Protein C Withhold anticoagulant
- Fresh frozen plasma: if bleeding severe
- VII
-
-------------------------------------
9 Time After Coumadin
VII/TF XII LIVER DISEASE
Mechanism of Coagulopathy of Liver Disease
XI

1. Decreased Synthesis of Coagulation Factors IX


Decreased factors: BLEEDING
VIII (v)

2. Decreased Degradation of Activated Coagulation Factors X


Increased activated factors: DIC
V v v
3. Synthesis of Abnormal Coagulation Factors v
Fibrinogen missing sialic acid: DYSFIBRINOGEN II vvvvvv

4. Platelet Sequestration I Ia (Fibrin)


Von Willebrand factor = v
Hypersplenism: THROMBOCYTOPENIA
Platelet =
Fibrin clot =

Laboratory Features of Liver Disease


DYSFIBRINOGEN OF LIVER DISEASE

z Prolonged PT and APTT


Deficiency of factors THROMBIN TIME (TT)
Time for clot to form after thrombin conversion of fibrinogen to fibrin

z Prolonged TT and RT
Dysfibrinogen, hypofibrinogenemia REPTILASE TIME (RT)
Time for clot to form after reptilase conversion of fibrinogen to fibrin
z Thrombocytopenia
NORMAL DYSFIBRINOGEN
Platelet sequestration, consumption, production TT = 18 seconds TT = 26 seconds
RT = 16 seconds RT = 25 seconds
z Platelet Functional Defects
Nonspecific platelet aggregation defects IIa (THROMBIN, REPTILASE)

z Fibrinolysis I Ia
Decreased fibrinogen, presence of FDP (FIBRINOGEN) (FIBRIN)
(DYSFIBRINOGEN)
Decreased plasmin inhibitor (2-antiplasmin)

CAUSES OF DISSEMINATED INTRAVASCULAR COAGULATION


LIVER DISEASE TREATMENT OF BLEEDING
CAUSE MECHANISM
1. Infection Endotoxin
BLOOD LOSS RBCs + 2x FFP
2. Tissue Injury Tissue Factor
FACTOR DEFICIENCIES FFP Trauma Thromboplastic Substances
THROMBOCYTOPENIA Platelet Concentrates Surgery
Burns
PLATELET DYSFUNCTION DDAVP
3. Obstetric Complications Amniotic Fluid
VITAMIN K DEFICIENCY Vitamin K Amniotic Fluid Embolus Thromboplastic Substances
Abruptio Placenta
HYPOFIBRINOGENEMIA Cryoprecipitate

FIBRINOLYSIS Aprotonin 4. Malignancy


Prostatic Cancer Mucin-Activation of Factor X
Adenocarcinoma (GI) Trypsin Activation
Promyelocytic Leukemia Procoagulant Materials of Promyeloblasts

5. Liver Disease Activated Coagulation Factors


DISSEMINATED INTRAVASCULAR COAGULATION VII/TF XII DISSEMINATED INTRAVASCULAR
COAGULATION (DIC)
XI
EXCESS
THROMBIN IX
ACTION
VIII (v)

FIBRINOGEN FIBRIN X
PLATELET COAGULATION XIIIXIIIa
AGGLUTINATION FACTORS FDP
XIII DEPLETION FM, FSP V v v
AGGREGATION ACTIVATION XIII DEFICIENCY v
DEPLETION INSOLUBLE FIBRIN
DEPLETION AT-III II vvvvvv
RES FACTOR NARROWED VESSELS
THROMBOCYTOPENIA DEFICIENCIES AT-III ACTIVATION
AT-III DEFICIENCY RBC FRAGMENTS I Ia (Fibrin)
BURR CELLS Von Willebrand factor = v
Platelet =
Fibrin clot =

DIAGNOSTIC CRITERIA FOR DIC VII/TF XII LUPUS ANTICOAGULANT - LAC

XI

1. Prolonged PT and APTT - Factor Deficiencies IX


2. Thrombocytopenia
VIII (v)
3. Hypofibrinogenemia LAC
4. Fibrin split products, monomer, dimer X

5. Decreased anti-thrombin III V v v


6. Red cell fragmentation v
II vvvvvv

I Ia (Fibrin)
Von Willebrand factor = v
Platelet =
Fibrin =

COAGULATION FACTOR INHIBITORS

SCREENING FOR COAGULATION INHIBITORS


1. Specific Inhibitors Inhibit specific factors
Autoantibodies Directed at Self (Factor)
Alloantibodies Directed at Infused Factor APTT MIXING TEST
2. Blocking Inhibitors Inhibit coagulation reactions
Lupus anticoagulant Directed at phospholipid APTT 0.1 ml plasma + 0.1 ml APTT reagent + 0.1 ml CaCl2 clotting time

APTT Mix 0.05 ml plasma + 0.1 ml APTT reagent + 0.1 ml CaCl2 clotting time
0.05 ml normal
INTERPRETATION OF APTT MIX CHARACTERISTICS OF COAGULATION INHIBITORS

Clinical State APTT APTT Mix Comments Specific Inhibitors Blocking Inhibitors
Normal 31 seconds 31 seconds No Effect
Immune Inhibitors Lupus Anticoagulant
Factor Deficiency 63 seconds 34 seconds 50% factor level Directed at Specific Factor Directed at Coagulation Reaction
corrects APTT Very Rare More Common (5% of SLE)
Bleeding Tendency Thrombotic Tendency
Inhibitor 63 seconds 62 seconds Cannot dilute effect Lab: Lab:
of protein inhibitor Prolonged APTT Prolonged APTT
Prolonged APTT Mix Prolonged APTT Mix
Factor Decreased Factors Normal

EXAMPLES OF COAGULATION INHIBITORS

Specific Inhibitors Blocking Inhibitors

Symptoms
Bleeding in elderly, no past history Recurrent pregnancy loss
Bleeding in hemophilic despite FVIII Stroke at any early age
Severe postpartum bleeding Thrombosis

Examples
Anti-VIII - Pregnancy, hemophilia A SLE
Anti-X - Amyloidosis Malignancy
Anti-XIII - Isoniazid Drugs: procainamide, penicillin, quinidine,
Anti-vWF - Thyroid disease, lymphoma dilantin, hydralazine, propanolol
TOPIC: HEREDITARY & ACQUIRED DISORDERS
OF COAGULATION

INSTRUCTOR: MARGARET V. RAGNI, M.D., M.P.H.

DATE: JANUARY 8, 2009

TIME: 11:00 AM

Primary Hemostasis Secondary Hemostasis

Defects: Platelet Plug Formation Fibrin Clot Formation

Disorders: Von Willebrand Disease Hemophilia A, B


Platelet Disorders Vitamin K Deficiency
Quantitative Liver Disease
Qualitative DIC

Bleeding Sx: Mild, early, mucosal Mod, severe, late, body cavity
II-1. DISORDERS OF PRIMARY HEMOSTASIS

Disorders of primary hemostasis include diseases that result in defective platelet plug
formation. As we discussed above, this may include disorders of platelets, both quantitative and
qualitative, or von Willebrand disease. Thrombocytopenia may be due to a production problem,
from bone marrow failure, leukemia, aplastic anemia; or it may be due to a destruction problem,
from immune destruction, such as in immune thromobocytopenic purpura (ITP) or drug-induced,
or non-immune destruction, such as with thrombotic thrombocyotpenic purpura (TTP) or
hemolytic uremic syndrome (HUS). Qualitative disorders of platelets may include, for example,
Bernard Soulier syndrome, the congenital GPIb/IX receptor defect which leads to defective
platelet adhesion; Glanzmann thrombasthenia due to congenital GPIIbIIIa receptor defect which
leads to defective platelet aggregation; or various storage pool defects.

A. Von Willebrand Disease

Von Willebrand disease (VWD) is the single most common bleeding disorder. It is
estimated to occur in 1-3% of the U.S. population and is characterized by mucosal bleeding,
including epistaxis, easy bruising, gastrointestinal bleeding, bleeding after trauma and surgery,
and genitourinary bleeding. In women, a common symptom is menorrhagia. VWD is caused by
quantitative or qualitative deficiency of Von Willebrand factor (VWF), an adhesive glycoprotein
which promotes platelet adhesion to damaged vascular endothelium. Thus, the clinical symptoms
in VWD, specifically mucosal bleeding in the oral, nasal, gastrointestinal, and genitourinary
tracts are due to the failure of platelet plug formation due to abnormalities or deficiency of VWF.

VII/TF XII Von Willebrand Disease


XI
In Vascular Injury
VWF & Menstrual Cycle
IX

VIII (v) Monthly sloughing endometrial lining


X Vessel injury
V v
v
v
Vasoconstriction of endometrial vessels

II vvvvv
Platelet plug formation

I Ia (Fibrin)
Von Willebrand factor = v Fibrin clot formation
Platelet =
Fibrin clot =

With no VWF, there are no occlusive platelet plugs in damaged arterioles and venules in
the mucosal vasculature after injury, and bleeding continues unchecked. One of the
distinguishing features of menorrhagia in women with Von Willebrand disease is that it starts at
menarche, in contrast to menorrhagia with fibroids, tumors, or following pregnancy. A diagnosis
of VWD is often not made, as many women consider it normal in their family to have heavy
periods. Rather, such women may not come to attention until years later when they bleed at
surgery. This is why a clinical bleeding history and family history and testing are so important
in diagnosis of VWD, and why the American College of Gynecology has recommended VWD
testing in women with menorrhagia, before they undergo laparoscopy or hysterectomy.
There are three major phenotypes. Type 1 VWD is a quantitative deficiency of VWF,
type 2 is a qualitative deficiency of VWF, and type 3 is a severe complete quantitative deficiency
of VWD.
Type 1 VWD is the most common form, occurring in 70%, and inherited in an autosomal
dominant manner. The defect is a quantitative deficiency of normal VWF. Bleeding symptoms
are mild to moderate, with menorrhagia often the first symptom in females and epistaxis in
males. Over 80% respond to desmopressin (DDAVP), an arginine analogue, which promotes the
release of VWF from Weibel Palade bodies in endotheial cells where it is stored.

Von Willebrand Disease

Type 1 VWD Type 2 vWD Type 3 VWD

65-70% 30-35% <5%


Autosomoal dominant Autosomal dominant Autosomal recessive
Defect: Low VWF Defect: Abnl VWF Defect: Absent VWF
Clinically mild Clinically moderate Clinically severe
Menorrhagia Epistaxis, bruise Epistaxis, bruise
Dx: Delayed Dx: Early Dx: Early
Rx: DDAVP Rx: VWF-concentrate Rx: VWF-concentrate

Type 2 VWD, occurring in 30-35%, is characterized by a qualitative defect in VWF and


associated with earlier and more severe symptoms. Most do not respond to DDAVP, and require
treatment with a VWF-containing concentrate when they bleed or anticipate surgery.
Type 3 VWD is very rare and characterized by autosomal recessive inheritance and
severe bleeding, e.g. epistaxis, bruising, or postoperative bleeding. These symptoms typically
occur, beginning at several years of age, causing these individuals to come to medical attention
and be diagnosed early. As they have no VWF in their endothelial cells, they have no response
to DDAVP and must be treated with VWF-concentrate. Women with type 3 disease often
experience postpartum or postoperative bleeding, and iron deficiency anemia is quite common.

Clinical Bleeding History Laboratory Diagnosis


Age at first bleeding symptoms
Frequency, severity Test Von Willebrand Hemophilia
Requirement for iron, transfusion VIII:C Low Low
Spontaneous vs. traumatic bleeding RCoF Low Normal
Postoperative bleeding VWF:Ag Low Normal
Family members affected Closure Time Prolonged Normal
Medication history

A diagnosis of Von Willebrand disease is suspected when a screening APTT and closure
time are prolonged, in the setting of a clinical and/or family history of bleeding. Because the
disease is heterogeneous, there may be a variable clinical picture. In VWD, the VWF activity
(RCoF), VWF antigen, and FVIII are decreased, and the closure time is prolonged. The VWF
activity is measured in a platelet aggregation assay with the aggregating agent ristocetin (RCoF).
The VWF antigen is measured in a Laurell immunoassay, and the FVIII clotting activity (F
VIII:C) is measured in a standard clotting test measured by the APTT (because F VIII is in the
intrinsic pathway). The reason the factor VIII:C is decreased is because VWF serves as a carrier
protein for factor VIII, to prevent its breakdown by proteolytic enzymes in the circulation. Thus,
when VWF is decreased, FVIII remains unprotected and is degraded and thus decreased in
VWD.

The factor VIII molecule, which includes both F VIII:C (clotting activity) and VWF or
(RCoF activity) is a very large molecule, > 2 million daltons. The FVIII:C is the low molecular
weight (LMW), while VWF is the high molecular weight (HMW) portion of the molecule.

APTT CLOTTING ASSAY

0.1 ml plasma + 0.1 ml APTT reagent + 0.1 CaCl2 Clotting Time

Clinical Condition APTT FVIII

Normal 35 seconds 0.85 U/ml


VWD 45 seconds 0.40 U/ml

The FVIII:C activity is measured in a clot-based assay, based on the APTT or activated
partial clotting time. An APTT reagent (which includes tissue thromboplastin) is added to
plasma (containing the FVIII:C), and in the presence of calcium, a clot forms: the time to clot
formation is the APTT. The FVIII:C level is read from a standard curve constructed from various
concentrations of FVIII: C.

The VWF activity is measured in an assay based on platelet aggregation in the presence
of ristocetin, an aggregating agent. Platelet-rich plasma, that is, plasma in which the patient's
platelets are dispersed, is used in this test. This is a turbid solution which lets no light through:
hence there is 0% light transmittance at baseline. When ristocetin is added to the platelet-rich
plasma, platelets of normals will aggregate and the platelets clump, letting light through around
the platelet clumps, with resulting >60% light transmittance. In individuals with Von
Willebrand disease, VWF is missing, and so platelet aggregation in the presence of ristocetin
does not occur, and there is no improvement in light transmittance (straight line).

The VWF:Ag assay is measured in a Laurell rocket assay in which patient plasma, which
contains VW:Ag, is electrophoresed into agar containing antibody to the VW:Ag. An antigen-
antibody reaction occurs, and the height of the precipitin peak) is proportional to antigen present.

SDS Gel Electrophoresis: vWF Multimers

A test which is useful in distinguishing the type 2 and 3 patients from type 1 patients is
the SDS (sodium dodecyl sulfate) gel electrophoresis. This method separates the FVIII protein
complex by molecular weight. This assay distinguishes type 1 vWD, in which all the protein is
present but quantitatively less is present and the bands are lighter, from type 2 vWD, in which
the high molecular weight bands (or multimers) are absent, and from type 3 vWD in which no
high or low molecular weight bands are present at all.
CLOSURE TIME

---------------------------------------------------------- C-EPI
----------------------------------/------------------------ C-EPI

----------------------------------------------------------- C-ADP
----------------------------------/------------------------ C-ADP

Blood

----------------------------------------------------------- C-EPI
::::::::
------------------------------::::/:::--------------------- C-EPI

----------------------------------------------------------- C-ADP
::::::::
------------------------------::::/:::--------------------- C-ADP

The closure time is the single best test of platelet function. It is performed in vitro in a
capillary tube coated with collagen (simulates basements membrane) and platelet aggregating
agents epinephrine (EPI) or adenosine diphosphate (ADP). Blood infused under constant vacuum
into the capillary tube simulates shear stress inside a blood vessel. Platelets attach to a small
nick (microscopic cut) in the capillary tube (adhesion), and then stick to each other
(aggregation).
A platelet plug forms, and the time to full occlusion of the aperture is the closure time.

II-2. DISORDERS OF SECONDARY HEMOSTASIS

Disorders of secondary hemostasis include diseases which result in defective fibrin clot
formation. As we discussed above, this may include congenital or acquired coagulopathies.
Coagulopathies are disorders of one or more of the plasma coagulation factors. Congenital
coagulopathies are usually inherited, or may arise as a spontaneous mutation. They are
characterized by a deficiency of a single coagulation factor, governed by a single defective gene.
Occasionally, they involve a dysfunctional coagulation factor, e.g. dysfibrinogen. In contrast,
acquired coagulopathies usually involve deficiencies of several coagulation factors.

Congenital Coagulation Deficiency Acquired Coagulation Deficiency


Rare Common
Involves single coagulation factor Involves multiple coagulation factors
Severe bleeding Mild to moderate bleeding
Chronic disability Minor, no disability
Among the congenital
Specialized coagulopathies,
care, treatment there iscare,
No specialized a range of incidence,
treatment severity, and

inheritance. The most common is type 1 Von Willebrand disease (VWD), with autoxomal
dominant inheritance and mild to moderate bleeding tendency. By contrast, type 3 VWD and
factors II, V, and X deficiency states are uncommon, with autosomal recessive inheritance and
severe bleeding tendency. The genes which encode for the coagulation proteins are known. A
variety of mutations, including point mutations, deletions, insertions, and substitutions, may
cause deficiency states. Among individuals with severe hemophilia A, for example, over half
have been found to have a specific inversion mutation, which is helpful in carrier testing and
prenatal diagnosis. Among those with type 1 Von Willebrand disease, the genetic defect(s)
remain unknown.
A. Hemophilia

Hemophilia is an X-linked disorder caused by deficiency of either factor VIII or factor


IX, which results in bleeding into joints, hemarthroses, bleeding into muscles, hematomas, or
bleeding into body cavities, e.g. central nervous system hemorrhages. These two diseases are
clinically indistinguishable: it is known that hemophilia A accounts for about 85% and
hemophilia B about 15% of all affected individuals. Because the inheritance is X-linked, males
are affected, at about 1 in 5,000 male births, and females are carriers. There are rare exceptions,
for example, in the Amish population where affected males marry carrier females and have girls
with hemophilia.

CONGENITAL PLASMA COAGULOPATHIES


_______________________________________________________________________
FACTOR NAME INCIDENCE BLEEDING INHERITANCE GENETICS
TENDENCY
_______________________________________________________________________
Ia Fibrinogen 1:1,000,000 ++++ AR 4q23-32
hypo 0/+ AD,AR 4q23-32
dys 0/+ AD,AR 4q23-32
II Thrombin < 1:1,000,000 +++ AR 11p11-q12
V 1:1,000,000 ++++ AR 1q21-q25
VII 1:500,000 +++ AR 13q34
VIII Anti-hemophilic 1:10,000 + to ++++ X Xq28
VWF Von Willebrand 1:100 + to ++ AD,AR 12pter-p12
IX Christmas 1:30,000 + to ++++ X Xq27
X Stuart 1:500,000 + to ++++ AR 13q34
XI < 1:500,000 0/+ AD 4q32-q35
XII Hageman ? 0 AD 5q33-qter
XIII-A < 1:1,000,000 ++++ AR 6p24-p25
-S < 1:1,000,000 ++++ AR 1q31-q32
_______________________________________________________________________

The gene for factor VIII, located on Xq28, and for factor IX, located on Xq27, have been
cloned and the entire sequence of these factors known, which has provided for genetically-
engineered treatment, including recombinant factor VIII and recombinant factor IX concentrate,
as well as has promoted development of gene therapy. The hemophilias are caused by numerous
gene mutations, including deletions, insertions, and point mutations, but there is no relation
between the amount of the protein deleted and the severity of the disease.

Clinically, the typical symptom in hemophilia is bleeding. The sine qua non for
diagnosis of hemophilia in an infant male is circumcision bleeding, and the typical child or adult
has recurrent joint hemorrhages with crippling and disability. In general, the severity, frequency,
and location of bleeding sites are largely dependent on the level of F VIII:C or F IX:C.

Those with factor levels less than 0.01 U/ml (remember 0.50-1.50 U/ml is normal) have
severe disease, accounting for 60% of affected individuals, and typically have bleeding episodes
every other week.
HEMOPHILIA
X-Linked Bleeding Disorder
Deficiency of Factor VIII, IX
Bleeding
Joints Hemarthroses
Muscles Hematomas
Infancy Circumcision Bleeding
Severity (FVIII, IX level)
Severe < 0.01 U/ml - Spontaneous, traumatic bleeds
Moderate 0.010.04 U/ml - Traumatic bleeds
Mild > 0.05 U/ml - Rare, traumatic bleeds

Clotting Factor Treatment


Reactive, not Preventive
Costly
Complications
More disabling than the disease (HIV, AIDS, Hepatitis, Inhibitor)

Those with levels of 0.01-0.04 U/ml have moderate disease, accounting for 15-20% of
affected individuals, typically experience traumatic, but not spontaneous bleeding. The real
danger for a patient with severe disease is a delay in or insufficient treatment, resulting in
crippling joint damage, or, if the hemorrhage occurs in the brain, death.

Those with levels > 0.05 U/ml have mild disease and account for 20-25% of affected
individuals, bleed infrequently, primarily with trauma, and thus may not be diagnosed until later
in life, usually with traumatic or postoperative bleeding. A classic mild patient is the young high
school student who plays baseball or basketball and experiences recurrent "pulled muscles"
which do not resolve spontaneously, and on examination are found to be muscle hematomas.
The real danger in the moderate or mild patients is delay or failure to recognize the diagnosis.

VII/TF XII HEMOPHILIA


BLEEDING SYMPTOMS IN HEMOPHILIA
XI

IX
Hemarthrosis Hematuria
VIII (v) Hematomas Retroperitoneal Hematoma
X Epistaxis Compartment Syndrome
V v v Oral Bleeding Psoas Muscle Hematoma
v
II vvvvvv Circumcision Bleeding Intracranial Hemorrhage
I Ia (Fibrin)
Postoperative Bleeding
von Willebrand factor = v
platelet =
fibrin clot =

Hemarthrosis is a particularly common symptom in hemophilia, and, after repeated


hemorrhages in the same joint, there is degeneration of the synovial lining and crippling chronic
osteoarthritis, which may lead to orthopedic surgery early in life, decreased mobility, and pain.
Gastrointestinal, intra-abdominal, and intracranial hemorrhages may be life-threatening.

Hemophilia is an ideal target disease for gene therapy. It is not a lethal disease and
current treatment is suboptimal, as treatment is reactive rather than preventive, i.e. after the
hemorrhage occurs, resulting in long-term orthopedic disability, with potential viral transmission
through ongoing treatment. In those in whom prophylactic (preventive) treatment is used, i.e.
treatment on a routine basis 3-4 times a week, to prevent hemorrhages, central venous catheters
are required for access, which are associated with line infection and sepsis. One advantage in
designing gene therapy for hemophilia is the exact level of coagulation factor is not crucial, as
long as a level of >1% is attained, which will prevent spontaneous hemorrhages. Further, the
exact location where the gene is inserted/implanted is not crucial, as long as the gene product is
secreted into the circulation; the exact duration of secretion is not important, as even repeated
dosing even one or two times a year would be better than current treatment requirements. A
number of approaches to gene transfer have been attempted, and although dogs have had
sustained correction for over 5 years, an effective approach in humans remains elusive.

HEMOPHILIA: IDEAL TARGET FOR GENE THERAPY

Hemophilia is not a lethal disease.


Treatment is suboptimal.
Complications of current therapy continue: infectious, orthopedic.
The factor level to be achieved not crucial.
The site of production of factor is not crucial, as long as secreted into circulation.
The duration of treatment is not crucial.

Coagulation products used in the treatment of congenital coagulopathies continue to


improve with recombinant technologies to avoid viral transmission. Although 80% of adults
with hemophilia have hepatitis C virus (HCV) infection and 40% HIV infection from plasma-
derived clotting factor of the past, hemophilic children receiving recombinant factor are free of
these infections.
Treatment of Congenital Factor Deficiency Disorders

Factor Deficiency Treatment Comment


I, XIII Cryoprecipitate No viral inactivation

II, V, VII, X, XI Fresh Frozen Plasma No viral inactivation


Retested plasma
Recombinant Factor VIIa Costly, frequent dosing

VIII Recombinant Factor VIII


Mild: DDAVP Tachyphylaxis, flushing

VWF Mild: DDAVP Tachyphylaxis, flushing


Others: VWF Concentrate Potential transmission of
Lipid-enveloped viruses
IX Recombinant Factor IX

XII None No bleeding

Another major complication of hemophilia treatment is the development of inhibitors:


these are antibodies to foreign infused factor concentrate. They occur in 20% or more of patients,
usually after 9-10 factor infusions, and are caused by a T-dependent immune response.
Preventing inhibitor formation is a major focus of hemophilia research. One approach successful
in the hemophilia A mouse model is FVIII-pulsed dendritic cell vaccination. Clinically, patients
who develop inhibitors are unresponsive to factor infusion (the inhibitor neutralizes its effect)
and must rely on more expensive and less effective treatments, e.g. factor VIIa, and have greater
morbidity than their counterparts without inhibitors.
B. Vitamin K Deficiency
Vitamin K is a fat-soluble vitamin necessary for the synthesis of functional clotting
proteins, factors II, VII, IX, and X and of functional clot regulatory proteins, protein C and S.
Vitamin K is a cofactor in the enzymatic conversion of glutamic acid into -carboxyglutamic
acid. The conversion of the precursor Vitamin K factor proteins, II, VII, IX, and X to their
activated forms, IIa, VIIa, IXa, Xa, and of protein C and S to their activated forms, activated
protein C (APC) and activated protein S (APS), occurs through the formation of -
carboxyglutamic acid in the precursor proteins, specifically in the NH2 terminus of these
proteins.

Vitamin K Deficiency
Fat soluble vitamin absorbed in ileum
Required for synthesis of clotting proteins
II, VII, IX, X, protein C, protein S
Cofactors: glutamic acid -carboxyglutamic acid
Site of action of oral anticoagulant (coumadin)

The oral anticoagulants known as coumarins inhibit Vitamin K at this important step of
clotting factor synthesis, thereby simulating Vitamin K deficiency. The -carboxyglutamic acid
residues are necessary for Ca++ and metal ion binding by Vitamin K-dependent proteins to
phospholipid and cell membranes, which is essential to many coagulation reactions in the
coagulation cascade. Thus, in Vitamin K deficiency, coagulation reactions and coagulation
regulation are impaired.

VITAMIN K METABOLISM

VII/TF XII VITAMIN K DEFICIENCY VITAMIN K


XI

IX Glutamic acid -Carboxyglutamic acid


VIII (v)

X
Factor IIIIa
V v
v
v
Factor VIIVIIa
II vvvvvv
Factor IXIXa
I Ia (Fibrin)
von Willebrand factor = v Factor XXa
platelet =
fibrin clot = Protein CAPC
Protein SAPS

The common causes of Vitamin K deficiency include dietary deficiency (green leafy
vegetables), but this is rare as the amount required is much lower than dietary intake. Antibiotics
may cause Vitamin K deficiency by destroying the gut flora that produce Vitamin K. Usually for
Vitamin K deficiency to occur, a combination of decreased oral intake and antibiotic therapy is
required. In patients with leukemia, for example, who are not eating because of chemotherapy-
associated nausea and receiving empiric broad spectrum antibiotics because of fever, Vitamin K
deficiency may occur within 1-3 day of initiation of antibiotic therapy. Malabsorption occurring
with obstructive jaundice and sprue may also cause Vitamin K deficiency because they interfere
with absorption of Vitamin K in the ileum. Inhibitors of Vitamin K, in addition to coumadin,
include pesticides, which are chemically similar to warfarin but longer-acting.

Causes of Vitamin K Deficiency

1. Dietary Deficiency
Green leafy vegetables: daily reqt 100-200 g/day

2. Hemorrhagic Disease of the Newborn


Day 2-7 bleeding in skin, mucosa
Third trimester deficiency, infant lack of gut bacteria

3. Lack of Synthesis in Gastrointestinal Tract


Antibiotic Therapy

4. Poor Gastrointestinal Absorption


Lack of Bile Salts - Obstructive Jaundice
Rapid Transit - Diarrhea, Sprue

5. Inhibitor of Synthesis
Coumadin
Pesticides

The best coagulation screening test for Vitamin K deficiency is the protime (PT), which
measures factor VII activity. Because factor VII has the shortest half life, about seven hours, it
is the first Vitamin K factor to decrease, and the earliest to correct, once treatment is given.
Thus, early Vitamin K deficiency is characterized by a prolonged PT. As Vitamin K deficiency
progresses, factors II, IX, and X become depleted, and both the PT and APTT are prolonged.
Treatment is with oral or subcutaneous or, rarely, parenteral Vitamin K. Responses occur quite
rapidly and normal levels may be reached within 24 to 48 hours.

Vitamin K Dependent Factors: t Vitamin K Deficiency

- Diagnosis
- PT Early deficiency: FVII t= 4-6 hr
- II PT, APTT Late deficiency: FII t= 72 hr
- X
-
Protein S Treatment
- Depends on clinical setting, bleeding severity
- Protein C
Vitamin K: 2.5 mg po SQ to 10-15 mg IV
-
- VII
Withhold anticoagulant
- Fresh frozen plasma, if bleeding severe
-------------------------------------
Time After Coumadin

C. Liver Disease

The liver plays a central role in coagulation as it is the site of synthesis of coagulation
factors and the site of degradation of clotting factors. When the liver suffers functional
Mechanisms of the Coagulopathy of Liver Disease

1. Decreased Synthesis of Coagulation Factors:


Decreased factors: BLEEDING

2. Decreased Degradation of Activated Coagulation Factors:


Increased activated factors: DIC
Increased activation of fibrinolytic system: FIBRINOLYSIS

3. Synthesis of Abnormal Coagulation Factors:


Fibrinogen missing sialic acid: DYSFIBRINOGEN
hepatocellular damage, such as in advanced cirrhosis, loss of synthetic and degradation function
may occur and result in a number of coagulation abnormalities, some of which may lead to
clinical bleeding. Clinically, there may be bruising, epistaxis, bleeding from venepuncture, or
hemorrhagic gastritis. The bleeding associated with liver disease may occur only with
venepuncture or with surgical procedures: the problem is that the coagulopathy is multifactorial.
i.e., more than one coagulation problem may be present, thrombocytopenia and decreased
factors.

Laboratory Features of Liver Disease


VII/TF XII LIVER DISEASE

XI
Prolonged PT and APTT
IX
Deficiency of factors
VIII (v) Prolonged TT and RT
X Dysfibrinogen, hypofibrinogenemia
V v v
Thrombocytopenia
v Platelet sequestration, consumption,
II vvvvvv
production
I Ia (Fibrin)
von Willebrand factor = v Platelet Functional Defects
platelet =
fibrin clot =
Nonspecific platelet aggregation defects
Fibrinolysis
Decreased fibrinogen, presence of FDP
Decreased plasmin inhibitor (2-antiplasmin)

A dysfibrinogen, an abnormally synthesized fibrinogen, is very common and an early


finding in liver disease, and the last abnormality to correct after transplantation. Because an
abnormal fibrinogen forms a fibrin clot poorly, it takes longer for the fibrin clot, and hence, there
is prolongation of the thrombin time and reptilase time, which measures formation of thrombin..

DYSFIBRINOGEN OF LIVER DISEASE

THROMBIN TIME (TT)


Time for clot to form after thrombin conversion of fibrinogen to fibrin

REPTILASE TIME (RT)


Time for clot to form after reptilase conversion of fibrinogen to fibrin

NORMAL DYSFIBRINOGEN
TT = 18 seconds TT = 26 seconds
RT = 16 seconds RT = 25 seconds
IIa

I Ia
(FIBRINOGEN) (FIBRIN)
(DYSFIBRINOGEN)
Treatment of liver disease is directed at replacing the factor deficiencies with FFP, and
correcting the thrombocytopenia by transfusion of platelets. When red cells are given for acute
blood loss, two units of FFP are given for each unit of RBCs. When fibrinolysis is present in
endstage liver disease, fibrinolytic inhibitors, such as aprotinin, may reduce blood loss. In those
who require liver transplantation, there is full correction of coagulation factor deficiencies when
the new liver is placed. Transplantation also cures (phenotypic not genotypic) individuals with
hemophilia A or B, as the new liver produces the missing factor VIII (of IX).

LIVER DISEASE TREATMENT OF BLEEDING

BLOOD LOSS RBCs + 2x FFP


FACTOR DEFICIENCIES FFP
THROMBOCYTOPENIA Platelet
PLATELET DYSFUNCTION DDAVP
VITAMIN K DEFICIENCY Vitamin K
HYPOFIBRINOGENEMIA Cryoprecipitate
FIBRINOLYSIS Aprotonin

D. Disseminated Intravascular Coagulation (DIC)

DIC is a dynamic disorder in which there is excess activation of the coagulation


mechanism within the circulation. This results in excessive bleeding associated with hyper-
utilization or consumption of clotting factors. Clinically, there is oozing from venepuncture
sites, with procedures, and surgery, and trauma. DIC may include both mucosal bleeding,
reflecting thrombocytopenia (defects in primary hemostasis: platelet plug formation), as well as

CAUSES OF DISSEMINATED INTRAVASCULAR COAGULATION

CAUSE MECHANISM

1. Infection Endotoxin

2. Tissue Injury Tissue Factor


Trauma Thromboplastic Substances
Surgery
Burns

3. Obstetric Complications Amniotic Fluid


Amniotic Fluid Embolus Thromboplastic Substances
Abruptio Placenta

4. Malignancy
Prostatic Cancer Mucin-Activation of Factor X
Adenocarcinoma (GI) Trypsin Activation
Promyelocytic Leukemia Procoagulant Materials of Promyeloblasts

5. Liver Disease Activated Coagulation Factors

musculoskeletal bleeding, reflecting factor deficiencies (defect in secondary hemostasis: fibrin


clot formation). Although DIC is uncommon disorder, the conditions underlying it are not
uncommon: e.g. severe infection/sepsis, severe tissue injury associated with trauma, obstetric
complications, malignancy, and liver disease. Whatever the trigger, foreign material , e.g.
endotoxin, tissue factor, amniotic fluid, thromboplastic into the materials from the fetus, directly
activate the coagulation system, with excess activation and formation of thrombin in the
circulation. As thrombin is a key multifunctional enzyme, with many essential roles in the
coagulation process, in DIC, these functions are enhanced and excessive.
DISSEMINATED INTRAVASCULAR COAGULATION

EXCESS
THROMBIN
ACTION

FIBRINOGEN FIBRIN
PLATELET COAGULATION XIIIXIIIa
AGGLUTINATION FACTORS FDP
XIII DEPLETION FM, FSP
AGGREGATION ACTIVATION XIII DEFICIENCY
DEPLETION INSOLUBLE FIBRIN
DEPLETION AT-III
RES FACTOR NARROWED VESSELS
THROMBOCYTOPENIA DEFICIENCIES AT-III ACTIVATION
AT-III DEFICIENCY RBC FRAGMENTS
BURR CELLS

The laboratory findings in DIC reflect excessive thrombin action. Specifically, in DIC,
neoplastic cells, or activated clotting factors, gain entrance into the circulation where they there
is thrombin-mediated fibrinogen conversion to fibrin. Fibrin is normally solubilized by cellular
mechanisms in the reticuloendothelial system or by enzymatic degradation to fibrin
degradation products, FDP, which include fibrin split products (FSP), fibrin monomer (FM), and
fibrin dimer (FD), all of which are soluble and circulate. With excess fibrinogen conversion to
fibrin, there is depletion of fibrinogen, resulting in hypofibrinogenemia. As more fibrin
monomers are produced, soluble fibrin degradation products (FDP, FM, FD) polymerize to
insoluble fibrin, which deposits in capillaries and small vessels.

VII/TF XII DISSEMINATED INTRAVASCULAR


COAGULATION (DIC)
XI DIAGNOSTIC CRITERIA FOR DIC
IX
1. Prolonged PT and APTT - Factor Deficiencies
VIII (v)
2. Thrombocytopenia
X
3. Hypofibrinogenemia
V v
v
v 4. Fibrin split products, monomer, dimer
II vvvvvv 5. Decreased anti-thrombin III
I Ia (Fibrin) 6. Red cell fragmentation
von Willebrand factor = v
platelet =
fibrin clot =

When erythrocytes pass through these narrowed vessels, they become damaged and
fragmented, forming red cell fragments, i.e. shistocytes and burr cells, readily seen on the
peripheral blood smear. Thrombin also triggers platelet aggregation and agglutination, and
excess thrombin action leads to excess platelet aggregation and agglutination, leading to
depletion of platelets, platelet consumption, and thrombocytopenia. Excess thrombin action also
leads to more rapid activation and deactivation of coagulation factors, which leads to factor
consumption, depletion, and relative coagulation factor deficiencies. As excess thrombin is
formed and utilized in DIC, there is excess utilization of anti-thrombin III, which complexes
with thrombin to inhibit excess clot formation, leading to depletion and antithrombin III
deficiency.
In order to make a diagnosis of DIC, there must be evidence of excess thrombin action in several
coagulation pathways, i.e. factor deficiencies, thrombocyotpenia, hypofibrinogenemia, fibrin
breakdown products, depletion of anti-thrombin III, or red cell fragmentation. Usually, at last
three of the following six criteria should be present, most importantly the first three, while the
last three criteria are less specific for DIC.
Treatment of DIC is aimed at treating the underlying disease, e.g. antibiotics for
infection, immediate delivery of the fetus, removal of burned tissue, chemotherapy of cancer,
and transplantation of endstage liver disease, basic replacement transfusion therapy is indicated.
Platelet transfusion is given for severe thrombocytopenia, fresh frozen plasma for severe factor
deficiencies, and cryoprecipitate for hypofibrinogenemia. Despite treatment, patients with DIC
do very poorly, with an overall survival rate under 40% no matter the underlying cause.

E. Inhibitors
Inhibitors are pathologic molecules in the circulation that directly inhibit clotting factors
or clotting reactions. These proteins are of two types, 1) specific inhibitors, those that inhibit
coagulation factors, the most common of which is factor VIII; or 2) blocking inhibitors, those
that inhibit coagulation reactions, the most common of which is the APTT reactions. Specific
inhibitors cause a clinical coagulopathy similar to that observed in a severe deficiency of the
involved clotting factor. By contrast, blocking inhibitors or "lupus anticoagulants" cause no
clinical bleeding, but rather are associated with thrombosis. These inhibitors were first
recognized in patients with lupus erythematosis, and hence the name, but also occur with certain
drugs, such as propranolol, procainamide, phenothiazines, and penicillin antibiotics, and with
certain conditions, including in women with recurrent pregnancy loss and young patients who
develop transient ischemic attacks (TIAs) and stroke.

VII/TF XII LUPUS ANTICOAGULANT - LAC

XI COAGULATION FACTOR INHIBITORS


IX

VIII (v)
1. Specific Inhibitors Inhibit specific factors
X
LAC Autoantibodies Directed at Self (Factor)
Alloantibodies Directed at Infused Factor
V v v
v
II vvvvvv
2. Blocking Inhibitor Inhibit coagulation reactions
I Ia (Fibrin)
von Willebrand factor = v
Lupus anticoagulant Directed at phospholipid
platelet =
fibrin =

Inhibitors are diagnosed by performing a "mixing experiment." In the standard APTT


assay, patient plasma is mixed with tissue factor (APTT reagent), calcium, and a clotting
recorded. In individuals with inhibitors, whether specific or blocking, the APTT is prolonged.
In order to distinguish an inhibitor from a deficiency state, the APTT mix is performed.
In the APTT mixing test, patient plasma is mixed with an equal volume of normal
plasma, tissue factor and calcium are added, and the clotting time recorded. In coagulation
deficiency, e.g. factor VIII deficiency, the APTT is prolonged, but the APTT mix is normal. This
is because by adding back normal plasma, you can attain a factor level of at least 50% (0.50
U/ml) which falls in the normal range and results in a normal APTT.
SCREENING FOR COAGULATION INHIBITORS
APTT MIXING TEST

APTT 0.1 ml plasma + 0.1 ml APTT reagent + 0.1 ml CaCl2 clotting time

APTT Mix 0.05 ml plasma + 0.1 ml APTT reagent + 0.1 ml CaCl2 clotting time
0.05 ml normal
INTERPRETATION OF APTT MIX

Clinical State APTT APTT Mix Comments


Normal 31 seconds 31 seconds No Effect

Factor Deficiency 63 seconds 34 seconds 50% factor level


corrects APTT

Inhibitor 63 seconds 62 seconds Cannot dilute effect


of protein inhibitor

Specific inhibitors are directed against a coagulation protein and are termed "immune
inhibitors", as the mechanisms of inhibitor formation is believed to be a T cell immune response,
both for autoantibodies and alloantibodies. Labs are investigating the role of immunomodulatory
molecules to specifically inhibit T cell response and inhibit or modulate inhibitor response.

CHARACTERISTICS OF COAGULATION INHIBITORS

Specific Inhibitors Blocking Inhibitors

"Immune Inhibitors" "Lupus Anticoagulants"


Directed at Specific Factor Directed at Coagulation Reaction
Very Rare More Common (5% of SLE)
Bleeding Tendency Thrombotic Tendency
Lab: Lab:
Prolonged APTT Prolonged APTT
Prolonged APTT Mix Prolonged APTT Mix
Factor Decreased Factors Normal

Blocking inhibitors, on the other hand, are directed against phospholipids. Phospholipids
are derived either from tissue or from platelets. Phospholipids, primarily phosphatidyl serine,
are negatively charged tissue or platelet surfaces on which coagulation reactions occur and on
which coagulation complexes assemble. These phospholipid surfaces trigger and enhance the
rate of coagulation reactions involving tissue factor and coagulation proteins. The interactions
between phospholipids, membrane surfaces, and coagulation reactions are not yet fully
understood.

EXAMPLES OF COAGULATION INHIBITORS

Specific Inhibitors Blocking Inhibitors

Symptoms
Bleeding in elderly, no past history Recurrent pregnancy loss
Bleeding in hemophilia despite FVIII Stroke at early age
Severe postpartum bleeding Thrombosis

Examples
Anti-VIIII - Pregnancy, hemophilia A SLE
Anti-X - Amyloidosis Malignancy
Anti-XIII - Isoniazid Drugs: procainamide, penicillin, quinidine
Anti-vWF - Thyroid disease, lymphoma dilantin, hydralazine, propanolol
HYPERCOAGULABLE
STATES
Franklin A. Bontempo, M.D.

January 2009
RISKS OF
VENOUS THROMBOSIS
1) 5% over lifetime in U.S.
2) Is age related:
1 in 100,000 per year at birth
1 in 100 per year at age 80
U.S. ANNUAL THROMBOSIS
MORBIDITY AND MORTALITY
ESTIMATES

Deep venous thromboses 5,000,000


Pulmonary emboli 500,000
Deaths 50,000

Moser KM, Am Rev Respir Dis 141:235-249 1990


DEEP VENOUS THROMBOSIS AND
PULMONARY EMBOLI

1) 2/3 are first time episodes


2) 1/3 are recurrences
3) Higher incidence several days after a
cold wave
4) Higher incidence in winter months
5) PE most frequently missed diagnosis
found at autopsy
DEATH RATE FROM
PULMONARY EMBOLI

1) No change in last 2 decades


2) Higher in males than females
3) Higher in nonwhites than whites
4) Correlates directly with age
HYPERCOAGULABLE STATES

1) Primary (Congenital)
2) Secondary (Acquired)
CONGENITAL HYPERCOAGULABLE
STATES
1) Antithrombin III deficiency
2) Protein C deficiency
3) Protein S deficiency
4) Heparin cofactor II deficiency
5) Factor XII deficiency
6) Dysfibrinogens
7) Plasminogen deficiency
8) Dysplasminogen
9) Plasminogen activator deficiency
10) Factor V Leiden mutation
11) Prothrombin variant
12) Homocysteinemia
13) Protein Z deficiency
XII
VII
XI

IX

VIII
X

II (Prothrombin-Thrombin)

I (Fibrinogen)
Plasminogen

Plasminogen
Activator

PAI*

Plasmin

= Inhibitory 2 Anti-plasmin
= Activating
* Plasminogen Activator Inhibitor
XII
Plasminogen
XI
Plasminogen
Activator
IX
VIII VII

X
PAI*

V
Plasmin
II

I
= Inhibitory 2 Anti-plasmin
= Activating Fibrin
* Plasminogen Activator Inhibitor
ANTITHROMBIN III (1)

1) Heterozygosity causes clinically


significant deficiency
2) Homozygosity probably not compatible
with life
3) Cofactor for heparin
4) Primarily inhibits factors II and X
XII
Plasminogen
XI
Plasminogen
Activator
IX
VIII VII

X
PAI*

V
Antithrombin III
Plasmin
II

I
= Inhibitory 2 Anti-plasmin
= Activating Fibrin
* Plasminogen Activator Inhibitor
ANTITHROMBIN III (2)
4) Decreased In:
(a) Liver disease
(b) DIC
(c) Post-operative state
(d) Women taking OCPs
(e) Sepsis
(f) Nephrotic syndrome
(g) Newborns
(h) Heparin therapy
(i) Acute thrombosis
XII
Plasminogen
XI

Plasminogen
Protein C IX
Activator
Protein S VIII VII

X
Activated
PAI*
Protein C
V
Plasmin

II

I
2 Anti-plasmin
= Inhibitory
= Activating Fibrin
Plasminogen Activator Inhibitor
PROTEIN C (1)
1) Vitamin K dependent
2) Similar in structure to vitamin K dependent
factors
3) Inactivates factors V and VIII
4) Inhibits PAI-1
5) Requires thrombin and thrombomodulin
6) Protein S is cofactor for activation
7) Homozygous deficiency causes neonatal
purpura fulminans
8) Heterozygous deficiency associated with
venous thrombosis
9) Coumadin may cause skin necrosis
COUMADIN-INDUCED SKIN
NECROSIS
1) Usually occurs on days 3-8 after
initiation of Coumadin
2) More common in females (75%)
3) Most common on the breast,
buttocks, or extremities, occ.
on penis in males
4) Not predictable by history or
protein C level
PLASMA HALF-LIVES OF VITAMIN K
DEPENDENT CLOTTING FACTORS

Hours
Factor VII 6
Factor IX 24
Factor X 30
Factor II 60
Protein C 6
Protein S 42
PROTEIN C (2)
10) Decreased In:
a) Liver disease
b) Acute thrombosis
c) Vitamin K deficiency
d) Coumadin therapy
e) DIC
f) Adult RDS
g) Post-operative state
h) Newborns
XII
Plasminogen
XI

Plasminogen Protein C IX
Activator
Protein S VIII VII

X
Activated
PAI* Protein C
V
Plasmin

II

I
2 Anti-plasmin
= Inhibitory
= Activating Fibrin
Plasminogen Activator Inhibitor
PROTEIN S (1)

1) Vitamin K dependent
2) Cofactor for protein C activation
3) Deficiency causes hypercoagulable
state similar to protein C deficiency
4) Approx. 60% bound to C4b binding
protein
5) Coumadin may cause skin necrosis
PROTEIN S (2)

6) Decreased in:
a) Coumadin therapy
b) Vitamin K deficiency
c) Liver disease
d) Pregnancy
e) Women taking OCPs, estrogens
f) Conditions elevating C4b protein
XII
Plasminogen
XI
Plasminogen
Activator Protein C IX
Protein S VIII VII

X
Activated
PAI*
Protein C
V
Plasmin Antithrombin III
Heparin
II
Cofactor II
I
2 Anti-plasmin
= Inhibitory
= Activating Fibrin
* Plasminogen Activator Inhibitor
FACTOR V LEIDEN MUTATION

1) The most common congenital thrombotic


disorder yet described
2) Eliminates the protein C cleavage site on
Factor V
3) Caused by mutation of guanine to adenine
(GA) in the factor V gene
4) Results in replacement of arginine by
glutamine (ArgGln) in the factor V amino
acid sequence
XII
Plasminogen
XI

Plasminogen
Activator IX
VIII VII

X
Activated
PAI*
Protein C
V
Plasmin
II

I
= Inhibitory 2 Anti-plasmin
= Activating Fibrin
* Plasminogen Activator Inhibitor
APC RESISTANCE ASSAY

1) Recommended for patients with thrombotic


history and normal APTT
2) APC ratio is developed where:
APC-APTT
APC RATIO =
APTT
3) Low APC ratios (APC resistant) are associated
with a thrombotic tendency
4) 80% of low APC ratios are associated with the
factor V Leiden mutation
FREQUENCY OF FACTOR V
MUTATION
Population Frequency
Holland 4%
United Kingdom 3.5%
Sweden 7%
Egypt 6.6%
United States 6%
American Blacks 1.4%
African Blacks 0%
East Asians 0%
FREQUENCY OF FACTOR V
MUTATION
Population Frequency
United States 6%
Hispanic Americans 2.2%
African Americans 1.4%
Native Americans 1.25%
Asian Americans 0.45%
West Indians 2.3%
African Blacks 0%
East Asians 0%
FREQUENCY OF FACTOR V
MUTATION
Population Frequency
Brazil 2%
Venezuela 1.6%
Costa Rica 2%
Argentina 5%
Spain 3.3%
Poland 5%
Bulgaria 9%
Hungary 9.8%
FREQUENCY OF FACTOR V
MUTATION
Population Frequency
India 1.3%
Syria 13.6%
Jordan 12.3%
Lebanon 14.4%
Greece-Cyprus 13.4%
Israel 12%
FACTOR V LEIDEN MUTATION
Thrombotic Risks

Heterozygous mutation
thrombosis and miscarriage 7:1
Heterozygous mutation
and OCP use 35:1
Heterozygous mutation 42:1
and 3rd gen. OCPs
Homozygous mutation 79:1
XII
Plasminogen
XI

Plasminogen
Protein C IX
Activator
Protein S VIII VII

X
Activated
PAI* Protein C
V
Plasmin Antithrombin III
Heparin
II
Cofactor II
I
2 Anti-plasmin
= Inhibitory
= Activating Fibrin
* Plasminogen Activator Inhibitor
PROTHROMBIN VARIANT
Factor II Mutation

1) Variant form of factor II


2) Found in 1-2% of the U.S. population
3) Caused by a guanine to adenine (GA) mutation
in the untranslated region of the factor II gene on
chromosome 11
4) Probably leads to increased circulating level of
normal factor II molecule
PROTHROMBIN VARIANT
Factor II Mutation
Thrombotic Risks

2.3 : 1 VT in heterozygotes
4-6 : 1 MI in young female smokers
5:1 Arterial stroke
7:1 MI in diabetics
10 : 1 Cerebral VT in heterozygotes
149 : 1 Cerebral VT in heterozygotes on OCPs
RELATIVE FREQUENCY OF CONGENITAL
THROMBOTIC DISORDERS

Antithrombin Ill deficiency 1:4000


Protein C deficiency 1:16000
Protein S deficiency 1:16000
Factor V Leiden mutation 1:17
Prothrombin variant 1:50-100
ACQUIRED OR SECONDARY
HYPERCOAGULABLE STATES (1)

Secondary
1) Homocystinuria
2) Sickle cell disease
SICKLE CELL DISEASE

1) Increased incidence of stroke, esp.


in children
2) Probably due to a variety of
abnormalities
ACQUIRED OR SECONDARY
HYPERCOAGULABLE STATES (2)

Platelet-Related
1) Myeloproliferative disorders
a) Polycythemia vera
(+JAK2 mutation - 90%)
b) Essential thrombocytosis
(+JAK2 mutation 50%)
2) Heparin-induced thrombocytopenia
HEPARIN-INDUCED THROMBOCYTOPENIA (1)

1) Occurs in ~5% of patients receiving standard heparin by


immune mechanism
2) May occur with minute doses, including heparin flushes
3) More commonly causes thrombocytopenia without
thrombosis
4) Some patients develop arterial or venous thrombosis
that may be life threatening
ACQUIRED OR SECONDARY
HYPERCOAGULABLE STATES (3)
Miscellaneous
1) Nephrotic syndrome
2) Paroxysmal nocturnal hemoglobinuria
3) Factor VII, VIII, IX concentrate therapy
4) Stasis
5) Artificial surfaces
6) Pregnancy
7) OCPs, estrogens
8) Malignancy
PAROXYSMAL NOCTURNAL
HEMOGLOBINURIA (PNH)
1) Due to a complement sensitive clone of RBCs
2) 60% of patients have intra-abdominal
thrombosis at autopsy
3) Hams test and sucrose lysis test used for
diagnosis
4) Pig-a gene may also be useful
STASIS
1) Post-operative state
2) Trauma
3) Immobilized medical patients
4) Air travel, long car trips
5) Obesity
ACQUIRED OR SECONDARY
HYPERCOAGULABLE STATES (4)

Autoimmune
1) Vasculitis
2) Varicella related autoimmune protein
S deficiency
3) Lupus anticoagulant
APA

LAC ACA BFPSTS


ANTIPHOSPHOLIPID ANTIBODY

Ab
hexagonal phase + ? B2 GPI
phospholipid
LUPUS ANTICOAGULANTS
DEFINITION
1) Are a heterogeneous group of antiphospholipid
antibodies
2) Found in a variety of clinical settings
3) Usually IgG or IgM
4) Prolong APTT and block coagulation factor activity
in vitro
5) Name is a misnomer since they are associated with
thrombosis rather than bleeding
LUPUS ANTICOAGULANTS
ETIOLOGY
1) Are antiphospholipid antibodies
2) May be produced in response to
hexagonal or non-bilayer form of
phospholipid
3) May require the presence of B2-
glycoprotein I to induce a conformational
change in phospholipid molecule
LUPUS ANTICOAGULANTS
Mechanism of Thrombosis

1) Remains poorly defined


2) Changes in thrombomodulin-protein
C complex
3) Inhibition of prostacyclin secretion
4) Enhancement of platelet aggregation
5) Depression of fibrinolytic activity
LUPUS ANTICOAGULANTS
Clinical Associations (1)

1) Seen in 2-4% of general population


2) May be asymptomatic in majority of patients
3) More frequent in elderly
4) Seen in 20-45% of patients with SLE, less
often in other autoimmune disorders
5) Recurrent venous or arterial thromboses,
antiphosholipid antibody syndrome
6) Seen commonly in patients with AIDS
LUPUS ANTICOAGULANTS
Clinical Associations (2)
6) Recurrent miscarriages, especially
2nd trimester
7) Stroke, including Sneddons syndrome
8) Migraine
9) Myocardial infarction
10) Thrombocytopenia
11) Seen in association with many drugs
DRUGS ASSOCIATED WITH LUPUS
ANTICOAGULANTS OR LUPUS-LIKE
SYNDROMES (1)
1) Procainamide (Pronestyl)
2) Hydralazine (Apresoline)
3) Isoniazid (INH)
4) Diphenylhydantoin (Dilantin)
5) Primidone (Mysoline)
6) Ethosuximide (Zarontin)
7) Carbamazepine (Tegretol)
DRUGS ASSOCIATED WITH LUPUS
ANTICOAGULANTS OR LUPUS-LIKE
SYNDROMES (2)
8) Phenothiazines
9) Haloperidol (Haldol)
10) ACE inhibitors
11) Sulfasalazine (Azulfidine)
12) Quinidine
13) -interferon
14) Antibiotics (In Children)
LUPUS ANTICOAGULANTS
LABORATORY ABNORMALITIES
1) Long APTT and mix
2) Long PT and mix
3) Dilute Russel Viper venom time ratio
4) Tissue thromboplastin inhibition index (TTI)
5) Hexagonal lipid neutralization
6) Interference with clotting factor assays
7) Anticardiolipin antibody
8) Antiphosphatidylserine antibody
9) 2-glycoprotein I
10) Platelet neutralization procedure
11) Thrombocytopenia
12) +ANA
13) +RPR
LUPUS ANTICOAGULANTS
MANAGEMENT
1) Accurate identification and differentiation
from specific factor inhibitor
2) Discontinuance of offending drug
3) Treatment of underlying autoimmune disorder
4) Anticoagulation for thromboses
5) Heparin and aspirin and/or prednisone for
recurrent miscarriage
ARTERIAL THROMBOSIS
Differential Diagnosis (1)
1) Atherosclerosis
2) Diabetes mellitus
3) Smoking
4) Hypertension
5) Lupus anticoagulant
6) Vasculitis
7) Heparin-induced thrombocytopenia
8) Hyperhomocystinemia
9) Myeloproliferative disorders
ARTERIAL THROMBOSIS
Differential Diagnosis (2)
10) Estrogens, OCPs
11) Sickle Cell Disease
12) Prothrombin gene variant
13) Factor V Leiden
14) Cocaine
Testing Recommendations for
Hypercoagulable States

< age 50 > age 60


1) Antithrombin III 1) Lupus anticoagulant
2) Protein C 2) APCr or FVL
3) Protein S 3) ? Prothrombin variant
4) APCr or FVL 4) ? MTHFR gene
5) Prothrombin variant
6) MTHFR gene
7) Lupus anticoagulant
FACTOR V CAMBRIDGE

1) Mutation of 2nd protein C cleavage site on FV


molecule
2) Associated with low APCr test
3) Heterozygosity associated with increased
thrombotic risk
4) Double heterozygote with FVL may worsen FVL
risk.
HR2 Haplotype

1) Associated with low APCr test


2) Increased thrombotic risk for homozygosity and
double heterozygosity with FVL
3) No increased risk with heterozygosity alone
CONGENITAL HYPERCOAGULABLE STATES
Clinical Characteristics

1) Thrombosis at young age


2) Family history of thrombosis
3) Recurrent thrombosis
4) Thrombosis at unusual sites
5) Venous and arterial thrombosis
January 2009 Franklin A.Bontempo, M.D.
University of Pittsburgh
School of Medicine

HYPERCOAGULABLE STATES

General Considerations

Most clotting factors and true circulating anticoagulants are made in the liver. In addition, an overall balance
exists between traditional clotting pathways and the fibrinolysis pathway. The purpose of the clotting pathways
is to form a stable fibrin clot; the purpose of the fibrinolytic pathway is to destroy clots naturally. In general,
any deficiency, which upsets this balance, leads to hypercoagulability or bleeding depending on which way the
balance is upset.

An estimated 2.5 to 5 million deep venous thromboses (DVT) occur in the U.S. annually, with 500,000 resulting
in pulmonary emboli (PE), and 50,000 resulting in death; and pulmonary embolus remains the most frequently
missed diagnosis at found at autopsy.

Hypecoagulable states are usually classified in two groups: 1) congenital or primary disorders (i.e. there is a
primary defect in the clotting cascade) and 2) acquired or secondary to an underlying disease. Congenital
hypercoagulable states are often manifest by the occurrence of a thrombotic problem before the age of 30 and
the most important of these are listed below:

Most Important

1) Antithrombin III deficiency


2) Protein C deficiency
3) Protein S deficiency
4) Factor V Mutations
5) Prothrombin variant

Antithrombin III Deficiency

Antithrombin III deficiency is considered to be autosomal dominant and is relatively rare but severe when it
occurs. Its natural function is to inhibit factor II primarily. However, when the level is congenitally deficient,
this natural anticoagulant action is diminished and leads to a significantly increased thrombotic risk.
Complicating the diagnosis of congenital deficiency of antithrombin III is that much more commonly a
deficiency occurs for an acquired reason, which does not usually predispose the patient to thrombosis. Common
causes of acquired deficiency of antithrombin III are as follows:

1. liver disease - due to decreased production of AT-III


2. DIC (disseminated intravascular coagulation) - due to consumption of AT-III
3. the postoperative state
4. Gram (+) and (-) septicemia
5. nephrotic syndrome - due to loss of AT-III in the urine
6. newborns - due to low levels of production
7. heparin therapy - due to binding of the AT-III, which is a cofactor for heparin
8. acute thrombosis - due to consumption of AT-III in clot
Protein C Deficiency

Protein C is a vitamin K dependent plasma protein similar in structure to factors II, VII, IX, and X which acts by
inactivating factors V and VIII (major function) as well as activating fibrinolysis by inhibition of plasminogen
activator inhibitor (PAI-1) in the fibrinolytic pathway (minor function). It must be activated from its inactive
form by protein S and thrombin among other molecules. Similar to antithrombin III, the diagnosis of protein C
deficiency is complicated by the frequent occurence of acquired deficiency which occurs in the following
settings:

1. liver disease - decreased production


2. acute thrombosis - consumption in clot
3. vitamin K deficiency - vitamin K necessary for production
4. Coumadin therapy - blocks vitamin K
5. DIC - consumption
6. adult RDS (respiratory distress syndrome)
7. post-operative state
8. newborns - low production

A heterozygous deficiency of protein C causes an increased risk of venous thrombosis but a homozygous
deficiency can occur at birth resulting in what is known as neonatal purpura fulminans where the baby will
usually die shortly after birth of widespread thrombosis. Protein C deficiency can also be associated with skin
necrosis soon after starting Coumadin because of further suppression of an already reduced level of protein C.

Protein S deficiency

Protein S is also a vitamin K dependent plasma protein and is a cofactor for protein C activation. Congenital
deficiency of protein S causes a hypercoagulable state similar to protein C deficiency. Unlike protein C it does
not have to be activated and approximately 60% of protein S exists in reversible equilibrium with C4b binding
protein, a regulatory protein of the complement system, which may increase in acute infection and be decreased
in newborns. Like protein C, an acquired deficiency occurs in a number of conditions, which can confuse the
diagnosis of congenital protein S deficiency:

1. pregnancy - increased binding protein


2. liver disease - decreased production
3. Coumadin therapy - decreased production
4. vitamin K deficiency - decreased production
5. patients taking oral contraceptives increased binding
6. patients taking postmenopausal estrogens - increased binding

Coumadin may induce skin necrosis in patients with protein S deficiency similar to patients with deficiency of
protein C but less frequently.

Congenital deficiencies of antithrombin III, protein C, and protein S are all associated primarily with venous and
not arterial thrombosis.

Factor V Leiden

The factor V Leiden mutation (not a deficiency) is the most common congenital hypercoagulable state yet
described. It is a point mutation of the factor V molecule causing a base pair substitution affecting 6% of U.S.
population. The mutated molecule prevents the natural anticoagulant action of activated protein C thereby
causing hypercoagulability, which is primarily venous rather than arterial. The activated protein C resistance
(APCr) test is the screening test for mutation, which is confirmed, by a DNA test as being either heterozygous or
homozygous. The relative risks of venous thrombosis are as follows:
a. heterozygotes 7:1
b. miscarriage 7:1
c. heterozygotes on OCP's 35:1
d. heterozygotes on 3rd gen. OCP's 42:1
e. homozygotes 79:1

Prothrombin Gene Variant (Factor II mutation)

The prothrombin gene variant is a base pair substitution mutation of the untranslated portion of the factor II
molecule affecting 1-2% of the U.S. population. The mutated molecule is associated with an elevated level of
factor II by unknown mechanism. A DNA test is available for diagnosis. Unlike the APCr test for the factor V
Leiden mutation, no screening test is available on plasma at present for the prothrombin variant which is
associated with an increased risk of both venous and arterial thrombosis:

Relative risks:

1. venous thrombosis (VT) 2-3:1


2. MI in women smokers 4-6:1
3. stroke 5:1
4. MI in diabetics 7:1
5. miscarriage 9.5:1
5. cerebral venous thrombosis 10:1
6. Cerebral VT with OCP's 149:1

RELATIVE FREQUENCY OF CONGENITAL THROMBOTIC DISORDERS

DEFICIENCY INCIDENCE

Antithrombin III deficiency 1:500,000 (est.)


Protein C deficiency 1:16000
Protein S deficiency 1:16000
Factor V Leiden mutation 1:17
Prothrombin Variant 1:50

Acquired or Secondary Hypercoagulable States

More common than the congenital hypercoagulable states are disorders which are acquired or secondary to a
disorder which is not a primary defect of the coagulation cascade. A fairly comprehensive list follows:

1) Sickle cell disease - increased risk of stroke in young pts.

2) Myeloproliferative Syndromes - such as polycythemia vera where there is a marked overproduction of


RBC's or essential thrombocytosis where there are too many platelets

3) Heparin-associated thrombocytopenia - which occurs in 5% of patients using any amount of heparin,


including flush heparin and is antibody mediated
4) Nephrotic syndrome - renal vein and other venous thromboses may occur due to antithrombin III loss in
urine
5) Paroxysmal nocturnal hemoglobinuria - frequent association with abdominal venous thromboses due to
abnormal clone of RBC's overly sensitive to C9 in the complement cascade
6) Factor VII, VIII, or IX concentrate therapy
7) Stasis such as that seen in the post-operative state, after trauma, from long term immobilization, a recent
orthopedic cast, air travel, or long car trips

8) Artificial surfaces - indwelling catheters, artificial hearts

9) Pregnancy - associated with a 6 fold increased thrombotic risk, especially in postpartum period

10) Oral contraceptives/estrogens

11) Malignancy - with mucinous adenocarcinoma being the cell type with the highest association with
thrombosis
12) Vasculitis/Arteritis

13) Varicella-related autoimmune protein S deficiency

14) Lupus anticoagulant (LAC)

Lupus Anticoagulants (Antiphospholipd Antibodies)

Lupus anticoagulants are antiphospholipid antibodies affecting 2-4% of the U.S. population. The name is a
terrible misnomer because although the first patient known to have one in 1948 had lupus, most patients with
lupus don't have one, most patients with a lupus anticoagulant don't have lupus or get lupus, and even though
they often prolong the aPTT they don't anticoagulate you. They do cause interference with the APTT or other
clotting tests which often fail to correct when mixed with normal plasma. In addition, they may require the
presence of beta-2-glycoprotein I or some other protein to occur and their mechanism of thrombosis is not clear.
They cause a variety of laboratory abnormalities but no single test defines them and examples of tests which
may be abnormal are listed below:

1. long aPTT and mix


2. long PT and mix
3. long dilute RVV and mix
4. abnormal platelet neutralization procedure
5. abnormal tissue thromboplastin inhibition index (TTI)
6. antiphospholipid antibody
7. anticardiolipin antibody
8. anti-nuclear antibody
9. abnormal clotting factor assays
10. thrombocytopenia
11. false positive syphilis tests

Most often there are no clinical symptoms associated with the presence of a lupus anticoagulant, but if there are
the following may be seen:

1. Deep venous thrombosis


2. Pulmonary embolus
3. Cerebral infarction and Sneddon's syndrome (stroke, lupus anticoagulant, livedo reticularis of the
skin)
4. Arterial occlusion
5. Renal vein thrombosis
6. Repeated spontaneous early abortions in pregnancy

They are also commonly associated with a variety of drugs including:


1) Procainamide (Pronestyl)
2) Hydralazine (Apresoline)
3) INH
4) Dilantin
5) Phenothiazines
6) Haloperidol (Haldol)
7) Primidone (Mysoline)
8) Ethosuximide (Zarontin)
9) Carbamazepine (Tegretol)
10) ACE inhibitors (Captopril, Vasotec, Lisinopril)
11) Sulfasalazine (Azulfidine)
12) Quinidine
13) -interferon
14) Penicillin (usually children only)

Finally, they may also occur at increased frequency inpatients with HIV/AIDS, idiopathically in the elderly,
and in rheumatoid arthritis or less often in other autoimmune disorders.

The treatment of LAC is to treat the underlying cause if one is identified, discontinue an offending drug, and if
the patient has had a thrombotic event to treat with anticoagulation. The use of prednisone not usually
beneficial.

The management of patients with hypercoagulable states in general includes:

1) DVT prophylaxis for surgical procedures

2) Avoidance of oral contraceptives in females

3) Avoidance of post-menopausal estrogens

4) Indefinite anticoagulation for 1 unprovoked thrombotic event, or 6 mos. of anticoagulation when a


reversible risk factor is identified

5) Screening family members for identified gene(s)


APPENDIX

Differential Diagnosis of Arterial Thrombosis

1) Embolus
2) Atherosclerosis
3) Smoking
4) Diabetes mellitus
5) Hypertension
6) Lupus anticoagulant/APA
7) Heparin-induced thrombocytopenia
8) Prothrombin variant
9) MTHFR homozygotes
10) Vasculitis
11) Oral contraceptives/estrogens
12) Myeloproliferative disorders
13) Factor V Leiden
14) Cocaine

Estimated Relative Venous Thrombotic Risks

Background 1/1000/yr
Postmenopausal estrogens 1.5
Non-O Blood type 1.6
MTHFR homozygotes 1.7
Prothrombin variant 2.3
Hyperhomocysteinemia 2-3
Oral Contraceptives 3-4
Pregnancy 6
Factor V Leiden 7
Lupus anticoagulants 8
Surgery with gen. anesthesia >100

Testing Recommendations for Hypercoagulable States

Age <50 Age >60 or arterial thrombosis

1) Antithrombin III 1) APC resistance of FVL


2) Protein C 2) Prothrombin variant
3) Protein S 3) ? MTHFR gene
4) Activated protein C resistance of factor V Leiden 4) Lupus anticoagulant
5) Prothrombin variant
6) MTHFR gene
7) Lupus anticoagulant
ANTITHROMBOTIC
THERAPY

Franklin A. Bontempo, M.D.

January 2009
ANTITHROMBOTIC THERAPY

1) Anticoagulant therapy
2) Antiplatelet therapy
3) Thrombolytic therapy
THE IDEAL ANTICOAGULANT
1) Would be effective for prophylaxis of clots,
prevention of clot extension, and clot lysis
2) Would be able to be given orally and parenterally
3) Would be inexpensive
4) Would be devoid of side effects
5) Would have a long half-life
6) Would be easy to monitor or not require monitoring
7) Would not interact with food or other medications
8) Would be rapidly and easily reversible
HEPARIN (1)
Description
1) Discovered in 1916 by McLean; isolated from liver, thus
the name heparin
2) Anionic glycosaminoglycan available as calcium or
sodium salt
3) Molecular weight 15,000 D (avg.)
4) Prepared from porcine intestinal mucosa and
bovine lung
5) Does not cross placenta
6) Little interaction with other medications
7) IV or SC administration only
8) Reversible with protamine (1 mg/200 U heparin)
HEPARIN (2)
Action

1) Binds to and potentiates antithrombin III


2) Heparin-antithrombin III complex inactivates
thrombin (factor IIa) and factor Xa
3) Secondary effect against platelet function
XII
Plasminogen
XI
Plasminogen
Activator
IX
VIII VII

X
PAI*

V
Antithrombin III
Plasmin
II

I
= Inhibitory 2 Anti-plasmin
= Activating Fibrin
* Plasminogen Activator Inhibitor
HEPARIN (3)
Laboratory
1) Half-life of 1-3 hours depending on dose, i.e. higher dose,
longer half-life
2) Prolongs APTT which is used for monitoring
3) Markedly prolongs the thrombin time
4) Prolongs the PT at high doses
5) PT and thrombin time not usually useful for monitoring
6) Therapeutic range is 1.5 to 2.5 x control APTT if APTT
sensitivity corresponds to heparin level 0.2 - 0.4 U/ml
heparin by protamine titration or 0.3 - 0.7 anti-factor Xa
U/ml
HEPARIN (4)
Indications
Full Dose: 5000 U or 80 U/kg IV bolus, followed by 1200-1600 U/hr
adjusted to therapeutic range
1) Acute deep venous thrombosis
2) Pulmonary emboli
3) Unstable angina and myocardial infarction
Low Dose: 5000 U sq q12 h
1) Postoperative prophylaxis of any major abdominal, thoracic,
gynecologic, or orthopedic procedure
2) Immobilized medical patients >40 yrs. with CHF, CVA,
malignant disease
3) Prophylaxis for underlying hypercoagulable state
Other Dose:
1) Extracorporeal bypass
2) Hemodialysis
3) After thrombolytic therapy
HEPARIN (5)
Contraindications
1) Thrombocytopenia
2) Aspirin or alcohol use
3) Hepatic or renal disease
4) Other platelet dysfunction
5) GI bleeding
6) Tumors, esp. CNS
HEPARIN (6)
Side Effects

1) Major side effect is bleeding


2) Osteoporosis with prolonged use
3) Thrombocytopenia
HEPARIN-INDUCED
THROMBOCYTOPENIA (1)
1) Occurs in 2-5% of patients receiving standard heparin by
immune mechanism
2) May occur with minute doses, including heparin flushes
3) More common with bovine than porcine heparin
4) Asymptomatic thrombocytopenia can occur in 30-50% of
pts who develop HIT antibodies
5) ~20-50% of thrombocytopenic patients develop arterial or
venous thrombosis that may be life threatening
HEPARIN-INDUCED
THROMBOCYTOPENIA (2)
1) Usually appears 3-15 days after starting heparin,
peak incidence day 8
2) Diagnosis is largely clinical despite availability
of several tests used to attempt confirmation
HEPARIN-INDUCED
THROMBOCYTOPENIA (3)
Alternative Anticoagulants
1) Lepirudin
2) Argatroban
3) Fondaparinux
LEPIRUDIN (1)
1) Recombinant form of hirudin
2) Highly specific direct thrombin inhibitor
3) Not associated with HIT
4) Short half-life 1-2 hours
5) Monitored by APTT or ecarin clotting time
6) Crosses placenta in rats, would not use in
pregnancy at present
7) No antidote
LEPIRUDIN (2)
8) Dosing:
Treatment - Bolus of 0.4 mg/kg (up to 110 kg)
by slow IV infusion, followed by 0.15 mg/kg/h
(up to 110 kg) by continuous IV infusion OR
1.12 mg/kg SC q12h

Prophylaxis: 20 mg SC q12h

9) Must be dose reduced in renal failure (cr > 2.0)


PENTASACCHARIDE
fondaparinux (Arixtra)

1) New type of anticoagulant


2) Indicated for DVT prophylaxis in
orthopedic surgery
3) Dose is 2.5 mg SC qd
4) Benefit in HIT
ARGATROBAN
1) Synthetic molecule
2) Direct thrombin inhibitor
3) Not associated with HIT
4) Monitored by APTT, also prolongs PT
5) Must be dose reduced in hepatic failure
6) Half-life 40-50 min.
7) No antidote
8) Dose:
2 microgm/kg/min IV adjusted to
1.5-3.0 x baseline APTT
BIVALIRUDIN
1) Second generation form of lepirudin
2) Direct thrombin inhibitor
3) Not associated with HIT
4) Shortest half-life of 25 minutes
5) Monitored by APTT
6) Must be dose reduced in renal failure
7) Is dialyzable
8) Might be used at low dose in pts with both
renal and hepatic failure
LOW MOLECULAR WEIGHT
HEPARIN
General
1) Molecular weight 3,000- 7,000 D
2) Inhibits factor Xa rather than thrombin
3) Factor Xa assay used for monitoring
4) Administered subcutaneously
5) Probably less antigenic than standard heparin
6) Recommended for prophylaxis and treatment
7) Enoxaparine, dalteparin, and tinzaparin available
in U.S.
LOW MOLECULAR WEIGHT
HEPARIN
Monitoring

1) PT, APTT not usually prolonged


2) May be monitored with anti-factor Xa assay
LOW MOLECULAR WEIGHT
HEPARIN
Doses

1) Enoxaparine, 30 mg sq. q 12h, for prophylaxis


1 mg/kg sq. q 12h, or 1.5 mg/kg qd for treatment of
established clots
2) Fragmin, 2500 or 5000 mg IU qd, for prophylaxis
120 IU/kg q 12h for unstable angina or
non-Q wave MI
3) Tinzaparin, 3500 IU 2 hrs. pre-op and qd post-op for
7-10d for general surgery; 75 IU/kg qd
for 7-10d post-op for hip/knee surgery
COUMADIN (1)
Description

1) Isolated by Link in 1939 after previous


observation that cattle developed bleeding
disorder after ingestion of spoiled clover
2) Is 4-hydroxycoumarin compound, similar in
structure to vitamin K
3) Administered p.o., rapid GI absorption
4) Crosses placenta easily
5) Interacts with a variety of drugs
6) Hereditary resistance has been described
COUMADIN (2)
Actions

1) Blocks the carboxylation of the vitamin K


dependent clotting proteins, factors II
VII, IX, and X, maintaining them in their
inactive forms
2) Also blocks the anticoagulant proteins
C and S
COUMADIN (3)
Laboratory
1) Prolongs the PT and APTT
2) PT and International Normalized Ratio
(INR) used for monitoring
INTERNATIONAL NORMALIZED
RATIO (INR)

ISI
INR = PATIENT PT
CONTROL PT

Ideal INR depends on clinical situation


ISI = Sensitivity Index for thromboplastin
COUMADIN (4)
Contraindications

1) Pre-existing hemostatic defects


2) GI bleeding
3) CNS hemorrhage
4) Pregnancy, esp. 1st trimester
COUMADIN (5)
Side Effects

1) Hemorrhage
2) Unmasking of underlying anatomic lesion
3) Surreptitious use
4) Fetal abnormalities
5) Skin necrosis with deficiencies of proteins
C or S usually on 3rd to 8th day of therapy
COUMADIN-INDUCED
SKIN NECROSIS
1) Usually occurs on days 3-8 after initiation of
Coumadin
2) More common in females (75%)
3) Most common on the breast, buttocks, or
extremities, occ. on penis in males
4) Not predictable by history or protein C level
COUMADIN (6)
Dosing
1) Usual recommendation is 5 mg initial daily dose
2) Larger loading dose not beneficial
3) Should be continued for at least 3-6 months after
initial DVT, 1 yr. for recurrent thrombi, and
indefinitely for atrial fibrillation and prosthetic
valves
4) Must be aware of changes in concomitant
drugs and diet
COUMADIN (7)
Interactions
POTENTIATORS:
Sulfas
Phenylbutazone
Cimetidine
Omeprazole
Amiodarone
Anabolic steroids

ANTAGONISTS:
Barbiturates
Rifampin
Penicillins
Antacids
COUMADIN (8)
Monitoring

1) Daily or every other day initially until INR stable in


therapeutic range
2) Followed by 1-2x weekly as OP
3) Stable pts should be monitored at least once per
month
4) More often when dietary or medications occur
ANTIPLATELET THERAPY
Aspirin Indications

1) Stroke, TIA
2) MI, recurrent MI
3) Unstable angina
4) CABG patency
TICLOPIDINE
1) Interferes with platelet-fibrinogen binding
2) Exerts its action for the life of the platelet
3) May prolong bleeding time
4) Useful for coronary artery stents and CVA
5) Methylprednisolone may reverse its effect
6) Associated with TTP, neutropenia, and
diarrhea
CLOPIDOGREL

1) Interferes with GP IIb/IIIa binding site


2) Exerts its action for the life of the platelet
3) May prolong bleeding time
4) Indicated for prevention of MI, CVA, and
vascular death in pts with ASCD
5) Fewer side effects than ticlopidine
6) Dose: 75 mg qd
Abciximab (ReoPro)

1) Human-mouse monoclonal Ab
2) Binds tp GP IIb/IIIa receptor on platelets
3) Half-life 10 min.
4) May block receptor for 10 days
5) Indicated for prevention of closure of coronary
vessels after angioplasty
6) May cause thrombocytopenia
7) Used with heparin and ASA
THROMBOLYTIC THERAPY (1)
1) Act by activation of plasminogen
2) Heparin often used after initial thrombolytic therapy
3) Streptokinase usage limited by allergic reactions
from prior use of streptococcal exposure, has
half-life of 18 min.
4) Urokinase not antigenic, half-life 11-16 min.
5) Tissue plasminogen activator (t-PA) is recombinant
and non-antigenic, biphasic half-lives of approx. 3
and 30-40 min.
6) Monitored with the thrombin time
7) Reversed with cryoprecipitate or FFP
THROMBOLYTIC THERAPY (2)
Indications
1) Controversial
2) Massive pulmonary emboli with
unstable hemodynamics less than
48 hrs. old
3) Extensive popliteal or more proximal
DVT less than 5 days old
4) Intracoronary thrombosis
5) Occluded venous access catheters
BIVALIRUDIN
1) Synthetic 20 amino acid peptide
2) Highly specific direct thrombin inhibitor
3) Reversibly binds to thrombin
3) Not associated with HIT
4) Half-life of 25 min.
4) Must be dose-reduced in renal failure
5) Is dialyzable
6) Dosing:
Bolus of 1.0 mg/kg by IV infusion
Follow by 2.5 mg/kg/h for 4 hrs. by continuous IV
then 0.2 mg/kg/hr
RECOMMENDED INR FOR VARIOUS
CLINICAL SITUATIONS
Treatment of DVT 2.0 - 3.0
Lupus Anticoagulant 2.5 - 3.5
Atrial Fibrillation 2.0 - 3.0
Acute Myocardial Infarction 2.0 - 3.0
Stroke Prevention 2.0 - 3.0
Recurrent MI Prevention 3.0 - 4.5
Tissue Valves 2.0 - 3.0
Mechanical Valves 2.5 - 3.5
PERIOPERATIVE
MANAGEMENT

1) Hold Coumadin 5 days pre-op


2) Start LMWH
3) Hold LMWH 24 hrs. pre-op
4) No monitoring required, ? check PT
5) Restart LMWH 12-24 hrs. post-op
6) Restart Coumadin at day 2-5
7) Hold LMWH
January 2009 Franklin A. Bontempo, M.D.
University of Pittsburgh
School of Medicine

ANTICOAGULANT THERAPY

General Antithrombotic therapy includes:

1) classic anticoagulant drugs like heparin and Coumadin that are directed against the clotting factor pathways

2) antiplatelet agents like aspirin, clopidogrel (Plavix) and persantine

3) thrombolytic agents such as streptokinase, urokinase, and tissue plasminogen activator

The development of new anticoagulants has recently been rapidly progressing and newer agents are improving
the risk/benefit ratio for anticoagulation for many patients.

Heparin Heparin was discovered in 1916 by McLean and isolated from the liver, thus the name heparin. It is
an anionic glycosaminoglycan available as the calcium or sodium salt prepared from either porcine intestinal
mucosa or bovine lung. Standard unfractionated heparin (UFH) preparations have an average molecular weight
of 15,000 D but consist of component molecules of widely varying sizes. Benefits of heparin are that it does not
cross the placenta and can safely be used in pregnancy, it has little interaction with other medications, and it is
easily reversible with protamine (approx. 1 mg/100 U heparin). Disadvantages are that it cannot be given orally
and that it causes heparin-induced-thrombocytopenia (HIT, the most common reason that hematologists in the
U.S are sued).

Standard unfractionated heparin (UFH) acts by binding to antithrombin III and potentiating (1000x) its ability to
inactivate thrombin, the active form of factor II in the clotting cascade. To a lesser extent it inhibits the active
form of factor X, factor Xa, and also inhibits platelet function. Heparin prolongs the APTT which is the usual
test used to monitor heparin dosing. The thrombin time and PT are also prolonged by heparin but these tests are
not usually suitable for monitoring.

Heparin has a half-life of 1-3 hours depending on the dose with higher doses having a longer half-life; the
therapeutic range is usually 1.5 to 2.5 times the control APTT value. However, because of the wide variety of
APTT reagents in use, this therapeutic range is only accurate if this therapeutic range corresponds to a heparin
level of 0.2-0.4 U/ml of heparin by the protamine titration method in the hospital laboratory.

Indications for heparin include acute deep venous thrombi (DVT) and pulmonary emboli (PE), unstable angina,
and myocardial infarction (MI). Heparin is also indicated for hemodialysis, cardiac bypass, and for use after
thrombolytic therapy.

INDICATIONS FOR THE USE OF HEPARIN

FULL DOSE: 5000 U IV bolus followed by 1200-1600 U/hr adjusted to therapeutic range OR
80 U/kg followed by 18 U/kg/hr

1) Acute deep venous thrombosis


2) Pulmonary emboli
3) Unstable angina and myocardial infarction
LOW DOSE: 5000 U SC q12h or q8h

1) Postoperative prophylaxis for any major abdominal, thoracic, gynecologic, or orthopedic procedure
2) Immobilized medical patients >40 yrs. with CHF, stroke, malignant disease
3) Prophylaxis for underlying hypercoagulable state

OTHER DOSE

1) Extracorporeal bypass
2) Hemodialysis

Clinical data provide strong support for the use of prophylactic heparin or an alternate anticogulant for most
surgical patients >40 yrs. of age in the post-operative period and for medical patients who are immobilized with
heart failure, stroke, or malignancy, or for patients with a known hypercoagulable state.

The major contraindications and side effects of heparin are listed below. In addition to the side effects shown,
heparin may rarely cause hypoaldosteronism, allergic reactions, or skin necrosis; however, those listed are
probably the most important side effects.

HEPARIN

Contraindications Side Effects

1) Thrombocytopenia 1) Bleeding
2) Aspirin or alcohol use 2) Thrombocytopenia
3) Hepatic or renal disease 3) Osteoporosis
4) Other platelet dysfunction 4) Alopecia
5) GI bleeding
6) Tumors, esp. CNS

Heparin induced thrombocytopenia (HIT) HIT occurs in 2-5% of patients who receive standard UFH,
including minute amounts (i.e. it is not dose related), usually 3-15 days (peak incidence day 8) after initiation of
the drug due to an immune mechanism. The bovine form is more highly associated with HIT than the porcine
form and the usual presentation is that of a persistent dropping of the platelet count to <100,000. About 1/5 of
patients with HIT may develop thrombosis which is potentially serious. A smaller number of patients may
develop a sudden dramatic thrombocytopenia with possible thrombosis immediately after starting heparin if the
patient had been previously exposed to heparin in the prior 6 weeks. This is due to the presence of pre-formed
antibodies not causing thrombocytopenia that may be particularly life threatening.

Cessation of the drug and use of an alternate, non-heparin, form of anticoagulation is the usual treatment. The
diagnosis of HIT remains largely clinical; laboratory testing may be helpful but can be cumbersome or
insensitive and is always necessary for making a diagnosis. Checking platelet counts at least every 3 days when
starting UFH to detect it early is strongly recommended.

Low Molecular Weight (LMW) Heparin LMW heparin contains more specific heparin fragments of molecular
weight 3,000 to 7,000 D. which act by inhibiting factor Xa rather than thrombin and require a factor Xa assay
for monitoring. LMW heparin is administered subcutaneously (SC) and is now recommended for DVT
prophylaxis as well as for the therapy of acute thrombosis. Studies indicate that LMW heparin produces a more
reliable dose-response relationship than UFH and significantly reduces post-operative DVT. LMW heparin is
associated with a lower risk (1-2%) of HIT than regular heparin but LMW heparin is contraindicated for use in
patients with known HIT previously induced by UFH because in that setting the cross-reactivity may be as high
as 80%. Enoxaparine (Lovenox), dalteparin (Fragmin), and tinzaparin (Innohep) are the three (3) LMW
heparins that are now available in the U.S. LMW heparins in contrast to UFH may need to be dose reduced for
patients with renal failure. They do not cross the placenta and may be used safely for most indications in
pregnancy.

Lepirudin (Refludan) Lepirudin is a recombinant leech venom with no cross-reactivity with regular (UFH)
heparin, which can usually be monitored with the APTT. It has a short half-life of 1-2 hr. and crosses the
placenta in rats making it contraindicated in pregnancy. Dosing is with a loading dose of 0.4 mg/kg IV over 15-
20 sec. followed by an infusion of 0.15 mg/kg/hr. IV for patients up to 110 kg. A subcutaneous dosing schedule
is also available. Because of renal excretion, it must be dose-reduced in renal failure. The latter is highly
important since protamine has only limited ability to reverse its action.

Argatroban Argatroban is a synthetic arginine derivative that competitively inhibits thrombin, both soluble and
thrombus-bound, has a half-life of 40-50 minutes, and was developed for use in patients with HIT. It is only
administered intravenously. The APTT is used for monitoring argatroban and there is no known cross-reactivity
with UFH in patients with HIT. Dose reduction is necessary for patients with hepatic failure because it is
excreted through the liver. One difficulty with its use is that it significantly prolongs the PT and may
complicate the transition from inpatient argatroban to outpatient Coumadin, requiring a higher INR level prior to
cessation of Argatroban to keep the INR in the therapeutic range during the transition.

Fondaparinux (Arixtra) Fondaparinux is a new class of LMW heparin called pentasaccharide. It is a piece of
the standard heparin molecule which has a longer half-life (17-21 hrs.) and has benefits for use for long term
DVT prophylaxis at a dose is 2.5 mg SC qd. It may also be monitored with the anti-Xa assay and may need to
be dose reduced in renal failure. It has also recently been shown to be effective for treatment of thrombosis at a
dose of 7.5 mg SC qd (for patients between 50 and 100 kg.) and may be useful in patients with HIT although not
officially approved for that use as yet.

Warfarin (Coumadin) Coumadin was isolated by Link in 1939 after the previous observation that cattle
developed a bleeding disorder after the ingestion of spoiled clover. It is a 4-hydroxycoumarin compound,
similar in structure to vitamin K, administered p.o. due to its rapid GI absorption. Because it crosses the
placenta easily and causes fetal abnormalities, it is contraindicated in pregnancy, especially in the first trimester.
Disadvantages of Coumadin include its interactions with a variety of drugs and foods. In addition, it usually
takes about 5 days until its full anticoagulant effect is reached. Occasional hereditary resistance has been
described but the usual reason for the lack of clinical effect is failure of the patient to take the medication.

Coumadin acts by inhibiting the formation of vitamin K, which is necessary for hepatic production of clotting
factors II, VII, IX, and X as well as the anticoagulant proteins C and S. Coumadin prolongs the PT and the
APTT but the PT is used for monitoring. Current recommendations are that the INR (International Normalized
Ratio), a calculated value taking into account the sensitivity of the reagent in the test system be used with the PT
to overcome variations in laboratory test results and to standardize the therapeutic range.

Indications for Coumadin are for prevention and treatment of deep venous thrombosis (DVT), for prevention of
stroke and recurrent thrombosis after myocardial infarction, for prevention of embolism in patients with atrial
fibrillation, and for patients with replaced cardiac valves, particularly mechanical valves.

Contraindications and side effects of Coumadin are listed below.


Skin necrosis is of particular interest with Coumadin use because patients who develop it may have a deficiency
of either protein C or S. This usually occurs during the 3rd to 8th day after starting Coumadin and develops
because of thrombosis of vessels in the underlying tissue and usually occurs on the buttocks, breast, or
extremities for unclear reasons. Treatment with heparin until the INR is stable in the therapeutic range is
recommended to avoid this complication.
COUMADIN

Contraindications Side Effects

1) Pre-existing hemostatic defects 1) Hemorrhage


2) GI bleeding 2) Unmasking of underlying
3) CNS hemorrhage anatomic lesion 3) Localized skin necrosis
4) Pregnancy, esp. 1st trimester 4) Fetal abnormalities
5) Surreptitious use

The recommended starting dose of Coumadin is 5 mg. followed by doses adjusted to the standard therapeutic
INR range as the vitamin K dependent factors become inactivated. Larger loading doses are not usually
recommended and may be dangerous. In some patients, particularly ethnic Chinese but also in others, lower (2
mg.) doses may be recommended by some studies due to the presence of polymorphisms that retard the
metabolism of Coumadin and render the patient more sensitive to its action. PT monitoring is usually done
daily or every other day in hospitalized patients until a stable level of anticoagulation is reached. The usual
therapeutic INR range for most indications is 2.0-3.0 with the exception of mechanical valves, which is 2.5-3.5.
Outpatients will often require a PT twice weekly initially and then biweekly or monthly after a steady state is
reached. Stable patients should be monitored no less than once monthly. However, because of the wide variety
of interactions of Coumadin with other drugs, any addition or deletion of a drug to the patient's medication
regimen should prompt consideration of more frequent PT testing to ensure maintenance of Coumadin levels in
the therapeutic range. A partial list of drugs that may interact with Coumadin is given below.

DRUGS, WHICH MAY INTERFERE WITH COUMADIN

POTENTIATORS: ANTAGONISTS:
Amiodarone Adrenocortical steroids
Anabolic Steroids Antacids
Aspirin Barbiturates
Cimetidine Oral contraceptives
NSAIDs Penicillins
Omeprazole Rifampin
Quinidine Vitamin C
Sulfas
Tamoxifen

Coumadin overdose may be treated with vitamin K, fresh frozen plasma, or clotting factor concentrates
including recombinant factor VIIa (Novoseven) with the type of treatment depending on the clinical situation.

ANTIPLATELET AGENTS

Aspirin Aspirin exerts its antiplatelet effect by acetylating the


endoperoxide cyclo-oxygenase irreversibly within minutes of the ingestion of a low dose of the drug. Thus for
the life of the platelet, normally 10 days, platelet function will be inhibited.

Clinically, the aspirin anti-platelet effect is more important in preventing arterial thrombi, where platelets are
more involved in the formation of clot, than in venous thrombi. Numerous trials have shown aspirin to be
beneficial for:
1) prevention of primary myocardial infarction (MI), particularly in males but also in females
2) reduction of the risk of MI and death in the setting of unstable angina
3) prevention of recurrent MI after initial infarction
4) prevention of new or recurrent stroke and death after transient ischemic attacks (TIAs) or minor stroke
5) prevention of coronary artery bypass graft closure.
Major side effects of aspirin are an increased risk of bleeding in general and GI irritation and bleeding from
ulceration of gastric mucosa in particular. Ototoxicity with tinnitus can occur but this as well as the other side
effects are seen less often with low doses of aspirin.

Clopidogrel (Plavix) Clopidogrel is an oral platelet antagonist which has been found to be useful in patients
with unstable angina and for prevention of MI and stroke in patients with transient ischemic attacks (TIA's). It
is also of benefit in patients with coronary artery stents, especially "drug-eluting stents" when used in
conjunction with ASA.

Clopidogrel exerts its antiplatelet action by inhibiting ADP for the life of the platelet, i.e. 10 days and may cause
prolongation of the bleeding time. It should be stopped 10-14 days prior to surgery. A first generation
clopidogrel-like drug, ticlopidine (Ticlid), was used earlier but was associated with thrombotic
thrombocytopenic purpura (TTP) and other side effects at a higher rate than that seen with clopidogrel, which
has only rarely been associated with TTP to date. Because of its lower toxicity profile, the use of clopidogrel
has largely replaced ticlopidine.

Abciximab (ReoPro) Abciximab is a human-mouse monoclonal antibody for intravenous infusion, which binds
to the glycoprotein IIb/IIIa receptor on human platelets and inhibits platelet aggregation. It may markedly
prolong the bleeding time and while it has a half-life of only 10 minutes, low levels of glycoprotein IIb/IIIa
receptor blockade may be present for up to 10 days after infusion. The major indication for abciximab is as
adjunctive therapy with heparin and aspirin for prevention of abrupt coronary vessel closure after angioplasty.
The major side effects are bleeding and thrombocytopenia (5%) and caution must be taken in patients with any
predisposing bleeding risks. The dose is 0.25 mg/kg IV given as a bolus 10-60 minutes prior to the start of
angioplasty followed by an infusion of 10 microgm (.01 mg)/min IV for 12 hours.

Thrombolytic agents

Thrombolytic agents act through the activation of plasminogen, are usually monitored with the thrombin time or
the APTT and can be reversed with fresh frozen plasma or cryoprecipitate. They are fast acting and are
advantageous because they lyse clot already formed. Heparin is often used as an adjunct to thrombolytic
therapy but its role has not been precisely defined. Streptokinase was the first thrombolytic agent but its use has
been limited by allergic reactions from prior use or streptococcal antigen exposure. Urokinase was used
previously but has been taken off the market due concern about viral contamination. Tissue plasminogen
activator (t-PA) is the most commonly used agent. T-PA is now used in recombinant form, is non-antigenic,
and has biphasic half-lives of approx. 3 and 30-40 min.

Indications for thrombolytic agents are controversial but they are usually recommended for intracoronary
thrombi associated with myocardial infarctions and also for occluded venous catheters. Current trials also
indicate benefit in selected patients with new-onset stroke. They may be used for massive pulmonary emboli
with unstable hemodynamics less than 48 hrs. old and occasional patients with extensive DVT's less than 5 days
old.

REFERENCES

1. Hirsh J: Heparin. N Engl J Med 324:1565-1574, 1991.


2. Hirsh J: Oral Anticoagulant Drugs. N Engl J Med 324:1865-1875, 1991.
3. Wessler S, Gitel SN: Warfarin: From Bedside to Bench. N Engl J Med 311:645-652, 1984
4. Roth GJ, Calverly DC: Aspirin, Platelets, and Thrombosis: Theory and Practice. Blood 83:885-898, 1994.
5. Hirsh J, Guyatt G, (eds.): Seventh ACCP Concensus Conference on Antithrombotic Therapy. Chest
(supplement) 126(3):163S-704S, 2004.
Objectives
To Review Practical Aspects of the
Coagulation in Clinical Evaluation and Treatment of Patients
Practice with Coagulation Disorders
Q/A Part 1 2009
Roy E. Smith MD, MS
smithre@upmc.edu

Simple Coagulation Cascade


Recombinant
Contact Activator + Ca + PL
TF + Ca + PL

PL
Contact Activation

IIA

Case 1 Case 1
A 42 yo male smoker with COPD and hypertension is
admitted with acute SOB and chest pain. He has no
bleeding hx.
Preprocedure Coagulation Tests
He has been non-compliant with his blood pressure
medicine. Citrate 3.2% 3.8%
A cardiac angiogram is planned.
PT (8-12s) 22.2 24.4
Px: Ruddy complexion. Distant heart sounds.
Bronchovesicular breath sounds. A palpable spleen aPTT (23-33s) 47.150.8
tip. Clubbing.
Labs: Hb 23.3 gm/dl (13.5 17.5)
Hct 73.2 ( 38-45)
What Do You Do Now? What Do You Do Now?
1. Angiogram All prolongations in PT or PTT
1. There is no must evaluated prior to invasive procedures
bleeding history.
Proceed with the 2. Repeat the PT and PTT with lower [citrate]
angiogram. 3. Mixing Studies on both PT and PTT Not
2. Repeat the PT and unreasonable, but with elevated Hct this is
PTT with lower unlikely to provide solution
[citrate] 0% 0% 0% 0% 4. Thrombin Time No bleeding history. Unlikely
3. Mixing Studies on to be useful. Fbn has to be really low to
both PT and PTT prolong PT and PTT.

e
m
...

...
...
ed

Ti
so
an

n
le

ie

bi
b

4. Thrombin Time ud

om
he
no

St
tt
is

hr
g

ea

T
in
re

ix
ep
he

M
R
T

Case 1 Citrate vs Hct


Pre-procedure Coagulation Tests Short vs Long Tube

Citrate 3.2% 0.25%


PT (8-12S) 22.2 12.2 1
aPTT (23-33s) 47.1 28
5

Clotting in the
vacutainer tube may
result in shortening
of the PT and/or
PTT.

If clot formation is
D Dimers Icterus XSive, the PT and/or
FDPs Lipemia PTT may be
Hemolysis
prolonged.
How Can Clinicians Influence Specimen Stability
Preanalytical Aspects of Test Specimen 18 24C 2 - 4C -70C
Testing? ACAs Serum 48h 2 wks 1 year

Ensure the patient is fasting. PTT tests Citrated 4h 4h 6mos


Plasma
Alert the Laboratory if PT tests Citrated 24h NR 6mos
Plasma
hematocrit is <25% or >55% FV and FII Whole 3 days NR NR
mutations blood
Laboratory should adjust [citrate] (EDTA)
Be aware of presence of
hemolysis, lipemia or jaundice

Case 2: 40 yo Female
What is the likely explanation?
No personal or family history of
bleeding. 1. Hemophilia A
Last cycle of chemotherapy for 2. Acquired FVIII
breast cancer 3 months ago via a inhibitor
central line. 3. DIC
4. Heparin Effect
0% 0% 0% 0%
PT (8-12s) 8.9 8.4 8.4

ct
IC
..

ffe
lia

h.

D
PTT (23-33s) 55 28 52

in
hi

E
op

in
III
FV

ar
em

ep
d
H

re

H
ui
Repeat Testing cq
A

What is the likely explanation?

1. Hemophilia A No bleeding history.


Patient is female.
Heparin Flush
2. Acquired FVIII inhibitor Variable
PTT. No bleeding.
3. DIC No bleeding. DIC is always 2nd
to something else.
4. Heparin Effect
Sample collected through a VAD
Heparin Contamination
Thrombin Reptilase

How to Ensure that Specimen


Fibrinogen
Collection is Optimal

Peripheral venipuncture
Fibrin Clot
not above running IV (saline or heparin)
Prolonged TT
Prolonged RT
Central/arterial line draws
Heparin Ensure wasting appropriate amount
Dysfibrinogenemia
Dysfibrinogenemia specimen
hypofibrinogenemia
hypofibrinogenemia
FDP/DDs
FDP/DDs

Case 3: Prolonged aPTT Case 3: Prolonged APTT


Preoperative clotting assays What do you order next?
45 year old female 1. Factors V, VII, X
No bleeding history 2. Factors XII, XI, IX,
aPTT 90 (21-33s); Pt 10 (8-12s) VIII
ThrombinTime normal 3. Factors II, VII, IX, X
1:1 mix 30s
4. Factor XII,
Consistent with factor deficiency
prekallikrein, High 0% 0% 0% 0%
Molecular Weight

X
Kinninogen

,X

.
...

l..
II,

I X

ka
X,
,V

re
,
I,

I
sV

VI
X

p
I,
II,

II,
or

sI
sX

X
ct

or

or
Fa

or

ct

ct
ct

Fa

Fa
Fa

Simple Coagulation Cascade


Contact Activator + Ca + PL
Recombinant What Do You Tell the Surgeon?
TF + Ca + PL
1. Preoperative
76% FFP
80%
2% 2. Preoperative
70%
80% rFVIIa
80%
3. Preoperative
plasma 0% 0% 0% 0%
exchange
IIa

IIA
FP

y
er
...
FV
eF

rg
ex
er
iv

su

4. Proceed with
a
at

tiv

sm

it h
er

la
op

w
er

ep
op

ed
re

iv
P

re

ce

surgery
at
P

ro
er

P
op
re
P
What Do You Tell the Surgeon? Prolonged
What Do YouaPTT with
Tell the No
Surgeon?
Bleeding Risk
1. Preoperative FFP This patient 1. Preoperative FFP
XII deficiency
is not at bleeding risk. FFP is 2. Preoperative rFVIIa
Severe deficiency prolongs
not indicated 3.
aPTT
Preoperative plasmarisk exchange
Not a bleeding
2. Preoperative rFVIIa - Same 4. Proceed with surgery prolonged
Other causes for
3. Preoperative plasma exchange aPTT that are not risks factors
for bleeding include:
- Same HMWK Def.
4. Proceed with surgery There is PK
no bleeding risk LAC (except a few with FII
inhibitor)

CASE 4: 45 yo man with


Case 4: What Next?
Ulcerative Colitis
Refractory to standard medical therapy 1. Obtain good
personal and
Scheduled for proctocolectomy. He is not family hx
receiving anticoagulants. 2. Thrombin time to
6/13/08 6/25/08 r/o heparin effect
PT (8-12s) 10.1 10.0 3. Mixing study to
r/o inhibitor
aPTT (21-33s) 42 35 activity 0% 0% 0% 0% 0%

4. Repeat aPTT and


What Do You Do Next?

..

...
PT for a third time

..
.

...
r..

to

r /

la
an
om dpe

to

p
tim

ith
dy

PT
o
5. Proceed with
n

u
go

w
ta
st
bi
n

ed
g

ea
ai

in

ce
planned surgery

ep
bt

hr

ix

ro
O

R
T

45 year Old Male


Case 4: What Next?
What Happened Next?
1. PHx/FHx- No bleeding or VTE Proctocolectomy performed
2. Heparin effect - No exposure to Massive intra-and postoperative bleeding
heparin Multiple blood products transfused
3. Inhibitor - No bleeding or thrombosis aPTT prolonged to 38s (22-30s).
4. Repeat PTT/PT will not be Dx
5. Proceed with surgery - dangerous
What would you Analyze
Prolonged PT or APTT Normal
Next?
Mix TT 1. Platelet Closure
times
2. Platelet
Correction No Correction Aggregations
3. Factors VIII, IX, XI
XS PL 4. Factor VII 0% 0% 0% 0%
Factor

II
Assays

I
X
.

V
.

t..
. .
Correction

X,

or
ga
re

,I

ct
su

re

III

Fa
lo

gg

sV
tC

tA

or
le

le

ct
te

te

Fa
la

la
LA

P
Simple Coagulation Cascade
What Would You Analyze Contact Activator + Ca + PL
Recombinant
TF + Ca + PL
1. Platelet Closure times Poor test of
platelet function. Not indicated 76%
80%
2. Platelet Aggregations Better than 70%
80%
Closure times, but not indicated. 23%
80%
3. Factors VIII, IX, XI Deficiency in any
of these will prolong the PTT only
4. Factor VII Deficiency will prolong IIA

the PT only

What was Missed?


APTT sensitivity to Factor IX
8 years prior bled after a tooth extraction
Personal and family are the most important
hemostatic test Sensitivity of PTT depends
The AML induction patient Walter Kriszik DOB 2.docx

on the reagent used


2/5 male siblings tested were affected
6 month F/U FIX = 9%
Patients with mild hemophilia A or B may have
variable FVIII or FIX levels and variable aPTTs

%[FIX]
% [Factor IX]
% [FIX]
Case 5: 70 yo male Case 5: 70 Year Old Male
Prolonged PTT
Not active bleeding or previous PT 10 (8-12s)
bleeding or thromboses
PTT 60 (22-33s)
Raised in orphanage Corrects with Mix
TT normal
No family medical history available
1:1 mix 28s
Born in Poland. Family disappeared Which Factor is
during WWII Consistent with deficient?
factor
deficiency
Congenital vs
acquired?

What Factor Would You What Factor Would You Assay?


Case 3: Prolonged aPTT
Assay?
1. Factors V a deficiency results in
1. Factors V 45 year old female
prolonged PT and PTT
2. Factor II 2. No
History
Factor II - ofSame
bleeding (spontaneous or
as above
3. Factor VII after challenge)
3. Factor VII a deficiency results in
4. Factor XI Preoperative clotting assays
prolonged PT only
PT 10 (8-12s)
4. Factor XI a deficiency results in a
aPTT 90 (21-33s)
prolonged PTT only
0% 0% 0% 0% What do you do next?
sV

II

I
I

X
I

V
or

or
or

or
ct

ct
ct

ct
Fa

Fa
Fa

Fa

Simple Coagulation Cascade


Contact Activator + Ca + PL
Recombinant Case 5: 70 Year Old Man
TF + Ca + PL
Eastern European Jewish background
76% Congenital FXI deficiency
80%
70%
<2%
Inhibitor not present (normal mix)
80%
80%
FXI levels do not correlate well with
clinical presentation. Therefore, must
be suspicious of this possibility
IIA
Case 6: 50 year old male with Epistaxis Case 6: What Do You Order?
Dx: Polycythemia Vera
Demands phlebotomy to control recurrent 1. Thrombin
epistaxis
No bleeding vessels on ENT exam. time/Reptilase
Normal BP time
Takes no medications 2. Factor Analysis
Personal and family Hx are N/C.
H/H 13/46, platelets 530,000, PT/PTT 13/30
3. Mixing study
(normal), platelet aggregations normal to all 4. Clot solubility 0% 0% 0% 0% 0%
agonists except ristocetin (poor response)
test

t
dy
sis

es
..

.
..
u
ly
R.

yt
st

ds
na
e/

lit
g

an
5. Von Willebrands

A
tim

in

bi
or

br
ix

lu
n

M
ct

ille
o
bi

ts
Fa

W
om
panel

lo

on
C
hr

V
T
Bleeding Conditions: normal PT/PTT
Case 6: What Do You Order?
Thrombocytopena/Platelet disorders
1. Thrombin time/Reptilase time-PT/PTT are nl. (including Quebec Platelet Disorder
Dysfibrinogenemia unlikely FXIII deficiency
2. Factor Analysis-He has epistaxis and PT/PTT are Mild congenital bleeding disorders (esp
nl. Factor analysis will likely not be helpful FIX def; VWD)
3. Mixing study-Not indicated, since PT/PTT are nl. Localized bleeding from anatomic disorder
4. Clot solubility test-The defect associated with a Vascular purpura
abnormal result is usually not associated with
epistaxis Diamond-Gardners Syndrome
5. Von Willebrands panel- Nl PT/PTT and epistaxis Alpha 2 anti-plasmin deficiency
indicates platelet defect. Aggregations nl except to Dysfibrinogenemia
ristocetin Plasminogen Activator inhibitor (PAI)-
deficiency

The Results of a Von Willebrands Multimers FVIII antigen


Dz Panel 2M

FVIII:C 75%
VWag 71%
VW:RiCOF <10%
VW:RiCOF/Vwag < 0.14
VWF multimers
What does this patient have?
1. A rare type of
VWD
2. No bleeding
disorder
3. Munchausen
Syndrome 0% 0% 0% 0%
4. He picks his

se
D

...

...
W

no
nose
Sy
iso
fV

is
n
d
o

sh
se
ng
pe

ck
au
di
ty

pi
ch
ee
re

e
un
bl

H
ra

M
A

Making the Diagnosis of VWD Case 7: 50 Year Old circumcised Male with Recent
Onset of Bleeding
Clinical Criteria
1. 1980-Tooth extraction No complications
1. Personal and family Hx of mucocutaneous
bleeding 2. 2005 - Root canal - 3 days later xs bleeding tracking
into the neck
2. and/or physical evidence of the same
3. 2005- Rifle recoil - a large bruise
Laboratory Criteria
6. 2006- Unexplained Intracerebral hemorrhage, full
1. VW:RiCOF< 30 IU/dl
recovery
2. RiCOF 30-50 as long as there is supporting
7. 2007- Requires elective surgery
clinical evidence
CBC, PT, PTT, platelet aggregations normal (including
3. RiCOF/Ag < 0.5-0.7 supports dx of Types 2A, to ristocetin)
B, M
He has no pertinent family history and takes no
4. VW:Ag in most types 2A,B, M <50 IU/dl, but medication
may be normal

Case 7: 50 Year Old Male with Recent


Case 7: 50 Year Old Male with Recent
Abnormal Bleeding and Normal PT/PTT
Abnormal Bleeding and Normal PT/PTT
What Would You Do Next?
What Would You Do Next?
1. Test for dys- or 1. Test for dys- or hypofibrinogenemia
hypofibrinogenemia (1:1M) Rare, not associated with
intracerebral hemorrhage
2. Check factor levels
2. Check factor levels PT/PTT nl.
3. Test for PAI-1 or A2
antiplasmin def 0% 0% 0% 0% 3. Test for PAI-1 or A2 antiplasmin def
very few cases reported. Very rare
4. Clot solubility test
st
l..

te
...

r..
ve
rh

4. Clot solubility test screening test for


ty
o
le

ili
o

or

ub
AI
ys

ct

ol
d

fa

FXIII deficiency (1:2M)


or

ts
or

ck

tf

lo
tf

he

es

C
es

T
T
FXIII Screen: Clot Solubility Test
FXIII Deficiency
5M
Urea FXIII <1% severe; 1-4% moderate; > 5% ? rare
or bleeding
1% T/2 : 7- 10 days
-
Citrated
Plasma MCA Hemostatic levels : 3-10% to prevent
spontaneous bleeding
Clinical features:
Umbilical stump bleeding 80%
Muscle hematoma 32%

Ca +/- Thrombin + 37C


+ Intra cerebral hemorrhage 30%
Trauma/postop, Miscarriage, Delayed
wound healing

Case 8: 75 YO WF with
Prolonged PT/APTT
Otherwise healthy. No meds.
No bleeding disorder or thrombosis
Physical examination: Livedo
reticularis only
PT 14 (10-12s)
PTT 65 (21-33s)

What is the most likely What is the most likely Diagnosis?


Diagnosis? 1. Coagulation Factor Deficiency She has no
bleeding history. This makes this highly unlikely
1. Coagulation
Factor Deficiency 2. Liver Disease This is possible since both the
PT and PTT are prolonged, but history not
2. Liver Disease consistent with liver injury
3. Factor Specific Inhibitor No bleeding. Would
3. Factor Specific have to be directed against a factor in the
Inhibitor common pathway. Very rare.
0% 0% 0% 0%
4. Lupus Anticoagulant No bleeding. Livedo
4. Lupus reticularis +. Highly likely.
t
se

an
..

. ..
o.

ul
n
ise
ct

Anticoagulant
cI

ag
Fa

r D

ifi

ico
n

ec
ve
io

nt
Sp
Li
at

s A
ul

or

pu
g

ct
oa

L u
Fa
C
Prolonged PT, APTT or dRVVT

Mix TT

Correction No Correction

XS PL
Factor
Assays Correction

LA

ISTH Diagnostic Criteria for LAC


Prolongation in PL dependent
screening test
Evidence of Inhibitory activity (mixing
studies)
Evidence that inhibitory activity is PL
dependent (confirmatory test)
R/O other inhibitor

Case 9: 80 yo female with 80 YO female with prolonged


prolonged PTT and Ecchymoses PTT and Ecchymoses
Recovering from a cat bite on the right PT 10 (8.4 12s)
index finger APTT 70 (21-33s)
Blood cultures grew out P. multicida 1:1 mix immediate correction;
Received 14 days of a cephalosporin prolonged to 50s after
incubation
antibiotic
HLN 70 (saline control 72s)
2 weeks after completion of ATBs:
dRVVT 42 (<45s)
generalized ecchymoses.
TT 19 (16-25s)
No joint bleeds.
10s
Prolonged PT, APTT or dRVVT What is Your Tentative Dx
70s
Mix TT 1. Lupus
Anticoagulant
2. Specific Factor
50s No Correction Deficiency
Correction
3. Heparin Effect.
XS PL No 0% 0% 0%
Factor correction
Assays Correction

nt

.
..

ct
la

e.

ffe
u

rD
ag

E
to

in
ico
Specific Inhibitor

ac

ar
nt

ep
F
ic
A

H
ci f
s
pu
LA

pe
L u

S
What is Your Tentative Dx Diagnostic Tests
1. Lupus Anticoagulant Criteria for Factor Assays
Lupus anticoagulant not met. No Reduced activity of a single factor at all
correction with xs phospholipid. dilutions (activity does not change with
increasing dilutions
2. Specific Factor Deficiency mixing
study corrected and then prolonged Inhibitor Assay
with incubation. Progressive inhibition of the factor in
question when normal plasma incubated
3. Heparin Effect The normal thrombin with test plasma
time r/o heparin effect. Inhibitor titer (eg. Bethseda assay)

FVIII specific standard


curve
Inhibitor
Time

[Factor VIII]
FVIII deficient plasma +
dilutions of normal
plasma PTT from mix of substrate
deficient plasma and patient
plasma with a known FVIII
deficiency
Apparent Factor Levels FVIII Inhibitor Titers
FVIII Inhibitor LAC
VIII IX XI XII VIII IX XI XII
Time
Plasma
Dilution
1:5 <1 28 20 38 12 14 5 10 Reciprocal of the
dilution that inhibits
1:10 <1 35 29 47 17 20 5 12
[Factor VIII] 50% of factor activity
1:20 <1 42 38 54 26 22 6 14
after 2hs at 37C

1:50 <1 60 51 72 50 30 12 16

1:100 <1 74 63 84 74 32 17 20
PTT from incubated mix of inhibitor plasma
and normal plasma

Review
The PT and PTT are artificial tests that
allow us to categorize coag pathways.
Abnormal test results: temp, lipemia,
hemolysis, etc.
Most important coagulation test:
history.
The w/u for a prolonged PT/APTT
begins with a mxing study.

Review Review
Sensitivities of APTT/PT vary with the
Heparin is a common cause of a prolonged reagents used.
PTT and can be excluded by doing a TT/RT. Spontaneous ICH occurs in FXIII def
Deficiencies of contact activation factors
The presence of multiple factor inhibitors
prolong the aPTT but does not cause
bleeding. should make you think of LAC.
Even minimal abnormalities in PT/APTT must LAC is no usually associated with bleeding
be evaluated. A factor specific inhibitor is usually
Congenital factor deficiencies may not be associated with diffuse ecchymoses (not
diagnosed until adulthood (esp VWD, IX, XI, hemarthrosis)
XIII).
Review
Lack of parallelism may indicate the
presence of an inhibitor
Multiple inhibitors may mean the
presence of LAC
58.The Hypercoagulable State
- -- -... - --
Jeffrey I. Zwicke~;MD, and Kenneth A. Bauer, MD

Risk Factors forVenousThromboembolism (VTE)


--

Accluired5 Risk Factors Inherited Risk Factors Mixed Risk Factors


I
1. Malignancy-associated 1. Antithrombin deficiency I. Hyperho~noc~steinc~nia
2. A~~riphospholipid antibody syndromc 2. Proccin C deficiency 2. Elcvaccd hctor VTII
3. Pregnancy-associated 3. Proccin S deficiency 3. Activated prorcin C rcsiswncc in absencc of fac-
4. Esrrogcn tlicmpy 4. Factor V Lcicicn a nun ti on tor V Lcidcn
5. Immobiliz~cion 5. Prochtombin GZOZlOA muclcion 4. Elevated factor TX
6. Trauma 6. Dysfibrinogencmia(rare) 5. Elevated factor XI
7. Ncphrocic syndromc 6. Elevated TAFI (thrombin-acci\*atablefibrinolysis
8. Hcparin-induccd rhfombocyropcnia inhibitor)
9. Mycloprolifernrivc iliso~.ders 7. Low frec TFPI (tissue hctor pathway inhibitor)
10. Paroxysrnnl nocturnal hemoglobinuria
1 I. Prolonged air travel
, 8. Low plasma fibrinolytic activity

12. Major sur~cry

Evaluation of a Patient After aVTE


Routine Evaluation
1. History and cxaminacion to idcnti5 acquired risk ficcors (scc abovc).This should include obscccric history in women because recnrrenr scc-
ond or child-trimcsccr fetal loss may suggest anriphospholipid antibody syndrome or hcrcdirary chcombopliilia
2. Decsilcd hmily history wich i n q ~ ~ i rregarding
y fcmalc fpnily niembcts who have cakcn oral conttzccpclvcs or sutFc~rdany vcnous clirvmbocic
cvcnrs during prcgnancy
. -
3. CBC and pcriphcml slnear to cvaluate for underlying disease (cg, ~n~cloprolifcracivc
disordcr such as csscntial chrornbo~~rhcmia
or poly-
cytl~c~nia
vcra, microangiopathic liemolysis)
4. Othcr laboratory cesrs as indicatcd including evaluarion for heparin-induced rhrombocytopenia
5. Exnnsivc scrccning for malignancy nor rccommended. Pcrforni age-appropriate scrccning as indicaccd. Lowcr threshold to scarch fix malig-
nancy bascd on sylnpcolns o r signs especially in older paticncs wich n smoking history
6. Tlirombophilia screcn as or~tlincdbelow for idiopxchic dccp vcin thrombosis

Laboratory Evaluation for Idiopathic VTE


Class~fyparicnr in strongly chro~nbo~liilic or wcakly cliro~iibo~hilic
catcgory bascd on chc following risk factors:
1. Dccp vcin rhrombosis bcfvrc ;igc 50
2. Reci~rrrncvenous thro~nbosis
3. Family history o f \'TE (in first-dcgrcc mlacives)

Laboratory Testing Based o n Risk G r o u p

Strongly Thrombophilic ( 2 1 o f the above risk kctors) Weakly T h ~ m u b o ~ h i l i c

I. Antica~diolipin;~nribodics I. r\nric:~rdiolipinantibodies
-7. LLIPLIS
anucoagulanc 2. Lupus ;~nticoagulanr
3. Hornocystcinc 3. Homoc!*stcinc
4. Facror V Lcidcn 4. hctor \.'Lcidcn
5. Prothronibin GZOIIOA mutation 5. Prothrombin G20210A nuta at ion
6. Protcin C deficiency
7. P~nccinS deficiency
8. Ancirlimmbin deficiency
I Laboratory Evaluation f o r Recurrent Arctrial T h r o m b o s i s
I. ChlY the prcsencc o l a lupus nncico;r~~nr/clevnccdc ~ r d i o l i ~ iantibody
n Icvcls 2nd hypertio~noc~reincmia
. r l r risk faccors 6,s ,trtrri,~liiirotnbrsi.;
che hmrdicacy rliromb~~hilias auc nor r ~ s kfacton
2. Consider ocher discasc scatcs, including paroxysmal nocrurnal liemoglobinuria. heparin-induced chr~nibocyco~cnia,
occulr rnalignanc);
~n~elr~p~uliferacivc
disorders, and cocaine ;tbuse

Acquired Risk Factors


1. Malignancy-Associated Thrombosis
General:
1. ,'\cioilna h r apRt-oximncely2@'%o f all cast:s of VTE
2. In solne prorpeccivr scirdies che incidaice of malignancy in rhr ti~srycar after diagnosis of .an idioparhicVTE is >7%1.klowevcr, rrials have
1101 LiwnonsmccJ itnprnved ptienc outcornel; and cosc-eiiccrivcncss o f cxtcnsivt. screening beyond ;iw-;lppropria~:r:or symptoin-dirccccJ can-
ccr bcrccning
Pachogcncsis:
1. E r ~ o l o ~nor
y clearly lrsrablishcd
7. be rclatcd to rissuc taccor e1al.oratcc.d by EUInOr cells
Managcmeat of tlrromlosis:
l a r heparin (LMWH)
1. A~utcrbronllosrs Heparin or low ~ ~ ~ o l e c uweight
1 7. Lor~p-rrrntrlzrctov; Parienu with canccr hove a liieher risk of recurrelicc rhan individuals wl~osirffer an tu~~rovoked
'9
'vTE in the A s e n i t of can-
iel: Anriioagu1;rre a minirni~~n of 6 tnonclis and 3s I O I I ~as the malig~lancgis ac~ive.Chmnic ;~ncico;rgulncioncan be considcred \vich low mol-
ecular weight heparin (LMWH) owing ro high rrcurrcncc race and pocendal 61. inil.ascd bleeding complicarions wirh anriioagul;rti~)r~ wirh
l ~ s 17'1.0 ill wnrfi~rintreated ~acictics
: In n randomized rrial comparing LMWI-I <d;llrcparin) wirh warfarin, recurrenet ar 6 m o ~ ~ rwits
.u,f.Iun.
conip:~rcd wirh 8% in rlir dnlcep;~ringroup (P = .0017), :rlthoi~~h
chcrc was no bignihcant differcmce in major blcrding raccs
, -

2. Antiphospholipid Antibody Syndrome


General:
diap~iosischanircrizcd by a chcombodc evenr (venous or arterial r h r o ~ ~ d o sor
1. ,A cli~~ical i s recurrcnr feral loss) in nssociacion wirh a pcrsisrmc
(LA) in spcci;rlizrti coagu1;rrioo :lss;tys and/or pcrsisrcntly elevated cjtcrs of wl-diolipin a~icibodi~s
Illpus ;~nticong~tlanr (IgG ;tnd Igkl')
2. L.4s .rw circly associated wirh bleeding, bur confer an inircascd risk for rcalrrcnr chroti~bosis
3. Cli~liial11innifc5v~ri0nsiticlirde rhr~n~bocycopcriia,ITCLII:~CIIC fcrirl loss, and arccrial or vcnous ctrmmbvsis
Pattlogencsis:
1. LAs rcsulr fiw~nanribodics rhar bind to phospliolipids and plas~naprorci~ls(P,-glycop~orcin I, prorliro~nbin)in vicro ;und prololiS elorring
rirncs i.criciwlly dcpcndcnc (rn rhc ;lmounc of phospholipid in ;~ssay)
2. Li\s ;irr often :rs.\ociaced with:
a. S~srniiicI U F ~ I J c ~ y c l i e ~ n a c o ~ ~ r ~
b. I>rugs (usually nor t~rochro~iibocic)
c. C:rnccr
d. Idiopathic
c. Infccrioris (ofcen rransicnc!
M.ulngemmc of thrombosis:
1. :liirrt rbr.;.t,rlvsa. Uni-iactionatcd heparin o r low molecular ~wiglicl~epnrin(LMWH)
7. l.o~!~-ir.n,r r1;rrap Warticin admi~~isrcrcd orally. liccmspcccivc scudics suggesccd char pacicncs wich ;uicipliospt,olipid .rnciboiiy syndn,,mc
rciluiircil .I r;trgcc LNR >3 ro obtain adcqu:rcc 3nrirhromLwric pcoccction. I-iowcvcr. subscclucnc pn,sprcrive srudirs sl~owedrhor an
lNr< = 1-3 is :~dcqu;~tc in pacicncs wich venous chl.ombosis

~:tu~vrl,cr
M:? vr rl. NEJM2LX13:5-1.9:1133-1 158
!'iu;rn.> <; cc .,I. 1 'l.lt!x>c& H:tcn~<ht2005:.;:ci+8-3.j.i -
T HE H YPERCO AGULABL E STATE 78 1

3. Pr egnanc y-Associa ted Thrombosis


G en er al:
1. T h ro m bosis is lead ing cause of maternal m o rt al it v in develop ed countries
.2. The risk o f" th rombosi s increases 5-6 rimes during ~'rcgnaIl c)' ,1I1d the in crea sed ris k p e rs ists LIp to S weeks pns rpJ n Unl
J. ;\: losr: [\ ~ lY~';I ';ur
DV -r~ an: Inl,l r.~:d in dw lcr'r leg duc [0 iliac vein co rn pres - io n by rh, righr i1i .1C ,l r rr r :.
;'vb nagt'ITI t' n t (an rcp ar rurn]:
\ V;ld:l rill is ;lb ~n lllccl}' co nrrmndic.ircd bet ween rlic (1rh and l 2d l weeks l)( prcgn.:mc;when risk of warfJri n c 1l1btTO P~1 [ h~' i~ grc:ltl'$[
l . .1"", ;l:r" l!Iioe,i;; Unfra crionarc d hepa rin o r low m ole cu lar \Vei ~ht he parin ( LM \V H)
L\l[\V I 1 prefcac d tor !ong-rl' rm ~lI1 ( i c o ag ll b (i o n over unfr .ictionarcd heparin because of ilh."t' (';lScd bionv.ulnbilicv and ;\ lower incidence 0f
ostco porovis :lnd hl'! )arin-indllc~'d tli rom bocvro pcnia
\V hell rhl'n11,\l'ut ic doses are used. mon iror .1l1t i-X ,1 levels monrhlv to ~I d j u s r t()r prcgnal1cy-;lssoci:lfCd wright: ( h ,ll1g C'.$
2. Pr~lpJJy lllx i5: S ~C A ntl co<lgu b n c prophylaxis during p reg na nc y below

l\1;ulagl'Il1l"Il t ( pc ri par cu lu ):
l'repomon:
DisLon rillll(' I.:\I\VH .ir least 24 ho urs bd~)n: Jelivcry [0 m inirniz blcl'ding risks ;l~'() ci ;l rt'd with cp i...[ural : lrll"'~ rhc~i:l :1J1 d ddi\:CI}
T he dccisinl1 to bridgl.' with untractiona tcd heparin shou ld be based on perceived risk of th rombos is ~iu rins r1w rum- o tl : ltl r i l~ {) ~lgl l b r i l }J1
U nfro crionarcd heparin sho uld be .idm iu istcrcd If
-The t unc int crv.rl tr om th e episode o f rhro m bosis has been <: I month
- t\ Lldir io nal p ro rh ro m l-o t ic risk !;}(rors exist or t ltc p.iricnt is o the rwise plTCl' i\'cd ;1$ having: :1 high risk 01: r11l'l,Hnb(}sis
D i s\~{) I H i n lic unir.icnona rcd hq\a rin 4-6 hours bd~)rc .icl ivcrv ( t1~ lJ ;.l. lly .ir rh c sta rr o f l::tbol.') r allow (or normal rz.irion t)f ,lPTT
iVlan.tgernent ( pos t part u m) :
1. J);lrring any bleed ing co nipiir .irio ns, rhcrnpcuri c an ricDaguLltion e m u:-t1:1lly be resumed 12.-18 hours a(t.. -r a \;tgin;tl dclivvry .m d 2-+ [lUlU'S
;\(rlT ccs .uc an scc rro n delivery

R<:sraning Anticoagularion Postpanmn


- - - - -----=--r
\ ';lgiual Deliv er y

12 hours

Tlu-ra pcu rir ,j o~i ng

2. \ Var r:,r in or L V1\ VH. dq'md ing on p.nicm pre!('!"c\1cr


:\ uriC"o aguLu ion is recomme nded f~'H' .u le ;IS( (, w6t :ks pnsrp:lrtlllT1 ( minimum 3-(, m onr!is ITl 1'0 Ll i af[cr d i ~' gn os ; s o r V T E)
Anticoagulant prophyla xis during pregnancy:
1. III wome n wirh :1 hi:-rorr of rhrombosi J~soc i.1I"cd wirl: ,1 rr.uisicnt risk i:l([ Ol'. J n ticoJg u b llf prOpI J ~.-L1X i . . is ~1' 1l{'r;} "~ ' no r 1\,'<]lIirC'd ;lnrCp;l rlll lll
due ro 10 \,,: rare IJf recurre n t thr om bosis, However. sho rr-rcrn, :mt icoJgllbtl~Hl tor 6-0 weeks poSrp ~l[tll l1l ShOliki be co ns ide red
) I II wouicn \\-' jrli lusro rv ~) f thrombosis .in d a hered itary rliroml-oplulic Llisl)rdc[ ( : . ec bl'!OW).
till' risk of recurre nce wit hout ;\llticnagulariDIl appL'al'S
1'0 bt' higher and anriC();lgulariol1 ,lI Hq' ;ll T ll ll l is jusriiicd. l-lowcv cr, rhc inrcn::-iry 1)1" a\1uctlagubri pn--riwra pt' IHll. \'lT~ IJS p r\ )p l 1\'J. lCrI(-h, l~ no r
bl'cn l'.'irabli:-.lH:d ,l nd :--h nuld br b,lS ~'d on pl'rcc i n~d risk 01" thrombl)si:o;. Durin g rhc Fllsrp:lrt urn t'~TiLJd , rhcr:lfH'uri c , 1I1 r i (u . l ~u h l' i o n "houkl he
a.I mmistcrcd t~) r 6-K weeks
,). The 1Tl.1Il :1gclTIl'nr or
.isym prom.u ic carr iers of hl'l'cdit.lry rhrolllbnp hili,lS is lX>Jltn) vt.'t's iaL Cllilsidcring thac d w l':,rilllarl'd risk n ( rh n)m bo sis jn
\VI llllcn with thl' prothromb in (J202 1(Jt\ 1l111 tJt iol1 o r [,cror V L ~id cn 1l111[;uion is only O,2 -0.5 (:/~ d tlring t' I.'Cg ll;lI1C)', pro ph yLl( [jc aor icl),lgub.
[ton ,ll1t \'l'arrllll1 is genc t':1 Il)' no r nyo m1TI l'nded, D uring the p()~rparrum per iod, t hl'l.';tf'('u tiL' ,lI1nco ag u l ii t illll \.' ;111 l\t' adlluni,Hc lvd I'o r 6~..~ \w r k:>
-t. In n.'(l ltT ('11I ;ld vn sc r rcg n a l1c y t) ll i C(Hncs . SOIllL' cvickncc :'lIggc~ {S thilt heredirary rhnllnbophdia. is a r i~k. I ~lcr or r;'r tT (lIl"ren l j ~'Ll I J o~:> ;1Ilt1
r11;lt pl:o\,h!'bd il J .\.:1 \VH ll1 a~' l'l'sul r in imp l'(l\'{:d 1'J'('gnanc:' \Jl H~~nll1l'S. ;\JLlirioIl:Il "'fu~ii{'s il l \ ' l'('( l ll i r{' ~ {

:;;':lJhi,,~,'!ci ;; !, c( ,li. ~l l'\'i 2 0 \)f):.11.1:1L;'J -1.14..J


! ~uci,.II::-; 'j '!:, ,\tr!:I,i. !' lw m:b H.h~ mll_';t 2 00~:') I ::()\)---:1 J
( ~ C d l ;l h h , \ , l ;,1. :-:r: j:'vl ..:!O\10: ,~-,1 2 :3 : l- J ~ O
(~n ~ ~ !.(: \' l ,Ii. 1),1,,1,.1 2nOI: 11).');,, (,') )-.1 ( )')~)
782 THE HYPERCOAGULABLE STATE

Inherited Risk Factors


Prevalen ce of D efects in Pati ent s with Idiop athic V TE
General Ourpar icn rs R elative Risk of
Hcre dirarv T hro ru bop lii lia Populat ion with DVT Thrombosis eel)

Anrithrornbin 1.)(;<1 5.0 ~ () . 7 ",~ -t ~


Ijp , j 1 ill 200U
T\pc n 1 Il1 (iOO
Prote in C: d d ; cir !1cr O,2-0.5S{1 40 1 J.t) ( U - IU)
'"
Protein S deficiency Unk nown 1- ,7(1/
, ll 2.4 CO.S- I .9]

Iacto r V Lciden 5-6'Xl 21 {}~, 6 .() : 3.6- 12.0)


Pmrhrol11bill ( i2il2 10 !\ _)l..V
, ,) 6 '~{ , 2.t) ( 1.+-5 .(,:

i 'J : I)\? r h ()lll\.'(y ~ t('i nl'Ill!,l ( 2::\>5rh !,'('[(cnril< 5(~; 1 lot?,:) 2.5 1,1.8- 3. .5')

[k'::\(cd I:, cltlr :<1 :, 2:')(l(h t'Lrccrnik ) 1O,~~) 1;)<:'(, ,_.-, ( 1.5-3.2)
--------
Elevate..l L,n or V III ( 2:')Oth pcrcc n tii c] IUlX) 25'>;) +.;\ ( 2 . ,~ - 1 u)

A..:!a ptn.l from Bauer K.\ . Zwickcrj l. Narurnl ;ll1 tl ( o ;; ~ \l i :l.n [:; and the prcrhrombonc sure. In: l-Iauciiu KL Lux Sf:. ~lU~ s d T l ~ cds, lilooci:
Pnnc.plcs .mcl [1r:1LE i((: vI' Hnn; H\J!ogy, Znd cd. Phibddph i;,: Lippincott \V iili;1m.S & \Vilkllh. 2l)03: 1,lU7

I , An tit hrombi n D eficiency


Gt'nl' r;tl:
<; "nritc,izcd in liH'" 'md ncurr.ilizcs rhrombin, f:lCtors IXa, Xc, :<1:,. Xl b
DiJI2:11v,S cic as says :
,.:\ nrirhr nml' in--}; (':p;llin (oElcrnr ass:'!)' 1:0 m easure b eror X~l inhibirlon ( pl'l' frrrcd) ; an rirh roml-i ;lI)tigen
1h-'l'.') of deflcic ncy stares:
1 Qunnritarivc deficiency
2. Fllncrionall...ldic i l' llC\'--IT ~I(ri vl' l,.~C l1 rcT or hepa rin-bindin g xirc l...ld~'cr
Acquired an tithn,Hnbi;l dc iic icucv:
1. Di e
2. Sc!"i;
,~. l.ivcr di",':lS('
-L Nl,.-phrotic syndrome
5, (.)('-'.l~ion;dly acute t h r~)lll b os is

(), r 'lq \ \I"i ll r!lct'ap)'


lvlan:tgrmrnt:
I. 11d l [( thrcll li\ ,'il.i : Hcp.uin, LIv1\VH . rarel y antith ro mb in concen trate with either heparin or L\1\VH
Hepa rin or LM\VH, H igher do,,,, mar b, required ru achieve a tl1l'r:l!',' uric aPT T. 'v!"" indi\'idll',j, can h,. , ucc'cs"h dk rrcarrd wirh
licp.mn or L!\'l\V H wrrhour the addi ti o n of .incirlirom bu: con ccn rrarc
Antithrombin (human ) con cun rra rc can !.ll' u::.cd 1"0 norm.rliz.: ;ltlril'h l'o1l1 bin levels 111 ,--hrnn icall:' ,tr1t:i l:u,lgub rcLJ !) a t l t' l1 ( S ill spl' n al ~ i r ll ;l
t ions rh.lr p r c~ (' n [ JI1 11I1J((cp[;1bl~' high rj ~k of bkcl...iillg witl: co ncomi r.uir anril'o:lguLUl t lI::.C, su,..-h JS l1('urn:-;u l"gcr y, trauma . .ind obstetric
mrcrvcntron
Antithrombin ( h uman ) conccn rrar..' wit h eith er heparin (I I' low mo lecula r weight heparin (Llvf\V H ), I II ,-~;l ~ C S of .-xtcnsivc \J ! ljt~-.
t1 1L'C ;ltcnlllg rliroml-osis
,.tY711!-'(\llliat:( {d rr: a~': Allfl cn agu iar l011 IS no r ro unnc lv rcconuncndcd l'xccpr in high-li:::.k "ir.U.HiOI1.S such a:::. ~ ll rt~l'l} .tnd pos siblv prc gn, lllC:\', B;b n :l
011 f undi;d pCl1cuJ.ncl..'" of VT E, il-)l1g-(erm prophviactic anticoagulation 111 be co nsidered

2, Prorein C D eficiency
Gl'!1cral:
h ",lt"! u c: ,. \ \ ir.llTlinK-,-lq'cll'-ll'llt pr() ("1l1.s~ lHh t ~si ;:cd in Elll..' JiVlT r.hal: l ' .\:t' ns : lIl r i ,-~o , l g t1 b n t , h:r i\" i r ~' aircr ,lCliv:lnnn bv t liroml-tu
l)i ;l gno ~tl( .issavs :
I , j) 'rn r:lc : ; :'Tl'c!m n cio J1c ("Jmt,j -!: \'icnna, :\u s(riJ ) d il"l'cr. ucrivation (If I'l'or('in C: :lnric\);lgubnr ,15:-:1\'
1

) P r OL1C:! , ll n l l. . h) l ~ ' r j ( .1:',$;1\' ro measure :"I([ i w site activit v


, , P ~"l Hi.'l n C ,1lH lgC!l

~\' ~ws of dt"fic lcncy states:


I, C!\I:Ultlr;ir:v(' dd icicncy
) luncriou.r. d(' ti(il'lh:: '-L) ;l~('d un .ib no rm a] .muciolvtic or JJ1((co agub nc .ictivir v
T H E HYPERCOAGUL A BL E ST A TE 78 3

Acq uired pro tein C defici ency:


I . \ \ '" ri :,,;n
2. Liver d isease
J. DIe
..J-. Sepsis
5. OC":;;lS ll) fiJ llr acu te chromblh is
:Vlan agc !l1C:l1t:
I . : l(!/u lht'rnl'(lHs: He pa r in . L?vl\VH , or wad;t l"ill
\ Varf:lr in can hl' lI:'cd bur p rc . .~ ;l ll r it) n j... ne cessar y wirh the mi ti.irio n p ( \\"arf:u'in ,,-i ll\.' ro GII'I' OC( UlT(' IlC,' (~f \V;lr t:l rin -indll \.~ c d skin Ill'crosis.
K~ q_' p.iticnr fully ,lJlticc1agub ccd wirl: hep arin when sell'ring w:11'!:ll'in. Th e inirial dose sho uj.j be f.1 il'ly low (2 111g t:)[ " ...1.1:'5) wirh 2- (0 ,)-
mg increment s until .1 t herape ut ic IN["Z is ach ieved
., .1~ ~lIlpro!l/.l!i( ;':1!T!('Y5: Jl1ricoJgu!Jr io ll not routine ly recomme nded except in high risk siruari ons such ;15 surgcl':'

J. Protein 5 D eficienc y
Gene ral:
Pm td ll 5: A vitam in K-Jepcnd r nr pro tein rhat enhances rhc an ricoJgtllan [ cHi.,et o f acnva rcd prote in C
Diagno sri(, ;l->S;lY.s:
I , APe .ll1ricoagub nr ass.1Y
2. To ral .md Ircc prote in S ;ll1rigl'l1 <jlu n rifil.'J rio n
~V P(,:5 of d d ic ien cy sra te s:

Total Protein S f ree Prot ei n 5 Protein 5 Activirj:

I .L .L .L
[J ~) ~ .L
rfr H 1 .L

;\ Cl..J ui n' l! p ro te in S d di("i cn c ~' :


I. \V;llt,u'i"
~: , P rc gn~lll( ~ ' ~ > I" rl'inll' ~ r n )
,), ( ) r.l l \,:~onr r; \(:t' p (, lO l1. 1)1<:
,k ,.\ ( U (C dlrn ]ll l'n ~i ~

S. LinT di ~\~ :l :\\'


(}. J nibmm;lt or~ ' dj ~()nJcrs
;\:1 J. n~l gc n lc n t:
S (a l1~hrd. therapy with hl'l);lrin and warfnrin

C) :l ;t' IlL bIll' III C I. N Ej:-'-l l')~ --l:J 1 1:1S 2.5 -- IS2~
:-;<.1'",\';l r:, H P ..: \ ,Ii. g l\l\.,1 19~' k(1 J: 12 1)7 -1 30n

General:
,\l'~ il1 il1l' ..5l)6 ro gluf:ll11 il1l' -ub srirurio n renders f:tcro r \ /:\ rclurivclv rcs isr.uir ro ;lcti v;lrcd p rotein C
D i:.lgno . . ti c J.'isay"';
1. (; clll)[ )l ) ltl g o r
'I .:\~.. tl \'~ll c d pn lrl'ill C '" I\ P ( ~ ) 1'c si sr an c c ,1~ '); l Y ", jrll conf i r m ;ll ' loJ} I .~' g(,llt'l r !'P in ~

Acqu irr-d !:Ict o r \' Lcidcu mu tat io ns:


".:nll\"
~vl ;m;l :C'Ill c n t:
1. ,-l/!!; ( (l,' rl~llIl\lJ/ -; ' S(<I nd;ll"~i rlu-r.ipv Wl d l lu-pari n and \\1;l rr;u,ill
) : t S\, III PIC!!/. /; /( .anurs:
5'::, o ( wlu rc r' l l p t d 3 ( ] n l"l .rrc curricrs t)( rhc ll1.U (; UI\)11. bur. IC I:) 1H1r tOllJ1I.l among n.rnvc Asian .md \ frl"': ;ln pnpuL!r.inlls
C:hrOJlll : lIJ[ I'~(}; l glli:H II Jl l ',\'l(hol\[ rh !\ ))ll b (l si s 1:-. n o r n'CDJ1ll1H:n, jnj
r\\)ph;iICTIC Illl':lsun's :-houid l'l: l~()l1sjJcrcd ,i Ll l"l l1~ b~h -rlS k si ru.mon. SIKh ;IS m;!Jor ')urger;' or rr.ruma
Ocd Cl' l1[ r~\CCt~n\\'s ':l.1ll r;lI l1 ln~ l'~ [ rog(,1l ,1l:C IhH 11.'C0 l1111 ll.' I1...1 r d due [ P ,1 .30-tOld IlKrc;lsc d usk nf thl"l)lllbosis III ,IS\ 'm p r O lll,l r l ( .:arncrs
L"';,r~l'-':-l;,k pl)l'llbnn n :-:'Ln~l'l1ing t()t ' 1:;1((0(" \ ' L,:iJefl is no r cos r-cilecnvc .1r"prOXL ma[cly bOOl) \.. .o me- n woulci llvcd h-J L1C :- c':':l'nvd [n prV\ 'lOIH
:I ,, " ~k D\T;

;),,:n:J I,) R.\l ,r :d. :'--::l,Uh' ! lj')..J.:.h'\):tl-!--l):


! 1 : lill h (~ . 1) ,'t :11. I'!'"..:...... atl ,.\<::. ,,1 ~ ~l l ;~ l\ 1 () ') 3 ; ~ ) O : i ()()' l -l (x)~
784 THE HYPERCOAGULABLE STATE

5 . Pro thromb in G20 2 101\ Mutat io n


General:
~1 u t.l ril' n ar pos ition 202 10 in rhc J -unrr.msl.ircd rl:'~ io n of pro th rom b in lc~l d s [I) increased f'fh,,-icl1l..":' or r";-orhro mh m l.. jl]synrih ' sis wirht'ut
;lfrc\~ rin~ rth' 'r:lu' o( rr.m- cri pr io n
D iag no; ric assay:
\~c l1l)r) "l . . illg
.Vcqui rr-d proth romh in <..-;20 2. 10 /\ ru ut at ions:
!'.:OIlC

i\.l U1:lgt?'I11 l"11r:


I . .1trl h' ,j; r~'u:b"SfJ: SC:lnl/ard rllerap)' wir l: h e p a r in and warfarin
2. A~)' l l lp(l'll/atit' c.nricrs:

Abo ut 2(~'~1 u f [,ill' vvh irc po pulatio n arc ca rrie rs o f t he mura t io n bu r it is nor fl)lll1d am ong na tive ,~ s i <l n an d AfricJll t''1 , )pu!arlllils
Cluonic .1Il tico'lguL! rion for J S)'lIl p co nl ;}(i ..: ca rri ers no r reco mm ended ( ~ l' C Factor V l.cidcn above)

Mixed Risk Factors


1. H yp crh omocvsr cin crn ia
G cn cr ]:
Slll (ll r-(( )ll t:l i n ll1~ ami no acid invo lved in torrnar iun of o rlu-r ;1I11{nc.) :lCids . Ek varl'd levels Gin be dill' to a varic rv o f ClC{(Jr~ . in.:luding IKl"l'd icll"y'
{ h(lmtrl.:'g( lS i t ~r ' l;)r ;vIT Hf-R C(,77T o r l~ysr~l rh i uni ll(, j3-:.- yntha')toB44 iIl ,,6~ mur nrions ) ;ml..l :ll,-qu in'd co nd it ion's
D iagnos t ic .\ 5.';;1)' :
1. F l..;r ing h I11110C;'S [ l' ill C 1(,\,\,1::.
2. AlrllOu\:!h \'rrH FR C677T or cvsra rluo ninc ,8 -~ v ll r h <1 ::' c 844 i n~ 6 o m utat ion s arc a s ~ o ci ;\.r(' d with h i~ht? t" ho mocvsrcinc levels, [L':\ring t~)r rht'~('
rnur nrio n is nor n-co m m cn d cd . . - ' -

Acq u ired defi ci en c ies:


I. FO!:J(c ddi(i (~n( y

2. \ 'ILun in R11 ddlc ic!1c!"


..). [\, d L"nCll' l1c )"

-i-. " 'led ic.ltinll s <eg, mcr horrcxa tc . rrunc rlro p u ui. cho !cstyr;ll,llil1C' , carbam az c pinc,:
S. i\ k nhll l
6. l.rvcr '"ilure
r , l, m ,,1 f lilurl'

,\1.lnag~rn t~ nl issu es:


1, :I (il~t' rl; r.lllll",.~I.\: S r:ln lb n1 rhl' r;l ~))' w irh he pa ri n and warfari n
~ . .'l~rmf';(, lJ/all{ .-lIITia,; :L l)W('[' homocvsrcinc levels. V ir.unin supplcmcnt.u ion wirl: low d oses of to l.icc Cur
re pla ceme n t \)f B1: or L\. V I (;lI 1\ in~ if
,k li('il'm : ca n ')lll ':- r. lI1 ri: 1I 1~' lower Ih I1l10 L~ YS 1T: il1L' ; !lm\'l'\'ct' . ir i.. . unclv.u whether Im\'t'ring lio mocvsn- rnc t ra ns lates int o lowt' r rliro rnb os is Lite;..

i""' Treatment of Thrombotic Events


I
I
Acute T reatm ent of Acute Thrombosis
I. lruna ] managelllell t not ~c fjl..'raily ;t1fc(tcLl b v the prCSCl1('l' (l( Il\TL' d i u r ~ ' rl:- k Ia ct ors
I
I ) l Jn l'r:kll lll1:.H('..i or Il }W m o it' cu l.ll' \\-\'i~I H hef'arin i ~ lI. .lf least 5 ..i.rvs
I ;. L nii.lCrin lU {l'll hq';lrin "hutl ld hI." .idrnu usrcrcd in Jdl'<'lll;![C dO~:lgC" l-cc.iu-c l.ulurc 1'0 t\';\cl1 ,I rlu-r.ipcunc In' l'! o l ;ll1{I ( O ; I ~P I LIl I ~) n wirlun rill'
, i r :; r '24 h() L1I' ~ 111(T l' :l.s e ;.. rilt..' ri:-k (PI' n ' l-U !TCI1CC
I
i + \Vl'l~iH -";\~c,l ," l()~ il1~ j~ H" u n fr;lc rln n:.tfl;j lll' parin wit h an inrr.ivcno us lv ~ld l n in i> r( n'd kudin~ dn-,,~', t;.,J]()\\'c,1 1)~' ; 1 co n rinuou., illrr:I\Vl1(lll~ 111f11-
! -,ton
THE HYP ERCOAGULABLE S TATE 785

Lon g-Term Therapy fol lowiny an Acm e Th rombo sis:


BaLUlcing Risk of Recurren ce Against Risk of Prol onged Anti coagulatio n
R isk of recu rrence
1. h1110w ing an idinpJ [hic event Ll f( l.'r d i~(~) Il[ in lt i ng J- 12 months l)( ~lJ H lc\.)ag\.l I J [inn : --5- 1S'~<) in the firsr year (25",)<>,l[ .5 :'(;J [~ ; 3()~(') .i r
8 ;,'e;n"S:,
.., Aft":T.1 provoked event such ;IS sllrgery . pregnancy. or ora l con tra cept ion. [he rate (. )f recur renc e is lower ( Ch ns tianscn SC cr .d. .1:\ ~'L-\
2005;293:D5.2-.236!; B"6lin cr al, L1I1 c,'( 20 03 ;262:52 3-2 6(1)
j. Generally con sidered highlT in rhc t~)ll ow in ~ situa t io ns: !'/ I)ld increased risk of recurrence in parcn t hcscs)
:vlJlign,lI1cy i> J ,,<)
f\nr iphosphoJ ipi,j Jfmbody syndrome ( ,...~ 2 X )
Compou nd lH"rer() /. ~go . . i r ~ i ~ H' 1:1(11 U' V Lcidl'll ;md prot hrombin C 20 2 10..\ m ururions ( ' - 2.5 X)

Hom \)zyg\.~s ir~~ tor t: tc[O [ \ ' Lcidcn or prot hro mb in G20 2 101\ m ur.i r ions ( --.2 X )
~t.'k crni kindrcds wit l: :,trong din i\."JI pCI H.' [[J nCc lo r .m rirhrom bin. pro te m C, o r prot ein S dclicicncv
The p rL~l' ncc o l two or I11dlT pro l'fll'l)lllb()[il" ri:-.k l:l (tO I'S ( l'g, lncror \ ' I..ciden ;l11d prot ein C ddlcic llcy)
l Pr.mduni P cr al. I3hxl 200 2: I 00 :3 +~4--3 4~f\; and T he f' !'<JC IlT Grou p Bluod C )agul r-ibrin" ly,i, 2000 ;I t :5 1 1-5 18)
-l. Nor ilklT:lScd in Iwrcl'l1zygnsiry tor LlCto r V Lcidcn or p rorh ro rn lu n G202 10;-\ murat ion alone

[ _k S l~f:,I;(l V ..:t :11. ~1 :.[;V f 19CJ 9:.')'11;1)0 l-fil)( 1


I-k i f .1 .\ ,":1 1. i\r.:h Ill fCrIl :"'{\d 21}(')( 1;16l1::'(11-7(")(1

! )r :l l kl ' li ll P n :11. : \ 1111 [u n-rn ;vkd 1\.)9 6 ; 12S: 1- :'

Hk eding risk with prolo nged ural ;lllricoagnb rion


I, Rare or major blcl,.d is . . . 2l)~ per :\.'Jr \vi[h t\ O.4 XI per ycar tJu li[y I
r~H C
.2. T he risk L1lel'ding i'l g rr: ll'Cl' in p.u u-nrs wid l ;l hi.'lro ry
0" 0 1' (~ I L'!c.lding. rcn .il or [ivcr l.u lurt-, d iJL1cres, o r uncuntro llcd J l~' lx rt\'~l~it>I1, or in
rlios wirl: ;l\.iV;J Jl\.'l:d .1gC(-;-.. 75 years,)

1 ~ " Hh R I " r .d. Am , M'.11 J ')\);-< : 1O~ : q J - ~) ~)


luzuuruncc 1) :\ cr ,d, I~l :'ld J 2()0 2: .~ 2S :~;2~-X.i,)
Prl.ucn (~ vr :11. Lancer I q()(';3 -1~~A 23 -~2,j

Long-Te rm T h era py; Ind icat ion s and Gui delines


Du rarion/ In rens iry of "Thera pv

Pr~~d~t'd rirs[ event wul: reversible or rune -lim ited risk


t:K fo r ,:c. pr"gn;ll1(y. "' ll rgcr~::
6 m on r ]. ' Lllp 'r IN! , :::: 2,0-.1 .0
At'rl,"r 6 mon r! , (1) 1 1:-. i~ k r illd l'iinirr .1Ilfi(\' ;lgtJ! arion

R t' (U rJ\' l H iliio]';lrhic VTE Illdl'li ll ire aJlt:iLt);)gu b rio ll


H igh recurre nce r.uc .,irtl;u illn:-i ( c~ . comb ined hcrcditarv Jd~'cr :-; ) -lOrger IGR = 2.()- J. () i scc bel", ,' )
,\((\.'I" ,I Ji l~'-lh re;Hl' lli llg. p .dmo ll:Jry
embol ism l:
CO l1sl..icr 1m..' moh-cul.u weigh t hq ):u'ill :.Jv1a ]i gl1: m ( ~ - ;, ::,s'-. )\.j ,Il ( ..
Thrombosis ar U IlU SU: l! sites (,eg. cereb ra l, m esen teri c. o r hcp .rric vein thrombosis]
Active m'l li~ n~lr1 .. ~,

.\:l' f(': . Dllr; ll'1rJ l1 :md inu- ns.rv I}f Jong- rerm rhtT;lpy
Rccr-rn r:lflJpmJ'I.l'li rn.d:-- l\':dll.\l lll~ :- ( r atl'gi~' " 10 prevc nr rC(U1'l'c:H t'\\'!l1:- , 1 (I ~T .m in ir.i;l! iJ iul' :HhlC VTE have Jel111 m"'(l.'arl'll:
1 -~(ti(J()' n( hnser-d u r] [iCl Il r111'r;)F'~'
13)\nT ru t en-, il y bcrrcr t h.in pbCtho bur ill ['criDr 1'0 Ingher l'.1rg,l'r 1i'1Z .mt:.. Oi1t! l d :ltl O n :I!i-(! rh{':':lfh'll lic ,mrico :l:p d.llio n 1~1 ;- .1-() :n Olllh:-.

PREV EN T Tr i,,1 EL:\TE Trial

INR 1.5- 2.0 IN R 1.5--1.9 IN R 2.0- ,1_0


.., ', .),
._ I>

K"' ,l rUIl {: d :11. ....... E[i\; I ~Yl\):3 '1(}:\)IlI -')\)7

R"H.,, I' " .d. ~I:J.\12 \\O .;:~ IS :1125 -11.' 1


786 VENOUS THROMBOEMBOLISM iJNr;; ACTfOi'i ATi:' D H LPA;; !!\' SODIUiV1

Therapeutic Options For Venous Thromboembolic Treatment Mo di ficat io ns


Events (VTE) D ose 0,.'lod ih cltiol1

REG IME N H "!':lril1 -iJ1d ucd Disl'llnrin lir .111


1 , . , ,.
t h nlJnbc, c: LOP l.'1l ;;1 IH'!-'J n n ,,1 J1 ~ i t: Ll ln g
UN FRACT lO NAT ED H EPARTN SO DIU M ( ~ n: Ch,lptLr 5\)) >
lill:...! I C~ I( n ot
o th erw ise
( l 'n rr;l i n cljcl r~'d.
miriarc rh l.~ l ';l l'~" wi!:h
Innavcu ous ( u n fract iot1atl"d) heparin sod iu m direct tlu-ombm
mhibn or 1.."\ ' (.'11 w ith I ll)
l ...Mdillg dose:
he t):\l'i n-ind uccd
H eparin sodi um RU units/kg b}' int ravenous push,J,l!OWL'..l by:
(hrolllho cyr.nl'cn i;t
A f fl ; II I W II1lU dOSL':

He par in ~' hi i ll m. hcgjn wit h 18 un ir:-/kg per hou r b)' conrinuous inrravcuoux inll l:-inn di luted J .i fc - r h rc :HC Jl i ll ~
in J co nvenien t volume \)f 0 .\)1.)/:) NaCI injectio n ( O . ~ {X1 0lS) 1) [ 5 l X1dextro se in ject ion ( D5\V) hlc<'t1ing ...

\Veigh t-Based No r m og ram for I u rravcn ou s H epa r in Ad m in ist rat ion


.II'PT Boills M ai nreu.incc D' lSing'
Adverse Effects
I
< 1.2 X cont rol fll unir,:; /kg DC(l"I';\:il' il1f(l~iOIl bv 4 unifs/ kg/ hour l . Hl T: I -..V~<l of ,),nil'ors !'l"n:i vin g
I
I un t"ra("[iOll<l [cd heparin tor > I week.
12 -1.5 X co n trol 20 il llirs/kg DC ":l.T;l~ C' infusion br 2 lln ir _~ /k:;./fl\)ll ::
I ,S r,- Ch"lorr:' 59 >
1.5-2,,' >; ...-o n uo l :.;" b~d ll ::: ~' ll ....~ h ,\l l ~C In muu cn .mcc LI ~ 1St.:' ") i':i.,n -tmmUnC-111l'lli:l rnt , lll'p:lrin ',b Stl(l :lLCll
- rliromborvt opr- ru.: ( H i\T \ l - :~ (Y~~.l
2..,-3 X con rro! ~o ch:lngc IllCrt:a::.c infusioll rare bv 2 llnib /kg/h oul'
R(' s u h~ ~-r() m rempnr:l ry pbukr ,l::;g:l\'g ;l-
; X ... \ H ll r~ ) 1 H"IJ ill fi..l~ioll ( Jr l ho ur
,
Increase in fusio ll r.ur bv -+ llnil~ / kg~/li ulir cion . 1l1:lrgillacion , J-nd p~ l' i p h r ra l ~LlJll~:'
- CLIt ion
D I.~ Ct'l.~J~L' d platel et ('l)UI)[ usu.il]v mild
and rrausicn r
l ]wck ,J!Y l-r ('\\'1")' ('I hours until dll' L1P\'llfic r~lng(' ;' 1.5-2.S X a PTT con rrol ) is sustained.

W lu-n rli cr.ipeutic r:lnge is ach ieved. dll'ck ,1JY I-r t()l' 2 consecutive rnc.isurem cnts six ho urs Onset (;'Picailr 2-4 lb !'~ a ((er SrJ IT of
111. pal'in
Jf'"rt: then check JPPT e\'e !';' 24 hour,
.Fhrorn bocvro pc ni.i reso lves ~k $ r~ i (r C~H1
:\,'1. Pn"T.m..~ hl'pari n i n fll~ i u n ~ in .1 srand.uciizcd (Orl(Cll[ra~jOl1 char f;lcilira[L's c()nvl~ITitlg
'/(:
ritlu il1g hepari n Jdnl in isrLltiol1
cl dmil1j ~rr;tti(Hl ra t es o f .. ul1irs/hour" [0 volume ," t im e (ml / h j" ; tor example, heparin so dium
3. () S (C O FK I lI J/ ( ~~ ( C ( 1 p()l"l)s i s
25,(1on llllir:-, in 25l) InL 0 .'-)1;'(1 NS produces ;J clin c{, 11l~;\ r ion ~)( 100 1IIlir; /mL
4 . l irpo:\ld os r l't\)ni~m /h~pl.'rka lcmla : 7_~ l\J
X,1/(: IrHl':I\ 'I.!lOI I... ( lIn fr:tCTlo n arcc{') lll~p:l rin so d iull 1 ~hl)l.dd 111)r t,(' u~cd wirl: paricnr-:; \V11O luv
of pan l. " IlIS
b;b llillc ,1bnor111;l! :IPT T
5. I n cn'a ~ed se ru m (ree T .. ,md T i wlr h nor-
Ill.,ITSH
(', Incl''':I'c'd Ilc'I'''ti,' :\ ST "".I ;\ 1T
Monitoring Th erapy
- I lIl~n_" I:'-c,.J "cnlm ["rj ~ I~ l ('ridl.'s
Ck,c]z ;ll)"1'T (-, hours a ([c ~' ~ LH' l'l l l ~ ... olltilllH..'U$ ilHL1VCl1lHlS Irlfu::.ion ;l1H.i ;lfrcr 1.'~Kh Ch;Hh!l' in
K. J-I~'tll'r:' r n:-iri\'i l~' rcac rron s r.ur lv, ll1dlld
I

r.ir r 0 1 :h.lm ill l:-- lT;mO I1, \ l. lI l1l.1 in ..l~i)TT J .5_2.5 .Y: rhc UPF'\T lun ir Ill' 1101'111.11 -
iug V,l ~ t)~ P~bll), cu r.uuo us l.,n~ . '. rs iroun ...i
(he in jl'Ct io n s ire . co n jun c n vi n s
~). F('\!r r
VEN OUS T H R O M B O E M B O LI S M ro w A1:JLr::CULA R \/iE lc;HT H EPA, RINS (UNNHs) 7 87

REG IM EN : LOW MOLECULAR WEIG HT Treatm ent Modifications

HEPARINS (LMWHs) Tlu:r is cu m u l.rtivc .m ri- I.icr o r :'\.1 .u-nvirv of :


L :\'1\VH in p.iticnts with renal imp.urmcnr .
ENOX t\ PAIUN SO DIU M, T INZ APARIN SO DIUM \\:hi('h (an increase [he ble'l'din,l co mplic.r-
uons of rhcrapv, In p.ui cnrs wi rh severe renal
ellUl:.'II :
f:lilure. auricnagLll.uio ll w it h u n fra c n o ua rcd
OQ,'" tor L:Y1\VH, or untr actio narcd h"f' "r in cannot he' mcd inrcl'change'"bh-
hep arin is rccou un cndcd . If LIv[\V I j is
Do nor us.. "n~' Livl\VI J in a patient wit h hcpanu-iuduccd rhro mboc vrop cma selected. d')Sl' mon iroring of anri-:\ .l ac t ivir}'
En oxap.u-in . .od ium 1 mg/kg (wice lb ily or 1.5 1l1g/kg once lb i J~ SLJb(l\ul)(,ullsl~ ' ~)nly is recom mended

T inz.rp.uin sod ium 175 JIHi -X:l un its/ kg once dady subcurancouslv ~\I!{\' Renal [mpaurm-nr

Hno xa pa ri n
----_.. . . .. '." .. ---.. _.. . . .. .. . _.. _.. .. .. . .. _.. --_.. . . . ----. ---. . .--_. . _.. _. .. .. --_. _ ". .. _ __ . . _ -'" -- . -----.. ---, (:rc:l r i n i llC 1L';\ 1':111(l' (\ d lll i n i ~ hT 1 mg/kg
<.;OmL/JI1ill OIlCC lJ .l i l~'

Toxicity ':'l l b n l f ,l Jl t"'O U :\ ly

1. Illcr(':l~t',-i risk of bleeding in pat ients who lise drugs that !Tl;\~' ,l n~'c r h L'IIlOS Cb is :1 nd in those Ti nza parin
vvir]: ,.:.nnw rbid F'cnhologics rhar predispose (0 hCl11nlThagc ~~r(,:Hinlllt clc.i ranr c :"In ~ pr .: i fi c l\'( O m lllf Jl-
( C "' r: <30 mL/min d;ltitln to .d ti'~ r
2. Thrombocvropcniu"
ljll"1.:I~'arin dose or
Incidence o f IIIT is less rh'1I1 wirh uutrac tionatcd hcpar rn
_ _ _ _ _ _ _ ~c,._I,_d_c _
Pla t elet C o un t In ciden ce LMW H Implicnrcd

1..31}~1 Enoxaparin
50,OOO-IOO,OOO/ 111l11' Monitoring Therapy
l tV
,0 Tin za p ari n
In 1110 ':.( par icnrs. no mo n itDring is J1L'cC'ss;lr~.
- 0.1;.I/~l Enox.ip.irin ,
In S I)Jll(' (ir<:t.lIJlSCIIlCC:-; such ;l S I.'regnancy.
<50,OOll /", ,,,;
rena l . . Irs(un crio ll. obes rtv, .uui k>n'::!.-rc rm
D.l.">;] Ti 11/. ;\ l-'_l J:lII
rhcr.ipv.. .1 dus t' adji. lsrtllc; ', t BU y be ~nt.:ccssaf :.
For rwicr .d aily ;h..1ll1 lnISrLlrioll. rJrgct rhcr.r-
pe litic r:1llgc is between 0.(' ,md lt) lInl r:-;/IllL
J. P:1in .u iJ1J l...... n o n ~i (' . . /i nJ('LTi{m -,ill' hCm arll\11:1
J - l hou rs. :1 rr cr S U b CU L U1Cl..)U$ d n ':. \,., bur for
4. Reversihle increase ill i\ ST. ALT t- .rlkaline ph"'I,h"r"st", LDI-I. .uui CK oncc-d.ulv dn~(' t he r.uv cr L lI1~T(' is ks\ W l"!1
5 . Decl"l":lscS ill bone m ine ra l dCI1~ i t ;' J lh.i osteoporos is ( b ur less fi'C'qllCI1( rh a n wi th un trac uo n- c$t.,blislt,:d (us u.i llv bl"t~\(cn ,lnJ to I
hq'"rin ) 2.0 un ir$/ ml,"
~
,l t d

h , LJrhl'r roxiciric: -
E I1 0, ,, pa rin
Notes
IhT ,.,..b k llli"
i\ \.I \',l1l [J gl~ o l~ 1. ;'v1\ VH s. over t1nfra([ioll;lr~d
Dt"b:'cJ hY ~'L' r~l'Il:> i ri'i i [y rcacrions
hrp. ui n:
Cur.mcous nC (l'():-> i ~ ,md I;l( necros is
T lu. nnboc vros i, [ a rely
I. Increased bioavailal-ilir
"') Lower incicicncc l)f nsrco pn ro ... is
T' in z .ip nri n J. Lower incidence or ht'~'aril1 - l!1d ll ~:cd
I n CIT:lscd :\('I" U It1 t"rt't' T j ;I nti T .. wirh 1101"m al TSH rh n' l l1boc~' rl)!'lnia
:\ ng ltH'd r lll ;J ;ll1d ;dkr gic purpucl
(,~ U r ;'lIl en li S I1l"(n)S l~ .1l1d r.ish cs
PrLlp isllI. r;trd y
788 VENOUS THROMBOEMBOLISMS ,',fAR rA i-Ui'! 50 D i Uivi

REGIMEN Adverse Effects

\VARFARI N SO D IU!v! 1. H Cl1lurrlt :tgc': P,lficll rs wirl: :J vari.mr l)rdynhlrphism /(1]' rhe ..~yr:()<h r(lnw 1>450 C1"P2C9
cnz;'mc mJ)' have Jil increa sed risk of .in l' ,'{J g g l' r J f Cd rCSp~)lb C [0 w :lrl':l r in ,1nd hcmc:Th ;\ i.~l'
") \Vad irin Clllhl'Y('l~).lrh:' or l~'ra l w a r J:I1..in -vncirom wiri: congcnir:ll :Ibno rm,diric's ,I( rc r
\ Va r far m sodium o r~l!!~' once ,.:Ltd }" conrinu - l'xFosun::: i ll U (c.:.'1\). 1"'(I~I~l ril1 is'l!;sllIHt((" "~l lJ l r, l i1iJintld il( ( \ \ 't'L'lI It L' lid: iIIld J 2:1: '. '.: iy L 4 pr:'gl/,//j::'l
ally ro maint"i" ,\I) INR o f 2-,1 ~. :\bdom in:d pain, (r:lJIlFi ng, Hatulcncc. blnaring , nausea. \:om iri:1g. di :lrrlll';l
-to Increased LFTs, cholcstaric liver injurv, jaund ice
lniu.ir l:hL'r ap\' wirh ,111 oral I,.iOSl () ( 5- 10
tllg/ d.iv in rhl,.: cvC'n i n~ J lh..i increase dose in 5. Ne phritis with acute renal fJillire
i n~rel1l :'nr, o] 1- 2 mg/day guid"d by 1,11>0- 6. U rolithiasis ( ('JICIUm oxalate)
rat orv v.ilucs T, In crc:lsed liver Cnl.ynH:s, ht' j,;tti[is, and a syndrome rliar ll1im ics vlrallh:p;lciris
8. Skin necrosis (risk [' lC t,or, include pro tein C and h'Tari n..induc ed t hro rn bo cvro p rm : C"c
\V,lri';lI:m is ;w;lilabic in th l' Unitl'o Srarl's in repurts o f prot ein S i..k ficlcocy and i.rcror V L{ j,..{rn)
man~ gl'ncn c f ~ \ l"1 rll1 b [ il l11 s
1;)1' oral adm uus -
~. R ash, ...icrmacitis, bullolls LTllp r JO l1.S, urri..: ar i;l , prunrus. ilJh.f purpu ric sk in rruprion
rr.irion. i'H..-luding um ncd iarc-rclc asc tablets
1G. Alo pecia
cl1 ntaining I. 2, 2.5, s. -t, 5. 7.5, and 10 mg
w.ut.u-in Slhllllllljr,lblcr I I . Os teoporos is

Mo n it o r ing Th erapy
" '!')JJ iror INR Initially 2 t imcs per week.
\Ylwrl :,ral'i1iLcd, gradually reduce ii'('q ucncy
ro U I1(t' !'cr \\o'('\k

Note s

L)m.; i! i1tT d a lL1/1.'.

Anribidrics ( eg. t'l'}'rh ro mycin. rnc trom d az ol c. rif:llnp in, Huconazolc :


Anr icpilcptics ( cark llllaz cp inr . barb irurarcs)
I.l r,id.!o\\'crillg ,lg en ts :>g, clohbrarc. cho lcstyram ine. lo vasr.uin )
l-:~ld i!ll ( r,ld i ..-us.
;vbn!' \'cgctabks conta in high levels of vi tami n K, but paricnrs do nor nt' cd to spcciricallv avoid such toods
P;ltil..'l1ls -, Iln ull.i l-c insrrllc[cd ro keep I.li cr. f:l i rly urul orin ;lI1d ;ll.lju:-> r rlh' w.u(:Ilin \.iUSt' ;IG:o rd i ng ro IN !\.
.:\lcoh(\j in small t.Jl I<lnr irir:, is lIsuall}' rnll.:T:l rC'1 1 wirhou r .dl tx ring I'l'orh rom bin riaw

Reco mmendations ~o r Ma naging Elevate d IN Ks or Bleeding in Paticncs Rece iving Vita mi n K ;\ ncago n ists"
C on d it ion D es cr ip tion

10-J1~ .lb,w(' ri tcrap r-ut ic r:1I1gc. 1.0Wt'l' dOSI: or omi t (ht:.;c. I1lt )l1iror morr (rnFH'nrly, ;\n d n-sunu- at 111\\,rl' .iosc when r:--:I{ is rlwr:1 t' l'lIf1C; if on1: mint -
btu <:. S.D; no '\ i~ll ifi( ;1n( mally above lhe rapeu tic r allgl'. Ill ) ...!O~l I( ...iucuo n m,ly LIt: n:o..jlli r('d

bl l"l' d i Jl ~

[~l.z 2::.:'.0. bur <>1.(1: no Onu r next 11 )1 2 dOSL::'. mouuor more I requ ern I", .lJ1d rcsunu- at lower d P:'(' when INR 1~ in dwr:ll'cutil. 1';lll \'r.. vltr-rn.rriv...lv,
~q~~r ll iil,."ll ll Ldll... il nl~ omit do::;.. ,H],..I gi\'t: vnarnin K' t :S 5 :n~ {)r:;ll~" ;. ~\:lf[il,.tdJd: if .u mcrc.rscd nsk D( blct:di l1~. If more L1p il,.i rc~~cr:-;I1 i... '
rrqum:d L"...-;ul:'t" the p: uit'l1l n~l'...l~ u r~Ynl sllIy;cry, vil:ll1lin K I : 2-4 m~~ 01'.111<, (' ;\ 11 Lie ~i \I.n wid l lh e CSpt:cl:ttilill lh:lt ,1
n:.Jucli ull o f dll' IN R will OCCll r III 2 ~ hUllr::;. [f the I\:R i:-> ~ti!l lligh. .tddiuoll,d viumin K 1 , 1- 2 111 ~ ol".lll< ( ;U1 l It..' g l\I,.n

r\: R 2:::. ~) .{); no '- 1 ~~ n i ti\,l1l r Hold \\',II.-(,ll' ll1 rher:1\.,y :1111..1 ~i\'e h i ~~ IH' r 1.-10"'e nl- \'lum!n K1 :, S- !() m~~ "f;}ll} ':' w ilh rlli: '-'; !)l{-Llti, m tl"':;tl : 1111' I:"':! ~ \','11 1
b j (,L'-.li l 12~ be rt.'Lluced s-ub:,u THiJllv 111 2 +- ..f ~ 11 \,)[ 11'':>. :\1Dl1 iwr 11101"e (n.:qul"Tu k .lll t! t1:'l' ,h:.kiili (.,n;lI , iLl lil ill f( l i( n C l,.- ": ~S;ln.
R.l::.tll11l. rbl:rJF~ ,n l ow~r I.l llsc \dw ll r:"':R l~ d1lT;tpl 'ilti .: . , ,

Sniolls bln'd l Jlg ,ll .111: ' Ho ld w:llfal'in dw r,l p: ,111d ~ I \I~ \"il.ll\1l11
K I 1{) mg h.\' .;Ill\\' 1\ ' lll fil -;ion '. SIIl'l'k JllcllH :\.J with (resh I' h" ll!-:1 1'1" !' ro-
1.!t" ,ILl IlIl uf I:"R l h r~ ) 1ll l\ 1l 1 co m ple x ( OI1( l'nl r;lU:, J l'f.'(J hj i l1 ~ 0 11 the nrg . . n .... \' o f {he ~ ll ll ;I ll() ll ; rl,.C'O lll b !ll;lIH !:ll,.-t n!" \"11.1 :11;t\' lH" ':1.~ n .. i,J
ned :IS ;l!l CrJ1:l11VC ['0 f-'I\llhro mbm coml'lex l"OI1CCllLr a lC. \ ' it.lmi n Kj ( .111 be rqw:l trd t ' \ ' (]" : 12 hour.. .

J-!\)ld w,lrt';)rin rhl'l":lpr :lJhi ~ i\T'prorhromhlll (OI11p k\ (O Il(c nf r;1{I: c'llppkmrll[ l'd \\'lrh \' ILlIl li l1 ;":1 : 10 m~ lw "';I1W 1\
1l1ill,S l l )n , R Ctomh1Jla m f:ll wr \" 11.1 U13 \ be (I H l :-d i~r(' d ;1 ... .dtcr n ;IU\,(' ro l' ro, hrom blll (:Dnl!,k.'i ,"(l l1t l n l r.H(': j"q';~ ;11 ~ t
n((c~s;.\I':". ...It:pt'' ndillg on r~F

"!! ":'.j ~lllll;'l il1::: .I" r! .' :lllliu:r.l


r) i:C.lll.d .d l c r ill ~ ; l ,j llSl.:S ,,( ql :I: r:;1t f< 1. ;:il t:i1 h t}' ,ll'lJ ; ( ' 1" L \ j \ \ 'H ...m L,C' ~ : \LllllJ I Uj ti l, ..tl~..:l ., I )t
I . I I I... \1t.' 1ll1l1 r~ 1 i).' hl'C~ll l"t''i': :;;t:.i .I lld tih' ~':l1 ; '::ll
~~\ " ll~ l " r...P(l:h l\"\: [,I ,\.-.;r.1rll1 lh \ : r" r~: :--.i'lIl th;;( l:-':R \: : j llt~ ) ;..1.5 :11\" It\:' n: Il.;hl.- lil:1 11 \ .;i-!,... wllhm <\I" 11\' .;1" dw rf,,r. H',I:U C' r.~ :l;!.,. l illl~, li w,-(' ::\ " , i ; j:~ :( .. :c ;: I",., (":H .:1 .1! ' I" " \I! ~ ) . l l ..
phi, ~ , > ; itl~~ii I>';"..
VENOUS TH RO MBOEMBOLIC EV E NTS ( V T E) ANT!T I';,~OI':iDiN il! (Hi) V :~. i'J ) 789

REGIMEN Treatm e nt Mo difications


Inf usion-rcl.trc.l DI.';:rcJSC t he r.uc uf
ANT IT H ROlv1BIN III (HU MAN) ( T H RO ~mAT E III)
ad nunisnarion. o r
Anr it hro m bin HI ( units required") inuuvcnouslv over 10-20 minutes. to llowcd D\': inte rrupt
Antith ro mb in Il l ~.6\"Y~'~ 1 df iniri:l( d()~e) evcry hou rs as ne eded ~ ryp;('. ;d lr 2- b 'day:-:~
24 .idmiu is u-uion until
sYJn ~' to m:> all.H:L": dl.::n
1. D,,)'j(' shDuld be ind ividuallv .icrcrm incd to increase plasma anritliroml -in III conccutra no n re sume :h 1111111I ;..t l.,;} f l u n

!..'k~lnd f :\T- rIl:i pb~ n);I) ro withi n ;l ran6i.:' of n{)rlT);~1 tt"


nl11 rlu- pn:-rhc::ai' )" 11.\cl :I,.. .l~c1i'w .1( ;1 -dower r;Hj,'

>\T-IIIJf,loslllo). l),,>; .c ma v be calcul.ircd from the loll ()w i l1 ~ lorm ula:

"E.\t' I \' ~ " ' ~i .1.) ":, :lfln~l;;l i .\ I:.HI 'pl.l~ nu 1';ls~'d ,lll fl.:lcritu lal :\-1~ T11 ;h YlI', rill' l~lr::llILl rs l,.l ~ \ , 1 on au cxpccrcd 111, 1,'

l : lcnr ;;i III \'1\',.1 1(','"vcr:' ~ I t: .l r.t" than b:"' ~\'l il1 \_' 1:\T-lllj Fia::-: m,l t.l ( lA% Fa ILl/kg ,hlmini:--ft';"r,i

2. E1 ill1i ll,\riull halfl ite is af'proxinl;" cly Hi ho ur,


l , .
A dve rse Effects Mo nit o r ing Th erapy
ln ru slo ll1'I.,Lt[('J even ts: I. Mon iror pe.ik .mtirhromloin Ien'l, 20
1. DizziI1L's</l ighrhc aclcc1l\c'ss minutes J.frcr infusion ;111l.1 tr nllgh levels
2 . Ches t rightness or pai n jusr prio r to ncxr dose ;lJ1d ,ld jusr [0 kcq.'>

" . ~,l ll$Ca


levels .il l\'\. ,' t)O~/{l or
norm al
2. ..v lso rCI.-'u11l iTl cnd ed ro . . hcck .uit uhrom lun
ok DYSgl~U$ iJ
levels 12 liours ;lft CT the illj[iJI in fusion
5. Chill,
. .l , S im u! u nL'o us heparin :h.{1Tl j n i~ [l:.l ;ion GHl
6. Cramps ;Iffc('[ half-life and more ti:t"(ltlC l1t mon i-
- Slwrl:l1L'sS or brc:lI"h t o nIl ~ ma y be rH'Cl' SSar r
1. The laboratory evaluation of unexplained prolongation in the PT or
PTT should include a mixing study.
2. The evaluation of a prolonged PT or PTT includes consideration of pre-
analytical variables eg proportion of plasma volume to rbc volume.
3. Prolongation of the PTT does not necessarily imply the presence of a
bleeding disorder.
4. Any prolongation in the PT or PTT should be evaluated prior to an
invasive procedure.
5. A common cause of a prolonged PTT is heparin exposure.
6. Levels of Factor XI do not correlate well with bleeding risk.
7. The level of any single coagulation factor does not correlate well with
the severity of prolongation in the PTT.
8. Patients with Von Willebrands Dz and mild Hemophilia A and B may
have normal or variably prolonged PTTs.
9. Family and Personal history are the most important test for a bleeding
disorder.
10. A Lupus anticoagulant may cause an apparent inhibition of several
different coagulation factors in the instrinsic pathway.
1. A risk factor of DVT is ethnicity.
2. AS patients age the risk ofVTE increases (esp. after the age of 45 years).
3. The recurrence risk for PE > DVT, the risk of mortality is PE>DVT.
4. Multiple risk factors for VTE increase are cumulative.
5. Practically speaking all malignancies (x in situ or skin) are associated
with an increased for VTE.
6. Relative Risk for incident VTE > RR for recurrent VTE. The Abs Risk for
incident VTE < AR for recurrent VTE.
7. Risk of PE in proximal DVT > risk of PE in distal DVT; however, there is
no difference in risk of PE by specific site.
8. The treatment for Proximal and distal DVT is similar.
9. Population studies indicate that 30% of PEs may be fatal.
10. Post-thrombotic syndrome and recurrent VTE occurs in about 30% of
VTE patients. Chronic Thromboembolic Pulmonary Hypertension
(CTHP) occurs in 3%.
11. VTE is a chronic and recurrent disease and should be treated as such.
12. When considering long term treatment ofVTE: Location, >l st episode,
idiopathic, risk factors.
13. Treatment can be divided into 2 phases: 1. Acute treatment, 2. 2 n d
prophylaxis (long term treatment or indefinite treatment).
14. Optimal therapy for provoked (correctable RFs) is at least 3 months.
15. Optimal therapy for idiopathic or high risk DVT > 3months.
16. Duration of indefinite therapy depends on Risk/Benefit.
17. Perioperative bridging therapy depends upon the pre-operative
effective plasma elimination t/2 of the AC used and the postoperative
bleeding risk.
18. HIT(T) must be evaluated based upon the type of AC used and pretest
probability and the detection of HIT antibodies.
19. Thrombocytopenia is not necessary for the dx of the HIT(T) syndrome.
Objective:

To Present Some Commonly


Coagulation in Clinical Seen Cases Involving Defects in
Practice C/A 2009. Part 2 Blood Clotting.
Roy E. Smith, MD, MS
smithre@upmc.edu To Gain and Understanding of
the Complexity involved in the
Care of These Types of
Patients.

25 yo Asian-American college student and No PHx/ FHx of thrombosis or other


Rugby player complains of RLE pain and medical disorder.
cramping in her calf.
She takes oral contraceptives. BMI
She does not recall injury her leg, but 24. Non-smoker
admits that she plays aggressively

Her Caucasian room mate has recently Calf - tender to deep palpation of
been diagnosed with a DVT. the muscle. Homans sign (-)

She wonders whether she could


also have DVT.

Does her Asian-American ethnic back-ground


VTE Incidence by Continent of Origin or
place her at a higher risk of venous thrombosis
than her Caucasian room mate? Ethnicity
1. Yes Ethnicity Incidence (SD)/100K
2. No African-American 138 6.5
White 103 2.1
Hispanic 61 2.8
As-Am/Pacific Isl. 29 2.4
Native Amer. 33
Her physical examination is non-specific and Annual Incidence of VTE
she has a known risk factor for venous in Olmsted County, MN: 1966-1995
thrombosis (ie. OCP), you decide to evaluate By Age and Gender
her further. 1,200

Annual incidence/100,000
Men
1,000
8/100k
800
What are the risk factors for a 1st episode of
600
DVT? Women
400

200

20 9
25 4
30 9
35 4
40 9
45 4
50 9
55 4
60 9
65 4
70 9
75 4
80 9
4
15 4

85
-1
-2
-2
-3
-3
-4
-4
-5
-5
-6
-6
-7
-7
-8
1
0-


Age group (yr)
CP1198340-1

Annual Incidence of VTE


by Event Type (PE vs. DVT)
1200
All DVT or PE
Annual incidence /100,000

1000 PE DVT
DVT alone
800

600 Pts with PE have higher risk of


recurrence
400

200

0
014 20 3034 40
2024 30 4044 50
5054 60
6064 70
7074 80
8084
15
1519 25
2529 35
3539 45
4549 55 6569 75
5559 65 7579 85
Age group (yr)
Silverstein, M.D. et al. Arch Intern Med 1998; 158:585-593. Copyright 1998, American Medical Association. All rights reserved.

Normal annual incidence of VTE = 0.008% or 8/100k


Although her physical examination is non-specific , Which location of a DVT is associated with
you decide to evaluate her further.
the lesser risk of subsequent PE?

Duplex Dopplers show that she has 1. Below Knee


a deep vein thrombosis of the
peroneal vein. The blood clot is
2. Above Knee
located just below the knee.

Is the risk of PE greater for a DVT


above the knee or below the knee?
0% 0%

e
e
ne

ne
K

K
ow

ve
bo
el
B

A
Although the frequency of pulmonary
emboli is less in patients with DVT 20%
below the knee, the risk of developing
a pulmonary embolus remains
substantial.
80%

How Should She Be Managed? You Conclude that She has DVT of the
choices 1-3 low risk for PE; 4-5 high risk for PE Peroneal V. Below the Popliteal V.
1. Reassurance
2. Anti-inflammatory How Should She Be Managed?
agent 1. Reassurance She has a DVT in an area that could
3. No intervention. easily extend into the popliteal space and above
Recheck Imaging increasing her risk of PE
Study later. 2. Anti-inflammatory agent Poor antithrombotic activity
3. No intervention. Recheck Imaging Study later.- This is
4. Coumadin only for
a
month. 0% 0% 0% 0% 0% reasonable option, although not highly recommended.
5. LMWH/UFH/ 4. Coumadin only for 1M Coumadin is inadequate for
Fondaparanux +
.
ce

acute DVT. 1M duration is inadequate.


...

fo
...

..
an

or

n.
n.

ly
ur

at

coumadin
on
tio

Fo

5. LMWH/UFH/Fondaparinux + coumadin This will be best


m
ss

H/
n
en
m
ea

LM adi

UF
rv
la
R

te
nf

H/

option to reduce risk of PE and post-phlebitic syndrome


in
ii

ou

W
o
nt

C
N
A
Although the frequency of pulmonary
Below the Knee DVTS that are
emboli is less in patients with DVT
proximal may be at greater risk for
below the knee, the risk of developing
pulmonary emboli and more severe
a pulmonary remains substantial.
post-phlebitic syndrome than those
located at a distal location below the
There is a reasonable dispute knee.
about how these patients should be
treated based upon the risk/benefit

U.S. VTEs
Non-Fatal VTEs 613,423
Fatal VTEs 295,370

Total VTEs 909,793

Things to Consider

VTE is A Chronic and Recurrent


Dz 1. Atypical Location
Patients with symptomatic VTE 2. 2nd Episode
have a Greater Risk of Recurrrence 3. Unprovoked (Idiopathic)
4. Non-reversible Risk Factors
This is an incident DVT. Should she
be evaluated for other risk factors?
1. Yes
2. No She should be thoroughly
evaluated for underlying
risk factors.

0% 0%

o
es

N
Y

Annual Incidence of VTE


in Olmsted County, MN: 1966-1995 She has peroneal DVT and takes oral
By Age and Gender contraceptive pills.
1,200
Annual incidence/100,000

1,000
Men You discover that she has a
8/100k
800 heterozygous Factor V Leiden Mutation.
600

400
Women Intensity and duration of treatment
200
partially depends upon her risk of
0
recurrence.
20 9
25 4
30 9
35 4
40 9
45 4
50 9
55 4
60 9
65 4
70 9
75 4
80 9
4
15 4

85
-1
-2
-2
-3
-3
-4
-4
-5
-5
-6
-6
-7
-7
-8
1
0-

Age group (yr)


CP1198340-1

Peroneal DVT, takes OCPs and is


How would you treat her DVT? hetero FLM.
1. She is young, has How would you treat her DVT?
extensive DVT AC
for 6m 1. Her DVT is not extensive and it
2. She has idiopathic is a first episode > 6 m of AC
(unprovoked) DVT - is not indicated.
AC indefinitely
3. She has provoked 2. Idiopathic DVT Indefinite AC
(reversible RF) DVT- 3. Provoked DVT by a corrected
Correct risk factor; 0% 0% 0% 0%
AC for at least 3m risk factor- , Correct RF; AC for
4. She has a provoked at least 3m
...
.
. ..

i..

..
d

ok
th
as

ke

DVT- AC for 1m
ov
pa
,h

4. 1 m of AC is inadequate
vo

pr
io
ng

ro
id

a
ou

as
as

as
y

h
h
is

he
he

he
he

S
S

S
S
DVT/PE Treatment and Prophylaxis Paradigm Factors influencing risk of recurrent
VTE
Heparin/LMWH Coumadin/LMWH Coumadin/LMWH site of initial VTE
5 days 3-6 months > 3-6 months PE > pDVT > dDVT

Acute Treatment Phase 2nd Prophylaxis circumstances around initial VTE


idiopathic > transient risk factor; SX vs ASX
Provoked initial anticoagulant treatment
early therapeutic anticoagulation
Idiopathic/2nd
Indefinite duration of anticoagulation
Aggressive Thrombophilia at least 3 - 6 months
Indefinite
presence of a continuing risk factor
cancer, aggressive thrombophilia

Cumulative Recurrence of VTE Aggressive Thrombophilias


Hazard ratio/1,000 person-

40 200
Homozygous mutations*
Cumulative recurrence (%)

30 150
Double heterozygous mutations*
APLS
20 100 Deficiencies: PC, PS, AT
person-days

10 50

0 0
0 1 2 3 4 5 6 7 8 9 10
* (FV Leiden/Prothrombin G20210A)
Years

DURAC I Study
6 weeks v 6 months oac after 1st VTE

Schulman et al, NEJM 1996


Optimal duration of OAC WODIT DVT Study
3m v 12m warfarin after idiopathic DVT
after initial heparinization

DURATION % RECURRENCE @ 2y
DURAC 1995 (897) 6w v 6m 18.1 9.5
1st DVT
transient risk factor 8.6 4.8
distal DVT only 11.4 5.8
prox. DVT only 21.3 11.2
pulmonary embolism 26.8 13.7

Schulman et al,
NEJM 1996

3m v long term warfarin


after 1st idiopathic VTE Bleeding complications with warfarin

fatal haemorrhage 0.25 - 0.64% per


year
major haemorrhage 1.1 - 2.7% per year
Achieved INR major bleeds (% per year) minor bleeds
<2 1.5 11
2-3 2.5 12
3-4 2.5 15
4-5 4 20
Kearon et al. 5-6 5 34
NEJM 1999, 6 9 96
340:901-7
van der Meer et al., Arch Int Med 1993

Determining Risk of Thrombosis: Heterozygous FVL


Duration of anticoagulation carriers.
RR for incident DVT (age 30) (10/100K) on OCP (30x) =
incidence 300/100k person-years= 0.3%/py
If recurrence risk is low
3-6 months RR for Incident DVT (age 55) (123/100k) on HRT (8-15x) =
incidence 900- 1800/100k py or 1.0 - 2%/py

If recurrence risk is moderate or high Recall: The Risk


of DVT increases Perimenopausal women
Annual incidence/100,000

consider long term anticoagulation with age. incidence of DVT = 123/100k

BALANCE
1,200

1,000
Men
800

Bleeding v Recurrent VTE RR (OCP) > RR (HRT)


600

400
Women
(4% 15y fatality rate 2%)
200

AR (OCP) <AR (HRT) 0


9

4
14


85
-1

-2

-2

-3

-3

-4

-4

-5

-5

-6

-6

-7

-7

-8
0-

15

20

25

30

35

40

45

50

55

60

65

70

75

80

Age group (yr)


CP1198340-1
RFC: How do you manage AC before,
Peri-Operative Management during, and after surgery?
1. She is two months from her DVT.
36 yo female with a DVT 2 months ago. No further AC is necessary.
She has developed a bowel obstruction 2. Begin prophylactic LMWH BID and
continue during after surgery re-
and is not able to take her oral AC establishing oral AC when able.
(coumadin) 3. Begin therapeutic LMWH qd, hold
the dose the evening prior to
Her INR is now non-therapeutic (1.5). surgery, and resume 2 hrs after
surgery is completed.
You are consulted by her surgeon for 4. Begin therapeutic LMWH BID, give 0% 0% 0% 0%
advice on how to manage her AC the daily dose the morning of the
day before surgery (24h prior) and
perioperatively.

..

t..
...

..
resuming 48-72 hrs after surgery is

ct

ti .
th

eu
y la

eu
on

ap
completed.

ph

ap
m

er
ro

er

th
wo

th

in
s t

in

in

eg
eg
ei

eg

B
B
S h

B
Association between residual anti-Xa level and
RFC: How do you manage AC before, interval from last dose of low-molecular-weight
during, and after surgery? heparin

1. She is incompletely treated and remains at risk for


recurrent DVT. Stopping AC is not a reasonable
option.
2. Prophylactic LMWH is insufficient in this setting.
3. Holding the dose of therapeutic LMWH the evening
prior to surgery and beginning 2h after increases the
risk of bleeding.
4. Present data indicates that giving the daily
threrapuetic dose 24h prior to surgery reduces risk of
plasma levels being in range that would predispose to O'Donnell, M. J. et. al. Ann Intern Med 2007;146:184-187
bleeding.

Recommendations Indwelling Catheter Related DVT


1. For patients receiving therapeutic
LMWH: give last dose 24 hours prior to 45 yo female with SII ER (-) Breast
the surgery or procedure. Cancer s/p Lx + LND with no personal
2. The last preoperative dose: the daily or family history of thrombosis has a
total dose. right sided central venous catheter in
which she received chemotherapy 1
3. For patients receiving UFH: Stop UFH 4
hours prior to the procedure.
year prior to the onset of DVT in her
right arm
4. Resume LMWH/UFH after surgery:
Minor surgery -24 h; Major- 48-72 h or
You are asked for advice on how to
LD LMWH/UFH when hemostasis manage this patient.
secure
Management of Catheter- Management of Catheter-related
related DVT DVT: Recommendations
1. Remove the catheter
immediately and follow 1. Do not remove the catheter if functional
expectantly. and there is a need.
2. Begin thrombolysis 2. Begin therapeutic LMWH, fondaparinux,
immediately.
or UFH with conversion to VKA for 3
3. Leave the catheter in
place and begin months.
prophylactic LMWH 0% 0% 0% 0%
3. Thrombolysis, thrombectomy,
for 1M. angioplasty, stenting or SVC filter are
4. Remove the catheter
...

..
.
l..

ra
m recommended only in select patients
..

p
i.

im

te
in
er

he
is
t

er
and begin therapeutic
he

ys

at
t
he
at

ec
bo
ec

at

th
m

ec
th

ro

LMWH and coumadin e


th

ov
th
e
ov

em
in

av
em

eg

R
Le

x 3M.
R

Previous HIT. Re-exposure to UFH.


Previous HIT episode. Re-exposure to
UFH. 600K

55 yo male had a CABG 6 months ago


complicated by HIT.
Pain in Right Leg
He is now admitted for a repeat CABG. 300K
He was exposed to UFH and began FVT.
prophylactic UFH 5000 q8h 48 hours after
surgery was completed
Inc. DDs, TT, and PTT
He received a total 6 U of PRCs and iv fluids
and was placed on a cardiac by-pass pump.
-D1 +D1 +D5 +D7

Previous HIT episode. Re-exposure to UFH. What is not likely to be the cause of the D1 platelet
decrease?
What is not likely to be the cause of the D1
1. Heparin activates platelets causing a mild decrease in
platelet decrease? the platelet count.
1. Non-immune 2. Hemodilution is common in patients newly hospitalized
heparin related or who have surgery.
effect 3. Cardiac by-pass pumps, hemofiltation, hemodialysis
2. Hemodilution activates platelets causing intravascular aggregates.

3. Platelet activation 4. HIT abs disappear within 30-100 days. The platelet count
dropped 50% of preop level 1 day after UFH exposure.
2nd to pump 0% 0% 0% 0% 0% There are other reasonable causes of the decrease in
4. HIT platelets.
IC
IT
n
t io

D
.
..

d.
r.

lu

2n
in

5. The elevated DDs are likely 2nd to the pump or tissue


i
od
r

n
pa

5. DIC
io
em
he

t
iva
H
e

insult. The prolonged TT and PTT are a residual UFH


un

ct
ta
m

le
im

te

effect.
on

la
P
N
Previous HIT. Re-exposure to UFH. Previous HIT episode. Re-exposure to
UFH. What about post op D7?
600K Fibrinogen normal
Inc. DDs, TT, and PTT 1. The platelet count is
unchanged from preop D1
and is of no consequence.
Pain in Right Leg 2. The decrease in the
300K platelet count from postop
FVT. D5 is due to the FVT.
3. The elevated DDs and
prolonged PTT indicate the 0% 0% 0% 0%
presence of DIC which
resulted in the FVT.

...

..
.
..

..

t ie
te
n.

pr
a
su

pa
d
pl

an

s
i

e
4. It is likely that this patient

nt

hi
t h

Ds

tt
ou

in

a
tc

th
ed
as
le
has HIT.

ly
t
e

re

va

ke
at

le
-D1 +D1 +D5 +D7

li
pl

de

is

he

he
he

It
T

T
T
Previous HIT episode. Re-exposure to Risk of HIT Categorized by Pt and Heparin
UFH. High Risk (>1%) postoperative patients receiving
What about post op D7? prophylactic/therapeutic UFH >4d

1. Although unchanged from D1, the D7 plt count Intermediate Risk (0.11%)
is 50% less than D5. The decrease in plts must 1. Med/Obst patients receiving prophylactic/therapeutic
be compared to the highest postop ct prior to UFH > 4d or after LMWH
drop.
2. Postoperative patients receiving LMWH > 4d or UFH
2. DVT rarely causes decreased plts and the flushes > 4d
changes are minimal
Low Risk (<0.1%)
3. DIC is characterized by low Fbn. DDs are non-
1. Med/Obst patients receiving LMWH > 4d or UFH flushes
specific
2. Any patient receiving UFH/LMWH < 4d
4. This patient likely has HIT(T).
Occurrence of symptomatic thrombosis after
Odds ratios for Risk of Thrombosis stopping heparin in patients confirmed to have
isolated HIT
14-year retrospective study
Prothrombin G20210A 2.0 Cumulative thrombotic event-rate (%)
Lupus Anticoagulant 5.4 100
90
Factor V Leiden 6.6 80 N = 62
Protein S Deficiency 10.9 70
60
52.8%
Dysfibrinogenemia 11.3 50
40
Protein C Deficiency 14.4 30
Antithrombin Deficiency 24.1 20
10
HIT 20-40 0
0 2 4 6 8 10 12 14 16 18 20 22 24 26 28 30
Days after isolated HIT recognized
Adapted from Warkentin TE, Kelton JG. Am J Med. 1996;101:502507.

57 yo female on the medicine


service receives UFH 5000U SC q8h.
On D5 she develops skin changes.
Her platelet count is normal.
Later, she develops necrosis at
injection sites
And Blue Toes
Dopplers of BLE are negative

HIT? HIT. Yes or No?

1. Yes
2. No
Erythematous plaques 1 Deep venous thrombosis 1

0% 0%
Skin necrosis 1 Venous gangrene 2
o
es

N
Y

1 Reproduced with permission Blackwell Publishing (Warkentin TE. Br J Haematol 1996


2 Warkentin TE et al. Ann Intern Med 1997
You suspect HIT and stop UFH. You begin
alternative AC immediately.After several days,
Patients developing the typical skin changes the results of the HIT panel which you had drawn
or evidence of thrombosis while receiving are returned as negative. Does this patient have
heparin should always be suspected to have HIT?
HIT and treated appropriately! 1. Yes
2. No

0% 0%

o
es

N
Y
REVIEW
It is likely that this patient has 1. Pulmonary emboli may come from both
HIT even if the platelet count proximal and distal DVT.
2. The incidence of DVT is dependent upon a
is not effected and the HIT number of factors including ethnicity,
panel is negative. heredity, and co-morbidities.
3. Venous thrombosis is a chronic dz
4. The duration of treatment of DVTs depends
upon the risk of recurrence and risk of
bleeding.

REVIEW REVIEW
4. Risk of recurrence and bleeding both 8. HIT may not be assoc with thrombocytopenia.
increase with age. 9. The probability of HIT can be estimated by the 4
5. Pre-operative AC should be adjusted to Ts and the risk by the type of patient and
prevent therapeutic blood levels at surgery. heparin.
6. Catheter related thrombosis should be 10. Thrombocytopenia and Thrombosis may occur
treated as a 1st time venous after heparin has been stopped.
thrombosis. 11. The presence of skin necrosis is evidence of
HIT regardless of the laboratory results.
7. HIT(T) may be early-, typical- or
delayed in onset.
Learning Objectives
Understand the clonal origin of hematologic malignancies
Understand the tools used to demonstrate clonality of these
Overview and Genetics of disorders
Comprehend the difference between lymphomas and
Hematologic Malignancies leukemias
Be familiar with some of the causes of hematologic diseases
Miroslav Djokic, MD Introduction to the WHO Classification
Division of Hematopathology
djokicm@upmc.edu
January 2009
keywords: clonality, phenotype, lymphoma, leukemia

Hematologic Malignancies Hematologic Malignancies


In general, hematologic malignancies are
Genetic Gene
caused by mutations, translocations or Cell Function abnormality Involved Disease
inversions of chromosomes containing genes
whose products regulate cell proliferation, Mantle cell
Cell cycle control t(11;14) PRAD1 lymphoma
differentiation or survival
Typically have a cytogenetic/molecular Follicular
Apoptosis t(14;18) BCL2 lymphoma
characteristic but often requires multiple
Acute
hits to produce full spectrum of disease promyelocytic
Differentiation t(15;17) RAR leukemia

Causes of Hematologic Malignancies Hematologic Malignancies


Chromosomal abnormalities and oncogenes
Clonal entire population of neoplastic cells
Inherited genetic factors
Down syndrome, Fanconi anemia, ataxia- arises from a single cell that has undergone
teleangiectasia (ATM)
genetic change
Viruses
EBV, HHV8 Difference is in clinical nature of disease
Environmental agents
Aggressive nature
H. pylori, gluten
Iatrogenic factors Infiltration into organs/tissues
therapy-related MDS/AML
Shortened life-span

1
Clonality Clonality
Genotype Numerical chromosomal changes
MDS monosomy 5, 7, 8
Karyotyping CLL trisomy 12
Fluorescence in-situ hybridization Reciprocal chromosomal translocations
Microarrays AML - t(8;21), t(15;17), inv(16)
ALL - t(12;21)
PCR CML - t(9;22)
Southern Blot NHL - t(11;14), t(8;14)
Deletions
Phenotype CEL del(4q12)
Flow Cytometry Point Mutations
Immunohistochemistry MPD - JAK2 V617F

Conventional Cytogenetic Analysis


Clonality (cont)
- Karyotyping
B cell - Ig heavy chain; light chain restriction; Resolution: 3-5 Mb
Ig gene rearrangement Requires fresh sample and dividing cells
T cell - TCR gene rearrangement Trained technologists
Plasma cell - secreted Ig or light chain only Labor intensive
(Bence-Jones protein) Not amenable to highly automated format

Fluorescence In Situ Hybridization


(FISH)
Resolution `100kb
Need to know which probe to use
Can be used on metaphase and
interphase cells
Archived fixed samples including paraffin
sections can be used
With completion of the human genome, all
t(15;17)(q22;q12-21) different probes available

t(15;17)(q22;q12-21)
Courtesy of Dr. U. Surti

2
Courtesy of Dr. U. Surti Courtesy of Dr. U. Surti

Microarrays MICROARRAY ANALYSIS


Array CGH will detect copy number
changes
Gene expression analysis can be
performed using RNA samples
DNA sequence changes in single
nucleotides (SNP)
Protein

Flow Cytometry
Immunophenotypic profile of neoplastic
cell population
Lineage assignment (myeloid, monocytic,
B-lymphoid, T-lymphoid)
Surface (or cytoplasmic) immunoglobulin
expression clonality
Follow-up of residual disease
Positive: CD19, CD34, CD10 (bright), TdT, CD22, CD45 (dim), HLA-DR, CD38,
CD13, CD33
Negative: CD20, surface Ig

3
Immunohistochemistry
Enables immunophenotypic assessment
of neoplastic cells within the context of
tissue architecture

CD138

lambda kappa

4
CD20 CD3

Lymphoma vs Leukemia
Lymphadenopathy or Bone marrow and
organ involvement peripheral blood
Always lymphoid: Can be lymphoid or
B-lineage myeloid
T/NK-lineage Acute or chronic
Hodgkin and non-
Hodgkin types

Many hematologic neoplasms can present both as


lymphoma and leukemia: CLL/SLL, Burkitt
leukemia/lymphoma, T-ALL/lymphoblastic lymphoma
Cyclin D1

Lymphoma Assembly of Immunoglobulin


Lymphomas are malignancies of lymphocytes
(B>>T)
Present with lymphadenopathy
Rarely high number of circulating malignant
cells found
Progressive infiltration of organs (liver, spleen;
occasionally others)
Range from indolent to highly aggressive

5
Modifications of Ig Lymphomas

Lymphomas Low Growth Fraction Lymphomas


Lymphomas are characterized by their rate of Genetic Abn Gene Fx Freq
proliferation FCL t(14;18) BCL2 apop 70-80%
A lymphoma with a low growth fraction MCL t(11;14) cyclinD1 prolif >90%
Follicular Cell Lymphoma CLL 13q14 del unk unk 50-60%
A lymphoma with a high growth fraction
Burkitts Lymphoma

High Growth Fraction


Lymphomas Burkitts Lymphoma

Genetic Abn Gene Fx Freq


DLBC t(3;N) BCL6 cell cycle 30-40%
Burkitts t(8;14) c-myc cell cycle 80%

6
Leukemia
Burkitts Lymphoma
Leukemia originates in marrow
Clinical manifestations range from pancytopenia (due to dense
marrow expansion of leukemia) to high WBC
Can see splenomegaly or adenopathy but is not dominant
feature in acute leukemias
Acute: represent arrested stages of differentiation
ALL
AML
Chronic: increased production and lack of apoptosis
CLL
CML

Acute Myelogenous Leukemia


Acute vs Chronic Leukemia
(AML)
Characterized by Characterized by
accumulation of accumulation of mature,
immature, differentiated cells
undifferentiated (granulocytes,
precursors (blasts) monocytes, lymphocytes)
Fulminant presentation Subclinical or incidental
with highly aggressive presentation with indolent
course course
Rapidly fatal if untreated, Survival in months to
but highly curable with years even without
chemoRx (pediatric ALL) therapy
May present with Usually present with high
peripheral cytopenias and white blood cell count
only rare blasts blood and splenomegaly
Cytogenetic Abnormalities in AML

Alteration in Transcription Factors- Acute Lymphoblastic Leukemia


AML (ALL)

7
Cytogenetic Abnormalities in
Plasma Cell Myeloma
ALL
Hematologic malignancy of terminally differentiated
B cells (plasma cells)
Function of plasma cells is secretion of Ig
Multiple genetic abnormalities have been reported
11q13 (cyclin D1)
6p21 (cyclin D3)
4p16 (fibroblast GF receptor-2)
16q23 (c-maf transcription factor)
20q11 (maf B - transcription factor)

Classification of Hematologic
Transformation
Malignancies
Change to different disease entity, usually Historically, classifications of hematologic
more aggressive disease malignancies were based on morphologic
Lineage specific and cytologic features
Examples: Not true biologic and clinical entities
CLL/SLL to large cell lymphoma (Richters) Lacked prognostic significance
MDS to AML Not uniformly accepted, leading to
PV to MF or AML confusion
CML: chronicacceleratedblast phase

WHO Classification Myelodysplastic Syndromes (MDS)


Emphasis on real Group of malignant hematopoietic stem cell
disease entities disorders characterized by:
Consensus approach pancytopenia
ineffective hematopoiesis
Stratifies neoplasms dysplastic bone marrow changes
according to lineage: significant risk of progression to acute myeloid
myeloid, lymphoid, leukemia
histiocytic/dendritic and IPSS (International Prognostic Scoring System):
mast cell* Estimates prognosis in MDS
Cell of origin concept Includes variables such as number of cytopenias,
percentage of marrow blasts, and cytogenetic
Will be continuously abnormalities
updated Patients divided into four subgroups according to risk;
low, intermediate-1 and -2, and high

8
Myelodysplastic Syndromes Myeloproliferative Neoplasms
Refractory anemia (cytopenia) Group of disorders characterized by
Refractory anemia with ringed sideroblasts unregulated, sustained, excess production
of one or more mature peripheral blood
Refractory cytopenia with multilineage elements with intact maturation and
dysplasia minimal dyspoiesis
Refractory anemia with excess blasts Genetic abnormalities:
Myelodysplastic syndrome with isolated BCR-ABL (CML)
del(5q) JAK2 V617F mutation
Cryptic del(4q12) - FIP1L1-PDGFRA (CEL)
Myelodysplastic syndrome, unclassifiable

Myeloproliferative Neoplasms
Chronic myelogenous leukemia, BCR-ABL1+
Chronic neutrophilic leukemia
Polycythemia vera
Primary myelofibrosis
Essential thrombocythemia
Chronic eosinophilic leukemia
Myeloid neoplasms associated with PDGFRA,
PDGFRB, and FGFR1 rearrangements
Mast cell disease
From Levine RL, Nat Rev Cancer. 2007 Sep;7(9):673-83
Myeloproliferative disease, unclassified

Myelodysplastic/Myeloproliferative
Lymphoid Neoplasms
Neoplasms
Group of neoplasms that exhibit features Three major categories of lymphoid neoplasms:
intermediate between myelodysplastic and B-cell neoplasms
myeloproliferative disorders T-cell/Natural killer (NK)-cell neoplasms
Hodgkin lymphoma (disease)
Chronic myelomonocytic leukemia
WHO Classification:
Atypical chronic myeloid leukemia
B-cell
Juvenile myelomonocytic leukemia Precursor B-lymphoblastic leukemia/lymphoma
Myelodysplastic/myeloproliferative neoplasm, Mature B-cell neoplasms
unclassifiable T-cell
Precursor T-lymphoblastic leukemia/lymphoma
Mature T/NK neoplasms

9
WHO Classification of Mature B-
Lymphoid Neoplasms
cell Neoplasms
Predominantly disseminated lymphoma/leukemia
The mature B-cell and T/NK-cell Chronic lymphocytic leukemia
Lymphocytic lymphoma/Waldenstroms macroglobulinemia
neoplasms are listed according to their Hairy cell leukemia
Splenic marginal zone lymphoma
clinical presentation: Plasma cell myeloma
Primary extranodal lymphomas
Predominantly disseminated/leukemic types Extranodal marginal zone B-cell lymphoma (MALT)
Predominantly nodal lymphomas
Predominantly nodal lymphomas Follicular lymphoma
Mantle cell lymphoma
Primary extranodal lymphomas Nodal marginal zone lymphoma

Diffuse large B-cell lymphoma


Primary mediastinal large B-cell lymphoma
Primary effusion lymphoma
Burkitt lymphoma

Lymphomatoid granulomatosis

WHO Classification of Mature T- WHO Classification of Hodgkin


cell and NK-cell Neoplasms Lymphoma
Leukemic disseminated


T-cell prolymphocytic leukemia
T-cell large granular lymphocytic leukemia
Classical Hodgkin Lymphoma (CHL)


Aggressive NK-cell leukemia
Adult T-cell leukemia/lymphoma
Nodular sclerosis CHL
Cutaneous
Mycosis fungoides Mixed cellularity CHL
Sezary syndrome


Primary cutaneous anaplastic large cell lymphoma
Lymphomatoid papulosis
Lymphocyte-rich CHL
Other extranodal
Extranodal NK/T cell lymphoma, nasal type
Lymphocyte-depleted CHL
Enteropathy-type T-cell lymphoma


Hepatosplenic T-cell lymphoma
Subcutaneous panniculitis-like T-cell lymphoma
Nodular lymphocyte predominant Hodgkin
Nodal
Angioimmunoblastic T-cell lymphoma
lymphoma
Peripheral T-cell lymphoma, unspecified
Anaplastic large cell lymphoma
Uncertain lineage and stage of differentiation
Blastic NK-cell lymphoma (? dendritic cell origin)

CML as a Paradigm Chronic Myeloid Leukemia (CML)


CML
Proliferative disorder of hematopoietic stem cells
Well-characterized clinical course
t(9;22) involves transfer Philadelphia (Ph) chromosome
of ABL1 oncogene to
Unique chromosomal abnormality
BCR on 22q resulting
in abnormal tyrosine Bcr-Abl1 tyrosine kinase
kinase activity
A single molecular abnormality that causes
transformation of a hematopoietic progenitor into a
malignant clone

10
Clinical Course: Phases of CML

Advanced phases
Chronic phase
Accelerated
phase Blast crisis

Median 56 Median duration Median survival


years 69 months 36 months
stabilization

Courtesy of Dr. U. Surti

t(9;22)(q34;q11.2) BCR/ABL interphase /metaphase FISH


Abnormal --
fusion
signals
BCR
BCR /ABL

no
ABL
signal
der (22) 22
9 der (9) Normal
cell

q11.2
q34 22
der (22) 22 der (22)
9 der (9)
9

From: Am Soc Clin Path


Courtesy of Dr. U. Surti

11
Gleevec (imatinib mesylate, formerly STI571) is the first
tyrosine kinase inhibitor that is commercially available for
clinical use. Gleevec selectively blocks cellular proliferation and
induces apoptosis in Philadelphia chromosome-positive (Ph+)
cells harboring the Bcr-Abl tyrosine kinase, the causative
abnormality of chronic myeloid leukemia (CML).

Gleevec (STI-571) blocks the activity of Bcr-Abl protein and halts


chronic myeloid leukemia

Points to Remember
Hematologic malignancies are caused by genetic
abnormalities that affect cell proliferation,
differentiation or survival
Genetic abnormalities are markers of the malignant
clone
WHO Classification stratifies malignancies
according to lineage and stage of differentiation
with emphasis on biological and clinical entities
Fusion proteins or dysregulated intermediaries of
proteins involved in proliferation, differentiation or
survival are targets for new therapies

12
Learning objectives
Pathologic evaluation of
Contrast normal and reactive morphology of
hematologic malignancies: leukocytes in peripheral blood to findings seen in
Evaluation of peripheral acute leukemia and chronic lymphoid leukemia.
Understand the indications for and standard
blood and bone marrow components of a bone marrow evaluation.
Contrast normal morphology of bone marrow to
selected pathology.
Sara A. Monaghan, MD Begin to appreciate the ancillary studies commonly
Division of Hematopathology used for evaluation of hematologic malignancy.
Department of Pathology
University of Pittsburgh

Indications for physician/pathologist to Peripheral blood film


review peripheral blood film Morphologic features to evaluate

Red cells
Anemia especially if hemolytic

White cells
Other unexplained cytopenias

Platelets
Unexplained leukocytosis and/or
lymphocytosis

a. Segmented neutrophil c. Eosinophil a. Basophil c. Lymphocyte


Identify the normal
b. Neutrophilic band d. Monocyte b. Lymphocyte, large granular d. Monocyte
leukocytes:

1
Normal
Acute leukemia: peripheral lymphocytes
Myeloblasts Lymphoblasts

blood film findings


Bicytopenia or pancytopenia.
Anemia
Absolute neutropenia
Thrombocytopenia
Abnormal cells in circulation
Blasts
Abnormal (dysplastic) neutrophil morphology may be seen with
acute myeloid leukemias and the related myelodysplastic
syndromes
May be associated with leukoerythroblastosis

Chronic lymphoid leukemias and


Leukoerythroblastosis lymphoma: peripheral blood film
Immature findings
granulocytes
(e.g. myelocyte)
Teardrop cells Cytopenias are much less common than with
acute leukemias.

Total white cell count may be high due to absolute


lymphocytosis.

Abnormal lymphoid cells may be seen

May be associated with leukoerythroblastosis.


Nucleated red
blood cells

Common myeloid progenitor Common lymphoid progenitor

Normal Reactive Neoplastic


lymphocytes lymphocytes lymphoid cells
Myeloblast
Pronormoblast

Promyelocyte Basophilic normoblast

Stages that can be


morphologically
recognized in Polychromatophilic
Myelocyte normoblast
marrow

Metamyelocyte Orthochromatic
normoblast

Megakaryocyte
N. Band Polychromatic
erythrocyte

Basophil Eosinophil Neutrophil Monocyte Platelets Erythrocyte Lymphocyte

2
Bone Marrow Evaluation: Components of a bone marrow
Indications evaluation
Peripheral blood film examination

Unexplained cytopenias - especially bicytopenia or


Bone marrow aspirate smears
pancytopenia Bone marrow trephine biopsy
Circulating blasts or abnormal cells Ancillary studies
Suspicion for neoplastic hematolymphoid proliferations Flow cytometry
Leukocytosis
Cytogenetic studies
Lymphadenopathy
Splenomegaly
+/- Molecular studies
Evaluation of iron stores and metabolism

See one, do one, teach one?

Good planning is essential.

The person performing the procedure Aspirate smears Core biopsy


needs to focus on the patient and
obtaining good samples.

An assistant can prepare smears and


handle the biopsy.

200 X 1000 X
Bone marrow
aspirate smears

Particle crush technique

Cytoplasmic and nuclear


details can be well assessed

Manual 500 cell differential


count

1000 X

3
1000 X 1000 X

Pro
Myelo

Meta

Seg

Band

1000 X

Selected pathology for comparison to normal

Normal Megaloblastic anemia

AML Chronic lymphocytic leukemia

Auer rod

40 X 500 X
Bone marrow
biopsy

Optimal biopsy:
at least 0.5 cm
of medullary cavity

Formalin or other fixative

Paraffin embedded sections

Assess cellularity and


architecture

500 X

4
Selected pathology for comparison to normal
Bone marrow evaluation:
Ancillary studies

Immunophenotyping by flow cytometry


Normal Acute leukemia
Cytogenetic analyses

Molecular studies

Aplastic anemia non-Hodgkin lymphoma

Immunophenotyping of
hematolymphoid malignancies: Immunophenotyping by flow cytometry
flow cytometry
Individual cells in fluid suspension pass thru a laser
beam.
Mature B-cell neoplasms
Collection of 5-8 properties of light emission for each
individual cell.
Acute leukemias Forward and side light scatter properties (inherent to various
cell types).
Fluorescence intensity of, typically, 3-6 fluorochrome-labeled
Mature T-cell neoplasms and other antibodies directed to cellular antigens.
disorders
Data displayed in 2-D dot plots

Example: A 4-color antibody combination CD5-


CD5-APC labels the T-
T-cells

to analyze lymphocytes in an aliquot of a CD


5

bone marrow sample CD5

CD5
CD

CD5
lambda
5

kappa

FITC PE CD19-
CD19-PerCP labels the B-
B-cells
CD19

kappa lambda
CD19

kappa CD19

lambda CD19
CD19
CD5
kappa

lambda

PerCP APC

CD19 CD5
Kappa-
Kappa-FITC labels some of the B-
B-cells Lambda-
Lambda-FITC labels some of the B-
B-cells

5
T-cells are CD5(+), CD19(-
CD19(-)
B-cells are
CD5(-
CD5(-), CD19(+)

Blood sample with mix of


T and B-
B-cells; B-
B-cells are
polytypic
B-cells with a mix of kappa
and lambda expression

CD19(+) B-
B-cells with
abnormal expression All B-
B-cells
of CD5 express only
one type of
light chain.

Reference: Preffer FI in Colvin RB, et al, eds. Diagnostic Immunopathology, 2nd ed.
Raven Press: New York. p. 726. Blood sample with mature B-
B-cell neoplasm

Flow cytometry: Mature B-cell


neoplasms Flow cytometry: Acute leukemia
Restricted (monotypic) immunoglobulin light chain
expression is strong indirect evidence of clonality for Lineage assignment
mature B-cells Myeloid blasts
Normal kappa to Lambda ratio = 0.6-2.5:1 B-lymphoblasts
T-lymphoblasts
Detection of a specific phenotype may be very helpful for
more precise classification of some B-cell neoplasms.
Documentation of immunophenotype for
For example, abnormal expression of the T-cell marker, CD5,
on a mature B-cell neoplasm raises the possibility of two main subsequent detection of a low level of
subtypes of B-lymphoid neoplasms: chronic lymphocytic residual or recurrent disease
leukemia or mantle cell lymphoma.

Cytogenetic analysis:
B-lymphoblastic leukemia
Cancer cytogenetics
CD10 = Common acute lymphoblastic leukemia antigen
(CALLA); common on B-
B-ALL, but not restricted to this. Looking for acquired abnormalities of
CD34 = On hematopoietic stem cells,
progenitors and some blasts
chromosomes in neoplastic cells.
In contrast to constitutional abnormalities.
Abnormalities may supply the following:
Precise diagnosis t(15;17); PML/RAR Acute promyelocytic leukemia
Prognostic information Trisomy 12 aggressive course in CLL
Evidence of disease progression New acquision of i(17q) in CML
CD13 = pan-
pan-myeloid marker; Prediction of response to directed therapies
CD19 = pan B-
B-cell marker it should not be found on t(15;17); PML/RAR enhanced response when treated with
non-
non-neoplastic B-
B-cells all-trans-retinoic acid (ATRA), a ligand for RAR
CD22 = pan B-
B-cell marker

6
Classic cytogenetics Acute promyelocytic leukemia
(Conventional karyotyping)

Evaluation of stained metaphase


chromosomes.
Fresh tissue for culture and dividing cells
required for analysis.
An abnormality is considered clonal when it is
seen in at least two metaphase spreads.

Image provided by Malini Sathanoori, Pittsburgh Cytogenetics Laboratory

Schematic representation of t(15;17)(q22;q21)

p11.2
Classic cytogenetics
p11.1
q11.1 p13
q11.2
p12
Numerical chromosomal abnormalities
q12
q13 p11.2
q14 p11.1

q15 q11.1
Reciprocal
and translocations are readily detected.
q11.2
q21 q12 exchange Other genetic defects play a role in
q22 q22
q21
q21 of segments
malignant transformation, but may not be
q23
q22
q23
detected:
q24
q24 Point mutations
q25
q25
Gene deletions Other cytogenetic studies
q26
(e.g. FISH) or molecular studies
Normal
Gene amplifications may be helpful
#17 with Der #15
Normal #15 with
#17 a break
Der Viral infections
#15 #17
a break at q21
at q22
t(15;17)(q22;q21)
Courtesy of Malini Santhanoori, Pittsburgh Cytogetics Laboratory

Key teaching points


Chronic lymphoid leukemias and acute leukemias are hematologic
malignancies that characteristically involve blood and bone marrow.
Chronic lymphoid leukemias present with an absolute lymphocytosis.
Acute leukemias present with cytopenias and, sometimes, circulating blasts.

Leukoerythroblastosis is an abnormal peripheral blood film finding that


indicates bone marrow infiltration and replacement.

Bone marrow evaluation includes review of a peripheral blood film,


aspirate smears and core biopsy.
biopsy

Proper procurement and handling of bone marrow samples are essential


for accurate interpretation.

Hematologic malignancies are classified based on morphology,


immunophenotype, genetic and molecular abnormalities.
abnormalities

7
Lecture: Pathologic Evaluation of Hematologic Malignancies Evaluation of
Peripheral Blood and Bone Marrow.

Date: Friday, January 8, 2009

Time: 9:00 10:00

Lecturer: Sara A. Monaghan, MD


monaghansa@upmc.edu
412-647-0382

Learning Objectives:

1. Understand the indications for a bone marrow evaluation.


2. Be aware of the standard components of a bone marrow evaluation.
3. Recognize the peripheral blood film findings associated with hematologic
malignancies, focusing on acute leukemias and chronic lymphoid leukemias.
4. Become familiar with the normal morphology of bone marrow as compared to
that of abnormal examples.
5. Begin to appreciate the use of ancillary studies commonly used for evaluation of
hematologic malignancy.
Indications for a bone marrow evaluation.
The primary purpose of a bone marrow examination is to diagnose, confirm, or stage a
hematologic disease. Hematologic disease involving the bone marrow may be neoplastic,
such as acute leukemias or chronic lymphoid leukemias, or non-nonneoplastic (e.g.
aplastic anemia). The major reasons to suspect a hematologic disease are unexplained
cytopenias (especially bicytopenia or pancytopenia), circulating blasts, persistent elevated
counts (leukocytosis, erythrocytosis, thrombocytosis), lymphadenopathy, and/or
splenomegaly. Uncommonly, a bone marrow evaluation may be needed to evaluate for
iron stores. A bone marrow evaluation may also rarely play a role in the detection of
some lysosomal storage diseases and in the evaluation of fever of unknown origin, but
these two types of disorders will not be discussed further here.

Unexplained cytopenias, especially affecting two or three cell counts (red cell
count/hemoglobin, absolute neutrophil count, or platelet count), raise clinical concern for
an acute leukemia, which can be either acute myeloid leukemia or acute lymphoblastic
leukemia. Pancytopenia refers to low counts among red cells, neutrophils and platelets,
while bicytopenia means that two of these counts are low. Possible non-neoplastic
causes to consider for bicytopenia or pancytopenia include nutritional deficiencies (B12,
folate, iron), toxic exposures (e.g. arsenic), affect of medications, infections (especially
viral), immune-mediated causes and non-neoplastic bone marrow failure (e.g. aplastic
anemia). Circulating blasts are also very worrisome for an acute leukemia. Detection of
circulating blasts in the blood is exceedingly rare under normal or reactive conditions.
Two instances in which occasional circulating blasts may be seen include stress reactions
in neonates and in patients treated recently with filgrastim [granulocyte colony-
stimulating factor (G-CSF)], which is used to treat neutropenia or to stimulate circulation
of hematopoietic stem cells for use in transplantation. Myelodysplastic syndromes,
which are clonal myeloid disorders related to acute myeloid leukemias, may also present
with cytopenias and/or circulating blasts. This fairly complex group of myeloid
neoplasms will not be discussed further here, but Dr. Boyiadzis will discuss
myelodysplastic syndromes during his lecture on Tuesday, January 13th. The present
lecture will illustrate the blood and bone marrow findings seen in acute leukemias.
Hematopathology Workshop II on Wednesday, Jan. 14th, will further reinforce the
understanding of how acute leukemias are diagnosed and classified by a pathologist.

Chronic lymphoid leukemias, including the prototypical chronic lymphocytic leukemia,


typically present with an absolute lymphocytosis (> 4x109/L). Cytopenias are less
common. Other causes for lymphocytosis to consider include infections (e.g. infectious
mononucleosis) and stress reactions, but these are usually transient. Some clarification of
terminology is warranted at this point. Chronic lymphoid leukemias is general
terminology that refers to neoplasms that result in an accumulation of mature lymphoid
cells that have a predilection to accumulate in the peripheral blood and bone marrow,
which is why a bone marrow evaluation is very valuable in diagnosing chronic lymphoid
leukemias. They are a subset of mature lymphoid neoplasms. Mature means that that
the lymphoid cells have differentiated beyond the blast stage, which contrasts with the
lymphoblasts that accumulate in acute lymphoblastic leukemia. Examples of blood and
bone marrow involvement by chronic lymphoid leukemias will be contrasted with acute
leukemia as well as with reactive lymphocytosis in this lecture. Neoplasms referred to as
lymphomas also include many mature lymphoid neoplasms (i.e. follicular lymphoma,
Burkitt lymphoma, etc). However, the lymphoid cells usually accumulate in solid tissue
organs (e.g. lymph nodes) in the disorders historically called lymphomas. Lymphomas
are therefore suspected when patients have lymphadenopathy, splenomegaly, a
mediastinal mass or other evidence of tissue infiltration. A bone marrow evaluation is
not usually an initial diagnostic procedure when lymphoma is suspected. The diagnosis
and classification of lymphomas will therefore be emphasized at other times during this
course. Yet lymphomas not uncommonly disseminate to other sites, especially bone
marrow, and this has prognostic significance. Along with imaging studies, bone marrow
evaluation has therefore been a standard component for clinical staging of lymphoma.

Elevated counts other than an absolute lymphocytosis (e.g. absolute neutrophilia,


monocytosis, eosinophilia, basophilia, erythrocytosis, and thrombocytosis) may raise
concern for a myeloid malignancy, especially a chronic myeloproliferative disorders (e.g.
chronic myeloid leukemia, polycythemia vera, essential thrombocythemia). There are
many non-neoplastic reasons also to consider for such elevated counts. If clinical
suspicion persists for a chronic myeloproliferative disorder, a bone marrow evaluation is
frequently indicated because most of the chronic myeloproliferative disorders are
associated with characteristic bone marrow findings that aid in diagnosis and
classification. However, details about the various chronic myeloproliferative disorders
and the differential diagnoses that should be considered are beyond the scope of the
present lecture. Please refer to the lectures by Drs. Tarhini (Chronic Leukemias) and
Boyiadzis (Myeloproliferative disorders and myelodysplastic syndromes) on Monday and
Tuesday, Jan 12th-13th.

Iron deficiency anemia can usually be documented based on biochemical tests, including
serum ferritin, total iron binding capacity, and transferrin saturation. Occasionally
distinguishing iron deficiency anemia from anemia of chronic disease can be difficult,
especially when the patient has an inflammatory disorder, malignancy, or acute liver
injury that might result in normal range ferritin levels despite iron depletion. A bone
marrow evaluation with a Prussian blue stain performed on the aspirate smears, which is
considered the gold standard for assessment of iron status, may then be indicated to
evaluate for iron deficiency.

Components of a bone marrow evaluation.


Morphologic review of a peripheral blood film is a standard component of a bone marrow
evaluation because cytomorphologic features of red cells, leukocytes and platelets often
provide very helpful clues for establishing either a diagnosis of a hematologic
malignancy or an alternate diagnosis. Bone marrow sampling usually includes both a
bone marrow aspirate specimen and biopsy specimen. These preparations give you
somewhat different information and, occasionally, one type of preparation is limited or
simply non-diagnostic, but the other is diagnostic. In adults, the hematopoietic bone
marrow is largely restricted to axial bones. The vast majority of bone marrow
evaluations are sampled from the posterior superior iliac crest. Some alternate sites may
be sampled. A sternal aspiration is occasionally performed, but a biopsy is not performed
at this site due to the risk of sternal perforation. In very young infants, the iliac bone is
not completely ossified; aspiration of the tibia may be performed instead. Good planning
is essential in obtaining optimal bone marrow samples. Suboptimal specimens contribute
to errors in interpretation. It is best to have an assistant who can do a good job in
preparing the air dried aspirate smears and who can handle the biopsy while the person
performing the procedure focuses on the patient and obtaining good samples.

A. Peripheral blood film.


It is helpful to realize where to focus your examination on a peripheral blood film.
The wedge technique is a common method for preparing a good quality peripheral
blood film although manual preparation is being supplanted by automated
preparations. Either method results in a preparation that is thick at one end and thin
at the opposite end. The area of optimal thickness should be about 2 cm long and this
is the main area that should be used for the morphologic evaluation.

Red cells, leukocytes and platelets are assessed whenever a peripheral blood film is
reviewed. Red cell morphology can be especially helpful in pointing toward a
diagnosis of one of the hemolytic anemias, but this has been discussed in other
lectures. Normal platelet development and function has been covered by Dr. Kiss
during lecture. Abnormalities in platelet morphology are uncommon and when
present are often subtle in most myeloid neoplasms and will not be covered. The
morphology of peripheral blood leukocytes (normal and some examples of reactive
changes) will be reviewed during lecture, as a basis for comparison with the findings
seen in acute leukemia and chronic lymphoid leukemia. Some time will be spent
during lecture illustrating the appearance of circulating blasts and the abnormal (i.e.
neoplastic) lymphoid cells associated with chronic lymphoid leukemias because these
are two specific peripheral blood findings that physicians should realize point toward
a diagnosis of a hematologic malignancy.

The leukocytes normally seen in the peripheral blood film include segmented
neutrophils, band stage neutrophils, lymphocytes, monocytes, eosinophils and
basophils (please also review Richard Steinmans lecture, Hematopoiesis
Overview). Segmented neutrophils are the most mature form in the neutrophil
series. The nucleus is segmented or lobated (2-5 lobes normally) and the segments
are connected by thin filaments of chromatin. The cytoplasm is pale with lilac
(specific) granules. The neutrophil band stage normally comprises about 5-10% of
leukocytes in blood. The nucleus is indented to more than half the distance from
nuclear margin, but does not constrict to the point of a filament. Bands are increased
with stress and infection. Normal lymphocytes are somewhat heterogeneous, but
many are small cells with round to oval nuclei that may be slightly indented or
notched. The chromatin is dense, coarse and clumped and the cytoplasm is often
scant. The large granular lymphocyte (LGL) is also a normal lymphocyte, but it
has more abundant cytoplasm and thus it is somewhat larger. LGLs often display a
few, coarse, azurophilic granules. LGLs are NK cells or CD8(+) suppressor/cytotoxic
T lymphocytes. Monocytes are large cells and usually display indented or folded
nuclei. Their chromatin is slightly less dense than that of a neutrophil. The monocyte
cytoplasm is gray to gray-blue and may contain fine, pink granules. Some monocytes
may show cytoplasmic vacuoles. The eosinophil cytoplasm displays coarse, orange-
red, refractile granules of uniform size. Most eosinophils have nuclei with two lobes,
but some will have 3. The basophil displays few to a moderate number of coarse,
dense blue-black granules of different sizes that may obscure the nucleus.

Circulating blasts likely herald acute leukemia unless the patient has received G-CSF
therapy or is a neonate. Occasional circulating blasts may also be seen with
myelodysplastic syndromes or chronic myeloproliferative disorders. Features of
blasts include scant to moderate cytoplasm, round or oval nuclei, fine chromatin and
one or more nucleoli. Morphology is not very reliable for determining the lineage of
the blasts (i.e. lymphoid or myeloid). However, if the blasts display Auer rods, then
the blasts are neoplastic myeloblasts and the diagnosis is likely acute myeloid
leukemia. Auer rods are needle-like cytoplasmic inclusions that represent an
abnormal aggregation of primary granules. However, only about 1/3rd of acute
myeloid leukemias will be associated with Auer rods. Compared to lymphocytes,
blasts are somewhat larger, display more scant cytoplasm, finer chromatin and
nucleoli.

With chronic lymphoid leukemias, abnormal lymphoid cells are expected in the
peripheral blood. Distinguishing neoplastic lymphoid cells from normal or reactive
lymphocytes can be difficult if the neoplastic lymphoid cells comprise only a
minority of the lymphoid cells. Normal lymphocytes are generally small and mature-
appearing with round nuclei, condensed chromatin, and inconspicuous nuclei.
Reactive lymphocytes invariably include a heterogenous mixture of lymphocytes with
a wide range of morphology that belongs to a spectrum of non-neoplastic
appearances. In general, neoplastic lymphoid cells tend to more monotonous
compared to normal and reactive lymphocytes. Mature neoplastic lymphoid cells
may range from predominantly small and mature-appearing (e.g. chronic lymphocytic
leukemia) to more overtly abnormal with unusual nuclear indentation, nuclear
folding, nuclear hyperchromasia, or prominent nucleoli (e.g. adult T-cell
leukemia/lymphoma). Examples of reactive and mature neoplastic lymphoid cells
will be compared to normal morphology during the lecture.

Leukoerythroblastosis is another abnormal peripheral blood film finding that may


indicate a hematologic malignancy, but it is not specific because it can be seen in
association with any reason for bone marrow infiltration and replacement. Thus, it
can be seen with acute leukemias, chronic leukemias, metastatic carcinoma, bone
marrow fibrosis, bone marrow necrosis, sarcoidosis, storage diseases, and
osteopetrosis. The combination of nucleated red blood cells and immature
granulocytes (promyelocytes, myelocytes, and metamyelocytes) constitute a
leukoerythroblastic picture. Teardrop-shaped red blood cells are also common.
Factors contributing to leukoerythroblastosis likely include disruption of the normal
bone marrow structure, cytokines and, possibly, extramedullary hematopoiesis.
Although leukoerythroblastosis is worrisome for bone marrow infiltration, profound
blood loss or hemolysis may also occasionally result in a leukoerythroblastic picture.
B. Bone marrow aspirate smears.
At the bedside during the procedure, the aspirate smears are prepared somewhat
similar to making a peripheral blood film. Fresh marrow particles are spread onto a
glass slide between two slides and air dried. Marrow particles may include portions
of blood vessels, stroma and entrapped hematopoietic cells. Similar to the peripheral
blood film, the aspirate smears are stained with Wright-Giemsa. This type of
preparation permits evaluation of fine cytologic detail of individual cells. However,
the bone marrow architecture is not well appreciated on this type of preparation.
Focal abnormalities will often not be detected on the aspirate smears, such as bone
marrow involvement by lymphoma.

The diagram below (fig. 1) best illustrates stages of differentiation of neutrophils and
erythroid precursors that are morphologically recognizable in the bone marrow
aspirate smears. Stages between stem cells and myeloblast or pronormoblast cannot
be morphologically distinguished and are not illustrated in the diagram. The
lymphoid stages of differentiation are not well addressed by this diagram. Students
may also refer to the lecture that Richard Steinman gave to them on hematopoiesis or
Fig. 13-1, Kumar: Robbins and Cotran: Pathologic Basis of Disease, 7th ed.

Figure 1
The aspirate smear is commonly examined under oil immersion using the 50X or
100X objective. The students are not expected to be able distinguish the precursor
cell types for their examination. However, they should be aware that for bone
marrow evaluation, a differential count is manually performed on an aspirate smear
by counting and categorizing hematopoietic cells, including granulocytic precursors
and erythroid precursors. During the lecture, an overview will be given of the normal
hematopoietic cells seen in the bone marrow. For reinforcement, the normal
components will be compared to some abnormal examples: megaloblastic erythroid
and granulocytic precursors (from a case with megaloblastic anemia), acute myeloid
leukemia, and chronic lymphocytic leukemia (fig. 2).

Megakaryocyte: The largest hematopoietic cell is the megakaryocyte. It has the


unique property of endomitosis: ongoing DNA synthesis without mitoses, resulting
in nuclear lobulation with increasing DNA content. The nuclei do not separate but
progressively fold and lobulate while remaining connected. These cells reside
adjacent to bone marrow sinuses and shed platelets into the blood.

Blasts: Blasts comprise less than 2% of the cellularity of the normal bone marrow.
Features of blasts include scant to moderate cytoplasm, round or oval nuclei, fine
chromatin and one or more nucleoli. Morphology is not very reliable in determining
into which lineage the blast will differentiate (i.e. lymphoid, erythroid, or
granulocytic). One exception to this latter comment occurs exclusively with acute
myeloid leukemia because if Auer rods are present, then the blast is neoplastic and it
is myeloid rather than lymphoid. However, only about 1/3rd of acute myeloid
leukemias will be associated with Auer rods. Auer rods are needle-like cytoplasmic
inclusions that represent an abnormal aggregation of primary granules.

Maturing granulocytes: The stages of neutrophilic granulocyte maturation beyond the


blast stage include promyelocyte, myelocyte, metamyelocyte, band and segmented
neutrophil. The promyelocyte is distinct because it is the largest and contains primary
granules, which are coarse and azurophilic; myeloperoxidase is a major enzyme
within the primary granules. As the neutrophilic granulocytes mature, they begin to
accumulate secondary granules. Secondary granules contain lactoferrin, vitamin B12
binding protein and lysozyme. There is progressive indentation of the nucleus and
ultimately segmentation. Eosinophils and basophils also progress through similar
stages of maturation, but are seen in very low numbers on the aspirate smears.
Monocytes arise from the same progenitor cells as neutrophils. As monocytes mature
they undergo acquisition of cytoplasmic granules and nuclear folding, but distinct
morphologic stages are difficult to identify on the aspirate smears.

Erythroid maturation: The earliest recognizable erythroid precursor is the


pronormoblast. During maturation, there is a reduction in cell size. Erythroid
precursors have very round nuclei. The chromatin becomes block-like or clumpy
as it condenses. The cytoplasm goes from basophilic to pink as cytoplasmic RNA
decreases and the cell synthesizes more hemoglobin.
Assessment for iron status: An iron stain (Prussian blue stain) performed on a well
prepared aspirate smear is typically very accurate for the assessment of iron stores.
Iron stores will be detected as diffuse or particulate iron that is colored blue to blue-
green.

Figure 2
A B

C D

Selected bone marrow pathology for comparison to normal (aspirate smears, 1000X).
(A) Normal maturing granulocytes and a few erythroid precursors. (B)
Megaloblastic anemia. Note the enlarged maturing erythroid and granulocytic
precursor cells with asynchronous maturation (i.e. nuclear development is lagging
behind cytoplasmic maturation). The nuclei have abnormally fine chromatin. (C) In
acute myeloid leukemia, the blasts are often medium to large and monotonous. The
acute leukemia cells also display immature-appearing nuclei with somewhat fine
chromatin and nucleoli. In this example, the myeloid lineage of the blasts is obvious
because there is an Auer rod (arrow). (D) In chronic lymphocytic leukemia, the
lymphoid cells resemble normal mature lymphocytes, but they are too numerous and
too monotonous.

A. Bone marrow biopsy.


An ideal biopsy is at least one centimeter in length. The specimen is fixed in
formalin or another preferred fixative. Then the specimen is processed and embedded
in paraffin. As for all histologic sections, very thin sections are cut (about 4 um),
placed on glass slides and then stained. Hematoxylin eosin stain is the standard stain
used for most histology.

The core biopsy is the best sample type for assessment of the bone marrow cellularity
and distribution of cells. The cellularity is determined as a cell to fat ratio. A rule of
thumb for what may be considered normocellular is 100 minus the patients age.
It is somewhat more difficult to identify individual cell types on the biopsy when
compared to the aspirate smear preparations. However as the granulocytes mature,
they do show evidence of granular cytoplasm and nuclear segmentation. The
erythroid precursors are recognized as cells with the very round, dense nuclei.
Megakaryocytes are easily appreciated on the biopsy and their numbers are best
assessed on the biopsy because the bone marrow architecture is well represented. In
contrast to the aspirate smears, the biopsy is ideal for identifying involvement by
focal processes such as carcinoma, lymphomas and granulomas. Selected bone
marrow pathology for comparison to normal is illustrated (figure 3).

Figure 3

Selected bone marrow pathology for comparison to normal (biopsy, 500X). (A)
Normal bone marrow histology displaying trilineage hematopoiesis. (B) Acute
leukemia. Note the large, monotonous blasts with fine chromatin in an acute
leukemia. Leukemias are not usually focal in the bone marrow, but are uniformly
interspersed throughout the interstitium or entirely replace the marrow. (C) Aplastic
Anemia. The cellularity is profoundly reduced, but here we see a few regenerating or
residual erythroid precursors. (D) Non-Hodgkin lymphoma. Involvement of the bone
marrow by lymphoma is often focal, which contrasts to acute leukemias that are more
evenly distributed throughout the marrow interstitium. Some non-Hodgkin
lymphomas (e.g. follicular lymphoma) have an abnormal predilection to infiltrate
around the bony trabeculae. Such a paratrabecular lymphoid aggregate is an
architectural feature that is essentially pathognomonic for lymphoma involvement
because benign lymphoid aggregates in the bone marrow are usually well
circumscribed and localized away from the bone (not depicted). In this case, notice
that the lymphoma cells are small and mature-appearing.

Ancillary studies.
For evaluation of a hematologic malignancy, flow cytometry, cytogenetic studies and
molecular studies can be extremely useful and sometimes essential. Some ancillary
studies need portions of the blood or bone marrow sample to be handled differently. For
example, flow cytometry and classical cytogenetic analysis require fresh sample that has
not been exposed to fixative.

A. Flow cytometry.
The lecture will introduce the use of flow cytometry for acute leukemias and mature B-
cell neoplasms (chronic B-lymphoid leukemias and B-lineage non-Hodgkin lymphomas).
The utility of flow cytometry will also be demonstrated during Hematopathology
Workshop II: Acute Leukemias (Wednesday, Jan. 14th, 3:00-5:00pm).

Flow cytometry lends itself to the evaluation of hematopoietic cells because these cells
are either already suspended in a liquid (blood or bone marrow) or can be fairly easily
dissociated from tissue by mincing and put into suspension. This is unlike mesenchymal
or epithelial tissue for which there is much greater adhesion between cells. Preparation
of the fluid sample includes being incubated with multiple antibodies directed toward
antigens of interest that might be present on the hematopoietic cells. The antibodies are
bound to different fluorochromes. The fluorochromes that have been developed for this
purpose can be excited by one or two laser beam wavelength(s) during flow cytometry.
However, each fluorochrome will emit a unique wavelength that can be used to discern
which antibodies have bound to the cells. Standard clinical flow cytometry utilizes 3-6
antibodies per aliquot of sample that is analyzed. See the tables below for examples of
common markers (antigens) that are evaluated by flow cytometry.

Tables: Examples of phenotypic markers useful for evaluation of hematologic


malignancy.
CD45 Also known as leukocyte common antigen (LCA). Present on all leukocytes.
CD34 Present on pluripotent hematopoietic stem cells and many progenitor cells of
hematopoietic lineage. Absent on cells differentiated beyond blast stage.

Some lineage associated antigens


CD3, CD4, CD5, CD8 T-lineage
CD19, CD20, CD22 B-lineage
CD13, CD33, Myeloperoxidase (MPO) Myeloid lineage
In addition to the evaluation of the presence of binding of the antibodies to the cells
during flow cytometry, each cell is also evaluated for two inherent properties of light
scatter: forward light scatter (FLS) and side scatter (SS) light characteristics. FLS is
collected at an angle of 2-10 from the incident laser light, and largely indicates cell size.
SS is collected at 90 from the incident light, and largely indicates cytoplasmic
complexity. Lymphocytes are small while monocytes and blasts are larger.
Lymphocytes and blasts emit very little side scatter light because they have scant
cytoplasm and round nuclei (i.e. non-complex), while monocytes, neutrophils, and
eosinophils emit substantially more due to a variety of characteristics related to
cytoplasmic granules and nuclear segmentation (i.e. more complex).

After the sample preparation, the sample is run through the flow cytometer. The cells are
hydrodynamically focused so that each will pass individually through the one or two laser
beams. Based on the light emitted from the cells, 5 to 8 properties of each cell are
collected (i.e. FLS, SS, and the presence of binding of the 3-5 fluorochrome labeled
antibodies). The data is displayed using multiple two dimension dot plots. Each dot
represents a cell and the 2 axes represent two of the 3-6 light scatter or emission
properties. Increments along the axes correspond to increased intensity of the light
scattered or emitted. See the lecture slides for examples of dot plots.

Flow cytometric immunophenotyping of acute leukemia is nearly essential nowadays for


a definitive determination of the lineage of the leukemic blasts (myeloid, B-lineage or T-
lineage). This will be illustrated in lecture and during Hematopathology Workshop II.
Furthermore, the immunophenotype of the leukemic blasts is well characterized at
diagnosis and this aids in the follow-up bone marrow evaluations after therapy.
Commonly, the immunophenotype of the leukemic blasts differs in some way from the
analogous normal blast immunophenotype (i.e. the leukemic blast has an aberrant
immunophenotype). For example, the phenotype of a leukemic myeloblast population
may differ from the phenotype of a non-neoplastic myeloblast population because it
aberrantly (i.e. abnormally) expresses CD19, which is a marker associated with B-
lineage. There are other more subtle, but more common aberrancies that can also help
with the distinction between a low level of residual or recurrent acute leukemia and
normal blast populations.

Flow cytometry of mature B-cell neoplasms is also frequently very useful. Most
neoplastic mature B-cell populations retain the ability to express immunoglobulin
receptor on their surface. However, a neoplastic population is monoclonal and will
express the same immunoglobulin receptor on nearly all cells and the light chain
component of the receptor will be of one type (either all kappa or all lambda among the
population). In contrast, a benign population of B lymphocytes will all express various
different immunoglobulin receptors, including a mix of cells with associated kappa light
chains and lambda light chains. The normal ratio of B-cells expressing kappa on their
surface to B-cells expressing lambda is 0.6-2.5 to 1. When the kappa to lambda ratio is
outside of this range (i.e. less than 0.6 or greater than 2.5), then we worry that there is a
monoclonal B-cell population present. In fact most times we evaluate a mature B-cell
neoplasm, we find almost exclusively a single type of light chain. This monotypic
expression of one type of light chain is an excellent surrogate marker indicating
monoclonality among mature B lymphoid populations. In addition, we further
characterize the neoplastic mature B-cell population by other markers. Often we find that
the population can be distinguished from normal mature B lymphocytes in ways other
than by monotypic light chain expression alone. The immunophenotypic signature of the
mature B-cell neoplasm can help to further classify it. For example, CD5 is generally
seen primarily among T lymphocytes. However, expression CD5 is also characteristic
for some mature B-cell neoplasms (i.e. chronic lymphocytic leukemia and mantle cell
lymphoma) and its detection begins to help us classify the neoplasm. These concepts of
flow cytometry as applied to mature B-cell neoplasms will be reinforced during lecture,
as well as during the lecture by Dr. Swerdlow (Pathology of lymphoproliferative
disorders, Wednesday, January 14th, 1:00-2:00pm), and during Hematopathology
Workshop III: Lymphomas (Friday, January 16th, 8:00-10:00am).

B. Cytogenetic studies.
For cancer cytogenetics, recall that we are looking for acquired abnormalities found
among the chromosomes of the neoplastic cells. Cytogenetic studies play a major role in
the diagnosis and classification of hematologic malignancies. Abnormalities may also
provide prognostic information. New abnormalities detected during follow-up of a
malignancy heralds evidence for disease progression. The most commonly used
cytogenetic studies utilized are (1) classical or conventional cytogenetic analysis (i.e. G-
banded metaphase analysis) and (2) fluorescence in situ hybridization (FISH). Here we
will review conventional cytogenetic analysis. The use of FISH will also be illustrated
during Hematolopathology Workshops II and III, and during Dr. Swerdlows lecture
(Pathology of lymphoproliferative disorders).

Evaluation of metaphase preparations from twenty cells is considered optimal for a


conventional cytogenetic analysis. An abnormality is generally considered clonal when
it is detected in at least two of the metaphase preparations. Abnormalities found in only
one metaphase preparation may be an in-vitro artifact. To obtain metaphase preparations,
the live cells must be able to grow in culture. The cells are arrested in mitosis, using
mitotic spindle inhibitors (e.g. colcemid). The cells are dropped onto a glass slide. The
chromosomes are stained and either photographed or imaged. A cytogenetic
technologist, nowadays with the help of software, arranges the chromosomes into pairs.
Recall that somatic cells contain 46 chromosomes and these comprise 22 homologous
pairs of autosomes and 2 sex chromosomes. A karyotype is a standard arrangement of a
photographed stained metaphase spread in which chromosome pairs are arranged in order
of decreasing length.

As a review, karyotypes are described using shorthand notations in the following order:
total number of chromosomes, sex chromosome complement, and any abnormalities in
ascending numerical order. The short arm of a chromosome is designated as p while the
long are is q. The chromosome arms are divided into two or more regions by
prominent bands seen morphologically and these are further divided into additional
bands. An example of acute promyelocytic leukemia in a male patient has the
following karyotype: 46,XY,t(15;17)(q22;q21)[15]/46,XY[5]. Fifteen of the 20 cells
display an abnormal karyotype with a reciprocal translocation between the long arm of
chromosome 15 at region 2, band 2 and the long arm of chromosome 17 at region 2, band
1. The PML gene is at 15q22 and the gene for the retinoic acid receptor alpha (RARA
gene) is at 17q21. The translocation results in production of an altered retinoic acid
receptor. Instead of its normal function in activating transcription, the altered receptor
(i.e. the RARA-PML fusion protein) represses target genes and blocks granulocyte
differentiation at a stage akin to blast/promyelocytes.

In addition to acute promyelocytic leukemia, there are 3 other acute myeloid leukemias
that are principally classified based on distinct cytogenetic abnormalities:
t(8;21)(q22;q22), inv16(p13;q22) and translocations involving 11q23 . The reason that
these four leukemias are classified based upon their cytogenetic abnormality is that these
abnormalities largely define their biology. Thus, each of these leukemias with their
pathognomic cytogenetic abnormality displays distinct clinical features (e.g. presentation,
prognosis, and response to therapy) and distinct pathologic features (e.g. morphology and
immunophenotype). Acute myeloid leukemia with t(15;17)(q22;q21) (i.e. acute
promyelocytic leukemia) is highlighted during lecture and during Hematopathology
Workshop II because it is so important to make this diagnosis promptly as it can be
rapidly fatal, and because therapy is directed at the tumor-specific genetic abnormality.
Fortunately, prompt recognition of this diagnosis and appropriate therapy, including high
doses of retinoic acid that overcome the block in differentiation, results in a cure rate of
60-70%. There are many other cytogenetic abnormalities that are well described and
associated with acute myeloid leukemias, acute lymphoblastic leukemias, mature B-cell
neoplasms, and all the other hematologic malignancies. As more is learned about
particular genetic and molecular abnormalities, the more they are used to classify
malignancies, to predict therapeutic outcomes, and to design targeted therapies.

C. Molecular studies
The use of PCR [DNA-PCR and reverse-transcription PCR (RT-PCR)] is fairly common
in the evaluation of hematologic malignancies. Southern blot analysis can also be used,
but has been largely replaced by PCR because PCR can be performed on fresh or
formalin-fixed tissue, can utilize DNA or RNA as a template, and is less time consuming.
One major use for molecular studies in hematopathology is to evaluate for clonal
rearrangement of the antigen receptor genes among mature lymphoid neoplasms (i.e.
chronic lymphoid leukemias and lymphomas). Clonal rearrangements among the antigen
receptor genes can be very helpful in distinguishing neoplastic from non-neoplastic
lymphoid proliferations, especially in cases for which morphology and
immunophenotyping do not provide enough information for definitive diagnosis.
However, hematologic malignancies other than B- and T-lymphoid neoplasms (e.g.
myeloid neoplasms) do not typically exhibit clonal gene rearrangement of the
immunoglobulin or T-cell receptor genes. Furthermore, antigen receptor gene
rearrangements do not define the biology of the malignancies. Therefore among most
types of hematologic malignancies, molecular studies are also used to look for disease
specific abnormalities (e.g. PML-RARA in acute promyelocytic leukemia; IGH-BCL2 in
follicular lymphoma or in a subset of diffuse large B-cell lymphomas), evaluate for
abnormalities with prognostic significance, and to evaluate for residual disease after
therapy. PCR is very sensitive and is able to detect a clone that represents 1 cell per
100,000 to 1,000,000. Therefore, PCR is very useful to evaluate for minimal residual
disease.

The immunoglobulin heavy chain gene (IGH) and the TCR gamma-chain are most
frequently utilized for the evaluation of clonality among B- and T-cells, respectively. In
a polyclonal population of lymphocytes, there are many possible rearrangements of the
exons that encode the IGH or TCR molecules. In a polyclonal population, PCR of the
DNA encoding either the IGH or TCR genes results in many products of variable sizes
due to different rearrangements as well as additional nucleotide insertions and deletions
between rearranged exons. Separation of PCR products based on size differences by gel
electrophoresis will result in a visual smear of differently sized products. Analysis of the
products by capillary electrophoresis will result in a tracing with multiple peaks. On the
other hand, lymphoid cells of a clonal population will usually all have the same
rearrangement. PCR of DNA in a clonal population then results in a discrete band with
identically sized products migrating the same distance on an electrophoretic gel, or will
result in a tracing with a dominant peak when using capillary electrophoresis. For an
example of tracings of the analysis of TCR-gamma gene by capillary electrophoresis, see
Figure 161-4 in Hoffman: Hematology: Basic Principles and Practice, 4th ed, available
through HSLS, as a Health Sciences E-Book:
http://www.mdconsult.com/das/book/body/83182686-2/0/1267/I4-u1.0-B0-443-06628-
0..50164-4--f4.fig?tocnode=49293926.
Pharmacology Part I
Chemotherapy of Hematological Lecture Objectives
Malignancies

To provide a broad overview of how drugs used


Pharmacology Part II for hematological malignancies are classified.
Targeted
Targeted Therapies To describe the mechanism of action and the
toxicities of a few prototype cytotoxic agents
John S. Lazo used for hematological malignancies.
January 9, 2009 To describe the mechanism of action of
prototype targeted agents used for
BST-
BST-3 10040 hematological malignancies.
Department of Pharmacology & Chemical Biology
Key Words:
lazo@pitt.edu Mechanism of Action, Toxicities, Targeted Therapy, Tyrosine Kinase Inhibitors

I. References Cancer victims Multiple myeloma

Lippincotts Illustrated Reviews. Pharmacology 3rd Ed. Howland, Mycek,


& Harvey. Chapter 39. pp 453-485. 2006. Sound and clear: telegraphic.

Goodman and Gilman, The Pharmacological Basis of Therapeutics, 11th


Ed. Brunton, Lazo & Parker, Ed. pp. 1315-1403, 2006. Comprehensive.

Katzung, Basic & Clinical Pharmacology, 10th Ed. Chapter 55, 2006.
Considerable information about clinical usage.

http://www.fda.gov/cder/cancer/approved.htm This lists all of the FDA


T-cell lymphoma Non-Hodgkins
approved anticancer drugs with indications. National Comprehensive
Comprehensive
Cancer Network (NCCN). http://www.nccn.org. Lists drug protocols. Hodgkins

Five-
Five-Year Relative Survival Rates Reasons for Ineffective Anticancer Drugs
1
32

Myeloma 24 Cancer is likely due to the clonal expansion of a single neoplastic cell
6 and proliferation of malignant cells is not fully controlled. Currently,
the most effective anticancer drugs act simply by killing diving cells.
86 Many tumors, however, do grow as rapidly as normal cells, such as those
Hodgkin's
Percent Survival

72 in bone marrow, intestinal epithelium and hair follicles. This leads to


Lymphoma
40 1995-2000 toxicity to normal tissues and limits therapy.
1974-1976 Many anticancer drugs do not target tumor stem cells.
Non- 60 1960-1963 Some tumors are in drug sanctuaries because of physical barriers to
Hodgkin's 48 the drugs, e.g. brain or testes or because of tumor biology, e.g. hypoxia.
Lymphoma 37 The size of the tumor can exceed the efficacy of the drugs used.
Drug resistance can develop.
48
Leukemia 35
14

0 20 40 60 80 100

Source: Surveillance, Epidemiology and End Results (SEER) Program.


Program. National Cancer Institute 2004
Acute Lymphoblastic Leukemia Treatment Regimens
Tumor Cell Burden and Relation to Phases of Therapy

Untreated Success: 10% complete remission in the 1960s to 70-90% now


Treatment Death
Visible
1012 Cancer
Remission Induction Consolidation Maintenance
1010 Palliation Intensification
# of 1-2 months >30 months
Cancer 108 Non-visible 3-9 months
Cells Cancer Vincristine Thiopurine
106 Prednisone Cytarabine Methotrexate
Anthracycline Methotrexate Vincristine
104 (doxorubicin or Hydrocortisone Prednisone
daunorubicin)
102 Cyclophosphamide
L-asparaginase

Cure

Time

Principles of Combining
Anticancer Drugs Cancer Drug Combinations
With >100 possible cancer drugs, there are tens of
thousands of possible combinations.
Non overlapping toxicities
Different mechanisms of action VBP = vinblastine, bleomycin & cisplatin; testicular cancer
ABVD = Adriamycin, bleomycin, teniposide, prednisone
Different mechanisms of resistance MOPP-BAP = mechlorethamine, Oncovin (vincristine), procarbazine,
bleomycin, Adriamycin (doxorubicin), prednisone)
CVP = cyclophosphamide, vincristine, prednisone
HDMTX-CF = high-dose methotrexate, citrovorum factor
VIP-B = VP-16 (etoposide), ifosamide, Platinol (cisplatin), bleomcyin

http://www.mtdaily.com/mt1/lists/chemo.html

Specific Anticancer Agents Sites of Inhibition by Anticancer Agents.


Most textbooks lists >50 anticancer drugs. We must organize them. The compounds Purine 6-Mercaptopurine Pyrimidine
can be grouped mechanistically (or the source) rather than by their therapeutic Synthesis 6-Thioguanine
Nalarabine
Synthesis

uses. Because of their low therapeutic indices, the doses of antineoplastic agents Inhibits purine synthesis

are sometimes calculated on the basis of the patients body surface (in square
meters) rather than in body weight. Dose intensity concept: mg/m2/wk. Ribonucleotides
Methotrexate
Pemetrexed
Inhibits dihydrofolate
reductase Alkylating agents,

Alkylating agents
Cyclophosphamide
Deoxyribonucleotides Cross link DNA
Cyarabine
Inhibits DNA polymerase Doxorubicin

Antimetabolites
Daunorubicin
Etoposide
Gemcitabine Inhibit topoisomerase II
Inhibits ribonucleotide
reductase

Natural Products Causes chain termination

DNA
Vorinostat
Inhibits histone deactylase Bleomycin

Hormones Bortezomib
Proteosome inhibitor
Imatinib
Dasatinib
RNA
Cleaves DNA

Arsenic Trioxide Inhibit growth factor

Anti-Signaling Agents Sulfhydryl reactive (?) signaling Vinca alkaloids


Thalidomide Rituximab Paclitaxel
Proteins
Lenalidomide Binds to CD20 Inhibit microtubule function
Antiangionesis, complex Gemtuzumab

Miscellaneous
Tretinoin Binds to CD33
Differentiation (?)
Microtubules
L-asparginase Enzymes, Growth Factors
Deplete aspargine & Receptors
IV. Drug Combinations V. Some Current Hot Topics in the
Treatment of Hematological Disorders
Combinations are the rule in the treatment of hematological
disorders. Personalized Chemotherapy
With >100 possible cancer drugs, there are tens of thousands - Biomarkers: Cellular, Protein, Cells, Genomics, or Proteomics
- Probably will first be done in hematological diseases
of possible combinations. As with the treatment of solid
tumors, it is almost impossible to remember the names of all
Highly selective vs. multitargeted compounds
of the combinations (nor should you at this stage!).
- Methotrexate vs pemetrexed; imatinib vs dasatinib
Examples of combinations include:
Endpoints
ABVD (Adriamycin (trade name for doxorubicin), bleomycin, - Overall survival: Disease-
Disease-free or progression-
progression-free
vinblastine, dacabazine) for Hodgkin disease; - Response: Tumor size or patient benefit
- Protection against cancer vs treatment
CVP (cyclophosphamide, vincristine, prednisone) for CLL;
Phase 0
ICE (ifosfamide, carboplatin,etopside) or DHAP - Phase I, II & III are well established clinical trial milestones.
milestones. Phose O is
microdosing with non-
non-therapeutic doses for pharmacokinetic and pharmacodynamic
(dexamethasone, high dose ara C (another name for data
cytarabine), Platinol (trade name for cisplatin) for multiple
myeloma. Vaccines
- Recently approved HPV vaccine for cervical cancer.
cancer.
http://www.mtdaily.com/mt1/lists/chemo.html

Therapeutic clinical trial based on in vitro sensitivity Therapeutic clinical trial based on gene array data

Ugurel, S. et al. Clin Cancer Res 2006;12:5454-5463

Some Current Hot Topics in the Alkylating Agents


Treatment of Hematological Disorders
Personalized Chemotherapy Chemistry. They have at least one CH2R group.
- Biomarkers: Cellular, Protein, Cells, Genomics, or Proteomics
- Probably will first be done in hematological diseases Nitrogen mustards: Melphalan, cyclophosphamide,
Highly selective vs. multitargeted compounds chlorambucil & melphalan
- Methotrexate vs pemetrexed; imatinib vs dasatinib
Endpoints
- Overall survival: Disease-
Disease-free or progression-
progression-free Mechanism of Action: These contain reactive groups that
- Response: Tumor size or patient benefit bind covalently with biologically important cell
- Protection against cancer vs treatment
Phase 0 constituents, e.g. DNA. Usually there are two reactive
- Phase I, II & III are well established clinical trial milestones.
milestones. Phose O is moieties and they cross link DNA.
microdosing with non-
non-therapeutic doses for pharmacokinetic and
pharmacodynamic data
Vaccines
- Recently approved HPV vaccine for cervical cancer.
cancer.
Mechanism of alkylation.
H2 H2 C C Cl
C C Cl H2
H2

CH3 Mechlorethamine
Mechlorethamine
Mechlorethamine H3 C N
C
H2
C
H2
Cl
H3 C N
+
C
CH2

Imonium ion
H2
NH2
Nitrogen mustard O

CH2CHCOOH
Melphalan
Melphalan
N

Melphalan HN

N
H2N N
ClH2CH2C
HN O
H2C C N C C Cl
R

H2 H2 H2
N +N Guanine in DNA
Cyclophosphamide
Cyclophosphamide
CH3
P O HN

ClH2CH2C Cyclophosphamide
N
O
1959
H 2N N

CH2CH2CH2COOH
Chlorambucil
Chlorambucil
Chlorambucil O
Interstrand DNA cross link by nitrogen mustard
N
HN O
H2 H2 O
+ C C
H2
N C
H2
C
+
N N
H 2N N N
HN CH3

Figure4.
4. Nitrogen mustard alkylating
alkylating agents
NH
R
Figure Nitrogen mustard agents
Nitrogen mustard alkylating agents
Guanine in DNA N
N N NH2
H2N N
R
R

Alkylating Agents (cont)

Absorption, Distribution and Metabolism:


i. Melphalan, chlorambucil, CCNU, BCNU and busulfan are orally active.
ii. The R-group of cyclophosphamide undergoes hepatic metabolism to give
the active alkylating agent, phosphoramide mustard, thus, it is really a
prodrug: cyclophosphamide itself is not active.

Cyclophosphamide
Uses of cyclophosphamide (Cytoxan
Cytoxan):
Acute lymphoid leukemia (ALL), acute myeloblastic leukemia (AML),
Hodgkin's disease, malignant lymphoma, multiple myeloma (MM).
Mechlorethamine: Chronic lymphoid leukemia (CLL), chronic myeloid
leukemia (CML) & Hodgkins disease.
Melphalan: MM & non-Hodgkins disease.
Chlorambucil: CLL, Hodgkins disease & lymphoma.

Cyclophosphamide metabolism Antimetabolites

Antimetabolites are compounds that bear a structural similarity to


normally occurring substances, such as vitamin, nucleoside, or amino acid.
They compete with the natural substrate for the active site on an
essential enzyme and/or substitute for the natural compound in DNA.
Most antimetabolites inhibit DNA synthesis; they kill cells in S-phase.
Although bone marrow depression occurs with all the drugs in this group,
oral and GI ulceration can be dose limiting for some (e.g.,
methotrexate). The antimetabolites require biotransformation to
produce their cytotoxic effects; conversion to the active form takes
place intracellularily and is referred to as lethal synthesis.

Hepatic cell cytoplasm


Pteridines & Antifolates Methotrexate
Chemical Mimicry Chemistry and Mechanism of Action.
Methotrexate (1953) is a folic acid H 2N N
analog that competitively inhibits
dihydrofolate reductase (DHFR). The
inhibition of tetrahydrofolate synthesis
interrupts one carbon transfer
reaction: the synthesis of thymidylate,
purine nucleotides and the amino acids
N
N glycine and methionine. Methotrexate
kills cells in S-phase. Methotrexate also
Folic acid
Aminopterin
R1
OH
NH2
Methotrexate NH2
R2
H
H
CH3

inhibits RNA and protein synthesis; it


can block cellular entry into S-phase.
R1

Figure 7. Mechanism of action of methotrexate.


Deoxyuridylic acid Thymidine monophophate
(dUMP) (TMP)
O
O

HN
Thymidylate synthetase HN CH3

N N
O O

deoxyribose-P deoxyribose-P
FH2
N5,N10-methylenetetrahydrofolate

Dihydrofolate
Reductase
FH4
Methotrexate
Leucovorin

Folate dependent biosynthesis. Cycle of reactions in the synthesis of thymidylate (dTMP)


dTMP) from dUMP.
dUMP.
Thymidylate synthase catalyzes the first reaction of this cycle producing dTMP.
dTMP. The other product of
the reaction, dihydrofolate,
dihydrofolate, must be reduced by NADPH in a reaction catalyzed by dihydrofolate
reductase before a methylene group can be added to regenerate 5,10- - methylene-
5,10 methylene - tetrahydrofolate.
Methylenetetrahydrofolate is regenerated in a reaction catalyzed by serine hydroxymethyltransferase.
hydroxymethyltransferase.

Methotrexate
Complexities
of folate
biosynthesis

Absorption, Distribution and Metabolism. It is well


absorbed orally. It is a weak acid, secreted, and
eliminated primarily by the kidney. It is plasma bound
(50%). It does not penetrate the blood-brain barrier
well. It has been administrated intrathecally.
Methotrexate is metabolized intracellularily to
polyglutamate derivatives; these are retained by cells
better and are better inhibitors of DHFR than the
parent compound.
Therapeutic uses include: ALL.
The toxic effects of methotrexate are bone marrow,
oral and GI mucosa.
Hepatic cell cytoplasm
Pteridines Pemetrexed - Alimta
This is a unique antifolate in that it has been shown to
inhibit at least five of the major folate-
folate-dependent
enzymes.
Thymidylate synthase
Dihydrofolate reducatase
Glycinamide ribonucelotide formyltransferase
5-aminoimidazole-
aminoimidazole-4-carboxamide ribonucleotide
H2N
H
N H
formyltransferase
C-1 tetrahydrofolate synthetase
N
N N NH 2

Premetrexed was approved in February 2004 for


N CH 3

N N

malignant plural mesothelioma


O H N
N COOH H
HOOC N NH 2

O COOH This is an example of a dirty drug


drug or a multi-
multi-targeted
targeted
drug.
COOH O

Pemetrexed Methotrexate

Pemetrexed Survival Improvement Pyrimidines:


Pyrimidines: Cytarabine (AraC)
AraC)

Chemistry. Cytarabine (also known as Ara-C or Cytosar-U)


(1969) is 1--D-arabinofuranosylcytosine.

Absorption, Distribution and Metabolism. Only poor bioavailability after


oral dose (~20%) because of cytidine deaminase in the GI and liver.
Must be given iv by continuous infusion because of short half life (~10
min). Less than 10% of the injected drug is excreted unchanged. Like
other antimetabolites, Ara-C must be anabolized to Ara-CTP.
NH 2

N H 2
N

O N
H N HO
O
HO

O N
HO

Cytosine Cytarabine

Pyrimidines:
Pyrimidines: Cytarabine (cont) Pyrimidines:
Pyrimidines: Cytarabine (AraC)
AraC)

Mechanism of Action. The most important cause of cell Acute nonlymphocytic leukemia
death is incorporation into DNA with subsequent inhibition of
DNA polymerase. Ara-C also induces terminal differentiation Acute lymphoid leukemia
of cells.
Toxicity. Ara-C is a potent myelosuppressive agent capable
Acute myeloid leukemia
of causing acute, severe leucopenia, thrombocytopenia and Chronic myeloid leukemia
anemia with striking megaloblastic changes. It can also cause
Non-Hodgkins lymphoma
GI toxicity, stomatitis and noncardiogenic pulmonary edema.
Purines: Mercaptopurine and Thioguanine
Purines: Mercaptopurine and Thioguanine

Administration, Distribution and Metabolism. Mercaptopurine and thioguanine


Chemistry. Mercaptopurine and thioguanine (1966) are analogs of are routinely given orally. They are metabolized extensively and the
hypoxanthine and guanine. metabolites are excreted in the urine; 50% of the administered drug is found
in the urine after 24 hr. Xanthine oxidase, which catalyzes the first step in
Mechanism of Action. Both of these drugs must be converted mercaptopurine breakdown, is inhibited by allopurinol. Allopurinol is often
intracellularily to the nucleotide forms by hypoxanthine-guanine used with anticancer drugs to prevent the hyperuricemia and uricosuria that
phophoribosyl transferase (HGPRT). Cytotoxicity occurs by several follows marked cell kill. The cytotoxic effect and toxicity of mercaptopurine
mechanisms including feedback inhibition of the first and rate- is increased in the presence of allopurinol; you must reduce the dosage of
mercaptopurine by or 1/3 of the usual dose. Thioguanine is not extensively
limiting step in purine biosynthesis and inhibition of purine
metabolized by xanthine oxidase; it can be used in combination with allopurinol
interconversions. In addition, the purine analogs have effects on without the dosage being reduced.
DNA and RNA synthesis, glycoprotein synthesis and chromosomal
replication.
Figure 8. Chemical structures of endogenous and clinically used purines.
Toxicity. Myelosuppression is a common side effect with both purine analogs.
Hepatotoxicity is less common with 6-thioguanine than with 6-mercaptopurine.
OH SH OH OH SH Approximately 1/3 of patients develop jaundice after 6-mercaptopurine; this
N
N
N
N
N N
N
N
N is associated with bile stasis and hepatic necrosis.
N

N N N N N N
H N N N N
H H H2 N H H 2N H
Hypoxanthine 6-Mercaptopurine Allopurinol Guanine 6-Thioguanine

Purine Metabolism Nelarabine


Chemistry. Nelarabine is a water-soluble nucleoside prodrug of 9-beta-D-
arabinofuranosylguanine (ara-G). It was FDA approved in October 2005
for the treatment of patients with T-cell acute lymphoblastic leukemia.

Administration, Distribution and Metabolism. Given iv. Primary route of


metabolism is O-demethylation by adenosine deaminase to form ara-G.

Mechanism of Action. Nelarabine is a prodrug of the ara-G. Nelarabine is


demethylated by adenosine deaminase to ara-G, mono-phosphorylated by
deoxyguanosine kinase and deoxycytidine kinase, and then converted to the
active 5' -triphosphate, ara-GTP. Accumulation of ara-GTP in leukemic
blasts allows for its incorporation into deoxyribonucleic acid (DNA), leading
to inhibition of DNA synthesis and cell death. There may also be other
mechanisms that contribute to the cytotoxic and systemic toxicity of
nelarabine .

Toxicity. Neurologic including severe somnolence, central nervous system


effects including convulsions, and peripheral neuropathy ranging from
numbness and parenthesis to motor weakness and paralysis.

Natural Products Anthracyclines

Unlike the other classes of antitumor agents, Chemistry. There are two anthracyclines: doxorubicin (Adriamycin) (1974)
and daunorubicin.
membership in this group is determined by the source of
the drug. They are obtained from lower organisms, such Absorption, Distribution and Metabolism. Both doxorubicin and
as plants and Streptomyces. With the exception of the daunorubicin are rapidly taken up by all tissues except the brain.
brain. They are
extensively bound to cellular components, which is responsible partially
partially for their
vincas and bleomycin the natural products are phase long plasma half-
half-life. They are cleared mainly by hepatic metabolism; severe
clinical toxicity may result if there is impaired hepatic function.
function.
nonspecific. The chemical structures are often complex
and, therefore, not always presented. a. Mechanism of Action. Inhibition of topoisomerase II enzyme catalyzes the
breaking and rejoining of DNA strands. Stabilization of the enzyme-DNA
complex leads to inhibition of DNA synthesis and cell death.
O OH
O

CH 2R
OH

O
OH O OH

R CH 3

Doxorubicin OH OH
Daunorubicin H
NH 2
A. Normal catalytic cycle of topoisomerase II

Doxorubicin Toxicity Topoisomerase II

Toxicity. Bone marrow depression and alopecia. The DNA


most serious toxicity (irreversible) is Transient cleavable complex
cardiomyopathy. There are two types: acute and
chronic. The acute form is characterized by
abnormal ECG changes; it is reversible and is not a
predictor of chronic cardiomyopathy. The second Noncleavable complex
form is a chronic cumulative does-related toxicity,
manifested by congestive heart failure that is B. Etoposide, daunorubicin, and doxorubicin causes double strand
strand
unresponsive to digitalis. For this reason the total DNA breaks
dose of doxorubicin is kept below 500 mg/sq. m. The daunorubicin
mechanism of cardiac toxicity is unknown but may be
due to the excessive local production of free radicals
in the myocardium. Irreversible double-strand breaks in DNA

Persistent cleavable complex

Etoposide Etoposide

Chemistry. Etoposide (1996) and teniposide are analogs of the


alkaloid, podophyllotoxin, which is obtained from the Mandrake Mechanism of Action. Both etoposide and teniposide
(may apple) plant (used by American Indians and early colonists). form a ternary complex with topoisomerase II and
DNA, preventing resealing of the topoisomerase II-
Absorption, Distribution and Metabolism. Etoposide is available induced DNA break. The enzyme remains bound to the
for iv use or oral administration, although absorption is variable free end of the broken DNA strand, leading to an
and ranges ~50%. After iv injection the terminal half-life is 6-8 accumulation of DNA strand breaks and cell death.
h. Approximately 40% of the drug is excreted in the urine Thus, they poison the enzyme. Cell in S and G2 are
unchanged so in patients with compromised renal function the most sensitive. Both drugs are affected by p-
dose must be reduced. glycoprotein.

Uses: Etoposide: Acute nonlymphocytic leukemia and


Non-Hodgkins lymphoma; Teniposide: ALL and Non-
hodgkins lymphoma.

Bleomycin
Chemistry. Bleomycin (1973) is a mixture of several different complex
glycopeptides extracted from Streptomyces.

Absorption, Distribution and Metabolism. Bleomycin is a peptide and


cannot be administered orally. More than of drug is excreted in the
urine unchanged within 24 hr. Bleomycin is inactivated in most tissues by
bleomycin hydrolase; the sensitivity of lungs and skin to the toxicity of
bleomycin is believed to be due to low levels of bleomycin hydrolase.

Mechanism of Action. Bleomycin forms a complex with iron and molecular


oxygen and then binds to double strand DNA and causes both single and
double strand breaks; the double strand breaks are believed to cause cell
death. The DNA damage is nucleotide-sequence specific. For most cells
the G2 phase is the most sensitive.
Bleomycin
Toxicity. The most serious adverse effect is pulmonary toxicity.
This manifestation begins with decreasing pulmonary function, fine
rales and cough; and it progresses to severe and sometimes fatal
pulmonary fibrosis. An unusual toxicity is a lethal anaphylactoid
reaction that occurs in patients with lymphoma; small test doses are
commonly given. It is important that bleomycin does not have
significant myelosuppressive effects and, thus, it is a popular agent
for combination chemotherapy.

Vinca Alkaloids
Vincas
Chemistry. The two members of this group are:
vincristine and vinblastine; both are derived from the
periwinkle plant. They have complex chemical structures.

Absorption, Distribution and Metabolism. Both


agents are given i.v. and they bind extensively to tissue
components; therefore long half-life. They do not pass
the blood-brain barrier. Both are metabolized by the Paclitaxel
liver and excreted in the bile, although vinblastine is Mechanism of action. Both
cleared 7x more slowly than vincristine. vincristine and vinblastine bind
specifically to the microtubule
proteins and blocks
polymerization. The dissolution
of the mitotic spindle interferes
with chromosome segregation and
leads of mitotic arrest.

Pharmacology III
Targeted Therapies
Imatinib
(Gleevec)

John S. Lazo FDA and Novartis, the drug's manufacturer, should be


January 8, 2009 commended for the rapid development and review that will
make this product available soon for the leukemia patients
who desperately need it," said Health and Human Services
BST-
BST-3 10040 Secretary Tommy G. Thompson. "It is also important to
recognize that today's approval is also a culmination of
Department of Pharmacology & Chemical Biology years of work and years of investment, by many people in
lazo@pitt.edu many different institutions, and even in different fields of
medicine. It's a testament to the groundbreaking scientific
research taking place in labs throughout America."
FDA News May 10, 2001
Imatinib
Anti-signaling agents: Imatinib mesylate Mechanism of Action. Imatinib is a protein-tyrosine kinase inhibitor that
inhibits the Bcr-Abl tyrosine kinase, the constitutive abnormal tyrosine kinase
created by the Philadelphia chromosome abnormality in CML. Imatinib is also an
inhibitor of the receptor tyrosine kinases for platelet-derived growth factor
Chemistry. Gleevec was discovered by screening many thousands of (PDGF) and stem cell factor (SCF), c-kit, & inhibits PDGF- and SCF-mediated
compounds (so called high throughput screening) and it is an analog cellular events. In vitro, imatinib inhibits proliferation and induces apoptosis in
of ATP and binds to the ATP binding pocket of the Bcr-Abl tyrosine
gastrointestinal stromal tumor (GIST) cells, which express an activating c-kit
kinase..
mutation. Imatinib resistance due to mutations in the kinase domain of BCR-ABL
Absorption, Distribution and Metabolism. Imatinib is well
absorbed after oral administration with a mean absolute
bioavailability of 98%. CYP3A4 is the major enzyme responsible for
metabolism of imatinib.

Dasatinib (Sprycel)
a. Chemistry. Dasatinib is another analog of ATP, which was Bortezomib
approved by the FDA in June 2006 for treatment of patients
with CML and Philadelphia-chromosome-positive ALL with
resistance or intolerance to prior therapy (second-line therapy). Chemistry. Bortezomib (2003) is an unique boronic acid containing small
molecule.
b. Absorption, Distribution and Metabolism. Dasatinib is orally Absorption, Distribution and Metabolism. Bortezomib is administered iv.
active with good distribution. It is highly protein bound (96%)
and primarily metabolized in the liver by CYP3A4. It has a half-life of 6 h and is metabolized with the loss of the broronic
acid by CYP3A4 and CYP2D6.
c. Mechanism of Action. Dasatinib is an inhibitor of several kinases
including BCR-ABL, SRC family kinases, c-KIT and PDGFR.

d. Toxicity. Severe thrombocytopenia, neutropenia, and anemia.

O OH
H
N N B
N OH
Imatinib H

O Dasatinib
N
O
N
S

Bortezomib
Bortezomib Cytokine
activation of
Cytokines

receptors

NF-B

Mechanism of Action. Bortezomib binds to the 20S


core of the 26S proteosome and is a reversible Activation of
NK-B by IB NF-B
inhibitor of its chymotrypsin-like activity. Inhibition degradation activates
transcription
of the proteosome blocks multiple signaling cascades Proteasome
most importantly it decreases NK-B. This occurs
because bortezomib inhibits the degradation of IB Decreased
cell adhesions
Decreased
so NF-B activities are blocked. molecules
inflammatory
molecules
Degraded IB
Decreased antiapoptotic factors
Toxicity. Thrombocytopenia & peripheral neuropathy
p50

P65

IB
Rituximab
Rituximab (Rituxan)

Chemistry. Rituximab is a genetically engineered, chimeric monoclonal


antibody (the first monoclonal antibody to be approved by the FDA for the
treatment of cancer (1997)) directed against the CD20 antigen on the surface
of malignant B lymphocytes (non-Hodgkins lymphoma). CD20 participates in
the activation process for cell cycle initiation and differentiation. The CD20
antigen is found on cells from the pre-B-cell stage through terminal
differentiation to plasma cells and is expressed on 90% of B-cell neoplasm.
The biological functions of CD20 are unknown.

Absorption, Distribution and Metabolism. Because it is an antibody,


Rituximab must be given iv. It has a serum half-life of ~75 h with a rather
large range in values.

Mechanism of Action. Monoclonal antibody binding to CD20 generates transmembrane signals


-ximab = chimeric antibody (~33% mouse antibody). that produce autophosphorylation and activation of serine/tyrosine protein kinases. This
-umab = humanized antibody (~5-10% mouse antibody). leads to antibody dependent cellular cytotoxicity (ADCC) and cell death. Toxicity. Rituximab
toxicities are mostly related to infusion reactions

Miscellaneous/Mysterious Agents
Gemtuzumab (MytotargTM)
Thalidomide O

Chemistry. Gemtuzumab is a humanized monoclonal antibody against CD33 N O


that is covalently linked to a semisynthetic derivative of the small NH
molecule calicheamicin. As such it is among the first hybrid large and O O

small molecule. Approved by the FDA March 2000 for CD33 positive
AML. Chemical structure. An old drug (developed in the 1950s
as a sedative) with a very simple structure.
Absorption, Distribution and Metabolism. Because it is an antibody,
Gemtuzumab must be given iv in two doses. Serum half-life of ~45-65 h; Absorption, Distribution, and Metabolism.
administer second dose after 14 days. Slow and highly variable oral absorption. Wide tissue
Mechanism of Action. Gemtuzumab ozogamicin is cytotoxic to the CD33
distribution including in semen. Rapid spontaneous
positive HL-60 human leukemia cell line. Gemtuzumab ozogamicin binds nonenzymatic hydrolysis leads to >50 metabolites.
to the CD33 antigen expressed by hematopoietic cells which results in
formation of a complex that is internalized. Once internalized, the Toxicity. Most common side effects are sedation and
calicheamicin derivative is released inside the lysosomes of the myeloid constipation. Is a notorious teratogen causing dysmyleia
cell and binds to DNA which results in DNA double strand breaks and (stunted limb growth).
cell death. This results in significant inhibition of colony formation in
cultures of adult leukemic bone marrow cells. Mechanism(s) of action. Complex including direct
apoptosis, reduced tumor adhesion to matrix, inhibition
Toxicity. Severe myelosuppression, hypersensitivity reactions (including of angiogenesis, inhibition of cytokine release, and
anaphylaxis), hepatotoxicity. enhanced natural killer cell functionality

Schematic overview of proposed mechanism of antimyeloma activity of thalidomide

NK functionality and production NK Cell

Thalidomide Tumor Growth arrest/death


Cell

adhesion

Tumor
Cell

Bone marrow
stromal cells
IL-6
IGF-1
Cytokine release VEGF
SDF-1
bFGF Angiogenesis
TNF
Lenalidomide (Revlimid) Arsenic Trioxide
Approved by FDA for Myelodysplastic Syndrome
(deletion of 5q) on December 28, 2006. Chemistry. Arsenic was used >2,400 years ago in Greece and
Rome. Arsenic trioxide was used a century ago for syphilis and
Structurally related to thalidomide so used under a parasitic diseases. It was FDA approved in 2000.
risk management plan (RevAssist) design to prevent Absorption, Distribution and Metabolism. It is administered
fetal exposure. as a 2 h iv infusion. The primary mechanism is through
enzymatic methylation but no dose reduction is recommended
in patients with hepatic or renal dysfunction.
Mechanism of Action. The mechanism of action remains
uncertain. I causes cellular differentiation and promotes
apoptosis. It is highly reactive to sulfhydryls and it generates
this may be important
Toxicity. Well tolerated with sine reversible hyperglycemia
NH2 O and fatigue. Can lengthen QT interval.
O Use. CML
O

Lenalidomide NH Thalidomide

Tretinoin Vorinostat (ZolinzaTM)


Chemistry. All-trans retinoic acid (Tretinoin) induces a high rate of complete Chemistry. Approved by the FDA October 2006.
remissions in acute promyelocytic leukemia patients as a single agent and
can be used in combination with anthracyclines.
Absorption, Distribution and Metabolism. Orally active, metabolized
Absorption, Distribution and Metabolism. Orally active. Metabolized primarily by glucuronidation and hydrolysis followed by beta-oxidation into 2
in the liver by CYP. inactive metabolites with a 2 hour half-life.
Mechanism of Action. The precise mechanism of action has not been
established but it is not a cytolytic agent. Under physiologic conditions, Mechanism of Action. The first approved histone deacetylase
the RAR- receptor dimerizes with the retinoid X receptor to form a inhibitor.
complex that binds tretinoin tightly, displacing a repressor of
differentiation. Tretionoin induces cytodifferentiation and decreases
proliferation of APL cells. In patients who achieve complete remission, Therapeutic Uses. Primary cutaneous T-cell lymphoma.
tretinoin therapy results in an initial maturation of the primitive
promyelocytes derived from the leukemic clone, followed by a Toxicities. Prolonged QT interval, anemia, deep venous thrombosis,
repopulation of the bone marrow and peripheral blood with normal,
polyclonal hematopoietic cells. thrombocytopenia
Toxicity. Mild such as dry skin, bone tenderness and hyperlipidemia.

Use. APL. O
H
N
N
H
O

Summary The Future of Cancer Chemotherapy

Remember the major classes. 2027 future


Cocktails of novel
Remember the irreversible toxicities. 2007-2017 antisignaling agents
Cytotoxics and hormones become mainstay.
Targeted therapies are emerging as remain foundation. Diagnostics combined with
individual therapy is
exciting new agents and will grow in
Emergence of targeted
1992-2007 standard. Cytotoxics
antisignaling compounds.
Cytotoxic &
numbers. Increasing use of biologics. phased out. Small
hormonal molecules replace
Novel combinations of
agents are biologics. Cancer is viewed
cytotoxics along with other
mainstays as a chronic disease.
modalities.
Tailored, individualized
chemotherapies.
Neglected Drugs

Antimetabolites
Fludarabine
Deoyxcorformycin
2-Chlorodeoxyadenosine
Decitamine (5-aza-2'-deoxycytidine)

Natural Products
Idarubicin (anthracycline)

Antisignaling
Nilotinib (Targets Bcr-Abl - TKI)
Alemtuzumab (Targets CD52)

Miscellanous
Bisphophonates
Interferon
Hydroxyurea (Targets Ribonucleotide Reductase)
Hematology 2009
Chemotherapy of Hematological Malignancies
January 2009

I. Objectives

To provide a broad overview of how drugs used for hematological malignancies are
classified.
To describe the mechanism of action and the toxicities of a few prototype cytotoxic agents
used for hematological malignancies.
To describe the mechanism of action of prototype targeted agents used for
hematological malignancies.

II. Key Words

Mechanism of Action, Toxicities, Cytotoxic Therapy, Targeted Therapy, Tyrosine Kinase


Inhibitors

III. References

Goodman and Gilman, The Pharmacological Basis of Therapeutics, 11th Ed. Brunton, Lazo, &
Parker, Eds. pp. 1315-1403, 2006. Comprehensive.

Katzung, Basic & Clinical Pharmacology, 10th Ed. Chapter 55, 2006. Considerable information
about clinical usage.

Lippincotts Illustrated Reviews. Pharmacology 3rd Ed. Howland, Mycek, & Harvey. Pp. 453-
484. 2005. Good illustrations and reasonably concise.

A useful site:http://www.fda.gov/cder/cancer/approved.htm This lists all of the FDA


approved anticancer drugs with indications. National Comprehensive Cancer Network
(NCCN). http://www.nccn.org. lists clinical protocols.

IV. Specific Anticancer Agents

The agents used to treat hematological malignancies share many of the same mechanisms of
action and toxicities that are seen with agents for solid tumors. Indeed, many agents are used
for both types of tumors. As you may remember from last year, the anticancer agents are
generally grouped mechanistically, structurally or based on the source (i.e. Natural Products)
rather than by their therapeutic uses. Because of their low therapeutic indices, the doses of
antineoplastic agents are usually calculated on the basis of the patients body surface (in square
meters) rather than in body weight.

V. Combinations

Combinations are the rule in the treatment of hematological disorders. As with the treatment of
solid tumors, it is almost impossible to remember the names of all of the combinations (nor
should you at this stage!). Examples of combinations include: ABVD (Adriamycin (trade name
for doxorubicin), bleomycin, vinblastine, dacabazine) for Hodgkin disease; CVP
(cyclophosphamide, vincristine, prednisone) for CLL; ICE (ifosfamide, carboplatin,etopside) or
DHAP (dexamethasone, high dose ara C (another name for cytarabine), Platinol (trade name for
cisplatin) for multiple myeloma.

VI. Hot Topics in Hematological Drug Therapy

Personalized Chemotherapy. There is considerable interest in applying modern methodology


(diagnostics) to individualize cancer treatment but there are serious scientific, cultural, and
regulatory issues. Many individuals are trying to assess the potential role of biomarkers in
prospectively determining treatment protocols. Methods being used include cellular tests
(immunofluorescence, culturing), protein (biochemical, ELISA, Western blots, proteomics). The
answer will probably first come with hematological diseases.

Targeted drugs. Some are beginning to question whether or not we should be seeking highly
selective drugs rather than multi-targeted (use to be called dirty) drugs. e. g., Methotrexate
vs pemetrexed; imatinib vs dasatinib.

Therapeutic Endpoints. With the advent of so-called targeted therapies (as opposed to true
cytotoxic drugs), there is increasing discussion about whether overall survival is the only or best
endpoint. Other options include Disease-free or progression-free survival, tumor size or patient
benefit or even protection against cancer vs treatment.

New FDA Clinical Trials Model. Phase 0: Microdosing with non-therapeutic doses for
pharmacokinetic and pharmacodynamic data.

Vaccines. The recently approved HPV vaccine for cervical cancer has stimulated lots of interest
in other vaccines.

VII. Classification of Cancer Chemotherapeutic Agents

Alkylating agents
Antimetabolites
Natural Products
Hormones not covered here.
Anti-Signaling Agents (Also called Targeted Therapies)
Biologicals
Miscellaneous

2
Schematic of Sites of Inhibition by Anticancer Agents.

Purine 6-Mercaptopurine Pyrimidine


6-Thioguanine
Synthesis Nelarabine Synthesis
Inhibits purine synthesis

Ribonucleotides

Methotrexate
Pemetrexed
Inhibits dihydrofolate Alkylating agents,
reductase Cyclophosphamide
Cross link DNA
Cytarabine Deoxyribonucleotides
Inhibits DNA polymerase

Doxorubicin
Daunorubicin
Etoposide
Inhibit topoisomerase II
Gemcitabine
Inhibits ribonucleotide
reductase
Causes chain
termination

DNA
Bleomycin
Cleaves DNA

RNA
Gleevec
Dasatinib
Inhibit growth Vinca alkaloids
factor signaling Paclitaxel
Rituximab Inhibit microtubule
Binds to CD20 Proteins
Bortezomib
Proteosome inhibitors
Arsenic Trioxide
Sulfhydryl reactive?
Thalidomide Microtubules
Tretinoin Enzymes, Growth Factors
& Receptors

CH3 Mechlorethamine
NH2

CH2CHCOOH
Melphalan
ClH2 CH2C
HN
N
P O
ClH2 CH2C Cyclophosphamide
O

CH2CH2CH2 COOH
Chlorambucil

Nitrogen mustard alkylating agents

3
Mechanism of alkylation. Nitrogen mustard forms a reactive cyclic intermediate that reacts
with 7-nitrogen of a guanine residue in DNA to create a covalent linkage. The second arm of the
mustard then cyclizes and reacts with a second guanine moiety in the same or opposite strand.

H2 H2C C Cl
C C Cl H2
H2
H3C N H3C N CH2
C C Cl +
H2 H2 C Imonium ion
H2

Nitrogen mustard O

N
HN

N
H2N N

R
O
H2C C N C C Cl
H2 H2 H2
+N Guanine in DNA
CH3
HN

N
H2N N

O
Interstrand DNA cross link by nitrogen mustard
N
HN O
H2 H2 O
+ C C
H2
N C
H2
C
+
N N
H2 N N N
HN CH3 NH
R
Guanine in DNA N
N N NH2
H2N N
R
R

1. Alkylating Agents

a. Chemistry and Types. They have at least one CH2R group. Includes:

i. Nitrogen mustards: cyclophosphamide (Cytoxan) (FDA approved: 1959),


mechlorethamine (1949).

b. Mechanism of Action. These contain reactive groups that bind covalently with
biologically important cell constituents, e.g., DNA. Usually there are two reactive
moieties and they can cross-link DNA.

c. Absorption, Distribution and Metabolism

i. Some (melphalan, chlorambucil, & cyclophosphamide) are orally active.

4
ii. The R-group of cyclophosphamide undergoes hepatic metabolism to give the
active alkylating agent, phosphoramide mustard. Cyclophosphamide itself is not
active; it is a prodrug.

d. Uses of cyclophosphamide. Acute lymphoid leukemia (ALL), acute myeloblastic


leukemia (AML), Hodgkin's disease, malignant lymphoma, multiple myeloma (MM).
Mechlorethamine: Chronic lymphoid leukemia (CLL), chronic myeloid leukemia
(CML) & Hodgkins disease. Melphalan: MM & non-Hodgkins disease. Chlorambucil:
CLL, Hodgkins disease & lymphoma.
Chemical structures of
methotrexate & folates.
2. Antimetabolites
H 2N N N
Antimetabolites are compounds that bear a structural
similarity to normally occurring substances, such as N
N CH2
vitamin, nucleoside, or amino acid. They compete with
the natural substrate for the active site on an essential R1
R2
N

enzyme and/or substitute for the natural compound in


DNA. Most antimetabolites inhibit DNA synthesis; they C
O

kill cells in S-phase. Although bone marrow depression HOOC NH


occurs with all the drugs in this group, oral and GI CH

ulceration can be dose limiting for some (e.g., H2 C


methotrexate). The antimetabolites require COOH

biotransformation to produce their cytotoxic effects;


conversion to the active form takes place intracellularily R1 R2
and is referred to as lethal synthesis. Folic acid OH H
Aminopterin NH2 H
A. Folates Methotrexate NH2 CH3
A1. Methotrexate

a. Chemistry and Mechanism of Action. Methotrexate (1953) is a folic acid analog


that competitively inhibits dihydrofolate reductase (DHFR). The inhibition of
tetrahydrofolate synthesis interrupts one carbon transfer reaction: the synthesis of
thymidylate, purine nucleotides and the amino acids glycine and methionine.
Methotrexate kills cells in S-phase.

b. Absorption, Distribution and Metabolism. It is well absorbed orally. It is a


weak acid, secreted, and eliminated primarily by the kidney. It is plasma bound
(50%). It does not penetrate the blood-brain barrier well. It has been administrated
intrathecally. Methotrexate is metabolized intracellularily to polyglutamate
derivatives; these are retained by cells better and are better inhibitors of DHFR than
the parent compound.

c. Therapeutic uses include: ALL, Non-Hodgkin's lymphoma, Burkitt's lymphoma.

d. Toxicity. The toxic effects of methotrexate are bone marrow, oral and GI mucosa.

5
A2. Pemetrexed

a. A new agent, chemically related to MTX, (2004; trade name Alimta) but acts on
multiple targets: thymidylate synthase, dihydrofolate reductase, C-1 tetrahydrofolate
synthetase and two formyltransferases. Used for mesothelioma.

NADPH + H+ NADP+ H Serine Glycine H


N N N N H N N H

H H
HN HN HN
N N N
H
O N O N O N
H R H R
R

Folic Acid/ MTX Tetrahydrofolate (THF) N5, N10-Methylene-


Methylene-THF

Mechanism of MTX Inhibition.

B. Pyrimidines
B1. Cytarabine

a. Chemistry. Cytarabine (also known as Ara-C or Cytosar-U) (1969) is 1--D-


arabinofuranosylcytosine.

b. Absorption, Distribution and Metabolism. Only poor bioavailability after oral


dose (~20%) because of cytidine deaminase in the GI and liver. Must be given iv by
continuous infusion because of short half life (~10 min). Less than 10% of the injected

6
drug is excreted unchanged. Like other antimetabolites, Ara-C must be anabolized to
Ara-CTP.

c. Mechanism of Action. The most important cause of cell death is incorporation


into DNA with subsequent inhibition of DNA polymerase. Ara-C also induces
terminal differentiation of cells.

d. Therapeutic Uses. ALL, CML & AML.


Cytarabine
NH2 e. Toxicity. Ara-C is a potent
N myelosuppressive agent capable of
O N causing acute, severe leucopenia,
thrombocytopenia and anemia with
HO
O
HO
striking megaloblastic changes. It can
HO
also cause GI toxicity, stomatitis and
Cytosine noncardiogenic pulmonary edema.
N

H2N N O
H

C. Purines:
C1. Mercaptopurine and Thioguanine

Chemical structures of endogenous and clinically used purines.

OH SH OH OH SH

N N N N
N N N N N
N

N N N N N N
H N H N H N H H2N N H
H2N
Hypoxanthine 6-Mercaptopurine Allopurinol Guanine 6-Thioguanine

a. Chemistry. 6-Mercaptopurine and 6-thioguanine (1966) are analogs of


hypoxanthine and guanine, respectively.

b. Administration, Distribution and Metabolism. Mercaptopurine and


thioguanine are routinely given orally. They are metabolized extensively and the
metabolites are excreted in the urine; 50% of the administered drug is found in the
urine after 24 hr. Xanthine oxidase, which catalyzes the first step in mercaptopurine
breakdown, is inhibited by allopurinol. Allopurinol is often used with anticancer
drugs to prevent the hyperuricemia and uricosuria that follows marked cell kill. The
cytotoxic effect and toxicity of mercaptopurine is increased in the presence of
allopurinol; you must reduce the dosage of mercaptopurine by or 1/3 of the usual
dose. Thioguanine is not extensively metabolized by xanthine oxidase; it can be used
in combination with allopurinol without the dosage being reduced.
7
c. Mechanism of Action. Both of these drugs must be converted within cells to the
nucleotide forms by hypoxanthine-guanine phosphoribosyl transferase (HGPRT).
Cytotoxicity occurs by several mechanisms including feedback inhibition of the first
and rate-limiting step in purine biosynthesis and inhibition of purine
interconversions. In addition, the purine analogs have effects on DNA and RNA
synthesis, glycoprotein synthesis and chromosomal replication.

d. Toxicity. Myelosuppression is a common side effect with both purine analogs.


Hepatotoxicity is less common with 6-thioguanine than with 6-mercaptopurine.
Approximately 1/3 of patients develop jaundice after 6-mercaptopurine; this is
associated with bile stasis and hepatic necrosis.

C2. Nelarabine

a. Chemistry. Nelarabine (also known as compound 506U78) is a water-soluble


nucleoside prodrug of deoxyguanosine analogue 9-beta-D-arabinofuranosylguanine
(ara-G), which was approved by the FDA in October 2005 for the treatment of
patients with T-cell acute lymphoblastic leukemia.

b. Administration, Distribution and Metabolism. Given iv. Primary route of


metabolism is O-demethylation by adenosine deaminase to form ara-G.

c. Mechanism of Action. Nelarabine is a prodrug of the ara-G. Nelarabine is

Chemical structures of endogenous and clinically used purines.

OH SH OH OH SH

N N N N
N N N N N
N

N N N N N N
H N H N H N H H2N N H
H2N
Hypoxanthine 6-Mercaptopurine Allopurinol Guanine 6-Thioguanine

demethylated by adenosine deaminase to ara-G, mono-phosphorylated by


deoxyguanosine kinase and deoxycytidine kinase, and then converted to the active 5' -
triphosphate, ara-GTP. Accumulation of ara-GTP in leukemic blasts allows for its
incorporation into deoxyribonucleic acid (DNA), leading to inhibition of DNA
synthesis and cell death. There may also be other mechanisms that contribute to the
cytotoxic and systemic toxicity of nelarabine .

d. Toxicity. Severe neurologic events have been reported with the use of nelarabine.
These events have included altered mental states including severe somnolence,
central nervous system effects including convulsions, and peripheral neuropathy
ranging from numbness and paresthesias to motor weakness and paralysis.

8
3. Natural Products

Unlike the other classes of antitumor agents, membership in this group is determined by the
source of the drug. They are obtained from lower organisms, such as plants and Streptomyces.
With the exception of the vincas and bleomycin the natural products are phase nonspecific. The
chemical structures are often complex and, therefore, not always presented. Cells with elevated
levels of p-glycoprotein are often more resistant to Natural Products.

A. Anthracyclines

a. Chemistry. There are 2 anthracyclines: doxorubicin (Adriamycin) (1974) &


daunorubicin.

b. Absorption, Distribution and Metabolism. Both doxorubicin and daunorubicin


are rapidly taken up by all tissues except the brain. They are extensively bound to
cellular components, which is responsible partially for their long plasma half-life. They
are cleared mainly by hepatic metabolism; severe clinical toxicity may result if there is
impaired hepatic function.

c. Mechanism of Action. Inhibition of topoisomerase II enzyme catalyzes the breaking


and rejoining of DNA strands. Stabilization of the enzyme-DNA complex leads to
inhibition of DNA synthesis and cell death. Both compounds are affected by p-
glycoprotein.

d. Therapeutic Uses. Doxorubicin: ALL, AML, Hodgkin's disease, malignant


lymphoma, & MM. Daunorubicin: AML & ALL.

e. Toxicity. Like other anticancer drugs, the anthracyclines cause bone marrow
depression. They also almost always cause alopecia. The most serious toxicity
(irreversible) is cardiomyopathy. There are two types: acute and chronic. The acute
form is characterized by abnormal ECG changes; it is reversible and is not a predictor of
chronic cardiomyopathy. The second form is a chronic cumulative does-related
toxicity, manifested by congestive heart failure that is unresponsive to digitalis. For
this reason the total dose of doxorubicin is kept below 500 mg/sq. m. The mechanism
of cardiac toxicity is unknown but may be due to the excessive local production of free
radicals in the myocardium.

B. Bleomycin

a. Chemistry. Bleomycin (1973) is a mixture of several different complex glycopeptides


extracted from Streptomyces.

b. Absorption, Distribution and Metabolism. Bleomycin is a peptide and cannot be


administered orally. It is primarily administered i.v.; some evidence for better
therapeutic effect if given by continuous infusion (some schedule dependency).
Bleomycin does not get into the CNS; it is very polar. More than of drug is excreted
in the urine unchanged within 24 hr. Bleomycin is inactivated in most tissues by

9
bleomycin hydrolase; the sensitivity of lungs and skin to the toxicity of bleomycin is
believed to be due to low levels of bleomycin hydrolase.

c. Mechanism of Action. Bleomycin forms a complex with iron and molecular oxygen
and then binds to double strand DNA and causes both single and double strand breaks;
the double strand breaks are believed to cause cell death. The DNA damage is
nucleotide-sequence specific. For most cells the G2 phase is the most sensitive. Unlike
most other Natural Products, p-glycoprotein does not affect bleomycin.

d. Therapeutic Uses. Non-Hodgkins and Hodgkins lymphomas.

e. Toxicity. The most common toxic effects are skin reactions including
hyperpigmentation. The most serious adverse effect is pulmonary toxicity. This
manifestation begins with decreasing pulmonary function, fine rales and cough; and it
progresses to severe and sometimes fatal pulmonary fibrosis. An unusual toxicity is a
lethal anaphylactic reaction that occurs in patients with lymphoma; small test doses are
commonly given. It is important that bleomycin does not have significant
myelosuppressive effects and, thus, it is a popular agent for combination
chemotherapy.

C. Vinca Alkaloids

a. Chemistry. The two members of this group are: vincristine and vinblastine (1965);
both are derived from the periwinkle plant. They have complex chemical structures.

b. Absorption, Distribution and Metabolism. Both agents are given i.v. and they
bind extensively to tissue components; therefore long half-life. They do not pass the
blood-brain barrier. Both are metabolized by the liver and excreted in the bile,
although vinblastine is cleared 7 x more slowly than vincristine.

c. Mechanism of Action. Both vincristine and vinblastine bind specifically to the


microtubule proteins and blocks polymerization. The dissolution of the mititotic
spindle interferes with chromosome segregation and leads to mitotic arrest. Both drugs
are affected by p-glycoprotein.

d. Uses. Vinblastine: Hodgkins disease and lymphoma. Vincristine: Non-Hodgkins and


Hodgkins leukemia.

e. Toxicities. Primarily leukopenia and neurological , especially with vincristine.

D. Etoposide

a. Chemistry. Etoposide (1996) and teniposide are analogs of the alkaloid,


podophyllotoxin, which is obtained from the Mandrake (may-apple) plant (used by
American Indians and early colonists).

b. Absorption, Distribution and Metabolism. Etoposide is available for iv use or


oral administration, although absorption is variable and ranges ~50%. After iv injection
the terminal half-life is 6-8 h. Approximately 40% of the drug is excreted in the urine
10
unchanged so in patients with compromised renal function the dose must be reduced.

c. Mechanism of Action. Both etoposide and teniposide form a ternary complex with
topoisomerase II and DNA, preventing resealing of the topoisomerase II-induced DNA
break. The enzyme remains bound to the free end of the broken DNA strand, leading to
an accumulation of DNA strand breaks and cell death. Thus, they poison the enzyme.
Cell in S and G2 are most sensitive. Both drugs are affected by p-glycoprotein.

d. Uses. Etoposide: Acute nonlymphocytic leukemia and Non-Hodgkins lymphoma;


Teniposide: ALL and Non-Hodgkins lymphoma.

e. Toxicities. Leukopenia, thrombocytopenia, and alopecia.

11
Hematology 2009

Targeted Therapies
January 2009

4. Anti-signaling agents

A. Imatinib mesylate (Gleevec or Glivec )

a. Chemistry. Imatinib (2001) was one of the first anticancer drugs discovered with high
throughput screening and is the prototype of a modern targeted small molecule. It is
considered an analog of ATP.

b. Absorption, Distribution and Metabolism. Imatinib is well absorbed after oral


administration with Cmax achieved within 2-4 hours post-dose. Mean absolute
bioavailability for the capsule formulation is 98%. the elimination half-lives of imatinib
and its major active metabolite, the N-desmethyl derivative, were approximately 18 and
40 hours, respectively. CYP3A4 is the major enzyme responsible for metabolism of
imatinib.

c. Mechanism of Action. Imatinib is a protein-tyrosine kinase inhibitor that inhibits the


Bcr-Abl tyrosine kinase, the constitutive abnormal tyrosine kinase created by the
Philadelphia chromosome abnormality in chronic myeloid leukemia (CML). Imatinib is
also an inhibitor of the receptor tyrosine kinases for platelet-derived growth factor (PDGF)
and stem cell factor (SCF), c-kit, and inhibits PDGF- and SCF-mediated cellular events. In
vitro, imatinib inhibits proliferation and induces apoptosis in gastrointestinal stromal
tumor (GIST) cells, which express an activating c-kit mutation.

d. Therapeutic Uses. CML and GIST.

e. Toxicities. Nausea, vomiting, edema and muscle cramps.

f. Resistance. Acquire resistance to imatinib predominately results form mutations in the


kinase domain that prevent drug inhibition, although some patients have amplification of
BCR-ABL.

B. Dasatinib (Sprycel)

a. Chemistry. Dasatinib is another analog of ATP, which was approved by the FDA in June
2006 for treatment of patients with CML and Philadelphia-chromosome-positive ALL
with resistance or intolerance to prior therapy (second-line therapy).

b. Absorption, Distribution and Metabolism. Dasatinib is orally active with good


distribution. It is highly protein bound (96%) and primarily metabolized in the liver by
CYP3A4.
c. Mechanism of Action. Dasatinib is an inhibitor of several kinases including BCR-ABL,
SRC family kinases, c-KIT and PDGFR..

d. Toxicity. Severe thrombocytopenia, neutropenia, and anemia.

C. Bortezomib (Velcade)

a. Chemistry. Bortezomib (2003) is an unique boronic acid containing small molecule.

b. Absorption, Distribution and Metabolism. Bortezomib is administered iv. It has a


half-life of 6 h and is metabolized with the loss of the broronic acid by CYP3A4 and
CYP2D6.

c. Mechanism of Action. Bortezomib binds to the 20S core of the 26S proteosome and is
a reversible inhibitor of its chymotrypsin-like activity. Inhibition of the proteosome
blocks multiple signaling cascades most importantly it decreases NK-B. This occurs
because Bortezomib inhibits the degradation of IB so NF-B activities are blocked.

d. Toxicity. Thrombocytopenia & peripheral neuropathy.

5. Biologics

A. Rituximab (Rituxan)

a. Chemistry. Rituximab is a genetically engineered, chimeric monoclonal antibody (the


first monoclonal antibody to be approved by the FDA for the treatment of cancer (1997)
directed against the CD20 antigen on the surface of normal and malignant B lymphocytes.
CD20 participates in the activation process for cell cycle initiation and differentiation. The
CD20 antigen is found on cells from the pre-B-cell stage through terminal differentiation
to plasma cells ad is expressed on 90% of B-cell neoplasm. The biological functions of
CD20 are unknown.

b. Absorption, Distribution and Metabolism. Because it is an antibody, Rituximab


must be given iv. It has a serum half-life of ~75 h with a rather large range in values.

c. Mechanism of Action. Monoclonal antibody binding to CD20 generates


transmembrane signals that produce autophosphorylation and activation of
serine/tyrosine protein kinases. This leads to antibody dependent cellular cytotoxicity and
cells death.

d. Toxicity. Rituximab toxicities are mostly related to infusion reactions.

B. Gemtuzumab (MytotargTM)

a. Chemistry. Gemtuzumab is a humanized monoclonal antibody against CD33 that is


covalently linked to a semisynthetic derivative of the small molecule calicheamicin. As
such it is among the first hybrid large and small molecule. Approved by the FDA March
2000 for CD33 positive AML.
2
b. Absorption, Distribution and Metabolism. Because it is an antibody, Gemtuzumab
must be given iv in two doses. Serum half-life of ~45-65 h; administer second dose after 14
days.

c. Mechanism of Action. Gemtuzumab ozogamicin is cytotoxic to the CD33 positive HL-


60 human leukemia cell line. Gemtuzumab ozogamicin binds to the CD33 antigen
expressed by hematopoietic cells which results in formation of a complex that is
internalized. Once internalized, the calicheamicin derivative is released inside the
lysosomes of the myeloid cell and binds to DNA which results in DNA double strand
breaks and cell death. This results in significant inhibition of colony formation in cultures
of adult leukemic bone marrow cells.

d. Toxicity. Severe myelosuppression, hypersensitivity reactions (including anaphylaxis),


hepatotoxicity.

6. Miscellaneous

A. Thalidomide.

a. Chemical structure. An old drug (developed in the 1950s as a sedative) with a very
simple structure.
O

b. Absorption, Distribution, and Metabolism. Slow


and highly variable oral absorption. Wide tissue N O
distribution, including in semen. Rapid spontaneous
NH
nonenzymatic hydrolysis leads to >50 metabolites.
O O

c. Mechanism of action. Complex including direct Thalidomide


apoptosis, reduced tumor adhesion to matrix,
inhibition of angiogenesis, inhibition of cytokine release, and enhanced natural killer cell
functionality

d. Toxicity. Most common side effects are sedation and constipation. Peripheral sensory
neuropathy can be serious. Is a notorious teratogen causing dysmyleia (stunted limb
growth).

3
Increase NK functionality and production

Anti-apoptosis
Cell growth arrest

Inhibition of adhesion

Inhibition of
cytokines

Decreased angiogenesis

Proposed mechanism(s) of action for thalidomide.

B. Lenalidomide (Revlimid)

e. Chemistry. An analog of thalidomide approved by FDA for Myelodysplastic Syndrome


(deletion of 5q) in December 2006. Now approved for MM.

f. Absorption, Distribution, and Metabolism. Good and rapid oral absorption. Wide
tissue distribution with primarily renal elimination.

g. Mechanism of action. Probably like thalidomide but still under study.

h. Toxicity. Similar to thalidomide but associated with neutropenia and thrombocytopenia.

C. Arsenic Trioxide

a. Chemistry. Arsenic was used >2,400 years ago in Greece and Rome. Arsenic trioxide
was used a century ago for syphilis and parasitic diseases. It was FDA approved in 2000.

b. Absorption, Distribution and Metabolism. It is administered as a 2 h iv infusion.


The primary mechanism is through enzymatic methylation but no dose reduction is
recommended in patients with hepatic or renal dysfunction.

4
c. Mechanism of Action. The mechanism of action remains uncertain. It causes cellular
differentiation and promotes apoptosis. It is highly reactive to sulfhydryls and it generates
this may be important

d. Toxicity. Well tolerated with sine reversible hyperglycemia and fatigue. Can length QT
interval.

e. Use. CML

D. Tretinoin

a. Chemistry. All-trans retinoic acid induces a high rate of complete remissions in acute
promyelocytic leukemia patients as a single agent and can be used in combination with
anthracyclines.

b. Absorption, Distribution and Metabolism. Orally active. Metabolized primarily in


the liver by CYP.

c. Mechanism of Action. The precise mechanism of action has not been established but it
is not a cytolytic agent. Tretionoin induces cytodifferentiation and decreases proliferation
of APL cells. In patients who achieve complete remission, tretinoin therapy results in an
initial maturation of the primitive promyelocytes derived from the leukemic clone,
followed by a repopulation of the bone marrow and peripheral blood with normal,
polyclonal hematopoietic cells.

d. Toxicity. Mild such as dry skin, bone tenderness and hyperlipidemia.

e. Use. APL.

5
TOPIC: CASE CONFERENCE II- COAGULATION
(SMALL GROUP)

DATE: January 12, 2009

TIME: 8:00-10:00 AM

Student Groups Small Group Rooms Facilitators

Group 65&66 Rooms 502 & 503 Patricia Kropf , MD

Group 67&68 Rooms 504 & 505 Margaret Ragni, MD, MPH

Groups 69&70 Rooms 506 & 507 Frank Bontempo, MD

Groups 71&72 Rooms 508 & 509 Joseph Kiss, MD

Groups 73&74 Rooms 510 & 511 Darrell Triulzi, MD

Groups 75&76 Rooms 512 & 513 G. David Roodman, MD, PhD

Groups 77&78 Rooms 514 & 515 Roy Smith, MD

Groups 79&80 Rooms 516 & 517 Bruce Hough, MD


COAGULATION CASES

CASE #1

A 6-year-old boy was referred to an ENT specialist for evaluation for tonsillectomy. He had
experienced recurrent upper respiratory infections, and his huge tonsils interfered with breathing and
airway patency at night. He had been placed on antibiotics intermittently over the last 3 months.
Screening coagulation tests revealed a prolonged APTT, normal PT, and prolonged closure times.
The coagulation service was asked to see the patient.
There had been no family history of bleeding, no previous epistaxis, excessive bruising, and
no prior surgeries. There was no excess bleeding with loss of deciduous teeth. The examination was
unremarkable. The patient was taking a semi-synthetic penicillin, as he had been intermittently over
the last several months.

Laboratory:

Hemoglobin 11.6 gm% (nl: 11.5-14.0)

WBC 4,500/l (nl diff) (nl: 4,000-10,000)

Platelets 163,000/l (nl: > 150,000)

APTT 39.8 seconds (nl: 22.0-34.0)

PT 12.5 seconds (nl: 10.5-14.0)

Closure times

CEPI 180 seconds (nl: 84-175)

CADP 120 seconds (nl: 65-117)


A coagulation profile was obtained.

COAGULATION PROFILE

PT 12.3 seconds (nl: 10.5 - 14.0)

APTT 41.0 seconds (nl: 22.0 - 34.0)

APTT Mix 40.0 seconds (nl: 22.0 - 34.0)

Fibrinogen 340 mg/dl (nl: 150 - 350)

Factor II 0.82 U/ml (nl: 0.60 - 1.40)

Factor V 0.78 U/ml (nl: 0.50 - 1.50)

Factor VII 0.79 U/ml (nl: 0.60 - 1.60)

Factor VIII 0.92 U/ml (nl: 0.50 - 1.50)

Factor IX 0.99 U/ml (nl: 0.65 - 1.35)

Factor X 0.84 U/ml (nl: 0.70 - 1.50)

Factor XI 0.66 U/ml (nl: 0.65 - 1.40)

Factor XII 0.65 U/ml (nl: 0.50 - 1.70)

Hexagonal Lipid Positive for (nl: negative)


Test phospholipid

Questions for discussion:

Question # 1: What is your differential diagnosis of the prolonged APTT ?

Question # 2: What is the probable cause of the prolonged closure time ?

Question # 3: What recommendations should be given to the surgeon ?


CASE # 2

A 56-year-old business executive was seen by his primary care physician for fatigue and easy
bruising. He had been well until the previous month when he noted blood-stained bowel
movements. The PT was prolonged at 16 seconds. The history was significant for heavy
consumption of alcohol (several shots/mixed drinks and beer daily)for the past thirty years. In
addition, the patient reported a gradual increase in abdominal girth over the past year. He denied
blackout spells, delirium tremens, alcohol withdrawal seizures, hepatitis, or significant time lost
from work due to illness. The only medication he took was Alka-Seltzer for 'indigestion.'

Physical examination was remarkable for the presence of spider angiomas on the face, back,
and upper arms. Gynecomastia and palmar erythema were present. The liver had a rounded edge
and the spleen tip was palpable. Rectal examination revealed prominent, distended hemorrhoidal
veins. A stool specimen was guaiac-positive. The remainder of the exam was unremarkable.

Laboratory:

Hemoglobin 10.3 gm% (nl: 13.2 - 17.1)

Hematocrit 29.3% (nl: 38.5 - 50.0)

WBC 5,400/l (nl: 3,800 10,800)

Platelets 100,000/l (nl: 150,000 400,000)

Peripheral smear: Macrocytic red cells, target cells, and hypochromic,


microcytic red cells. Platelets were slightly decreased.
A coagulation profile was obtained.

COAGULATION PROFILE

PT 15.0 seconds (nl: 10.5 - 14.0)

APTT 36.0 seconds (nl: 22.0 - 34.0)

APTT Mix 31.0 seconds (nl: 22.0 - 34.0)

Thrombin Time 26.0 seconds (nl: 13.0 - 18.0)


Reptilase Time 23.0 seconds (nl: 13.0 - 18.0)
Fibrinogen 220 mg/dl (nl: 150-350)
Factor II 0.46 U/ml (nl: 0.60 - 1.40)
Factor V 0.41 U/ml (nl: 0.50 - 1.50)
Factor VII 0.40 U/ml (nl: 0.60 - 1.60)
Factor VIII 2.60 U/ml (nl: 0.50 - 1.50)
Factor IX 0.38 U/ml (nl: 0.65 - 1.35)
Factor X 0.35 U/ml (nl: 0.70 - 1.50)
Factor XI 0.89 U/ml (nl: 0.65 - 1.40)
Factor XII 0.92 U/ml (nl: 0.50 - 1.70)
Closure Time
CEPI 190 seconds (nl: 84-175)
CADP 130 seconds (nl: 65-117)

Platelet Aggregation Tests:

ADP 20 M 50% (nl: 60-100)


ADP 10 M 50% (nl: 60-100)
ADP 5 M 45% (nl: 50-100)

Collagen 25% (nl: 60-100)


Risto 1.5 mg 60% (nl: 60-100)
Arach Acid 10% (nl: 60-100)

Questions for discussion:

Question # 1: What are the causes of the patients elevated PT and closure
time?

Question # 2: What is the function of vitamin K ?


CASE #3

A 72 year-old man was admitted to same-day-surgery for a transurethral resection of the


prostate (TURP). He had been in good health until six years prior to admission when he began to
notice hesitancy, urgency, and post-micturition dribbling. Progression of these symptoms led him to
seek medical attention. Preoperatively, the coagulation studies, PT, APTT, and the CBC and platelet
count were normal. A preoperative urine specimen showed pyuria and bacteriuria.

The operative procedure was uneventful. The patient was discharged the fourth post-
operative day on Percocet for analgesia and Keflex for the urinary tract infection. Six days later,
he experienced fever to 38.5o C and presented to his primary care physician with dysuria and flank
pain. A urine specimen revealed too numerous to count white cells, and a blood culture revealed
numerous gram-negative rods. He was admitted to his local hospital where intravenous antibiotics
were started. Within one hour of admission, the patient experienced a shaking chill and became
tachycardic and hypotensive. The blood pressure was 80/50, and the temperature rose to 40.1 C.
Repeat blood and urine cultures were obtained, and the patient was treated with vasopressors, fluids,
and glucocorticoids. Antibiotic therapy was changed to Zosyn and Cefipime. Twelve hours
after the acute event the patient passed a melanotic bowel movement and began to ooze blood from
recent venepuncture sites.

A repeat CBC showed : Hemoglobin = 9.5 gm%


WBC = 22,000/l with left shift
Platelets = 14,000/l

Peripheral smear: normochromic, normocytic red cells, many schistocytes, decreased


platelets, increased WBCs, with many bands and metamyelocytes.
A coagulation profile was obtained.

COAGULATION PROFILE

PT 18.4 seconds (nl: 10.5 - 14.0)

APTT 39.1 seconds (nl: 22.0 - 34.0)

APTT Mix 33.9 seconds (nl: 22.0 - 34.0)

Thrombin Time 24.0 seconds (nl: 13.0 - 18.0)

Reptilase Time 20.0 seconds (nl: 13.0 - 18.0)

Fibrinogen 95 mg/dl (nl: 150 - 350)

Factor II 0.33 U/ml (nl: 0.60 - 1.40)


Factor V 0.34 U/ml (nl: 0.50 - 1.50)
Factor VII 0.37 U/ml (nl: 0.60 - 1.60)
Factor VIII 1.80 U/ml (nl: 0.50 - 1.50)
Factor IX 0.28 U/ml (nl: 0.65 - 1.35)
Factor X 0.39 U/ml (nl: 0.70 - 1.50)
Factor XI 0.75 U/ml (nl: 0.65 - 1.40)
Factor XII 0.65 U/ml (nl: 0.50 - 1.70)
Fibrin Monomer ++++ (nl: 0)
Fibrin Split Products 40.0 g/ml (nl: 0.0 - 2.5)

Platelet Count 21 x 103/l (nl: 150-450)

Closure Time
CEPI 180 seconds (nl: 84-175)

CADP 120 seconds (nl: 65-117)

Questions for discussion:

Question # 1:What is the differential diagnosis of the patients


thrombocytopenia ?

Question # 2: What is the appropriate treatment ?


Case # 4

A 52 year-old man was admitted for elective hip replacement. He had been in good health with no
other significant medical problems. Preoperatively, the coagulation studies, PT, APTT, and the CBC
and platelet count were normal (Platelets = 250,000/l). The operative procedure was uneventful.
He received sc heparin for DVT prophylaxis. On day 7 after his operation his platelet count
decreased to 100,000 /l. Your differential diagnosis include heparin induced thrombocytopenia
(HIT).

Questions for discussion:

Question # 1: Should treatment be delayed until laboratory


confirmation of HIT ?

Question # 2: What is the treatment of HIT and HIT associated with


thrombosis ?

Question # 3: What are the complications associated with HIT?


Comments CASE #1

The differential diagnosis of a prolonged APTT includes blocking inhibitor (or lupus
anticoagulant, LAC) versus von Willebrand disease versus congenital coagulation deficiency of a
factor in the intrinsic pathway, e.g FVIII or IX or XI or XII. The lack of previous history of
bleeding would go against a coagulation factor deficiency, although those with mild congenital
deficiency might have bleeding only with trauma or surgery, and are often diagnosed by coagulation
screening prior to surgery. Diagnosis of a congenital coagulation deficiency of the APTT system
would require the presence of a prolonged APTT which corrects in a mix. For example, a patient
with FXI deficiency might have an APTT of 43 seconds, which corrects in a 1:1 mix with normal
plasma to 26 seconds, and a FXI level of 0.03 U/ml. (The normal APTT is 22 seconds to 34 seconds,
and the usual range for factors is 50% to 150% or 0.50 U/ml to 1.50 U/ml).

A prolonged APTT associated with a prolonged closure time strongly suggest the possibility
of von Willebrand disease. Such individuals are usually heterozygous and have bleeding primarily
with trauma or surgery. However, the APTT mix should be normal. A prolonged APTT which fails
to correct in a 1:1 mix with normal plasma is diagnostic of a blocking inhibitor or lupus
anticoagulant (LAC). That is what our patient has. The prolonged closure time in this patient could
indicate antibiotic-related platelet dysfunction, such as with penicillin, which persists until the
antibiotic is stopped. Aspirin and ibuprofen also inhibit cyclooxygenase. A history should help
determine if drugs are the cause. Aspirin should be stopped at least 3 weeks before surgery, and
ibuprofen, at least 48 hours before surgery to avoid bleeding.

We should inform the ENT surgeon that the patient may safely undergo surgery without
bleeding risk, as long as he stops the offending drug causing the prolonged closure time. There is no
need to give blood products prophylactically before or after surgery. Lupus anticoagulants (LAC) or
blocking inhibitors (as they are interchangeably called) do not result in clinical bleeding, but rather
may be associated with thrombosis. In fact, if a patient identified with a lupus anticoagulant gives a
history of past thrombosis, anticoagulation should be considered perioperatively. The clinical
situations in which blocking inhibitors occur are many: these include the use of antibiotics (e.g.
penicillin in children), anti-arrhythmic and antipsychotic drugs, recurrent pregnancy loss in women,
and strokes or related neurologic conditions in young individuals. In up to half of patients with
lupus anticoagulants, no cause is found.

The coagulation profile reveals:

1. Prolonged APTT and mix...............................................................Inhibitor


2. Normal coagulation factorsBlocking inhibitor
3. Positive Hexagonal lipid test..............................Sensitive test confirming inhibitor

Comments CASE #2
The history and physical examination reveal findings of liver disease. The peripheral blood
smear reveals a dimorphic population of red cells, both with macrocytes and target cells, typical of
liver disease, and with hypochromic microcytes typical of blood loss anemia. The prolongation of
the PT and the moderate thrombocytopenia would also suggest the possibility of liver disease.
Vitamin K deficiency is also part of the differential diagnosis of a prolonged PT, and could be
superimposed on underlying liver disease, especially if the patient has not been eating or has
diarrhea.

The use of aspirin [here Alka-Seltzer] and aspirin-containing drugs is ubiquitous and
knowledge of over-the-counter and prescription drugs is important. Platelet functional defects are
typical of ASA-containing drugs, and, in patients with an underlying coagulopathy, such as liver
disease, as in this case, abnormal platelet function may contribute to the overall bleeding tendency in
such patients. Closure tine is used to evaluate platelet function.

The coagulation profile reveals:

1. Prolonged closure time............................................................................ASA effect


2. Prolonged thrombin time, reptilase time.............................................Dysfibrinogen
3. Prolonged PT...........................................................................Extrinsic F deficiencies
4. Prolonged APTT.......................................................................Intrinsic F deficiencies
5. Low platelet count.............................................................Hypersplenism, portal HBP
6. Low factors.............................................................................Hepatic synthetic defect
7. Decreased plat aggregation tests..ASA effect

Vitamin K is necessary for postribosomal carboxylation of the terminal glutamic acid residues of all
vitamin K dependent blood coagulation factors X,IX,VII, II and the two coagulation factor inhibitors
(C and S). This carboxylation allows the binding of calcium, which is needed for the expression of
the phospholipids binding sites and activation of the vitamin K dependent proteins.
Comments CASE #3

The occurrence of sepsis in a previously healthy person may result in both Vitamin K
deficiency secondary to antibiotics, and disseminated intravascular coagulation.

The coagulation profile reveals:

1. Prolonged closure time.......................................................due to thrombocytopenia


2. Prolonged PT..............................................................................due to decreased FVII
3. Prolonged APTT.....................................................due to factor consumption with DIC
4. Prolonged TT, RT................................................................due to hypofibrinogenemia
5. Elevated FVIII..........................................................nonspecific acute-phase-reactant
6. Decreased FII,VII,IX,X.............................................Vit. K deficiency due to antibiotics
7. Decreased FI,FV...................................................... due to factor consumption in DIC
8. Fibrin monomer, split products............................. due to excess thrombin action on fibrin

DIC: Fibrin monomers, split products, thrombocytopenia hypofibrinogenemia, AT-III deficiency


abnormal smear (burr cells)

TREATMENT

1. Treat the underlying cause of DIC: in this case, sepsis.


2. Use blood products in patients who are bleeding:
A. Cryoprecipitate as a fibrinogen source.
B. Fresh frozen plasma (FFP) as a factor source.
C. Platelets for thrombocytopenia.

DIFFERENTIAL DIAGNOSIS

1. Pseudothrombocytopenia (ex vivo clumping)


2. Decreased production of platelets
Acute alcoholism, thiazide diuretics, diethylstilbestrol, cocaine, HIV
3. Increased destruction or clearance of platelets
Sepsis, acute viral infection
Drugs (abciximab, acetaminophen, amiodarone, amphotericin B, cimetidine, danazol,
diclofenac, digoxin, heparin, quinidine, quinine, rifampin, sulfasalazine, trimethoprim-
sulfamethoxazole, vancomycin, many others)
Disseminated intravascular coagulation (DIC), idiopathic thrombocytopenic purpura (ITP),
HIV infection, cardiopulmonary bypass, antiphospholipid syndrome
4. Splenic sequestration
5. Pregnancy associated
Comments CASE #4

Half of the patients who develop HIT will develop thrombosis (HITTS) within 30 days, unless
they are treated appropriately with non-heparin anticoagulants.

Clinical presentation of HIT and other sequelae of HIT


Deep vein thrombosis (50%)
Pulmonary embolism (25%)
Skin lesions at injection site (10% - 20%)
Acute limb ischemia (5% - 10%)
Acute thrombotic stroke or MI (3% - 5%)
Acute systemic reactions following IV heparin bolus ( 25% of at risks
patients)
Adrenal hemorrhagic infraction (rare)
Cerebral dural sinus thrombosis (rare)

An appropriate alternative anticoagulant to heparin or LMWH should not be delayed for results
of HIT antibody testing in patients strongly suspected of having HIT

Any dose of heparin can produce HIT, even the heparin on coated intra-vascular devices

LMWH is less likely than UFH to cause HIT, but LMWH must not be substituted for UFH in a
patient who already has HIT
Additional info

CLOSURE TIME

Test of platelet function


In vitro test with 80-90% sensitivity, 94% specificity
Simple, rapid
Measures primary hemostasis
Platelet adhesion
Platelet aggregation
Simulates blood vessel injury in capillary tube

METHOD:
CAPILLARY TUBES COATED WITH BIOACTIVE MEMBRANE COATED
WITH COLLAGEN + EPINEPHRINE OR + ADENOSINE DIPHOSPHATE

----------------------------------------------------------- C-EPI
----------------------------------/------------------------ C-EPI
Blood
----------------------------------------------------------- C-ADP
----------------------------------/------------------------ C-ADP

1. Blood is drawn into blue top (citrated tube).


2. Blood is aspirated under constant vacuum into capillary tube coated with
collagen and epinephrine or ADP, simulating Shear Stress.
3. Platelets attach (Adhesion) at microscopic cut in membrane, become activated,
and stick to each other (Aggregation).
4. Platelet plug forms.
5. Time to obtain full occlusion of aperture is the Closure Time.

----------------------------------------------------------- C-EPI
::::::::
------------------------------::::/:::--------------------- C-EPI
Blood
----------------------------------------------------------- C-ADP
::::::::
------------------------------::::/:::--------------------- C-ADP

Normal Range: CEPI 84-175 seconds


CADP 65-117 seconds

Limitation: Not interpretable when platelets <100,000/l or hematocrit low


Test Abnormal in: uremia, cirrhosis, vWD, ASA, Bernard Soulier syndrome,
Glansmans thrombasthenia
PROTHROMBIN TIME (PT)

Patient Plasma

37C

Tissue Thromboplastin (TF) -CaCl2

Time for clot to form

Principle: Tissue thromboplastin is a lipoprotein found in many mammalian


tissues. In the presence of calcium ions, it is capable of activating
the extrinsic pathway (FVII).

Methodology: Photometric (optical density or light scatter).

Use: Monitoring oral anticoagulant therapy.


Detection of vitamin K deficiency or deficiency of factors II,
VII, IX, and X.

Limitations: 1. Overcitrated specimens (underfilled tubes or high


hematocrit) can prolong clotting time due to CaCl2 -
citrate imbalance.
2. Unsuspected heparin contamination.
3. Very low fibrinogen (< 50 mg/dL) - instruments
cannot detect clot.
4. Extremely quick-clotting specimens clot before
optical system "turned on" - instrument reports
maximum clotting time.
5. Excessive hemolysis, icterus, or lipemia may
interfere.
PARTIAL THROMBOPLASTIN TIME, ACTIVATED (APTT)
(also called Partial Thromboplastin Time, PTT)

Patient Plasma

37C

Phospholipid reagent; surface activator (kaolin)

CaCl2 (0.025 m)

Time for clot to form

Principle: Phospholipids (lipid-rich thromboplastin) and kaolin, a


particulate activator of the contact factors, are used to activate the
clotting mechanism. The specimen is incubated with the
phospholipid and activator, then recalcified, and the clotting time
determined. Prolongation of the clotting time is indicative of
factor deficiency or inhibitor. The test is called partial
thromboplastin because it is only the lipid-rich thromboplastin
used in this assay.

Methodology: Photometric (optical density or light scatter).

Use: Monitoring heparin therapy (unfractionated).


Detection of factor deficiency (except FVII) and inhibitors.

Limitations: 1. Overcitrated specimens (underfilled tubes or high


hematocrit) can prolong clotting time due to CaCl2 -
citrate imbalance.
2. Unsuspected heparin contamination.
3. Very low fibrinogen (< 50 mg/dL) - instruments
cannot detect clot.
4. Extremely quick-clotting specimens clot before
optical system "turned on" - instrument reports
maximum clotting time.
5. Excessive hemolysis, icterus, or lipemia may
interfere.
Objectives
1. To be able to list the key differences between MGUS and
Plasma Cell Disorders multiple myeloma.

2. To be able to identify criteria for the diagnosis of multiple


myeloma.
G. David Roodman, M.D., Ph.D.
3. To know the major clinical manifestations of symptomatic
multiple myeloma.
January 2009 4. To understand the differences between primary amyloidosis
and multiple myeloma.

Key words: multiple myeloma, amyloidosis, monoclonal protein

Plasma Cell Disorders Distribution of Monoclonal Gammopathies


Mayo Clinic Experience
Multiple Myeloma n=1296
SMM,
SMM, 3.5%
3.5% (44)
(44)
Other Disorders Lymphoproliferative
Lymphoproliferative Solitary
Solitary or
or extramedullary,
extramedullary,
Monoclonal gammopathy of undetermined 2.5%
2.5% (31)
(31) 1.5%
1.5% (20)
(20)
significance (MGUS) Amyloidosis
Amyloidosis Macro,
Macro, 3%
3% (41)
(41)
Smoldering multiple myeloma (SMM) (AL)
(AL) 10%
10% (130)
(130) Other,
Other, 2.5% (34)
2.5% (34)
Solitary Plasmacytoma
Bone Myeloma
Extramedullary 15% (193)
Waldenstrms Macroglobulinemia
Primary Amyloidosis (AL) MGUS
62% (803)
Heavy chain disease
POEMS syndrome / osteosclerotic myeloma
Type I and II cryoglobulinemia
Light chain deposition disease
Kyle RA and Rajkumar SV. Cecil Textbook of Medicine, 22nd Edition, 2004

Criteria for Diagnosis of Myeloma MGUS: Natural History

1384 patients
MGUS Smoldering MM Active MM Median F/U 15.4 years
<3 g M spike 3 g M spike 10% PC
<10% PC
OR 10% PC M spike +
115 progressed to MM or related disorder
AND AND
32 progressed to SMM
No anemia, bone lesions Anemia, bone lesions,
normal calcium and high calcium or
kidney function abnormal kidney function

Kyle RA. N Engl J Med 2002; 346: 564 Kyle RA et al. N Engl J Med 2002;346:564-69

1
Risk of Progression of MGUS over time Diagnosis of Patients Who Have Progressed
60

from MGUS
50
40

50 46.0
45
percent

40

Relative Risk
30

35
progression
30 25.4
25
20

20
15
8.4 8.5
10
5 2.4 0.9
10

0
MM IgM Primary Macro- Chronic Plasma-
progression, accounting for death as a competing risk lymphoma amyloidosis globulinemia lymphocytic cytoma
leukemia
0

0 5 10 15 20 25 30 Disease
years Kyle RA. N Engl J Med. 2002;346:564
Rajkumar SV et al. Blood 2005;106:812-817

MGUS Management of MGUS


Size of monoclonal (M) protein and risk of
progression to myeloma or related malignancy
No therapy has been shown to alter the natural
M-Protein Size Risk of Progression Risk of Progression
(g/dL) at 10 years (%) at 20 years (%)
history of the disease.
0.5 6 14
Repeat studies in 6 months; if stable then yearly
1.0 7 16
thereafter.
1.5 11 25 Traditionally patients with MGUS are followed
2.0 20 41
annually; however, in low-risk MGUS less
frequent follow up can be considered.
2.5 24 49
3.0 34 64
Bisphosphonates are not indicated

Kyle RA and Rajkumar SV. Cecil Textbook of Medicine, 22 edition, 2004 Elsevier Inc.

Multiple Myeloma
Multiple Myeloma
Prevalence
50,000 Americans have multiple myeloma
Malignancy of plasma cells nd
2 most common hematologic malignancy
Hallmarks: Median age at diagnosis is 71 years
Anemia Median survival from diagnosis: 3-5 years

Renal failure 15,000 people are diagnosed with MM each year in the US
Bone destruction (lytic bone lesions) More than 11,000 American MM patients die each year
Hypercalcemia Population subgroups
Presence of monoclonal protein Incidence twice as high in African Americans
More frequent in men than women
Increased risk of infection
This disease remains incurable in most patients
Jemal A. CA Cancer J Clin 2005;55:10. Kyle RA. Mayo Clin Proc 2003;78:21
Bergsagel DE. Blood 1999;94:1174 Seer Cancer Statistics Review 1975-2001

2
Immunophenotyping Major Symptoms at Diagnosis

Myeloma Cells Bone Pain 58%


Exhibit light-chain restriction: kappa/lambda Fatigue 32%
Typically CD 138+ and CD 38+/CD45- Weight Loss 24%
Paresthesias 5%
CD20 expressed in only 15-20% of MM patients

11% are asymptomatic or have only


mild symptoms at diagnosis

Kyle RA. Mayo Clin Proc 2003;78:21


San Miguel Baillieres Clinical Haematol 1995;4:735-59

Initial Diagnostic Evaluation


Clinical Features at Presentation
Hx and physical examination
Blood work-up
Monoclonal (M) protein (93%) CBC with diff and platelet counts
BUN, creatinine
Lytic bone lesions (67%)
Electrolytes, calcium, albumin, LDH
Increased plasma cells in the bone marrow (96%) Quantitative immunoglobulins
Anemia (normochromic normocytic; 73%) Serum protein electrophoresis (SPEP) and immunofixation
Beta2-microglobulin, C-reactive protein (CRP)
Hypercalcemia (corrected calcium >11) (13%) Urine
Renal failure, serum creatinine >2.0 (19%) Bence Jones quantitation (24 hour urine)
24-hr protein electrophoresis (UPEP) and immunofixation
Infection
Other
Skeletal survey
Unilateral bone marrow evaluation with cytogenetics

Kyle RA. Mayo Clin Proc 2003;78:21-33 NCCN Practice Guidelines. v.1, 2006

Normal Serum Protein Electrophoresis Serum Protein Electrophoresis in Myeloma

Kyle RA and Rajkumar SV. Cecil Textbook of Medicine, 22nd Edition, 2004 Kyle RA and Rajkumar SV. Cecil Textbook of Medicine, 22nd Edition, 2004

3
Immunofixation to Determine Type of Monoclonal Protein

IgG kappa
M protein

Kyle RA and Rajkumar SV. Cecil Textbook of Medicine, 22nd Edition, 2004

FISH for Deletion 13 Distribution of Monoclonal


Proteins in Multiple Myeloma
M protein found in serum or urine or both at time of
diagnosis in 97% of patients (3% are non-secretory)
Serum M spike by protein electrophoresis: 80%
Abnormal serum immunofixation: 93%
Abnormal urine immunofixation: 75%
Abnormal urine or serum immunofixation: 97%

Of the 3% with non-secretory myeloma with negative


serum and urine immunofixation, 60% will have detectable
serum free light chains on the serum free light chain assay

Normal Deletion 13 Kyle RA et al. Mayo Clin Proc. 2003;78:21


International Myeloma Working Group. British Journal of Haematology. 2003;121:749
Fonseca R. Cancer Res. 2002 62: 715-720 Jacobson, Jl et al. British Journal of Haematology. 2003;122:441

Categories of Monoclonal Gammopathies


Categories of Monoclonal Gammopathies
Classification Characteristics Management
Classification Characteristics Management Indolent MM Presence of serum/urine M protein Monitoring every
MGUS Serum M protein <3 g/dL and Observation Bone marrow plasmacytosis 3 mo, with
with treatment treatment
Bone marrow plasma cells Mild anemia or few small lytic bone
beginning at
<10% if done and beginning at lesions
disease
disease Absence of symptoms
Absence of anemia, renal progression
progression
failure, hypercalcemia, and lytic Symptomatic MM Presence of serum/urine M protein Immediate
bone lesions treatment
Bone marrow plasmacytosis
Smoldering MM Serum M protein >3 g/dL and Observation, Anemia, renal failure, hypercalcemia,
Bone marrow plasma cells with treatment or lytic bone lesions
10% and beginning at Patients with primary systematic
disease
Absence of anemia, renal amyloidosis and bone marrow
progression plasma cells 30% are considered to
failure, hypercalcemia, and lytic
have both MM and amyloidosis
bone lesions

Br J Haematology 2003, 121 749-757 Br J Haematology 2003, 121 749-757

4
Durie-Salmon Staging System for Myeloma International Staging System for
Stage Criteria Myeloma cell mass
( 1012 cells/m2)
Symptomatic Myeloma
I All of the following: <0.6 (low)
Hemoglobin >10 g/dL
Serum calcium level 12 mg/dL (normal) Stage 1 2M <3.5 and
Normal bone or solitary plasmacytoma on x-
ray
ALB 3.5
Low M component production rate:
IgG <5 g/dL
IgA <3 g/dL
Bence Jones protein <4 g/24 hr Stage 2 Not Stage 1 or 3
II Not fitting stage I or III 0.61.2 (intermediate)
III One or more of the following: >1.2 (high)
Hemoglobin <8.5 g/dL
Serum calcium level >12 mg/dL
Multiple lytic bone lesions on x-ray
High M-component production rate:
IgG >7 g/dL
IgA >5 g/dL
Bence Jones protein >12 g/24 hr Stage 3 2M 5.5
Subclassification Criteria
A Normal renal function (serum creatinine level <2.0 mg/dL) 2M=serum 2 microglobulin in mg/dL; ALB=serum albumin in g/dL
B Abnormal renal function (serum creatinine level 2.0 mg/dL)

Durie B, Salmon S. Cancer. 1975;36(9):842-854; Greipp PR, et al. Blood 2005; 102: 190a

Major Adverse Prognostic Factors Del 13: Unfavorable Prognosis

Karyotypic deletion 13 or hypodiploidy Deletion of chromosome 13


is the single most powerful
High plasma cell labeling index adverse prognostic factor for
Molecular genetics: t(4;14), t(14;16) or 17p- all times to events in patients
High LDH, 2M, or CRP referred for high-dose
therapy
Increased circulating plasma cells OS 65.1 9.8 vs 26.7
Plasmablastic morphology 4.1 months
Low albumin

Facon, T et al. Blood 97:1566-1571, 2001.

2-microglobulin Challenges in Management


An elevated 2- Currently incurable in most patients
microglobulin (2.5) is an Chemotherapy response rates = 50% to 70%
adverse prognostic factor Long-term complete responses = rare
IgA subtype as well Median survival with standard therapy = 3 years
Model using 2m 2.5, IgA Stem cell transplant prolongs survival, but not
isotype associated to 2m curative
2.5 Treatment of relapse
Median survival >111 No standard therapy
mos. (0 factors) vs. 43.1
Existing options inadequate
(1) vs. 17.8 mos. (2)
New treatment options needed
MP = melphalan/prednisone

Facon, T et al. Blood 97:1566-1571, 2001. NCCN Practice Guidelines. Rajkumar SV, et al. Mayo Clin Proc. 2002;77:813-822.

5
Initial Therapy Considerations Smoldering (Asymptomatic) MM

Ensure patient does not have smoldering MM Treatment Options


(asymptomatic MM)
Approach to therapy for MM is based on whether
a patient is a transplant candidate
Consider clinical trials if available
Improving complete response rates is a key goal Observation Clinical Trial
of current trials
*Use of Thalidomide is not indicated outside of a
clinical trial in smoldering myeloma

Kyle & Greipp. N Engl J Med 1980; 302:1347-1349 Rajkumar SV et al. Leukemia. 2003;17:775-779
Hjorth et al. Eur J Haematol 1993; 50:95-102 Weber D et al. J Clin Oncol. 2003;21:16
Witzig. Br J Haematol 1994; 87:266-72

Initial Approach to Treatment of MM Melphalan/Prednisone Treatment


Has long been the standard therapy for non-transplant
candidates
Clearly not a transplant candidate
based on age, performance status
Potential transplant Alkylating agent + corticosteroid dosing
and comorbidity candidate
Dosing: 0.25 mg/kg/day melphalan for 4 days
+ 20 mg prednisone tid for 6 days every 4-6 weeks

MPT, 7 Day dosing: 4mg/m2/day Melphalan + 40mg/m2/day


Non-alkylator based
MP or clinical trial* induction x 4 cycles Prednisone each given 7 days per month
40% remission (at least 75% reduction in serum myeloma
protein, 95% reduction in Bence Jones proteinuria and
>5% marrow plasma cells)
*Thal/Dex or Dex are additional
Stem cell harvest
options especially if immediate Alexanian R et al. JAMA. 1969;208:680
response is needed Dimopoulos. NEJM. 1994;330;7;484; Kyle RA & Rajkumar SV. N Engl J Med 2004;351:1860-73.

Mortality of Combination Chemotherapy vs


Melphalan/Prednisone
Years
0 1 2 3 4 5 6+
100
90
Estimated percentage still alive

80
70
60
50
40

30 24.4%
19.4%
1.4% SD 1.4
20 23.0% (log-rank
Allocated cct (% SD) 18.0% 2P > .1; NS)
10 Allocated MP (% SD)

Deaths/person-years:
CCT 642/1999 392/1456 305/1044 196/724 133/506 255/1130
MP 576/1968 407/1423 294/983 194/652 130/444 215/839

Myeloma Trialists' Collaborative Group. J Clin Oncol. 1998;16;12:3832

6
MP versus MP plus thalidomide (MPT) in Elderly MP versus MP plus thalidomide (MPT) in Elderly
Patients With Multiple Myeloma Patients With Multiple Myeloma
GIMEMA Phase III Randomized Controlled Trial GIMEMA Phase III Randomized Controlled Trial:
Incidence of thromboembolism
Response MPT Arm % MP Arm% P Value

CR + nCR 28 7 MPT with no DVT prophylaxis (n=65)

ORR 76 48 <0.001
22% deep-vein thrombosis
Median PFS, mo 33 14 <0.001
MPT with Enoxaparin 40 mg/d for 4 months (n=64)
2 yr survival rate 82 65 0.2
3% deep-vein thrombosis

Palumbo et al. Lancet 2006;367:825-831 Palumbo et al. Lancet 2006;367:825-831

Response (RRC/IMWG)
MM003: Trial Design
PR+VGPR+CR
80 PR VGPR
Thalidomide 50200* mg d 1-28 69.4%*
CR
Response Rate (%)

Dex 40 mg, d 1-4, 9-12, 17-20 TTP


60 25.5% 51.1%*
Same except
OS * p=0.001

X 4 COURSES Dex d 1-4 RR 40 35.3%


* CR+VGPR
Continue until PD
Safety 20 35.7% *
p<0.0001

Placebo d 1-28 12.8% *


8.1% * 3.0% *
Dex 40 mg, d 1-4, 9-12, 17-20 0
Thalidomide/ Placebo/
* Thalidomide dose escalated in all patients, over 4 weeks, to improve tolerability Dex Dex
Thalidomide/Dexamethasone MM003 Thalidomide/Dexamethasone MM003

Time to Progression

HR (95% CI)
0.43 (0.32, 0.58)

Thalidomide/dex median time to progression: 22.4 months


Placebo/dex median time to progression: 6.5 months

P<0.0001

Thalidomide/Dexamethasone MM003

7
Lenalidomide Plus Standard or Low-Dose
Dexamethasone in Newly Diagnosed Myeloma
ECOG 4A03: Phase III Randomized Study
Courses repeat every 28 days up to 1 yr in the
absence of PD or unacceptable toxicity If PD within 4 mo

Arm I. Lenalidomide 25 mg/day PO, Arm III. Salvage therapy


Days 1
121
Thalidomide 200 mg/day PO,
Standard-Dose Dexamethasone Days 1
128
40 mg/day PO, Days 1
14, 9
912, 17
1720
Standard-Dose Dexamethasone
Newly (n=206) 40 mg/day PO, Days 1
14, 9
912, 17
1720
diagnosed,
untreated MM
Arm II. Lenalidomide 25 mg/day PO, Arm IV. Salvage therapy
(n=412) Days 1
121
Thalidomide 200 mg/day PO,
Low-Dose Dexamethasone Days 1
128
40 mg/day PO, Days 1, 8, 15, 22
Low-Dose Dexamethasone
(n=206)
40 mg/day PO, Days 1, 8, 15, 22

Endpoints: Response Rate, Safety in Arms I, II; Response Rate in Arms III, IV

E4A03: Serious adverse events occurring


during first 4 months of therapy Bortezomib

Toxicity
Arm A Arm B
pvalue
Potent, Ki = 0.6 nM
(N=223) (N=222)
Any non Hem toxicity (Grade Reversible inhibitor of O OH
54.3% 39.6% 0.002 H
>=3) the 26S proteasome N N B
Toxicity of Any Type (Grade N OH
19.3% 11.3% 0.025
>=4) H
O
Death (Grade 5) 4.9% 0.5% 0.006 N

Phase II Trial of Bortezomib in Previously


Initial Therapy: Bortezomib Untreated MM: Response and Adverse Events
Combinations
n=60
Study No. of Response
patients rate
Response n (%) Most Common Adverse Events n (%)
Bortezomib + Dex (Harousseau
(Harousseau et al) 30 73% CR 6 (10) Peripheral neuropathy 36 (55)
PR 17 (28) Fatigue 6 (21)
PAD (Oakervee
(Oakervee et al) 21 95% MR 15 (25) Rash 5 (17)
SD 19 (32) Nausea 3 (10)
VTD (Alexanian
(Alexanian et al) 36 92%
PD 3 (5) Constipation 3 (10)
Varicella zoster virus 3 (10)
Upper respiratory infection 2 (7)

Harousseau et al ASCO 2004; Oakervee et al. Br J Haematol 2006;129:776


Wang et al ASH 2005 Richardson P et al. Blood. 2005;106:716a [abstract 2548]

8
Autologous Stem Cell Transplantation Stem Cell Transplantation

Key issues
Mel 200/m2 standard conditioning regimen
Sufficient performance status, and adequate liver, Efficacy compared to conventional
pulmonary, cardiac function needed chemotherapy
Higher PR and CR rates than conventional Timing: early versus delayed
chemotherapy Single versus tandem
Higher overall and event-free survival than
conventional Rx Role of allogeneic and mini-allogeneic
Advanced age and impaired renal function are by transplants
themselves not contraindications. Maintenance post SCT

Attal M, et al. N Engl J Med. 1996;335:91-97. NCCN Practice Guidelines. v.1.2001.

Randomized Comparison of Conventional Transplant versus Conventional Chemotherapy


vs. High Dose Therapy
For newly diagnosed myeloma
Patients CR EFS OS
(n) (%) (median, mo) (median, mo)
Conv 100 5 18 37
Attal,
Attal, 1996 HDT 100 22 27 52
Conv 255 -- 14% at 7 yrs 38% at 7 yrs
Barlogie,
Barlogie, 2006 HDT 261 -- 17% at 7 yrs 38% at 7 yrs
Conv 91 -- 13 24
Fermand HDT 94 -- 39 64.6
Conv 83 11 34.3 66.9
Blad
Blad, 2001 HDT 81 30 42.5 67.4
Conv 200 8.5 19.6 42.3
Child, 2003 HDT 201 44 31.6 54.1

Attal M et al. N Engl J Med. 1996;33 Blad J Blood. 2001;98:815a


Barlogie, et al. J. Clin Oncol 2006, 24(6), 929-936 Child JA et al. N Engl J Med. 2003;348:1875
Fermand J et al. Blood. 1998;92:3131 Attal M. N Engl J Med 1996; 335:97; Child J. N Engl J Med 2003; 348:1875

SCT: Early Vs Delayed Single Versus Tandem Auto SCT: IFM 94 trial

Tandem SCT
399 pts Single SCT (M 140 mg/m2; )
2
(M 140 mg/m ; vs then
8Gy) (M 140 mg/m2; 8Gy)

CR/VGPR 42% 50% p=0.10


7 yr prob EFS 10% 20% p=0.03
7 yr prob OS* 21% 42% p=0.001

*Survival benefit with tandem SCT restricted to patients failing to achieve CR or


VGPR with first SCT

Fermand. Blood 1998; 92:3131 Attal et al N Engl J Med 2003; 349: 2495

9
Treatment of Bone Metastases
Treatment of Bone Metastases
Chemotherapy
Chemotherapy
Chemotherapy

Orthopaedic
Orthopaedic intervention
intervention
Orthopaedic intervention
Analgesics
Analgesics
Analgesics
Radiotherapy
Osteoclast inhibition
Radiotherapy
Radiotherapy
Osteoclastinhibition
Osteoclast inhibition

Waldenstroms Macroglobulinemia Amyloidosis


1. Bone marrow infiltration by plasmacytoid Pathogenesis
lymphocytes Maybe primary (plama cell dyscrasia)
2. High levels of IgM or secondary (chronic inflammatory
3. No signs of myeloma conditions or infection or familial
4. No bone disease fevers)
5. Lymphadenopathy or splenomegaly Primary amyloid due to deposition of
present in 40% light chains in organs
6. Serum viscosity elevated in 30% patients Other types of amyloid due to
7. Median survival 5 years deposition of fragments of serum
amyloid protein A or other proteins.

10
Amyloidosis
Treatment:

Melphalan and prednisone or PBSCT for


amyloidosis due plasma cell dyscrasia.
Cochicine for secondary amyloidosis
Treat underlying disease if possible.

Organ transplantation if indicated but may


recur in the transplanted organ. Cardiac or
renal transplant followed by PBSCT is being studied.

11
Multiple Myeloma and Bone Disease

Multiple Myeloma (MM) is a hematologic malignancy characterized by


neoplastic clonal proliferation of plasma cells typically resulting in the presence of a
monoclonal immunoglobulin (Ig) present in the blood and/or urine. MM is responsible
for 1% of malignant diseases, and comprises 10% of all hematologic malignancies,
making it the second most common hematologic malignancy in adults, second to only
Non-Hodgkin's lymphoma. The estimated annual incidence of MM is approximately 4
per 100,000 in Caucasians and 8 per 100,000 in African-Americans with a projected
19,900 new cases to be diagnosed in 2007 in the United States and approximately 10,790
deaths due to myeloma to occur [1]. The incidence is lower in the Asian population. The
incidence of MM increases with age with median age at diagnosis being 68 years, with a
greater incidence of the disease in males as compare to females.

Clinical Manifestations of Myeloma

Approximately 80% of Multiple Myeloma patients manifest bone involvement, so


that bone destruction is the hallmark of the disease. With bone being the primary site of
disease, it is no surprise that roughly 2/3 of patients experience bone pain at diagnosis
and 60% of patients suffer a pathologic fracture during the course of their disease. [2,3].
Other clinical manifestations of the disease include renal involvement, hypercalcemia,
anemia, and infections.

Bone destruction

Bone destruction in MM can cause significant clinical morbidity. A study of over


250 myeloma patients found that the areas of bone most likely to be involved include the
spine (49%), skull (35%), pelvis (34%), ribs (33%), humeri (22%), femora (13%) and
mandible (10%) [4]. The most common radiographic findings of bone involvement
include osteolysis, osteopenia, pathologic fractures, or a combination of the above.
Eighty percent of patients experience bone pain. Bone pain typically presents in the back
or chest and is exacerbated by movement and is less intense at nighttime.

Hypercalcemia

Hypercalcemia occurs in approximately 15% of myeloma patients. Causes of


hypercalcemia in myeloma include increased bone resorption, decreased bone formation,
impaired renal function, and in a minority of patients, increased levels of PTHrP. In
contrast to humoral hypercalcemia of malignancy, hypercalcemia of myeloma is more
often secondary to widespread bone involvement and renal impairment as opposed to
elevated levels of PTHrP. The level of hypercalcemia in myeloma correlates with tumor
burden and has not been found to correlate with serum PTHrP levels [5]. The uncoupling
of normal bone resorption/formation favors bone resorption in myeloma resulting in an
overall net efflux of calcium into the extracellular fluid. Signs of symptoms of
hypercalcemia include dry mouth, anorexia, renal stones, confusion, depression, nausea,
vomiting, polydypsia, and polyuria. Renal impairment in myeloma is thought to cause
hypercalcemia not only by an inability to clear the excessive calcium in the serum from
increased bone resorption, but there is also increased renal tubular calcium reabsorption.
The etiology of the increased renal tubular calcium reabsorption remains unclear, as
elevation of PTHrP is not a consistent finding among myeloma patients.

Neurologic Symptoms

The most common presenting neurologic complication in myeloma is


radiculopathy usually of the thoracic or lumbosacral regions. The pathophysiology of
this is due to expansion of the primary tumor leading to compression of the nerve or
collapse of the bone. Spinal cord compression which should be treated as an oncologic
emergency is seen in 5-10% of patients. Peripheral neuropathy occurs but is typically
associated with amyloidosis or more commonly as a side effect of therapy.

Other systemic complications

Other complications associated with myeloma include anemia, thrombocytopenia,


and leukopenia. Despite extensive bone marrow involvement in some patients with
myeloma, thrombocytopenia and leukopenia are rare, thus this is unlikely as the primary
etiology of anemia. Rather, increased production of IL-6, fas-ligand, and MIP-1 are
thought to be the primary causes of anemia. Erythropoietin levels are also probably
decreased secondary to increased cytokine production. Patients with IgA myeloma are
more likely to develop bleeding complications as well as hyperviscosity due to the
propensity for IgA immunoglobulins to associate in polymeric formations.

Diagnosis

Criteria for diagnosis of myeloma involve the classic triad of myeloma requiring
[6] > 10% abnormal plasma cells in the bone marrow plus either [7] osteolytic bone
lesions or [8] elevation of serum and/or urine M protein to > 3g/dl or > 1g/dl respectively
[9].

Evaluation of bone involvement

Evaluation of bone involvement should be done using conventional radiography


which is superior to Technetium-99m bone scanning. Nuclear medicine scanning reflects
osteoblastic activity thus underestimating the osteolytic lesions characteristic of myeloma
bone disease. If conventional radiography is inconclusive or negative in the setting of
high clinical suspicion for bone disease, CT without contrast or MRI may be used, both
are more sensitive than conventional radiography.

Traditionally, bone surveys have been used to both diagnose lytic lesions, and
monitor response to therapy of patients with myeloma bone disease. An adequate survey
includes imaging x-rays of the skull, vertebral column, pelvis, and extremities. Due to
the numerous images acquired during each bone survey, the accuracy of the interpretation
of the images can be limited. The limited reproducibility of bone surveys has led to use
of newer modalities such as CT scan without contrast, MRI, and PET scans to evaluate
the extent of myeloma one disease. In comparison trials, MRI has been shown to have
greater sensitivity than plain radiographs in detecting asymptomatic bone disease [10]. In
one study of 53 patients with multiple myeloma, 55% of patients with presumed normal
plain radiographs had evidence of diffuse and nodular bone involvement on MRI [10].
MRI is also superior to plain radiography for staging patients with MM. One study
evaluated 12 patients with presumed solitary plasmacytomas on bone survey found 4
patients to have additional evidence of marrow involvement on MRI evaluation thus
changing their diagnosis to MM. Thus MRI imaging of the head, spine, and pelvis is
recommended in all patients with a suspected diagnosis of solitary plasmacytoma to rule
out any other bone lesions.

Prognosis

While survival of multiple myeloma is improving, average length of survival even


with treatment is almost 5 years with a small percentage of patients surviving for greater
than 7 years. Several factors have been identified that contribute to the prognosis in MM,
including karytypic abnormalities in the myeloma cells, serum levels of albumin and 2
microglobulin, which reflects tumor burden in patients without renal failure.

Treatment of Myeloma

The treatment of myeloma has greatly improved over the last decade. There are
three new agents approved for treating myeloma including the immunomodulatory drugs,
thalidomide, and lenalinomide, and the proteosome antagonist, bortezomib. In addition,
autologous stem cell transplant after high dose melphalan offers prolonged disease
survival for eligible patients. Allogeneic stem cell transplant is usually not performed in
myeloma patients.

References

1. Jemal A, Siegel R, Ward E, Murray T, Xu J, Thun MJ (2007) Cancer Statistics


2007. Cancer J Clin 57:43-66.

2. Lahtinen R, Laakso M, Palva I, Virkkunen P, Elomaa I (1992) Randomised,


placebo-controlled multicentre trial of clodronate in multiple myeloma. Lancet
340:1049-1052.

3. Riccardi A, Gobbi PG, Ucci G, Bertoloni D, Luoni R, Rutigliano L, Ascari E


(1991) Changing clinical presentation of multiple myeloma. Eur J Cancer
27:1401-1405.

4. Kyle RA, Therneau TM, Rajkumar SV, Larson DR, Plevak MF, Melton LJ 3rd
(2004) Incidence of multiple myeloma in Olmsted County, Minnesota: Trend over
6 decades. Cancer 101:2667-2674.
5. Oyajobi BO (2007) Multiple myeloma/hypercalcemia. Arthritis Res Ther 9:S4.

6. Abe M, Hiura K, Wilde J, Moriyama K, Hashimoto T, Ozaki S, Wakatsuki S,


Kosaka M, Kido S, Inoue D, Matsumoto T (2002) Role for macrophage
inflammatory protein (MIP)-1alpha and MIP-1beta in the development of
osteolytic lesions in multiple myeloma. Blood 100:2195-2202.

7. Aisenberg AC, Kaplan MM, Rieder SV, Goldman JM (1970) Serum alkaline
phosphatase at onset of Hodgkin's disease. Cancer 26:318-326.

8. Bataille R, Boccadoro M, Klein B, Durie BG, Pileri A (1992) C-Reactive Protein


-2 microglobulin produce a simple and powerful myeloma staging system.
Blood 80:733-737.

9. Kyle RA (1992) Diagnostic criteria of multiple myeloma. Hematol Oncol Clin


North Am 6:347-358.

10. Dimopoulos MA, Moulopoulos LA, Datseris I, Weber D, Delasalle K, Gika D,


Alexanian R (2000) Imaging of myeloma bone diseaseimplications for staging,
prognosis and follow-up. Acta Oncol 39:823-827.
Learning Objectives
Understand the cause, symptoms and the
treatment of leukostasis
Chronic Leukemias Understand the pathophysiology in the
development of CML
Ahmad A Tarhini, MD, Msc
Know the phases of CML
Assistant Professor of Medicine Know the treatment of CML and the
Dept. of Medicine, Division of Hematology-
Hematology-Oncology
Email: tarhiniaa@upmc.edu monitoring needed
Know the diagnosis and treatment of CLL
January 2009
Know the disorders associated with CLL

CASE

50 year old woman presents with fatigue


and early satiety
PE: spleen, 15 cm below left costal margin
WBC 250 x 109/l, Plts
WBC Plts 600,000/microL
Peripheral Blood smear

Leukemias
What next step will most likely yield a Subdivided into acute and chronic subsets
diagnosis of CML? Also subdivided into myeloid or lymphoid
Acute:
Acute: represent arrested stages of
CT abdomen differentiation (reduced capacity to
Bone marrow aspirate with cytogenetics for differentiate into more mature cellular
t(9;22) elements)
Peripheral blood testing with flow cytometry blasts in marrow, circulation and other tissues
for B cells expressing CD5, CD19, CD20,
CD23 Chronic:
Chronic: increased production of maturing
Splenectomy
(CML) or mature (CLL) cells and lack of
apoptosis

1
Leukostasis Frequency of Leukostasis
Definition:
Definition: vascular plugging secondary to Total Cases Incidence
hyperleukocytosis with high absolute
leukocyte count (>50,000) AML 179 5%
Morbidity:
Morbidity: CML 85 15%
CNS:
CNS: intracerebral hemorrhage
ALL 143 4%
Pulmonary:
Pulmonary: hypoxemia secondary to poor
oxygen exchange CLL 89 1%

Blood; 60: 279, 1982

Size of the Leukocyte Affects the


Therapy of Leukostasis
Risk of Leukostasis
Goal
Vascular plugging Rapid reduction of peripheral leukocyte count
relates to size of cells
Myeloblasts are larger
Methods
and contribute to
more volume with Mechanical (leukapheresis
(leukapheresis))
less numbers of cells Chemotherapy
CLL is a disorder of Hydroxyurea
small mature
Induction chemotherapy (acute leukemia)
lymphocytes

Chronic Myelogenous Leukemia


CML: Epidemiology
(CML)
Clonal myeloproliferative disorder of pluripotent 15% of all leukemias (~25,000
(~25,000 new cases
stem cells
Uncontrolled production of maturing of acute leukemia annually)
granulocytes, mainly neutrophils,
neutrophils, also
eosinophils,
eosinophils, basophils Peak incidence of occurrence is 5th and 6th
proliferation, apoptosis decade
Cytogenetic hallmark: Philadelphia chr.
chr. t(9;22)
Molecular hallmark: BCR/ABL (initiation Slight male predominance (1.4X)
causative event)
Indolent disorder; excellent prognosis Predisposition:
Estimated 5 year survival 90% radiation exposure (<5%)
Unknown (95%)

2
BCR-ABL
History of CML
1960 - Philadelphia chromosome described 95% have Philadelphia chromosome t(9;22)
1973 - chromosome banding studies Abl is a tyrosine kinase
demonstrated a reciprocal translocation Chimeric bcr-
bcr-abl has increased kinase activity in
between 9 and 22
cytoplasm activating downstream kinases that
1983 - t(9;22) results in cellular oncogene
(bcr/abl) prevent apoptosis
1987 - product of abl shown to have tyrosine Interacts with several cytoskeletal proteins,
proteins, the
kinase activity Ras signaling pathway and other growth factor
1988 - beginning of synthesis of drugs that receptors pathways
specifically inhibit tyrosine kinases

Natural History of CML

CML Workup Phases of CML


History and PE Chronic phase (85%)
Performance status, splenomegaly Past: 3-
3-5 years
Present: estimated 25+ years
Labs: CBC, Plts,
Plts, diff, electrolytes, Cr
Accelerated phase
BM Asp/Bx
Asp/Bx with CG
Past: 12-
12-18 months
+/-
+/- molecular studies (FISH, RT-RT-PCR) Present: 4-
4-5 years
Blast crisis (AML 66%; ALL 33%)
3-9 months
6-12 months

3
Chronic phase Accelerated Phase
50% asymptomatic Blasts 15%
Symptoms:
Symptoms: fatigue, early Blasts + Pros 30%
satiety Basophils 20%
Exam:
Exam: splenomegaly Plts 100,000
Lab:
Lab: leukocytosis (often Clonal evolution
>100,000) with full Increasing acquisition of new chromosomal abnormalities
spectrum of precursors including second t(9,22), trisomy 8, del 17p (p53)
Basophilia Symptoms:
Symptoms: more constitutional symptoms (fever, weight
Thrombocytosis loss) and increased difficulty controlling counts
None of criteria for Exam:
Exam: increasing splenomegaly
accelerated or blast
phase

Blast phase D.D.: Leukemoid Reaction


Conversion to acute leukemia A leukemoid reaction may be confused with CML
Patients have a robust leukocytosis associated with
70% AML/30% ALL infections or following surgery
There is no accompanying basophilia
Blasts 30%
The majority of immature forms are neutrophils and
Extramedullary disease with localized bands (vs
(vs promyelocytes or myelocytes in CML)
immature blasts Leukocyte alkaline phosphatase (LAP): if the
granulocytes are normal, the LAP score is elevated.
In CML, LAP scores are zero or very low reflecting an
intrinsic abnormality of the granulocytes.

Therapy of CML
Bone marrow transplant is the only
cure - eradication of disease

Imatinib Mesylate (TKI)-


(TKI)- functional
cure
cure

4
CML - Modern Therapy CML Therapy
Chronic Phase Chronic Phase

Frontline treatment: Imatinib mesylate Historically oral chemotherapy used: could


Second line: Allogeniec SCT, new tyrosine control counts but not induce a remission
kinase inhibitors Hydroxyurea (inhibitor of ribonucleotide
Other reductase)
Busulfan (alkylating agent)
Hydroxyurea,
Hydroxyurea, Busulfan
IFN- (+ ara-
IFN- ara-C)

CML Therapy CML Therapy


Chronic Phase Chronic Phase
Imatinib:
Imatinib: first molecularly targeted drug
Interferon-
Interferon-a - alone or in combination with marketed
cytarabine Oral medication with mild side effects
(diarrhea, edema, myalgias, fever)
Interferon:
Interferon: cytostatic agent; with multiple side
effects (fevers, myalgias, depression) Use in chronic phase induces a
Could both control the counts and would
hematologic remission (WBC <10K; nl Hb
induce elimination of bcr/abl clone in <20% and platelets) in 90%
(molecular remission) Induces a molecular remission in ~70%

5
Monitoring Response in CML
Hematologic remission:
remission: normalization of blood
counts
Cytogenetic remission:
remission: loss of Ph1+ clone
Requires metaphases (20 metaphases counted)
Sensitivity 10-2
Molecular remission:
remission:
FISH (200 cells counted)
Sensitivity 10-3
RT-
RT-PCR (10,000 cells counted)
Sensitivity 10-5

Druker. NEJM. December 2006

CML Therapy
Tyrosine Kinase Inhibitors
Accelerated Phase
Imatinib is also being used in:
GI stromal tumors (express stem cell ligand)
ligand)
Hypereosinophilic syndromes (express PDGFR) Imatinib or other TKI (+
(+ IFN, ara-
ara-C,
chemo)
FDA has approved two new tyrosine kinase inhibitors for
patients resistant/intolerant to imatinib Allo SCT if no major CG response after 3
months
Dastinib binds both active and inactive conformations of abl
kinase domain
Nilotinib has higher affinity and selectivity for abl kinase

6
CML Therapy Results of SCT in CML
Blast Phase 40-
40-50% cure rate
Time to blast phase prior Most frequent cause of failure is relapsed
to imatinib 3 years underlying CML
Survival following Donor lymphocyte infusion can control the clone
conversion to blast phase 1-year mortality 10% to 50% (depending on age,
6 months degree of matching, source - related vs. MUD)
Not cured by conventional Another cause of death is uncontrolled graft vs.
chemotherapy for acute host disease (5-
(5-10%)
leukemia Morbidities: GVHD, sterility, cataracts, hip
Chemo + Imatinib/Dasatinib necrosis, second cancers, QOL
then Allo SCT

CASE
Blood Smear
H: 70 y.o.
y.o. man presents with fatigue, neck
swelling, recurrent fevers and night
sweats, 20 pound weight loss in the past 7
months
Small mature
lymphocytes
LN
PE: palpable bilateral cervical LN Smudge cells
cells

Labs: WBC 213,000/microl, absolute


lymphocyte count 200,000/microl, HGB
13, Plts 200,000/microl

QUESTION
What is the most appropriate next step for
establishing the diagnosis?

Cervical lymph node biopsy


Bone marrow aspirate with cytogenetics for
t(9;22)
Peripheral blood testing with flow cytometry

PET/CT (whole body)

7
CLL
Chronic Lymphocytic Leukemia
Epidemiology
Dx made by flow cytometry: circulating
Most common form of leukemia B cells (CD19+, CD20+, CD23+), clonal
Peak incidence between ages 60-
60-80 (k or l restricted), and co-express CD5

Predisposing factors:
factors: inherited or acquired
No characteristic cytogenetics but cells
immune deficiency diseases over-express bcl-2 (prevents apoptosis)

Cytogenetics in CLL
80% will have an
abnormality
None diagnostic
Most common
abnormality is 13q
deletion (>50%)
followed by others:
11q deletion
Trisomy 12
17p deletion

Other Prognostic Factors in CLL Rai Staging System


Stage 0: lymphocytosis only (>8,000 absolute
lymphocytes at two points in time)
Stage 1: stage 0 + lympadenopathy
Stage 2: stage 1 + hepatosplenomegaly
Stage 3: stage 2 + anemia (HGB < 11)
Stage 4: stage 3 + thrombocytopenia
(<100,000)

*not always so orderly

8
Survival by Stage
Rai
classification

Binet classification

CLL - Cause of Death


CLL - Therapy

30% of patients die of other causes


No therapy needed unless doubling
Infections (hypogammaglobulinemia, time short, bulky disease, anemia,
neutropenia) thrombocytopenia, constitutional
symptoms
Complications of cytopenias
Oral alkylating agents (chlorambucil,
cyclophosphamide)
Fludarabine (intravenous) single most
active agent

CLL - Mechanisms and Side Effects of


CLL - Mechanisms and Side Effects
Drugs
of Drugs
Purine analogues [fludarabine; deoyxcoformycin
Chlorambucil (Pentostatin), 2-chlorodeoxyadenosine
Mech: direct damage to DNA (Cladrabine)]
Side effects: cytopenias (bone marrow
suppression), nausea Mech: inhibits adenosine deaminase
Cyclophosphamide Side effects: bone marrow suppression, nausea,
Mech: same induction of T cell immunodeficiency
Side effects: cytopenias, hemorrhagic cystitis

9
CLL - Immunotherapy CLL - Immunotherapy
Rituximab (Rituxan)
Rituxan) Alemtuzumab (Campath)
Campath)

Anti-
Anti-CD20 used in CLL and NHL Anti-
Anti-CD52 used in lymphoid malignancies
Mechanism Mechanism
induces ADCC, induces apoptosis, activates induces ADCC, activates complement
complement; can be used to target Side effects
radiotherapy
immune reaction: fever, rigors, urticaria
Side effects
can induce tumor lysis syndrome
immune reaction: fever, rigors, urticaria
can induce profound immundeficiency
can induce tumor lysis syndrome

Associated Syndromes with


CLL
Immune-mediated hemolytic anemia (IgG) (10%)
Immune-mediated thrombocytopenia (5%)
Both = Evans Syndrome
Hypogammaglobulinemia
Increased second malignancies (8X)
Richters Syndrome-conversion to large cell
lymphoma (<5%)
Conversion to prolymphocytoid leukemia (<5%)

10
CHRONIC MYELOGENOUS LEUKEMIA
Incidence
The incidence of chronic myelogenous leukemia (CML) is 1.5 per 100,000 people per year, and
the age-adjusted incidence is higher in men than in women (2.0 versus 1.2). The incidence of
CML increases slowly with age until the middle forties, when it starts to rise rapidly. CML
incidence for males decreased slightly (4.4%) between 1997 and 2003 as compared to 1977
1997.

Definition
The diagnosis of CML is established by identifying a clonal expansion of a hematopoietic stem
cell possessing a reciprocal translocation between chromosomes 9 and 22. This translocation
results in the head-to-tail fusion of the breakpoint cluster region (BCR) gene on chromosome
22q11 with the ABL (named after the abelson murine leukemia virus) gene located on
chromosome 9q34. Untreated, the disease is characterized by the inevitable transition from a
chronic phase to an accelerated phase and on to blast crisis in a median time of 4 years.

Etiology
No clear correlation with exposure to cytotoxic drugs has been found, and no evidence
suggests a viral etiology. In the pre-imatinib era, cigarette smoking accelerated the
progression to blast crisis and therefore adversely affected survival in CML. Atomic bomb
survivors had an increased incidence; the development of a CML cell mass of 10,000/ L took
6.3 years. No increase in CML incidence was found in the survivors of the Chernobyl accident,
suggesting that only large doses of radiation can induce CML.

Pathophysiology
The product of the fusion gene resulting from the t(9;22) plays a central role in the
development of CML. This chimeric gene is transcribed into a hybrid BCR/ABL mRNA in which
exon 1 of ABL is replaced by variable numbers of 5' BCR exons. Bcr/Abl fusion proteins,
p210BCR/ABL, are produced that contain NH2-terminal domains of Bcr and the COOH-terminal
domains of Abl. A rare breakpoint, occurring within the 3' region of the BCR gene, yields a
fusion protein of 230 kDa, p230BCR/ABL. Bcr/Abl fusion proteins can transform hematopoietic
progenitor cells in vitro. Furthermore, reconstituting lethally irradiated mice with bone marrow
cells infected with retrovirus carrying the gene encoding the p210BCR/ABL leads to the
development of a myeloproliferative syndrome resembling CML in 50% of the mice. Specific
antisense oligomers to the BCR/ABL junction inhibit the growth of t(9;22)-positive leukemic
cells without affecting normal colony formation.

The mechanism(s) by which p210BCR/ABL promotes the transition from the benign state to the
fully malignant one is still unclear. Messenger RNA for BCR/ABL can occasionally be detected
in normal individuals. However, attachment of the BCR sequences to ABL results in three
critical functional changes: (1) the Abl protein becomes constitutively active as a tyrosine
kinase (TK) enzyme, activating downstream kinases that prevent apoptosis; (2) the DNA-
protein-binding activity of Abl is attenuated; and (3) the binding of Abl to cytoskeletal actin
microfilaments is enhanced.
Clinical Presentation
SYMPTOMS
The clinical onset of the chronic phase is generally insidious. Accordingly, some patients are
diagnosed while still asymptomatic, during health-screening tests; other patients present with
fatigue, malaise, and weight loss or have symptoms resulting from splenic enlargement, such
as early satiety and left upper quadrant pain or mass. Less common are features related to
granulocyte or platelet dysfunction, such as infections, thrombosis, or bleeding. Occasionally,
patients present with leukostatic manifestations due to severe leukocytosis or thrombosis such
as vasoocclusive disease, cerebrovascular accidents, myocardial infarction, venous
thrombosis, priapism, visual disturbances, and pulmonary insufficiency. Patients with
p230BCR/ABL-positive CML have a more indolent course.

Progression of CML is associated with worsening symptoms. Unexplained fever, significant


weight loss, increasing dose requirement of the drugs controlling the disease, bone and joint
pain, bleeding, thrombosis, and infections suggest transformation into accelerated or blastic
phases. Fewer than 1015% of newly diagnosed patients present with accelerated disease or
with de novo blastic phase CML.

PHYSICAL FINDINGS
Minimal to moderate splenomegaly is the most common physical finding; mild hepatomegaly
is found occasionally. Persistent splenomegaly despite continued therapy is a sign of disease
acceleration. Lymphadenopathy and myeloid sarcomas are unusual except late in the course
of the disease; when they are present, the prognosis is poor.

HEMATOLOGIC FINDINGS
Elevated white blood cell counts (WBCs), with increases in both immature and mature
granulocytes, are present at diagnosis. Usually <5% circulating blasts and <10% blasts and
promyelocytes are noted with the majority of cells being myelocytes, metamyelocytes and
band forms. Cycling of the counts may be observed in patients followed without treatment.
Platelet counts are almost always elevated at diagnosis, and a mild degree of normocytic
normochromic anemia is present. Leukocyte alkaline phosphatase is low in CML cells. Serum
levels of vitamin B12 and vitamin B12binding proteins are elevated. Phagocytic functions are
usually normal at diagnosis and remain normal during the chronic phase. Histamine
production secondary to basophilia is increased in later stages, causing pruritus, diarrhea, and
flushing.

At diagnosis, bone marrow cellularity is increased, with an increased myeloid to erythroid


ratio. The marrow blast percentage is generally normal or slightly elevated. Marrow or blood
basophilia, eosinophilia, and monocytosis may be present. While collagen fibrosis in the
marrow is unusual at presentation, significant degrees of reticulin stainmeasured fibrosis are
noted in about half of the patients.

Disease acceleration is defined by the development of increasing degrees of anemia


unaccounted for by bleeding or therapy; cytogenetic clonal evolution; or blood or marrow
blasts between 10 and 20%, blood or marrow basophils 20%, or platelet count <100,000/ L.
Blast crisis is defined as acute leukemia, with blood or marrow blasts 20%. Hyposegmented
neutrophils may appear (Pelger-Huet anomaly). Blast cells can be classified as myeloid,
lymphoid, erythroid, or undifferentiated, based on morphologic, cytochemical, and
immunologic features. Occurrence of de novo blast crisis or following imatinib therapy is rare.

CHROMOSOMAL FINDINGS
The cytogenetic hallmark of CML, found in 9095% of patients, is the t(9;22)(q34;q11.2).
Originally, this was recognized by the presence of a shortened chromosome 22 (22q-),
designated as the Philadelphia chromosome, that arises from the reciprocal t(9;22). Some
patients may have complex translocations (designated as variant translocations) involving
three, four, or five chromosomes (usually including chromosomes 9 and 22). However, the
molecular consequences of these changes are similar to those resulting from the typical
t(9;22). All patients should have evidence of the translocation molecularly or by cytogenetics
or FISH to make a diagnosis of CML.

Treatment
The therapy of CML is changing rapidly because we have a proven curative treatment
(allogeneic transplantation) that has significant toxicity and a new targeted treatment
(imatinib) with excellent outcome based on 5-year follow-up data. Therefore, physician
experience and patient preference must be factored into the treatment selection process.
Discussion of both treatment options with a patient is indicated. The decision should focus on
the outcomes, risks, and toxicities of the various approaches.

At present, the goal of therapy in CML is to achieve prolonged, durable, nonneoplastic,


nonclonal hematopoiesis, which entails the eradication of any residual cells containing the
BCR/ABL transcript. Hence the goal is complete molecular remission and cure.

ALLOGENEIC SCT
Allogeneic SCT is complicated by early mortality owing to the transplant procedure. Outcome
of SCT depends on multiple factors including: (1) the patient (e.g., age and phase of disease);
(2) the type of donor [e.g., syngeneic (monozygotic twins) or HLA-compatible allogeneic,
related or unrelated]; (3) the preparative regimen (myeloblative or reduced intensity); (4)
GVHD; and (5) posttransplantation treatment. In the imatinib era, allogeneic transplantation
should be used when possible for patients with accelerated/blastic phases of the disease or
those whose disease fails to respond or progresses on imatinib.

IMATINIB MESYLATE
Imatinib mesylate (Gleevec) functions through competitive inhibition at the ATP binding site of
the Abl kinase in the inactive conformation, which leads to inhibition of tyrosine
phosphorylation of proteins involved in Bcr/Abl signal transduction. It shows specificity for
Bcr/Abl, the receptor for platelet-derived growth factor, and Kit tyrosine kinases. Imatinib
induces apoptosis in cells expressing Bcr/Abl.

In newly diagnosed CML, imatinib (400 mg/d) is more effective than IFN- and cytarabine.
The complete hematologic remission rate, at 18 months, of patients treated with imatinib was
97% compared to 69% in patients treated with IFN- and cytarabine. Similarly, the complete
cytogenetic remission rate was 76% with imatinib compared to 14% with IFN- and
cytarabine.

All imatinib-treated patients who achieved major molecular remission (26%), defined as 3
log reduction in BCR/ABL transcript level at 18 months compared to pretreatment level, were
progression-free at 5 years. The progression-free survival (PFS) at 5 years for patients
achieving complete cytogenetic remission but less pronounced molecular remission is 98%.
The 5-year PFS for patients not achieving complete cytogenetic remission at 18 months was
87%. These results have led to a consensus that molecular responses can be used as a
treatment goal in CML. For example, chronic-phase CML patients who do not achieve any
cytogenetic remission following six months of imatinib are unlikely to achieve major molecular
remission and should be offered other treatment approaches.

Progression to accelerated/blastic phases of the disease was noted in 3% of patients treated


with imatinib as compared to 8.5% of patients treated with IFN- and cytarabine during the
first year. Over time, the annual incidence of disease progression on imatinib decreased
gradually to <1% during the fourth and fifth years, and no patient who achieved complete
cytogenetic remission during the first year of imatinib treatment progressed to the
accelerated/blastic phases of the disease.

Imatinib is administered orally. The main side effects are fluid retention, nausea, muscle
cramps, diarrhea, and skin rashes. The management of these side effects is usually
supportive. Myelosuppression is the most common hematologic side effect. Myelosuppression,
while rare, may require holding drug and/or growth factor support.

Mutations at the kinase domain are being targeted by novel TK inhibitors that have a different
conformation than imatinib, demonstrating activity against most imatinib-resistant mutations.
Nilotinib (Tasigma), like imatinib, binds to the kinase domain in the inactive conformation.
Dasatinib (Sprycel) binds to the kinase domain in the open conformation and also inhibits the
SRC (sarcoma) family of kinases, addressing the last mechanism of resistance. CML with the
T315I mutation is resistant to imatinib, nilotinib, and dasatinib.

Dasatinib is approved by the FDA for the treatment of all stages of CML with resistance or
intolerance to prior therapy, including imatinib. Nilotinib will likely follow suit. Both are oral
agents given twice daily, with toxicity profiles similar to imatinib with small but significant
differences. Dasatinib was shown to cause pleural effusion in 22% of patients with 7%
developing grade 3-4 toxicity. Nilotinib was associated with sudden death in six of
approximately 550 CML patients. A suspected relationship to nilotinib was reported in two of
these cases.

These new agents have changed the treatment algorithm of CML. For example, patients who
do not achieve any cytogenetic remission at six months on imatinib will now be offered either
dasatinib or SCT. IFN- , though FDA-approved for CML, will only be offered if all other options
have failed.

The encouraging results with imatinib have led clinicians to offer it as first-line therapy for
newly diagnosed CML patients, including those who otherwise would have benefited from
transplant (e.g., young patients with a matched sibling donor). Prior exposure to imatinib does
not affect transplant outcome. However, delaying BMT for high-risk patients may result in
disease progression. SCT after disease progression is associated with poorer outcome.
Therefore, we recommend close monitoring of imatinib response, especially in these patients.

TREATMENT OF BLAST CRISIS


Treatments for primary blast crisis, including imatinib, are generally ineffective. Only 52% of
patients treated with imatinib achieved hematologic remission (21% complete hematologic
remission), and the median overall survival was 6.6 months. Patients who achieve complete
hematologic remission or whose disease returns to a second chronic phase should be
considered for allogeneic SCT. Other approaches include induction chemotherapy tailored to
the phenotype of the blast cell followed by imatinib, with or without additional chemotherapy
and SCT. Blast crisis following initial therapy with imatinib carries a dismal prognosis even if
treated with dasatinib or nilotinib.
Excerpted from Harrisons Online
CHAPTER 104. ACUTE AND CHRONIC MYELOID LEUKEMIA
CHRONIC LYMPHOCYTIC LEUKEMIA
Essentials of Diagnosis
Lymphocytosis > 5000/mcL.

Coexpression of CD19, CD5 on lymphocytes.

General Considerations
Chronic lymphocytic leukemia (CLL) is a clonal malignancy of B lymphocytes. The disease is
usually indolent, with slowly progressive accumulation of long-lived small lymphocytes. These
cells are immunoincompetent and respond poorly to antigenic stimulation.

CLL is manifested clinically by immunosuppression, bone marrow failure, and organ infiltration
with lymphocytes. Immunodeficiency is also related to inadequate antibody production by the
abnormal B cells. With advanced disease, CLL may cause damage by direct tissue infiltration.

Information about CLL is now evolving rapidly, with new findings in biology and new treatment
options.

Clinical Findings
SYMPTOMS AND SIGNS
CLL is a disease of older patients, with 90% of cases occurring after age 50 years and a
median age at presentation of 65 years. Many patients will be incidentally discovered to have
lymphocytosis. Others present with fatigue or lymphadenopathy. On examination, 80% of
patients will have lymphadenopathy and 50% will have enlargement of the liver or spleen.

The long-standing Rai classification system remains prognostically useful today: stage 0,
lymphocytosis only; stage I, lymphocytosis plus lymphadenopathy; stage II, organomegaly;
stage III, anemia; stage IV, thrombocytopenia. These stages can be collapsed in to low-risk
(stages 0I), intermediate risk (stage II) and high-risk (stages IIIIV).

CLL usually pursues an indolent course, but some subtypes behave more aggressively; a
variant, prolymphocytic leukemia, is more aggressive. The morphology of the latter is
different, characterized by larger and more immature cells. In 510% of cases, CLL may be
complicated by autoimmune hemolytic anemia or autoimmune thrombocytopenia. In
approximately 5% of cases, while the systemic disease remains stable, an isolated lymph
node transforms into an aggressive large cell lymphoma (Richter syndrome).

LABORATORY FINDINGS
The hallmark of CLL is isolated lymphocytosis. The white blood count is usually greater than
20,000/mcL and may be markedly elevated to several hundred thousand. Usually 7598% of
the circulating cells are lymphocytes. Lymphocytes appear small and mature, with condensed
nuclear chromatin, and are morphologically indistinguishable from normal small lymphocytes,
but smaller numbers of larger and activated lymphocytes may be seen. The hematocrit and
platelet count are usually normal at presentation. The bone marrow is variably infiltrated with
small lymphocytes. The immunophenotype of CLL demonstrates coexpression of the B
lymphocyte lineage marker CD19 with the T lymphocyte marker CD5; this finding is commonly
observed only in CLL and mantle cell lymphoma. CLL is distinguished from mantle cell
lymphoma by the expression of CD23, low expression of surface immunoglobulin and CD20,
and the absence of overexpression of cyclin D1. Patients whose CLL cells have mutated forms
of the immunoglobulin gene have a more indolent form of disease; these cells typically
express low levels of the surface antigen CD38 and do not express the zeta-associated protein
(ZAP-70). Conversely, patients whose cells have unmutated IgV genes and high levels of ZAP-
70 expression do less well. The assessment of genomic changes by fluorescence in-situ
hybridization (FISH) provides important prognostic information. The finding of deletions of
chromosome 17p or 11q confers a poor prognosis, whereas those whose only genomic change
is deletion of 13q have a very favorable outcome.

Hypogammaglobulinemia is present in 50% of patients and becomes more common with


advanced disease. In some, a small amount of IgM paraprotein is present in the serum.

Differential Diagnosis
Few syndromes can be confused with CLL. Viral infections producing lymphocytosis should be
obvious from the presence of fever and other clinical findings; however, fever may occur in
CLL from concomitant bacterial infection. Pertussis may cause a particularly high total
lymphocyte count. Other lymphoproliferative diseases such as Waldenstrm
macroglobulinemia, hairy cell leukemia, or lymphoma (especially mantle cell) in the leukemic
phase are distinguished on the basis of the morphology and immunophenotype of circulating
lymphocytes and bone marrow.

Treatment
Most cases of early indolent CLL require no specific therapy, and the standard of care for early
stage disease has been observation. However, with advances in therapy and with new
information on biologic prognostic factors, clinical trials will investigate whether there is a role
for early intervention in subsets of patients with early stage disease. Indications for treatment
include progressive fatigue, symptomatic lymphadenopathy, or anemia or thrombocytopenia.
These patients have either symptomatic and progressive Rai stage II disease or stage III/IV
disease. The initial treatment of choice is the combination of the chemotherapeutic agent
fludarabine plus the antibody rituximab, with or without the addition of the chemotherapeutic
drug cyclophosphamide. The addition of cyclophosphamide appears to increase the risk of
treatment-related infection, and the question of whether this increase in toxicity is warranted
by improved anti-leukemic effectiveness is currently being studied. Chlorambucil, 0.61
mg/kg orally every 3 weeks for approximately 6 months, was the standard treatment prior to
the development of fludarabine. This treatment is convenient, well tolerated, and remains a
reasonable first choice for elderly patients for whom frequent trips to the clinician's office is a
hardship. The monoclonal antibody alemtuzumab is approved for treatment of refractory CLL
and can be especially useful in clearing the blood and bone marrow of disease. However, it
produces significant immunosuppression, and its role in primary therapy has been limited due
to the risk of severe and fatal infections.
Associated autoimmune hemolytic anemia or immune thrombocytopenia may require
treatment with rituximab, prednisone, or splenectomy. Fludarabine should be avoided in
patients with autoimmune hemolytic anemia since it may exacerbate this condition. Patients
with recurrent bacterial infections and hypogammaglobulinemia benefit from prophylactic
infusions of gamma globulin 0.4 g/kg/month), but this treatment is very expensive and can be
justified only when these infections are severe.

Allogeneic transplantation offers potentially curative treatment for patients with CLL, but it
should be used only in patients whose disease cannot be controlled by standard therapies.
Nonmyeloablative allogeneic transplant has produced encouraging results and may expand the
role of transplant in CLL. Some subtypes of CLL with genomic abnormalities such as 17p
deletions have a sufficiently poor prognosis with standard therapies that early intervention
with allogeneic transplant is being studied to assess whether it can improve outcomes.

Prognosis
New therapies are changing the prognosis of CLL. In the past, median survival was
approximately 6 years, and only 25% of patients lived more than 10 years. Patients with
stage 0 or stage I disease have a median survival of 1015 years, and these patients may be
reassured that they can live a normal life for many years. Patients with stage III or stage IV
disease had a median survival of less than 2 years in the past, but with fludarabine-based
combination therapies, 2-year survival is now greater than 90% and the long-term outlook
appears to be substantially changed. For patients with high-risk and resistant forms of CLL,
there is evidence that allogeneic transplantation can overcome risk factors and lead to long-
term disease control.

EXCERPTED FROM CURRENT MEDICAL DX & TX


CHAPTER 13. BLOOD DISORDERS > LEUKEMIAS & OTHER MYELOPROLIFERATIVE
DISORDERS
Human Hemopoiesis Principles of hematopoietic stem cell transplantation

allogeneic transplantation autologous transplantation


conditioning conditioning

Hematopoietic Stem Cell


Transplantation Patient Patient
TXP TXP
Markus Y. Mapara MD PhD Donor

January 2009 BM PBSCT

Maximal cytoreduction (*) Maximal cytoreduction


Exchange of hematopoiesis Retransfusion of autologous
allogene cellul. Immunotherapy hematoppoietic Progenitor cells

Types of Transplants Stem Cells and Transplantation


Types of Transplants Rationale
Peripheral Blood Stem Cells (PBSCT) AUTO: High dose chemo/radiation rescue
Autologous Transplant A dose-response effect has been shown for many cancers
Stem cells collected peripherally using apheresis (cell separator ie: doubling of certain chemo agents may increase the cell kill by a factor of
Patients own stem cells machine) 10
Less invasive; less discomfort; less morbidity than BM
Dose-intense treatment may improve response rates or result in cures, even
Outpatient procedure those that have failed conventional doses of chemo
Allogeneic Transplant PBSCT results in more rapid hematopoietic recovery than BM Profound myeloablation would result in death without hematopoietic stem cell
No difference in treatment outcome rescue
Stem cells from someone else=donor stem cells Quickly replacing traditional BM ALLO: Actual killing of cancer cells by donor stem cell progenitors: Graft
versus Tumor Effect
Using cytokine stimulation (G-CSF injections) Donor immune cells attack recipient malignant cells
BM releases large number CD34 stem cells into circulation Recognize recipient malignant cells as foreign
Stem cells harvested via peripheral line

1
Harvesting of Stem cells using Leukapheresis Stem Cell Processing under cGMP conditions Enumeration of hematopoietic progenitor
cells by CD34 staining using FACS

Mobilization of CD34 cells Stem Cell Collection and Transplant Procedure Retransfusion of Stem Cells
Autologous:
A. Steady state G-CSF 5 days of SC cytokine injections to mobilize stem cells from
BM into PB (G-CSF)
Day 5 & 6: PB collected and apheresed to separate/isolate
CD34+ stem cells
Volume 200 mL cryopreserved in dimethylsulfoxide (DMSO)
and stored at -196 C using liquid nitrogen
High dose chemotherapy +/- irradiation (TBI) given over 2-5
days
2-3 days later stem cells are thawed and infused through
central line
Stem cells home in on BM and begin differentiation and BM
B. Rebound after cytotoreductive Chemotherapy plus G-CSF recovery (14 days)
Patient supported with transfusions (PLT<10;Hb<80) and
given G-CSF until neutrophils recover
Prophylactic antibiotics given; fevers or infections treated
aggressively

2
Rationale for high dose chemotherapy and
autologous stem cell transplantation
Dose effect relation Role of chemotherapy sensitivity on Disease
free survival after transplantation
High dose
chemotherapy

Stem cell harvest Transplantation

Chemotherapy sensitivity

Linear Dose-effect relation

3
Gene marking studies
LTR Neo LTR

marking Neo-
R Gene

Retransfusion
graft afterHD-CT

Detection of
Marker gene
Barlogie ASBMT 2007
In relapse
Barlogie ASBMT 2007

4
Positive and negative selection approaches
Positive Selection Negative Selection Allogeneic stem cell transplantation
Fe

Fe
Selection of
Biotin Avidin
CD34+stem cells
Induction of Breaking of
Avidin Immune Complement tolerance tolerance
Biotin
Immunotoxins

Suicide gene transfer

DACS
(Density adjusted cell
separation) Induction of apoptosis (anti-CD95 mAb)

Types of Graft rejection in Transplantation Major Histocompatibility complex (MHC)


GVL & GVHD is Immune Mediated Minor Ag
Solid Organ Transplantation
Donor Immune cells recognize Recipient cells as non-self
HVG
Immunsuppression T-cell & NK cell response
GVH for prohpylaxis of graft rejection
Attack host cells: malignant and normal host cells

Balance of this immune response:


Allogeneic BMT
Minimize GVHD + Maximize GVL
Immunsuppression 1) Immunosuppressive Therapy with BMT
HVG for GVHD-Prophylaxis
2) HLA-Match Donor & Recipient
Match major antigens to decrease GVHD
GVH TOLERANCE Mismatch of minor antigens results in GVL
DONOR Patient

Major Ag

5
MHC-HLA Types of Transplants
HLA Balance for alloBMT
MHC molecules discriminate self & non-self Allogeneic Transplants
In humans = HLA molecules Stem cells donated from another
function is Ag presentation to T-cells HLA match most important (HLA- A,B and DR)=6/6= match
Major Class I & II HLA matched for allogeneic
Via Antigen Presenting Cells (Dendritic Cells)
T-Cells trained early in thymus to recognize this
BMT
Matched Sibling
Decreases severe GVHD Best match for major and minor antigens
Minor HLA antigens not matched
Major HLA molecules (Class I & II) most important: Matched Unrelated
Results in GVL and GVHD BMT bank now over 8 million donors
Class I (HLA-A, HLA-B) molecules interact with CD8 Cytotoxic T-
cells Only recently being characterized
Class II (HLA-DR) interact with CD4 Helper T-cells Mismatched
Single mismatch (5/6) or Haploidentical (3/6)

Graft versus host disease


Types of Transplants Types of Transplants Toxicity of conditioning therapy (cytokine storm)
Translocation of LPS in the GI-system
Alloreactivity of donor cells (extent of MHC disparity)
Non-myeloablative or Mini-allogeneic T cell polarization (Th1 vs Th2)
Syngeneic Transplants transplants Lungs
Identical twins Non-myeloablative conditioning chemotherapy
Not very successful: Less toxic than full myeloablative traditional BMTs Liver
major AND minor HLA perfectly matched Takes advantage of the graft vs tumor effect
No GVL effect Decreased mortality of transplant Gut
Allogeneic transplants now possible for elderly &
medically compromised
Recent trials showing promising results
Skin

6
Conditioning-induced inflammatory cascade for Influence of acute GVHD on leukemia free survival
Role of inflammatory reaction for development of GVHD
development of GVHD: Crucial role for LPS
SPF housed mice have less GVHD


A B
Prophylactic antibiotic gut decontamination reduces
incidence of GVHD clinically

BM + 5 x106 T cells d0
Donor cells from TLR4 mutant mice induce less
severe GVHD
A B
B975, a synthetic lipid-A analogues reduces GVHD
BM d0
TNF-a levels after conditioning predict development 5 x106 T-cells d56 Ringden, Bone Marrow Transplant, 1996

of GVHD

aGVHD IV
Acute GVHD of the skin

7
aGVHD of the skin: grade I-II
GVHD and TRM
Therapy of acute GVHD
Prophylaxis:
Cyclosporine and Steroids ( MTX)
T-cell depletion of the graft

Therapy: high dose steroids,


In vivo T cell depletion (ATG, OKT3), IL-
2 R)

Ringden, Bone Marrow Transplant, 1996

Sole administration of donor lymphocyte infusions for induction of


Graft-versus-Leukemia (GVL) effects in GVL effects Regression of pulmonary metastases after NMA
Leukemia patients HSCT for RCC (Childs et al. 1999 JCO)
Donor
PBSCT Donor lymphocyte
80 infusions
% relapse probability

70 AML GVL
60 ALL
CML in CP Relapse
50

40

30
Patient
Disease Patients % Response
20
CML cytogenet. relapse 43/53 (81%)
10 CML hematolog. relapse 113/148 (76%)
CML acceleration/BC 18/54 (33%)
0
AML 18/81 (22%)
en D D D D Horowitz, Blood, 1990
ALL 3/37 (8%)
ng TC HD vH vH vH
sy aG cG
GV cG MDS 5/14 (36%)
a+
No Luznik and Fuchs 2002; Cancer Control 9: 123-137

8
Newly dx MM
Auto-auto 58 mo MST
2-3 x VAD Auto-allo not yet reached

Auto-auto EFS 33 mo
CTX Auto-Allo EFS 43 mo

Mel 200

2GyTBI/ Mel 200


CsA/MMF

No TCD ab!!

Bruno et al, 2007 NEJM Bruno et al, 2007 NEJM

Approaches for separating GVHD and GVL Infectious complications following allogeneic BMT Adverse Effects

Inflammatory response Autologous


Delayed DLI Well tolerated/ low mortality rate (<1%)
Reduced conditioning Allogeneic
Chemokines Graft Versus Host Disease (GVHD)
Host
MHC I Adhesion molecules Donor T lymphocytes recognize recipient as foreign and mount
HSV
CMV immune reaction
Necrotic cell death factors ?
VZV 3 main organs affected: Skin, GI, Liver toxicities
Donor T cell
Cellular therapies (GVHD): Infections
Regulatory T cells Significant immune disruption can result in severe opportunistic
Candida
Tolerogenic DC Aspergillus Aspergillus
infections (Fungal, Viral:CMV, bacterial)
Host APC Pneumocystis carinii
Graft Rejection- very rare
Cellular therapies (GVL):
Mi-Ag specifc T cells
TCR transfer

9
Autologous Stem Cell Transplant
Allogeneic Transplants
Advantages Advantages Disadvantages
Lower Mortality (TRM) Disadvantages Graft Vs Tumor=CURE Higher Mortality
<1% Lower Cure Donor immune response GVHD
against malignant cells Infections
No rejection No Graft Vs Tumor
With prophylactic abx,
No GVHD Potential for tumor antifungals &
Less infections/ less contamination immunosuppressive tx
immune suppression Graft Rejection
Now only 1% or less
z
z
65-70 yo or older <60 yo

10
AccessMedicine | Print: Chapter 108. Hematopoietic Cell Transplantation 12/10/08 10:16 PM

Note: Large images and tables on this page may necessitate printing in landscape mode.

Copyright The McGraw-Hill Companies. All rights reserved.

Harrison's Online > Part Six: Oncology and Hematology > Section 2: Hematopoietic Disorders > Chapter 108. Hematopoietic
Cell Transplantation >

HEMATOPOIETIC CELL TRANSPLANTATION: INTRODUCTION


Bone marrow transplantation was the original term used to describe the collection and transplantation of
hematopoietic stem cells, but with the demonstration that the peripheral blood and umbilical cord blood are
also useful sources of stem cells, hematopoietic cell transplantation has become the preferred generic term for
this process. The procedure is usually carried out for one of two purposes: (1) to replace an abnormal but
nonmalignant lymphohematopoietic system with one from a normal donor, or (2) to treat malignancy by
allowing the administration of higher doses of myelosuppressive therapy than would otherwise be possible. The
use of hematopoietic cell transplantation has been increasing, both because of its efficacy in selected diseases
and because of increasing availability of donors. The International Bone Marrow Transplant Registry
(http://www.ibmtr.org) estimates that about 50,000 transplants are performed each year.

THE HEMATOPOIETIC STEM CELL


Several features of the hematopoietic stem cell make transplantation clinically feasible, including its
remarkable regenerative capacity, its ability to home to the marrow space following intravenous injection, and
the ability of the stem cell to be cryopreserved. Transplantation of a single stem cell can replace the entire
lymphohematopoietic system of an adult mouse. In humans, transplantation of a few percent of a donor's
bone marrow volume regularly results in complete and sustained replacement of the recipient's entire
lymphohematopoietic system, including all red cells, granulocytes, B and T lymphocytes, and platelets, as well
as cells comprising the fixed macrophage population, including Kupffer cells of the liver, pulmonary alveolar
macrophages, osteoclasts, Langerhans cells of the skin, and brain microglial cells. The ability of the
hematopoietic stem cell to home to the marrow following intravenous injection is mediated, at least in part, by
the interaction of cell-surface molecules, termed selectins, on bone marrow endothelial cells with ligands,
termed integrins, on early hematopoietic cells. Human hematopoietic stem cells can survive freezing and
thawing with little, if any, damage, making it possible to remove and store a portion of the patient's own bone
marrow for later reinfusion following treatment of the patient with high-dose myelotoxic therapy.

CATEGORIES OF HEMATOPOIETIC CELL TRANSPLANTATION


Hematopoietic cell transplantation can be described according to the relationship between the patient and the
donor and by the anatomic source of stem cells. In ~1% of cases, patients have identical twins who can serve
as donors. With the use of syngeneic donors, there is no risk of graft-versus-host disease (GVHD) that often
complicates allogeneic transplantation, and unlike the use of autologous marrow, there is no risk that the stem
cells are contaminated with tumor cells.

Allogeneic transplantation involves a donor and recipient who are not immunologically identical. Following
allogeneic transplantation, immune cells transplanted with the stem cells or developing from them can react
against the patient, causing GVHD. Alternatively, if the immunosuppressive preparative regimen used to treat
the patient before transplant is inadequate, immunocompetent cells of the patient can cause graft rejection.

http://online5.hsls.pitt.edu:2206/popup.aspx?aID=2866250&print=yes_chapter Page 1 of 15
AccessMedicine | Print: Chapter 108. Hematopoietic Cell Transplantation 12/10/08 10:16 PM

the patient before transplant is inadequate, immunocompetent cells of the patient can cause graft rejection.
The risks of these complications are greatly influenced by the degree of matching between donor and recipient
for antigens encoded by genes of the major histocompatibility complex.

The human leukocyte antigen (HLA) molecules are responsible for binding antigenic proteins and presenting
them to T cells. The antigens presented by HLA molecules may derive from exogenous sources (e.g., during
active infections) or may be endogenous proteins. If individuals are not HLA-matched, T cells from one
individual will react strongly to the mismatched HLA, or "major antigens," of the second. Even if the individuals
are HLA-matched, the T cells of the donor may react to differing endogenous, or "minor antigens," presented
by the HLA of the recipient. Reactions to minor antigens tend to be less vigorous. The genes of major
relevance to transplantation include HLA-A, -B, -C, and -D; they are closely linked and therefore tend to be
inherited as haplotypes, with only rare crossovers between them. Thus, the odds that any one full sibling will
match a patient are one in four, and the probability that the patient has an HLA-identical sibling is 1 (0.75) n
, where n equals the number of siblings.

With current techniques, the risk of graft rejection is 13%, and the risk of severe, life-threatening acute
GVHD is ~15% following transplantation between HLA-identical siblings. The incidence of graft rejection and
GVHD increases progressively with the use of family member donors mismatched for one, two, or three
antigens. While survival following a one-antigen mismatched transplant is not markedly altered, survival
following two- or three-antigen mismatched transplants is significantly reduced, and such transplants should
be performed only as part of clinical trials.

Since the formation of the National Marrow Donor Program, it has become possible to identify HLA-matched
unrelated donors for many patients. The genes encoding HLA antigens are highly polymorphic, and thus the
odds of any two unrelated individuals being HLA-identical are extremely low, somewhat less than 1 in 10,000.
However, by identifying and typing >7 million volunteer donors, HLA-matched donors can now be found for
~50% of patients for whom a search is initiated. It takes, on average, 34 months to complete a search and
schedule and initiate an unrelated donor transplant. Results so far suggest that GVHD is somewhat increased
and survival somewhat poorer with such donors than with HLA-matched siblings.

Autologous transplantation involves the removal and storage of the patient's own stem cells with subsequent
reinfusion after the patient receives high-dose myeloablative therapy. Unlike allogeneic transplantation, there
is no risk of GVHD or graft rejection with autologous transplantation. On the other hand, autologous
transplantation lacks a graft-versus-tumor (GVT) effect, and the autologous stem cell product can be
contaminated with tumor cells that could lead to relapse. A variety of techniques have been developed to
"purge" autologous products of tumor cells. Some use antibodies directed at tumor-associated antigens plus
complement, antibodies linked to toxins, or antibodies conjugated to immunomagnetic beads. In vitro
incubation with certain chemotherapeutic agents such as 4-hydroperoxycyclophosphamide and long-term
culture of bone marrow have also been shown to diminish tumor cell numbers in stem cell products. Another
technique is positive selection of stem cells using antibodies to CD34, with subsequent column adherence or
flow techniques to select normal stem cells while leaving tumor cells behind. All these approaches can reduce
the number of tumor cells from 1000- to 10,000-fold and are clinically feasible; however, no prospective
randomized trials have yet shown that any of these approaches results in a decrease in relapse rates or
improvements in disease-free or overall survival.

Bone marrow aspirated from the posterior and anterior iliac crests has traditionally been the source of
hematopoietic stem cells for transplantation. Typically, anywhere from 1.5 to 5 x 10 8 nucleated marrow cells
per kilogram are collected for allogeneic transplantation. Several studies have found improved survival in the
http://online5.hsls.pitt.edu:2206/popup.aspx?aID=2866250&print=yes_chapter Page 2 of 15
AccessMedicine | Print: Chapter 108. Hematopoietic Cell Transplantation 12/10/08 10:16 PM

per kilogram are collected for allogeneic transplantation. Several studies have found improved survival in the
settings of both matched sibling and unrelated transplantation by transplanting higher numbers of bone
marrow cells.

Hematopoietic stem cells circulate in the peripheral blood but in very low concentrations. Following the
administration of certain hematopoietic growth factors, including granulocyte colony-stimulating factor (G-CSF)
or granulocyte-macrophage colony-stimulating factor (GM-CSF), and during recovery from intensive
chemotherapy, the concentration of hematopoietic progenitor cells in blood, as measured either by colony-
forming units or expression of the CD34 antigen, increases markedly. This has made it possible to harvest
adequate numbers of stem cells from the peripheral blood for transplantation. Donors are typically treated with
4 or 5 days of hematopoietic growth factor, following which stem cells are collected in one or two 4-h pheresis
sessions. In the autologous setting, transplantation of >2.5 x 10 6 CD34 cells per kilogram, a number easily
collected in most circumstances, leads to rapid and sustained engraftment in virtually all cases. Compared to
the use of autologous marrow, use of peripheral blood stem cells results in more rapid hematopoietic recovery,
with granulocytes recovering to 500/ L by day 12 and platelets recovering to 20,000/ L by day 14. While this
more rapid recovery diminishes the morbidity of transplantation, no studies show improved survival.

Hesitation in studying the use of peripheral blood stem cells for allogeneic transplantation was because
peripheral blood stem cell products contain as much as one log more T cells than are contained in the typical
marrow harvest; in animal models, the incidence of GVHD is related to the number of T cells transplanted.
Nonetheless, clinical trials have shown that the use of growth factormobilized peripheral blood stem cells from
HLA-matched family members leads to faster engraftment without an increase in acute GVHD. Chronic GVHD
may be increased with peripheral blood stem cells, but in trials conducted so far, this has been more than
balanced by reductions in relapse rates and nonrelapse mortality, with the use of peripheral blood stem cells
resulting in improved overall survival.

Umbilical cord blood contains a high concentration of hematopoietic progenitor cells, allowing for its use as a
source of stem cells for transplantation. Cord blood transplantation from family members has been explored in
the setting where the immediate need for transplantation precludes waiting the 9 or so months generally
required for the baby to mature to the point of donating marrow. Use of cord blood results in slower
engraftment and peripheral count recovery than seen with marrow but a low incidence of GVHD, perhaps
reflecting the low number of T cells in cord blood. Several banks have been developed to harvest and store
cord blood for possible transplantation to unrelated patients from material that would otherwise be discarded.
A summary of the first 562 unrelated cord blood transplants, facilitated by the New York Blood Center, reported
engraftment in ~85% of patients but at a slower pace than seen with marrow. Severe GVHD was seen in 23%
of patients. The risk of graft failure was related to the dose of cord blood cells per kilogram infused. The low
cell content of most cord blood collections has limited the use of this approach for adult patients.

THE TRANSPLANT PREPARATIVE REGIMEN


The treatment regimen administered to patients immediately preceding transplantation is designed to
eradicate the patient's underlying disease and, in the setting of allogeneic transplantation, immunosuppress the
patient adequately to prevent rejection of the transplanted marrow. The appropriate regimen therefore
depends on the disease setting and source of marrow. For example, when transplantation is performed to treat
severe combined immunodeficiency and the donor is a histocompatible sibling, no treatment is required
because no host cells require eradication and the patient is already too immunoincompetent to reject the
transplanted marrow. For aplastic anemia, there is no large population of cells to eradicate, and high-dose
cyclophosphamide plus antithymocyte globulin are sufficient to immunosuppress the patient adequately to

http://online5.hsls.pitt.edu:2206/popup.aspx?aID=2866250&print=yes_chapter Page 3 of 15
AccessMedicine | Print: Chapter 108. Hematopoietic Cell Transplantation 12/10/08 10:16 PM

cyclophosphamide plus antithymocyte globulin are sufficient to immunosuppress the patient adequately to
accept the marrow graft. In the setting of thalassemia and sickle cell anemia, high-dose busulfan is frequently
added to cyclophosphamide in order to eradicate hyperplastic host hematopoiesis. A variety of different
regimens have been developed to treat malignant diseases. Most of these regimens include agents that have
high activity against the tumor in question at conventional doses and have myelosuppression as their
predominant dose-limiting toxicity. Therefore, these regimens commonly include busulfan, cyclophosphamide,
melphalan, thiotepa, carmustine, etoposide, and total-body irradiation in various combinations.

Although high-dose treatment regimens have typically been used in transplantation, the understanding that
much of the antitumor effect of transplantation derives from an immunologically mediated GVT response has
led investigators to ask if less-intensive "nonmyeloablative" regimens might be effective and more tolerable.
Evidence for a GVT effect comes from studies showing that posttransplant relapse rates are lowest in patients
who develop acute and chronic GVHD, higher in those without GVHD, and higher still in recipients of T cell
depleted allogeneic or syngeneic marrow. The demonstration that complete remissions can be obtained in
many patients who have relapsed posttransplant by simply administering viable lymphocytes from the original
donor further strengthens the argument for a potent GVT effect. Accordingly, a variety of less-intensive
nonmyeloablative regimens have been studied, ranging in intensity from the very minimum required to achieve
engraftment (e.g., fludarabine plus 200 cGy total-body irradiation) to regimens of more immediate intensity
(e.g., fludarabine plus melphalan). Studies to date document that engraftment can be readily achieved with
less toxicity than seen with conventional transplantation. Furthermore, the severity of GVHD appears to be
decreased because less tissue damage is done by the lower doses of drugs in the preparative regimen.
Complete sustained responses have been documented in many patients, particularly those with more indolent
hematologic malignancies. The role of nonmyeloablative transplants in any disease, however, has not been
fully defined.

The Transplant Procedure


Marrow is usually collected from the donor's posterior and sometimes anterior iliac crests with the donor under
general or spinal anesthesia. Typically, 1015 mL/kg of marrow is aspirated, placed in heparinized media, and
filtered through 0.3- and 0.2-mm screens to remove fat and bony spicules. The collected marrow may undergo
further processing depending on the clinical situation, such as the removal of red cells to prevent hemolysis in
ABO-incompatible transplants, the removal of donor T cells to prevent GVHD, or attempts to remove possible
contaminating tumor cells in autologous transplantation. Marrow donation is safe, with only very rare
complications reported.

Peripheral blood stem cells are collected by leukophoresis after the donor has been treated with hematopoietic
growth factors or, in the setting of autologous transplantation, sometimes after treatment with a combination
of chemotherapy and growth factors. Stem cells for transplantation are generally infused through a large-bore
central venous catheter. Such infusions are usually well tolerated, although occasionally patients develop fever,
cough, or shortness of breath. These symptoms usually resolve with slowing of the infusion. When the stem
cell product has been cryopreserved using dimethyl sulfoxide, patients more often experience short-lived
nausea or vomiting due to the odor and taste of the cryoprotectant.

Engraftment
Peripheral blood counts usually reach their nadir several days to a week posttransplant as a consequence of
the preparative regimen, then cells produced by the transplanted stem cells begin to appear in the peripheral
blood. The rate of recovery depends on the source of stem cells, the use of posttransplant growth factors, and
the form of GVHD prophylaxis employed. If marrow is the source of stem cells, recovery to 100 granulocytes/
http://online5.hsls.pitt.edu:2206/popup.aspx?aID=2866250&print=yes_chapter Page 4 of 15
AccessMedicine | Print: Chapter 108. Hematopoietic Cell Transplantation 12/10/08 10:16 PM

the form of GVHD prophylaxis employed. If marrow is the source of stem cells, recovery to 100 granulocytes/
L occurs by day 16 and to 500/ L by day 22. Use of G-CSFmobilized peripheral blood stem cells speeds the
rate of recovery by ~1 week when compared to marrow. Use of a myeloid growth factor (G-CSF or GM-CSF)
posttransplant can further accelerate recovery by 35 days, while use of methotrexate to prevent GVHD delays
engraftment by a similar period. Following allogeneic transplantation, engraftment can be documented using
fluorescence in situ hybridization of sex chromosomes if donor and recipient are sex-mismatched, HLA-typing if
HLA-mismatched, or restriction fragment length polymorphism analysis if sex- and HLA-matched.

Complications Following Hematopoietic Cell Transplant


EARLY DIRECT CHEMORADIOTOXICITIES
The transplant preparative regimens commonly used cause a spectrum of acute toxicities that vary according
to the specific regimen but frequently result in nausea, vomiting, and mild skin erythema (Fig. 108-1).
Regimens that include high-dose cyclophosphamide can result in hemorrhagic cystitis, which can usually be
prevented by bladder irrigation or with the sulfhydryl compound mercaptoethanesulfonate (MESNA); rarely,
acute hemorrhagic carditis is seen. Most preparative regimens will result in oral mucositis, which typically
develops 57 days posttransplant and often requires narcotic analgesia. Use of a patient-controlled analgesic
pump provides the greatest patient satisfaction and results in a lower cumulative dose of narcotic. Patients
begin losing their hair 56 days posttransplant and by 1 week are usually profoundly pancytopenic.

Figure 108-1

http://online5.hsls.pitt.edu:2206/popup.aspx?aID=2866250&print=yes_chapter Page 5 of 15
AccessMedicine | Print: Chapter 108. Hematopoietic Cell Transplantation 12/10/08 10:16 PM

Major syndromes complicating marrow transplantation. VOD, venoocclusive disease; GVHD, graft-versus-host
disease; HSV, herpes simplex virus; CMV, cytomegalovirus; VZV, varicella-zoster virus. The size of the shaded area
roughly reflects the risk of the complication.

Approximately 10% of patients will develop venoocclusive disease of the liver, a syndrome resulting from
direct cytotoxic injury to hepatic-venular and sinusoidal endothelium, with subsequent deposition of fibrin and
the development of a local hypercoagulable state. This chain of events results in the clinical symptoms of
tender hepatomegaly, ascites, jaundice, and fluid retention. These symptoms can develop any time during the
first month posttransplant, with the peak incidence at day 16. Predisposing factors include prior exposure to
intensive chemotherapy, pretransplant hepatitis of any cause, and use of more intense conditioning regimens.
The mortality of venoocclusive disease is ~30%, with progressive hepatic failure culminating in a terminal
hepatorenal syndrome. Both thrombolytic and antithrombotic agents, such as tissue plasminogen activator,
heparin, and prostaglandin E, have been studied as therapy, but none has proven of consistent major benefit
in controlled trials, and all have significant toxicity. Early studies with defibrotide, a polydeoxyribonucleotide,
seem encouraging.

Although most pneumonias developing posttransplant are caused by infectious agents, in ~5% of patients a
diffuse interstitial pneumonia will develop that is thought to be the result of direct toxicity of the preparative
regimen. Bronchoalveolar lavage typically shows alveolar hemorrhage, and biopsies are typically characterized
by diffuse alveolar damage, although some cases may have a more clearly interstitial pattern. High-dose
glucocorticoids are often used as treatment, although randomized trials testing their utility have not been
http://online5.hsls.pitt.edu:2206/popup.aspx?aID=2866250&print=yes_chapter Page 6 of 15
AccessMedicine | Print: Chapter 108. Hematopoietic Cell Transplantation 12/10/08 10:16 PM

glucocorticoids are often used as treatment, although randomized trials testing their utility have not been
reported.

LATE DIRECT CHEMORADIOTOXICITIES


Late complications of the preparative regimen include decreased growth velocity in children and delayed
development of secondary sex characteristics. These complications can be partly ameliorated with the use of
appropriate growth and sex hormone replacement. Most men become azoospermic, and most postpubertal
women will develop ovarian failure, which should be treated. Thyroid dysfunction, usually well compensated, is
sometimes seen. Cataracts develop in 1020% of patients and are most common in patients treated with total-
body irradiation and those who receive glucocorticoid therapy posttransplant for treatment of GVHD. Aseptic
necrosis of the femoral head is seen in 10% of patients and is particularly frequent in those receiving chronic
glucocorticoid therapy.

GRAFT-VERSUS-HOST DISEASE
GVHD is the result of allogeneic T cells that were either transferred with the donor's stem cell inoculum or
develop from it, reacting with antigenic targets on host cells. GVHD developing within the first 3 months
posttransplant is termed acute GVHD, while GVHD developing or persisting beyond 3 months posttransplant is
termed chronic GVHD. Acute GVHD most often first becomes apparent 24 weeks posttransplant and is
characterized by an erythematous maculopapular rash; persistent anorexia or diarrhea, or both; and by liver
disease with increased serum levels of bilirubin, alanine and aspartate aminotransferase, and alkaline
phosphatase. Since many conditions can mimic acute GVHD, diagnosis usually requires skin, liver, or
endoscopic biopsy for confirmation. In all these organs, endothelial damage and lymphocytic infiltrates are
seen. In skin, the epidermis and hair follicles are damaged; in liver, the small bile ducts show segmental
disruption; and in intestines, destruction of the crypts and mucosal ulceration may be noted. A commonly used
rating system for acute GVHD is shown in Table 108-1. Grade I acute GVHD is of little clinical significance,
does not affect the likelihood of survival, and does not require treatment. In contrast, grades II to IV GVHD
are associated with significant symptoms and a poorer probability of survival, and they require aggressive
therapy. The incidence of acute GVHD is higher in recipients of stem cells from mismatched or unrelated
donors, in older patients, and in patients unable to receive full doses of drugs used to prevent the disease.

Table 108-1 Clinical Staging and Grading of Acute Graft-versus-Host Disease

Clinical Stage Skin LiverBilirubin, mol/L Gut


(mg/dL)

1 Rash <25% body surface 3451 (23) Diarrhea 5001000


mL/d
2 Rash 2550% body 51103 (36) Diarrhea 10001500
surface mL/d
3 Generalized 103257 (615) Diarrhea >1500 mL/d
erythroderma
4 Desquamation and bullae >257 (> 15) Ileus

Overall Clinical Skin Stage Liver Stage Gut Stage


Grade

I 12 0 0

http://online5.hsls.pitt.edu:2206/popup.aspx?aID=2866250&print=yes_chapter Page 7 of 15
AccessMedicine | Print: Chapter 108. Hematopoietic Cell Transplantation 12/10/08 10:16 PM

II 13 1 1
III 13 23 23
IV 24 24 24

One general approach to the prevention of GVHD is the administration of immunosuppressive drugs early after
transplant. Combinations of methotrexate and either cyclosporine or tacrolimus are among the most effective
and widely used regimens. Prednisone, antiT cell antibodies, mycophenolate mofetil, and other
immunosuppressive agents have also been or are being studied in various combinations. A second general
approach to GVHD prevention is removal of T cells from the stem cell inoculum. While effective in preventing
GVHD, T cell depletion is associated with an increased incidence of graft failure and of tumor recurrence
posttransplant; as yet, little evidence suggests that T-cell depletion improves cure rates in any specific setting.

Despite prophylaxis, significant acute GVHD will develop in ~30% of recipients of stem cells from matched
siblings and in as many as 60% of those receiving stem cells from unrelated donors. The disease is usually
treated with glucocorticoids, antithymocyte globulin, or monoclonal antibodies targeted against T cells or T cell
subsets.

Between 20 and 50% of patients surviving >6 months after allogeneic transplantation will develop chronic
GVHD. The disease is more common in older patients, in recipients of mismatched or unrelated stem cells, and
in those with a preceding episode of acute GVHD. The disease resembles an autoimmune disorder with malar
rash, sicca syndrome, arthritis, obliterative bronchiolitis, and bile duct degeneration and cholestasis. Single-
agent prednisone or cyclosporine is standard treatment at present, although trials of other agents are under
way. In most patients, chronic GVHD resolves, but it may require 13 years of immunosuppressive treatment
before these agents can be withdrawn without the disease recurring. Because patients with chronic GVHD are
susceptible to significant infection, they should receive prophylactic trimethoprim-sulfamethoxazole, and all
suspected infections should be investigated and treated aggressively.

GRAFT FAILURE
While complete and sustained engraftment is usually seen posttransplant, occasionally marrow function either
does not return or, after a brief period of engraftment, is lost. Graft failure after autologous transplantation can
be the result of inadequate numbers of stem cells being transplanted, damage during ex vivo treatment or
storage, or exposure of the patient to myelotoxic agents posttransplant. Infections with cytomegalovirus (CMV)
or human herpes virus type 6 have also been associated with loss of marrow function. Graft failure after
allogeneic transplantation can also be due to immunologic rejection of the graft by immunocompetent host
cells. Immunologically based graft rejection is more common following use of less-immunosuppressive
preparative regimens, in recipients of T celldepleted stem cell products, and in patients receiving grafts from
HLA-mismatched donors.

Treatment of graft failure usually involves removing all potentially myelotoxic agents from the patient's
regimen and attempting a short trial of a myeloid growth factor. Persistence of lymphocytes of host origin in
allogeneic transplant recipients with graft failure indicates immunologic rejection. Reinfusion of donor stem
cells in such patients is usually unsuccessful unless preceded by a second immunosuppressive preparative
regimen. Standard preparative regimens are generally tolerated poorly if administered within 100 days of a
first transplant because of cumulative toxicities. However, use of regimens combining, for example, anti-CD3
antibodies with high-dose glucocorticoids, fludarabine plus low-dose total-body irradiation, or
cyclophosphamide plus antithymocyte globulin have been effective in some cases.

http://online5.hsls.pitt.edu:2206/popup.aspx?aID=2866250&print=yes_chapter Page 8 of 15
AccessMedicine | Print: Chapter 108. Hematopoietic Cell Transplantation 12/10/08 10:16 PM

INFECTION
Posttransplant patients, particularly recipients of allogeneic transplantation, require unique approaches to the
problem of infection. Early after transplantation, patients are profoundly neutropenic, and because the risk of
bacterial infection is so great, most centers initiate antibiotic treatment once the granulocyte count falls to
<500/ L. Fluconazole prophylaxis at a dose of 200400 mg/kg per day reduces the risk of candidal infections.
Patients seropositive for herpes simplex should receive acyclovir prophylaxis. One approach to infection
prophylaxis is shown in Table 108-2. Despite these prophylactic measures, most patients will develop fever and
signs of infection posttransplant. The management of patients who become febrile despite bacterial and fungal
prophylaxis is a difficult challenge and is guided by individual aspects of the patient and by the institution's
experience. The general problem of infection in the immunocompromised host is discussed in Chap. 126.

Table 108-2 Approach to Infection Prophylaxis in Allogeneic Transplant Recipients

Organism Approach

Bacterial Ceftazidime 2 g IV q8h while neutropenic


Fungal Fluconazole 400 mg PO qd to day 75 posttransplant
Pneumocystis Trimethoprim- 1 double-strength tablet PO bid 2 days/week until day 180 or off
carinii sulfamethoxazole immunosuppression
Viral
Herpes simplex Acyclovir 800 mg PO bid to day 30
Varicella zoster Acyclovir 800 mg PO bid to day 365
Cytomegalovirus Ganciclovir 5 mg/kg IV bid for 7 days, then 5 (mg/kg)/d 5 days/week to day
100

Once patients engraft, the incidence of bacterial infection diminishes; however, patients, particularly allogeneic
transplant recipients, remain at significant risk of infection. During the period from engraftment until about 3
months posttransplant, the most common causes of infection are gram-positive bacteria, fungi (particularly
Aspergillus) and viruses including CMV. CMV infection, which in the past was frequently seen and often fatal,
can be prevented in seronegative patients by the use of seronegative blood products. The use of ganciclovir,
either as prophylaxis beginning at the time of engraftment or initiated when CMV first reactivates as evidenced
by development of antigenemia, can significantly reduce the risk of CMV disease in seropositive patients.
Elimination of white blood cells from transfused blood products is another method to prevent CMV
transmission. Foscarnet is effective for some patients who develop CMV antigenemia or infection despite the
use of ganciclovir or who cannot tolerate the drug.

Pneumocystis jiroveci pneumonia, once seen in 510% of patients, can be prevented by treating patients with
oral trimethoprim-sulfamethoxazole for 1 week pretransplant and resuming the treatment once patients have
engrafted.

The risk of infection diminishes considerably beyond 3 months after transplant unless chronic GVHD develops,
requiring continuous immunosuppression. Most transplant centers recommend continuing trimethoprim-
sulfamethoxazole prophylaxis while patients are receiving any immunosuppressive drugs and also recommend
careful monitoring for late CMV reactivation. In addition, many centers recommend prophylaxis against
varicella zoster, using acyclovir for 1 year posttransplant.

http://online5.hsls.pitt.edu:2206/popup.aspx?aID=2866250&print=yes_chapter Page 9 of 15
AccessMedicine | Print: Chapter 108. Hematopoietic Cell Transplantation 12/10/08 10:16 PM

TREATMENT OF SPECIFIC DISEASES USING HEMATOPOIETIC CELL


TRANSPLANTATION
Nonmalignant Diseases: Treatment
IMMUNODEFICIENCY DISORDERS
By replacing abnormal stem cells with cells from a normal donor, hematopoietic cell transplantation can cure
patients of a variety of immunodeficiency disorders including severe combined immunodeficiency, Wiskott-
Aldrich syndrome, and Chdiak-Higashi syndrome. The widest experience has been with severe combined
immunodeficiency disease, where cure rates of 90% can be expected with HLA-identical donors and success
rates of 5070% have been reported using haplotype-mismatched parents as donors (Table 108-3).

Table 108-3 Estimated 5-Year Survival Rates Following Transplantation a

Disease Allogeneic, % Autologous, %

Severe combined immunodeficiency 90 N/A


Aplastic anemia 90 N/A
Thalassemia 90 N/A

Acute myeloid leukemia


First remission 5560 50
Second remission 40 30
Acute lymphocytic leukemia
First remission 50 40

Second remission 40 30
Chronic myeloid leukemia
Chronic phase 70 ID
Accelerated phase 40 ID
Blast crisis 15 ID

Chronic lymphocytic leukemia 50 ID


Myelodysplasia 45 ID
Multiple myeloma 30 35
Non-Hodgkin's lymphoma
First relapse/second remission 40 40

Hodgkin's disease
First relapse/second remission 40 50
Breast cancer
High-risk stage II N/A 70
Stage IV N/A 15

http://online5.hsls.pitt.edu:2206/popup.aspx?aID=2866250&print=yes_chapter Page 10 of 15
AccessMedicine | Print: Chapter 108. Hematopoietic Cell Transplantation 12/10/08 10:16 PM

aThese estimates are generally based on data reported by the International Bone Marrow Transplant Registry.
The analysis has not been reviewed by their Advisory Committee.

Note: N/A, not applicable; ID, insufficient data.

APLASTIC ANEMIA
Transplantation from matched siblings after a preparative regimen of high-dose cyclophosphamide and
antithymocyte globulin can cure up to 90% of patients <40 years with severe aplastic anemia. Results in older
patients and in recipients of mismatched family member or unrelated marrow are less favorable; therefore, a
trial of immunosuppressive therapy is generally recommended for such patients before considering
transplantation. Transplantation is effective in all forms of aplastic anemia including, for example, the
syndromes associated with paroxysmal nocturnal hemoglobinuria and Fanconi's anemia. Patients with Fanconi's
anemia are abnormally sensitive to the toxic effects of alkylating agents and so less intensive preparative
regimens must be used in their treatment (Chap. 102).

HEMOGLOBINOPATHIES
Marrow transplantation from an HLA-identical sibling following a preparative regimen of busulfan and
cyclophosphamide can cure 7090% of patients with thalassemia major. The best outcomes can be expected if
patients are transplanted before they develop hepatomegaly or portal fibrosis and if they have been given
adequate iron chelation therapy. Among such patients, the probabilities of 5-year survival and disease-free
survival are 95 and 90%, respectively. Although prolonged survival can be achieved with aggressive chelation
therapy, transplantation is the only curative treatment for thalassemia. Transplantation is being studied as a
curative approach to patients with sickle cell anemia. Two-year survival and disease-free survival rates of 90
and 80%, respectively, have been reported following matched sibling transplantation. Decisions about patient
selection and the timing of transplantation remain difficult, but transplantation represents a reasonable option
for younger patients who suffer repeated crises or other significant complications and who have not responded
to other interventions (Chap. 99).

OTHER NONMALIGNANT DISEASES


Theoretically, hematopoietic cell transplantation should be able to cure any disease that results from an inborn
error of the lymphohematopoietic system. Transplantation has been used successfully to treat congenital
disorders of white blood cells such as Kostmann's syndrome, chronic granulomatous disease, and leukocyte
adhesion deficiency. Congenital anemias such as Blackfan-Diamond anemia can also be cured with
transplantation. Infantile malignant osteopetrosis is due to an inability of the osteoclast to resorb bone, and
since osteoclasts derive from the marrow, transplantation can cure this rare inherited disorder.

Hematopoietic cell transplantation has been used as treatment for a number of storage diseases caused by
enzymatic deficiencies, such as Gaucher's disease, Hurler's syndrome, Hunter's syndrome, and infantile
metachromatic leukodystrophy. Transplantation for these diseases has not been uniformly successful, but
treatment early in the course of these diseases, before irreversible damage to extramedullary organs has
occurred, increases the chance for success.

Transplantation is being explored as a treatment for severe acquired autoimmune disorders. These trials are
based on studies demonstrating that transplantation can reverse autoimmune disorders in animal models and
on the observation that occasional patients with coexisting autoimmune disorders and hematologic
malignancies have been cured of both with transplantation.

http://online5.hsls.pitt.edu:2206/popup.aspx?aID=2866250&print=yes_chapter Page 11 of 15
AccessMedicine | Print: Chapter 108. Hematopoietic Cell Transplantation 12/10/08 10:16 PM

Malignant Diseases: Treatment


ACUTE LEUKEMIA
Allogeneic hematopoietic cell transplantation cures 1520% of patients who do not achieve complete response
from induction chemotherapy for acute myeloid leukemia (AML) and is the only form of therapy that can cure
such patients. Cure rates of 3035% are seen when patients are transplanted in second remission or in first
relapse. The best results with allogeneic transplantation are achieved when applied during first remission, with
disease-free survival rates averaging 5560%. Chemotherapy alone can cure a portion of AML patients, and so
the relative merits of transplanting all patients during first remission versus only transplanting very-high-risk
patients and those who relapse continue to be discussed. Autologous transplantation is also able to cure a
portion of patients with AML. The rates of disease recurrence with autologous transplantation are higher than
those seen after allogeneic transplantation, and cure rates are somewhat less.

Similar to patients with AML, adults with acute lymphocytic leukemia who do not achieve a complete response
to induction chemotherapy can be cured in 1520% of cases with immediate transplantation. Cure rates
improve to 3050% in second remission, and therefore transplantation can be recommended for adults who
have persistent disease after induction chemotherapy or who have subsequently relapsed. Transplantation in
first remission results in cure rates around 55%. While transplantation appears to offer a clear advantage over
chemotherapy for patients with high-risk disease, such as those with Philadelphia chromosomepositive
disease, debate continues about whether adults with standard-risk disease should be transplanted in first
remission or whether transplantation should be reserved until relapse. Autologous transplantation is associated
with a higher relapse rate but a somewhat lower risk of nonrelapse mortality when compared to allogeneic
transplantation. On balance, most experts recommend use of allogeneic stem cells if an appropriate donor is
available.

CHRONIC LEUKEMIA
Allogeneic hematopoietic cell transplantation is the only therapy shown to cure a substantial portion of patients
with chronic myeloid leukemia (CML). Five-year disease-free survival rates are 1520% for patients
transplanted for blast crisis, 2550% for accelerated-phase patients, and 6070% for chronic phase patients,
with cure rates as high as 80% at selected centers. Use of unrelated donors results in more GVHD and slightly
worse survival than seen with matched siblings, although 3-year disease-free survival rates of 70% have been
reported at some large centers. The timing of transplantation in CML has become more complicated with the
introduction of imatinib mesylate, a remarkably effective, relatively nontoxic oral agent. Even though imatinib
is not generally regarded as curative, given its favorable toxicity profile, most physicians favor its use as initial
therapy for CML, with transplantation being reserved for those who fail to achieve a complete cytogenetic
response with imatinib, relapse after an initial response, or are intolerant of the drug (Chap. 104).

Allogeneic transplantation has been used to only a limited extent for chronic lymphocytic leukemia, in large
part because of the chronic nature of the disease and because of the age profile of patients. With allogeneic
transplantation, complete remissions have been achieved in the majority of patients so far reported, with
disease-free survival rates of ~50% at 3 years. However, treatment-related mortality has been substantial,
and further follow-up is needed. Encouraging results have been seen using reduced intensity preparative
regimens before allogeneic transplantation.

MYELODYSPLASIA
Between 40 and 50% of patients with myelodysplasia appear to be cured with allogeneic transplantation.
Results are better among younger patients and those with less-advanced disease. However, some patients

http://online5.hsls.pitt.edu:2206/popup.aspx?aID=2866250&print=yes_chapter Page 12 of 15
AccessMedicine | Print: Chapter 108. Hematopoietic Cell Transplantation 12/10/08 10:16 PM

Results are better among younger patients and those with less-advanced disease. However, some patients
with myelodysplasia can live for extended periods without intervention, and so transplantation is generally
recommended only for patients with disease categorized as intermediate risk I or greater according to the
International Prognostic Scoring System (Chap. 102).

LYMPHOMA
Patients with disseminated intermediate- or high-grade non-Hodgkin's lymphoma who have not been cured by
first-line chemotherapy and are transplanted in first relapse or second remission can still be cured in 4050%
of cases. This represents a clear advantage over results obtained with conventional-dose salvage
chemotherapy. It is unsettled whether patients with high-risk disease benefit from transplantation in first
remission. Most experts favor the use of autologous rather than allogeneic transplantation for patients with
intermediate or high grade non-Hodgkin's lymphoma, because fewer complications occur with this approach
and survival appears equivalent. For patients with recurrent disseminated indolent non-Hodgkin's lymphoma,
autologous transplantation results in high response rates and improved progression-free survival compared to
salvage chemotherapy. However, late relapses are seen after transplantation. The role of autologous
transplantation in the initial treatment of patients is under study. Nonmyeloablative preparative regimens
followed by allogeneic transplantation result in high response rates in patients with indolent lymphomas, but
the exact role of this approach remains to be defined.

The role of transplantation in Hodgkin's disease is similar to that in intermediate- and high-grade non-
Hodgkin's lymphoma. With transplantation, 5-year disease-free survival is 2030% in patients who never
achieve a first remission with standard chemotherapy and up to 70% for those transplanted in second
remission. Transplantation has no defined role in first remission in Hodgkin's disease.

MYELOMA
Patients with myeloma who have progressed on first-line therapy can sometimes benefit from allogeneic or
autologous transplantation. Autologous transplantation has been studied as part of the initial therapy of
patients, and both disease-free survival as well as overall survival were improved with this approach in
randomized trials. The use of autologous transplantation followed by nonmyeloablative allogeneic
transplantation has shown encouraging results.

SOLID TUMORS
Among women with metastatic breast cancer, 1520% disease-free survival rates at 3 years have been
reported, with better results seen in younger patients who have responded completely to standard-dose
therapy before undergoing transplantation. Randomized trials have not shown superior survival for patients
treated for metastatic disease with high-dose chemotherapy plus stem cell support. Randomized trials
evaluating transplantation as treatment for primary breast cancer have yielded mixed results. No role for
autologous transplantation has been established in the treatment of breast cancer.

Patients with testicular cancer who have failed first-line chemotherapy have been treated with autologous
transplantation; ~1020% of such patients apparently have been cured with this approach.

The use of high-dose chemotherapy with autologous stem cell support is being studied for several other solid
tumors, including ovarian cancer, small cell lung cancer, neuroblastoma, and pediatric sarcomas. As in most
other settings, the best results have been obtained in patients with limited amounts of disease and where the
remaining tumor remains sensitive to conventional-dose chemotherapy. Few randomized trials of
transplantation in these diseases have been completed.

http://online5.hsls.pitt.edu:2206/popup.aspx?aID=2866250&print=yes_chapter Page 13 of 15
AccessMedicine | Print: Chapter 108. Hematopoietic Cell Transplantation 12/10/08 10:16 PM

Partial and complete responses have been reported following nonmyeloablative allogeneic transplantation for
some solid tumors, most notably renal cell cancers. The GVT effect, well documented in the treatment of
hematologic malignancies, may apply to selected solid tumors under certain circumstances.

POSTTRANSPLANT RELAPSE
Patients who relapse following autologous transplantation sometimes respond to further chemotherapy,
particularly if the remission following transplantation was long. More options are available for patients who
relapse following allogeneic transplantation. Of particular interest are the response rates seen with infusion of
unirradiated donor lymphocytes. Complete responses in as many as 75% of patients with chronic myeloid
leukemia, 40% in myelodysplasia, 25% in AML, and 15% in myeloma have been reported. Major complications
of donor lymphocyte infusions include transient myelosuppression and the development of GVHD. These
complications depend on the number of donor lymphocytes given and the schedule of infusions, with less GVHD
seen with lower dose, fractionated schedules.

FURTHER READINGS
Appelbaum FR: Haematopoietic cell transplantation as immunotherapy. Nature 411:385, 2001 [PMID:
11357147]

Baron F, Storb R: Hematopoietic stem cell transplantation after reduced-intensity conditioning for older adults
with acute myeloid leukemia. Curr Opin Hematol 14:145, 2007 [PMID: 17255792]

Bensinger WI et al: Transplantation of bone marrow as compared with peripheral-blood cells from HLA-identical
relatives in patients with hematologic cancers. N Engl J Med 344:175, 2001 [PMID: 11172139]

Copelan EA: Hematopoietic stem-cell transplantation. N Engl J Med 354:1813, 2006 [PMID: 16641398]

Petersdorf EW et al: Major-histocompatibility-complex class I alleles and antigens in hematopoietic-cell


transplantation. N Engl J Med 345:1794, 2001 [PMID: 11752355]

BIBLIOGRAPHY
Child JA et al: High-dose chemotherapy with hematopoietic stem-cell rescue for multiple myeloma. N Engl J
Med 348:1875, 2003. [PMID: 12736280]

Grunebaum Eet al: Bone marrow transplantation for severe combined immune deficiency. JAMA 295:508, 2006
[PMID: 16449616]

Rubinstein P et al: Outcomes among 562 recipients of placental-blood transplants from unrelated donors. N
Engl J Med 339:1565, 1998 [PMID: 9828244]

Schmitz N et al: Aggressive conventional chemotherapy compared with high-dose chemotherapy with
autologous haemopoietic stem-cell transplantation for relapsed chemosensitive Hodgkin's disease: A
randomized trial. Lancet 359:2065, 2002 [PMID: 12086759]

Schouten HC et al: High-dose therapy improves progression-free survival and survival in relapsed follicular
non-Hodgkin's lymphoma: Results from the randomized European CUP trial. J Clin Oncol 21:3918, 2003 [PMID:
14517188]
http://online5.hsls.pitt.edu:2206/popup.aspx?aID=2866250&print=yes_chapter Page 14 of 15
AccessMedicine | Print: Chapter 108. Hematopoietic Cell Transplantation 12/10/08 10:16 PM

14517188]

Syrjala KL et al: Recovery and long-term function after hematopoietic cell transplantation for leukemia or
lymphoma. JAMA 291:2335, 2004 [PMID: 15150205]

Copyright The McGraw-Hill Companies. All rights reserved.


Privacy Notice. Any use is subject to the Terms of Use and Notice.

http://online5.hsls.pitt.edu:2206/popup.aspx?aID=2866250&print=yes_chapter Page 15 of 15
Genetic Origins of Human Acute Leukemia
Robert L. Redner MD
Acute Leukemias Pro-B Cell
(cIg-)
Pre-B Cell
(cIg+)
B Cell
(sIg+) Plasma Cell

To understand the pathophysiology of acute Lymphoid


Stem Cell
leukemias Pro-T Cell Pre-T Cell T Cell

To recognize the different treatment strategies


employed for AML, APL, and ALL, Pluripotent Erythrocytes
Stem Cell Erythroblast
Proerythroblast
To recognize the major prognostic factors that
determine treatment outcome Progranulocyte Granuloocyte
Myeloid Myeloblast
Keywords: AML, ALL, APL, induction Stem Cell
chemotherapy, remission AT Look Promonocyte Monocyte

DFCI Megakaryoblast

2003
Promegakaryocyte Platelets

Flt3 Receptor Ras Mutations in 15-30% of AML

Internal Tandem
Duplication
15-30% of AML

Proliferation of Leukemic Cells


in Marrow
Normal Leukemic

QuickTime and a TIFF (Uncompressed) decompressor are needed to see this picture. QuickTime and a TIFF (Uncompressed) decompressor are needed to see this picture.

1
Monotony of blasts Diagnosis
Bone marrow
Normal Leukemic
20% blasts
Chloroma
QuickTime and a TIFF (Uncompressed) decompressor are needed to see this picture.
Auer rods
QuickTime and a TIFF (Uncompressed) decompressor are needed to see this picture.

Genetic Origins of Human Acute Leukemia


Auer Rods
Pro-B Cell Pre-B Cell B Cell
(cIg-) (cIg+) (sIg+) Plasma Cell

Lymphoid
Stem Cell

Pro-T Cell Pre-T Cell T Cell

Pluripotent Erythrocytes
Stem Cell Erythroblast
Proerythroblast

Progranulocyte Granuloocyte
Myeloid Myeloblast
Stem Cell

Promonocyte Monocyte
AT Look
DFCI Megakaryoblast

2003
Promegakaryocyte Platelets

Clinical Presentation Clinical Presentation


anemia: fatigue, pallor DIC-disseminated intravascular coagulation
in M3 (promyelocytic)
leukopenia: infection
organ infiltration: skin nodules, gingival
thrombocytopenia: mucosal bleeding, hypertrophy, lymphadenopathy,
petechiae splenomegaly (more common in M4 and M5
abnormal monocytes)

2
Clinical Presentation Incidence of Acute Leukemia
hyperleukocytosis: >100,000 myeloblasts/ul.
with leukostasis; vascular injury and hypoxia; a
medical emergency requiring leukaphoresis
hypermetabolic: high uric acid; tumor lysis
syndrome (high phosphate, potassium, and uric
acid levels in blood from dying cells) a
medical emergency

Risk Factors Pro-B Cell Pre-B Cell B Cell


(cIg-) (cIg+) (sIg+) Plasma Cell

Cause unknown for most patients Lymphoid


Stem Cell
Exposure to radiation or other mutagens
Pro-T Cell Pre-T Cell T Cell
Atomic war, medicinal or diagnositic radiation
Chemotherapy esp. alkylating agents
Pluripotent Erythrocytes
Environmental chemical exposure Stem Cell Proerythroblast
Erythroblast

Inherited defects in DNA repair


Fanconi anemia, Blooms syndrome, Ataxia- Progranulocyte Granuloocyte
telangiectasia, Li-Fraumeni Myeloid Myeloblast
Stem Cell
Constitutional chromosomal abnormalities
Promonocyte Monocyte
trisomy 8, monosomy 7 AT Look

Preexisting myelodysplastic or myeloproliferative DFCI Megakaryoblast

disorder 2003
Promegakaryocyte Platelets

FAB classification of AML


M0 Undifferentiated
M1 Myeloblastic
M2 Myeloblastic with differentiation
M3 Promyelocytic
M4 Myelomonocytic
M5 Monocytic
M6 Erythroleukemia
M7 Megakaryocytic

3
Results with Chemotherapy

Cytogenetics in AML Treatment


Good prognostic Poor prognostic
factors factors:
inversion 16 loss of part or all
of 5 or 7
t(8;2 1)
QuickTime and a TIFF (Uncompressed) decompressor are needed to see this picture.

multiple
t(15;17) abnormalities
(Promyelocytic) 11q23 (seen in
therapy related
leukemia esp. VP-
16)

AML-Induction Therapy Pro-B Cell Pre-B Cell B Cell


(cIg-) (cIg+) (sIg+) Plasma Cell

Ara-C plus anthracycline Lymphoid


Stem Cell

Pro-T Cell Pre-T Cell T Cell


Nonspecific killing of dividing cells
skin Pluripotent
Erythroblast Erythrocytes
Gastrointestinal Stem Cell Proerythroblast

Normal hematopoietic cells


Progranulocyte Granuloocyte
Myeloid Myeloblast
Stem Cell

Promonocyte Monocyte
AT Look
DFCI Megakaryoblast

2003
Promegakaryocyte Platelets

4
Chemotherapy kills marrow
cells

QuickTime and a TIFF (Uncompressed) decompressor are needed to see this picture.

Remission

is NOT the same as a INDUCTION CHEMOTHERAPY


to induce a remission
Cure CONSOLIDATION CHEMOTHERAPY
1012 to consolidate the remission
usually four cycles of Ara-C

Results with Chemotherapy Stem Cell Transplant

QuickTime and a TIFF (Uncompressed) decompressor are needed to see this picture.

5
Myelotarg Differentiation Therapy

QuickTime and a TIFF (Uncompressed) decompressor are needed to see this picture.

CD33
Expressed on 80% AML
Expressed on normal committed
progenitors
Not on hematopietic stem cells
Not on nonhematologic cells

Differentiation Therapy ATRA in APL: Transcriptional Targeting

ncompressed)
compressor are

FAB classification of AML APL Survival


M0 Undifferentiated
M1 Myeloblastic
M2 Myeloblastic with differentiation
M3 Promyelocytic
M4 Myelomonocytic
M5 Monocytic
M6 Erythroleukemia
M7 Megakaryocytic

6
Acute Lymphocytic Leukemia ALL Presentation

Leukocytosis
Lymphadenopathy
Spelenomegaly
Mediastinal Mass

Protected Sanctuary Sites for


ALL
ALL Blasts
Central Nervous System Testicles
Precursor B rarely t(9;22)
T-lineage
Mature B (Burkitts) t(8;14) or t(8;22)

ALL Treatment ALL Treatment


Induction Vincristine/Prednisone
multiple drugs Dependent on Age Cure
Consolidation multiple drugs
CNS prophylaxis Childhood >90%
Maintenance monthly for two years Young Adult 40%
Old Adult 10%

7
Acute Myeloid Leukemia

Incidence
The incidence of acute myeloid leukemia (AML) is ~3.7 per 100,000 people per year, and the
age-adjusted incidence is higher in men than in women (4.6 versus 3.0). AML incidence
increases with age; it is 1.9 in individuals <65 years and 18.6 in those >65. A significant
increase in AML incidence has occurred over the past 10 years.

Etiology
Heredity, radiation, chemical and other occupational exposures, and drugs have been
implicated in the development of AML. No direct evidence suggests a viral etiology.

HEREDITY
Certain syndromes with somatic cell chromosome aneuploidy, such as trisomy 21 noted in
Down syndrome, are associated with an increased incidence of AML. Inherited diseases with
defective DNA repair, e.g., Fanconi anemia, Bloom syndrome, and ataxia telangiectasia, are
also associated with AML.

Radiation
Survivors of the atomic bomb explosions in Japan had an increased incidence of myeloid
leukemias that peaked 57 years after exposure. Therapeutic radiation alone seems to add
little risk of AML but can increase the risk in people also exposed to alkylating agents.

CHEMICAL AND OTHER EXPOSURES


Exposure to benzene, a solvent used in the chemical, plastic, rubber, and pharmaceutical
industries, is associated with an increased incidence of AML. Smoking and exposure to
petroleum products, paint, embalming fluids, ethylene oxide, herbicides, and pesticides, have
also been associated with an increased risk of AML.

DRUGS
Anticancer drugs are the leading cause of therapy-associated AML. Alkylating agent
associated leukemias occur on average 46 years after exposure, and affected individuals
have aberrations in chromosomes 5 and 7. Topoisomerase II inhibitorassociated leukemias
occur 13 years after exposure, and affected individuals often have aberrations involving
chromosome 11q23. Chloramphenicol, phenylbutazone, and, less commonly, chloroquine and
methoxypsoralen can result in bone marrow failure that may evolve into AML.

Classification
The World Health Organization (WHO) classification includes different biologically distinct
groups based on immunophenotype, clinical features, and cytogenetic and molecular
abnormalities in addition to morphology. In contrast to the previously used French-American-
British (FAB) schema, the WHO classification places limited reliance on cytochemistry. A major
difference between the WHO and FAB systems is the blast cutoff for a diagnosis of AML as
opposed to myelodysplastic syndrome (MDS); it is 20% in the WHO classification and 30% in
the FAB. Importantly, the WHO schema is the first leukemia classification system to consider
genetic along with morphologic features to define different subsets of AML.

IMMUNOPHENOTYPE AND RELEVANCE TO THE WHO


CLASSIFICATION
The immunophenotype of human leukemia cells can be studied by multiparameter flow
cytometry after the cells are labeled with monoclonal antibodies to cell-surface antigens. This
can be important for separating AML from acute lymphoblastic leukemia (ALL) and identifying
some types of AML. For example, AML that is minimally differentiated (immature morphology
and no lineage-specific cytochemical reactions) is diagnosed by flow-cytometric demonstration
of the myeloid-specific antigens cluster designation (CD) 13 or 33. Similarly, acute
megakaryoblastic leukemia can often be diagnosed only by expression of the platelet-specific
antigens CD41 and/or CD61. While flow cytometry is useful, widely used, and, in some cases,
essential for the diagnosis of AML, it is only supportive in establishing the different subtypes of
AML through the WHO classification.

CLINICAL FEATURES AND RELEVANCE TO THE WHO


CLASSIFICATION
The WHO classification considers clinical features in subdividing AML. For example, it identifies
therapy-related AML as a separate entity and subclassifies this group based on the specific
types of prior chemotherapy received. It also divides AML with multilineage dysplasia based
upon the presence or absence of an antecedent MDS. These clinical features contribute to the
prognosis of the specific type of AML.

GENETIC FINDINGS AND RELEVANCE TO THE WHO


CLASSIFICATION
The WHO classification is the first AML classification to incorporate genetic (chromosomal and
molecular) information. Indeed, AML is first subclassified based on the presence or absence of
specific recurrent genetic abnormalities. For example, AML FAB M3 is now designated acute
promyelocytic leukemia (APL), based on the presence of either the t(15;17)(q22;q12)
cytogenetic rearrangement or the PML/RAR product of the translocation. Thus, the WHO
classification separates APL from all other types of AML as a first step and forces the clinician
to correctly identify the entity and tailor treatment(s) accordingly.

Chromosomal Analyses
Chromosomal analysis of the leukemic cell provides the most important pretreatment
prognostic information in AML. Two cytogenetic abnormalities have been invariably associated
with specific morphologic features: t(l5;17)(q22;q12) with APL and inv(16)(p13q22) with AML
with abnormal bone marrow eosinophils. Many other chromosomal abnormalities have been
associated primarily with one morphologic/immunophenotypic group, including
t(8;21)(q22;q22) with slender Auer rods, expression of CD19, and abundance of normal
eosinophils, and t(9;11)(p22;q23), as well as other translocations involving 11q23, with
monocytic features. Many of the recurring chromosomal abnormalities in AML have been
associated with specific clinical characteristics. More commonly associated with younger age
are t(8;21) and t(l5;17); with older age, del(5q) and del(7q). Myeloid sarcomas (see below)
are associated with t(8;21) and disseminated intravascular coagulation (DIC) with t(15;17).

MOLECULAR CLASSIFICATION
Molecular study of many recurring cytogenetic abnormalities has revealed genes that may be
involved in leukemogenesis; this information is increasingly being incorporated into the WHO
classification. For instance, the t(15;17) encodes a chimeric protein, promyelocytic leukemia
(Pml)/retinoic acid receptor (Rar ), which is formed by the fusion of the retinoic acid
receptor (RAR ) gene from chromosome 17 and the promyelocytic leukemia (PML) gene
from chromosome 15. The RAR gene encodes a member of the nuclear hormone receptor
family of transcription factors. After binding retinoic acid, RAR can promote expression of a
variety of genes. The 15;17 translocation juxtaposes PML with RAR in a head-to-tail
configuration that is under the transcriptional control of PML. Three different breakpoints in
the PML gene lead to various fusion proteins. The Pml-Rar fusion protein tends to suppress
gene transcription and blocks differentiation of the cells. Pharmacologic doses of the Rar
ligand, all-trans-retinoic acid (tretinoin), relieve the block and promote differentiation (see
below). Similar examples exist with a variety of other balanced translocations and inversions,
including the t(8;21), t(9;11), t(6;9), and inv(16).

Molecular aberrations are also being identified that are useful for classifying risk of relapse in
patients without cytogenetic abnormalities. A partial tandem duplication (PTD) of the
MLL gene is found in 510% of patients with normal cytogenetics and results in short
remission duration. FMS-like tyrosine kinase 3 (Flt3) is a tyrosine kinase receptor important in
the development of myeloid and lymphoid lineages. Activating mutations of the gene FLT3 are
present in ~30% of adult AML patients due to internal tandem duplications (ITDs) in the
juxtamembrane domain or mutations of the activating loop of the kinase. These occur more
commonly in patients with normal karyotype. Continuous activation of Flt3 and downstream
target kinases, including signal transducer and activator of transcription protein 5,
Ras/mitogen-activated protein kinase, and phosphatidylinositol 3-kinase/Akt, provides
increased proliferation and antiapoptotic signals to the myeloid progenitor cell. Presence of
FLT3 ITD in patients with normal cytogenetics predicts for short remission duration and
inferior survival. Other molecular prognostic factors in patients with normal karyotype AML
include mutations of the nucleophosmin gene (NPM1) and C/EBP that are associated with
improved treatment outcome. In contrast, overexpression of genes such as brain and acute
leukemia, cytoplasmic (BAALC) predicts for poor outcome. Gene expression profiles to predict
outcome in normal karyotype AML patients are under active investigation.

Clinical Presentation
SYMPTOMS
Patients with AML most often present with nonspecific symptoms that begin gradually or
abruptly and are the consequence of anemia, leukocytosis, leukopenia or leukocyte
dysfunction, or thrombocytopenia. Nearly half have had symptoms for 3 months before the
leukemia was diagnosed.

Half mention fatigue as the first symptom, but most complain of fatigue or weakness at the
time of diagnosis. Anorexia and weight loss are common. Fever with or without an identifiable
infection is the initial symptom in ~10% of patients. Signs of abnormal hemostasis (bleeding,
easy bruising) are noted first in 5% of patients. On occasion, bone pain, lymphadenopathy,
nonspecific cough, headache, or diaphoresis is the presenting symptom.

Rarely patients may present with symptoms from a mass lesion located in the soft tissues,
breast, uterus, ovary, cranial or spinal dura, gastrointestinal tract, lung, mediastinum,
prostate, bone, or other organs. The mass lesion represents a tumor of leukemic cells and is
called a granulocytic sarcoma, or chloroma. Typical AML may occur simultaneously, later, or
not at all in these patients. This rare presentation is more common in patients with t(8;21).

PHYSICAL FINDINGS
Fever, splenomegaly, hepatomegaly, lymphadenopathy, sternal tenderness, and evidence of
infection and hemorrhage are often found at diagnosis. Significant gastrointestinal bleeding,
intrapulmonary hemorrhage, or intracranial hemorrhage occur most often in APL. Bleeding
associated with coagulopathy may also occur in monocytic AML and with extreme degrees of
leukocytosis or thrombocytopenia in other morphologic subtypes. Retinal hemorrhages are
detected in 15% of patients. Infiltration of the gingivae, skin, soft tissues, or the meninges
with leukemic blasts at diagnosis is characteristic of the monocytic subtypes and those with
11q23 chromosomal abnormalities.

HEMATOLOGIC FINDINGS
Anemia is usually present at diagnosis and can be severe. The degree varies considerably,
irrespective of other hematologic findings, splenomegaly, or duration of symptoms. The
anemia is usually normocytic normochromic. Decreased erythropoiesis often results in a
reduced reticulocyte count, and red blood cell (RBC) survival is decreased by accelerated
destruction. Active blood loss also contributes to the anemia.

The median presenting leukocyte count is about 15,000/ L. Between 25 and 40% of patients
have counts <5000/ L, and 20% have counts >100,000/ L. Fewer than 5% have no
detectable leukemic cells in the blood. The morphology of the malignant cell varies in
difference subsets. In AML the cytoplasm often contains primary (nonspecific) granules, and
the nucleus shows fine, lacy chromatin with one or more nucleoli characteristic of immature
cells. Abnormal rod-shaped granules called Auer rods are not uniformly present, but when
they are, myeloid lineage is virtually certain . Poor neutrophil function may be noted by
impaired phagocytosis and migration and morphologically by abnormal lobulation and deficient
granulation. Platelet counts <100,000/ L are found at diagnosis in ~75% of patients, and
about 25% have counts <25,000/ L. Both morphologic and functional platelet abnormalities
can be observed, including large and bizarre shapes with abnormal granulation and inability of
platelets to aggregate or adhere normally to one another.

PRETREATMENT EVALUATION
Once the diagnosis of AML is suspected, a rapid evaluation and initiation of appropriate
therapy should follow. In addition to clarifying the subtype of leukemia, initial studies should
evaluate the overall functional integrity of the major organ systems, including the
cardiovascular, pulmonary, hepatic, and renal systems. Factors that have prognostic
significance, either for achieving complete remission (CR) or for predicting the duration of CR,
should also be assessed before initiating treatment. Leukemic cells should be obtained from all
patients and cryopreserved for future use as new tests and therapeutics become available. All
patients should be evaluated for infection.

Most patients are anemic and thrombocytopenic at presentation. Replacement of the


appropriate blood components, if necessary, should begin promptly. Because qualitative
platelet dysfunction or the presence of an infection may increase the likelihood of bleeding,
evidence of hemorrhage justifies the immediate use of platelet transfusion, even if the platelet
count is only moderately decreased.

About 50% of patients have a mild to moderate elevation of serum uric acid at presentation.
Only 10% have marked elevations, but renal precipitation of uric acid and the nephropathy
that may result is a serious but uncommon complication. The initiation of chemotherapy may
aggravate hyperuricemia, and patients are usually started immediately on allopurinol and
hydration at diagnosis. Rasburicase (recombinant uric oxidase) is also useful for treating uric
acid nephropathy and often can normalize the serum uric acid level within hours with a single
dose of treatment. The presence of high concentrations of lysozyme, a marker for monocytic
differentiation, may be etiologic in renal tubular dysfunction, which could worsen other renal
problems that arise during the initial phases of therapy.

Prognostic Factors
Many factors influence the likelihood of entering CR, the length of CR, and the curability of
AML. CR is defined after examination of both blood and bone marrow. The blood neutrophil
count must be 1000/ L and the platelet count 100,000/ L. Hemoglobin concentration is not
considered in determining CR. Circulating blasts should be absent. While rare blasts may be
detected in the blood during marrow regeneration, they should disappear on successive
studies. Bone marrow cellularity should be >20% with trilineage maturation. The bone
marrow should contain <5% blasts, and Auer rods should be absent. Extramedullary leukemia
should not be present. For patients in morphologic CR, reverse transcriptase polymerase chain
reaction (RT-PCR) to detect AML-associated molecular abnormalities and either metaphase
cytogenetics or interphase cytogenetics by fluorescence in situ hybridization (FISH) to detect
AML-associated cytogenetic aberrations are currently used to detect residual disease. Such
detection of minimal residual disease may become a reliable discriminator between patients in
CR who do or do not require additional and/or alternative therapies.

Age at diagnosis is among the most important risk factors. Advancing age is associated with a
poorer prognosis, in part because of its influence on the patient's ability to survive induction
therapy. Age also influences outcome because AML in older patients differs biologically. The
leukemic cells in elderly patients more commonly express CD34 and the multidrug resistance
1 (MDR1) efflux pump that conveys resistance to natural productderived agents such as the
anthracyclines (see below). With each successive decade of age, a greater proportion of
patients have more resistant disease. Chronic and intercurrent diseases impair tolerance to
rigorous therapy; acute medical problems at diagnosis reduce the likelihood of survival.
Performance status, independent of age, also influences ability to survive induction therapy
and thus respond to treatment.

Chromosome findings at diagnosis are important independent prognostic factors. Patients with
t(15;17) have a very good prognosis (approximately 85% cured), and those with t(8;21) and
inv(16) a good prognosis (approximately 50% cured), while those with no cytogenetic
abnormality have a moderately favorable outcome (approximately 40% cured). Patients with a
complex karyotype, t(6;9), inv(3), or 7 have a very poor prognosis. This emphasizes the
importance of cytogenetic as well as the previously discussed molecular assessment of the
leukemia cells at diagnosis and relevance of storing samples for potential later use.

A prolonged symptomatic interval with cytopenias preceding diagnosis or a history of an


antecedent hematologic disorder is another pretreatment clinical feature associated with a
lower CR rate and shorter survival time. The CR rate is lower in patients who have had
anemia, leukopenia, and/or thrombocytopenia for >3 months before the diagnosis of AML
when compared to those without such a history. Responsiveness to chemotherapy declines as
the duration of the antecedent disorder(s) increases. Secondary AML developing after
treatment with cytotoxic agents for other malignancies is usually difficult to treat successfully.

A high presenting leukocyte count is an independent prognostic factor for attaining a CR.
Among patients with hyperleukocytosis (>100,000/ L), early central nervous system bleeding
and pulmonary leukostasis contribute to poor outcome with initial therapy.

In addition to pretreatment variables such as age, cytogenetics, and leukocyte count, several
treatment factors correlate with prognosis in AML, including, most importantly, achievement of
CR. In addition, patients who achieve CR after one induction cycle have longer CR durations
than those requiring multiple cycles.

Acute Myeloid Leukemia: Treatment


Treatment of the newly diagnosed patient with AML is usually divided into two phases,
induction and postremission management. The initial goal is to quickly induce CR. Once CR is
obtained, further therapy must be used to prolong survival and achieve cure. The initial
induction treatment and subsequent postremission therapy are often chosen based on the
patient's age. The influence of intensifying therapy with traditional chemotherapy agents such
as cytarabine and anthracyclines in younger patients (<60 years) appears to increase the cure
rate of AML. In older patients the benefit of intensive therapy is controversial; novel therapies
are being pursued.

INDUCTION CHEMOTHERAPY
The most commonly used CR induction regimens (for patients other than those with APL)
consist of combination chemotherapy with cytarabine and an anthracycline. Cytarabine is a
cell cycle S-phasespecific antimetabolite that becomes phosphorylated intracellularly to an
active triphosphate form that interferes with DNA synthesis. Anthracyclines are DNA
intercalaters. Their primary mode of action is thought to be inhibition of topoisomerase II,
leading to DNA breaks. Cytarabine is usually administered as a continuous intravenous
infusion for 7 days. Anthracycline therapy generally consists of daunorubicin intravenously on
days 1, 2, and 3 (the 7 and 3 regimen). Treatment with idarubicin for 3 days in conjunction
with cytarabine by 7-day continuous infusion is at least as effective and may be superior to
daunorubicin in younger patients. The addition of etoposide may improve the CR duration.

After induction chemotherapy, the bone marrow is examined to determine if the leukemia has
been eliminated. If 5% blasts exist with 20% cellularity, the patient is usually re-treated
with cytarabine and an anthracycline in doses similar to those given initially, but for 5 and 2
days, respectively. Our recommendation, however, is to change therapy in this setting.
Patients who fail to attain CR after two induction courses should immediately proceed to an
allogeneic stem cell transplant (SCT) if an appropriate donor exists. This approach is only
applied to patients under the age of 70 with acceptable end-organ function.

With the 7 and 3 cytarabine/daunorubicin regimen outlined above, 6575% of adults with de
novo AML under the age of 60 years achieve CR. Two-thirds achieve CR after a single course
of therapy, and one-third require two courses. About 50% of patients who do not achieve CR
have a drug-resistant leukemia, and 50% do not achieve CR because of fatal complications of
bone marrow aplasia or impaired recovery of normal stem cells. Higher induction treatment
related mortality and frequency of resistant disease have been observed with increasing age
and in patients with prior hematologic disorders (MDS or myeloproliferative syndromes) or
chemotherapy treatment for another malignancy.

High-dose cytarabine-based regimens have very high CR rates after a single cycle of therapy.
When given in high doses, more cytarabine may enter the cells, saturate the cytarabine-
inactivating enzymes, and increase the intracellular levels of 1- -D-arabinofuranylcytosine-
triphosphate, the active metabolite incorporated into DNA. Thus, higher doses of cytarabine
may increase the inhibition of DNA synthesis and thereby overcome resistance to standard-
dose cytarabine. In two randomized studies, high-dose cytarabine with an anthracycline
produced CR rates similar to those achieved with standard 7 and 3 regimens. However, the CR
duration was longer after high-dose cytarabine than after standard-dose cytarabine.

The hematologic toxicity of high-dose cytarabine-based induction regimens has typically been
greater than that associated with 7 and 3 regimens. Toxicity with high-dose cytarabine
includes myelosuppression, pulmonary toxicity, and significant and occasionally irreversible
cerebellar toxicity. All patients treated with high-dose cytarabine must be closely monitored
for cerebellar toxicity. Full cerebellar testing should be performed before each dose, and
further high-dose cytarabine should be withheld if evidence of cerebellar toxicity develops.
This toxicity occurs more commonly in patients with renal impairment and in those over age
60. The increased toxicity observed with high-dose cytarabine has limited the use of this
therapy in elderly AML patients.

SUPPORTIVE CARE
Measures geared to supporting patients through several weeks of granulocytopenia and
thrombocytopenia are critical to the success of AML therapy. Patients with AML should be
treated in centers expert in providing supportive measures.

Recombinant hematopoietic growth factors have been incorporated into clinical trials in AML.
These trials have been designed to lower the infection rate after chemotherapy. Both G-CSF
and granulocyte-macrophage colony-stimulating factor (GM-CSF) have reduced the median
time to neutrophil recovery by an average of 57 days. This accelerated rate of neutrophil
recovery, however, has not generally translated into significant reductions in infection rates or
shortened hospitalizations. In most randomized studies, both G-CSF and GM-CSF have failed
to improve the CR rate, disease-free survival, or overall survival. Although receptors for both
G-CSF and GM-CSF are present on AML blasts, therapeutic efficacy is neither enhanced nor
inhibited by these agents. The use of growth factors as supportive care for AML patients is
controversial. We favor their use in elderly patients with complicated courses, those receiving
intensive postremission regimens, patients with uncontrolled infections, or those participating
in clinical trials.

Multilumen right atrial catheters should be inserted as soon as patients with newly diagnosed
AML have been stabilized. They should be used thereafter for administration of intravenous
medications and transfusions, as well as for blood drawing. Antibiotic-impregnated catheters
should be considered if the risk of line-related infection is high.

Adequate and prompt blood bank support is critical to therapy of AML. Platelet transfusions
should be given as needed to maintain a platelet count >10,00020,000/ L. We believe that
the platelet count should be kept at higher levels in febrile patients and during episodes of
active bleeding or DIC. Patients with poor posttransfusion platelet count increments may
benefit from administration of platelets from human leukocyte antigen (HLA)-matched donors.
RBC transfusions should be administered to keep the hemoglobin level >80 g/L (8 g/dL) in the
absence of active bleeding, DIC, or congestive heart failure. Blood products leukodepleted by
filtration should be used to avert or delay alloimmunization as well as febrile reactions. Blood
products should also be irradiated to prevent transfusion associated graft-versus-host disease
(GVHD). Cytomegalovirus (CMV)-negative blood products should be used for CMV-
seronegative patients who are potential candidates for allogeneic SCT. Leukodepleted products
are also effective for these patients if CMV-negative products are not available.

Infectious complications remain the major cause of morbidity and death during induction and
postremission chemotherapy for AML. Prophylactic administration of antibiotics in the absence
of fever is controversial. Oral nystatin or clotrimazole is recommended to prevent localized
candidiasis. For patients who are herpes simplex virus antibody titerpositive, acyclovir
prophylaxis is effective in preventing reactivation of latent oral herpes infections.

Fever develops in most patients with AML, but infections are documented in only half of febrile
patients. Early initiation of empirical broad-spectrum antibacterial and antifungal antibiotics
has significantly reduced the number of patients dying of infectious complications. An
antibiotic regimen adequate to treat gram-negative organisms should be instituted at the
onset of fever in a granulocytopenic patient after clinical evaluation, including a detailed
physical examination with inspection of the indwelling catheter exit site and a perirectal
examination, as well as procurement of cultures and radiographs aimed at documenting the
source of fever. Specific antibiotic regimens should be based on antibiotic sensitivity data
obtained from the institution at which the patient is being treated.

TREATMENT OF PROMYELOCYTIC LEUKEMIA


Tretinoin is an oral drug that induces the differentiation of leukemic cells bearing the t(15;17).
APL is responsive to cytarabine and daunorubicin, but about 10% of patients treated with
these drugs die from DIC induced by the release of granule components by dying tumor cells.
Tretinoin does not produce DIC but produces another complication called the retinoic acid
syndrome. Occurring within the first 3 weeks of treatment, it is characterized by fever,
dyspnea, chest pain, pulmonary infiltrates, pleural and pericardial effusions, and hypoxia. The
syndrome is related to adhesion of differentiated neoplastic cells to the pulmonary vasculature
endothelium. Glucocorticoids, chemotherapy, and/or supportive measures can be effective for
management of the retinoic acid syndrome. The mortality of this syndrome is about 10%.

Tretinoin (45 mg/m2 per day orally until remission is documented) plus concurrent
anthracycline chemotherapy appears to be among the safest and most effective treatments for
APL. Unlike patients with other types of AML, patients with this subtype benefit from
maintenance therapy with either tretinoin or chemotherapy.

Arsenic trioxide produces meaningful responses in up to 85% of patients refractory to


tretinoin. The use of arsenic trioxide is being explored as part of initial treatment in clinical
trials of APL. Additionally, studies combining arsenic trioxide with tretinoin in the absence of
chemotherapy are ongoing.

The detection of minimal residual disease by RT-PCR amplification of the t(15;17) chimeric
gene product appears to predict relapse. Disappearance of the signal is associated with long-
term disease-free survival; its persistence predicts relapse. With increases in the sensitivity of
the assay, some patients with persistent abnormal gene product have been found who do not
suffer a relapse. Studies are underway to determine whether a critical threshold level of
transcripts uniformly predicts for leukemia relapse.

POSTREMISSION THERAPY
Induction of a durable first CR is critical to long-term disease-free survival in AML. However,
without further therapy virtually all patients experience relapse. Once relapse has occurred,
AML is generally curable only by SCT.

Postremission therapy is designed to eradicate residual leukemic cells to prevent relapse and
prolong survival. Postremission therapy in AML is often based on age (younger than 5565
and older than 5565). For younger patients, most studies include intensive chemotherapy
and allogeneic or autologous SCT. High-dose cytarabine is more effective than standard-dose
cytarabine. The Cancer and Leukemia Group B (CALGB), for example, compared the duration
of CR in patients randomly assigned postremission to four cycles of high (3 g/m2, every 12 h
on days 1, 3, and 5), intermediate (400 mg/m2 for 5 days by continuous infusion), or
standard (100 mg/m2 per day for 5 days by continuous infusion) doses of cytarabine. A dose-
response effect for cytarabine in patients with AML who were 60 years was demonstrated.
High-dose cytarabine significantly prolonged CR and increased the fraction cured in patients
with favorable [t(8;21) and inv(16)] and normal cytogenetics, but it had no significant effect
on patients with other abnormal karyotypes. For older patients, exploration of attenuated
intensive therapy that includes either chemotherapy or reduced intensity allogeneic SCT has
been pursued. Postremission therapy is a setting for introduction of new agents

Allogeneic SCT is used in patients <70 years old with an HLA-compatible donor who have
high-risk cytogenetics. In the subset with normal cytogenetics and high-risk molecular
features such as FLT3 ITD, allogeneic SCT is best applied in the context of clinical trials, as the
impact of aggressive therapy on outcome is unknown. Relapse following allogeneic SCT occurs
in only a small fraction of patients, but toxicity is relatively high from treatment; complications
include venoocclusive disease, GVHD, and infections. Autologous transplantation can be
administered in young and older patients and uses the same preparative regimens. Patients
subsequently receive their own stem cells collected while in remission. The toxicity is lower
with autologous SCT (5% mortality rate), but the relapse rate is higher than with allogeneic
SCT, and randomized studies have not demonstrated outcome superior to postremission
conventional-dose chemotherapy. The increased relapse rate is due to the absence of the
graft-versus-leukemia (GVL) effect seen with allogeneic SCT and possible contamination of the
autologous stem cells with tumor cells. Purging tumor from the autologous stem cells has not
lowered the relapse rate with autologous SCT.

Randomized trials comparing intensive chemotherapy and autologous and allogeneic SCT have
shown improved duration of remission with allogeneic SCT compared to autologous SCT or
chemotherapy alone. However, overall survival is generally not different; the improved
disease control with allogeneic SCT is erased by the increase in fatal toxicity. While stem cells
were previously harvested from the bone marrow, virtually all efforts currently collect these
from the blood following mobilization regimens, including growth factors with or without
chemotherapy. Prognostic factors may help select patients in first CR for whom transplant is
most effective.

RELAPSE
Once relapse occurs, patients are rarely cured with further standard-dose chemotherapy.
Patients eligible for allogeneic SCT should receive transplants expeditiously at the first sign of
relapse. Long-term disease-free survival is approximately the same (3050%) with allogeneic
SCT in first relapse or in second remission. Autologous SCT rescues about 20% of relapsed
patients with AML who have chemosensitive disease. The most important factors predicting
response at relapse are the length of the previous CR, whether initial CR was achieved with
one or two courses of chemotherapy, and the type of postremission therapy.

Because of the poor outcome of patients in early first relapse (<12 months), it is justified (for
patients without HLA-compatible donors) to explore innovative approaches, such as new drugs
or immunotherapies. Patients with longer first CR (>12 months) generally relapse with drug-
sensitive disease and have a higher chance of attaining a CR. However, cure is uncommon,
and treatment with novel approaches should be considered if SCT is not possible. One
promising therapy is decitabine, a nucleoside analog that inhibits DNA methyltransferase and
subsequently reverses aberrant methylation in AML cells. Interestingly, inhibiting DNA
methyltransferase occurs at a much lower dose than previously used to produce a cytotoxic
effect in AML. Low-dose decitabine yields CR in a small subset of patients with relapsed AML,
including those with unfavorable karyotypes. New agents are needed.
For elderly patients (age >60) for whom clinical trials are not available, gemtuzumab
ozogamicin (Mylotarg) is another alternative. This therapy is an antibody-targeted
chemotherapy consisting of the humanized anti-CD33 antibody linked to calicheamicin, a
potent antitumor antibiotic. The CR rate is ~30%. Its effectiveness in early relapsing (<6
months) or refractory AML patients is limited, possibly due to calicheamicin being a potent
MDR1 substrate. Toxicity, including myelosuppression, infusion toxicity, and venoocclusive
disease, can be observed with gemtuzumab ozogamicin. Pretreatment with glucocorticoids can
diminish many of the infusion reactions associated with gemtuzumab ozogamicin. Studies are
examining this treatment in combination with chemotherapy for both young and older patients
with previously untreated AML.
World Health Organization Classificationa

I. AML with recurrent genetic abnormalities

AML with t(8;21)(q22;q22);RUNX1/RUNX1T1b

AML with abnormal bone marrow eosinophils [inv(16)(p13q22) or


t(16;16)(p13;q22);CBFB/MYH11]b

Acute promyelocytic leukemia [AML with t(15;17)(q22;q12) (PML/RAR ) and


variants]b

AML with 11q23 (MLL) abnormalities

II. AML with multilineage dysplasia

Following a myelodysplastic syndrome or myelodysplastic syndrome/myeloproliferative


disorder

Without antecedent myelodysplastic syndrome

III. AML and myelodysplastic syndromes, therapy-related

Alkylating agentrelated

Topoisomerase type II inhibitorrelated

Other types

IV. AML not otherwise categorized

AML minimally differentiated

AML without maturation

AML with maturation

Acute myelomonocytic leukemia

Acute monoblastic and monocytic leukemia

Acute erythroid leukemia

Acute megakaryoblastic leukemia

Acute basophilic leukemia

Acute panmyelosis with myelofibrosis

Myeloid sarcoma

French-American-British (FAB) Classificationc Incidence

M0: Minimally differentiated leukemia 5%

M1: Myeloblastic leukemia without maturation 20%

M2: Myeloblastic leukemia with maturation 30%


M3: Hypergranular promyelocytic leukemia 10%

M4: Myelomonocytic leukemia 20%

M4Eo: Variant: Increase in abnormal marrow eosinophils

M5: Monocytic leukemia 10%

M6: Erythroleukemia (DiGuglielmo's disease) 4%

M7: Megakaryoblastic leukemia 1%


Acute Lymphocytic Leukemia

For patients with ALL, evaluation is usually completed after a complete blood count, chemistry
studies reflecting major organ function, a bone marrow biopsy with genetic and immunologic
studies, and a lumbar puncture. The latter is necessary to rule out occult CNS involvement. At
this point, most patients would be ready to begin therapy. In ALL, prognosis is dependent
upon the genetic characteristics of the tumor, the patient's age, the white cell count, and the
patient's overall clinical status and major organ function.

The most common cancer in childhood is B cell ALL. Although this disorder can also present as
a lymphoma in either adults or children, presentation as lymphoma is rare.

The malignant cells in patients with precursor B cell lymphoblastic leukemia are most
commonly of pre-B cell origin. Patients typically present with signs of bone marrow failure
such as pallor, fatigue, bleeding, fever, and infection related to peripheral blood cytopenias.
Peripheral blood counts regularly show anemia and thrombocytopenia but might show
leukopenia, a normal leukocyte count, or leukocytosis based largely on the number of
circulating malignant cells. Extramedullary sites of disease are frequently involved in patients
who present with leukemia, including lymphadenopathy, hepato- or splenomegaly, CNS
disease, testicular enlargement, and/or cutaneous infiltration.

The diagnosis is usually made by bone marrow biopsy, which shows infiltration by malignant
lymphoblasts. Demonstration of a pre-B cell immunophenotype and, often, characteristic
cytogenetic abnormalities confirm the diagnosis. An adverse prognosis in patients with
precursor B cell ALL is predicted by a very high white cell count, the presence of symptomatic
CNS disease, and unfavorable cytogenetic abnormalities. For example, t(9;22), frequently
found in adults with B cell ALL, has been associated with a very poor outlook. The bcr/abl
kinase inhibitors have improved the prognosis.

PRECURSOR B CELL LYMPHOBLASTIC LEUKEMIA: TREATMENT


The treatment of patients with precursor B cell ALL involves remission induction with
combination chemotherapy, a consolidation phase that includes administration of high-dose
systemic therapy and treatment to eliminate disease in the CNS, and a period of continuing
therapy to prevent relapse and effect cure. The overall cure rate in children is 90%, while
~50% of adults are long-term disease-free survivors. This reflects the high proportion of
adverse cytogenetic abnormalities seen in adults with precursor B cell ALL.

PRECURSOR T CELL LYMPHOBLASTIC LEUKEMIA


Precursor T cell malignancies can present either as ALL or as an aggressive lymphoma. These
malignancies are more common in children and young adults, with males more frequently
affected than females.

Precursor T cell ALL can present with bone marrow failure, although the severity of anemia,
neutropenia, and thrombocytopenia is often less than in precursor B cell ALL. These patients
sometimes have very high white cell counts, a mediastinal mass, lymphadenopathy, and
hepatosplenomegaly. Precursor T cell lymphoblastic leukemia is most often found in young
men presenting with a large mediastinal mass and pleural effusions. Both presentations have
a propensity to metastasize to the CNS, and CNS involvement is often present at diagnosis.

Precursor T Cell Lymphoblastic Leukemia: Treatment


Children with precursor T cell ALL seem to benefit from very intensive remission induction and
consolidation regimens. The majority of patients treated in this manner can be cured. Older
children and young adults are also often treated with "leukemia-like" regimens. Patients who
present with localized disease have an excellent prognosis. However, advanced age is an
adverse prognostic factor. Adults with precursor T cell leukemia who present with high LDH
levels or bone marrow or CNS involvement are often offered bone marrow transplantation as
part of their primary therapy.

Excerpted from Harrisons Online


Myeloproliferative disorders/neoplasms & Learning Objectives
Myelodysplastic Syndromes
Understand the common origin of the MPN
Michael Boyiadzis MD, MHSc JKA2 mutations in the MPN
Assistant Professor of Medicine
Diagnosis and treatment of polycythemia vera
Division of Hematology-Oncology
University of Pittsburgh Cancer Institute Diagnosis and treatment of essential thrombocythemia
Diagnosis and treatment myelofibrosis
Myelodysplastic syndromes

Key words: Myeloproliferative Neoplasms, Polycythemia vera,


Essential thrombocythemia, Primary myelofibrosis, Myelodysplastic syndromes
January 2009

Bone marrow stem cell


Myeloproliferative Neoplasms (MPN)
(Myeloproliferative Disorders, MPDs) Clonal abnormality

Myeloproliferative neoplasms (MPN) are clonal disorders Principal


cellular
of hematopoietic stem cells that clinically manifest as proliferation Granulocyte Red cell Mega- Reactive
precursor precursor karyocytes fibrosis
overproduction of cells that contribute to the myeloid
BCR/ABL
lineage

Clinical Chronic Polycythemia Essential Primary


entity myelogenous vera thrombocythemia myelofibrosis
leukemia

Acute leukemia

Myeloproliferative neoplasms (MPN) Myeloproliferative neoplasms


(2008 WHO classification)
The cardinal features of the three main MPNs are:
Chronic myelogenous leukemia (CML)
Polycythemia vera (PV) an increased red-cell mass in polycythemia vera

Essential thrombocythemia (ET) a high platelet count in essential thrombocythemia

Primary myelofibrosis bone marrow fibrosis in idiopathic myelofibrosis


Chronic neutrophilic leukemia
Chronic eosinophilic leukemia, not otherwise categorized
Each can clinically mimic the other MPN as well
Hypereosinophilic syndrome as many nonclonal disorders
Mast cell disease

1
Myeloproliferative neoplasms JKA2 mutations in the MPN
In 2005, several groups reported a single, acquired point
mutation in the Janus kinase 2 (JAK2) gene in
patients with myeloproliferative disorders

JAK2, a cytoplasmic tyrosine kinase, is critical for


intracellular signaling by the receptors for
erythropoietin, thrombopoietin,
granulocyte colony-stimulating factor, and
granulocytemacrophage colony-stimulating factor

JKA2 mutations in the MPN JKA2 mutations in the MPN


JAK2 has an enzymatically active kinase domain (JH1) JAK2 with V617F mutation
and a catalytically inactive domain (JH2)
Results in expression of a constitutively
JH2 domain has an auto inhibitory function that normally activated JKA2 tyrosine kinase in the
suppresses the kinase activity of JAK2 absence of binding of Epo
or Tpo to their respective receptors
Quanine to thymine mutation encoding a valine to
phenylalanine substitution at position 617 in the JH2
domain of JAK2

Erythropoietin and thrombopoietin regulation in PV and ET Polycythemia vera (PV)

Both PV and ET are characterized by increased Incidence: 0.8 to 2.6/100,000 population per year
sensitivity of committed hematopoietic cells to their
Median age: 60 years
respective primary humoral growth factors: erythroid
Incidence of PV is slightly higher in men than women
precursors to erythropoietin (Epo) in PV and (2.8 versus 1.3 cases/100,000 per year)
megakaryocytes to thrombopoietin (Tpo) in ET
95 to 97 % of patients with PV have the V617F
mutation in exon 14 of the JAK2 gene, which is absent
in normal subjects as well as those with secondary
polycythemia

2
P. Vera Symptoms Clinical Findings in PV
Splenomegaly
Headache
Hepatomegaly
Pruritus
Facial and/or conjuctival plethora
Weakness
Engorgement of the veins of the optic fundus
Dyspnea
Hypertension
Dizziness
Cutaneous ulcers or gouty features
Visual change
Signs of arterial or venous thrombosis
Weight loss
Epigastric pain
Erythromelalgia
Epistaxis, gastrointestinal bleeding
Thrombosis (venous or arterial)

Secondary Polycythemia
P. Vera- laboratory findings
Physiologically appropriate: response to hypoxia
Elevated hemoglobin/hematocrit 1. Reduced PaO2
a. Chronic lung disease
b. Pickwickian (obesity-hypoventilation) syndrome
Elevated red blood cell mass c. Sleep apnea
d. High altitude
e. Cyanotic heart disease
Low serum Epo levels 2. Normal PaO2
a. Smokers and CO-induced polycythemia
Elevated platelet count and white blood cell count ( ~40-60%) Physiologically inappropriate
1. Tumors (Renal cell carcinoma, Wilms tumor, hepatoma, uterine fibroma,
cerebellar hemangioma, atrial myxoma)
Elevated lactate dehydrogenase level (LDH) 2. Benign renal disease (Polycystic kidney disease, hydronephrosis, renal artery
stenosis)
3. Postrenal transplantation erythrocytosis
Elevated uric acid level 4. Endocrine disorders (Pheochromocytoma, primary aldosteronism, Cushing
syndrome)
5. Administration of erythropoiesis-stimulating hormones
a. Epo
b. Androgens

Diagnostic algorithm for suspected PV


Diagnostic criteria for PV were proposed in the late
1960s by the Polycythemia Vera Study Group (PVSG)

While these criteria continue to be widely used in


clinical practice, they were developed prior to the
general availability of assays for erythropoietin
endogenous erythroid colony formation, and specific
karyotypic and clonal analyses

3
2008 WHO diagnostic criteria for PV
Clinical complications of PV
Major criteria:
1.
Hgb >18.5 g/dL in men, Hgb >16.5 g/dL in women Myelofibrosis, myelodysplasia and acute leukemia
or
Hgb or Hct >99th percentile of reference range for age, sex or altitude of residence
or Bleeding and thrombosis
Hgb>17 g/dL in men or>15 g/dL in women if associated with a sustained increase
of 2 g/dL from baseline that cannot be attributed to correction of iron deficiency
or
elevated red cell mass >25% above mean normal predicted value

2. Presence of JAK2V617F or similar mutation

Minor Criteria: 1. BM trilineage myeloproliferation


2. Subnormal serum Epo level
3. EEC growth

Diagnosis of PV requires meeting either both major criteria and


one minor criterion or the first major criterion and 2 minor criteria

PV treatment options Essential thrombocythemia (ET)


Phlebotomy- goal hematocrit <0.45 Incidence: 0.2 to 2.5/100,000 population per year

Mean age : 60 years


Cytoreductive therapy
- Risk stratification ( age>60, history of thrombosis)
Female predominance (f/m ~2:1)
- Agents : hydroxyurea as first line therapy
interferon (younger pts, pregnant women)
JAK2 mutation ~ 57%

Antithrombotic therapy
- Aspirin if not contraindicated

Inhibition of JAK2 is a potential therapeutic target for treatment of MPN

Secondary thrombocytosis Clinical Findings in ET


Transient Processes Up to one-half of patients with ET may be asymptomatic at presentation

1. Acute blood loss Vasomotor


2. Acute infection or inflammation
Headaches, visual disturbances, dizziness, burning
3. Drug reactions
erythromelalgia, cognitive deficits, syncope
Sustained Processes
Thrombotic
1. Iron deficiency Arterial and venous
2. Hemolytic anemia
3. Asplenic state
4. Chronic inflammatory or infectious diseases Hemorrhagic
5. Cancer Gastrointestinal, mucosal, epistaxis,
deep hematoma, hemathrosis

4
Clinical Findings in ET Diagnostic algorithm for suspected ET

Splenomegaly ~ 40%

Hepatomegaly uncommon

2008 WHO diagnostic criteria for ET Treatment options for ET


1. Platelet count 450 x 109 /L
Most patients with ET are asymptomatic and can
2. Megakaryocyte proliferation with large and mature morphology
No or little granulocyte or erythroid proliferation be observed for months or years after the
diagnosis is established before requiring
3. Not meeting WHO criteria for CML, PV, PMF, MDS or other treatment
myeloid neoplasm

4. Demonstration of JAK2V617F or other clonal marker or Treatment strategies are based primarily on the
no evidence of reactive thrombocytosis presence or absence of risk factors for
thrombosis
Diagnosis of ET requires meeting all four major criteria

Treatment options for ET Treatment options for ET

Interferon-
High risk patients The risk of recurrent
thrombosis is unacceptably high in patients with a effective in reducing platelet count but significant
previous history of thrombosis or age >60 years or side effect
platelet > 1,500 x 109/L
Anagrelide
block megakaryocyte maturation resulting in platelet
Platelet-lowering agent underproduction
- hydroxyurea as first line therapy common side effects: headache, palpitation, fluid retention
diarrhea, fatigue, nausea, dizziness

Antithrombotic therapy
- Aspirin if not contraindicated

5
Primary myelofibrosis (PMF) Clinical manifestations of PMF

Incidence of 1.5 per 100,000 Severe fatigue


Dyspnea, palpitations, light headedness (anemia)
Median age: 67 years Pain, early satiety (splenomegaly)
Left upper quadrant pain, nausea (splenic infarct)
Hallmark feature: marrow fibrosis and myeloid Bleeding, bruising (thrombocytopenia, dysfunctional
metaplasia (extramedullary hematopoiesis) platelets)
Joint pain (hyperuricemia)
Bone pain
Abdominal pain (portal hypertension)
GI bleeding

Laboratory findings in PMF Laboratory findings in PMF

Anemia
Leukocytosis (50%)
Leukopenia (7%)
Thrombocytosis (28%)
Thrombocytopenia (37%)
Elevated LDH, bilirubin, uric acid

Immature cells from the bone marrow into the blood is


characteristic but not sensitive for the diagnosis of PMF

Conditions associated with bone marrow fibrosis 2008 WHO diagnostic criteria for PMF
Major criteria
Malignant diseases 1. Megakaryocyte proliferation and atypia accompanied by either reticulin
MPN and/or collagen fibrosis or
in the absence of reticulin fibrosis, the megakaryocyte changes must be
Myeloid malignancies accompanied by increased marrow cellularity, granulocytic proliferation and
often decreased erythropoiesis (i.e. pre-fibrotic PMF)
Lymphoid malignancies 2. Not meeting WHO criteria for CML, PV, MDS or other myeloid neoplasm
Non-hematologic malignancies- adenocarcinoma 3. Demonstration of JAK2V617F or other clonal marker or no evidence of
reactive marrow fibrosis

Non-malignant Disorders Minor Criteria


1. Leukoerythroblastosis
Chronic infection 2. Increased serum LDH
3. Anemia
Autoimmune diseases 4. Palpable splenomegaly

Diagnosis of primary myelofibrosis (PMF) requires meeting all three major criteria
and two minor criteria

6
Course and prognosis Treatment options for PMF

Morbidity and mortality are relayed to Allogeneic hematopoietic cell transplantation is


hematopoietic failure, thrombosis, the only curative treatment for this disorder, it is
hypersplenism, advanced age and evolution to associated with a high treatment-related
AML mortality, and may not be available for older
patients with severe comorbidities and those
without a suitable donor

Treatment options for PMF MYELODYSPLASTIC SYNDROMES

In the absence of allogeneic HSCT as a Myelodysplastic syndromes are clonal disorders


therapeutic option- supportive and palliative characterized initially by ineffective hematopoiesis
modalities for the symptomatic patient and subsequently by the development of acute
leukemias

Splenectomy: can be considered in selected Peripheral blood cytopenias in combination with a


patients with a painfully enlarged spleen, anemia hyper cellular bone marrow exhibiting dysplastic
and other refractory cytopenias, and/or severe changes are the hallmark of MDS
degrees of portal hypertension

WHO CLASSIFICATION OF MYELODYSPLASTIC SYNDROMES


MYELODYSPLASTIC SYNDROMES
Incidence: Increases with age
Overall: 4.1 per 100,000

Age 50 59: 5.3 per 100,000


Age 60 69: 15 per 100,000
Age 70 79: 49 per 100,000
Age > 80: 89 per 100,000

Mean age: 68 years

M/F: 1:1

7
MYELODYSPLASTIC SYNDROMES INTERNATIONAL PROGNOSTIC SCORING SYSTEM (IPSS)

Clonal cytogenetic abnormalities: 30% -79% Score value


Deletions are more frequent than translocations
Prognostic 0 0.5 1 1.5 2
variable
Most frequent chromosomal aberrations in MDS patients
BM blasts (%) <5 5 - 10 - 11 - 20 21 - 30
Numerical Translocations Deletions
Karyotype* Good Intermedia Poor
Cytogenetics (%) Cytogenetics (%) Cytogenetics (%)
te
+8 19 inv 3 7 del 5q 27 Cytopenias 0-1 2 or 3
7 15 t (1;7) 2 del 11q 7 lineage lineages
+ 21 7 t (1;3) 1 del 12q 5 *Good: normal, Y, del (5q), del (20q)
Poor: complex ( 3 abnormalities) or chromosome 7 abnormalities
5 7 t (3;3) 1 del 20q 5 Intermediate: all other abnormalities
t (6;9) <1 del 7q 4 Cytopenias defined as: Hemoglobin level < 10 g/dL; ANC * < 1800/mm3; Platelet count
< 100,000/mm3
t (5;12) <1 del 13q 2

: loss of chromosome; +: additional chromosome; inv: inversion;


t: translocation; del: deletion

MYELODYSPLASTIC SYNDROMES MDS treatment options

Supportive care: Transfusions and cytokine support


Score Group

IPSS Risk Group Score Years for 25% of patients to Low intensity therapy: Hypo-methylating agents,
evolve to AML azacitidine and decitabine have been approved by the US
Low 0 9.4
FDA for treatment of MDS, and lenalidomide for patients
with MDS associated with a deletion 5q cytogenetic
Intermediate-1 0.5 1 3.3 abnormality
Intermediate-2 1.5 2 1.1
High intensity therapy
High 2.5 0.2
(1) Hematopoietic stem cell transplantation (HSCT)

(2) Intensive chemotherapy

MDS treatment options


Treatment Recommendation depends on:
Patients IPSS Risk Group, age, performance status and
comorbid conditions

Therapy for Lower Risk Patients (IPSS Low/Intermediate-1)


Supportive care and consider low intensity therapy

Therapy for Higher Risk Patients (IPSS Intermediate-2/High)


HSCT or intensive chemotherapy

If patient is not candidate for high intensity therapy -


supportive care and low intensive therapy

8
Learning Objectives

Understand the common origin of the MPN


JKA2 mutations in the MPN
Diagnosis and treatment of polycythemia vera
Diagnosis and treatment of essential thrombocythemia
Diagnosis and treatment myelofibrosis
Myelodysplastic syndromes

Myeloproliferative disorders/neoplasms

The myeloproliferative disorders/ neoplasms (MPD/MPN) arise as clonal expansions of


multipotential hematopoietic progenitor cells in which one cell type dominates the
clinical picture; over-production of erythrocytes (polycythemia vera; PV), neutrophils
(chronic myeloid leukemia; CML), or platelets (essential thrombocythemia; ET). Primary
myelofibrosis (PMF) is characterized by fibrosis within the bone marrow.
In addition to the above disorders the 2008 WHO classification include as part of MPNs
chronic neutrophilic leukemia, chronic eosinophilic leukemia, not otherwise categorized,
hypereosinophilic syndrome and mast cell disease.
Part of the lecture will review the differential diagnosis of erythrocytosis,thrombocytosis
and marrow fibrosis and will discuss the diagnosis and treatment of three of the MPNs:
PV, ET and PMF. CML was presented in the chronic leukemias lecture.

Polycythemia vera:

1. Background: Patients have a median age of 60 yrs at presentation but patients


at all ages have been reported.

2. Clinical presentation: Patients with PV may be diagnosed while


asymptomatic by detection of an elevated Hb found during routine bloodwork. However,
patients may present due to thrombosis secondary to the erythrocytosis (e.g., hepatic vein
thrombosis - Budd-Chiari syndrome). Other symptoms of hypervolemia may be present:
chest pain, dyspnea, headache, or visual disturbances. Patients with PV describe intense
pruritus following hot showers (aquagenic pruritus). Secondary gout due to increased
protein turnover and elevated uric acid levels may arise. Finally, patients may complain
about intense burning pain and erythema of the feet (erythromelalgia).
3. Physical examination: Patients with PV often have ruddy complexions due to
the erythrocytosis. Funduscopic exam may reveal vascular engorgement and
hemorrhages. Moderate splenomegaly is seen in the majority (~80%) of patients.
4. Laboratory abnormalities: Patients with PV typically have Hb >18 and
HCT >55%. In addition, 40% of patients have an elevated WBC >12,000 and 60% have
an increased platelet >600,000. This is referred to as panmyelosis. Mutation in a
cytoplasmic tyrosine kinase (JAK2; V617F) has been described and is present in >95%
of patients with PV. The mutation causes auto-activation of cytokine receptors for
erythropoiesis causing cell proliferation independent of erythropoietin.

5. Differential diagnosis: The major points in the differential diagnosis is the


determination of a relative or absolute increase in the red cell mass and whether the
absolute increase in the red cell mass is appropriate (i.e., sensitive to Epo) or
inappropriate (Epo-independent as in PV)

a) Relative vs Absolute
1) relative: decreased plasma volume (dehydration; diuretics; burns)
2) absolute: Epo-sensitive vs Epo-independent
b) Absolute erythrocytosis
1) Epo-sensitive: Appropriate vs Inappropriate
(1) Appropriate:
cardiac
pulmonary
altitude
high affinity hemoglobinopathy
(2) Inappropriate

Renal lesions (tumors, cysts, diffuse parenchymal disease,


hydronephrosis, renal artery stenosis, transplantation)
Endocrine lesions (adrenal tumors)
Miscellaneous tumors (cerebellar hemangioblastoma, uterine
fibroids, bronchial carcinoma)

2) Epo-insensitive: P. vera

Finding Secondary erythro. Polycythemia vera

red cell volume increased increased


splenomegaly absent present
arterial oxygen sat normal/decreased normal
leukocytosis absent present
thrombocytosis absent present
marrow erythro. hyperplasia pan-hyperplasia
epo level increased decreased
Criteria for PV

Major criteria:
1.
Hgb >18.5 g/dL in men, Hgb >16.5 g/dL in women
or
Hgb or Hct >99th percentile of reference range for age, sex or altitude of residence
or
Hgb>17 g/dL in men or>15 g/dL in women if associated with a sustained increase of 2
g/dL from baseline that cannot be attributed to correction of iron deficiency
or
elevated red cell mass >25% above mean normal predicted value

2. Presence of JAK2V617F or similar mutation

Minor Criteria: 1. BM trilineage myeloproliferation


2. Subnormal serum Epo level
3. EEC growth

Diagnosis of PV requires meeting either both major criteria and


one minor criterion or the first major criterion and 2 minor criteria

6. Treatment: Phlebotomy- goal hematocrit <0.45

Cytoreductive therapy
- Risk stratification ( age>60, history of thrombosis)
- Agents : hydroxyurea
interferon (younger pts, pregnant women)

Antithrombotic therapy
- Aspirin if not contraindicated

Previously, 32P or alkylator chemotherapy (e.g., chlorambucil) had been employed


as therapy; however, these agents increase the risk of transformation to acute
leukemia.

7.Prognosis: Untreated, the mortality of PV is 50% within the first 18 months following
the development of symptoms. Thrombotic complications are seen in 20% at diagnosis
and another 30% will develop complications during the course of the disease. The risk of
thrombosis is greatest for patients >70.
Essential thrombocythemia (ET):

1. Background: Median age of patients is 60 but also seen in young adults (10-
25% of cases). Slightly more prevalent in women

2. Clinical presentation: Often (1/3) found incidentally during routine


bloodwork. Majority of patients present with symptoms related to vasomotor activity
(vascular headaches or visual disturbances). Erythromelalgia also reported in patients
with ET. 10-25% of patients present with thrombosis while another 10% have
hemorrhagic complications.

3. Physical examination: Examination is relatively unrewarding. Mild


splenomegaly is present in 20%. Massive splenomegaly is more common in other MPS.

4. Laboratory abnormalities: Patients are usually not anemic but may have a
mild leukocytosis. Pseudohyperkalemia may be seen secondary to release of potassium
by the platelets during clotting. Qualitative defects in platelet aggregation can be
observed (prolonged bleeding time and defective aggregation following addition of
epinephrine) but none are diagnostic of ET. The JAK2 mutation is observed in ~50% of
patients with ET.

5. Differential diagnosis: Reactive (secondary) thrombocytosis can be seen in


several situations. Post-splenectomy or in states of hyposplenism (e.g., amyloidosis)
platelet counts may be >600,000. A rebound thrombocytosis is observed after repleting
folate or vitamin B12 or following discontinuation of alcohol. In chronic inflammatory
states (infection, inflammatory bowel disease, collagen vascular disease) or malignancies
thrombocytosis may occur. Iron-deficiency is a prevalent cause for thrombocytosis
reportedly secondary to the structural homology of Epo to thrombopoietin.

Essential Reactive
Thrombocythemia Thrombocythemia

chronic thrombocytosis present absent


known cause for reactive process absent present
thrombosis/hemorrhage present absent
splenomegaly present absent
marrow fibrosis present absent
abnormal cytogenetics present absent
6. Criteria for ET
1. Platelet count 450 x 109 /L

2. Megakaryocyte proliferation with large and mature morphology


No or little granulocyte or erythroid proliferation

3. Not meeting WHO criteria for CML, PV, PMF, MDS or other myeloid neoplasm

4. Demonstration of JAK2V617F or other clonal marker or no evidence of reactive


thrombocytosis

Diagnosis of ET requires meeting all four major criteria

7.Treatment for ET

High risk patients The risk of recurrent thrombosis is high in patients with a
previous history of thrombosis or age >60 years or
platelet > 1,500 x 109/L

Platelet-lowering agent
- hydroxyurea as first line therapy

Anagrelide
block megakaryocyte differentiation and
proliferation resulting in platelet underproduction
side effects: headache, palpitation, fluid retention, diarrhea
Interferon-
effective in reducing platelet count but significant side effect

7. Prognosis: Majority of patients (60-80%) have a normal life expectancy. Conversion


to acute leukemia is seen in 3-4% of patients primarily those treated with alkylator
therapy or 32P.

Primary Myelofibrosis( PMF) (agnogenic myeloid metaplasma)

1. Background: Primarily affects older patients with median age of 60.


2. Clinical presentation: Patients typically present with symptoms related to
pancytopenia and splenomegaly.

3. Physical examination: Marked splenomegaly is characteristic.

4. Laboratory abnormalities: The peripheral blood smear demonstrates


findings of extramedullary hematopoiesis (tear-drop red cells, nucleated red cells, left
shift (promyelocytes, myelocytes). Increased uric acid may be seen. Attempts at
aspiration of bone marrow typically result in a dry tap due to the extensive fibrosis.
Radiographs may shown osteosclerosis.

5. Differential diagnosis: Myelofibrosis may be seen in any of the other MPS


(CML or PV). Thus, these disorders need to be excluded prior to making the diagnosis of
AMM. Secondary myelofibrosis has been reported in many other disorders:

hematologic diseases: AML (especially M7); ALL; myelodysplastic syndrome;


hairy cell leukemia; Hodgkins and non-Hodgkins lymphoma; multiple
myeloma; systemic mastocytosis
metastatic carcinoma (breast, prostate, stomach)
infections (TB, histoplasmosis, HIV)
renal osteodystrophy
vitamin D deficiency
hypo- or hyperthyroidism
auto-immune disorders: SLE; scleroderma
exposures: radiation, benzene
Gauchers disease
osteopetrosis

6. Treatment: In young patients allogeneic hematopoietic stem cell


transplantation is curative. For patients that are not candidate for stem cell
transplantation therapy is primarily directed at alleviating symptoms and improving the
quality of life.

7.Prognosis: In distinct contrast to the other MPS, the prognosis of AMM is poor
with a median survival of 4 years. Causes of death include infection, hemorrhage and
cardiac failure. There is an intrinsic rate (10%) of evolution to acute leukemia.
MYELODYSPLASTIC SYNDROMES
The myelodysplastic syndromes (MDS) comprise a heterogeneous group of malignant
stem cell disorders characterized by dysplastic and ineffective blood cell production (ie,
apoptotic death) and a variable risk of transformation to acute leukemia . These disorders
may occur de novo or arise years after exposure to potentially mutagenic therapy (eg,
radiation exposure, chemotherapy)
Signs and symptoms at presentation of MDS are non-specific. Many patients are
asymptomatic, with the diagnosis established upon routine laboratory screening. Others
present with symptoms resulting from a previously unrecognized anemia, including
fatigue, weakness, exercise intolerance, angina, dizziness, cognitive impairment, or an
altered sense of well being. Less commonly, infection, easy bruising, or bleeding
precipitates hematologic evaluation. Systemic symptoms such as fever and weight loss
are uncommon, and generally represent late manifestations of the disease or its attendant
complications.

1. CBC with Leucocytes: ANC < 2500/mm3: 50%


differential ANC < 1500/mm3: 20% - 35%
WBC: Rare (Except patients with chronic myelomonocytic leukemia)
Erythrocytes: Hgb < 10 g/dL: 80%
Anemia as an isolated cytopenia: 30 - 35%
Platelets: Thrombocytopenia: 25% - 50%

2. Peripheral blood Peripheral blood smear abnormalities are a hallmark of MDS and include:
smear 1. Hypogranulated neutrophils with abnormal nuclei
2. Giant platelets
3. Polychromasia
4. Macrocytosis and anisopoikilocytosis
5. Reticulocytosis is rare
3. Bone marrow biopsy Marrow cellularity is normal or increased
and aspiration with No single characteristic feature of marrow morphology distinguishes MDS
iron stains Dysplastic features seen in MDS marrow include:
1. Megaloblastoid changes
2. Ringed sideroblasts
3. Micromegakaryocytes
4. Abnormal megakaryocyte nuclei
5. Increased myeloblasts
6. Hypogranular promyelocytes
7. Increased monoblasts
4. Cytogenetic analysis Clonal cytogenetic abnormalities: 30% -79%
of bone marrow cells Deletions are more frequent than translocations
MOST FREQUENT CHROMOSOMAL ABERRATIONS IN MDS PATIENTS
Numerical Translocations Deletions
Cytogenetics (%) Cytogenetics (%) Cytogenetics (%)
+8 19 inv 3 7 del 5q 27
7 15 t (1;7) 2 del 11q 7
+ 21 7 t (1;3) 1 del 12q 5
5 7 t (3;3) 1 del 20q 5
t (6;9) <1 del 7q 4
t (5;12) <1 del 13q 2
: loss of chromosome; +: additional chromosome; inv: inversion;
t: translocation; del: deletion
5. HLA typing For patients under age 60 years and their siblings
Consider HLA typing for older patients with good performance status
6. Chemistry RBC folate, serum B12, serum iron/TIBC/ferritin, serum erythropoietin level (prior to
RBC transfusion)
WHO CLASSIFICATION OF MYELODYSPLASTIC
SYNDROMES

Category Peripheral Blood Bone Marrow


RA Anemia Erythroid dysplasia only
No or rare blasts Blasts < 5%
- Ringed sideroblasts < 15%

RARS Anemia Erythroid dysplasia only


No blasts Blasts < 5%
Ringed sideroblasts 15%

RCMD Cytopenias (bi- or pancytopenia) Dysplasia 10% of cells in two or


more myeloid cell lines
No or rare blasts Blasts < 5%
Monocytes < 1000/mm3 -
- Ring sideroblasts < 15%
RCMD-RS Cytopenias (bi- or pancytopenia) Dysplasia 10% of cells in two or
more myeloid cell lines
No or rare blasts Blasts < 5%
Monocytes < 1000 mm3 -
- Ring sideroblasts 15%
RAEB-I Cytopenias Unilineage or multilineage dysplasia
Blasts < 5% Blasts 5 - 9%
Monocytes < 1000 mm3 -
RAEB-II Cytopenias Unilineage or multilineage dysplasia
Blasts 5 - 19% Blasts 10 - 19%
Monocytes < 1000/mm3 -
Auer rods Auer rods
MDS-U Cytopenias Unilineage dysplasia in granulocyte
or megakaryocytes
No or rare blasts Blasts < 5%
Anemia -
MDS, Isolated del (5q) Blasts < 5% Isolated del (5q), Blasts < 5%

Platelets: normal or increased Megakaryocytes: normal or increased

RA, refractory anemia; RARS, RA with ringed sideroblasts; RCMD, refractory cytopenia with
multilineage dysplasia; RCMD-RS, RCMD with ringed sideroblasts; RAEB, RA with excess blasts
INTERNATIONAL PROGNOSTIC SCORING SYSTEM (IPSS)

SCORE VALUE
Prognostic 0 0.5 1 1.5 2
variable
BM blasts (%) <5 5 - 10 - 11 - 20 21 - 30
Karyotype* Good Intermediate Poor
Cytopenias 0 - 1 lineage 2 or 3
lineages
* Good: normal, Y, del (5q), del (20q)
Poor: complex ( 3 abnormalities) or chromosome 7 abnormalities
Intermediate: all other abnormalities
Cytopenias defined as: Hemoglobin level < 10 g/dL; ANC * < 1800/mm3; Platelet count
< 100,000/mm3

SCORE GROUP
IPSS Risk Score Years for 25% of patients to evolve to
Group AML
Low 0 9.4
Intermediate-1 0.5 1 3.3
Intermediate-2 1.5 2 1.1
High 2.5 0.2

Treatment Recommendation depends on: patients IPSS Risk Group, age, performance
status

Supportive care: Transfusions and cytokine support

Low intensity therapy: Azacitidine and decitabine have has been approved by the FDA
for treatment of MDS, and lenalidomide for patients with MDS associated with a deletion
5q cytogenetic abnormality.

High intensity therapy


(1) Hematopoietic stem cell transplantation (HSCT)
(2) Intensive chemotherapy

Therapy for Lower Risk Patients (IPSS Low/Intermediate-1)


Supportive care and consider low intensity therapy

Therapy for Higher Risk Patients (IPSS Intermediate-2/High)


HSCT or intensive chemotherapy

If patient is not candidate for high intensity therapy -


supportive care and low intensive therapy
Objectives & Key Words
(always obey the authorities)
Pathology of lymphoproliferative disorders
Begin to develop an understanding of
biological & clinical aspects of malignant
lymphomas
Steven H. Swerdlow, M.D.
Begin to know how lymphomas are
January, 2009 diagnosed & classified for clinical
purposes
Key words lymphoma classification, non-
Hodgkin lymphoma, Hodgkin lymphoma,
lymphocyte development, pathology

Pathology of
lymphoproliferative disorders

Malignant lymphomas: biology,


diagnosis & therapy

Components of our magical


mystery tour
This lecture
An approach to the malignant Lecture on the clinical aspects of
lymphomas
lymphomas
Laboratory session on malignant
Not a list of facts to memorize for lymphomas (fewer cartoons, more real
an examination! histologic images)
An oversimplification & will
emphasize lymph nodes rather
than extranodal lymphomas

1
Object: begin to develop an
understanding of biological &
clinical aspects of malignant
lymphomas and how they are
diagnosed & classified for clinical
purposes
To gain some appreciation of
how we recognize & deal with
patients who have malignant
lymphomas

At the end of this experience you A day in the life of a hematopathologist


should be able to
Define what lymphomas are & explain the
similarities and differences from the cells
of the normal immune system.
Explain the reasons for the diversity
among the malignant lymphomas.
Know, in general, how a
hematopathologist would go about
diagnosing & classifying the malignant
lymphomas.

Plan for the lecture What is a malignant lymphoma?


What are lymphomas? Malignant neoplasms of the cells of the immune system
generally based in lymph nodes & other extramedullary
How does one approach the subject without sites (vs leukemias that are generally based in the bone
having to memorize the features of more than 70 marrow)
different types/subtypes of lymphomas? Most of B-cell origin, some of T-cell origin, few of NK-cell
origin.
How does a hematopathologist go about Not 1 neoplasm but many different types of neoplasms
diagnosing & classifying the malignant that vary from indolent possibly requiring no therapy to
lymphomas so that the clinical very aggressive requiring extensive therapy.
hematologist/oncologist knows how to proceed. So to learn about lymphomas is to learn about many
different disorders with increasing interest these days in
precise diagnoses & additional prognostic factors.

2
How to start thinking about Cells of the immune system
lymphomas
Be sure you
understand the basic
features of the normal
cells of the immune
system and their
immunoarchitecture
(ie, how they grow
when they are normal
& happy) Natural killer cells
and how they go
bad Macrophages
Dendritic cells

B-cell is
There are many different subsets of B- B-
B-cell is cells, T-
T-cells & the other cells of the
B-cell is immune system
B-cell. Vary based on their stage of development, their
functional properties & state of transformation
(resting or blast transformed)
Expressed as differences in their location & growth
there is no such thing as pattern, their morphologic appearance,
immunophenotype (the antigens they express) &
repetitionYou will see that when I kept genotype (status of their immunoglobulin or T-cell
on saying something was something ., I receptor genes).
changed it just a little bit until I got a
whole portrait. Gertrude Stein, How writing
is written

Major B-
B-cell compartments
Memory
Immunoblasts
B-cells
Plasma cells
B-cell differentiation
Antigenic selection,
Transformation
Antigenic
somatic stimulation
hypermutation
& isotype switching Centroblasts
B-blast T
And this is an oversimplified
Post-follicular
T B-cells view of B-cell development!
Bone Nave But you already now have a good idea of
marrow B-cell Centrocytes why there are so many different types of B-
cell lymphomas and what types there might
Tingible body be!
macrophage Primaryfollicle
Secondary
follicle or
germinal Marginal zone
Monocytoid B-cells follicular center B-cells

3
The normal reactive lymph node
Also remember Sinuses
Follicles in
Any type of resting B-cell can transform cortex
just like the follicular center cells do ie, to Monocytoid
more rapidly dividing cells that are B-cells
recognized generally because they get Marginal
larger, have nucleoli and somewhat more zone B-cells
abundant cytoplasm. Paracortical
T-cell rich
Hilum area
Medulla (plasma cells &
small lymphs)

How else do we identify these


Major B-cell different types of B-
B-cells &
compartments characterize their proliferations?
Histopathology (architectural & cytologic features)
Immunophenotype (use of antibodies to characterize
cells based on their antigenic expression)
Genotypic studies (use of molecular genetic
techniques to characterize cells based on studying
genes of interest and sometimes their expression
DNA & RNA studies)
Cytogenetic studies (classical & fluorescence in situ
hybridization to look for chromosomal abnormalities
associated with neoplasms in general or specific
types of neoplasms)

Immunophenotypic studies: eg, Genotypic (molecular) studies


identifying germinal center cells varied techniques/varied information
Southern blot analysis (fresh/frozen
Flow cytometry Immunohistochemistry tissue, not rapid)
B-cell clonality (even if admixed
polyclonal population)
T-cell clonality
Lymphoma associated translocations
(eg, bcl-2, c-myc)
Polymerase chain reaction
Similar to above
RT-PCR looking for specific mRNA
Anti-CD10 antibody (brown is positive) (qualitative or quantitative)

4
Looking for t(14;18) --
Gene Profiling IGH@/BCL2
expression analysis Classical FISH using
of thousands of cytogenetics probes for IGH@
genes using & BCL-
BCL-2
DNA/oligonucleotide
microarrays

NEJM 348:1777, 2003

So, what happens when good


lymphocytes go bad? Not a static process
Sequence of molecular/cytogenetic abnormalities
that lead to monoclonal proliferation of abnormal
neoplastic lymphoid cells. Additional
The neoplastic lymphoid cells proliferate & lead molecular/cytogenetic
to destruction of the normal underlying tissues. abnormalities can
Many will disseminate to distant sites. occur over time and
Although having variably blatant abnormal lead to increased
features, many neoplastic lymphoid cells also aggressiveness of the
often retain many of their normal characteristics
(eg, many morphologic & phenotypic features of neoplastic lymphoid
their normal counterparts, ability to interact cells.
with other lymphoid/dendritic cells, etc).

Example development of Example neoplasms of follicular


neoplasms of follicular center cells center cells
IGH@/BCL2 translocation plus
unspecified other abnormalities
leads to abnormal BCL2
expression in follicular/germinal
center cells that do not normally
express BCL2
Overexpression of the anti-
apoptotic bcl-2 protein
B-cell
TOGETHER WITH other
B-cell
Chromosome 14 abnormalities lead to neoplastic
Chromosome 14 Chromosome 18 Chromosome 18 FCC

5
So, what happens in the lymph
BCL-2 IMMUNOSTAIN
node after IGH@/BCL2 & other
neoplastic events occur in FCC?

FOLLICULAR FOLLICULAR
HYPERPLASIA LYMPHOMA

Additional genotypic/molecular
Bone T
What else might happen?
marrow events, eg, TP53 mutation
TT

TT

T
T

TT

TT
Transformation

B-cell monoclonality is another


Diffuse proliferation of
critical element of B-
B-cell neoplasms
transformed B-
B-cells - more rapid another indicator of a
growth in the patient T
homogeneous population
T

Malignant lymphomas composed of B-cells


will demonstrate monoclonality very
helpful although not 100% indicative of
lymphoma.
T
Reactive proliferations are usually
polyclonal.

6
Reactive
What does this mean? Remember lymphoid B-cell lymphoma
that the immunoglobulin in hyperplasia
individual B-
B-cells will either include
the kappa or lambda light chain

but not both.



How can you & So, we


weve reviewed some of the basic
pathogenesis of malignant lymphomas
document this? using FCC lymphomas as an example,
& looked at how malignant lymphomas
express themselves.
Also reviewed the ways in which lymphoid
only proliferations can be evaluated (building on
prior lecture of Dr. Monaghan).
Morphology
Phenotype
Genotype
Cytogenetics findings

Let
Lets get practical! Step 1
You are evaluating a Be sure the pathologist gets the lymph node
patient who comes to biopsy FRESH so that material is available
see you because of an for the following (assuming they are
enlarged lymph node appropriate):
that is growing, painless Morphologic preparations
& hasnt responded to Flow cytometric immunophenotypic
the antibiotics his PCP studies
gave him. A decision is
Molecular studies
made to perform a
lymph node biopsy. Cytogenetic studies

7
Lymph node biopsy
(extranodal proliferation)

Benign Malignant

Specific/non-specific Lymphoma Other (eg metastatic


reactive proliferation (B/T/NK) carcinoma)

Hodgkin lymphoma Non-Hodgkin lymphoma

DECISION TIME! Precise type Precise type

Is it a lymphoma? If a malignant lymphoma, what type?


Evaluate lymph node architecture intact vs Hodgkin lymphoma (aka Hodgkins
destroyed. disease)
Evaluate cytology of proliferating cell population(s) lymphoma with Reed-Sternberg cells
heterogeneous vs homogeneous. surrounded by reactive non-neoplastic chronic
Is there phenotypic evidence for a monoclonal inflammatory cells (variable #s of lymphs,
population or a population with an aberrant plasma cells, eosinophils, histiocytes,
phenotype? neutrophils) and variable degrees of fibrosis.
Do cytogenetic studies suggest a neoplastic clonal several different types
population?
If still no answer, do molecular studies support a Non-Hodgkin lymphoma
monoclonal proliferation and if so of B or T-cells? Is Multiple different types
there evidence of other genotypic abnormalities (like
a chromosomal translocation)?

Current classification of Hodgkin


Hodgkins Classical Hodgkins lymphoma
lymphoma (WHO)
Classical Reed-Sternberg cells and RS
Nodular lymphocyte predominant Hodgkin variants that usually have a
lymphoma characteristic immunophenotype very
Classical Hodgkin lymphoma different than NLPHL
Nodular sclerosis classical Hodgkin lymphoma B-cell & T-cell antigen negative
Lymphocyte-rich classical Hodgkin lymphoma CD15/LeuM1+
Mixed cellularity classical Hodgkin lymphoma CD30/Ki-1+
Lymphocyte depleted classical Hodgkin lymphoma
CD45/LCA-

8
Classical Hodgkin lymphoma Nodular sclerosis Hodgkins
Nodular sclerosis classical Hodgkin lymphoma
lymphoma - birefringent collagen bands and
lacunar type RS variants
Lymphocyte-rich classical Hodgkin
lymphoma - numerous small lymphs
Mixed cellularity classical Hodgkin
lymphoma- moderate number of classic RS cells
admixed with numerous inflammatory cells
Lymphocyte depleted classical Hodgkin Gross
lymphoma-very numerous RS cells or very photograph
extensive diffuse fibrosis Lacunar cell

Non-Hodgkins lymphomas

WARNING: Learning
about the non-
Hodgkin lymphomas
may be hazardous to
your mental health.

Numerous types of non-


non-Hodgkin
lymphoma need to be
distinguished for clinical purposes Lancet 1974
Many have taught us how to think about dividing
7;2(7880):586
up the non-Hodgkin lymphomas.

9
So, especially in terms of the B-
B-
cell lymphomas, just remember
the beginning of this lecture &
you
youll be off to a running start!
As best we can, relate the non-Hodgkin
lymphomas to the cells of the normal
immune system and their varied
compartments B-cells, T-cells, NK cells.
Some entities in particular also require the
input of molecular/cytogenetic & clinical
data
Lukes & Collins, Cancer 34:1488, 1974 The current object of our classification is
the identification of distinct
clinicopathologic entities.

Classification of Neoplastic Diseases of the


Hematopoietic and Lymphoid Systems

Classification Committees with Clinical Advisory Committee


November, 1997

Nothing in WHO classification of the


medicine lymphomas
stands still! Precursor lymphoid neoplasms
New monograph B lymphoblastic leukemia/lymphoma (8 types)
published T lymphoblastic leukemia/lymphoma (1)
September, 2008 Mature B-cell neoplasms (~38 choices)
(as of 10/08 -- Mature T-cell & NK-cell neoplasms (~22 choices)
7000 copies sold!) Hodgkin lymphoma
Nodular lymphocyte predominant
Classical (4 types)

10
Mature B-cell neoplasms
Precursor lymphoid neoplasms Chronic lymphocytic leukaemia/small
lymphocytic lymphoma


Mantle cell lymphoma
Diffuse large B-cell lymphoma (DLBCL),
B-cell prolymphocytic leukaemia NOS
Splenic marginal zone lymphoma T-cell/histiocyte rich large B-cell lymphoma
B lymphoblastic leukaemia/lymphoma Hairy cell leukaemia Primary DLBCL of the CNS
Primary cutaneous DLBCL, leg type
B lymphoblastic leukaemia/lymphoma, NOS Splenic lymphoma/leukaemia, EBV+ DLBCL of the elderly
unclassifiable
B lymphoblastic leukaemia/lymphoma with recurrent Splenic diffuse red pulp small B-cell lymphoma DLBCL associated with chronic
Hairy cell leukaemia-variant inflammation
cytogenetic abnormalities Lymphoplasmacytic lymphoma Lymphomatoid granulomatosis
B lymphoblastic leukaemia/lymphoma with t(9:22) Waldenstrm macroglobulinemia Primary mediastinal (thymic) large B-cell
(q34;q11.2); BCR/ABL Heavy chain diseases lymphoma
Alpha heavy chain disease Intravascular large B-cell lymphoma
B lymphoblastic leukaemia/lymphoma with t(v;11q23); MLL Gamma heavy chain disease ALK positive large B-cell lymphoma
rearranged Mu heavy chain disease
Plasmablastic lymphoma
B lymphoblastic leukaemia/lymphoma with t(12;21) Plasma cell myeloma
Large B-cell lymphoma arising in HHV8-
(p13;q22); TEL/AML1(ETV6-RUNX1) Solitary plasmacytoma of bone associated multicentric Castleman
Extraosseous plasmacytoma disease
B lymphoblastic leukaemia/lymphoma with hyperdiploidy
Extranodal marginal zone lymphoma of Primary effusion lymphoma
B lymphoblastic leukaemia/lymphoma with hypodiploidy mucosa-associated lymphoid tissue Burkitt lymphoma
(Hypodiploid ALL) (MALT lymphoma)
B-cell lymphoma, unclassifiable, with
B lymphoblastic leukaemia/lymphoma with Nodal marginal zone lymphoma features intermediate between diffuse
t(5;14)(q31;q32)(IL3-IGH) Paediatric nodal marginal zone lymphoma large B-cell lymphoma and Burkitt
Follicular lymphoma lymphoma
B lymphoblastic leukaemia/lymphoma with Paediatric follicular lymphoma B-cell lymphoma, unclassifiable, with
t(1;19)(q23;p13.3); E2A-PBX1;TCF3/PBX1) Primary cutaneous follicle centre features intermediate between diffuse
lymphoma large B-cell lymphoma and classical
T lymphoblastic leukaemia/lymphoma Hodgkin lymphoma

Mature T-
T-cell & NK-
NK-cell neoplasms
T-cell prolymphocytic leukaemia
T-cell large granular lymphocytic
Mycosis fungoides
Szary syndrome
Example of diagnosing and
leukaemia
Chronic lymphoproliferative
Primary cutaneous CD30 positive T-
cell lymphoproliferative disorders classifying malignant
disorder of NK-cells Lymphomatoid papulosis
Aggressive NK cell leukaemia Primary cutaneous anaplastic large
cell lymphoma
lymphoma
Systemic EBV positive T-cell
lymphoproliferative disease of Primary cutaneous gamma-delta T-
childhood cell lymphoma
Hydroa vacciniforme-like Primary cutaneous CD8 positive Pop quiz only counts for 5% of
lymphoma aggressive epidermotropic cytotoxic
Adult T-cell leukaemia/lymphoma T-cell lymphoma your grade.
Extranodal NK/T cell lymphoma, Primary cutaneous CD4 positive
nasal type small/medium T-cell lymphoma
Peripheral T-cell lymphoma, NOS

Only joking bad joke, sorry!


Enteropathy-associated T-cell
lymphoma Angioimmunoblastic T-cell
Hepatosplenic T-cell lymphoma lymphoma
Subcutaneous panniculitis-like T- Anaplastic large cell lymphoma,
cell lymphoma ALK positive
Anaplastic large cell lymphoma,
ALK negative

79 year old male with axillary


lymphadenopathy

11
Small lymphocytic
lymphoma
Follicular hyperplasia
9Follicular lymphoma

Mantle cell lymphoma
Marginal zone lymphoma

What tools can you use to solve Flow cytometric studies


this mystery?

Cyclin D1 Immunophenotype
CD20+

CD5+ CD5-

Cyclin D1- Cyclin D1+ CD10+ CD10-

Variable CD23+ CD23- Bcl-6+ Bcl-6- Bcl-6+


FMC7- FMC7+

DLBCL CLL/SLL MCL FL MZL/MALT DLBCL


DLBCL LPL
Burkitt DLBCL
Different case illustrated

12
FICTION studies (FISH for CCND1/IgH
translocation + IHC for cyclin D1)

Normal Cyclin D1+, t(11;14)+

Non-Hodgkins Lymphoma
Mantle cell Classification Project, Blood 89:3909, 1997

lymphoma
The lymphoma
formerly known as
centrocytic T-LB
(intermediate PTCL
lymphocytic) MCL

Relative incidence of ML
Hodgkin
DLBLC
FL
Closed MALT
today B-CLL/SLL
MCL
MedLBCL
ALCL
Burkitt
MZL,nodal
LPL
When is tea time? PTCL
prec T-LB
Other NHL
Seer data (HL vs NHL) & NHL Classification project, Blood 89:3909

13
Philosophy & tools behind making And in addition, an introduction to
the following diagnostic decisions the biology of the malignant
lymphomas
Lymph node biopsy
(extranodal proliferation) The clinical lecture, lymphoma PBL and
Benign Malignant
hematopathology laboratory on lymphomas
all a part of this package.
Specific/non-specific Lymphoma Other (eg metastatic
reactive proliferation (B/T/NK) carcinoma) Why do pathologists bother with all of this?
Hodgkin lymphoma Non-Hodgkin lymphoma

Precise type Precise type

14
Diagnosis and Treatment of Evaluation of Suspected Lymphoma
Lymphoproliferative Disorders Patient History
January 2009 Painless lymphadenopathy
Patricia Kropf, M.D. B symptoms: fever, night sweats and weight loss
Assistant Professor, Hematology/Oncology, Pain
-alcohol induced (HD)
University of Pittsburgh Cancer Institute
-abdominal (adenopathy, splenomegaly)
-bone (destruction, infiltration)
-neurogenic (nerve root, meninges, shingles)
-back (spinal cord compression, retroperitoneal adenopathy)

Evaluation of Suspected Lymphoma Evaluation of Suspected Lymphoma


Patient History Physical Examination
Screen patient
patients history for:
The physical examination must be directed to all potentially involved
involved
Diseases: immunodeficiency states, autoimmune
lymphoid sites:
disease, HIV, Hepatitis B and/or C
Standard lymph node sites (cervical, axillary, inguinal)
A personal or family history of lymphoma Waldeyer
Waldeyers ring (tonsils, base of tongue, nasopharynx)
Personal history of prior radiation Liver and spleen
Occupational exposure: herbicides and other toxins Abdominal nodal sites (mesenteric, retroperitoneal)
Less commonly involved nodal sites (occipital, preauricular,
epitrochlear, popliteal nodes)

Evaluation of Suspected Lymphoma Evaluation of Suspected Lymphoma


Biopsy Procedures
Differential Diagnosis
Bottom line: get tissue! An excisional biopsy is preferred
Lymphadenopathy caused by infection or autoimmune (not a fine needle aspirate)
disorder If a peripheral node is involved, excise the entire node
Unilateral lung masses are frequently due to lung cancer If the only site of disease is intrathoracic or retroperitoneal, a
Splenomegaly may be secondary to infections, portal core biopsy should be pursued either through CT guidance or
hypertension, storage disease, myeloproliferative disorders surgery
Isolated bilateral hilar adenopathy without a mediastinal
Bone marrow biopsy with aspiration may lead to diagnosis
mass suggests sarcoidosis
and will complete staging
Evaluation of Suspected Lymphoma Evaluation of Suspected Lymphoma
Baseline Laboratory Tests Imaging Studies

Complete blood count, differential and platelets Routine radiologic studies should include:
Tests of renal and hepatic function (Cr, ALT, AST, Alk Phos)
LDH (reflects tumor burden)
HIV serology, in selected patients CT scans with contrast of the neck, chest, abdomen
Hep B and C serology, in selected patients and pelvis*
Uric Acid
PET scan (in addition to CT) in certain instances (e.g.
Beta 2 microglobulin level
Peripheral blood for flow cytometry and molecular biology if
DLBCL, Hodgkin's)
indicated

*NCCN guidelines state a chest x-


x-ray and CT of the
abdomen and pelvis is sufficient in most cases

Evaluation of Suspected Lymphoma Hodgkins Lymphoma


Evaluation of the Central Nervous System Epidemiology

Accounts for 1% of all new cancers in the U.S.


Examination of the spinal fluid is routinely used The incidence of HL is bimodal, with the first peak in adults
to exclude occult CNS lymphoma in Burkitt occurring at age 20-
20-30 years and the second peak after age 50
Burkitts
lymphoma or lymphoblastic lymphoma In young patients (less than 10 years of age) there is a 3:1 male
male
to female ratio
The imbalance returns to an equal ratio in adult disease

Hodgkins Lymphoma Hodgkins Lymphoma


Etiology Pathology
The etiology of HL is unknown, however
however HLs are defined by the presence of Reed Sternberg cells
There an association with EBV in up to 40% of cases and their variants
There is some familial and geographic clustering to The Reed-
Reed-Sternberg cells are within a background of
suggest an inherited susceptibility, environmental, or normal appearing lymphocytes, plasma cells,
infectious contribution in its etiology eosinophils, and neutrophils
A higher incidence is found in families of higher social HL almost always originates in the lymph node and
status, advanced education, smaller family size (perhaps spreads through the lymphatics in an orderly fashion
suggesting delayed exposure to infectious or
environmental antigen)
Hodgkins Lymphoma
Pathology and Classification
The World Health Organization (WHO) divides HL into
two entities:
Nodular Lymphocyte Predominant HL
Classical HL

Nodular lymphocyte predominant HL is now recognized to


be an indolent B-
B-cell NHL and not true HL

Hodgkins Lymphoma Hodgkins Lymphoma


Pathology and Classification Sites of Involvement
Classical HL is divided into four subtypes: Lymph nodes in the cervical and supraclavicular
Nodular sclerosis classical HL region are involved in >70% of cases
Mixed cellularity classical HL In nodular sclerosing HL the anterior mediastinum
Lymphocyte depleted HL in the prime location
Lymphocyte rich HL Lung involvement may occur by direct contiguity
or by hematogenous dissemination

Hodgkins Lymphoma
Sites of Involvement
If abdominal sites become involved: the spleen, splenic
hilar lymph nodes, and celiac nodes are the earliest sites
Retroperitoneal lymph node involvement occurs late
Pleural effusions secondary to mediastinal compression of
vascular lymphatic drainage may occur
Bone marrow rarely involved at the time of diagnosis
If bone involvement occurs: bone lesions on plain films are
osteoblastic
Hodgkins Lymphoma
Staging Evaluation
Adequate surgical biopsy
CT scan in combination with PET of neck, chest,
abdomen, and pelvis
Bone marrow aspiration and biopsy

Hodgkins Lymphoma Hodgkins Lymphoma


Treatment Philosophy Case Presentation
Most patients at all stages will be cured 24 year old WF noted painless, enlarged neck lymph
Virtually all patients (all stages) are treated up front nodes
with chemotherapy that includes doxorubicin,
bleomycin, vinblastine, and dacarbazine (ABVD) +/- +/- CBC:
consolidation radiation therapy WBC 5 x 109/L with normal differential
Selected patients with very early stage disease may be hemoglobin 13.9 g/dL
treated with radiation alone platelets 329 x 109/L

An excisional biopsy was performed


Hodgkins Lymphoma Hodgkins Lymphoma
Case Presentation Case Presentation
The evaluation of the biopsy revealed nodular
sclerosing Hodgkin
Hodgkins disease

Hodgkins Lymphoma
Hodgkins Lymphoma Case Presentation
Case Presentation
The patient was treated with combination
chemotherapy: doxorubicin, bleomycin,
vinblastine, and dacarbazine for a total of 6 cycles

Repeat CT/PET performed after therapy


Non-Hodgkins Lymphoma
Epidemiology
NHLs are the most common occurring hematologic
malignancy in the U.S.
Represent 4-
4-5% of all new cancer cases
Are the 5th leading cause of cancer death
Worldwide the highest reported incidence is in the
U.S., Europe, and Australia
Lowest incidence is in Asia

Non-Hodgkins Lymphoma Non Hodgkin's Lymphoma


Epidemiology Etiology
There are well known geographic differences in The cause of most cases of NHL is unknown
histologic subtypes of NHL However there are several strong associations with
diseases and infectious agents
Endemic form of Burkitt
Burkitts Lymphoma: Africa
Immunodeficiency states:
Gastric Lymphoma: Northern Italy
Severe combined immunodeficiency
Adult T Cell Lymphoma: Japan, Caribbean
Hypogammaglobulinemia
Common variable immunodeficiency
AIDS
Solid organ transplant recipients

Non Hodgkin's Lymphoma Non Hodgkins Lymphoma


Etiology Etiology
Patients with auto-
auto-immune disorders have an increased risk Studies of occupational exposure and environmental
of developing NHL: risk have been inconsistent, however:
Rheumatoid arthritis there appears to be an increased prevalence in
Psoriasis farmers, forestry workers, agricultural workers
Sjogren
Sjogrens syndrome (increase risk 30-
30-40 fold)
several studies have shown a slightly increased
Several infectious agents are linked to NHL: risk with herbicide exposure
Epstein Barr Virus (EBV)
HTLV-
HTLV-1
Human Herpes Virus 8
Clinical Classification:
Non Hodgkins Lymphoma
The following three terms are used to define the NHLs:

Indolent -survival of untreated disease measured in years


Aggressive -survival of untreated disease measured in
months
Highly aggressive -survival of untreated disease measured
in weeks

Non Hodgkins Lymphoma Non Hodgkin's Lymphoma


Indolent Lymphomas Aggressive Lymphomas
The indolent lymphomas are associated with reasonably The Aggressive NHLs (survival of untreated disease
long survival (years) if left untreated measured in months)
Are unlikely to be cured with conventional chemotherapy Most common subtypes include:
Most common subtypes: - Diffuse large B cell lymphoma
Follicular lymphoma
- Peripheral T cell lymphoma
Small lymphocytic lymphoma
- Anaplastic large cell lymphoma
Marginal zone lymphoma
Mantle cell lymphoma (may be indolent or aggressive)

Non Hodgkin's Lymphoma


Highly Aggressive Lymphomas
The Highly Aggressive NHLs (survival of untreated
disease measured in weeks)
Most common subtypes include:
- Burkitt's lymphoma
- Precursor B lymphoblastic leukemia/lymphoma
- Precursor T lymphoblastic leukemia/lymphoma
- Adult T cell leukemia/lymphoma
Non-Hodgkins Lymphoma Non-Hodgkins Lymphoma
Follicular Lymphoma Follicular Lymphoma
The second most common lymphoma in the U.S. FL is composed of a mixture of centrocytes
A disease of older adults (small, cleaved follicle center cells) and
Slight female predominance centroblasts (larger, non-
non-cleaved follicle center
cells)
The clinical course is indolent, but the disease is
unlikely to be curable The proportion of centroblasts varies from case to
case
The tumors are assigned a grade based on the
number of centroblasts per high power field
Non-Hodgkins Lymphoma
Follicular Lymphoma
Characteristics of the tumor cells in FL:
Cells are CD10+
Cells are CD5-
CD5-
Cells have surface immunoglobulin
Cells have high levels of BCL-
BCL-2 protein
expression (85% of cases)

Non-Hodgkins Lymphoma Non-Hodgkins Lymphoma


Follicular Lymphoma Follicular Lymphoma
A characteristic translocation t(14;18)(q32;31) is Average survival is 6-
6-10 years
present in 85% of cases Disease is indolent and may go years without treatment
This translocation joins the BCL-
BCL-2 gene to an IgH Treatment for advanced or symptomatic disease consists
of:
segment
CVP-
CVP-R (cyclophosphamide, vincristine, prednisone,
Results in constitutively high levels of BCL-
BCL-2 rituxan) or CHOP-
CHOP-R (cyclophosphamide
(cyclophosphamide,, doxorubicin,
protein which is a potent inhibitor of apoptosis vincristine,
vincristine, prednisone, rituxan)
rituxan)
Radiation is given in select cases
Follicular NHL
Case

Follicular NHL
Case
Biopsy and flow cytometry of abdominal lymph node
Follicular Lymphoma Follicular NHL
Case Case
Flow Cytometric Immunophenotypic studies: Patient was treated with R-
R-CHOP for 6 cycles
CD20+, CD19+, CD10+, CD5-
CD5-, with cell surface kappa
monoclonal B-
B-cells
Post-
Post-treatment CT/PET ordered
Diagnosis: FL, grade 1,
focal grade 2
(follicular growth pattern)

Follicular NHL: Case Non-Hodgkins Lymphoma


Diffuse Large B Cell NHL
Diffuse Large B Cell (DLBC) is the most common
NHL
30% of patients present with systemic symptoms:
fevers, weight loss, night sweats
Up to 40% of cases are extra-
extra-nodal: common
extra-
extra-nodal sites include the GI tract, bone, CNS
Diffuse Large B Cell NHL Diffuse Large B Cell NHL
DLBCL arises from a mature B cell, usually a DLBCL can arise de novo or as a transformation
centroblast or an immunoblast from other indolent lymphomas
The cell
cells nuclear size is more than twice the size Standard treatment:
of a normal lymphocyte Stage I, II: CHOP-
CHOP-R followed by local radiation
The cells express one or more B cell antigens: Stage III, IV: CHOP-
CHOP-R x 6-
6-8 cycles
CD19, CD20, and surface immunoglobulin
Diagnosis and Treatment of Lymphoproliferative Disorders
January 2009
Patricia Kropf, M.D.

Introduction
Lymphomas are neoplastic transformations of normal lymphoid cells. They are
morphologically divided into two major categories: non-Hodgkins lymphoma and
Hodgkins lymphoma. These neoplasms range from the most indolent to the most
aggressive human malignancies.

Non-Hodgkins Lymphoma

Epidemiology
The non-Hodgkins lymphomas (NHLs) are the most common occurring hematologic
malignancies in the United States. They represent 4-5% of all new cancer cases and are
the fifth leading cause of cancer death in the United States. Worldwide, the highest
reported incidence rates are in the U.S., Europe and Australia. The lowest incidence rates
have been reported in Asia. There has been a striking increase in NHL incidence rates
over the past four decades; this has been referred to as an epidemic of NHL. The
reasons for the increased incidence are not entirely clear. The increased prevalence of
some NHLs is clearly related to the acquired immunodeficiency syndrome (AIDS).
However, it should be noted that the increase began before the AIDS epidemic.
There are well known geographic differences in histologic subtypes of NHL. Examples
include the endemic form of Burkitts lymphoma which is seen most commonly in
children in equatorial Africa. Higher rates of gastric lymphoma occur in northern Italy.
Adult T cell leukemia/lymphoma occurs most commonly in Japan and the Caribbean.

Etiology
The cause of most cases of NHL is unknown. However, several genetic diseases,
environmental agents, and infectious agents have been associated with the development
of lymphoma.
Several rare inherited immunodeficiency states are associated with as much as a 25% risk
of developing lymphoma. These include severe combined immunodeficiency,
hypogammaglobulinemia, and common variable immunodeficiency. In addition,
acquired immunodeficiciency states (AIDS, solid organ transplant recipients) are
associated with an increased risk of NHL. Patients with a variety of autoimmune
disorders, including rheumatoid arthritis, psoriasis and Sjogrens syndrome also have an
increased risk of developing lymphoma.
Several infectious agents are linked to the pathogenesis of lymphoma. The Epstein Barr
virus (EBV) is associated with 95% of endemic Burkitts lymphoma, is linked to post-
transplant lymphoproliferative disorders and to some AIDS-associated lymphomas. The
human T-cell lymphotropic virus type 1 (HTLV-1) is a human retrovirus responsible for
Adult T cell Leukemia/Lymphoma. Human herpes virus 8 (HHV-8) has been linked to
primary effusion lymphoma.
Studies of occupational and environmental NHL risk have been inconsistent and
contradictory. The risk appears to be increased in several occupations including farmers,
forestry workers, and agricultural workers. Several studies have shown an increased risk
associated with herbicide exposure.

Diagnosis
The initial evaluation of a patient with suspected NHL must establish the histologic
subtype, the extent of disease, and the performance status of the patient. The patients
past medical history should be screened for a number of diseases, infectious agents, or
toxins that may be associated with NHL. These include immunodeficiency states, auto-
immune disease, and infections such as HIV, HTLV-1, EBV, and hepatitis B and C. The
presence or absence of B symptoms: night sweats, fevers, fatigue, weight loss, should be
determined. The physical exam needs to be directed to all potentially involved lymphoid
sites, including the standard lymphoid sites (e.g. cervical, supraclavicular, axillary,
inguinal nodes), liver and spleen, and Waldeyers ring. A tissue biopsy must be obtained;
an excisional biopsy of lymph node is always preferred over a fine needle aspirate. Only
an excisional biopsy of an intact node allows sufficient tissue for histologic,
immunologic, and molecular biologic assessment. In addition, a bone marrow biopsy is
performed as marrow involvement occurs in 30 to 50 percent of all patients with NHL.
To assess the extent of disease, CT scans of the neck, chest, abdomen, and pelvis should
be performed. In certain subtypes of lymphoma, such as diffuse large B cell NHL
(DLBC) and in Hodgkins lymphoma, a PET scan is performed in conjunction with the
CT scan.

Classification
In 1995 the Society for Hematopathology and the European Association of
hematopathologists jointly developed a classification of hematologic neoplasms for the
World Health Organization (WHO). The goals were to classify the lymphoid neoplasms
using a combination of morphology, immunotyping, genetic features, and clinical
syndromes. See slide # 37.
A straightforward way to think of the NHLs is as follows:
-Indolent: survival of untreated disease measured in years
e.g.: Follicular, Marginal Zone, Small Lymphocytic Lymphoma
-Aggressive: survival of untreated disease measured in months
e.g.: Diffuse Large B Cell, Anaplastic Large Cell, Peripheral T-Cell
-Highly Aggressive: survival of untreated disease measured in weeks
e.g.: Burkitt's, Precursor B Cell, Adult T cell

Staging
A clinical stage serves as a guide in determining prognosis and treatment. The goals of
staging are to determine the extent of disease and define the location of disease. The
staging system accepted for use in both NHL and HL is the Ann Arbor system (see slide
#22 )

Follicular Non-Hodgkins Lymphoma


Follicular NHL is the second most common lymphoma in the U.S., accounting for 30%
of all NHLs. Follicular lymphoma is a disease of older adults, with a slight female
predominance. While the clinical course of this disease tends to be indolent, it is not
usually curable. Follicular lymphoma is composed of a mixture of centrocytes (cleaved
follicle center cells) and centroblasts (large non-cleaved follicle center cells) and by
definition has at least a partially follicular pattern. The proportion of centroblasts varies
from case to case and the clinical aggressiveness of the tumor increases with increasing
number of centroblasts. The tumors are assigned a grade (1-3) based on the number of
centroblasts counted per high power field. The tumor cells of follicular lymphoma
express surface immunoglobulin, are CD10+, and CD5-. A characteristic translocation:
t(14;18)(q32;31) is present in 85% of cases; this translocation joins the BCL-2 gene to an
IgH segment and leads to constitutively high levels of BCL-2 protein which is a potent
inhibitor of apoptosis. The average survival for a patient with FL is 6-10 years. Disease
progression may be slow and not need immediate treatment. Treatment for advanced
disease includes CVP-R (cyclophosphamide, vincristine, prednisone, rituximab) or
CHOP-R (cyclophosphamide, doxorubicin, vincristine, prednisone, rituximab). Five to
10% of cases may transform to a more aggressive lymphoma, such as diffuse large B cell
lymphoma.

Diffuse Large B Cell Non-Hodgkins Lymphoma (DLBCL)


DLBCL NHL is the most common lymphoma. Patients typically present with a rapidly
enlarging symptomatic mass. Systemic symptoms: fatigue, weight loss, fevers are
present in 30% of patients at diagnosis. Up to 40% of cases are extranodal: common
extranodal sites include the GI tract, bone, and CNS. DLBCL arises from a mature B-
cell, usually a centroblast or immunoblast. It is a neoplasm of large B cells whose
nuclear size is more than twice the size of a normal lymphocyte. The malignant cells
express one or more B cell antigens (CD19, CD20) as well as CD45 and surface
immunoglobulin. In addition to occurring de novo, DLBCL can arise through the
transformation of many different types of low grade NHL. Standard treatment for stage I
and II disease includes CHOP-R x 3 cycles followed by local radiation. Therapy for
stage III and IV disease consists of 6-8 cycles of CHOP-R. Prognosis in DLBCL is
highly associated with the International Prognostic Index (IPI score) which assigns a
point for each characteristic: Age >60, abnormal LDH, ECOG performance status 2,
Ann Arbor Stage III or IV, and the number of extranodal disease sites >1 (see slide # 43).
Hodgkins Lymphoma

Epidemiology
HL accounts for 1% of all new cancers in the U.S. The incidence of HL is bimodal, with
the first peak occurring in adults 20 to 30 years of age and the second peak after age 50.
In young patients (less than 10 years of age) there is a 3:1 male to female ratio. This
imbalance returns to an equal ratio in adult disease.

Etiology
The etiology of HL is unknown, but there does appear to be a causal association with
EBV in up to 40% of cases. There are also familial and geographic clusters to suggest an
inherited susceptibility, environmental, or infectious contribution to the etiology of HL.
In Western countries, there tends to be a higher incidence in families of higher social
status, advanced education and smaller family size. This suggests delayed exposure to
common infectious or environmental antigens.

Diagnosis and Classification


The HLs are defined by the presence of Reed- Sternberg cells in a background of non-
neoplastic cells. Cases are sub classified according to the morphology and
immunophenotype of the RS cells and the composition of the cellular background.
Hodgkin lymphoma is comprised of two disease entities: Nodular lymphocyte
predominant HL and Classical HL. Nodular lymphocyte predominant HL is now
considered to be one of the indolent NHLs, and not true HL.
Classical HL is further subdivided into the following four subtypes: nodular sclerosis
classical HL, mixed cellularity classical HL, lymphocyte depleted classical HL, and
lymphocyte-rich classical HL. Hodgkins disease almost always originates in the lymph
nodes and spreads through the lymphatics in an orderly fashion.

Staging
A clinical stage serves as a guide in determining prognosis and treatment. The goals of
staging are to determine the extent of disease and define the location of disease. The
staging system accepted for use in both NHL and HL is the Ann Arbor system (see slide )

Treatment and Prognosis


Patients with Stage I-II disease are usually categorized as having favorable or
unfavorable disease based on the bulk of disease, systemic symptoms, the erythrocyte
sedimentation rate, and the number of disease sites. Patients with Stage I or II disease are
considered unfavorable of they have ANY of the following characteristics: an ESR >50,
and ESR >30 plus B symptoms, a mediastinal mass exceeding 0.35 of the intrathoracic
diameter, or four or more sites of disease.
In the U.S. the favored chemotherapy regimen in ABVD: adriamycin, bleomycin,
vinblastine, and dacarbazine/
Stage I/II, favorable prognosis: ABVD x 3-4 cycles, followed by involved field radiation
to the initially involved lymph node site -or- chemotherapy alone x 6 cycles or-
radiation alone to the involved site. However, it appears that combined modality
treatment with chemo and radiation may result in a decreased recurrence risk, therefore
this approach is preferred.
Stage I/II: unfavorable prognosis: ABVD x 4-6 cycles, followed by radiation.
Stage III-IV: Chemotherapy with ABVD. Patients receive CT scans throughout the
course of treatment course and are to receive 2 cycles of chemotherapy beyond best
response, this generally translates into 6-8 cycles.
Involved field radiation may be recommended for sites of bulky disease or when there
has been less than a complete remission to chemotherapy.

The cure rate for all patients with HL approaches 90%.


Laboratory objectives
Evaluate three cases in order to

Reinforce the definition of acute leukemia.


Workshop: Hematopathology II
Acute leukemias Highlight your knowledge of the four major
categories of acute myeloid leukemia [WHO
classification (2008)].
Division of Hematopathology
Department of Pathology Know the role of flow cytometry and
cytogenetic studies in the characterization of
UPMCHS acute leukemia.

Review question: What is the difference


between leukemia
leukemia and lymphoma?
lymphoma?
Leukemia Neoplastic proliferation of
hematolymphoid cells (myeloid or lymphoid) that
presents with involvement of bone marrow and, What is the major difference between diseases
usually, peripheral blood.
that are called acute leukemia
leukemia and diseases
that are called chronic leukemia?
leukemia?
Lymphoma Lymphoid neoplasm generally
presenting as a solid tissue proliferation or
infiltrate. The degree of differentiation of
the neoplastic cells
Do myeloid neoplasms ever present as a tissue
mass?
Myeloid sarcoma (a.k.a. granulocytic sarcoma or
chloroma)

Fig. 13-1, Kumar:


Robbins and Cotran:
Pathologic Basis of
Disease, 7th ed.
Acute leukemia
Neoplasm of clonal myeloid or lymphoid
stem cells usually involving bone marrow
This shows the cellular compartments in which the neoplastic
Acute with 2 major characteristics:
lymphoblastic
cells are accumulating in various hematolymphoid neoplasms
leukemias Impaired differentiation
Survival advantages

Acute myeloid Accumulation of abnormal blasts, leading to


leukemias
suppression and dysfunction of maturing
Chronic myelo-
myelo- Mature hematopoietic cells
proliferative B, T or NK cell
disorders neoplasms

1
What would you expect as a clinical
Detection of acute leukemia presentation for acute leukemia?

Clinical suspicion Symptoms and signs of anemia, neutropenia


and thrombocytopenia
CBC with differential & peripheral blood Fatigue
film review Fever and infections
Spontaneous bleeding, petechiae, ecchymoses,
gum bleeding
Bone marrow examination with flow
cytometry and cytogenetic analysis
Lymphadenopathy or infiltration of solid tissue is less
common

What findings would you expect with


the CBC and peripheral blood film review? Case 1
The patient is a 22-year-old male presents with
abdominal pain, nausea, vomiting and sore throat.
Bicytopenia or pancytopenia should raise
suspicion of acute leukemia His history is significant for non-metastatic Ewing
sarcoma involving T4 spinal vertebrae (2 years
prior to this presentation). At that time, he was
Usually, blasts seen in blood treated with resection, chemotherapy and radiation.

Physical exam was fairly unremarkable, except for


well-healed surgical scars.

Peripheral blood
Patient Normal range
L (6.0 - 17.5)
WBC 4.6 x 109/L (3.8-10.6)
RBC 2.67 x 1012/L (4.13-5.57)
HGB 10.2 g/dL (12.9-16.9)
Hct 28.7 % (38.0-48.8)
MCV 107.6 fl (82.6-97.4)
RDW 16.1 % (11.8-15.2)
PLT 55 x 109/L (156-369)

Differential:
% Absolute Absolute Normal Range

Neutrophils 45 % 2.1 x 109/L (2.3-7.8)


Lymphs 33 % 1.5 x 109/L (0.8-3.6)
Monos 16 % 0.7 x 109/L (0.3-0.9)
Baso 1% 0.1 x 109/L (0.0-0.1)
Blasts 5% 0.2 x 109/L

Nucleated RBC/100 WBC 3

2
Peripheral blood
The manual 500 cell bone marrow differential count
demonstrated the following:

Patient % Normal Range (%)

Blasts 29% (0.0 - 2.0)


Promyelocytes 0 (2.0 - 4.0)
Myelocytes 2 (8.0 - 16.0)
Metamyelocytes 7 (10.0 - 25.0)
Bands 7 (9.0 - 18.0)
Segmented PMN 6 (7.0 - 14.0)
Monocytes 4 (0.0 2.0)
Pronormoblasts 1 (0.0 - 1.0)
Normoblasts 40 (16.0 - 32.0)
Lymphocytes 3 (11.0 - 23.0)
Plasma cells 1 (0.0 3.0)

Bone marrow aspirate smear Bone marrow aspirate smear

Characterization of leukemic clone

Bone marrow examination


Aspirate smear
Trephine biopsy

Phenotype (Myeloid, B, or T)
Immunophenotype
Flow cytometry
Cytochemistry

Cytogenetics

3
Cytochemical studies can be performed on the
air dried peripheral blood or aspirate smears

Reaction Normal reactive cells Diagnostic utility

Myeloperoxidase Neutrophils, eosinophils AML

Non-specific Monocytes AML with monocytic


esterase differentiation

Myeloperoxidase cytochemical
reaction considered generally specific
for myeloid differentiation

Purpose of flow cytometric


analysis for acute leukemia
Lineage assignment
Myeloid blasts
Precursor T-lymphoblasts
Precursor B-lymphoblasts

Documentation of immunophenotype for


Cytochemical detection of subsequent detection of a low level
non-specific esterase is evidence
non- residual or recurrent disease
for monocytic differentiation.

Review: Flow cytometry in the analysis of CD2 CD33

hematolymphoid neoplasms FITC PE

Individual cells in fluid suspension pass thru a CD2 CD33

laser beam

Collection of 6-8 properties of light emission for


each individual cell per aliquot of sample
Forward and side light scatter characteristics CD45 CD34

Fluorescent intensity of 4-6 fluorochrome-labeled


antibodies directed to cellular antigens of interest PerCP APC

CD45 CD34
Data displayed in 2-D dot plots

4
In this case, the blasts are positive for CD45,
CD45,
CD34 and CD33,
CD33, but negative for CD2

Reference: Preffer FI in Colvin RB, et al, eds. Diagnostic


Immunopathology, 2nd ed. Raven Press: New York. p. 726.

Case 1: Flow cytometry performed on bone marrow.


Some phenotypic markers useful for
evaluation of acute leukemias Grans and Monos

CD34
SSC

CD45 Present on all leukocytes; also known as


leukocyte common antigen (LCA)
CD34 Present on pluripotent hematopoietic stem cells
and progenitor cells of many hematopoietic cell
lineages Lymphocytes
CD117 Present on hematopoietic stem cells, progenitor FSC CD45
cells and mast cells

Some lineage associated antigens


HLA-DR
CD11b

T-lineage CD2, CD3, CD4, CD5, CD8

B-lineage CD19, CD20

Myeloid CD13, CD33, Myeloperoxidase


CD13 CD33

Bone marrow
chromosome analysis

47,XY,t(3;21)(q26;q22),del(7)(q22q34),
+21[6]/48,XY,t(3;21)(q26;q22),+8,+21[13]

5
Review: Banding pattern
nomenclature
(e.g. normal X-chromosome)
Short arm is p
Long arm is q
Each arm is divided into
regions based on
prominent bands.
Numbered outward from the
centromere
Each region is further
subdivided into bands and
sub-bands
Fig. 5-22, Kumar: Robbins and Cotran:
Pathologic Basis of Disease, 7th ed.

Evolution of AML classification


Subtype-specific biology

Case 1: Diagnosis Morphology

Therapy-
Therapy-related acute myeloid leukemia

Cytogenetic/genetic-based classification

WHO classification of acute Abbreviated prognostic grouping of AML


myeloid leukemia based on chromosomal findings

I. AML with recurrent genetic abnormalities Favorable Inv16 or t(16;16)


t(8;21)
II. AML with myelodysplasia-related changes t(15;17)
Close biologic relationship between II and III
Intermediate +8
III. Therapy related AML & myelodysplastic normal
syndromes (MDS) Unfavorable -7, t(6;9),
inv(3)(q21;q26.2),
IV. AML not otherwise specified Complex abnormalities (>3)

6
Standard therapy for AML
18-60 y.o. Follow up on patient (case 1)
Induction chemotherapy Patient attained CR after induction, but
Goal: Empty out marrow; induce remission at high risk for relapse. Thus,
(defined as <5% marrow blasts) hematopoietic stem cell transplant
planned. However, no siblings.
Risk adapted post-remission chemotherapy AML was refractory to several courses
Goal: Reduce undetectable burden of of chemotherapy.
leukemic cells to effect long-term disease-free
survival He underwent allo-MUD peripheral
blood stem cell transplant

Follow up on patient (case 1) Case 2


Suffered severe graft-versus-host The patient is a 50-year-old male who over
disease involving the liver and the last 2 weeks has become fatigued. He
infections. has also noted bleeding of his gums. He has
had a low grade fever that has persisted
AML relapsed. despite antibiotic therapy.
A decision was made against heroic
measures, he was made comfortable Physical exam is notable for petechiae and
and died 9 months after diagnosis. ecchymoses of all extremities.

Patient Normal range


L (6.0 - 17.5)
WBC 53.4 x 109/L (3.8-10.6)
RBC 2.70 x 1012/L (4.13-5.57)
HGB 9.1 g/dL (12.9-16.9)
Hct 25.0 % (38.0-48.8)
MCV 92.5 fl (82.6-97.4)
RDW 15.9 % (11.8-15.2)
PLT 12 x 109/L (156-369)

Differential:
% Absolute Absolute Normal Range

Neutrophils 2% 1.07 x 109/L (2.2-7.7)


Lymphs 6% 3.20 x 109/L (0.8-3.6)
Monos 2% 1.07 x 109/L (0.3-0.9)
Myelocytes 1% 0.53 x 109/L
Metamyelocytes 1% 0.53 x 109/L
Leukemic
promyelocytes/
blasts 88% 46.97 x 109/L

7
Characterization of leukemic clone

Bone marrow examination


Aspirate smear
Trephine biopsy

Phenotype
Immunophenotype
Flow cytometry
Cytochemistry

Cytogenetics

8
Case 2: Flow cytometry performed on bone marrow.
The manual 500 cell bone marrow differential count
demonstrated the following

Blasts/leukemic 68.4% (0.0 - 2.0)


promyelocytes
Myelocytes 0.3 (8.0 - 16.0)
Metamyelocytes 1.0 (10.0 - 25.0)
Bands 0.3 (9.0 - 18.0)
PMNs 0.3 (7.0 - 14.0)
Pronormoblasts 0 (0.0 - 1.0)
Normoblasts 14.3 (16.0 - 32.0)
Lymphocytes 13.0 (11.0 - 23.0)
Plasma cells 1.7 (0.0 3.0)

Some phenotypic markers for


evaluation of acute leukemias
CD45 Present on all leukocytes; also known as
leukocyte common antigen (LCA)
CD34 Pluripotent hematopoietic stem cells and
progenitor cells of many hematopoietic cell
lineages
CD117 Present on hematopoietic stem cells, progenitor
cells and mast cells

Some lineage associated antigens

T-lineage CD3, CD4, CD5, CD8

B-lineage CD19, CD20

Myeloid CD13, CD33, Myeloperoxidase

9
Schematic representation of t(15;17)(q22;q21)

p11.2
p11.1
q11.1 p13
q11.2
q12 p12
q13 p11.2

Case 2: Diagnosis
q14 p11.1

q15 q11.1
q11.2
Reciprocal
q21 q12 exchange
q21
q22 q22 q21 of segments
q22
q23

q24
q23 Acute promyelocytic leukemia
q24
q25
q25 AML with t(15;17)(q22;q12), (PML/RAR)
q26

Normal #17 with Der #15


Normal #17 Der
#15 with a break
#15 #17
a break at q21
at q22
t(15;17)(q22;q21)
Courtesy of Malini Santhanoori, Pittsburgh Cytogetics Laboratory

WHO classification of acute Abbreviated prognostic grouping of AML


myeloid leukemia based on chromosomal findings

I. AML with recurrent cytogenetic abnormalities Favorable Inv16 or t(16;16)


t(8;21)
t(15;17)
II. AML with multilineage dysplasia
Intermediate +8
normal
III. AML & MDS, therapy related
Unfavorable -7, t(6;9),
inv(3)(q21;q26.2),
IV. AML not otherwise categorized Complex abnormalities (>3)

Acute promyelocytic leukemia with t(15;17) Acute promyelocytic leukemia: therapy


(a.k.a. APL and AML-M3)
Very important to promptly recognize this subtype of AML
because of its association with severe disseminated
intravascular coagulation (DIC) and potentially fatal
Translocation of retinoic acid receptor alpha (RAR) hemorrhage.
locus on chromosome 17 to PML locus on chromosome
15. Pharmacologic doses of vitamin A derivative, all-trans-
RAR is a transcription factor, normally. retinoic acid (ATRA), a ligand for RAR, will induce
PMLRAR fusion protein represses expression of target genes remission of APL
and blocks granulocyte differentiation at blast/promyelocyte Binds to fusion protein, leading to dissociation of
stage co-repressor complexes & degradation of fusion protein
Promyelocytes are then able to differentiate and undergo apoptosis
Second mutation likely is needed for pathogenesis of
APL Patients will still need cytotoxic chemotherapy to eliminate
the leukemic stem cells.

Cure rate with standard APL therapy is 60-70%

10
Case 3 Patient Normal Range (2-6 years)

WBC 23.0 x 10^9/L (5.0 - 17.0)


The patient is a 4-year-old boy who has had RBC 1.71 x 10^12/L (3.90 - 5.30)
HGB 6.30 g/dl (11.5 - 13.5)
an acute upper respiratory infection that has Hct 19.40% (34.0 - 40.0)
not resolved after one week of antibiotic MCV 76.0 fl (75.0 - 87.0)
therapy. His mother states that he has had a PLT 50 x 10^9/L (150 - 450)
persistent low grade fever, joint pain and
cough for approximately two weeks.
Differential:
% Absolute Normal Range (2-6 years old)
Physical examination reveals mild
Neutrophils 5% 1.15 x 10^9/L (1.50 - 8.50)
generalized lymphadenopathy and mild Lymph 15% 3.45 x 10^9/L (3.00 - 9.50)
splenomegaly. Other cells 80% 18.40 x 10^9/L

Characterization of the
neoplastic cells
Bone marrow examination
Aspirate smear
Trephine biopsy

Phenotype
Immunophenotype
Flow cytometry
Cytochemistry

Cytogenetics

11
The differential count of the bone marrow aspirate
demonstrates the following:

Patient ( %) Normal range (%) (2-6 years)

Blasts 89.0 (0.0 - 0.8)


Promyelocytes 0.0 (0.0 - 3.5)
Myelocytes 0.0 (0.0 - 8.5)
Metamyelocytes 0.0 N/A
Bands 0.0 (4.2 - 31.2)
PMNs 1.0 (9.6 - 66.9)
Pronormoblasts 0.0 (0.0 - 1.4)
Normoblasts 3.0 (0.0 - 34.3)
Lymphocytes 7.0 (11.2 - 29.3)

AML Summary of cytologic features


of acute leukemias
AML ALL
Blast size Larger, usually uniform More variable, usually
small to medium
Cytoplasm Moderate amount; granules Usually scant; coarse
frequent granules rarely
Auer rods Present in 60-70% Not present

Chromatin Usually fine May be fine or more


coarse
Nucleoli 1-4, often prominent Absent or 1-2, often
inconspicuous

ALL

Case 3: Flow cytometry performed on bone marrow.

Immunopheno-
Immunopheno-
type of leukemic
cells:

Positive for
CD19, CD22,
CD10, TdT &
CD34.

Negative for
markers of
myeloid
differentiation.

12
Some phenotypic markers for
evaluation of acute leukemias
CD45 Present on all leukocytes; also known as
leukocyte common antigen (LCA)
CD34 Pluripotent hematopoietic stem cells and
progenitor cells of many hematopoietic cell
lineages
TdT Terminal deoxynucleotidyltransferase. Expressed
by lymphoblasts and, less commonly, leukemic
myeloblasts.

Some lineage associated antigens

T-lineage CD3, CD4, CD5, CD8


B-lineage CD19, CD20, CD22

Myeloid CD13, CD33, Myeloperoxidase

Prognosis of
acute lymphoblastic leukemia
To be covered by Dr. L. Krishnamurti, Pediatric
Case 3: Diagnosis Malignant Hematologic Conditions (Thurs., Jan.
15th)

B lymphoblastic leukemia with So far, this patients presentation suggests a


hyperdiploidy favorable prognosis because of the following:
Age < 10 y.o.
WBC < 50 x 10^9/L
Cytogenetic findings, including hyperdiploidy and
trisomy 4, 10, and 17

Acute lymphoblastic leukemia Therapy for childhood ALL


3-4 risk groups based on prognostic factors
Acute lymphoblastic leukemia (ALL) accounts for (Low)
25% of acute leukemia cases. Standard
About 2/3rds of the cases of ALL occur in children. High
ALL accounts for 1/3rd of childhood cancers Very high: Philadelphia chromosome positive or
It is the cancer with highest incidence in childhood t(4;11)/MLL-AF4

Precursor B-cell phenotype accounts for ~85% of Induction chemotherapy is similar


cases of acute lymphoblastic leukemia
Precursor T-lymphoblastic leukemia/lymphomas are
much less common Postremission treatment differs based on risk
group
Refer to Dr. Krishnamurtis lecture

13
Hematopathology II: Acute Leukemias
Children with acute lymphoblastic leukemia
Key take-home points
have cure rates approaching ~80-
~80-85%
with contemporary therapy Acute leukemias are neoplastic proliferations of clonal
hematopoietic stem cells, resulting in an accumulation of
blasts.
Cure rate is about 35% in adults 60.
Standard evaluation for acute leukemia includes
morphologic assessment, flow cytometry and cytogenetic
Cure rate is about 5% in adults > 60. studies.

Acute promyelocytic leukemia is a subtype of acute


myeloid leukemia that is associated with severe coagulopathy
and potentially fatal, spontaneous hemorrhage.

Acute lymphoblastic leukemia is rare, but is one of the most


common cancers occurring in children.

14
Objectives
Define palliative care and understand the
need for palliative care
Overview of Palliative Care Distinguish between palliative care and
hospice
Winifred Teuteberg, MD Describe different aspects of palliative
University of Pittsburgh School of care and how they are provided to patients
Medicine and families
January 2009

Dictionary Definitions Textbook Definition


Palliate: To reduce the violence of; to
moderate the intensity of Palliative medicine is the study and
Merriam-Webster Dictionary management of patients with active,
Palliative: Reducing the severity of; progressive, far-advanced disease, for
denoting the alleviation of symptoms whom the prognosis is limited and the
without curing the underlying disease. focus of care is quality of life.
Stedmans Medical Dictionary, 27th edition
Oxford Textbook of Palliative Medicine, 3rd Edition

In other words Hospice


Palliative care focuses on maximizing Provides 24/7 palliative care to terminally
quality of life for patients with life- ill patients and services to their families
threatening illness by Can be inpatient or outpatient
addressing physical, spiritual and Medicare hospice benefit
psychosocial distress 65 or older
optimizing clinician-patient communication Prognosis of 6 months or less
assisting in advanced care planning and Goals of care must be palliative NOT curative
coordination of care Most other insurers now also cover
using an interdisciplinary approach hospice

1
Palliative Care and Hospice History of Palliative Care
Death in the 20th century
Conquering of illness
Penicillin, polio vaccine, cancer chemotherapy
Palliative Care
Shift in focus of medicine towards curing illness and
prolonging life
Institutionalization of medicine
Care moved from home to hospitals and clinics
Most deaths now occur outside of the home
Hospice
Change in popular experience of death
No longer commonplace
Fear of the unfamiliar

History of Palliative Care History of Palliative Care


Philosophy Cicely Saunders opens St. Christophers
Modern medicine often falls short in the care Hospice in 1967 in London
of patients at the end of life The hospice movement comes to the US
Active care for patients and their loved ones Early home-based programs, 1974
should not end when illness is no longer
curable First palliative care unit
By focusing on the treating the whole patient Montreal, 1975
and his or her loved ones, we can improve
quality of life at the end of life

History of Palliative Care Who should receive palliative care?


Patients with life-threatening illness who
Emergence of palliative care programs in
have distressing symptoms or problems
US teaching hospitals in the 1990s
related to their illness including
AAHPM board certification 1996
have spiritual or existential distress
ACGME accreditation as subspecialty with need assistance discussing goals of care and
government subsidy for fellowship training code status
2008
need assistance with coordination of care
Fellowship training in palliative care
have loved ones who need support
Moving from academic centers into the
community

2
When should palliative care start? Who provides palliative care?
Traditional Model of Care
Interdisciplinary team approach to care
Curative Care Palliative Better suited to treat total patient (mind, body,
Care
spirit)
Onset of disease Death
Allows for collaboration in problem solving
Optimal Model of Care Allows for support of those involved in caring
for patients
Curative Care Palliative
Care

Onset of disease Death

Adapted from Lipman, AG, J of Pharmaceutical Care Pain Symptom Control, 1999

Who provides palliative care? Where is palliative care delivered?


Physicians Psychologists Acute care hospital
Nurse practitioners Chaplains Palliative care consult service
Dedicated palliative care unit
Physician assistants Nurses
Outpatient clinic
Social workers Complementary Extended care facility
Bereavement medicine practitioners
Rehabilitation facility
counselors Palliative home care
Hospice
Home
Inpatient

How is palliative care delivered? Clinician-patient communication


Focus on treating the total patient and his
or her loved ones Hear the patients story
Use open-ended questions
Four main aspects of care
Make eye contact
Clinician-patient communication
Respond to patients emotions
Assessment and treatment of symptoms
Demonstrate empathy
Psychosocial, spiritual and bereavement
support Conversation should be patient-centered
Care coordination

3
Clinician-patient communication Clinician-patient communication
Discuss goals of care
Discuss patients hopes and fears Plan for continued care
Outline medical facts Address patients questions and concerns
Identify realistic and attainable goals Reassurance of non-abandonment
Advanced directives and code status Plan for follow-up
Discuss and offer advice on decisions
regarding artificial nutrition, prolonged life
support and cardiopulmonary resuscitation

Assessment and treatment of Assessment and treatment of


symptoms symptoms
Provide therapy with attention to the
Pain Shortness of breath following
Fatigue Nausea Underlying cause of the symptom
Depression Constipation Patients history of past effective therapies
Anxiety Anorexia Mode of delivery (IV, SC, IM, PO, PR)
Insomnia Delirium Side-effect profile
Availability and cost
Patient prognosis and personal goals

Assessment and treatment of Psychosocial, spiritual and


symptoms bereavement support
Titration and adjustments of therapy Psychosocial support
Carefully assess response Determining patient and family coping
Maximize comfort Assessing support systems
Minimize bothersome side-effects Offer assistance from a social worker or
psychologist for the patient and/or loved ones

4
Psychosocial, spiritual and Psychosocial, spiritual and
bereavement support bereavement support
Address spiritual or existential concerns
Helpful psychosocial questions Are you a spiritual person?
How has the illness affected you? Has faith been important to you?
What has helped you get through this? Do you ever think about why this has
How have your loved ones been affected? happened to you?
What has helped you during difficult Offer support from a chaplain or offer to
situations in the past? bring in the patients own spiritual advisor

Psychosocial, spiritual and Psychosocial, spiritual and


bereavement support bereavement support
Bereavement support is an avenue for
Address death and dying continued care of loved ones after the
Have you been thinking about dying? patient has died
What thoughts or worries do you have? Organized bereavement program through
What plans have you made? hospice or palliative care program
Simply allowing the patient to talk about Personal cards or letters, phone calls when
death is often very helpful appropriate
Attending a funeral or memorial service if it
Answer questions about what to expect
feels comfortable

Care coordination Sources


The Oxford Textbook of Palliative Medicine, 3rd
Helping to navigate the healthcare system edition, Oxford University Press, 2004.
Morrison SR, Meier DE. Palliative care. NEJM
Care coordination involves 2004.
Addressing and readdressing goals of care Lipman AG. Pain management and palliative
Referral to hospice, palliative care or other care: a natural-and needed-synergy. American
Pain Society Bulletin 2002.
interdisciplinary care services Billings JA. What is palliative care? J of Palliative
Addressing financial planning Medicine 1998.
Arranging support for loved ones and Billings JA. Caring for dying patients and their
encouraging or facilitating their visits families. Cecil Textbook of Medicine, 22nd
edition, 2004.

5
......
PALLIATIVE CARE AND HOSPICE


Lovejoy, Arthur O. 1961. Reflections on Human Nature. Balti and shortness of breath are the most feared by patients, along
more: Johns Hopkins Universiry Press. with fear of the unknown. Because considerable medical
LuzZartO,Moshe Hayyim. 1982. The Kno~ing Heart: The Phi technology now exists that can postpone death, costs are
0/ God's Oneness, rr. Shraga Stlverstem. Jerusalem:
losophy often high and most people die in hospitals or nursing
Feldheim. homes, attended by strangers. For patients who die at home,
Reich, Warren T. 1989. "Speaking of Suffering: A Moral Ac the financial, physical, and emotional burdens of caregiving
counr of Compassion." Soundings 72( 1): 83-108. fall heavily on isolated nuclear families, and predominantly
Scarry, Elaine. 1985. The Body in Pain: The Making and Unmak on women.
ingo/the World. New York: Oxford Universiry Press.
Simundson, Daniel]. 1980. Faith under Fire: Biblicallnterpreta
tions o/Suffering. Minneapolis, MN: Augsburg. The Early Days of the Hospice Movement
Solomon, R. C. 1972. "Hegel's Concept of 'Geisr.'" In Hegel: A The "hospice movement," as it is popularly known, is
Collection o/Critical Essays, ed. Alasdair C. MacInryre. Notre generally agreed to have started in 1967 with the opening of
Dame, IN: Universiry of Notre Dame Press.
Sf. Christopher's Hospice in London under the charismatic
Whitehead, Alfred North. 1974. Religion in the Making. New leadership of Dr. Cicely Saunders. Hospices were a feature of
York: New American Library. the Middle Ages in Europe, usually run by religious orders,
Whitehead, Willard, III, and Kuhn, Wolfgang F. 1990. "Chronic and offered safety, healing, and rest to weary and often
Pain: An Overview." In va!. 1 of Chronic Pain, pp. 5-48, ed. wounded travelers. It was therefore an obvious name to give
Thomas W. Miller. Madison, CT: International Universities
to institutions founded in France, Ireland, and England
Press.
around the turn of the nineteenth century to care for the
dying. What made Sf. Chrisropher's and those that followed
different was Saunders's insistence on scientific rigor and
professional education and training.
;.i;

Few people were likely to return home from these


",
pioneer hospices, but they would get skilled relief of their
PALLIATIVE CARE AND

pain and suffering, whatever its nature or origin, in a (


HOSPICE

~l'
sensitively nourished environment oflove, safety, and peace
for them and their relatives. That better care ofthe dying was
needed was attested to by many comments of the dying i;
'..
; I ~
The terms palliative care and hospice are frequently used
themselves, grieving relatives who looked back in horror and !, ;;
sadness at what patients had had to suffer, and by an i i'l
interchangeably to describe an approach to the care of
individuals who are likely to die in the relatively near future
increasing number of papers published in reputable medical 1 jt
journals detailing this suffering. At what most must have felt

from serious, incurable disease, for whom the principal focus the lonelieSt time of their life, the dying described them .',\'. I"

of care is quality of life and support for the patient's family. selves as having no attention paid to their suffering and 1_.;' 1~. 1
The terms gained currency in the last third of the twentieth getting no answers to their questions. They not only experi ~'.l
centuty as a result of significant changes in the leading causes enced a spectrum of physical suffering, but endured fear,
tl"
I.,i'.'. .
f I

~,' ".1/'i
of death in the developed countries of the industrialized depression, loneliness, and a sense of being undervalued by
world. In these countries prior to 1900, most people died
relatively quickly, usually from acute, infectious diseases.
society. They often felt deserted by their doctors, whom they ~'i
~
found difficult to trust when so rarely were they told the true 1

They typically died at home, attended by family and friends. nature of their mortal illness and what lay ahead. The dying
Because little in the way of medical technology was available either lived with relatives who, hoping to protect them, (
to prevent or delay death, the costs of care were low, and the conspired with the doctors to keep them in ignorance, or in , 1
.
dying person and her caregivers could emphasize the inter hospitals where the focus of attention was sophisticated I
personal and spititual aspects of dying. investigations and aggressive treatments designed to cure.
By contrast, at the beginning of the twenty-first centuty
.i i
most people in the developed world die from chronic,
degenerative diseases such as cancer, cardiovascular disease, Palliative Care
lung disease, and degenerative neurological disease. Death It was soon recognized that the word hospice, though widely
usually follows a prolonged period of progressive loss of undersrood and accepted by the English-speaking world,
function and numerous distressing symptoms, ofwhich pain would never be universally acceptable because it had a

ENCYCLOPEDIA OF BIOETHICS 3rd Edltion 1969

..,....

PALLIATIVE CARE AND HOSPICE


different meaning in French and Spanish. Balfour Moum, of relatives-are approptiate from the time of diagnosis. In
who established a specialized unit at the Royal Vicroria an attempt to produce a more succinct definition, called for
Hospital in Montreal in 1974 based on the principles he had when palliative medicine was recognized as a medical spe
learned at Sf. Christopher's, coined the term palliative care cialty in the United Kingdom in 1987, palliative care was
to circumvent the language problem. Because it was already defined as the study and care of patients with active,
in medical parlance, the healthcare professions accepted this progressive, far-advanced disease and a limited life expect
term. Today physicians working in this field describe them ancy, for whom the focus of care is the quality of life.
selves as palliative medicine physicians and nurses as pallia
This definition does not limit palliative care to people
tive care nurses, while the services where they work (the
with malignant disease, nor does it state a prognosis in terms
original hospices) are called specialist palliative care services.
of months or weeks. Ie is worded so as not to be confused
The acceptance and adoption of palliative care by other with care of the elderly, care of the chronically ill, or care of
healthcare professionals has not always been straightforward, the incurable (which would embrace many of the conditions
however. Many claimed they were already providing it, in seen daily by physicians). Unfortunately, it omits mention
spite of the many reportS of uncontrolled suffering. A few of relatives, or the fact that palliative care can be provided
suspected it was euthanasia under another name. Some were only by an interdisciplinary team. Its strength lies in its
convinced it was not based on well-proven therapeutic unequivocal focus on quality of life rather than on cure Ot
regimens but was simply complementary or alternative prolongation of life, the declared objectives of much of
medicine applied to the dying. Others questioned why it modern medical care.
seemed to focus on the care of people with malignant disease
]. Andrew Billings, who in 1998 reviewed many of the
when patients suffering end-stage cardiac, neurological, and
competing definitions, concluded that the following defini
respiratory disease, or AIDS, had similar and often unmet
tion achieves the best balance of completeness and concision:
needs (Addington-Hall).
Palliative care is comprehensive, interdisciplinary
care, focusing primarily on promoting quality of
Definition and Scope life for patients living with a terminal illness and
It was easy to define hospice care when it focused on the final for their families. Key elements for helping the
days of life. Ie soon became apparent, however, that better patient and family live as well as possible in the face
care was needed long before this terminal phase. Hospital of life-threatening illness include assuring physical
comfort, psychosocial and spiritual support, and
based teams were created to provide care for patients in the
provision of coordinated services across various
hospital units where they were still receiving treatmenrs
sites of care. (p. 80)
inrended to cy.re or slow the progress of their underlying
disease. Thing~tould also be improved when people were Two further statements, endorsed by the government
being cared for at home, where most wanted to remain as of the United Kingdom, have been found challenging and
long as possible, though, contrary to what has always been helpful:
said, not necessarily to die there (Hinton; Ward). A range of
It is the right of every person who needs
services was developed to assist primary physicians caring for
it to receive high quality palliative
people at home, including home visits by nurses and other
care, irrespective of his or her
professionals and day-care units for patients who could be
diagnosis.
brought inro a center for clinical assessment and creative
It is the responsibility of every clinician
occupational therapy.
to provide high quality palliative
Palliative care was no longer synonymous with "care of care. (Doyle, p. 6)
the dying." Yet, as the field has developed, it has struggled to
In applying these principles in the complex, highly differen
define itself in a way that caprures its broader scope
tiated world of the health professions, it is helpful to note
reflecting its appropriateness for patients earlier in their
that palliative care can be provided at three levels: principles,
disease process, who are not imminently dying-without
techniques, and specialist care.
resorting to euphemisms chosen to disguise the fact that the
care is for people who, sooner rather than later, will die of Palliative care principles ate integtal to all good clinical
their illness. The most commonly used definition is that care, and they are applicable at every stage of a patient's care,
devised by the World Health Organization. It emphasizes whatever the nature of the illness. Every doctor and nurse
that the principles of palliation-the relief of physical, should be applying these principles, even when they are still

"J,;
psychosocial, and spiritual distress, and respect for the needs defining the nature and cause of an illness or its symptoms.
I
I

1970 ENCYCLOPEDIA OF BIOETHICS 3rd Edition


'.;,r ,~
it,.. '
" ...
"
PALLIATIVE CARE AND HOSPICE
;;..:
Palliative techniques are usually the responsibility of the value of her life, while remaining vigilant for the effects
rofessionals such as surgeons and interventional radiolo- of depression or social isolation, presents one of the most
P
gists, who, for examp I ' Insert
e, create ostomies, . stents, and profound clinical and ethical challenges in palliative care.
provide palliative radiation. None of these procedures is Yet, the skills for eliciting and responding to this form of
intended to cure, but each can bring about relief in suffering. suffering are seldom addressed in medical and nursing
Specialist palliative care is provided by those who have schools.
undergone specialist training as stipulated by their accredit The third concern is meaning of life. When, and only
ing professional body. In such countries and regions as the when, their physical suffering has been relieved and their
United Kingdom, Australasia, and Hong Kong, specialist families cared for, do dying people begin to ask existential
palliative care units are those where all senior doctors and questions. Though a diminishing proportion of people in
nurses are accredited specialists and where members of the West now claim to have a meaningful religious faith,
professions allied to medicine (physiotherapists, occupa more than 75 percent of dying people want to discuss the
tional therapists, clinical pharmacists, clinical psychologists, meaning of life, suffering, and death, and they may be
and music, art, and stoma therapists) have all had additional disappointed if no one is interested in helping them. Once
training pertinent to palliative care. Such services are usually again, in the absence of some training in the humanities,
affiliated with local medical and nursing colleges. doctors and nurses in the increasingly secularized Western
society find themselves ill-equipped to help with this issue.

Quality, Value, and Meaning of Life


As palliative care continues to develop, it is being recognized The Development of Palliative
that with the drugs and techniques currently available, and Care Worldwide
li

1
the increasing skills to use them, it is relatively easy to From the handful in operation in 1967, there are now more
!

I
achieve physical comfort, but that even when that has been than 6,200 palliative care programs in over 100 countries. In
achieved a person may still feel frightened, lonely, un its birthplace, the United Kingdom, palliative care services
wanted, or undervalued. Those working in the field now are readily and freely accessible to all. The National Health
realize that, beyond the management of physical symptoms, Service runs one-fifth of these services, and 25 percent of the
palliative care is primarily concerned with three things. operating costs of the others are met by government, the t;
f
vi.
First is quality of life. Many quality-of-life assessment balance being met from voluntary funding of more than
tools specific to palliative care are now available to healthcare
professionals (Clinch, Dudgeon, and Schipper; Higginson).
US$450 million annually. A typical palliative care in-patient
unit in the United Kingdom, with 10 to 100 beds, admits
,.

H
I'

:1
Each attempts to measure quality as perceived by the patient annually twenty to twenty-five patients per bed, where each
or relative and not by the attending professionals. Robust will stay for an average of eleven to fourteen days. The
,
"

research is now confirming what has long been suspected, portion of patients able to return home varies between 40 :1
that patients not given the information they seek experience and 60 percent, higher if there is an effective community
II
i'
more physical and psychosocial suffering and describe a palliative care service and a day unit. Seldom do more than ill
lower quality of life than those kept informed according to 15 percent of patients who have conditions other than 'i
;1
their wishes. To many people's surprise, this has proven to cancer receive palliative care in the United Kingdom, a 'I
be the case not only in the West but also in diverse cultures considerably smaller percentage than in the United States. !iI
and among peoples of various faiths in the Middle and
Far East.
Though palliative care services are being developed in I:

The second concern is value of life. As people approach


many countries, most are modeled on those of the United
Kingdom and the United States, rather than being designed f
death they increasingly wonder whether their lives have been
of any value to others and to the community, and whether
to meet local needs and cultures. Palliative care is still not
available to the 75 percent of the world's population, for
J
~'
they still have any value as persons when they are incapaci whom curative treatment of life-threatening disease is either
tated by a fatal illness, dependent on others, and, as they are unavailable or inaccessible. There are still only a relative
often reminded, expensive to care for. Surveys in the United handful of medical schools worldwide that include palliative
States have shown that patients' loss of independence and care in the curriculum, and fewer still where a specialist
fears of being a burden to others are more often the primary teaches it. Even when it is mentioned in undergraduate
motivations in requesting assisted suicide or euthanasia than medical courses it is rarely included in the training of
is physical pain (Emanuel et al.; Sullivan, Hedberg, and subspecialists who-in the West-provide the bulk of the
Fleming). Respecting the individual patient's assessment of care to critically ill patients. Only in those countries where

ENCYCLOPEDIA OF BIOETHICS 3rd Edition 1971


PALLIATIVE CARE AND HOSPICE


there are doctors working full time in the field is palliative more recent development in the United States. As recently as
care rapidly gaining credibility and acceptance. 1998, only 15 percent of U.S. hospjtals reported having any
services devoted to end-of-life care (Pan et al.). In a survey of
5,810 member hospitals by the American Hospiral Associa
Palliative Care in the United States tion in 2000, 13.8 percent of the 4,856 respondents re
The first hospice program in the United States opened in ported having a palliative medicine service, while 22.7
percent reported a hospital-based hospice program, and 42
Connecticut in 1974. Most early programs relied heavily for
percent reported a pain management service.
financial support on private, local philanthropy and grants.
Beginning in 1983, patients over the age of sixty-five could Inpatient palliative care units on the British or Cana
elect to receive a "hospice benefit" under the Medicare dian model are still relatively rare in the United States.
program. A patient certified by his physician as "terminally Hospital-based palliative care teams primarily provide con
ill" (defined as having a life expectancy ofsix months or less) sultation for symptom management, patient and family
may waive access to Medicare coverage of curative treat counseling, and conversations designed to determine appro
ments for the terminal illness, in return for a package of priate goals of care (Pan et al.). Financial pressures on acute
services aimed at symptom control and improved quality of care hospitals in the United States usually dictate the swift
life. These services would otherwise not be covered or would discharge (to home or nursing facility) of any patient for
be provided in an uncoordinated manner. The Medicare whom acute hospital interventions are no longer indicated.
hospice benefit (payable as a per diem reimbursement to This restricts the ability ofthe hospital palliative care team to
Medicare-certified hospice providers) includes nursing care assist in the course of the patient's dying. The role of the
in the home (up to sixteen to twenty hours per week, with team at that point is most often to assure as smOOth a transfer
temporary twenty-four-hour care available under limited as possible to another setting, which mayor may not include
"crisis" circumstances); medical appliances and drugs; home ongoing palliative care by specialist professionals.
makers, home health aides, and volunteers for personal and Unlike in Great Britain, where there are now more
respite care; physician services; short-term hospitalization; specialist palliative medicine physicians than oncologists,
physical and occupational therapy; psychological and spiri palliative medicine has not been recognized as a medical
tual support; social services; and bereavement counseling subspecialty in the United States. Beginning in 1996, how
(Center for Medicare Education). ever, the American Boatd of Hospice and Palliative Medi
Medicare requires hospices to conform to several proce cine began to administer a certifying examination for physi
dural and staffing requirements in order to receive federal cians who wished to be known for special competence in the
funds. Among rhe most significant requirements are that the field. A separate organization, the Hospice and Palliative
hospice must have a core, interdisciplinary team made up of Nurses Association, administers a certifying examination for
at least a physician, a registered nurse, a social worker, and a nurses and began a certification program for palliative care
chaplain or other counselor; that patients must have an nursing assistants in 2002.
identified primary-care provider in the home (usually a
family member or someone else who is available on a rwenty
four-hour-per-day basis); and that no more than 20 percent Ethical and Policy Issues in Palliative Care
of the total aggregate number of days ofcare provided by the and Hospice
hospice may be in inpatient settings. Many of the ethical issues that arise in the care of the dying
are similar to issues that arise in many other areas of
Since Medicare funding became available, the number
healthcare, such as truthfulness and confidentiality, decision
of hospice programs in the United States has increased
making authority in the professional-patient relationship,
dramatically. From 1982 ro 2000, the estimated number of
the appropriate use and allocation of technology and other
providers grew from 500 to 3,100. The number of patients
healthcare resources, the conduct of research, and the locus
served increased from approximately 1,000 to approxi
of ethical responsibility when care is provided by a team
mately 700,000 berween 1975 and 2000. Cancer patients
(Randall and Downie), Other issues are more commonly
made up 57 percent of hospice admissions in the United
associated with the care of the terminally ill, though not
States in 2000, followed by patients with heart disease
absent from Other arenas, such as decision making for
(l 00/0), dementia (60/0), lung disease (60/0), end-stage kidney
patients who have lost the capacity to make or communicate
disease (30/0), and end-stage liver disease (20/0) (NHPCO).
their own decisions, withholding or withdtawing life
In contrast to community- and home-based hospice sustaining treatment, and hastening death by assisting in
care, hospital-based palliative care programs are a much suicide or through active euthanasia.

1972 ENCYCLOPEDIA OF BIOETHICS 3rd Edition

~
"'!"'l

~""'''''"''"""
f1 -'~:
7

PALLIATIVE CARE AND HOSPICE


l ""

The latter issue tends to receive the greatest attention hospice and palliative care remain near the margins of the
from bioethics scholars and policymakers. Moral distinc American healthcare system. In the realm of education, a
tions between various actions or choices that can hasten the 1997 survey of fourth-year medical students and third-year
time of death can be exquisitely fine (Quill, La, and Brock). medical residents found that both groups rated their prepa
Yet, for all the persistent and intense debate surrounding the ration in end-of-life care worse than for many other com
issues of suicide and euthanasia (Battin, Rhodes, and Sil mon clinical tasks (Block and Sullivan), and analyses of
vers), "terminal sedation" and the doctrine of double effect leading medical t~xtbooks reveal that, on average, end-of-life
(Fohr), or the differences, if any, between "allowing to die" issues are addressed on only 1.6 percent of the pages (Block).
and "causing to die" (Brock; Clouser), another set of issues In the realm of financing of services or research, the desire to
are no less vexing and affect far more people. These are the forestall or prevent death ovetwhelms support for hospice
questions of access to and quality of palliative care services. and palliative care. Precise data are difficult to obtain, but
The dimensions of the problem of access to palliative one indicator of the relative lack of support for palliative as
care are suggested by the following data from the United opposed to curative medicine is presented in a 1997 report
States. According to the National Hospice and Palliative from the Institute of Medicine of the National Academy of
Care Organization (NHPCO), of the 2.4 million people Sciences. The report cites a personal communication from
who died in the United States in 2000, approximately one an official from the National Institutes of Health (NIH),
fourth died while receiving hospice care. Approximately half who estimated that in fiscal year 1996, NIH spent about $70
died in hospitals, 25 percent died in a nursing facility, and million on pain research out of an overall budget of $12
another 25 percent died at home; the percentage of home billion.
deaths has remained relatively stable for several decades, From the policy perspective, the greatest challenge
despite Gallup polls that consistently indicate that over 85 facing palliative care in the United States at the beginning of
percent ofAmericans would prefer to die at home. the twenty-first century is to fashion a system of financing
It is true that dying at home is an imperfect marker for and delivery of care that is flexible enough to provide services
the adequacy of palliative care. In fact, in most developed as they are needed along the complete continuum from
countries, the better the palliative care provision in hospitals diagnosis of life-threatening illness through the (often un
and the community, the fewer the number ofpeople who die predictable) period of disability and functional decline, into
at home, with home deaths now approaching 20 percent in the last phases of active dying and family bereavement
most European countries. A more telling statistic is that of (Lynn). The system would, at a minimum, encourage the
patients who received hospice care in 2000, one-third died open acknowledgment by physicians and patients of the
within seven days of admission, despite the six months of possibility of dying, advance planning to anticipate compli
benefits allowed under the Medicare hospice program. The cations and likely needs for care, meticulous attention to
median length of stay for hospice patients in the United physical symptoms and to psychological and spiritual suffer
States has been dropping steadily for several years; the ing, support for the family, and the creation of settings for
","
NHPCO reports that it was only twenty-five days in 2000. care that respect the personal and spiritual significance of
Although the reasons for these trends are still being investi death and loss.
gated, the following are likely to be significant contributing Worldwide, the challenge ofaccess to competent pallia
factots: the difficulty of making precise estimates of life tive care is no less daunting. Among the principal causes for
expectancy-as is required for Medicare hospice eligibility alarm are the number of people living with HIV/AIDS
especially for diseases other than cancer (Teno et al.); estimated by the United Nations at 40 million at the end of
patients' reluctance to accept the label "terminally ill"; the 2001-and the large projected increase in deaths from
requirement that patients forgo Medicare reimbursement tobacco products, which the World Health Organization
for treatments with curative intent; and many physicians' predicts could triple by 2020 from the 2000 level of 3.5
identification of a hospice referral with "giving up." million (Brundtland). In both cases, almost all of the
In the United States, hospice and palliative care have inctease is expected to occur in the developing world. Global
not yet fully overcome the legacy of opposition to main effortS to teach the principles of modern palliative care, and
stream scientific medicine that characterized their begin to incorporate them in healthcare systems, are lagging far
nings in the 1970s. The growth of rigorous scientific re behind the manifest need, despite curative technologies and
search in palliative care, the publication of textbooks, and medications remaining unavailable or unaffordable for most
the growth ofa cadre of palliative medicine specialists with a of the world's poor.
base in academic medical centers should ameliorate this Where palliative care is available, there is the challenge
problem in the years to come. For the present, however, of providing cate in ways that respect different cultural and

ENCYCLOPEDIA OF BIOETHICS Hd Edition 1973


PALLIATIVE CARE AND HOSPICE

r
religious views. Most professionals who enter the field do so Brunddand, Gro. 2001. 'The Future of the World's Health." In
because they want to help people die well. But what does it Critical Issues in Global Health, ed. C. Everen Koop, Clarence
Peterson, and M. Roy Schwartz. San Francisco: Jossey-Bass.
mean to "die well"? What is a "good death"? There is no
single, universal answer to either of these questions. That the Center for Medicare Education. 2001. The Medicare Hospice
modern hospice movement was first promulgated largely by Benefit. Washington, D.C.: Author.
Christians may have hindered its development among peo Clinch, Jennifer; Dudgeon, Deborah; and Schipper, Harvey.
ple of other faiths for whom the "hospice philosophy" may 1998. "Quality of Life Assessment in Palliative Care." In
Oxford Textbook of Palliative Medicine, ed. Derek Doyle,
have been hard to separate from theological commitments
Geoffrey Hanks, and Neil MacDonald. New York: Oxford
that they did nor share. Even with respect to elements of a University Press.
"good death" on which most people could probably agree
Clouser, K. Danner. 1977. "Allowing or Causing: Another
freedom from pain, resolution of personal affairs, the sup
Look." Annals ofInternal Medicine 87: 622-624.
portive presence of loved ones-there is room for consider
Doyle, Derek. 1997. Dilemmas and Directions: The Future of
able personal variation. People differ in their willingness to
Specialist Palliative Care: A Discussion Paper. London: National
face the reali ty of their imminent death; in their desire to talk Council for Hospice and Specialist Palliative Care Services.
about their feelings to friends, family, or caregivers; in how
Doyle, Derek; Hanks, Geoffrey; and MacDonald, Neil, eds.
they balance pain relief against alertness; and in their
1998. OxfOrd Textbook of Palliative Medicine. New York:
willingness to tolerate increasing weakness, dependency, and Oxford University Press.
uncertainty rather than trying to control the timing and
Emanuel, Ezekiel; Fairclough, Diane; Daniels, Elisabeth; et al.
manner of their death through an act of suicide or euthana 1996. "Euthanasia and Physician-Assisted Suicide: Attitudes
sia. This variability requires health professionals to approach and Experiences of Oncology Patients, Oncologisrs, and the
patients and families as individuals, in an effort to provide Public." Lancet 347: 1805-1810.
care that is consistent both with patient and family values Fohr, S. Anderson. 1998. "The Double Effect of Pain Medica
and with their own conscience. tion: Separating Myth from Reality." Journal of Palliative
Medicine 1(4): 315-328.
DEREK DOYLE
Higginson, Irene. 1992. Quality, Standards, Organizational, and
DAVID BARNARD ClinicalAuditfOr Hospice and Palliative Care Services. London;
National Council for Hospice and Specialist Palliative Care
Services.
SEE ALSO: AIDS: Healthcare and Research Issues; Cancer,
Ethical Issues Related to Diagnosis and Treatment; Care; Hinton, John. 1994. "Can Home Care Maintain an Acceptable
Compassionate Love; Death; Dementia; Healthcare Resources, Quality of Life for Patients with Terminal Cancer and Their
Allocation oj: Informed Consent; Life, Quality oj: Life Relatives?" Palliative Medicine 8(3): 183-196.
Sustaining Treatment and Euthanasia; Long-Term Care; Joint United Nations Programme on HIV/AIDS. 2002. Report
Nursing, Profession oj: Pastoral Care and Healthcare on the Global HIV/AIDS Epidemic 2002.
Chaplaincy; Social Work in Healthcare; Teams, Healthcare
Lynn, Joanne. 2001. "Serving Patients Who May Die Soon and
Their Families; The Role of Hospice and Other Services."
Journal ofthe American Medical Association 285(7): 925-932.
BIBLIOGRAPHY
Mount, Balfour. 1997. 'The Royal Victoria Hospital Palliative
Addington-Hall, Julia. 1998. Reaching Out: Specialist Palliative Care Service: A Canadian Experience." In Hospice Care on the
Care fOr Adults with Non-Malignant Diseases. London: National International Scene, ed. Cicely Saunders and Robert Kastenbaum.
Council for Hospice and Specialist Palliative Care Services. New York: Springer.
Battin, Margaret; Rhodes, Rosamond; and Silvers, Anita; eds. National Hospice and Palliative Care Organization (NHPCO).
1998. Physician-Assisted Suicide: Expanding the Debate, New 2001. Facts and Figures. Alexandria, VA: Author.
York: Routledge.
Pan, Cynthia; Morrison, R. Sean; Meier, D.; et al. 2001. "How
Billings, J. Andrew. 1998. "What Is Palliative Care?" Journal of Prevalent Are Hospital-Based Palliative Care Programs? Status
Palliative Medicine 1(1): 73-81. Report and Future Directions." Journal ofPalliative Medicine
Block, Susan. 2002. "Medical Education in End-of-Life Care: 4(3): 315-324.
The Status of Reform." Journal of Palliative Medicine 5(2): Quill, Timothy E.; Lo, Bernard; and Brock, Dan W. 1997.
243-248. "Palliative Options of Last Resort; A Comparison ofVoluntar
Block, Susan, and Sullivan, Amy. 1998. "Attitudes about End-of ily Stopping Eating and Drinking, Terminal Sedation, Physician
Life Care: A National Cross-Sectional Study." Journal of Assisted Suicide, and Voluntary Active Euthanasia." Journal of
Palliative Medicine 1(4): 347-355. the American Medical Association 278(23): 2099-2104.
Brock, Dan. 1992. "Voluntary Active Eurhanasia." Hastings Randall, Fiona, and Downie, R. S. 1996. Palliative Care Ethics: A
Center Report 22(2): 10-22. Good Companion. Oxford: Oxford University Press.

1974 ENCYCLOPEDIA OF BIOETHICS 3rd Edition

.........

PASTORAL CARE AND HEAL THCARE CHAPLAINCY


Saunders, CiceJy. 1998. "Foreword." In Oxford Textbook of Historically, pastors have extended their care to a wide
Palliative Medicine, ed. Derek Doyle, Geoffrey Hanks, and range of personal needs and concerns, from struggles of
Neil MacDonald. New York: Oxford Universiry Press. faith, doubr, mora! failure, and problems of conscience to
Sullivan, Amy; Hedberg, Katrina; and Fleming, David. 2000.
r
"Legalized Physician-Assisted Suicide in Oregon-The Sec
marriage and family conflict and the suffering involved in !:;
illness, tragedy, and death. In Christian care, the historic,
ond Year." New EnglandJoumal ofMedicine 342(8): 598-604.

I
ritualized "means of grace"-sacrament, scripture, prayer
Teno, Joan; Weitzen, Sherry; Fennell, Mary; et al. 2001. "Dying continue to be important resources of pastoral care, espe
Trajectory in the Last Year of Life: Does Cancer Trajectory Fit
cial1y in situations of crisis (e.g., dying). But in many
Other Diseases?" Joumal ofPalliative Medicine 4(4): 457-464.
situations conversational methods predominate. Pastoral
U.S. Institute of Medicine. Division of Health Care Services. r
conversation emphasizes the caregiver's psychological un
Committee on Care at the End of Life. 1997. Approaching
Death: Improving Care at the End ofLife, ed. Marilyn Field and derstanding and abiliry to foster a therapeutic or healing !
Christine Cassel. Washington, D. c.: National Academy Press. mode of relationship and sryle of conversation with the f
person receiving care. This includes empathic listening, the l
Ward, Audrey. 1985. "Home Care Services for the Terminally
Ill." Sheffield, UK: Universiry of Sheffield, Medical Care abiliry to form emotionally honest, trusting relationships, I
Research Unit. and the care receiver's active participarion with the pastor in f
I,
World Health Organization. 1990. Cancer Pain ReliefandPallia the search for healing and wholeness. Ar the root of their
tive Care. Geneva, Switzerland: Author. care, pastoral caregivers help persons find the kind of faith
and value commitments that can sustain, enrich, and give
redemptive meaning to their lives, and "to experience as fully
INTERNET RESOURCE
as possible the realiry of God's presence and love in their
Joint United Nations Programme on HIV/AIDS. 2002. Report
lives" (Holst, p. 46).
on the Global HIVIAJDS Epidemic 2002. Available from <http://
www.unaids.org/publications/>. Pastoral care and healthcare chaplaincy are often distin
guished from another ministerial specialization-pastoral rt.
counseling. When this distinction is made, pastoral counsel J
ing is commonly defined as a specialized form of ministry
f
characterized by an intentional contract between the pasto
ral caregiver and the person or family seeking help, usually
PASTORAL CARE AND
involving a series of prearranged counseling sessions. This
structured form of care contrasts with the more casual and
HEALTHCARE CHAPLAINCY
varied forms of caring relationships that parish pastors and
healthcare chaplains rypically form. Though many minis .~ ,

ters, priests, rabbis, and healthcare chaplains provide short t;;t


term counseling of the more formal kind, pastoral counsel [;
Pastoral care normally refers to the help given by ordained
ing as a specialized ministry is devoted entirely to this work. ~
ministers, priests, and other persons with designated relig
To a large extent it is a form of psychotherapy or family
ious roles (such as deacons and members of Roman Catholic
therapy (and is often called "pastoral psychotherapy"), and
religious orders) to suffering, troubled, or perplexed persons.
usually involves a number of sessions and the payment of a
In the simplest and most profound sense, pastoral care has
fee. Pastoral counselors, like healthcare chaplains, have
been defined from a Christian perspective as "the attempt to
specialized training requirements, professional organizations
help others, through words, acts, and relationships, to
(principally, the American Association of Pastoral Counsel
experience as fully as possible the realiry of God's presence
ors), and standards of certificarion. They serve on the staffs
and love in their lives" (Holst, p. 46). The term is primarily
of larger churches, in pastoral counseling centers, and in
Christian but it is sometimes used analogously in other faith
other professional settings, and are often licensed by state
traditions (e.g., the rabbi's care in Judaism). Recently the
governments as pastoral (or other) counselors, psychologists,
term spiritual care has been introduced into secular healthcare
or marriage and family rherapists.
settings as a less specifically Christian alternative term. In
any case, when pastoral or spiritual care is provided in
healthcare facilities by pastors or rabbis sponsored by the Pastoral Care in Healthcare Settings: The
institution, it is known as healthcare chaplaincy. This article
Healthcare Chaplain
largely focuses on healthcare chaplaincy because it is rhe
primary way in which contemporary pastoral care becomes FUNCTIONS AND ROLE. Much ofwhat health care chaplains
involved with the issues of bioethics. do involves helping persons and families (of all faiths) with

ENCYCLOPEDIA OF BIOETHICS 3rd Edition 1975


JOURNALo r I'ALLIATIVEMEDICINE
Vo lume 1, Numbe r 1, 1998 Essay- Change
Mary Ann Lieb ert, Inc.

What is Palliative Care?

J. AN DREW BILLINGS, M.D.

. .. the meaning of a wo rd is its use in the care" IOJI- this latt er term some times referr ii
lan gua ge.' to comfort care, some times to su pport of tl
comp romised host or critically ill pa tient, par-
ticularly those suffering from the ad verse ef-
ilENACARDIOLOGIST introduces himself to fects of cancer treat ment. The di versity of
W a patient or fa mily member or is asked,
"What d o you do?" a clear, concise, simple, and
mean ings of these terms and their unfamilia r-
ity to man y person s can bew ilder patients, fam -
read ily und erstood answer mig ht be, "1 am a ily members, and colleag ues in the health pro-
hea rt specia list." An intern ist can say, "a physi- fessions. A lack of ag reeme nt on the meaning
cian for ad ults," and then perh ap s specify of the terms also reflects conflict among pa llia-
so mething about nonsurgical man agem ent of tive care practitioners abou t the natu re of the
nonpregn ant ad ults. But wh at d oes a palliati ve field. How we label ou rselves and how we and
care specialist say? As a hospice med ical di - others d efine our work may have profou nd ef-
rector for man y years, I am famili ar with the fects on the future of the hosp ice movement
awkwa rd ness of introd ucing myself to a new and on pa lliative ca re as a d iscipline. Altho ugh
pat ient or family member. Word s like " ter mi- 1cannot resolve all of the bafflement a nd con-
nal care" or "de a th" stick in the mouth, a nd one tenti on that underlie these issu es, and certainly
sea rches for eu phemisms. Now, as a newly la- do not pretend to any au thority in arriving at
beled palliat ive med icine specialist, I continue a final d efinition, 1hope this essay will provide
to struggle with how to explain myself. some clarifica tion and stimulate furt her di s-
Balfou r Moun t coined the term, "pa lliative cuss ion abou t the mean ing of palliati ve care.
care" around 1973 to d escribe his new program
at the Royal Victoria Hospi tal in Montreal,
mod eled on St. Christop her 's Hospi ce.' "Pal- DEFINITIONS OF PALLIATI VE CAR E
liati ve care" and the related term, "palliative
medi cine," have become the labels of choice To palliate, of course, literally mea ns "to
throu ghou t the world for program s based on cloak." Palliation can be viewed d isapprov-
the hosp ice phil osophy, and are now being ingly as merely covering up problems. How-
used increasi ng ly in the United States.3-7 The eve r, as cu rrently used in America n medi cine,
Institu te of Med icine recom mend s that "Pallia- "palliative care" is becoming a wide ly accepted
tive care should become, if not a med ical term for an approac h to the ma nageme nt of a
specia lty, at least a defined area of expertise, termin al illness that focuses on sy mptom con-
ed uca tion, and resea rch.'?' "Palliative care" trol and sup port ra ther than cure or life pro-
overlap s with "terminal ca re," "d eath-and - longa tion.
d ying," "hospice," "end -of-life care," "tha na- The term "palliative care" currently enjoys
tology," "comfort care,"? and "su pportive two wid ely cited d efinitions.P Both of them are

Palliative Care Service, Massachusetts General Hospital, and Department of Medicine, Harvard Med ical Schoo l,
Boston , Massachusetts.

73
74 BILLINGS

easily criticized, which I interpret as evidence avoids pre tentious or confus ing im plication of
not so much of their coine r's shor tcomings but the othe r terms.l? O ther cha rac teristics of pal-
of how the field ha s evo lved, matu red , and ex- liative care that might be related to or sub-
panded . One com mo nly quoted int er pretation sumed by the term "com prehensive" are "in-
com es from the World Health Organization terdisciplinary," "coord ina ted," "integrated,"
(WH O) in its 1990 publication, Cancer Pain Re- "hu ma nistic," and "accessible" care.
lief and Palliative Care: "the active total care of This WHO d efinition is ty pically followed by
pati ents whose di sease is no t resp on sive to cu- a lon ger, so mewha t awkward attempt at clari-
rative treatment.vl-' ficatio n:
Try that on on e of your pa tients ! Or maybe
suggest to your medical colleagues that they Co ntrol of pain, of othe r sym ptoms, and
only deal with ina ctive or partial care or with of psychological, social, and spiritua l
cu ra tive treatment! The term "a ctive" is pre- proble ms is par amount. The goa l of pal-
su ma bly included here to d ispe l notion s that liati ve care is ac hieve ment of the best p os-
palliative care is passive or focused simply on sible qua lity of life for pati ents and their
av oiding interventions, but see ms to add litt le fami lies. Man y as pec ts of palliative care
to the meaning of the definit ion-what is inac- are also a pp licable ea rlier in the cou rse of
tive care? We certainly do not wa nt to claim ex- the illn ess, in conjunction wi th antican cer
pertise on the vast number of di seases that do treatment.' ?
not respond to cura tive treatment. Ideally, our
de finition shou ld focus on the posit ive as pec ts These additional assertions are help ful.P
of the work, suc h as helping patients and fam- thou gh they d o not clearly dis ting uis h our
ilies live we ll or promote their qu ali ty of life. work from that of othe r clinical field s. Man y
Here, the em phasis on failure-"not resp on sive other clinicians recogn ize the importan ce of
to cu ra tive treatmen t"-seems unnecessaril y comfort and su ppor t in terminal di sease, as
gloomy , but perha ps is fairly gentl e and ac- well as in nonterminal di sease. Arg ua bly, all of
ceptabl y eu phe mistic abou t terminal care and med icine can be viewed as seeking the goa l of
death. The com mo n but probl em at ic d istinc- quality of life. Moreov er , the ass ociation of pa l-
tion s bet ween palliation and cura tive or life- liative care solely with cance r is misleading.
prolon gin g (or life-extend ing) treatment (or A second definition of palliat ive care, used
treatment with "aggress ive intent") are not in- by the au tho rs of The Oxford Textbook of Pallia-
voked . tive Medicine, first publish ed in 1993, also has
On e streng th of thi s definition is the asser- been w idely cited: "The study a nd man age-
tion that ca re sho uld add ress all forms of suf- ment of pati ents with ac tive, progressive, far-
fering : "total care." Related terms ar e "total ad vanced di sease for w hom the p rognosis is
pain or suffe ring" or "holistic care." Unfortu- lim ited and the focus of care is the quality of
nately, suc h claims ca n sound a bit overinflated life." IB This definition is more concise and more
or unrealisti cally ambitious . "Ho lism" un for- precise than the WH O phrases. By choosing a
tunately is a bankrupt term, a red light that of- word like "focus" (O T "em phasis" or "conce n-
ten signa ls nonsense. It has lost its cache in tra te"), we avoid making pall iat ive care a
thoughtful social science circles,"! and "holis- conflicting or totally sepa rate ap proach from
tic care" is now regul arl y u sed sy no ny mo us ly "conventional," "curative," "aggressive" or
with "altern ative" or "com plementary" medi- "life-p rolonging" measures. A Ca na d ian Pal-
cine. A d ero gat ory term, "symptomatologists" liative Ca re Associati on definition also stresses
has been introduced by Michael Kearncy" and that palliati ve care "ma y be combined with
might be used to d escrib e careg ivers who fo- therapies aimed at redu cing or curing th e ill-
cus on vari ous di seases and problems but d o ness, or it ma y be the total focus of care." !" We
not add ress the tota l su ffering of the perso n.!" sho u ld not be consigned to the fina l days of life
"Comp rehensive care" is m y term of choice, es- when ot her a pproac hes are abandoned .
pecially becau se it already has establis hed Ho wever, this defi nition is replete with jar-
meaning in health se rv ices litera ture, and it go n and confus ing terminology. Wh at is a "lim-
WHAT IS PALLIATIVE CARE? 75

ited " (or "unli mited") progn osis? Is th is d esig- or really regard dying as a n abno rma l process?
nati on p referabl e to "incurable" or "termi nal?" Theology seems to be atte m pt ing to impose it-
Wh o will attend to the subtle di stinction ab out self on clinical work. Ce rtain ly, pa lliati ve care
the disease bein g bo th active a nd progressive? may not always view death as a failure or as
Whe re is the family in th is expli cation ? an inevitable ene my, a nd may see positive op-
None of these definition s are brief or clea r po rtu nities for growth and reconciliati on in the
eno ug h to answer a pati ent or family member' s face of dyin g. A more d istinguish ing charac-
questions : "What does palliati ve care mean?" teristic of palliative care wo u ld be tha t it openly
Or, "W ha t does the palliative care service do?" ackno w ledges dying .
Ind eed, the d efin ition s ma y be too abstr use a nd Similarly, we regularl y read that hosp ice or
too vague for eve n clinicians or health-car e pol- palliat ive care "neithe r hasten s nor postpon es
icy experts who are familiar w ith the jar gon. death," which ap pears to be a statement of ide-
These definition s are often acco m pa nied by ology, reflectin g perhaps an aversion to eu-
a host of precep ts that I find largely meaning- than asia and, tellin gly, to life-prolon gin g treat-
less, silly, grandiose, and even inaccurat e, ment. This maxim may reflect so me of the
many of w hich would seem quite out of place religious orientation of the hospice movement,
in a discussion of a no the r clinical specialty, re- but does not seem appropria te for defining a
ga rd less of its scope or sense of mission . We field of health care. Regardless, the statement
sho uld be mindful of Dere k Doyle's ad moni- certa inly d oes not reflect my own sense tha t I
tion "never to believe that we have a mon op- am often postpon in g or hastening deat h. For
oly on care, concern or compassio n," and of instan ce, pat ients w ho are not ea ting, losin g
hospice's ten d ency to "self-righteous ness .v-" weight, and becoming progress ively weakened
Some terms and phrases- for instan ce, "pa- but then receive carefu l mouth ca re, nutrition al
tien t-centered care," "care versu s cure," "treat- su pport, pain con tro l, and othe r comfort mea-
ing the perso n, not the d isease," "com passion," sures see m to live lon ger (and wish to kee p li v-
"sk ill," "d ignity," "recognition of pati ent val- ing lon ger ) with good palliative interve ntion.
ues," or "cu ltura lly sens itive services"-may Vigo rous application of opioid analgesics or
suggest stan dards d isting u ishing good from seda tives to treat a pa tient's severe pain or dys-
bad palliat ive ca re, but do not cons titute es- pn ea may cause drowsi ness and redu ced in-
sential parts of a d efinition. They seem to im - take of food an d fluid s, as well as pr edispos-
ply that pa lliative ca re has a spec ial claim on ing to aspiratio n, and th us po tentia lly hasten
such vir tues. The u se of suc h terms can muddy death.
the wa ters and, for me, engende r misgivings.
For ins tance , although palliative care shou ld
certainly be tailored to the need s and wishes of HOSPICE AND PALLIATIVE CARE:
the pat ient and family, this feature does not d is- A PARTING OF WAYS OR
tin guish us from ot her fields of medi cine any A NEW COALITION?
mo re than do skill or compassion. Q ua lity of
life is a conce rn in all areas of medicine, and Co mpe tition is grea test between those
any in tellige nt approach to assessing it begins w ho occupy the same position in the econ-
with an understanding of pati ents' kn owl ed ge omy of nature.21
abou t their cond itio n and po ten tial man age-
men t strategies, their values, and their person al Palliat ive ca re emb races the hospice philoso -
cost- be nefi t calcula tions. Here, also, the term ph y of care and seeks to bring th is approac h to
"d ignity" is often used, altho ugh it seems a wid er grou p of pati ents than cu rre ntly served
vag ue and potentially lad en with care by hospice progra ms in this country. It also
providers' values . seeks to integrate the hos pice approach into
We also often read that hosp ice or palliati ve clinica l pract ice. I do no t believe that most pal-
ca re "affirms life and regards d yin g as a nor- liati ve ca re practiti on ers have forsaken hospice,
mal process." Bu t what does it mean not to af- but rather a re tryin g to ap ply the mod el more
firm life? Do other clinicia ns d isa pprove of life broad ly and also more sensibly than currently
76 BILLINGS

fostered by hospi ce regulat ions and reim - vice, chaplaincy, volu nteers, or bereavem ent
bursem en t in the Un ited States . I view all hos- care). Moreover, as recently d ocumented for
p ice care as a seg me nt of palliati ve care. hom e care pa tients with amyotrophic lat eral
In the United States, hospice has come to sclerosis, althoug h hosp ice staff may be per-
mean primaril y a go vernmentally regul ated or- ceived as more knowl ed geable and em pa the tic
ga niza tion or p rogr am for dyin g persons and than conve ntiona l home care wo rke rs, hosp ice
their families,22 typi cally focus ing on hom e may provide far fewe r hours of form al care.23
care, and limit ed to patients with: Thus, pa tients and families are often forced to
choose between hospice care with insuffi cient
1. An expec ted progn osis of 6 months or less hom e health aide sup po rt and a conventiona l
2. A focus on comfort measures-this is some- hom e care ap proach that includes significantly
tim es (bu t not always) defin ed by hospice more hom e healt h aide hours.
progr am s as a desir e to foreg o a variety of Eligibility requirements that may mak e sense
"aggressive" and often expens ive man age- from a fiscal vantage in d esigning a hospice
ment ap proac hes (us ua lly includ ing car- ben efit or in ru nning a progr am under the cur-
diopulmonary resuscitati on, blood product ren t reimbu rseme nt scheme make littl e sense
replaceme nt, a nd some forms of radiother- to a clinician concerned with overall care of the
apy, surgery, chemothera py, and acute care d yin g and their families. For instan ce, many
hospitalizati on ), at least insofar as these pat ients wh o are receiving purely com fort care
treatment modalities are bein g used in an at- an d seem ap propriate for hosp icelike services
tempt to cure or prolon g life rather than to can be expected to live for years. Many ag-
palliate sy m ptoms gress ive or high -technology or sim ply expen-
3. A gene ral pr eference for care at hom e (ex- sive in ter ventio ns are ap pro pr iate for pati ents
cep t where in patient hospi ce is available in the very lat e p hases of a terminal illness and
and spe cifically soug ht) sho uld not be foregon e just to qua lify for com-
4. A willingn ess to sign a for m ack nowledg ing prehensive hospice home care services. The us e
that they wan t to enter a hospi ce program of a ntire troviral regimen s or of treatments to
and focus on comfort care pr event blin dness from cyto mega lovi rus in far-
5. Health insuran ce that cove rs hosp ice ad vanced AIDS would be common exa m ples.
Similarly, pa tients wh o ma y be ineligible for
Many hosp ice programs also requ ire that the some hosp ice progr am s becau se they do not
pa tient have a primary care-g ive r in the hom e have a prima ry care-g iver still may wa nt to re-
or readily ava ilable. Ano ther set of eligibility ceive care at home and can benefit grea tly fro m
requirem ents, w hich I hear occasio nally from the su p po rt offered by hospice. Patients wh o
pati ents or family members but which are not need the gr eater home health aide hours of-
embod ied in Fed eral hosp ice regulati ons or the fered by conve ntional hom e care p rogr ams a nd
Patient Self-Determination Act, is that the pa- th us choose to forego hospice enrollme nt still
tien t and family ag ree not to call emergency may wis h a pa lliative care app roac h. Patients
services or to be rehospitalized . w ho are ave rsive to the wo rd "hos pice" or w ho
Hospi ce program s in the United States have are relu ctant to sign forms that redefi ne their
been increasin gly boxed in by these req uire- insuran ce benefits or who have di fficulty ac-
ments. They have becom e extreme ly cau tious knowledg ing that they are imminently facing
with ad miss ion or recerti fication in the face of death may benefit from and sho uld be ab le to
the threat posed by an unsym pathetic and per- receive palliative serv ices.
haps ill-con ceived govern me nt au dit that scru- Hospi ces rightly object to terms such as "hos-
tinizes long-stay patients and those with non- picelike" care because so man y conventional
can cer diagnoses. At the sa me time, health hom e care progr am s have claime d to provid e
maintenan ce organizations and insurers have services that are equa l to that of hospice but, in
atte mpted to "unbu ndle" hosp ice services, pro- fact, d o not offer man y of the standa rd benefits
viding and payin g for only pa rt of the hospice of hospice, including inte rd isciplinary care,
package (e.g., hom e nursing without socia l ser- specially trained and su ppo rted clinicians, vol-
WHAT IS PALLIATIVE CARE? 77

un teer an d bereavement services, and free lack a p rimary caregiver or are receiving pal-
medications and d urab le medical equip ment. liativ e radiation, and then gene ralize this rule
At the same time, some hospice programs a re to all hospices. From both within an d ou tside
establis hing "bridge" or prehospi ce program s of hospice, I have freq ue ntly witn essed that
tha t may facilitate ea rly ad missio n a nd avoid many hospice nurses communicate poo rly with
some of the d ifficulties posed by hospice ad - ph ysician s and hospital-based colleag ues a nd
miss ion or recertification requirements. Bridge do not appreciate pot entially ap pro priate "ag-
programs rep rese nt an effort with in hosp ice gressive" man agem ent ap proaches . Hospice
and home care orga niza tions to exten d hospice staff often lack a sense of collegiality with non-
services to pa tients and family that curren tly hospi ce health-care providers, apparently be-
a re not receiving such services, as well as to fa- lieving that only hospice trul y "ca res." "Letting
cilita te ear lier and more approp riate transfe rs go" of various treat ments or hopes for cu re or
to hospice. The bridge pro grams may be pre- prolongation of life becomes a wei rd virtue, a
sen ted as hospicelike, but they ha ve not been ho ly state, the only right way to die. Physicians,
sys tema tically studie d in such a way as to as- particularl y, become a butt of criticism an d are
sess their impact or allow a mean ingful com- treat ed in a manner that see ms a t least sub tly
pa riso n with hospice care. Similar qu est ions d emeaning, so metimes overtl y hostile. In my
arise with palliati ve care services, whi ch lack community, a freque nt emphasis on exclusion-
meaningful standards of care or ap propria te ary criteria ("We can' t provi de around -the-
accred itation of p roviders. Bridge pro gra ms clock care.. .. Have they given up chernother-
a nd palliativ e care program s both exemplify, a py yet?. . Where will he go if he gets
in part, an a ttempt to extend the hosp ice p hi- sicker?" ) has conveyed a with holding, negative
losophy of care to more pa tients and famili es att itude amo ng hospi ce staff abou t sharing in
whil e sides tep ping the regul atory con straints the care of dyi ng patients and families.
of certified hospice programs, as we ll as the Finally, mu ch to the d ismay of palliativ e care
cu rrent constriction of hospice services. providers, hospice in the United States has be-
Hospice clinicians hav e assert ed that pallia - come a program for imminently dying persons,
tive car e is an atte mp t to avoid the negativ e caring for many pat ients only in the last few
connotation of the wo rd , "hospice,"24 but I days or weeks of life.2s Only a sma ll propor-
think they misp erceive, underestimate, or sim- tion (roug hly 20%) of d ying person s are cared
ply ove rloo k problems with hospi ce programs, for by hospice program s in this cou ntry. Pal-
which provide some of the motive behin d us- liativ e care seeks involvemen t with pati ents
ing a n alternative term. I personally d o not be- and families as soon as the diagnosis of a life-
lieve that "hospice" is a bad word in my com- threaten ing illness is confirmed, occas ionally
mun ity. I continue to foster my ident ifica tion even earlier. Palliative care is not jus t for the
with hospice, and I feel free to use the term in immine ntly d ying, nor should be hosp ice."
describing my work, past a nd present. At the In the United States, the greatest threa t to
same time, having switched recent ly from be- hos pice from palliative care wo uld see m to be
ing a hospice med ical director to a palliati ve the poss ibility that the hospi ce philosop hy will
care service d irector w ho regularl y refers to be d istorted and su pplanted by the newer p ro-
hospice, I have bee n struc k with how hospice grams. Insofar as pa lliative care appears to be
programs can confuse referral sou rces and tar- more academic, scientific, doctor driven, inpa-
nish their reputation through a lack of rela- tient based , inclusive of specia lists, and con-
tively uni form standards of care and by see m- sulta tive, rather than community based, patient
ing to show grea ter concern, at times, for their centered, nurse dri ven , genera list a nd primary
program goa ls a nd fina ncial constraints than care d omi nated, psychosocially a nd spiritually
for the need s of pati ents, families, and referring oriented, and committed to comprehensive, in-
clinicians. Clinicians, who ma y not d istin gui sh terdisciplinary se rvices, these fears see m well
one hosp ice pro gram from another, may be told founded. A pain or symptom control team that
that one program do es not p rovid e continuous does not provide com prehensive, inte rdis cipli-
care or respite or does not accept patient s who nary ca re to patien ts and families may provide
78 BILLINGS

a needed service but it sho u ld not be confused cou ntry? The answer to this question is pa rt of
wit h palliative care. Until clea r s tandards are defin ing ourselves a nd may have important
established for palliati ve care progr ams, in- implicatio ns for reimbu rsem ent in the age of
cluding valid, p rofessionally recogn ized ere- managed care. Although I will not offer a con-
d entialin g of clini cian s, and accou ntability for clusion on the d ifficult question of whe the r pal-
standards of qu ality of care, apprehens ion is in- liative medi cine sho uld become a specialty or
evitable abo u t diluting or d istorting the hos- an area of exceptiona l competence w ithin ex-
pice philosophy and reversin g the ga ins from isting field s, pa rt of the justificati on for a new
the establishme nt of certified hospice programs field is to ad d ress unmet pa tient and family
in this cou ntry ov er the past 15 years. need s, offer expe rtise with diffi cult cases and
Man y opportunities rem ain for coope ration unfamiliar treatment method s, train medical
amo ng palliative ca re program s a nd hospi ce. stude nts and grad ua te ph ysicians, a nd carry
Briefly, hospice is the home care program of ou t resea rch, all of which pertain to palliative
cho ice for eligible pati ents and families. Pallia- medicine.
tive care programs provid e a co nd uit for wider An early and ongoi ng distin ction amo ng
ed ucation abou t and ea rlier referrals to hos- kinds of doctors is based on the type of tech-
pice. Insofar as many pa lliative care se rvices nology they princip ally u se in their trea t-
are based in hospitals, they a re ge nera lly bet- me nt- between physicians who primaril y use
ter able than hospice progra ms to pa rticipa te in medica tion and those that perform surgery.
the key treatment decisions, includi ng the tran - Palliative care teams do have a treatment
sition to comfort care, wh ich often occur in the method that differs from most (but definitely
inpatient setting. Palliative care programs tend not all) ot her field s of care: comp rehensive care
to be based in acad emic institutions and can and case managem ent by a specially trained
provide broad er training of physician s and and su ppo rted interdisciplinar y team. Notably,
othe r health-care profession als and stude nts non e of the d efiniti ons of palliative care cited
abou t goo d end -of-life care, w hich includes ea rlier in this essay include interdisciplinary
hospice care. Few academic palli ati ve care pro- care as a basic featu re.
grams will wa nt to start their own home hos- If we are specia lists, we canno t d elineate ou r
pice programs, and most will want to work wo rk as follows:
closely with hospices in a va riety of commu ni-
ties to assu re contin ui ty of excelle nt care when 1. Orga n or organ-sys tem based (nephro logists
pati ents go home. Hospices a re needed as train- principally take care of the kidney, neuro l-
ing sites for students in the health p rofes- ogists the ner vous system),
sio ns.i? Add itiona lly, altho ug h a great streng th 2. Disease based (onco logists principally tak e
of hosp ice in the United States has been its em- care of cance r)
phasis on qu ality home care and the manage- 3. Age based (ped iatricians p rov ide ge nera l
ment of ch ronic, progressiv e, fatal di sease, pal- medical care to child ren).
liative medicine can contribu te to care in a
vari ety of other settings- the acut e care hospi - Palliative care is end -of-life care, directed to dy-
tal, including the intensive care un it and the ing pers ons and their families. It canno t be a
eme rge ncy ward, as well as offices and ex- subse t of onco logy or a ny other specialty be-
tended care facilities-and has a role in deaths cause we d eal w ith a broad range of terminal
fro m acu te co ndi tio ns. illnesses. Simi larly, although pall iative medi-
cine may be developing prim ar ily as hospital
consulta tion services.P' and might be viewed as
A NEW SPECIALTY OR an inpa tient specialty, analogous to "inten-
GENERALISTS WITH AN AREA sivis ts" or "hos pitalists," the bulk o f pati ents
OF SPECIAL COMPETENCE? requiring palliati ve care are outpa tients. A fo-
cus only on inst itu tional care would und ermine
Where do palliative care ph ysicians fit in the our com prehens ive focus and contribu te to fur-
panoply of ge neralists and specia lists in thi s ther fragm entation of end-of-life care.
WHAT IS PALLIATIVE CARE? 79

The refore, it might make sense to ca ll our- have the capability of offering a ran ge of con-
selves generalists, provid ing comprehensive, sulta tive and primary care serv ices. A sim ple
accessible, first-line care, but only to a subset consulta tive approach tha t focus es on sym p-
of pati en ts and their families-thos e facing a toms, pa rticu larly ph ysical sym ptoms, without
terminal illness. Th is approach is similar to add ress ing broader psychosocial a nd spiritua l
how ge riatricia ns may define themselves as as pec ts of pat ient and famil y suffering-----exem-
ge ne ralists for the elderly. However, bot h spe- plifi ed by some pain se rvices or pain a nd sym p-
cia lists and ge neralists populat e the field of pal- tom control teams-is neither state-of-the-a rt
liati ve care, and various medical subgro ups sym p tom control nor true palli ative care.
may ha ve different a ttitud es abou t and train -
ing for specialized consu ltation versu s com-
prehensive, int erdisciplinary primary care. ELEME NTS OF A D EFINI TI O N
Regardless of our orientation as specia lists or
ge neralists, I believe that we need to int erface Pall iative care is characterized as follows
effectively w ith patients, fami lies, and health-
care pro viders who have a va riety of need s, 1. Limiting itself to a particular clinica l condi-
wishes, a nd resources. We need to wo rk close ly tion : terminal illness or care at the end of
and comfortab ly with our clinical colleagues life. Unlike hosp ice, as currently regul ated
w ho provid e the bulk of preterminal care. For in this cou ntry, we do not need to specify
insta nce, w hen a skilled , ded icat ed primary a progn osis. In d escribing ourselves as
ca re provider is man agin g a case, we might act caring for the dying, we want to avoid eu-
solely as cons u ltants, providing ad vice directly phemisms, but also not be so blunt as to
to the referring ph ysicia n. Only pa rt of the pal- frighten pa tients and their fam ilies (e.g.,
liat ive care team (e.g., the socia l worker or speaking abou t "incurable" or "termi nal"
cha plain or volunteer ) mig ht become d irectly disease), nor be so kindl y as to become hope-
involved with the pati ent or family, com ple- lessly vag ue (e.g., describing pati ents as "ad-
menting the wo rk of the pr imar y care doctor. van ced "). With our health-care colleagues,
On the othe r han d , if the patien t is bein g ob- "termina l illness" is a rela tively clear notion
served, for instan ce, by a neu rosurgeon who and allows us the flexibili ty to participate in
views his or her job as largely com pleted after the earlier ph as es of "act ive, prog ressiv e" fa-
recovery from su rgery, patients, fami lies, and tal conditions that eventually becom e "far-
health-care provide rs may prefer that the pal- advanced ." For pati ents and families, "life-
liative care team ass ume a primary care role, threatening illness" may be the most
takin g respon sibili ty for not only the man age- a ppro priate d escriptive term, though it in-
ment of th e terminal illness but also for coo r- cludes con d itions, such as acu te trauma, that
dinating the input of the specialists, assuring are not typically within our domain.
goo d communica tion and oversee ing ge nera l 2. Employing a di stin ct me thod of eva lua tion
medical managem ent. For a pa tient u nd ergo- a nd management, a specia l expertise: (a)
ing chemo thera py or radiati on for cancer, we com prehens ive and (b) interdisciplinary (or
may want to coma nage care wi th the oncolo- collaborative) care.
gist or rad iation therapist, sha ring some re- 3. Directing care to the patient and the family
sponsibility for sym ptom man agem ent, bu t and , by im plicati on, ex tend ing care into the
perhaps taking a domin ant role in su pe rvising peri od of bereav em en t.
home care services or providing psychosocial 4. Focusin g on a specific man agem ent goal:
and sp iritua l su pport. prom oti ng quality of life (or livin g as well
Thus, in my view , palliati ve care in the as possible). This goa l includes a lleviatin g
United Sta tes mu st be flexible and collabo ra- suffering, prov iding su p port, and making
tive yet retain responsibility for ass u ring coor- the best of remaining time, but need not ex-
dinati on of co m prehens ive care and, at tim es, clude othe r goals, including cure or remi s-
provid ing a full ran ge of appropriate servi ces. sio n. Alterna tive terms that may be more
I believe that palliati ve care programs must acceptabl e to pati ents and families arc "com-
80 BILLINGS

fort care" or "su pportive care," altho ug h I that you and yo ur family rece ive exce lle nt
find that the former tends to imply passiv- pain con trol and other comfort measures, ge t
ity and withhold ing, as suggested by "com- the informa tio n yo u wa nt to pa rticipa te in de-
fort measu res only," wh ereas the latter also cisions abo ut yo ur care, receiv e em oti on al and
can mean trea tme nts clearl y aimed at pro- spi ritua l suppo rt and p rac tical ass ista nce, ob-
lon gin g or sus ta ining life. tain expert help in planning for care ou tside
the hosp ita l, continue ge tti ng good services in
Hence, palliative care is comp rehensive, in- the comm un ity , and overa ll enjoy life as best
terdisciplinary care, focus ing pr imarily on pro- yo u can, give n yo ur condi tion. We tr y to co-
moting qua lity of life for pati en ts living wit h a ord inate a nd tailor a package of se rv ices that
term ina l illness and for their families. Key ele- best su its your va lues, beliefs, wis hes, and
me nts for helping the pati ent and family live need s in w ha tever se tting yo u are receiv ing
as we ll as possible in the face of life-th reaten- care ."
ing illness include ass uring physical comfo rt,
psychosocial and spiritua l suppor t, and p rov i-
sion of coord ina ted services across va rious sites CONCLUSION
of care.
This exp lication, which still is a mouthful, Througho u t the world now, pa lliative care
does not mention any thing abou t su p po rting is developing as an area of specia l clinical
the service provid ers, an esse ntial feature of com pe tence . As a fled gling field, it now can
any palliati ve care program , ye t one that does boast of multiple clinical centers a nd training
not see m to deserve inclu sion in a bri ef d efi- progra ms, a va rie ty of fine textbooks, journals,
nit ion stateme nt. The definit ion also does not and educa tiona l conferences , and a sma ll re-
specifically ad d ress the compo nents of an in- sea rch enterprise . Pall iati ve care has attracted
terd isciplina ry team and, like othe r defini tions clini cian s fro m disparate backgrou nds and in-
above, does not speci fically me ntion volun teers terests, and hen ce th e field currently em braces
or bereav em ent serv ices. a d ivers ity of views abo ut its sco pe, goa ls, and
method s. This di versit y is a vir tue. Wh ere dif-
ferent vie wpoi nts and expertise a re sha red,
WHAT DO YOU SAY? cross- fertiliza tio n occu rs, a nd un tes ted as-
su m p tio ns are cha llenged . Ho we ve r, di versit y
For sta teme nts that are intended p rima rily im plies disagreeme nt or co nflict with in the
for clinicians and othe r health professio na ls, I field, and hen ce confusio n for those trying to
speak of "comp rehensive care, provided by an understan d us. Critics will eas ily find oppor-
interdisciplinary team, for pat ients and fami- tu nities for d eri sion. Dive rsi ty also can mean
lies living w ith a life-threatening or terminal ill- lack of meani ngful sta ndards. Our cha llenge
ness, particularly wh ere care is focused on al- is to avoid ortho doxy ye t move ahea d w ith
leviating suffe ring and promoting qua lity of grea ter u na nimity abou t the nature of the
life." I might then go on to clarify: "Major con- field .
cerns a re pai n a nd symptom man agemen t, in-
forma tion sharing and advance care planning,
psychosocial and spi ritua l su pport, and coor- ACKNOWLEDGMENTS
dination of care, includ ing arrangi ng for excel-
lent services in the comm unity." Dr. Billin gs is a Soros Facu lty Scholar of the
In talking with patients, I might say, "Pal- O pen Society Inst itute Project on Death in
liative care is a spe cia l se rvice, a team ap- Am er ica and was also su pported by a Na tiona l
p roach to providing co mfort a nd support for Ca ncer Institute gra nt R25CA 66818-01 for Hos-
persons living with a life-th reatening illness pice in Ge neral Medi cal Ed ucation and Prima ry
a nd for their families. We a re a nurse, socia l Ca re. The au tho r is gratefu l for helpful com-
worker, cha plain, and ph ysicians who wo rk me nts fro m Drs. Dav id Weissm an and Susa n
with your current health-care team to ass u re D. Block.
WH AT IS PALLIA TIVE CA R E? 81

REFERENCES 16. Cassell ES: 11,e nature of suffering an d th e goa ls of


medi cin e. N Engl I Med 1982;306:639-M5.
1. Wittgcn stcin L: PI,i1osoplrical lm.'eStigatiows, Part 1, sec- 17. Ca rlso n RW, Devi ch L, Frank RR: Devel opment of a
tio n 43, 1953. com p rehe nsive su p po rtive ca re team for th e ho pe-
2. Mount BM: The Roya l Victo ria Hospital Palliati ve lessly ill o n a university hospice medical center.
Care Se rvice : A Ca nad ia n ex pe rie nce. In: Saund ers C, l AMA 1988;259:378-383.
Kest enbaum R, eds. Hospice Care on the International 18. Doy le D, Hanks ewc, MacDon ald : Introduction .
Scene. ew Yo rk: Spri nger, 1997, pp. 73-85. In: Doyle D, Hanks ewe. MacDonald N , ed s, Oxford
3. Walsh TO: Continu ing care in a medica l center: The Textb<",k of Palliatitx: Medici"e. Oxfo rd , England : Ox-
Cleveland C linic Foun dation Palliative Ca re Service. ford University Pres s, 1993, p. 3.
J Pain Symptom Mgmt 1990;5:273-278. 19. Ferris FD, Cu m m ing s I, Ied s.): For the Canad ian Pal-
4. Weissma n DE, G riffie J: The Pa lliat ive Care Consul- liative Ca re Associa tion. Palliath: Can'; Toward a Con-
tati on Serv ice of the Med ical Co llege of Wisconsin. J sensus ill Standardiu d Principles of Practice. Firs t Phase
Pain Sym p to m Mgm tl 994;9:474-479. Workin g Docu me nt. O ttawa: Canadia n Palliati ve
5. Weissman DE: Co ns ulta tion in palli ative med icine. Ca re Association, 1995, p. 12.
Arch Intern Med 1997;157:733-737. 20. Doy le D: Facing th e 1990's: special issues. Hospice
6. Paw ling-Kaplan, O'Con no r P: Hospice care for m i- Upda te 1990;2:1- 9.
noriti es: An anal ysis of a hospital-based inn er city pa l- 21. Darwin C: TIlt' Origin of Spt'cil'S. 1859. Am herst. Ne w
liati ve care service . Am J Hospice Ca re 1989;july / Au - York: Prometheus Books, 1973.
gust:I3-21. 22. Field M], Cassel CK, ed s.: Com m ittee o n Ca re at th e
7. Gomez C F: Hospice and hom e ca re: oppo rt u nities for End of Life, Division o f Health Ca re Servin's, Insti-
trainin g. In : ABlM Committee 011 Etaiuatien of Clhlical tute of Medicine. A pproaclJillg Death: Imprtlvi" s Can' llt
Competence, Caring for the DyinS: ldentiiicotion tmd Pro- till' End of Life. Washingt on , DC: Na tio nal Academy
motio/l of Physician Competency. Philad elphia : Am eri - Press , 1997, p. 31.
ca n Board of Interna l Medicin e, 1996, pp. 27- 30. 23. Kriv icka s LS, Shockley L, Mitsumoto H: Home care
8. Field Mj, Cassel C K, eds.: Committee on Ca re at the of pa tients wi th amyotrophic later al sclerosis (ALS).
End of Life, Division of Healt h Can' Services, lnsti- I Neurological Sciences 1997;152 s up pl 1:582- 589 .
tute of Medi cine. Approachillg Death: Improving Careat 24. Man n SM, Welk TA: Ho spice and /or pa llia tive care?
the End of Life. Washington, DC: National Academ y Am I Ha sp Palliat Care 1997;(Novempcr / Deccm-
Press, 1997, Pl'. 9- 10. ber):314-315.
9. Bascom PB: A hospital-ba sed co m fort ca re team; Co n- 25. Ch ristakis NA, Escar ce J): Su rv iva l of Medicare pa-
su ltation for serious ly ill a nd d yin g patient s. Am I tient s after enrollme nt in hospice programs. N Eng l I
Hosp ice Palliati ve Ca re 1997;14:57-60. Ml'<l 1996;335:172- 178.
to. Ca rlso n RW, Devich L, Frank RR; Devel o pm ent of a 26. C leary [F, Carbone PI': Palliati ve med icine in th e el-
co m p rehe nsive su p po rt ive care team for th e hope- derly. Ca ncer 1997;80:1335-1 347.
lessly ill o n a uni versity hosp ice m edica l center. 27. Billings lA , Block SD: Pall iat ive care in u ndergra d u-
lA MA 1988;259:378-383. ate medical ed ucat ion: Sta tus report and fu tu re di -
11. Berge r AM, Port enoy RK, Weissm an DE: Preface. In; rections. lA MA 1997;278:733-738.
Berger A, Portenoy RK, Weissma n DE, (ed s.): Princi- 28. Weissman DE: Co ns ulta tio n in palliat ive medi cine.
pit'S and Practice of Supporth't' Ollcology. Philadelp hia: Arch Intern Med 1997;157:733-737.
Lippi ncott-Ra ve n, 1998, p p. xix-xx.
12. jones A, Faulkner A: \Vhat d oes palliat ive can." mea n?
Contempo rary professional opinion. j Cancer Ca re
1996;5:39-43. Address reprint requests to:
13. World Heal th Orga niza tion : Cancer Pain Rt'1it1and Poi- /. Alldrew 8illillgs, M.D.
liatiix Can'. Technical Report Series 8o.t. Ge neva, Palliatinc Care Scroice
Switze rlan d : Wo rld Health O rganiza tion, 1990, p. 11.
14. Ph illips DC: Holistic Thought ill Social Sci('IICt'. Stan-
Foum/as 600
ford , CA : Stanfo rd University Press, 1976. Massachusetts Gelleral Hospital
15. Kearney M: Palli ative m ed icin e-s-just a no the r s pe- 55 Fm it Street
cialty ? Pa lliat Med 1992;6:39--46. 80stOll, MA 021 14-2696
Copyright 1997 by the American Medical Association. All Rights Reserved. Applicable FARS/DFARS
Restrictions Apply to Government Use. American Medical Association, 515 N. State St, Chicago, IL 60610.
Volume 277(20), 28 May 1997, pp 1633-1640

An 88-Year-Old Woman Facing the End of Life


[Clinical Crossroads]

Lynn, Joanne MD, MA, MS

Dr Lynn is Professor of Health Care Sciences and Medicine and Director of the Center to Improve Care of
the Dying, George Washington University Medical Center, Washington, DC.

Clinical Crossroads at Beth Israel Deaconess Medical Center is produced and edited by Thomas L.
Delbanco, MD, and Jennifer Daley, MD; Erin E. Hartman, MS, is managing editor. Clinical Crossroads
section editor: Margaret A. Winker, MD, Senior Editor, JAMA.

DR DALEY: Mrs J is an 88-year-old widow who has had multiple recent episodes of congestive heart failure
[CHF] requiring hospitalization. She lives alone in an assisted living center for senior citizens in a suburb
of Boston, Mass. She has 1 daughter, Mrs V, who is very involved in her care. She has Medicare insurance
and a supplemental Medigap insurance policy.

Mrs J has long-standing essential hypertension, coronary artery disease with stable angina pectoris, aortic
stenosis, and an elevated alkaline phosphatase suggestive of Paget disease of the bone. She has chronic
CHF, paroxysmal atrial fibrillation, 2 to 3 episodes of angina per week relieved by nitroglycerin, and
recurrent episodes of flash pulmonary edema, necessitating 4 hospital admissions in the previous 10
months. Within the past year, Mrs J refused needle biopsy for evaluation of a multinodular goiter.
Functionally, she needs assistance only with bathing, but notes fatigue and some shortness of breath
following any activity, including moving about her apartment. Current medications include furosemide (80
mg daily), isosorbide dinitrate (20 mg 3 times daily), aspirin (325 mg daily), digoxin (0.25 mg daily), and
nitroglycerin as necessary for angina pectoris.

Mrs J is a talkative, enthusiastic, and gracious woman. On physical examination, her skin tone is pallid and
her extremities show mild cyanosis. Her blood pressure is 110/70 mm Hg, pulse is 110 beats per minute
and regular, and her respiratory rate is 26 breaths per minute. She has delayed carotid upstrokes
bilaterally, diffuse wheezing and bibasilar rales, a 3/6 systolic ejection murmur at the left sternal border
radiating to the neck, and a 2/6 holosystolic murmur radiating to the left axilla. She has poor distal pulses
and 3+ to 4+ pitting edema of the feet and legs to the knees. Her neurological examination is normal
except for moderate bilateral hearing loss. After echocardiography last year showed an ejection fraction of
less than 20% and aortic stenosis with a 35-mm gradient, cardiac catheterization was performed and
demonstrated 3-vessel coronary artery disease; aortic stenosis with a 40-mm gradient; and an ejection
fraction of 35%. Mrs J wanted her daughter to make decisions, and so Mrs V and the physicians decided
that any invasive procedure to relieve Mrs J's symptoms would be too burdensome and would put her at
too much risk, and that medical management was a better way to handle her illness. Mrs J would like to
live as she is doing now, in the assisted living facility, as long as possible.

MRS J: HER UNDERSTANDING AND PERCEPTIONS

I'm not afraid. If I have to go, I go. I have a nice family, my daughter, her husband, and 2 beautiful
grandchildren who have accomplished a lot. My daughter is very good to me. So when the time comes I'll
say good-bye. I'm happy here [at the assisted living center], and I've got a lot of company that comes to
see me.
We [my friends] never talk about [dying]. People here sometimes get very sick. Sometimes they take
them out, and I'm not really sure what happens. If I'm very sick, I'll be in a hospital. I wouldn't stay here.
I hope that I'll be getting along. I'm at that age, you know.

MRS V, MRS J's DAUGHTER: HER UNDERSTANDING AND PERCEPTIONS

I have not confronted my mother about dying, about her very limited life span. My feeling regarding my
own mother is that whether she has 1 week, 1 month, or 6 months, her life is not going to be lived any
differently. She is not going to take a trip, or spend a lot of money that she doesn't have. I'm not sure
whether, for her, knowing precisely when the end of life will be is going to matter. As I see it, she has no
unfinished business. I feel very fortunate that I have time with my mom, knowing that on a daily basis
she feels OK. I'm not seeing her suffering now, and we have time to talk.

Everyone is an individual. Paying attention to the person may be even more important than the medical
problem. I know different families respond differently. I absolutely defer to the medical community
because I have no medical knowledge, and I am willing to understand and to read. I trust the people I
have chosen to take care of my family.

My mother and I talked about resuscitation vaguely a few years ago, in relationship to my dad. I have not
discussed it recently with my mom. I have discussed it with Dr Z, and I will be in charge of making
decisions if she can't make them. At her age, and with her joy of life, and with the great risks of
resuscitation, I think we're talking about a machine breathing for her, which may result in her being a
nonvital being afterward.

DR Z: HIS UNDERSTANDING AND PERCEPTIONS

I've been approaching end-of-life issues with this patient in a subtle fashion. Although she is intelligent
and wants to know about her health, I deciphered over the time that I've known her, as well as through
conversations with her daughter, that she does not want to spend lengthy periods of time talking about
the topic of death. I have spoken to her recently-given her coronary artery disease coupled with severe
aortic stenosis-about the prospect that her situation is going to be one of insidious decline. I spoke in
broad generalities about her prognosis-that there was very little more that we could do without taking
great risks to her well-being. She fully understood it.

Expressing to her exactly how much time she has left is troubling not just to me, but to her daughter.
None of us has a clear-cut answer to that, but we do know that 6 months is probably a reasonable range.
Her daughter and I have the sense that the patient doesn't want to know this.

We've talked about what we would do if Mrs J goes into pulmonary edema. We'll avoid hospitalization if
possible, use diuretics as much as possible and even a morphine drip. Hospice care is being investigated.
It's the wish of the patient, her daughter, and myself to try to keep her at the assisted living community
as long as possible. Hospice is wary, however, since her prognosis is somewhat uncertain, and her needs
are not what they usually deal with. The assisted living center seems to be uneasy about her possibly
dying there unless hospice is supervising things.

If the patient did awaken one morning and asked me to help her die because she was in terrible straits, I
would try to reassure her that we would do everything possible to make her comfortable, including using
morphine. I could not be responsible for taking her life. I can only help her to be more comfortable in her
dying.

AT THE CROSSROADS: QUESTIONS TO DR LYNN

What are the issues for patients, families, and care providers as a person comes to the end of life? What
constitutes excellence for a primary physician serving a dying person? How should the physician approach
diagnosis, prognosis, life prolongation, and symptom management? How can the primary physician and
other caregivers respond to the patient's and family's fears of impoverishment? How can health care
professionals relate to the dying person's search for meaning and personal growth?
DR LYNN: All of us will die, and, like Mrs J, most of us will die at the end of a long life. Only a few
generations ago, people usually died quickly and dying happened at any age. The median age at death
now is 77 years, and 84 years for women who survive to age 65. [1] Disability for those past the age of
65 years affects an average of 6 of the last 15 years for men and 8 of the last 19 years for women. [2]
Only recently have health care providers and administrators begun to focus attention on how to serve
those with slowly fatal conditions.

Medical texts attribute dying to severe physiological dysfunction. A patient's experience of dying is quite
different. Beyond pain and dysfunction, the dying are absorbed with the annihilation of the self, the impact
on loved ones, the terror of the unknown, and the opportunity for transcendence beyond the mortal.
Health care professionals must learn how to take an appropriately supportive role in this human drama.
Doing so requires not only diagnostic and therapeutic skills, but also learning to function creatively within
ambiguous situations, enabling patients to experience personal growth and leave-taking, and planning
ahead to prevent emergencies.

Living With Ambiguity: Diagnosis and Prognosis for the End of Life

Diagnosis is important for establishing prognosis and evaluating treatment, and often for reassurance.
Threatening situations are tamed somewhat by being named. For many patients nearing the end of life,
however, the time remaining might be spent better in some personally meaningful way, rather than in
enduring burdensome diagnostic procedures. [3] Mrs J's heart condition has been definitively
characterized by cardiac catheterization to determine whether a potential surgery would have needed to
include coronary bypass as well as valve replacement. Yet, she had refused biopsy of her enlarged thyroid
1 year earlier and seems complacent about death's timing. Would she really have had open heart surgery?
The catheterization might not really have been an advantage to her.

Dr Z says that Mrs J has a life expectancy of "6 months." What precisely can this mean? A prognostic
statement provides an estimate based on survival rates of similar patients. For most illnesses, in most
patients, authoritative data about outcomes and covariates have not been collected. Readily available data
for CHF, for instance, include only prognostic statements for symptomatic aortic stenosis alone (mostly in
much younger patients) [4] or survival time for younger hospitalized end-stage CHF patients. [5] The first
would predict half of all patients to be dead within a year of a first episode-but this may be misleadingly
optimistic in Mrs J's case since her paroxysmal symptoms probably reflect ischemia rather than aortic
stenosis. The second would have predicted (on the third hospital day) about 58% survival at 6 months
and about 82% at 1 month. Either way, no data directly address Mrs J's condition and the best Dr Z can
say is something like this: "Mrs J, if we had 100 persons just like you, the best estimate is that about half
of them would be alive in 6 months."

Congestive heart failure is less predictable than many causes of death and is generally discussed as a
chronic condition rather than an eventually fatal illness. A patient dying of colon cancer, for example, has
a long period of functional stability, eventually ending with a month or two of progressive disability and
weight loss [6] (Figure 1). While survival times of similarly ill cancer patients in the declining phase may
vary a few weeks, their trajectory toward death is ordinarily apparent. A CHF patient will most often
exhibit a slow and lengthy decline in daily function, with periodic bouts of severe symptoms and disability
(Figure 1). While death may occur in the throes of a severe exacerbation, or after a very slow background
decline, most deaths probably occur suddenly and relatively unpredictably, from arrhythmias. In the Study
to Understand Prognoses and Preferences for Outcomes and Risks of Treatments (SUPPORT) study, CHF
patients had surprisingly optimistic prognoses even up to the day before death. The median prognosis on
the day before death was a 50% chance to live 2 months (Figure 2) [5,7]
Figure 1.-Trajectory of dying for colon cancer or congestive heart failure.

Figure 2.-Median prognosis by day before death for 3 illnesses in the Study to Undertand Prognoses and
Preferences for Outcomes and Risks of Treatments (SUPPORT) study. CHF indicates congestive heart
failure; ARF, acute renal failure; and MOSF, multiorgan system failure. Adapted and printed with
permission from the Society of Critical Care Medicine (Lynn et al [7]).

Younger patients (<80 years) usually come to the end of life through a single, serious illness and may
attempt to prolong life, even against long odds, to attain what they believe to be a full life span. Our
culture seems to presume that younger patients with established serious disease are to be much more
aggressively diagnosed and treated than those older than 80 years. [8,9] Those of advanced years
(roughly past 80 years), with substantial established disease and disability, often have a confluence of
illnesses and lack of reserve. They, their families, and physicians seem to find it appropriate not to pursue
vigorous diagnosis and treatment, even with uncertain prognoses. The implicit age-based heuristic
presumes that "aggressive" treatment is appropriate for those younger than about 80 years, and "comfort
care" is appropriate for those older. If further research confirms this pattern, we must then ask if it is
unjust or deserves some attention. Overtreatment of younger patients may be at least as much a problem
as undertreatment of older ones.
Mrs J is aware that she will die but finds pleasure in continuing. Her life accomplishments are satisfying
and the thought of death is not particularly disturbing. Dr Z has recognized that the patient will die of her
disease. He has, commendably, discussed at least general preferences with her and helped plan for a
reasonable quality of life. Precision in prognosis is not available and not particularly important to Mrs J or
her family. The patient should know that death will likely come suddenly, when it finally comes. If she has
important things to "close," she might want to do so in advance. Most people dying in old age have had a
few episodes of exacerbations of serious illness, as Mrs J has had. Gillick points out that these can function
as "rehearsals" for death, and experience with them can be used to help the patient and family make
more specific plans and cope better when death occurs. [10]

A Good Life at the End of Life

In past centuries, people had less opportunity to dwell on the exigencies of death because it often came
swiftly and at an early age. Today most of us will have an extended period in which to experience life
shaped by a serious and eventually fatal illness.

For too many, this will be a grim undertaking amidst palpable adversity-facing debilitating symptoms,
fragmented support services, devaluation as persons, isolation, and impoverishment. In a national
telephone survey, family members reported that about half of recent deaths of conscious patients were
marked by moderate to severe pain most or all of the time. [11] Pain afflicts about one third of cancer
patients near death [12] even though the rates in special programs focused on relieving cancer pain are
less than 10%. [13] Other reports have substantiated serious pain in other illnesses. [14] Depression,
delirium, anxiety, and fatigue all too commonly obliterate the opportunity for a meaningful closing to life.
Each is potentially treatable, or preventable, but is often unrecognized against the background of disease
or is accepted as a matter of course. [15,16] Both emotional and financial hardship generate even more
distress for patients and families.

Deficiencies in the care of the dying do not result from a vacuum of understanding about what constitutes
a good dying, or the conditions required to achieve it. Physical comfort is a prime ingredient along with
the availability of a place to live where daily needs are easy to meet. Satisfaction of these fundamental
requirements allows the patient to attend to spiritual and emotional concerns. Dying persons value human
contact (physical touch), being heard, feeling empathy, and feeling that care is trustworthy. Practitioners,
however, rarely have received instruction in caring for those coming to the end of life. The end of life is a
potentially rich and rewarding period for human development. Our goal must be to maximize the
opportunity for each dying individual to live well on their own terms, right up to the time of death.

Clinical Strategies

Excellence in caring for a person at the end of life requires that the clinician understand 4 critical
elements. First, understand the patient's story, including the role of the family and the ways that the
patient and family make sense of life and its vicissitudes. The clinician then can assume a helpful and
fitting role within that story. Second, understand the body, and the limits and possibilities determined by
its ailments over time. Third, understand the care system and what can be done, routinely and in
exceptional situations. The clinician must understand regional health care resources and relevant law and
ethics. Finally, understand oneself. Only then can the practitioner be an effective instrument of healing in
the context of fatal illness. All of these are difficult and the last is often a lifelong quest. Understanding the
illness is included in medical education, albeit superficially. The rest are not generally addressed in
training. Thus, to achieve excellence, physicians must find other ways to learn, including reading and
sharing knowledge from other disciplines and learning from patients and families. In fact, the most
important single guidepost to good care at the end of life is for the professional to listen and come to
understand the patient and family (Table 1).
Table 1.-Clinical Strategies for Good Care at the End of Life*

Other specific strategies prove helpful. Time-limited trials of aggressive diagnosis and treatment can teach
all involved a great deal about the disease and the body as well as about how the patient's body and
family's system respond. Key participants agree to reassess the merits of such procedures on a scheduled
basis, and to stop the procedures if they are not helping. For Mrs J, for example, one might plan to treat
the next CHF exacerbation with a 2-day trial of aggressive monitoring and pharmacological management
(with morphine for dyspnea and vasodilation, but without ventilator or resuscitation). If Mrs J were losing
ground, all efforts at rescue would be stopped and only symptom management would be pursued while
she died. If she were clearly improving, however, she would probably go home in much the same state as
she has now.

I find it helpful to tell patients and families that I cannot know the patient's reserve and the behavior of
the disease well enough to stop treatments at just the right time. Generally, I tell them that the best I can
do is to overtreat each patient a little, since that is often what I need to do to understand the body's
reserves. I promise, however, that I will make every effort to keep these trials brief. Once it is clear that
they are not warranted, I will avoid repeating treatment that has failed because the patient's dwindling
reserves limit the capacity to rebound back to baseline function. Patients and families are usually
reassured by the overall approach and are generous in forgiving our uncertainties. Confident that a good
care plan can be crafted, patient and family fears about ambiguity and uncertainty are generally relieved,
not heightened, by honesty.

Good care of the dying depends critically on the ability to promise future behavior-to "make deals."
Knowing that the physician or nurse will be there throughout the dying process and that they will keep
their promises is reassuring to patients and families. Sometimes patients feel that dying will be so awful
as to drive others away. The fact that caregivers calmly expect a comfortable passing and expect to be
there to attend to any needs is profoundly therapeutic. Although written documentation of such a plan
might suffice to guide care with multiple discontinuous care teams, [17] it is probably less reassuring than
the continued personal presence of a trusted professional. Good care of dying persons is not compatible
with schedules that have physicians "rotating off" every month, or in which the patient loses all of his or
her trusted professionals with a change in venue to hospice, hospital, or nursing home.

Medicine, ethics, and law describe care as if it were marked by a series of decisions. Near the end of life,
the disease progressively constrains the options. The care system makes it easy to have only certain
patterns of care. To patients and families, the experience may feel more like an ongoing negotiation with
fate, trying to find ways to get through each day. Sometimes the course of care comes to a point in time
when the goal changes from prolonging life to comfort care. That image has been promoted extensively in
the care of cancer patients. [18]

Many persons, however, will arrive at death's door with ambiguous prognoses and uncertain diagnoses.
For these patients, it will not be foolish to hope for some recovery with treatment, but we would do well to
note that death is a prominent possible outcome, even with treatment. These patients may need a "mixed
management" model of care in which the physician provides time-limited treatment trials, even while
helping the patient to find meaning in the end of life, including having the chance to say farewells, to
forgive, and to share love. Multiple goals will sometimes be conflicting; a simple formula should not be
crafted to make these difficult situations seem falsely routine.

Unfortunately, we do not know much about Mrs J's search for meaning or her spiritual concerns or growth
as she neared death. Mrs J noted that "We never talk about it." Our culture shares few stories and myths
about how to live well while dying. A patient recently said to me, "I have looked in the Bible for guidance
as to how to live, but no one there faced what I face, dying so slowly. How am I to learn what to do?" Mrs
J says that people go to the hospital from her senior living facility and only a few come back. However,
she does not say they die. Indeed, she may not really know what happens to them or be able to speak of
it. Her language is eerily reminiscent of the future world in The Time Machine, [19] where the sick or old
are quietly harvested, and those left behind do not speak about it.

Our culture has made it difficult to address certain issues and our impoverished language reduces us to
speaking merely of the tangible: "Would you want us to try to restart your heart?" We desperately need
formulations such as these: "You will die from this, though we can't be sure when. What is it that is really
important to do in the time left?" or "Are you concerned about what will come after death?" or "How do
you see your life now-are you at peace with yourself?" A good clinician can share insights and stories,
identify resources such as useful writings, [20-23] and recognize that many people need a guide through
this unfamiliar and threatening terrain. To be an excellent diagnostician or therapeutic technician is not
enough.

Communication and Advance Care Planning

The conversations that most physicians have with patients about dying are much like those that Mrs J and
her daughter shared with the interviewer-imprecise, inexplicit, and uncomfortable, yielding no particularly
strong guidance for the course of care. Mrs J was asked if she wanted "everything possible done to keep
you alive." Her answer, not surprisingly, was "Yes." Surely this exchange claims more than it merits! She
almost proudly states that she was too old to have an operation and she contended that when her time
came, she would just "go." She noted that she trusted her daughter, a view confirmed by the daughter,
who added that her trust was in the doctors.

Talking about dying is unfamiliar, but professional caregivers often find it fairly easy. The most important
general rule is to talk less and listen more. Ask patients and families what they think is likely to happen,
and ask them to say what is most worrisome and what is most to be desired. Usually this yields an array
of leads to follow: "Tell me more about how your mother died," "Do you really think that you will get over
this illness?" or "Why do you think that your time will be so short?" Physicians under time pressures often
avoid such conversations, fearing being pulled into long exchanges. Structuring the time so that patient
and family know when the doctor must leave, bringing other members of the care team into the
conversation (including chaplains, social workers, nurses, and trained volunteers), and having multiple
conversations over time are helpful ways to balance other demands. Mostly, though, I have been
astonished at how little time really important conversations usually take. Often, an understanding that is
sufficient to ground a therapeutic relationship that lasts a lifetime takes less than 15 minutes. Since this
time avoids frantic phone calls to make urgent decisions and dealing with anger over misdirected care, it
almost certainly saves time in the long run.

What choices does Mrs J need to make in advance? Serious chronic disease as a cause of death means
that problems can usually be anticipated and optimal plans could largely avoid emergency interventions or
frightening situations. Unfortunately, contingency planning is rare. It is not enough to have Mrs J sign a
standard living will; they are just too vague. [17,24,25] Instead, persons with serious illness or greatly
advanced old age and their caregivers need to make specific plans and preparations for the complications
that are likely in their situations. Persons with metastatic cancer to the brain need preparations for
possible seizures, persons with severe obstructive lung disease need to prepare for dyspnea, those with
CHF should expect sudden death from arrhythmia, and so on. The right drugs need to be available. Clear
plans should address hospitalization, use of the emergency medical system, attempts at resuscitation, and
so forth. Persons living alone need alarm systems to summon help, and emergency medical response
teams must have ready and reliable access to care plans. [26,27] In addition, persons coming to the end
of life need the opportunity to have completed financial planning and addressed family issues, to have
made appropriate plans about memorials and burial, and to have attended to meaningful cultural and
religious rituals. None of this happens unless the possibility of impending death is recognized and
discussed. As Mrs J's daughter said, "Paying attention to the person may be even more important than the
medical."

If a patient or the family is avoiding the subject of death at a point when establishing communication is
likely to be important, encouraging them to take on some specific task that relates to the time after the
death is often strikingly effective in reorienting them to reality. The physician might ask the patient about
plans for the eventual memorial service and burial, or for advice as to what help certain family members
are likely to need in the time after the death. The nurse might ask the patient or family to name the
funeral director or to consider who should be present at or shortly after the time of death. If the patient is
leaving behind small children or grandchildren, patients and families could make audiotapes about the
patient's history, character, and accomplishments. Giving everyone permission to think about the time
after the death makes the dying more real, and also more manageable.

What planning does Mrs J really need to guide the end of her life? Physicians and hospital staff expend
much anxiety over orders against resuscitation. With good communication about priorities and
possibilities, the decision about resuscitation is usually obvious and not controversial. Most dying persons
will have periods of delirium, somnolence, or inattention and will need someone else to make decisions.
Mrs J's surrogate decision maker is obvious. If her family situation were more complicated, the primary
care physician should sort out who would best negotiate her care and protect her interests. The patient's
chosen surrogate(s) usually benefit from discussion with the patient and the primary physician. Ethicists
and lawyers advise designating the surrogate with a formal durable power of attorney for health care.
Doing so, however, is rarely needed unless the best surrogate is contentious, or the care is to be given in
a state where only formal designations are likely to be honored, eg, New York State. [28] If a patient has
no friend or family suitable to serve as a surrogate, the physician must realize that no state has
effectuated reliable and effective procedures to ensure good decision making. Thus, such a person should
be encouraged to make more decisions about health and personal care in advance, aiming to guide
physicians, caregivers, and possibly courts of law.

Mrs J, her daughter, and physician have made some plans. They have discussed treatment options
targeted toward the symptoms this patient is likely to suffer. Diuretics and oxygen will be mainstays of
therapy. Morphine will be offered to relieve shortness of breath if other treatments are insufficient. It is
critical in her situation to decide between a respirator and sedation since severe dyspnea creates an
emergency. The decision to use a ventilator would need to be revisited over time, as her condition
worsened. In Mrs J's situation, whether she would stay at home or be hospitalized seems to remain
unsettled, pending further negotiations between hospice, the assisted living center, and her physician and
family.

Opportunities for Meaningful Reform

A profoundly dysfunctional care system regularly thwarts good care at the end of life. When patients and
families need continuity, they meet fragmentation. Seriously ill persons are not desirable in capitated
Medicare, since their needs far exceed the payment rate. They are not well served by fee-for-service
arrangements either, since needed services are not well reimbursed and are often best provided by those
who cannot directly bill, such as chaplains, social workers, sitters, and nursing aides. "Merely" supportive
services are excluded from Medicare, and what is available through Medicaid and private arrangements is
often unreliable. Very few care systems arrange for the same professional caregivers to be available in
multiple settings, and few ensure that plans made in 1 setting are carried forward into others. Enduring
change will require restructuring the incentives to encourage and sustain excellent care and to drive out
the mediocre.

To guide change, measurement of quality care is essential. Care systems do not generally measure quality
of care for the end of life. Recently, a few dozen national patient and professional organizations issued a
statement articulating the elements of care that need measurement and called on providers, funders, and
certifiers of care to require good performance (Table 2). [29] Doing so will require gaining experience in
measuring quality components, in seeing whether improved practices affect outcomes, in establishing
benchmarks for performance, and in setting standards.
Table 2.-Components of Quality Care at the End of Life*

What is the optimal setting for living at the end of life? Just a decade ago, most people who died at the
end of a serious illness spent the last weeks in a hospital. Hospice programs have shown that many
persons can live well while dying, if comprehensive services are mobilized to their homes. Many more
deaths for the elderly are now occurring in nursing homes. Avoiding the isolation and disruption of
hospitalization might justify these shifts, although they probably are motivated as much by broader cost-
containment pressures in hospitals. Nevertheless, nursing homes and home care are not automatically
better. We will need supplemental resources and careful monitoring to be sure that they follow the
encouraging lead of hospice rather than serving merely as a way to remove costs from the hospital
system. (Table 3) shows some representative reimbursement rates for Medicare hospice care.

Table 3.-Hospice Payment Rates*

Mrs J's situation illustrates the current dysfunctions of our health care system. She meets the formal
qualifications for hospice, but her needs as she dies of heart failure cause most hospice programs to react
quizzically since the typical hospice nurse, who is quite comfortable with morphine pumps, will often be
unfamiliar with treatments for pulmonary edema. Likewise, the assisted living center has few resources
and little familiarity with treating serious illness. The patient assumes that she would go back to the
hospital if she were sick again, but the physician clearly knows that hospitalization does not generally
comport with a comfortable dying. The possible roles of these various settings and service providers is
now quite unsettled, and Dr Z will have to respond to the real possibilities confronting this patient. In
addition, the physician and others should take an active role in defining the roles of institutions and
service providers. If persons like her are to live well under the shadow of death, some care setting will
have to become competent at serving such a patient's needs. Hospitals could learn to care for the dying,
or assisted living centers could learn to collaborate with home care teams, or emergency response teams
for end-of-life care could come to these settings. One can see various possibilities, but for now, each care
setting has some serious shortcoming, and no one has thought through a reliably excellent clinical
pathway for patients like Mrs J.
Practitioners who serve the dying certainly need to assure excellence in their direct work with patients, but
they also bear responsibility to create change. If, for example, instructions for relief of patient suffering
are lost on transfer between the nursing home and hospital, practitioners need to ensure that the system
improves handling of important information. If patients and families feel that they were left adrift and
isolated, practitioners must redesign their care system to better meet these needs. The tools and
techniques of continuous quality improvement seem well suited to this work. [30] Since those who are
dying are dependent and transitory, policymakers don't usually hear their voices or feel compelled to
respond. We who serve the dying, in part because we anxiously note that we too will eventually be at the
end of our own lives, will have to shoulder responsibility for defending their interests. Meeting the needs
of the dying is, in the end, serving the interests of us all.

QUESTIONS AND DISCUSSION

DR DALEY: Mrs J was admitted to the Beth Israel Deaconess Medical Center early this month with chest
pain and pulmonary edema. She was very ill. She was treated with morphine, and she waxed and waned
in her consciousness. Her doctor, the house staff, and the family decided after several days that it would
be best to transfer her to the palliative care unit. She died there a few days ago.

MRS V: Everything Dr Lynn said, we lived. With Dr Z and my family, I think we did a lot of the good things
that Dr Lynn said. My mother went into the hospital on Saturday. She was very upbeat, even though she
knew she was very ill. Monday I received a call from Dr Z, and he told me about the conversation he had
just had with my mother. She said, "This is the end, isn't it?" Dr Z said, "I think this is the beginning of
the end." And my mom said, "I'm not quite ready." She said, "I would like to see my children and
grandchildren, once again, if I could." My children had seen her over the Thanksgiving holiday, and I
thought they had said good-bye, and they did too. Consequently they did not see her before her death.
On Tuesday, it was suggested to us that hospice be put in place at the palliative care center. She totally
trusted her physician from the moment she met him. I felt that if she knew that she were within a hospital
setting, even though it was a hospice, this would make her very comfortable. On Wednesday, she came to
the hospice, and she was more aware. In the late afternoon she encouraged me to go home. She passed
away early Thursday morning. I am convinced that she did not have a painful death. I believe she told Dr
Z, as she told me, she did not want to suffer. She did not want to be in pain. My mother lived a very kind
life. She had no unfinished business that I could see. She always had peace with everybody, and I think
the end was a very good one for her.

AN INTERNIST: Working with dying patients and their families takes a lot of time and I don't get
reimbursed for all the time it takes. It requires time for the patient, time to get together with the family,
and time to make arrangements with hospice agencies. At least in my life, time is a scarce commodity.

DR LYNN: The current incentives both in fee for service and managed care are dramatically contrary to
good care, although for different reasons. Those are the kinds of incentives that ought to be undone and
rearranged because at the present time you can go broke doing good care. You cannot advertise that
you're doing excellent care in this arena, because you'll attract costly patients for whom you cannot be
paid well. On the other hand, the actual time costs to a doctor working in a good care team are very
modest. You would be astonished how quickly you can learn an awful lot about the patient just by being
open to it, and listening as they talk and picking up on some cues. It really can be exceedingly efficient.
So it's certainly much more efficient, but it isn't well paid. Almost all really well-functioning and
sustainable care systems work with teamwork. It's not just the doctor whose skills may be in demand, but
other people who are good at counseling. Involve whoever is good at supporting the family and providing
the counseling, so that the doctor doesn't have to Figure outthe care system alone. The doctor who has to
Figure outhospice eligibilities, what the local hospice will do, and what the home care agency will do, will
do it once or twice a year. But a social worker or administrator who knows these things can do it so
efficiently that you can lower your costs. The impact of good care on costs in a managed care arena are
enormous.

AN INTERNIST: How should we train young doctors about caring for the dying? They seem very distant
from the current internal medicine programs that we have, and I think surgical programs are probably not
much different.
DR LYNN: Surgical training programs have almost no experience except with calamitous death. Internal
medicine trainees should be required to care for at least 6 patients who are dying. Trainees should learn in
systems of continuity where they stay with the patient, do the bereavement follow-up calls, see what
questions the families still have, and learn the tremendous rewards of working in that kind of an
environment. There are concrete skills to be learned. In 1 study, the majority of oncologists did not know
how to manage morphine. [31] We can start by demanding performance, creating accountability for
outcomes, and learning the relevant skills.

AN INTERNIST: In addition to the time and resource issues for individual physicians, how should a medical
center like Beth Israel Deaconess Medical Center, known for excellence in this area, respond-given current
reimbursement? Isn't it financially irresponsible of the leadership of this hospital to invest in systems that
would attract patients to see us as excellent in this area, until we are appropriately reimbursed?

DR LYNN: You are right-we are in a very difficult situation in which it is financially risky to develop
excellence. [32] The issues are different in capitated care and in fee for service. In capitated Medicare, the
monthly rate is obviously lower than the dying patient's service needs. The plan and provider do not want
to attract people this sick and may discourage enrollment, encourage disenrollment, and seek transfer out
of plan to hospice. Even when the patient's care is unavoidable, incentives are strongly toward
underservice and perhaps hasty acceptance of early death. In contrast, fee for service distorts good care
because it underpays support, counseling, and symptom-relieving drugs and overpays for procedures.

There are some clever, small-scale, sustainable foci of excellent care, [33] and, of course, for some
patients for short times, hospice services. In the long run, however, we must all raise our voices that it is
just not a tenable situation. We didn't develop a health care system to serve the healthy. We developed a
health care system because we are going to get very sick and die. A health care system that cannot
support excellence at the end of life isn't worth sustaining. In the meantime, institutions like this one are
called on to achieve excellence as a matter of mission and calling, despite adverse financial incentives.
Leadership institutions cannot just accept indefensible patterns of care.

DR DELBANCO: If you were her doctor and she asked you to help her die, because she was feeling
terrible, what would you say?

DR LYNN: I would give much the same answer as Dr Z gave. That that's just not among the things that
I'm available to do. On the other hand, I'll see to it that you do not die of suffocation and that you will be
sedated enough that you will not feel short of breath. I'd redouble my efforts to make sure that is true. If
we accept physician-assisted suicide now, I'm fearful that poor patients will be at great risk for being killed
because our society doesn't choose to serve them. The American Geriatrics Society brief for the Supreme
Court [34] says that, if we legalize physician-assisted suicide for patients whose lives are being made
miserable by the community's choices about supportive care, then we are not much different from the
Nazi doctors. It has to be possible to live through to the end of life with confidence that you will be
comfortable, comforted, and in relationship with others. I think we should not be ready to kill people
because we are unready to change the care system.

MR V, MRS J's SON-IN-LAW: I must say that we had excellent medical guidance, but very little guidance
as to options in moving from an assisted living center to a nursing home or hospice. We were going
through a learning curve that we will never really need to use again. It's foolish for individuals to have to
go through that whole curve, rediscovering what somebody else has already found out.

DR LYNN: You're certainly correct. The care for seriously chronically ill people has been orphaned in this
country, and we have been blind to it. We don't have regular systems in place that your doctor can call
on. He or she can call on a hospital social worker to get you discharged, but he or she can't call on
someone who will follow through with you and make sure you know the relevant options, can see the
landscape ahead, and feel confident that you aren't going to be bushwhacked by something unexpected or
fail to be made aware of something that really could have been a substantial advantage. We need those
kinds of arrangements. This approach is called "Medicaring," a system in which people who are seriously,
chronically ill with the illness that will kill them will get comprehensive services from that point until death.
A Medicaring approach would ensure that someone would help families through all these things. Important
services including spiritual counseling and family support are not currently on the agenda. We could do
better.

Clinical Crossroads is made possible by a grant from the Robert Wood Johnson Foundation. We thank the
patient, her family, and her doctor for sharing their stories in person and in print.

Dr Lynn wishes to acknowledge the assistance of Judith Hsia, MD, a cardiologist at George Washington
University.

This conference took place at the Medicine Grand Rounds of the Beth Israel Deaconess Medical Center,
East Campus, Boston, Mass, on January 23, 1997.

Reprints: Erin E. Hartman, MS, Division of General Medicine and Primary Care, Beth Israel Deaconess
Nedical Center, East Campus, 330 Brookline Ave, LY318, Boston, MA 02215.

REFERENCES

1. US Bureau of the Census. Statistical Abstract of the United States: 1995. 115th ed. Washington, DC: US Bureau of the Census;
1985:86 [Context Link]

2. Verbrugge LM. Aging and the quality of life. In: Abeles RP, Gift HC, Ory MG, eds. Disability in Late Life. New York, NY: Springer
Publishing Co; 1994. [Context Link]

3. Welch HG, Albertsen PC, Nease RF, et al. Estimating treatment benefits for the elderly: the effect of competing risks? Ann Intern
Med. 1996;124:577-584. Ovid Full Text Bibliographic Links Library Holdings [Context Link]

4. Braunwald E. Heart Disease: A Textbook of Cardiovascular Medicine. 4th ed. Philadelphia, Pa: WB Saunders Co; 1992. [Context
Link]

5. Knaus WA, Harrell FE Jr, Lynn J, et al. The SUPPORT prognostic model: objective estimates of survival for seriously ill hospitalized
patients. Ann Intern Med. 1995;122:191-203. Ovid Full Text Bibliographic Links Library Holdings [Context Link]

6. Morris JN, Suissa S, Sherwood S, et al. Last days: a study of the quality of life of terminally ill cancer patients. J Chronic Dis.
1986;39:47-62. Bibliographic Links Library Holdings [Context Link]

7. Lynn J, Harrell FE Jr, Cohn F, et al. Prognoses of seriously ill hospitalized patients on the days before death: implications for
patient care and public policy. New Horiz. 1997;5:56-61. [Context Link]

8. Hakim RB, Teno JM, Harrell FE Jr., et al. Factors associated with do-not-resuscitate orders: patients' preferences, prognoses, and
physicians' judgement. Ann Intern Med. 1996;125:284-293. Ovid Full Text Bibliographic Links Library Holdings [Context Link]

9. Hamel MB, Phillips RS, Teno JM, et al. Seriously ill hospitalized adults: do we spend less on older patients? J Am Geriatr Soc.
1996;44:1043-1048. Ovid Full Text Bibliographic Links Library Holdings [Context Link]

10. Gillick M. Choosing Medical Care in Old Age: What Kind, How Much, When to Stop. Cambridge, Mass: Harvard University Press;
1994. [Context Link]

11. Benson J, Cantor J, Lynn J, et al. A national perspective on dying in America: does place of death matter? Presented at the New
York Symposium on Health Services Research; New York, NY; December 10, 1996. [Context Link]

12. Addington-Hall J, McCarthy M. Dying from cancer: results of a national population-based investigation. Palliat Med. 1995;9:295-
305. Bibliographic Links Library Holdings [Context Link]

13. Quill TE, Cassel C, Meier DE. Assisted death and physician-assisted suicide. N Engl J Med. 1993;328:965-966. Bibliographic
Links Library Holdings [Context Link]

14. Lynn J, Teno JM, Phillips RS, et al. Perceptions by family members of the dying experience of older and seriously ill patients. Ann
Intern Med. 1997;126:97-106. Ovid Full Text Bibliographic Links Library Holdings [Context Link]

15. Chochinov HM, Wilson KG, Enns M, et al. Prevalence of depression in the terminally ill: effects of diagnostic criteria and symptom
threshold judgements. Am J Psychiatry. 1994;151:537-540. [Context Link]
16. Breitbart W. Suicide risk and pain in cancer and AIDS patients in current and emerging issues. In: Chapman CR, Foley KM, eds.
Current and Emerging Issues in Cancer Pain: Research and Practice. New York, NY: Raven Press; 1993:49-65. [Context Link]

17. Teno JM, Lynn J. Putting advance-care planning into action. J Clin Ethics. 1996;7:205-213. Bibliographic Links Library
Holdings [Context Link]

18. World Health Organization. Cancer Pain Relief and Palliative Care: Report of a WHO Expert Committee. Geneva, Switzerland:
World Health Organization; 1990. WHO Technical Report Series 809. [Context Link]

19. Wells HG. The Time Machine. New York, NY: Berkeley; 1963. [Context Link]

20. Byock I. Dying Well: The Prospect for Growth at the End of Life. New York, NY: Riverhead Books; 1997. [Context Link]

21. Dunn H. Hard Choices for Loving People. Herndon, Va: A&A Publishers, Inc; 1994. [Context Link]

22. Kubler-Ross E. On Death and Dying. New York, NY: Macmillan Publishing Co Inc; 1969. [Context Link]

23. Kubler-Ross E. Living With Death and Dying. New York, NY: Macmillan Publishing Co Inc; 1981. [Context Link]

24. Teno JM, Licks S, Lynn J, et al. Do advance directives provide instructions which direct care? J Am Geriatr Soc. 1997;45:508-
512. [Context Link]

25. Teno J, Lynn J, Wenger N, et al. Advance directives for seriously-ill hospitalized patients: effectiveness with the Patient Self-
determination Act and the SUPPORT intervention. J Am Geriatr Soc. 1997;45:500-507. [Context Link]

26. Miles SH, Crimmins TJ. Orders to limit emergency treatment for an ambulance service in a large metropolitan area. JAMA.
1985;254:525-527. Bibliographic Links Library Holdings [Context Link]

27. Miles SH. Advanced directives to limit treatment: the need for portability. J Am Geriatr Soc. 1987;35:74-76. Bibliographic
Links Library Holdings [Context Link]

28. New York State Task Force on Life and the Law. Life-Sustaining Treatment: Making Decisions and Appointing a Health Care
Agent. Albany: New York State Task Force on Life and the Law; July 1987. [Context Link]

29. AGS Ethics Committee. Measuring quality of care at the end of life: a statement of principles. J Am Geriatr Soc. 1997;45:526-
527. [Context Link]

30. Berwick DM. Quality comes home. Ann Intern Med. 1996;125:839-843. Ovid Full Text Bibliographic Links Library Holdings
[Context Link]

31. Von Roenn JH, Cleeland CS, Gonin R, et al. Physician attitudes and practice in cancer pain management-a survey from the
Eastern Cooperative Oncology Group. Ann Intern Med. 1993;119:121-126. Ovid Full Text Bibliographic Links Library Holdings
[Context Link]

32. Jones S. Why Not the Best for the Chronically Ill? Washington, DC: Health Insurance Reform Project, George Washington
University; January 1996. Research agenda brief. [Context Link]

33. Shute N. Death with more dignity. US News & World Report. February 24, 1997;122:61-62. [Context Link]

34. Lynn J, Cohn F, Pickering JH, et al. American Geriatrics Society on physician-assisted suicide: brief to the United States Supreme
Court. J Am Geriatr Soc. 1997;45:489-499. Ovid Full Text Bibliographic Links Library Holdings [Context Link]

CLINICAL CROSSROADS (Delbanco TL, Daley J, Hartman EE, Walzer J, Winker MA, eds); Death; Elderly; Heart Failure, Congestive;
Physician-Patient Relations; Terminal Care
Pediatric Non-
Non-Malignant Iron Deficiency During
Hematological conditions Childhood: Etiology
Part I & II Blood Loss
GI tract
Lakshmanan Krishnamurti, MD
Whole cow milk
Associate Professor Pediatrics Parasitic infections (esp. hookworm)
Division of Hematology/Oncology/BMT Esophageal varices (portal hypertension)
Childrens Hospital of Pittsburgh Other anatomic lesions (Meckels, polyp, duplication,
1) objectives of lecture ulcer)
To recognize clinical presentation and diagnosis of common pediatric non- Inflammatory bowel disease
malignant hematological conditions
To recognize the treatment, outcomes and long term effects of treatment of
Epistaxis
common pediatric non-malignant hematological Menstrual loss
conditions
Intrapulmonary or renal loss (rare)
2) key words : Thalassemia, sickle cell disease iron deficiency anemia,
autoimmune hemolytic anemia, thrombosis, Hemostasis, Thrombocytopenia Chronic intravascular hemolysis
Idiopathic pulmonary hemosiderosis

Iron Malabsorption Why Excessive Whole Cow Milk


An infrequent primary etiology of iron deficiency
Intake Predisposes to
during infancy and childhood Iron Deficiency During Infancy
Most common causes:
Celiac disease Contains minimal iron (<1 mg/L)
Medications (proton pump inhibitors, H2 Iron poorly absorbed (5-10%) compared to
receptor antagonists)
breast milk (50% absorption)
Chronic giardiasis
Isolated defect
Leads to reduced intake of other foods and
medicinal iron
Diagnosed by iron absorption test (serum iron
level before and 1 hour after 1 mg/kg oral iron) May cause GI bleeding

Iron Deficiency During Lead Poisoning


Adolescence
Blood loss
Basophilic stippling in erythrocytes
Menstrual
Elevated free erythrocyte
Gastrointestinal and renal in protoporphyrin
athletes, especially long-
distance runners
Rapid growth
H. pylori infection

1
Diagnosis of Iron Deficiency
History: children rarely symptomatic
Physical examination
Non-specific findings of anemia
Specific findings (koilonychia,
chlorosis, etc.) rarely seen in
children
Laboratory tests
5-5-97
From Dallman et al, Nathan & Oski, 4th Edition 1993

Laboratory Diagnosis of Iron


Deficiency Anemia During
Childhood
Result that is
suggestive or Measurement
diagnostic of iron deficiency
Serum ferritin < 12 g/L
Serum iron < 40 g/dL
Serum transferrin (TIBC) > 400 g/dL
Transferrin saturation < 10%
MCV < 70 fl
RDW > 16%

Monitoring, Duration and


Toxicity of
The
The Best Test For Iron Therapy
Iron Deficiency
Deficiency Monitoring after initiation of therapy:
1-2 weeks: In moderate or severe cases only;
document rise in reticulocyte
count and hemoglobin ( 1-2
gm/dl)
Complete response to a 4-6 weeks: In all patients; document nearly
complete correction of anemia
therapeutic trial of iron Duration:
3-4 months (at least 2 months after correction
of anemia)
Side Effects:
GI intolerance
Dark stools
Stained teeth

2
Folate and Cobalamin
Reasons for Lack of Response to
Deficiency
Oral Iron Therapy Clinical and Laboratory Findings
Parents not administering Extremely common Non-specific signs and symptoms of anemia
iron according to
instructions ( administering iron Jaundice due to ineffective erythropoiesis
with formula)
Diagnosis is incorrect; Common Macrocytic anemia
child doesnt have iron
deficiency Relatively low reticulocyte count
Dose of iron is incorrect Occasionally Hypersegmentation of neutrophils
Child is malabsorbing Rare Mild thrombocytopenia and/or neutropenia
iron
Megaloblastic changes in marrow
Neurological findings (B12 deficiency only): loss of
position sense, ataxia,5-5-97
psychomotor retardation,
seizures

Macrocytosis During Childhood Folic Acid Deficiency


Normal newborn infant Etiology
Reticulocytosis Decreased intake (rare due to supplementation
Marrow failure of packaged foods)
Drugs Severe malnutrition
Cyanotic congenital heart disease Sick premature infant
Down syndrome Unpasteurized goat milk
Hypothyroidism
Liver disease Intestinal malabsorption (celiac disease,
Megaloblastic anemia (B12, folate inflammatory bowel disease, anticonvulsants)
deficiency) Increased requirements
Chronic hemolytic anemia
Pediatrics 1992;89:1063-67 Pregnancy 5-5-97

3
Folic Acid Deficiency Vitamin B12 (Cobalamin)
Diagnosis and Management Present in meat and dairy products
Typical history Intrinsic factor (IF) required for absorption
Absence of neurologic manifestations of IF produced by gastric parietal cells
cobalamin deficiency
IF-B12 complex binds to receptors in terminal
Megaloblastic changes in CBC marrow ileum and absorbed
Reduced serum folate (reflects current status) Transported by plasma by transcobalamin (TC-I
and TC-II)
Reduced red cell folate (reflects tissue levels)
Transcobalamin II (TC-II) required for entry into
Complete response to therapy with physiologic cells
doses of folate (200-400 g/day)
5-5-97 Abundant B12 stores in liver and other tissues
5-5-97

Cobalamin Deficiency Congenital (Juvenile)


Etiology
Reduced B12 intake
Pernicious Anemia
Vegan (no meat or dairy products) Presents at 1-2 yr of age (hepatic stores derived
Breast-feeding infant of vegan from mother suffice until then)
Decreased intestinal B12 absorption
Reduced intrinsic factor Autosomal recessive
Adult-type pernicious anemia
Not associated with autoimmune disorders
Juvenile pernicious anemia
Malabsorption despite normal intrinsic factor Diagnosis:
Ileal resection
Inflammatory bowel disease Low serum cobalamin level
Imerslund-Grsbeck syndrome
Abnormal Schilling test, corrected by IF
Increased intestinal utilization
Blind loop or other intestinal
5-5-97
stasis Treatment: IM cobalamin
5-5-97
monthly
Fish tapeworm infection

Treatment of Vitamin B12


Imerslund -Grsbeck
Imerslund-Grsbeck Deficiency with High Dose Folic
Syndrome Acid
Malabsorption of cobalamin (abnormal
Schilling test, not corrected by IF) Megaloblastic anemia of B12 deficiency
responds to folic acid in high doses ( 1 mg/d)
Ileal receptor defect
Proteinuria Underlying B12 deficiency may be masked

Autosomal recessive inheritance Neurologic deficits of B12 deficiency do not


respond to folate
Presents in early infancy
Treatment: IM cobalamin Bottom line: Correct diagnosis extremely
5-5-97 important! 5-5-97

4
Hemolytic Anemias -
Anemia of Inflammation
Overview
Also called anemia of chronic disease
Membrane Disorders
A common cause of mild anemia in infants and
Enzyme Disorders
young children
Hemoglobin Disorders
Due primarily to decreased RBC production
(plus hemolytic component in some cases) Extrinsic Causes of Hemolysis

May follow mild acute (otitis media,


gastroenteritis, etc.) or more severe or chronic
infections
Often self-limited

Clinical Presentation of Extravascular vs Intravascular


Hemolytic Anemia Hemolysis
Intravascular Extravascular
Pallor Location of RBC Inside vessels In spleen and or
Icterus, jaundice Clearance liver (RES)
Antibody Type* IgM (occ. IgG) IgG
Fatigue
Splenomegaly Mechanism of Complement Macrophages
Hemolysis mediated digest RBCs
Gallstones
Lab Findings Hgbinemia  Bilirubin
Dark urine Hgbinuria  LDH
 Haptoglobin
Example PCH*, PNH AIHA*, HDN*, HS

Membrane disorders:Hereditary
Spherocytosis
Clinical Manifestions of HS
Most common cause of non-immune
hemolytic anemia Hemolytic anemia
Degree of anemia varies with different mutations
Autosomal dominant transmission 25% with compensated hemolysis and no anemia
25-30% sporadic mutations Pallor, fatigue
Loss of membrane surface area relative to Jaundice
intracellular volume  spheres and Neonatal jaundice in first 24 hours of life
decreased deformability Splenomegaly
Gallstones
Abnormalities of spectrin and/or ankyrin,
Positive family history
and less commonly Protein 4.2 or Band 3
May present with parvovirus associated aplasia

5
Laboratory Manifestations of HS Incubated Osmotic Fragility Testing

Spherocytes on Red cells are incubated in varying


peripheral blood smear concentrations of saline (0 0.9%)
for up to 48 hours
Reticulocytosis
As concentration of saline
Increased incubated decreases, cells take on water and
osmotic fragility Normal Osmotic Fragility are hemolyzed
Negative DAT Normal cells around 0.5%
Increased MCHC > 36% HS cells at higher NaCl
concentrations
due to relative cellular
dehydration Degree of hemolysis is detected
by spectrophotometry
Increased bilirubin, LDH
Not reliable < 6-12 months of age
Increased Sensitivity to Lysis

Enzyme disorders: G6PD Enzyme disorders: Pyruvate


Deficiency Kinase Deficiency
Most common red cell enzymopathy
X linked inheritance
Decreased production of NADPH with inability to maintain reduced Reduction in ATP production, with loss
glutathione levels
Hemolysis occurs in response to oxidative stresses such as
of membrane stability, water loss, cell
infections, drugs, fava beans (favism), naphthalene (moth balls) shrinkage
Common variant found in African Americans rarely associated with
sever hemolysis Increased shunting through Rapaport-
Anemia may be low grade and chronic (CNSHA) or acute after
exposure to oxidant Luebering shunt resulting in increased
Denatured hemoglobin seen as Heinz bodies on blood smear; also 2,3 DPG production
with blister cells on smear
Reticulocytes have 5X higher G6PD, so assay after resolution of This increased 2,3 DPG facilitates O2
hemolytic crisis
Avoid drugs such as co-trimoxazole, sulfonamides, nitrofurantoin, release from hemoglobin to the tissues
primaquine, dapsone and partially compensates for anemia

Pathophysiology
of SCD
Point mutation in
globin gene
Single amino acid
substitution in globin
Polymerization of
hemoglobin under
hypoxic conditions
Vaso-
Vaso-occlusion by
distorted RBCs
Steinberg,M.H. Bailliere
Baillieres Clin Hematol
. 11:163-
11:163-84, 1999

6
Vasoocclusion is a two step process
Interaction of sickle RBC with
endothelium
Activated
v3 Platelet
SO4Gly

Endothelial cell
Thrombospondin
CD36 CD36
SS
RBC
VCam-
VCam-1 41
Dense cell trapping and
retrograde blockage Fibronectin
Inflammatory Cytokines

Role Of NO In SCD Pathophysiology

Steinberg, M.H. BJH 129(4)465-


481.2005
Reiter & Gladwin. Curr Opinion Hematology 2003.10(2) 99-
99-107

Painful swelling
of hands and feet
6 months - 2
years
One of the first
manifestations

7
10% of children with SCD will get a stroke
Medical Emergency Caused by
sickling of red
Admit PICU, Exchange Transfusion cells in pulmonary
vasculature
Chronic transfusion to prevent recurrence
Major cause of
Transcranial Doppler ultrasound a screening death in childhood
tool for risk of stroke
Indistinguishable
from pneumonia
Simple/Exchange
transfusion

Sickle Cell Pain Crisis Sickle Cell Pain Crisis


Increased Sickling Vaso-
Vaso-occlusion Increased Sickling Increased Sickling Vaso-
Vaso-occlusion Increased Sickling

Hyperviscocity Hyperviscocity

Ischemic tissue injury regional hypoxia Ischemic tissue injury regional hypoxia
and acidosis and acidosis

Dehydration Dehydration

PAIN PAIN

Pain Management Distributions of Tricuspid Regurgitation


Velocity in Patients With Sickle Cell Disease
4.0

Goals
m/sec))

3.5
TR jet velocity ((m/sec

Assess the clinical problem


3.0
Treat the pain aggressively in a 33%

supportive environment 2.5

Make an appropriate diagnosis: Pain 2.0


67%

in a patient with sickle cell disease


does not equal pain due to sickle cell 1.5

1.0
Sickle cell African American
patients controls

8
Sickle Cell Pain Crisis
Impact of pulmonary
hypertension on survival Increased Vaso-
Vaso-
Adults with SCD Sickling occlusion Increased Sickling
1.0

TRV < 2.5 m/s


0.9

Fraction Hyperviscocity
0.8
survival
p < 0.001
0.7
Ischemic tissue injury regional hypoxia
0.6 TRV 2.5 - 2.9 m/s and acidosis
TRV 3 m/s
0.5
Relative Dehydration
0 10 20 30 40 50 60 Risk
Months 10 PAIN
Gladwin et al. N Engl J Med.
2004;350:886

Pain Management
Why transplant for sickle cell
disease?: it is a serious illness
Goals
Pain crises, stroke, chest syndrome
Assess the clinical problem pulmonary hypertension and organ
Treat the pain aggressively in a damage affect the quality and quantity of
supportive environment life.
Make an appropriate diagnosis: Pain Loss of 20-
20-30 years life expectancy
in a patient with sickle cell disease Morbidity and premature mortality in
does not equal pain due to sickle cell adulthood is unpredictable.
HCT is the only treatment that offers the
hope of cure

Indications for HCT in SCD Indications for HCT in SCD


Patients <16 years of age with symptomatic Stage I or II sickle lung disease,
SCD with organ damage with an HLA identical Sickle nephropathy
sibling donor with one or more of the following:
Stroke, CNS hemorrhage or a neurological Bilateral proliferative retinopathy and major
event, or abnormal cerebral MRI and impaired visual impairment in at least one eye,
neuropsychological testing, Osteonecrosis of multiple joints with
Acute chest syndrome with a history of recurrent documented destructive changes,
hospitalizations or exchange transfusions, Requirement for chronic transfusions but
Recurrent vaso-
vaso-occlusive pain 3 / year for 3 with RBC alloimmunization >2 antibodies
years or recurrent priapism, during long term transfusion therapy.

9
Matched-
Matched-related donor
Current Outcomes of HCT for SCD transplantation for sickle cell
disease: report from the CIBMTR
126 patients transplanted between 1989 and
2002.
Comprehensive data available on 67
indications stroke (38% ) and recurrent vaso-
occlusive crisis (37%).

Probability
median age at transplantation 10 years
67% of patients had received >10 red blood cell
transfusions prior to HCT.
27% had a poor performance score prior to
HCT.
M. C. Walters et al., Blood 95, 1918-24 (Mar 15, 2000).
Panepinto JA, Walters MC, Carreras J, et al. Br J Haematol. Apr 24 2007.
Panepinto et al, Br. J Hematol 2007

Limitations of current New Approaches to HCT for SCD


approaches New approaches to overcome current
Only 1 in 4 ( 25%) chance that a full sibling limitations
will be a HLA match Regimen
80% of otherwise eligible patients will not
have matched sibling donor
Donor
Adults and patients with organ damage Recipient
have not been considered for HCT Indications
Concerns about toxicity, risk of death,
infertility and CGVHD may be barriers to
acceptability

Geographic Distribution of Magnitude of the Problem


Thalassemia
100,000 live births/ year of patients with
Thalassemia in the world
10,000 -20,000 hydrops fetalis/ year

From: Weatherall: BMJ, Volume 314(7095).June 7, 1997.1675-


1678

10
Genetic Control of Hemoglobins


Portland 2 2
2 2

Hb Gower
2
Hb
Portlan
d
From: Weatherall: BMJ, Volume 314(7079).February 15, 1997.492-
1997.492-496

Genetic Control of Hemoglobins Genetic Control of Hemoglobins


Thalassemia

4
4 2 2E
Hb Barts Hb E
Hb H

From: Weatherall: BMJ, Volume 314(7079).February 15, 1997.492-


1997.492-496 From: Weatherall: BMJ, Volume 314(7079).February 15, 1997.492-
1997.492-496

THALASSEMIA:
PATHOPHYSIOLOGY
Thalassemia
chromosome 16
Decreased or absent production of one or
more globin chains

Normal /

Silent carrier /
Trait / or ( Low / MCV
)
Globin chain Imbalance
Hgb H disease / ( Low
MCV, Anemia
Ineffective erythropoiesis )
Hydrops Fetalis /
HbCS( chain termination mutant ). Unstable
mRNA

11
Pathophysiology of
Hydrops
-Thalassemia Fetalis due to
alpha
Thalassemia
Reported
primarily in
SEA
populations
5/1000
conceptions

Maternal complications from


Hydrops fetalis
Hydrops fetalis
Hypoxia onset 6 weeks of gestation
(onset in Rh incompatibility 18 weeks) Pre-
Pre-eclamptic toxemia 61%
Six reported survivors in the world Polyhydramnios 59%
One at Marshfield clinic Antepartum Hemorrhage 11%
Severe Neurologic Sequelae Malpresentation 37%
Limb reduction and hypoplasia of lung, Mean duration of gestation 31 weeks
thymus, adrenals and kidneys PPH, DIC, abruptio placentae,
retained placenta, renal failure,
pleural effusion

Mutations causing Thalassemia and Hgb E


- Thalassemia trait chromosome 11
-1 Thalassemia trait /- - ( in SEA ) G A
2
2 Thalassemia trait - / - (Med, Decreased or absent production of globin chains
Africa) 0 Thalassemia
/ / no synthesis of globin chains
/ / Thalassemia
+

/ / decreased synthesis of globin chains


/ /
/ / Hemoglobin E functions as a mild + Thal
/ /
Hgb EE (Homozygous HgbE) is mild
Hgb E/ 0 Thalassemia can be severe

12
Thalassemia Major: Thalassemia Major
Clinical Picture
Peripheral Smear

Cardiac and Hepatic Endocrine Complications


Complications in Thalassemia* in Thalassemia*

Thalassemia Intermedia: Thalassemia Minor:


Clinical Syndromes Clinical Syndromes

13
Management of Thalassemia
Red Cell Tranfusions
Mainstay of therapy for Thalassemia Major
Goal is to maintain Hg >9.5g/dl
Requires transfusions every 2 to 4 weeks
Advantages include:
Improved physical and psychologic well being
Decreased cardiomegaly
Decreased hepatosplenomegaly
Fewer bony changes and orthodontic problems
Normal or near normal growth until
adolescence
However, patients become severely iron
overloaded

1965-80 1980-2008 Progress in Thalassemia


Progress in
Thalassemia

2005 MRI
quantification of
iron overload
2005 Deferasirox
Chronic anemia
Transfusion keeping 1997 Hu,Butyrate
Admission for transfusion at
Hgb>10 1982 BMT
Hgb 4gm/dL
Regular chelation 1980 Hypertransfusion
Skeletal changes, growth
Normal growth,activity,
retardation, impaired sexual
school, sexual maturation
maturation 1979 Desferrioxamine
Improved quality of life
Poor quality of life Improved survival 1975 Genetic Counseling
Death in adolescence

HSCT For Thalassemia: Outcomes


HCT for Thalassemia
Pesaro Overall Thalassemia
Thalassemia major or severe forms of Risk Survival Free Survival
E/ or Thalassemia (four -globin
E/ group % %
gene deletion)
I 95 90
HSCT performed in > 1600 patients
The Pesaro risk stratification : II 87 84
hepatomegaly III 79 58
portal fibrosis Adults 66 62
adequacy of chelation

J. Gaziev, G. Lucarelli, Curr Opin Pediatr 15, 24 (Feb, 2003).

14
Clinically Important Hemoglobin
Unstable Hemoglobins
Mutants
II. Structural variants that cause a
Most are autosomal dominant mutations that alter the
Thalassemic phenotype (15) solubility of hemoglobin in the red cell
. Hb CS, G Philadelphia, Hasharon Most lead to alterations in tertiary or quaternary structure of
hemoglobin
. E, Very unstable HbTerre haute (a.k.a. Heat or isopropanol stability test are screening tests
Hb Indianapolis), Geneva, Showa- Heinz bodies present in RBCs with supravital stain and
Showa- lead to extravascular hemolysis
Yakushiji May see abnormal smeared band on electrophoresis
III. Unstable Hgb that cause CHBA (100) Examples include:
Hb Zurich with increased affinity for CO
Hb Koln, Zurich, Hammersmith, Gun Hill, Hb Kln
Hb Poole is an unstable gamma chain variant
Medicine Lake

Extrinsic and Acquired Causes of Neonatal Alloimmune Hemolytic Anemia


Hemolytic Anemia (Erythroblastosis Fetalis or HDN)
Immune mediated hemolysis
Transplacental passage of maternal alloantibody
Mechanical Destruction directed against fetal antigens, leading to
Microangiopathic Hemolytic Anemia (MAHA) hemolysis of fetal RBCs with resulting anemia,
Drug Induced Hemolysis hyperbilirubinemia, hydrops fetalis, kernicterus
Thermal Burns
May be due to Rh incompatibility, ABO
Toxins incompatibility, or other blood groups (Kell, Duffy,
Hypersplenism Lewis and others)
Complement Mediated Destruction Fetomaternal hemorrhage leads to maternal
Paroxysmal Nocturnal Hemoglobinuria immune response
May occur spontaneously or following amniocentesis,
trauma, abortions, external cephalic version

Rh Hemolytic Disease ABO Incompatibility


Isohemagglutinins are naturally occurring
Rh is the most immunogenic of blood groups antibodies
Hemolysis does not occur with first pregnancy Typically IgM, but only IgG can cross placenta
Alloimmunization does occur with first pregnancy Can occur in the first pregnancy
Infants Direct Antiglobulin Test (DAT) will be ABO Set up with Group O mom and Group A
positive or B infant (20% of pregnancies, but only 2%
Smear with NRBCs, polychromasia, but not affected by HDN)
usually spherocytes (RBCs dont exit spleen) DAT is usually positive (may be weak)
RhIgG is given to Rh- mothers to prevent Peripheral smear with polychromasia, NRBCs,
alloimmunization spherocytes
Given at 28 weeks, at delivery, and after any invasive
procedure (amniocentesis, chorionic villus sampling, ABO antigens not expressed in early fetal
version) RBCs, thus ABO HDN is not usually severe

15
Warm Reactive Autoimmune
Cold Agglutinin Disease
Hemolytic Anemia (AIHA)
IgG mediated IgM mediated
Extravascular clearance primarily via the IgM-RBC immune complex forms at 4C
reticuloendothelial system (spleen)
May be idiopathic or associated with SLE, Activates complement when warmed centrally
lymphoid malignancies, immunodeficiency Often react with I/i blood group system
Antibodies usually against common (Rh) Can be associated with Mycoplasma, EBV
antigens
DAT positive (IgG + C3) DAT + for C3, thus intravascular lysis
Treatment: Steroids, splenectomy, other Treatment: Keep patient warm, supportive
immunosuppressive drugs, + IVIG, transfusion therapy, plasmapheresis for severe
with least incompatible blood disease

Microangiopathic Hemolytic Platelet disorders: Idiopathic


Anemia (MAHA) Thrombocytopenic Purpura
The most common cause for acute onset of thrombocytopenia in an
Red cell fragmentation disorders otherwise well child
One to 4 wk after exposure to a common viral infection, a small number of
Shearing of red cells (schistocytes) children develop an autoantibody directed against the platelet surface.
Epstein-Barr (EBV) and HIV. EBV-related ITP and other viruses associated
May also see spherocytes on smear with ITP
May occur with vasculitic disorders, DIC, previously healthy 14 yr old child who has the sudden onset of
post stem cell transplantation, pregnancy,
generalized petechiae and purpura.
bleeding from the gums and mucous membrane, particularly with profound
drugs including cocaine, cyclosporine A, thrombocytopenia (platelet count < 10 109/L).
presence of abnormal findings such as hepatosplenomegaly or remarkable
FK506 (tacrolimus), and congenital heart lymphadenopathy suggests other diagnoses.
When the onset is insidious, especially in an adolescent, the possibility of
disease chronic ITP or that thrombocytopenia is a manifestation of a systemic illness
such as systemic lupus erythematosus (SLE) is more likely.
In children Hemolytic uremic syndrome is
more common than Thrombotic
thromobytopenic purpura

Clinical course of ITP Treatment options for ITP


spontaneous resolution of ITP will occur in 7080%, within 6 mo. 1. Intravenous immunoglobulin (IVIG). IVIG in a dose of
Therapy does not appear to affect the natural history of the illness. 0.81g/kg/day 12 days, induces a rapid rise in platelet
Less than 1% of cases develop intracranial hemorrhage. count (usually >20 109/L) in 95% of patients within 48hr.
objective of early therapy is to raise the platelet count to more than
20 109/L and prevent the rare development of intracranial IVIG therapy is both expensive and time-consuming to
hemorrhage. administer.
Ten to 20 percent of children who present with acute ITP go on to 2. Prednisone. of 14mg/kg/24hr of prednisone appear to
develop chronic ITP.
Severe thrombocytopenia (platelet count < 20 109/L) is common
induce a more rapid rise in platelet counts than in untreated
platelet size is normal or increased, reflective of increased platelet patients with ITP.
turnover. 3. IV Anti- D Therapy. RBC-antibody complexes bind to
hemoglobin value, white blood cell (WBC) count, and differential macrophage Fc receptors and interfere with platelet
count should be normal.
destruction. 8085% of patients receiving anti-D in a dose
bone marrow examination, when done, reveals normal granulocytic
and erythrocytic series with characteristically normal or increased of 50g/kg have demonstrated a rise in platelet count to
numbers of megakaryocytes. levels above 20 109/L within 2 days.

16
Neutropenia: General Classification Immune Neutropenias of Childhood
I. Marked decrease bone marrow reserve
A. Primary disorders: Kostmann syndrome, Shwachman- Marrow production normal to increased, storage pool
Diamond syndrome, cyclic neutropenia normal to decreased
B. Secondary: Chemotherapy, drug induced (non-
immune), nutritional, viral infection Increase in turnover of neutrophils, vascular
C. Part of a complex phenotype, e.g., Glycogenosis Ib,
Cartilage-hair hypoplasia compartment decreased
D. Other: e.g., idiopathic
Categories:
II. Normal marrow reserve - Alloimmune
A. Immune: Chronic benign neutropenia of childhood, - Chronic benign neutropenia of childhood
autoimmune, alloimmune, drug induced, infection - Autoimmune
B. Non-immune: Infection, hypersplenism, excessive
margination - Drug-induced

Chronic Benign Neutropenia of Autoimmune Neutropenia


Childhood
Mechanism: anti-neutrophil antibody; many have specificity for neutrophil Mechanism: antibody mediated; turnover. May be associated with other
specific antigens (NA series). Antibody against FCR III common. hematologic antibodies and/or immune deficiency/autoimmune
Clinical features: common syndrome (~ 1/100,000) disorders.
- Median age diagnosis 8-11 months (range 3-38) Clinical features:
- Usually no significant predisposition to infection, occasional patient - May also find ITP, AIHA, other hematologic antibodies
will have significant or severe infection - SLE, immunodeficiency states (dysgammablobulinemia,
- Most detected before 14 months of age hypogammaglobulinemia, HIV, etc.), Feltys syndrome, Hashimoto or
- Median length illness 20 mos (range 6-54) Graves diseases. Chronic active hepatitis may have associated
- Counts usually <500, spontaneous remission autoimmune neutropenia.
- Antibody test usually positive - Variable ANC, marrow shows normal cellularity, late maturation arrest
- Marrow shows normal to increased myeloid series with a decrease at Management:
the band and/or seg level. Some show earlier arrest. - Treat primary autoimmune disorder and/or hematologic antibodies
Management: supportive care; risk for infection not great. Infections - G-CSF may be helpful if marrow storage pool depleted
treated with antibiotics. G-CSF may be required in a few with
infection.

Alloimmune Neutropenia Congenital marrow failure


Mechanism: maternal alloimmunization to neutrophil-specific antigens (NA),
transplacental passage and binding to neonatal neutrophils.
syndromes: Fanconi Anemia
Clinical features:
Autosomal recessive
- Neutropenia usually for 2-4 weeks, occasionally up to 3-4 mos. Can be
profound. Genetically and phenotypically
- Affected patients may be asymptomatic or may develop skin infections, heterogeneous, with the common
pneumonia, sepsis or meningitis
- Commonly confused with sepsis associated neutropenia
features of
- Marrow shows myeloid hyperplasia with maturation arrest at mature In vitro and in vivo sensitivity to DNA cross-
precursors linking agents
Management:
Congenital malformations
- Antibiotics and supportive care for infections
- IVIG infusion not always effective Progressive BM failure
- Consider G-CSF in face of severe infection Predisposition to AML

17
Transplantation for Fanconi's anaemia: long-term follow-up of fifty patients transplanted from a
sibling donor after low-dose cyclophosphamide and thoraco-abdominal irradiation for conditioning
(used on Patient #1)
Chromosomal rearrangements reflect a loss
of fidelity in repairing double strand DNA
breaks. These lesions are corrected
normally by 2 primary pathways:

NHEJ (non homologous end


joining)
HRR (homologous recombinational
repair)

Absence of either pathway results in


genomic instability and increased
radiosensitivity

Congenital marrow failure


syndromes: Diamond Blackfan Diamond Blackfan Anemia
Anemia
Autosomal macrocytic anemia
recessive reticulocytopenia
20-25% have a normal bone marrow cellularity with
mutation in RPS19 decreased red cell precursors
on chromosome 19 normal or slightly decreased WBC
Short stature normal or slightly increased platelet
Congenital count
anomalies

18
Diamond Blackfan Anemia Aplastic anemia: Definition
Pancytopenia
Corticosteroids and red blood cell Anemia
Neutropenia
transfusions are the mainstays of Thrombocytopenia

therapy. Reticulocyto-penia
Aplastic bone marrow
82% initially responsive to steroids Hypocellular with all
elements down; mostly fat
36% of patients need chronic red cell and stroma
transfusions Residual hematopoietic
cells are normal
BMT from a matched donor is curative No malignancy or fibrosis
No megaloblastic
Increased risk of solid tumors and hematopoiesis
leukemia

Hepatitis Associated Aplastic


Severity of Aplastic Anemia
Anemia
Severe Aplastic Anemia (SAA) Typically in boys and young men
Marrow of less than 25% normal cellularity OR Severe aplasia occurs 2-3 months after acute
marrow <50% normal cellularity with fewer than 30% hepatitis
of the cells being hematopoietic
2-5% of aplastic anemias in West have h/o hepatitis
2 of 3 abnormal peripheral blood values
4-10% of aplastic anemias in Far East have h/o
Absolute reticulocyte count < 40,000
hepatitis
ANC<500
Platelets <20,000 BM failure can be precipitous and fatal
Very Severe Aplastic Anemia (VSAA) Etiology of hepatitis is not obvious
Above with ANC <200 Non-A, non-B, non-C
High incidence after OLT for fulminant non-A, non-B
hepatitis

Definitive therapy:
Supportive Treatment for AA
Immunosuppression
Prophylactic platelet transfusion-<10,000 Immunosuppression is NOT curative
or <20,000 and fever1 Goal is sustained remission
20-36% have recurrent aplastic anemia
Fe chelation 20-36% develop clonal disorder, PNH, MDS or acute
Epogen not effective or safe2 leukemia
Combination therapy is best
Transfusions should be CMV-, Antithymocyte globulin (ATG)
leukoreduced, irradiated, no family donors Toxic side effect is serum sickness, tx with steroid
Can lower platelet counts, transfuse prn
Cyclosporine
1.Consenus conference on platelet transfusion 1997
High dose corticosteroids
2. Guidelines for diagnosis and management of acquired aplastic anemia, Br J of
Hematology, 2003

19
Bleeding infant with prolonged
Definitive Therapy: BMT APTT without known bleeding
Therapy choice influenced by age and disease
disorder
severity Bleeding after circumcision or spontaneous
<20 years old mucosal bleed( only 25% of hemophiliacs
Allogeneic BMT if matched sib available
bleed after circumcision)
50-80% cure rate, with low incidence clonal disorders
Condition pre-transplant with ATG/cyclophosphomide Obtain detailed HPI family history
Consider unrelated donor, but survival only half matched sib
20-45 years old Prolonged APTT: Differential diagnosis VIII,
Allogeneic BMT if in excellent health w/fully matched sib IX, XI, deficiency or VWD type III
>45 years old Draw blood for diagnostic studies
?Immunosuppression only
?BMT with conditioning before BMT showing increased Recombinant Factor VIII concentrate
survival Immediate cessation of bleeding

The Hematologic Focused Bleeding Symptoms:


History Factor Deficiency vs
Easy bruising, petechiae
Platelet/Vessel
Nose/gum/mouth bleeding( commonest cause
of nose bleed: inquisitive fingers) Factor Deficiency
Abnormal nose bleed is one that lasts.15-30 Platelet/Vessel
minutes, needs visit to ER or nasal cautery
Menorrhagia/bleeding post childbirth - Delayed - Immediate
Bleeding after tonsillectomy, wisdom teeth
extraction - Deep - Superficial
Easy bruisability - Joint/muscle - Mucosal
Hemarthroses/ muscle bleeding
GI/GU bleeding

Congenital Thrombophilia
Prevalence in Patients
with Thrombosis
Deficiency of anticoagulants
AT-III, protein C or S ATIII Deficiency 1- 8%
Resistance to cofactor proteolysis
Factor VLeiden* Protein C Deficiency 1- 8%
High levels of procoagulants Protein S Deficiency 1- 5%
Prothrombin 20210 mutation * APC Resistance 15 - 65%
Damage to endothelium
Homocysteinemia

* Common

20
Thrombophilia
Acquired
zLines, Lines, Lines( most common cause of deep
vein thromobsis) z Malignancy
zSurgery z PNH
zImmobility z Myeloproliferative
zInfection/Inflammation syndrome
zPregnancy z Antiphospholipid
zOCP and the Patch Antibody
zVasculitis z Nephrotic

zIBD
Syndrome

21
Path: K:/AAP-PPC-06-1201/Application/AAP-PPC-06-1201-278-PT-9-Five.3d
Date: 26th August 2008 Time: 19:32 User ID: abhaskar 1BlackLining
Enabled

CHAPTER 287  Iron-Deficiency Anemia 2201

as they syndrome of androgen resistance, the testis cases, this process necessitates transfer of the
may instead be brought into the scrotum where it patient to a medical center with a neonatal inten-
can be observed for the development of malignant sive care unit and the appropriate pediatric sub-
features. specialists. The patient should never be sent home
Long-term psychological support is required for the and referred to subspecialty care as an outpatient.
majority of patients with DSD. Help should be pro- Indications for referral include the following:
vided to the patient by a mental health professional, Male infant born at term:
preferably someone with experience in dealing with  Bilateral nonpalpable testes
DSD. National and local support groups are also avail-  Micropenis
able for families and may be beneficial.  Perineal hypospadias
Gender identity is the personal conception of one-  Single undescended testes with hypospadias of
self as male or female and is the result of a complex any degree
interplay between both biological and environmental Female infant:
factors. It is distinct from sexual orientation and gen-  Clitoral hypertrophy of any degree
der role. Hormonal effects (specifically androgen  Posterior labial fusion (not adhesion)
effect on the prenatal brain), brain structural differen-  Inguinal or labial mass
ces, assigned sex of rearing, and sex-steroid effects at
the time of puberty have all been shown to influence AAP POLICY STATEMENT
gender identity. Thus some patients who are assigned
Lee PA, Houk CP, Ahmed SF, et al, in collaboration with the
a particular gender are later dissatisfied. Some adults participants in the International Consensus Conference on
with this history have influenced medical practices Intersex organized by the Lawson Wilkins Pediatric Endo-
surrounding gender assignment, particularly with crine Society and the European Society for Paediatric
respect to early surgery before the patient acquires Endocrinology. Consensus statement on management of
the ability to consent to the procedure. intersex disorders. Pediatrics. 2006;118(2):488-500. AAP
Unfortunately, outcome data for gender identity in endorsed.
DSD are relatively sparse. For example, the largest
study to date showed that genetic males with active SUGGESTED RESOURCES
prenatal androgen effects should be raised as males, Achermann JC, Hughes IA. Disorders of Sex Development.
given their high rates of male gender identity. Addi- In: Kronenberg HM, Melmed S, Polonsky KS, Larsen PR,
tionally, female infants virilized as a result of CAH eds. Williams Textbook of Endocrinology. Philadelphia,
should be given a female gender assignment because PA: WB Saunders; 2008.
more than 90% of these patients are content with an American Academy of Pediatrics, Committee on Genetics.
assigned female gender. Gender identity outcomes are Evaluation of the newborn with developmental anomalies
more difficult to predict in other disorders. of the external genitalia. Pediatrics. 2000;106(1 pt 1):138-
142.
CONCLUSION Conte FA, Grumbach, MM. Abnormalities of sexual determi-
nation and differentiation. In: Gardner DG, Shoback D,
The differential diagnosis of the infant born with
eds. Greenspans Basic and Clinical Endocrinology. 8th ed.
developmental anomalies of the external genitalia is New York, NY: McGraw-Hill Medical; 2007.
extensive, given the complicated process of human Lee PA, Houk CP, Ahmed SF, et al. Consensus statement on
sexual differentiation. The birth of such patients natu- management of intersex disorders. Presented at the Inter-
rally creates a stressful situation for the family of the national Consensus Conference on Intersex. Pediatrics.
patient, which can be attenuated by the appropriate 2006;118(2):e488-e500.
evaluation and management by an experienced multi- Reiner WG. Gender identity and sex-of-rearing in children
disciplinary team. Because fatal results may occur with with disorders of sexual differentiation. J Pediatr Endocri-
improper vigilance, hospitalization and close monitor- nol. 2005;18(6):549-553.
Styne DM. Sexual differentiation. In: Styne DM, ed. Pediatric
ing of the infant is indicated in all cases. Given that a
Endocrinology (Core Handbook Series in Pediatrics). Phila-
patients gender identity cannot be predicted based delphia, PA: Lippincott Williams & Wilkins; 2003.
solely on the phenotype or the biological defect, gen-
der assignment in these infants requires several con-
siderations. In some cases, the assigned gender may
not be concordant with the patients gender identity as
the child ages. Throughout this process the general Chapter 287
pediatrician plays an important role in helping coordi-
nate the care of these patients. IRON-DEFICIENCY ANEMIA

WHEN TO REFER Lakshmanan Krishnamurti, MD

Given the possibility of significant electrolyte dis-


turbances, cortisol insufficiency, and possible Iron-deficiency anemia (IDA) is the most common
shock and death, all patients with ambiguity of nutritional deficiency worldwide. In the United States,
the external genitalia should be evaluated immedi- IDA is most common among women and young chil-
ately by an experienced multidisciplinary team dren. Iron is an important ingredient of hemoglo-
that includes a pediatric endocrinologist. In many bin and is involved in numerous cellular processes.
Path: K:/AAP-PPC-06-1201/Application/AAP-PPC-06-1201-278-PT-9-Five.3d
Date: 26th August 2008 Time: 19:32 User ID: abhaskar 1BlackLining
Enabled

2202 PART 9: SPECIFIC CLINICAL PROBLEMS

Children with iron deficiency in infancy continue to ETIOLOGY AND PATHOPHYSIOLOGY


have poor cognition and school achievement and The 4 most important factors in the development of
increased behavior problems into middle childhood.1 iron deficiency in children are (1) the iron endowment
Iron deficiency is also associated with deficits in work at birth, (2) the iron needs during rapid body growth,
productivity, and severe anemia is associated with (3) exogenous iron absorption, and (4) blood loss. The
maternal and child mortality.2 Prevention of iron defi- causes of iron deficiency are summarized in Table 287-1.
ciency is therefore an important public health issue. During gestation, the level of fetal iron stores is
related to maternal iron status, and the maternal-fetal
unit is dependent on exogenous iron.6 The ratio of iron
IRON-CONTAINING COMPOUNDS content to weight in the human fetus remains constant
IN THE BODY throughout gestation. The healthy full-term newborn
Iron is the most abundant heavy metal in the body. has sufficient iron stores to last for 6 months, if suffi-
The multiple iron-containing compounds found within cient small amounts of iron are ingested from the diet.
the body can be grouped into two major categories: The infants iron endowment can be compromised by
(1) those serving metabolic functions and (2) those blood loss during the pregnancy or the perinatal
involved with iron storage and transport. period. Common causes of blood loss include third-
The first category includes heme- and nonheme- trimester bleeding, such as abruptio placentae, placenta
containing compounds. Heme is composed of a proto- previa, fetomaternal hemorrhage, and twin-to-twin
porphyrin ring with noncovalently bound iron in the transfusions.
ferrous form (Fe++). The most abundant heme-contain- Gestational conditions that result in lower newborn
ing protein in the body is hemoglobin, which trans- iron stores include severe maternal iron deficiency,
ports oxygen from the lungs to the tissues and maternal hypertension with intrauterine growth retar-
accounts for more than 60% of total body iron. Myo- dation, and maternal diabetes mellitus. Stable, very
globin, which accounts for 10% of total body iron, is a lowbirth-weight premature infants are also at risk for
heme protein that provides oxygen for use during early postnatal iron deficiency because they accrete
muscle contraction. The other major heme proteins, less iron during gestation, grow more rapidly after
the cytochromes, are found in the mitochondria and
are necessary for the oxidative production of cellular
energy. Several nonheme iron proteins are also Table 287-1 Causes of Iron Deficiency
present, such as the iron-sulfur complexes and fla-
voproteins. Many of these proteins are found in
INCREASED
the mitochondria and are also involved in oxidative
metabolism. IRON INADEQUATE
The second category of iron compounds includes REQUIREMENTS IRON ABSORPTION
molecules involved in iron transport and storage.
Transferrin is a b1-globulin capable of binding 2 Blood loss Diet low in bioavailable
Menstruation iron
atoms of iron in the ferric form. It transports iron
Gastrointestinal tract Formula not fortified
from the intestinal epithelium to the bone marrow,
Milk enteropathy with iron
where it binds to transferrin receptors on the surface
Food sensitivity Cows milk before the
of differentiating red blood cells (RBCs). The trans- Inflammatory bowel age of 6 mo
ferrin-receptor complex is then internalized and thus disease Strict vegetarian diet
supplies iron for the synthesis of hemoglobin. Trans- Meckel diverticulum Poor dietary habits in
ferrin also plays a major role in the recycling of iron Peptic ulcer disease adolescents
from senescent RBCs. Ferritin, an iron storage com- Reflux esophagitis Impaired absorption
pound found in all cells of the body, is composed of Hookworms Intestinal
a hollow protein shell encapsulating iron molecules. Malignancy malabsorption
Hemosiderin, which also serves to store intracellular Genitourinary tract Gastric surgery
iron, is thought to be a partially degraded form of Respiratory tract Hypochlorhydria
ferritin. Idiopathic pulmonary
Hemosiderosis
Cystic fibrosis
INCIDENCE Pulmonary tuberculosis
Iron deficiency is common, despite the generally good Cardiac
standard of nutrition and the widespread use of iron- Hemosiderinuria due to
fortified foods. The prevalence of iron deficiency is cardiac hemolysis
greatest among toddlers ages 1 to 2 years (7%) and Blood donation
adolescent girls and adult women ages 12 to 49 years Pregnancy
(9%-16%). The prevalence of iron deficiency is 2 times Growth
higher among non-Hispanic black and Mexican- Prematurity
American women (19%-22%) than among non-Hispanic Infancy
Adolescence
white women (10%).3 Currently, prevalence of IDA
remains higher than the goal of the national health
Modified from: Centers for Disease Control and Prevention. Recommenda-
objectives for 2010 to reduce iron deficiency in vulnera- tions to prevent and control iron deficiency in the United States. MMWR
ble populations by 3 to 4 percentage points.3-5 Recomm Rep. 1998;47(RR-3):1-29.
Path: K:/AAP-PPC-06-1201/Application/AAP-PPC-06-1201-278-PT-9-Five.3d
Date: 26th August 2008 Time: 19:32 User ID: abhaskar 1BlackLining
Enabled

CHAPTER 287  Iron-Deficiency Anemia 2203

birth, are typically undertreated with enteral iron, and the first hematologic abnormalities; at the second
receive fewer RBC transfusions than term infants.7 stage, the mean corpuscular volume (MCV) and mean
Iron is needed not only for many metabolic func- corpuscular hemoglobin (MCH) decline; and the final
tions and tissue replacement, but also for growth. stage of iron deficiency is associated with a low MCH
Growth rates vary with age and are maximal during count, a hemoglobin concentration below 9 g/dL, and
infancy and adolescence, the same periods associated a transferrin saturation of less than 16%.9
with the highest frequency of iron deficiency. Functional iron deficiency is a condition that occurs
Iron balance is maintained by regulation of iron primarily as a result of treatment with recombinant
absorption. The amount of iron absorbed depends human erythropoietin. During the supraphysiologic
both on the amount and bioavailability of dietary iron burst of RBC production after a pharmacologic dose
and on regulation of iron absorption by the intestinal of recombinant human erythropoietin, the small circu-
mucosa. Most dietary iron occurs in the nonheme lating iron pool (0.1% of total body iron) may be insuf-
form and is much less bioavailable than that in heme ficient to supply the stimulated erythron. Thus, even
proteins. The iron in hemoglobin and myoglobin is with normal iron stores, IDE may occur, with further
particularly bioavailable; up to 30% is directly iron repletion often required for normal erythropoie-
absorbed by the gastrointestinal tract. Human milk sis to resume.10 Inflammation may also be a contribu-
and cows milk contain small amounts of iron (0.5- tory factor.11
1 mg/1000 mL). However, 50% of the iron in human Specific laboratory findings are associated with
milk is absorbed, compared with only 10% in cows each of the 3 stages of iron deficiency. The laboratory
milk. Full-term infants who are exclusively breastfed test findings characteristic of each stage are summar-
for the first 6 to 9 months do not become iron defi- ized in Table 287-2.
cient.8 Nonheme iron absorption is inhibited by bran
in cereals, polyphenols in many vegetables, and tan- DIFFERENTIAL DIAGNOSIS
nins in tea. The addition of solids to an infants diet can Diagnosis of iron deficiency is made by the combina-
greatly impair iron absorption and puts the infant at tion of RBC indices and serum transferrin saturation
risk for developing iron deficiency. The introduced sol- or ferritin. Transferrin saturation and ferritin may be
ids should therefore contain abundant amounts of iron altered by infection, inflammation, malignancy, and
(eg, iron-fortified cereals). starvation. Serum transferrin receptor levels help dis-
Blood loss causes iron deficiency in children less criminate iron deficiency from the anemias of chronic
frequently than in adults. In infancy and childhood, disease.
iron deficiency caused by blood loss is most commonly Although the absence of iron stores in the bone
associated with the ingestion of unprocessed cows marrow remains the gold standard for making the
milk and with parasitic infections. Hypersensitivity to diagnosis of iron deficiency, this test is rarely per-
whole cows milk causes an exudative enteropathy and formed for this purpose because of the obvious dis-
frequently leads to gastrointestinal blood loss (Fig- comfort involved and the difficulty of standardizing
ure 287-1). Other less-common causes of blood loss in bone marrow iron stain.
children include Meckels diverticulum, intestinal Once the diagnosis of IDA is made, efforts should
duplication, peptic ulcer disease, hemorrhagic telan- be undertaken to establish the cause of the deficiency
giectasia, and the chronic use of medications that pro- (see Table 287-1). In infancy (a period during which
long the bleeding time (eg, aspirin). iron demands resulting from rapid growth may out-
strip the supply of iron), in adolescence, and during
STAGES OF IRON DEFICIENCY pregnancy, iron deficiency is the result of a physiologi-
Iron deficiency occurs when total body iron content is cal increase in iron requirement, which is not being
diminished. When absorption exceeds losses, the iron met by the oral supply of iron. Beyond infancy, blood
surplus is stored in the reticuloendothelial system, loss is the most common cause of IDA.
principally the liver, spleen, and bone marrow. Iron is Iron deficiency must be distinguished from other
removed from the reticuloendothelial storage pool to hypochromic microcytic anemias (Figure 287-2). RBC
compensate for negative iron balance. The develop- indices in infancy and childhood are described in
ment of iron deficiency proceeds through a series of Table 287-3.
overlapping stages.
The first stage of iron deficiency is iron storage Thalassemia Trait
depletion. During this stage, no deficit of iron supplied IDA and thalassemia trait are the most common causes
to the erythroid marrow for RBC production occurs. If of mild microcytic anemia with hemoglobin level of
the negative iron balance continues, then the second 9 g/dL or more. RBC count is often increased above
stage, iron-deficient erythropoiesis (IDE), will occur. normal despite the presence of a mild anemia and
During this stage, erythroid iron supply is diminished, microcytosis in thalassemia trait, whereas RBC count is
but the hemoglobin concentration remains in the nor- reduced in IDA. The RBC distribution width (RDW) is
mal range. If the negative iron balance persists, then increased in iron deficiency. The Mentzer index, defined
IDA finally develops. This third stage is characterized as the MCV divided by the RBC count in millions, can
by a decrease in the hemoglobin concentration and a help distinguish the anemia of iron deficiency from that
reduction in RBC size and hemoglobin content. Hema- of b-thalassemia trait.12 In IDA the Mentzer index is
tologic abnormalities in iron deficiency progress as often greater than 13.5; in b-thalassemia trait, it is less
impairment of hematopoiesis9 progresses. Anisocyto- than 11.5 with 82% specificity. An RDW index (RDWI),
sis and an increased percentage of microcytic cells are which is calculated by the formula RDWI = (MCV
Path: K:/AAP-PPC-06-1201/Application/AAP-PPC-06-1201-278-PT-9-Five.3d
Date: 26th August 2008 Time: 19:32 User ID: abhaskar 1BlackLining
Enabled

2204 PART 9: SPECIFIC CLINICAL PROBLEMS

Child with iron deficiency


anemia or fecal occult
blood (beyond infancy)

Milk enteropathy?

Yes No

Symptomatic Asymptomatic

Eliminate milk and soy Site-Specific Colonoscopy


milk from diet. Investigation If colonscopy is negative,
Introduce ironcontaining EGD and perform EGD
solid food. Colonoscopy
Treat with oral iron.

Negative and Negative


symptomatic Negative

Consider small intestine Trial of iron therapy


work-up
No response
Negative and asymptomatic

Re-evaluate cause
of anemia

Other Tests
Computed tomography
Meckel scan
Angiogram

Figure 287-1 Algorithm for approach to iron deficiency Disease. 7th ed. Copyright ' 2002 Saunders, An Imprint of
with occult intestinal blood loss. (Modified from Feldman Elsevier.)
M. Sleisenger & Fordtrans Gastrointestinal and Liver EDG, esophagogastroduodenoscopy.

RBC  RDW), of at least 220 is indicative of IDA, of a-thalassemia trait can be assumed when a patient
whereas an index of less than 220 is indicative of thalas- with a familial hypochromic microcytic anemia has
semia trait with a specificity of 92%.13 RBC count and normal results of iron studies (including ferritin), nor-
RDWI are the most reliable discrimination indices in mal levels of hemoglobin (Hb) A2 and Hb F, and a nor-
differentiation between b-thalassemia trait and IDA.14 mal hemoglobin electrophoresis. It is a diagnosis of
If a- and b-thalassemia trait or hemoglobin E dis- exclusion except in the newborn period, when infants
ease is suspected, then the diagnosis can be estab- with a-thalassemia trait have 3% to 10% Hb Barts (g4),
lished by review of the newborn screen or by which may be detected in the newborn screen.
obtaining a hemoglobin electrophoresis (only in cases
of b-thalassemia and not a-thalassemia). Because tha- Hb H Disease
lassemia trait is frequently not associated with hemo- Hb H disease, another form of a-thalassemia, results
globin of less than 9 g/dL, it is not included in the from deletion of three of the four a-globin genes. It also
differential diagnosis in severe anemia. The diagnosis is characterized by hypochromia and microcytosis, but
Path: K:/AAP-PPC-06-1201/Application/AAP-PPC-06-1201-278-PT-9-Five.3d
Date: 26th August 2008 Time: 19:32 User ID: abhaskar 1BlackLining
Enabled

CHAPTER 287  Iron-Deficiency Anemia 2205

Lead Poisoning and Anemia


Table 287-2 Laboratory Abnormalities Lead poisoning and IDA are both associated with high
in the Three Stages levels of erythrocyte zinc protoporphyrins (ZnPP). Iron
of Iron Deficiency deficiency and lead poisoning frequently coexist.
Although the nature of their relationship is not com-
STAGE DESCRIPTION pletely elucidated, characterization of a common iron-
lead transporter and epidemiologic studies among
IIron depletion Serum ferritin children strongly suggest that iron deficiency may
Bone marrow iron increase susceptibility to lead poisoning.15 In cases of
IIIron-deficient Serum ferritin lead poisoning associated with iron deficiency, the
erythropoiesis Bone marrow iron RBCs are morphologically similar, but coarse baso-
Serum iron philic stippling of the RBCs is frequently prominent.
TIBC Increases in blood lead, ZnPP/heme (ZnPP/H) ratio,
IIIIron-deficiency Serum ferritin and urinary coproporphyrin levels are seen.
anemia Bone marrow iron
Serum iron
TIBC EVALUATION
Hemoglobin History
Hematocrit
MCV The onset and progression of iron deficiency is usually
RDW gradual, and most children will not have major symp-
toms. Iron deficiency in infants and children is associ-
indicates increased; , decreased. ated with generalized weakness, irritability, easy
MCV, Mean corpuscular volume; RDW, red blood cell distribution width; fatigability, headaches, poor feeding, anorexia, pica,
TIBC, total iron-binding capacity. and poor weight gain.
From Roper D, Stein S, Payne M, Coleman M. Anemias caused by impaired
production of erythrocytes. In Rodak BF, ed. Diagnostic Hematology. Phila-
delphia, PA: WB Saunders; 1995. Reprinted by permission. Physical Examination
The physical examination is usually unremarkable
in addition, a mild hemolytic component is present except for marked pallor of the mucous membranes
from instability of the b-chain tetramers (Hb H) result- and skin. Other physical findings associated with IDA
ing from a deficiency of a-globin chains. Beyond but that are rarely observed include mild hepatosple-
infancy, Hb H is readily identified by hemoglobin elec- nomegaly, lymphadenopathy, glossitis, stomatitis, blue
trophoresis. During the newborn period, the moder- sclerae, and koilonychia (spoon-shaped nails).
ately severe a-globin deficiency allows for the
accumulation of more g chains, and the concentration Laboratory Evaluation
of Hb Barts is over 20%. Laboratory tests for identifying iron deficiency include
screening and definitive tests. Screening tests identify
Anemia of Chronic Disease and Inflammation IDE by demonstrating either a reduced supply of
Inflammation impairs the supply of iron to the plasma plasma iron or poor hemoglobinization of circulating
and ultimately results in a form of anemia called ane- RBCs. Definitive tests identify IDA by measuring iron-
mia of chronic disease or anemia of inflammation. It related proteins derived from either the iron storage
is usually normocytic, although it may occasionally compartment in macrophages or the iron utilization
be slightly microcytic. Because inflammation alters compartment in RBC precursors.
screening tests for iron status in the same manner as
true iron deficiency, the distinction between anemia of Screening Measurements
chronic disease and IDA requires tissue-related iron Hemoglobin Concentration
measurements. However, the usefulness of the serum Hemoglobin concentration is the most commonly used
ferritin in the diagnosis of IDA is compromised by the screening test for iron deficiency. Someone with nor-
effect of inflammation on the value. Consequently, a mal body iron stores must lose a large portion of body
serum ferritin concentration above 30 mg/L in an ane- iron before the hemoglobin falls below the laboratory
mic patient does not exclude IDA in the presence of definitions of anemia. Furthermore, low hemoglobin
chronic inflammation. does not distinguish among the causes of anemia other
C-reactive protein is generally considered to be the than iron deficiency, and additional testing is required.
best laboratory marker of inflammation. If it is less
than 30 mg/L, then inflammation is generally consid- Serum Iron and Transferrin Saturation
ered unlikely to be sufficient to raise the serum ferritin. Serum iron levels normally fluctuate daily, with maxi-
A suggested ferritin level of less than 40 mg/L and less mal levels occurring in the morning and minimal levels
than 70 mg/L are used to diagnose IDA in anemic in the evening. The total iron-binding capacity (TIBC)
patients without and with inflammation, respectively. varies less than serum iron but is harder to measure
The serum transferrin receptor (STfR) measurement accurately. The normal TIBC is 250 to 400 mg/dL, but
for identifying IDA is one in which the concentration as serum iron levels decrease, the TIBC increases to
is not affected by inflammation. Use of the STfR/ferri- 450 mg/dL or more. Iron and TIBC measurements are
tin ratio further improves the specificity of the diagno- useful in distinguishing IDA from anemia of chronic
sis of IDA and may eliminate the need for bone disease. Serum iron levels decrease with both, but the
marrow examination for assessment of iron stores. TIBC levels also decrease in chronic disease states
Path: K:/AAP-PPC-06-1201/Application/AAP-PPC-06-1201-278-PT-9-Five.3d
Date: 26th August 2008 Time: 19:33 User ID: abhaskar 1BlackLining
Enabled

2206 PART 9: SPECIFIC CLINICAL PROBLEMS

Microcytic hypochromic anemia without elevated reticulocyte count

RBC Normal/Increased RBC Normal/Decreased


RDW Normal RDW Increased

FEP

Abnormal Markedly Elevated


newborn elevated
screen
Hemoglobin
electrophoresis
Otherwise Inflammation or
High levels chronic disease
healthy patient in
of blood lead
high risk group

C-RP 30%
Therapeutic
Thalassemia trait Lead poisoning trial of iron

Ferritin 30% Ferritin 70%

sTfR

Elevated Normal

Anemia of
Iron deficiency
chronic disease

Determine cause
of iron deficiency

Treat iron Treat cause of


deficiency iron deficiency

Figure 287-2 Algorithm for an approach to microcytic anemia. RBC, red blood cell; RDW, red blood cell distribution
width.

(Table 287-4). The degree of iron saturation of plasma inexpensive measurement is widely available. How-
transferrin is calculated as follows: transferrin satura- ever, marked diurnal variation in plasma iron values
tion = (serum iron concentration TIBC)  100. and the numerous clinical disorders that affect the
Serum iron and TIBC levels help confirm the diagno- transferrin saturation limit its use in the clinical setting.
sis of iron deficiency, with a low serum iron and a high Normal or high transferrin saturation is as useful for
transferrin level resulting in a transferrin saturation of excluding IDA as a low value is for identifying it.
less than 10% to 15%. Transferrin levels are increased
in iron-deficiency states because of increased hepatic Red Blood Cell Indices
synthesis of the protein and greater liberation of The development of electronic counters has made the
apotransferrin (the transport protein without iron) use of RBC indices widely available for the initial
from hemoglobin-synthesizing sites. This relatively screening of infants and children for iron deficiency.
Path: K:/AAP-PPC-06-1201/Application/AAP-PPC-06-1201-278-PT-9-Five.3d
Date: 26th August 2008 Time: 19:33 User ID: abhaskar 1BlackLining
Enabled

CHAPTER 287  Iron-Deficiency Anemia 2207

Table 287-3 Red Blood Cell Indices During Infancy and Childhood

HEMOGLOBIN
(g/dL) HEMATOCRIT (%) RETICULOCYTES (%) MCV (fL)

AGE MEAN RANGE MEAN RANGE MEAN LOWEST


CHILD
Cord blood 16.8 13.7-20.1 55 45-65 5.0 110
2 wk 16.5 13.0-20.0 50 42-66 1.0 107
3 mo 12.0 9.5-14.5 36 31-41 1.0 80
6 mo-6 yr 12.0 10.5-14.0 37 33-42 1.0 70-74
7-12 yr 13.0 11.0-16.0 38 34-40 1.0 76-80
ADULT
Female 14 12.0-16.0 42 37-47 1.6 80
Male 16 14.0-18.0 47 42-52 1.6 80

MCV, Mean corpuscular volume; WBC, white blood cells.


Modified from Behrman RE. Nelson Text Book of Pediatrics. 17th ed. Philadelphia: WB Saunders; 2004. Copyright ' 2004, Elsevier, with permission.

Table 287-4 Laboratory Findings Associated With the Differential Diagnosis


of Microcytic Anemias

b-THALASSEMIA CHRONIC
FINDING IRON DEFICIENCY LEAD POISONING TRAIT DISEASE
Ferritin Normal Normal
Serum iron Normal Normal
Total iron-binding capacity Normal Normal
Erythrocyte zinc Normal
protoporphyrin
Red blood cell distribution Normal Normal Normal
width
Serum transferrin receptor Normal Normal

, increased; , very increased; , decreased. Iron deficiency and lead poisoning frequently co-exist.

These tests are highly reproducible and less subject to instrument called a hematofluorimeter. Initially, ZnPP
sampling error compared with hemoglobin determi- was erroneously characterized as metal-free protopor-
nations because tissue fluid dilution does not affect phyrin or free erythrocyte protoporphyrin or erythro-
RBC size. The RBCs become smaller than normal with cyte protoporphyrin.17 In fact, most presumed metal-
decreased MCV, and their hemoglobin content free protoporphyrin in erythrocytes is now known to
decreases with decreased MCH. The RDW approxi- be largely an artifact of the analytical procedures used
mates the standard deviation of the RBC population. at the time and is still used in some laboratories that
Normal RDWs occur in the range of 12% to 17%. In required an acid extraction that removed zinc to form
IDA, a marked dispersion exists in cell volumes (sizes) metal-free or free protoporphyrin. Because approxi-
such that the RDW increases. RBC indices in infancy mately 95% of the nonheme protoporphyrin in eryth-
and childhood are described in Table 287-3. rocytes is ZnPP, this procedure does not create a
diagnostic problem in most cases. The ZnPP/H ratio is
Zinc Protoporphyrin an indicator of iron available to the developing eryth-
A simple and reliable measurement of IDE is the eryth- rocytes in the bone marrow regardless of the cause,
rocyte ZnPP, a product of abnormal heme synthesis. such as iron deficiency, inflammation, or functional
Normally, a trace of zinc rather than iron is incorpo- iron deficiency such as in chronic renal failure.
rated into protoporphyrin during the final step of Another significant limitation of ZnPP is that it
heme biosynthesis. In states of IDE, ZnPP formation is increases with lead toxicity, and even the normal
enhanced.16 An increase in the zinc protoporphyrin to range varies with environmental lead exposure, infec-
heme ratio (ZnPP/H) of greater than 80 mcmol/mol is tions, inflammatory diseases, and protoporphyria.
demonstrated to be a sensitive, specific, and cost- However, the ZnPP/H ratio is not increased in thalas-
effective test for identifying preanemic iron defi- semia trait, which makes ZnPP/H ratio determinations
ciency.16 A major advantage of this well-established helpful in distinguishing iron deficiency from a- or b-
assay is the ability to measure the ZnPP/H ratio thalassemia trait, in addition to its role in screening for
directly on a drop of blood using a dedicated portable iron deficiency (see Table 287-4).
Path: K:/AAP-PPC-06-1201/Application/AAP-PPC-06-1201-278-PT-9-Five.3d
Date: 26th August 2008 Time: 19:34 User ID: abhaskar 1BlackLining
Enabled

2208 PART 9: SPECIFIC CLINICAL PROBLEMS

Reticulocyte Hemoglobin as with liver disease. Although low serum ferritin is


The mean reticulocyte hemoglobin content (CHr) is diagnostic of iron deficiency, a high ferritin level asso-
analogous to the RBC mean corpuscular hemoglobin ciated with inflammation or liver disease does not rule
but with the advantage of monitoring the hemoglo- out concomitant iron deficiency.
binization of the most recently produced RBCs. A
CHr of less than 26 pg is an early indicator of Serum Transferrin Receptor
iron-restricted hematopoiesis and IDA in children.18 The proteolytic cleavage of transferrin receptors can
The diagnostic power of CHr is limited in patients be measured in the serum as sTfR. sTfR directly corre-
with high MCV or with RBC disorders such as lates with the total mass of erythroid precursor. The
thalassemia.19 sTfR is high in iron deficiency and in conditions result-
ing in increased production of RBCs, including thalas-
Peripheral Blood Smear semia and sickle cell disease.21 sTfR is not affected by
Examination of the blood smear in IDA reveals hypo- inflammation, and hence it is useful in distinguishing
chromic microcytes, poikilocytes, elliptocytes, and tar- iron deficiency from chronic inflammatory states that
get cells (Figure 287-3). The presence of basophilic do not have high sTfR (see Table 287-4). Infants have
stippling suggests associated lead poisoning. How- higher baseline sTfR levels than children and adults,
ever, the RBC changes seen on the blood smear are indicating the need to establish age-specific references
not specific for iron deficiency. The white blood cell values. The ratio of sTfR to serum ferritin has been
count and morphology in IDA are usually normal. shown to have excellent performance in estimating
Both thrombocytosis and thrombocytopenia occur body iron stores but is limited by the lack of standardi-
with iron deficiency. The latter is more common in zation for sTfR assays.22,23
severe iron deficiency and resolves once iron therapy
is begun. Bone Marrow Iron
The staining of a normal bone marrow aspirate sample
Definitive Tests with Prussian blue dye reveals the presence of iron in
The absence of stainable iron in the bone marrow is a RBC precursors (normoblasts) and serves as a reliable
definitive test for IDA but is not routinely applicable index of body iron stores. In iron deficiency, the num-
for obvious reasons. The 2 key definitive measure- ber of iron granules in normoblasts is decreased, and
ments for diagnosing iron deficiency are the serum stainable iron in the marrow aspirate is almost com-
ferritin, which measures the size of iron stores, and pletely absent.3
the serum transferrin receptor, which measures the
extent of tissue iron deficiency.20 MANAGEMENT
Serum Ferritin Therapeutic Trial of Iron
Definitive serum ferritin levels vary with age during Therapeutic trial of iron has been proposed as a con-
infancy and childhood. In healthy individuals, serum venient method to diagnose iron deficiency in patients
ferritin levels reflect body iron stores; levels below with anemia. Although this approach is reasonable in
30 ng/L indicate iron deficiency. Ferritin is an acute- otherwise-healthy individuals, in those at high risk of
phase reactant. Serum ferritin levels are increased deficiency, such as infants, teenage girls, and pregnant
during infections and inflammatory processes, as well women, making a definitive laboratory diagnosis at
the outset is preferable.

Treatment of Iron Deficiency


The treatment of choice for iron deficiency is the oral
administration of iron. Although various iron salts are
available, ferrous sulfate is inexpensive and well toler-
ated, although adverse effects may occur, such as nau-
sea, dyspepsia, constipation, and diarrhea. Adverse
effects can be managed by administering the iron with
or immediately after meals. If symptoms persist, then
reductions in the amount of iron in each dose or
reduction in frequency to a single daily dose may help
control the side effects. If intolerance is persistent,
then switching to ferrous gluconate may be helpful.
Iron polysaccharide complex also has the advantage
of availability as tablets or elixir and is well tolerated.
Approximately twice as much iron is absorbed on an
empty stomach as at mealtime. Consumption of milk
Figure 287-3 Iron-deficiency anemia. Many red blood should be limited, which will allow increased intake of
cells are microcytic (smaller than the nucleus of the normal
iron-rich foods, and blood loss from intolerance to
lymphocyte near the center of the field) and hypochromic
cows milk proteins is reduced.
(with central areas of pallor that exceed one half of the diame-
ter of the cells). (Goldman L. Cecil Textbook of Medicine. 22nd
Because soy-based formulas can also lead to blood
ed. Copyright ' 2004 WB Saunders, with permission.) loss,24 children with milk enteropathy should be switched
to a primary diet of iron-containing solids. In the absence
Path: K:/AAP-PPC-06-1201/Application/AAP-PPC-06-1201-278-PT-9-Five.3d
Date: 26th August 2008 Time: 19:34 User ID: abhaskar 1BlackLining
Enabled

CHAPTER 287  Iron-Deficiency Anemia 2209

of ongoing blood losses, response to iron therapy is when oral therapy is not possiblefor example, in
rapid and predictable (see Figure 287-1). A response patients who have inflammatory bowel disease. Three
of decreased irritability and increased appetite to oral drugs are licensed in the United States. Iron dextran
iron therapy has been noted within 12 to 24 hours has been used for several years but is associated with
(Table 287-5). The reticulocyte response peaks at 5 to significant risks. Iron sucrose and iron gluconate may
7 days after the institution of iron therapy. In an have a better safety profile than iron dextran.
otherwise-healthy individual, the recovery from ane-
mia is approximately two-thirds complete within
1 month. The hemoglobin be measured again at
1 month to check the therapeutic progress and to em- BOX 287-1 Recommendations
phasize compliance. of the Centers for Disease
If after 4 weeks the anemia does not respond to iron Control and Prevention
treatment despite compliance with the iron supple-
for the Primary Prevention
mentation regimen and the absence of acute illness,
then the anemia can be further evaluated by using of Iron Deficiency in Infants,
other laboratory tests, including MCV, RDW, and Children, and Adolescents
serum ferritin concentration. For example, a serum
ferritin concentration of 15 mcg/L or less confirms BREASTFEEDING AND IRON-FORTIFIED FORMULA
iron deficiency, and a concentration of more than  Encourage exclusive breastfeeding of infants (without
15 mcg/L suggests that iron deficiency is not the cause supplementary liquid, formula, or food) for 4-6 months
of the anemia. Once the diagnosis of iron deficiency is after birth.
confirmed, either by a response to a therapeutic trial  When exclusive breastfeeding is stopped, encourage
or by further laboratory tests, oral therapy with ele- use of an additional source of iron (approximately
mental iron at 3 to 6 mg/kg per day should be contin- 1 mg/kg/day of iron), preferably from supplementary
ued for 2 to 3 months after normal hemoglobin levels foods.
have been restored. This regimen allows the repletion  For infants aged <12 months who are not breastfed or
of body iron stores. Anemia, microcytosis, and who are partially breastfed, recommend only iron-
increased free erythrocyte protoporphyrin levels are fortified infant formula as a substitute for human milk.
corrected completely with 3 to 5 months of treatment.  For breastfed infants who receive insufficient iron from
Use of intramuscular or intravenous iron is rarely supplementary foods by age 6 months (ie, <1 mg/kg/
warranted. Parenteral iron administration may be indi- day), suggest 1 mg/kg/day of iron drops.
cated in the face of ongoing blood loss that exceeds the  For breastfed infants who were preterm or who had
bodys ability to replenish iron stores through oral low birth weight, recommend 2-4 mg/kg/day of iron
absorption, in the presence of iron malabsorption, or drops (to a maximum of 15 mg/day) starting at
when the patient cannot tolerate or will not take oral 1 month after birth and continuing until 12 months
iron preparations.20 Intramuscular injections are pain- after birth.
ful, and skin discoloration is common. Anaphylactic  Encourage use of only human milk or iron-fortified
reactions have occurred with both intramuscular and infant formula for any milk-based part of the diet (eg,
intravenous injection, and deaths have been reported. in infant cereal) and discourage use of low-iron milks
Parenteral treatment should therefore be used only (eg, cows milk, goats milk, soy milk) until age
12 months.
 Suggest that children ages 1-5 years consume no more
than 24 oz of cows milk, goats milk, or soy milk each day.
Table 287-5 Responses to Iron Therapy SOLID FOODS
in Iron-Deficiency Anemia  At age 4-6 months or when the extrusion reflex
disappears, recommend that infants be introduced to
plain, iron-fortified infant cereal. Two or more servings
TIME AFTER
per day of iron-fortified infant cereal can meet an
IRON infants requirement for iron at this age.
ADMINISTRATION RESPONSE  By approximately age 6 months, encourage one
feeding per day of foods rich in vitamin C (eg, fruits,
12-24 hr Replacement of intracellular vegetables, juice) to improve iron absorption,
iron enzymes; subjective preferably with meals.
improvement; decreased
 Suggest introducing plain, pureed meats after age
irritability; increased appetite
6 months or when the infant is developmentally ready
36-48 hr Initial bone marrow response;
to consume such food.
erythroid hyperplasia
48-72 hr Reticulocytosis, peaking ADOLESCENT GIRLS AND NONPREGNANT WOMEN
at 5-7 day  Encourage adolescent girls and women to eat iron-rich
4-30 day Increase in hemoglobin foods and foods that enhance iron absorption and to
level optimize their dietary iron intake.
1-3 mo Repletion of stores
From Centers for Disease Control and Prevention. Recommendations to pre-
Modified from Behrman RE. Nelson Text Book of Pediatrics. 17th ed. Phila- vent and control iron deficiency in the United States. MMWR Recomm Rep.
delphia: WB Saunders; 2004. Copyright ' 2004, Elsevier, with permission. 1998;47:1-29.
Path: K:/AAP-PPC-06-1201/Application/AAP-PPC-06-1201-278-PT-9-Five.3d
Date: 26th August 2008 Time: 19:34 User ID: abhaskar 1BlackLining
Enabled

2210 PART 9: SPECIFIC CLINICAL PROBLEMS

A blood transfusion is indicated only when severe (3) persistent or unrecognized blood loss, (4) mal-
anemia leads to congestive heart failure and cardio- absorption of iron (eg, primary gastrointestinal dis-
vascular compromise. If a blood transfusion is clini- ease), (5) other diagnoses (eg, a- or b-thalassemia trait
cally warranted, then packed RBCs should be given and Hb E disease), and (6) poor iron utilization (eg,
slowly or a partial exchange transfusion performed. chronic inflammatory disease, sideroblastic anemia,
Vital signs should be monitored carefully. lead poisoning, congenital atransferrinemia).

Failure to Respond to Therapy


When a patient fails to respond to oral iron treat- PREVENTION
ment, the following factors should be considered: (1) non- Increased iron intake among infants has resulted in a
compliance with oral therapy, (2) inadequate iron dose, decline in childhood IDA in the United States. Conse-
quently the use of screening tests for anemia has
become a less efficient means of detecting iron defi-
ciency in some populations, whereas for women of
BOX 287-2 Recommendations childbearing age, iron deficiency has remained preva-
of the Centers for Disease lent. The Centers for Disease Control and Prevention
has therefore developed recommendations for use by
Control and Prevention primary health care physicians for primary prevention
for the Screening for Iron of iron deficiency through appropriate dietary intake
Deficiency in Infants, Children, (Box 287-1) and secondary prevention through detect-
and Adolescents ing and treating IDA (Box 287-2).8

 Anemia screening before age 6 months for preterm


infants and low-birth-weight infants who are not fed
iron-fortified infant formula
WHEN TO REFER
 Annual assessment of children ages 2-5 years for risk
factors for iron-deficiency anemia (IDA) (eg, a low-iron  Cause of anemia is unknown.
diet, limited access to food because of poverty or  Gastrointestinal blood loss is suspected.
neglect, special health care needs)  Anemia is not explained by nutritional imbalance.
 Assessment at ages 9-12 months and 6 months later (at  Anemia is refractory to treatment.
ages 15-18 months) for infants and young children for  Patient requires intravenous iron.
risk factors for anemia:  Diagnosis of iron deficiency is questionable.
n Preterm or low-birth-weight infants
n Infants fed a diet of non-iron-fortified infant formula
for >2 months WHEN TO ADMIT
n Infants introduced to cows milk before age
12 months  Patient exhibits signs of cardiac failure.
 Patient requires intravenous iron.
n Breastfed infants who do not consume a diet
 Patient has moderate to severe blood loss.
adequate in iron after age 6 months (ie, who receive
insufficient iron from supplementary foods)
n Children who consume >24 oz daily of cows milk TOOLS FOR PRACTICE
n Children who have special health care needs (eg, Engaging Patient and Family
children who use medications that interfere with
 Anemia FAQ (fact sheet), American Academy of Pedia-
iron absorption and children who have chronic
infection, inflammatory disorders, restricted diets, or trics (www.aap.org/publiced/BR_Anemia.htm).
extensive blood loss from a wound, an accident, or  Anemia and Your Young Child (brochure), American
surgery) Academy of Pediatrics (patiented.aap.org).
 In populations of infants and preschool children at high
risk for IDA (eg, children from low-income families, RELATED WEB SITE
children eligible for the Special Supplemental Nutrition
 Centers for Disease Control and Prevention: Iron Defi-
Program for Women, Infants, and Children,21 migrant
children, or recently arrived refugee children), screen ciency Anemia (www.cdc.gov/nccdphp/dnpa/nutrition/
all children for anemia between ages 9 and 12 months, nutrition_for_everyone/iron_deficiency/index.htm).
6 months later, and annually from ages 2 to 5 years
 Starting in adolescence, screen all nonpregnant SUGGESTED RESOURCES
women for anemia every 5-10 years throughout their Centers for Disease Control and Prevention. Recommenda-
childbearing years during routine health examinations tions to prevent and control iron deficiency in the United
 Annually screen for anemia women having risk factors States. MMWR Recomm Rep. 1998;47(RR-3):1-29.
for iron deficiency (eg, extensive menstrual or other Cook JD. Diagnosis and management of iron-deficiency
blood loss, low iron intake, a previous diagnosis of IDA) anaemia. Best Pract Res Clin Haematol. 2005;18:319-332.
Sandoval C, Jayabose S, Eden AN. Trends in diagnosis and
From Centers for Disease Control and Prevention. Recommendations to pre- management of iron deficiency during infancy and early
vent and control iron deficiency in the United States. MMWR Recomm Rep. childhood. Hematol Oncol Clin North Am. 2004;18:
1998;47:1-29 1423-1438.
Path: K:/AAP-PPC-06-1201/Application/AAP-PPC-06-1201-278-PT-9-Five.3d
Date: 26th August 2008 Time: 19:34 User ID: abhaskar 1BlackLining
Enabled

CHAPTER 288  Juvenile Idiopathic Arthritis 2211

REFERENCES Chapter 288


1. Grantham-McGregor S, Ani C. A review of studies on
the effect of iron deficiency on cognitive development in JUVENILE IDIOPATHIC
children. J Nutr. 2001;131:649S-666S, discussion 666S-
668S. ARTHRITIS
2. Stoltzfus RJ. Iron-deficiency anemia: reexamining the
nature and magnitude of the public health problem. Sum- David M. Siegel, MD, MPH; Harry L. Gewanter, MD
mary: implications for research and programs. J Nutr.
2001;131:697S-700S, discussion 700S-701S.
3. Centers for Disease Control and Prevention. Iron Juvenile idiopathic arthritis (JIA; previously referred to
deficiencyUnited States, 1999-2000. MMWR Morb as juvenile rheumatoid arthritis or juvenile chronic
Mortal Wkly Rep. 2002;51:897-899.
arthritis) is an uncommon collection of clinical syn-
4. Looker AC, Cogswell ME, Gunter EW. Iron deficiency
United States, 1999-2000. (Reprinted from MMWR.
dromes that have the common feature of chronic child-
2002;51:897-899.) JAMA. 2002;288:2114-2116. hood arthritis. The diagnosis is applied to any child
5. Davis RM. Healthy people 2010: national health objec- younger than 16 years who has persistent arthritis of
tives for the United States. BMJ. 1998;317:1513-1517. one or more joints lasting for more than 6 weeks in
6. Blot I, Diallo D, Tchernia G. Iron deficiency in pregnancy: whom all other diseases have been excluded. JIA is
effects on the newborn. Curr Opin Hematol. 1999;6: classified further into 7 subtypes, with most patients
65-70. fitting into systemic-onset, oligoarthritis, or polyarthri-
7. Rao R, Georgieff MK. Perinatal aspects of iron metabo- tis based on the clinical course over the first 6 months
lism. Acta Paediatr Suppl. 2002;91:124-129.
of illness.1
8. Centers for Disease Control and Prevention. Recommen-
dations to prevent and control iron deficiency in the Although JIA is the most common of the pediatric
United States. MMWR Recomm Rep. 1998;47:1-29. rheumatic diseases, its true incidence and prevalence
9. England JM, Ward SM, Down MC. Microcytosis, aniso- are unknown. For all subtypes taken collectively, the
cytosis and the red cell indices in iron deficiency. Br J peak age at onset is between 2 and 4 years of age, with
Haematol. 1976;34:589-597. a smaller peak later in childhood. Overall, a general
10. Schaefer RM, Bahner U. Iron metabolism in rhEPO- female predominance is found, but not in all subtypes.
treated hemodialysis patients. Clin Nephrol. 2000;53: The best estimate of prevalence is approximately 0.5 to
S65-S68. 1 case per 1000 children; thus approximately 40,000 to
11. Macdougall IC, Cooper AC. Hyporesponsiveness to
100,000 children in the United States have JIA at any
erythropoietic therapy due to chronic inflammation. Eur
J Clin Invest. 2005;35(suppl 3):32-35.
given time.2
12. Mentzer WC Jr. Differentiation of iron deficiency from
thalassaemia trait. Lancet. 1973;1:882. ETIOLOGY
13. Sandoval C, Jayabose S, Eden AN. Trends in diagnosis The exact cause of JIA is unknown. Data on the fre-
and management of iron deficiency during infancy and quency of certain subtypes of human leukocyte anti-
early childhood. Hematol Oncol Clin North Am. 2004;18: gens (HLA) in JIA (eg, HLA-DR5 and -DR8 in younger
1423-1438. girls who have oligoarticular JIA, HLA-DR4 in rheu-
14. Demir A, Yarali N, Fisgin T, et al. Most reliable indices in matoid factor [RF]-positive polyarticular JIA, and
differentiation between thalassemia trait and iron defi- HLA-B27 in older boys who have enthesitis-related
ciency anemia. Pediatr Int. 2002;44:612-616.
JIA)3 have led to the concept of a genetic predisposi-
15. Kwong WT, Friello P, Semba RD. Interactions between
iron deficiency and lead poisoning: epidemiology and tion for the development of an inflammatory arthritis
pathogenesis. Sci Total Environ. 2004;330:21-37. that may be triggered by any of several events, such as
16. Rettmer RL, Carlson TH, Origenes ML, et al. Zinc proto- trauma, infection,4 or emotional stress.5
porphyrin/heme ratio for diagnosis of preanemic iron Other areas of interesting research include investi-
deficiency. Pediatrics. 1999;104:e37. gations of immunologic abnormalities involving auto-
17. Labbe RF, Dewanji A. Iron assessment tests: transferrin antibodies,6 cytokines, immunoregulation,7 and the
receptor vis-a-vis zinc protoporphyrin. Clin Biochem. function of and communication between T and B lym-
2004;37:165-174. phocytes and antigen-presenting cells.3 The precise
18. Brugnara C, Zurakowski D, DiCanzio J, et al. Reticulo-
nature of these interactions and how they result in the
cyte hemoglobin content to diagnose iron deficiency in
children. JAMA. 1999;281:2225-2230.
development of JIA remain to be discovered, but the
19. Mast AE, Blinder MA, Lu Q, et al. Clinical utility of the success of specific cytokine-directed therapy (eg, eta-
reticulocyte hemoglobin content in the diagnosis of iron nercept, anakinra) supports the central role of these
deficiency. Blood. 2002;99:1489-1491. proinflammatory mediators in the clinical manifesta-
20. Cook JD. Diagnosis and management of iron-deficiency tions and course of JIA.
anaemia. Best Pract Res Clin Haematol. 2005;18:319-332.
21. Cook JD. The measurement of serum transferrin recep-
tor. Am J Med Sci. 1999;318:269-276. RHEUMATOLOGIC DISORDERS
22. Brugnara C. Iron deficiency and erythropoiesis: new WITH ARTHRITIS
diagnostic approaches. Clin Chem. 2003;49:1573-1578.
Juvenile ankylosing spondylitis and the other spondy-
23. Cook JD, Flowers CH, Skikne BS. The quantitative
assessment of body iron. Blood. 2003;101:3359-3364. loarthropathies can present as a subtype of JIA at their
24. Nickerson HJ, Silberman T, Park RW, et al. Treatment of onset, especially in an older child who is HLA-B27 pos-
iron deficiency anemia and associated protein-losing itive. Acute rheumatic fever, although long in decline
enteropathy in children. J Pediatr Hematol Oncol. 2000; in the United States and other industrialized nations,
22:50-54. has seen a resurgence in the last 2 decades. Patients
Review in translational hematology

Current concepts in the pathophysiology and treatment of aplastic anemia


Neal S. Young, Rodrigo T. Calado, and Phillip Scheinberg

Aplastic anemia, an unusual hematologic are telomere repair gene mutations in the efits of transplantation to patients who
disease, is the paradigm of the human target cells and dysregulated T-cell activa- are older or who lack family donors. Re-
bone marrow failure syndromes. Almost tion pathways. Immunosuppression with cent results with alternative sources of
universally fatal just a few decades ago, antithymocyte globulins and cyclospor- stem cells and a variety of conditioning
aplastic anemia can now be cured or ine is effective at restoring blood-cell regimens to achieve their engraftment
ameliorated by stem-cell transplantation production in the majority of patients, but have been promising, with survival in
or immunosuppressive drug therapy. The relapse and especially evolution of clonal small pediatric case series rivaling con-
pathophysiology is immune mediated in hematologic diseases remain problem- ventional transplantation results. (Blood.
most cases, with activated type 1 cyto- atic. Allogeneic stem-cell transplant from 2006;108:2509-2519)
toxic T cells implicated. The molecular histocompatible sibling donors is cura-
basis of the aberrant immune response tive in the great majority of young pa-
and deficiencies in hematopoietic cells is tients with severe aplastic anemia; the
now being defined genetically; examples major challenges are extending the ben- 2006 by The American Society of Hematology

Introduction
More than 25 years have passed since our first, highly speculative amelioration in most patients, based both on high-quality clinical
review of aplastic anemia; in that fortunately obscure publication, trials and mechanistic insights from the experimental laboratory.
pathophysiology was addressed tentatively and immunosuppres- Our intention in this circumscribed review is to emphasize the
sive therapies hardly at all. The article did reflect both the dismal most current aspects of aplastic anemia. As a complement, the
prospects for patients with the severe form of marrow failure and reader is referred to monographs, textbook chapters, and other
the formidable practical difficulties of experimentation in a rare recent reviews.1-6
disorder in which the cells of interest had disappeared. Aplastic
anemia was considered heterogenous in origin and virtually
impossible to study systematically. At the bedside, the clinical Etiologies
emphasis was the identification of a putative causal factor
Clinical associations
exposure to benzene or a culpable pharmaceuticalto allow
classification in an otherwise doomed patient. As progenitor assays Since Ehrlichs description of the first case of aplastic anemia in a
were developed, diverse factors could be held theoretically respon- pregnant woman,126 precipitating factors have been sought from
sible for failure to form colonies in tissue culture, ranging from the individual patients history. An enormous literature, dating
quantitative and qualitative defects in stem cells and blocks in from the beginning of the 20th century, described chemical- and
differentiation to a lack of stroma support or inadequate cytokine drug-induced disease, stimulated by observations of the effects of
production, or the effects of a chemical poison. benzene on blood counts, of dipyrones association with agranulo-
In the intervening decades, our understanding of aplastic cytosis, and a seeming epidemic of aplastic anemia after the
anemia has cohered around a unified immune mechanism of introduction of chloramphenicol.
hematopoietic-cell destruction, which was inferred from but also These associations are worth reassessment in the context of the
has informed effective immunosuppressive therapies for the dis- immune hypothesis of marrow failure. Pregnancy appears a real
ease (Figure 1). Technical advances in cell biology, flow cytometry, association, as deduced more from the documented improvement
molecular biology, and immunology have provided methods to of blood counts with its termination than from formal epidemio-
measure numbers and function of very limited numbers of cells. As logic study.7 The unusual syndrome of eosinophilic fasciitis also is
a result, we have a more unified and rational view of aplastic strongly linked to aplastic anemia. Five to 10% of cases of aplastic
anemias pathophysiology; the disease is understood in its relation anemia follow an episode of seronegative hepatitis, in which
to other related marrow failure syndromes; and in many important immune activation is inferred from the pattern of T-cell activation,
respects an unusual blood syndrome can model more common cytokine production, and HLA association.8 Despite intensive
autoimmune diseases of other organ systems (Figure 2). Particu- efforts, including sophisticated molecular and immunologic ap-
larly satisfying is that aplastic anemia is now amenable to cure or proaches and animal inoculations, an infectious agent has not been

From the Hematology Branch, National Heart, Lung, and Blood Institute, N.S.Y. wrote Introduction and Etiologies as well as contributed to
National Institutes of Health (NIH), Bethesda, MD. Pathophysiology and Treatment; R.T.C. wrote Pathophysiology; and P.S.
wrote Treatment. All authors contributed to the final paper.
Submitted March 22, 2006; accepted May 30, 2006. Prepublished online as
Reprints: Neal S. Young, 10 Center Dr, Bldg 10/CRC, Rm 3E-5140, Bethesda,
Blood First Edition Paper, June 15, 2006; DOI 10.1182/blood-2006-03-010777.
MD 20892-1202; e-mail: youngns@mail.nih.gov.
Supported by NIH Intramural Research program. 2006 by The American Society of Hematology

BLOOD, 15 OCTOBER 2006 VOLUME 108, NUMBER 8 2509


2510 YOUNG et al BLOOD, 15 OCTOBER 2006 VOLUME 108, NUMBER 8

Figure 1. Pathophysiology of acquired aplastic ane-


mia. The figure stresses the crucial and related roles of
the hematopoietic stem-cell compartment as a target for
the immune response. An inciting event, such as a virus
or medical drug, provokes an aberrant immune response,
triggering oligoclonal expansion of cytotoxic T cells that
destroy hematopoietic stem cells (left panel, Onset).
Bone marrow transplantation or immunosuppressive
therapy leads to complete response (CR) or partial
response (PR) by eradicating or suppressing pathogenic
T-cell clones (middle panel, Recovery). Relapse occurs
with recurrence of the immune response, and the immu-
nologically stressed and depleted stem-cell compartment
also allows selection of abnormal hematopoietic clones
that manifest as paroxysmal nocturnal hemoglobinuria,
myelodysplasia (MDS), and occasionally acute myelog-
enous leukemia (AML) (right panel, Late Disease).

identified. Benzene, or more correctly its metabolites, is a marrow little difference between patients with idiopathic aplastic anemia
toxin in animals and humans, but in the West benzene exposure as and those with an assumed drug etiology, in demographics,
an etiology of aplastic anemia is now rare. Ancient case reports and response to therapy, or survival.14 In contrast, very few chemo-
series leave doubt as to whether marrow failure in benzene workers therapeutic agents, despite being designed as cell poisons and
was not often myelodysplasia rather than aplastic anemia. Addition- administered in milligram or gram quantities, directly result in
ally, benzene also has effects on immune function.9 irreversible marrow destruction without obvious effects on other
Of greatest practical import is the relationship of medical organs. Claims of permanent aplastic anemia after idiosyncratic
drug use to aplastic anemiaunpredictable marrow failure in exposure to minuscule quantities of chloramphenicol, for ex-
this setting is devastating to the patient and physician and has ample (as in ophthalmic solutions), more likely reflect observa-
serious legal ramifications for pharmaceutical drug develop- tion and reporting biases than a mechanism of extreme sensitiv-
ment.10 The study of idiosyncratic drug reactions, by definition ity to a hidden metabolite.
extremely rare, is difficult. That genetic differences in drug
metabolism, especially in detoxification of reactive intermediate Epidemiology
compounds, underlie susceptibility is best supported by one
Two more reliable approaches to identifying etiology are epidemi-
study of a single individual exposed to carbamazepine, pub-
ology and laboratory identification of antigens. Unfortunately,
lished more than 20 years ago.11 Overrepresentation of deletions
in the drug-metabolizing glutathione-S-transferase genes neither has yielded conclusive results. Two large, controlled,
(GSTM1, GSTT1, which would increase concentrations of toxic population-based studies have been conducted, the International
drug intermediates) has been observed in some series.12,13 Aplastic Anemia and Agranulocytosis Study in Europe and Israel in
Astonishingly, no satisfactory mechanism has been developed the 1980s15 and the recently completed Thai NHLBI Aplastic
for the most notorious pharmaceutical, chloramphenicol, or for Anemia Study in Bangkok and a northeast rural region.16 The
other heavily inculpated agents such as penicillamine or gold. incidence of aplastic anemia in the West is 2/million and about 2- to
Many drugs on black lists also more commonly cause mild 3-fold higher in Asia. Benzene and pesticides, while significantly
marrow suppression, and possibly regular but only modest associated, accounted for only a small number of cases in both
destruction of marrow cells is a prerequisite for a much more studies, and medical drugs have a negligible role in Asia. In rural
infrequent immune response to an exposed neoantigen. A Thailand, exposure to nonbottled water, as well as to certain
parallel mechanism is supported by the clinical observation of animals (ducks and geese), to animal fertilizer, and also to
pesticides, suggested an infectious etiology.
Autoantigens

A few putative antigens have been teased from screening


antibodies in patients sera against a peptide library (by
expression of genes in fetal liver or leukemic-cell lines).
Kinectin, a widely expressed protein, bound to antibodies from
about 40% of aplastic patients.17 Another antigen that bound to
antibodies, in a smaller minority of marrow failure patients, was
diazepam-binding related protein-1, an enzyme essential in the
oxidation of unsaturated fatty acids and broadly distributed in
tissues.18 The relevance of these autoantibodies to a cellular
pathophysiology of aplastic anemia is unclear. For kinectin,
reactive cytotoxic T cells could be generated in vitro and
inhibited human hematopoietic colony formation, but antikinec-
tin T cells were not found in patients.17 For diazepam-binding
related protein-1, a putative T-cell epitope derived from this
Figure 2. Venn diagram of the clinical and pathophysiologic relationships protein could stimulate cytotoxic T cells obtained from one
among the bone marrow failure syndromes, leukemia, and autoimmune
diseases. Overlapping circles indicate difficulties in diagnostic discrimination and patient, and T-cell precursors with peptide-binding activity were
shared underlying mechanisms. present in 2 cases.
BLOOD, 15 OCTOBER 2006 VOLUME 108, NUMBER 8 CURRENT CONCEPTS IN APLASTIC ANEMIA 2511

culture, and their addition to normal marrow inhibited hemato-


Pathophysiology poiesis in vitro (reviewed in Young20). The effector cells were
identified by immunophenotyping as activated cytotoxic T cells
In most cases, aplastic anemia is an immune-mediated disease. expressing Th1 cytokines, especially -interferon. CD8 cells
Cellular and molecular pathways have been mapped in some detail containing intracellular interferon may now be measured di-
for both effector (T lymphocyte) and target (hematopoietic stem rectly in the circulation,21 and oligoclonal expansion of CD8
and progenitor) cells (Figure 3). Exposure to specific environmen- CD28 cells, defined by (1) flow cytometric analysis for T-cell
tal precipitants, diverse host genetic risk factors, and individual receptor (TCR) V subfamilies; (2) spectratyping to detect
differences in the characteristics of the immune response likely skewing of CDR3 length; and (3) sequencing of the CDR3
account for the diseases infrequency, variations in its clinical region to establish a molecular clonotype.22 In general, patients
behavior, and patterns of responsiveness to treatment. at presentation demonstrate oligoclonal expansions of a few V
subfamilies, which diminish or disappear with successful therapy;
Immune-mediated T-cell destruction of marrow
original clones re-emerge with relapse, sometimes accompanied
An immune mechanism was inferred decades ago from the by new clones, consistent with spreading of the immune
recovery of hematopoiesis in patients who failed to engraft after response. Very occasionally, a large clone persists in remission,
stem-cell transplantation, when renewal of autologous blood-cell perhaps evidence of T-cell tolerance.
production was credited to the conditioning regimen. Also sugges- The impact of T-cell attack on marrow can be modeled in vitro
tive was that the majority of syngeneic transplantations in which and in vivo. -Interferon (and tumor necrosis factor-) in increas-
bone marrow was infused without conditioning failed.19 The ing doses reduce numbers of human hematopoietic progenitors
responsiveness of aplastic anemia to immunosuppressive therapies assayed in vitro; the cytokines efficiently induce apoptosis in CD34
remains the best evidence of an underlying immune pathophysiol- target cells, at least partially through the Fas-dependent pathway of
ogy: the majority of patients show hematologic improvement after cell death.23 In long-term culture of human bone marrow, in which
only transient T-cell depletion by antithymocyte globulins (ATGs); stromal cells were engineered to constitutively express -inter-
relapse also usually responds to ATG; and dependence of adequate
feron, the output of long-term culture-initiating cells (LTCI-ICs)
blood counts on administration of very low doses of cyclosporine is
was markedly diminished, despite low concentrations of the
not infrequent. As immunosuppression has intensified, from early
cytokine in the media, consistent with local amplification of
attempts with corticosteroids to aggressive strategies such as
toxicity in the marrow milieu.24 Immune-mediated marrow failure
high-dose cyclophosphamide, and the proportion of responders has
risen, the willingness to ascribe an immunologic mechanism also has been modeled in the mouse: infusion of parental lymph node
increased. Indeed, little distinguishes responders to immunologic cells into F1 hybrid donors caused pancytopenia, profound marrow
therapy from refractory patients (other than age, as children show aplasia, and death.25 Not only a murine version of ATG and
higher rates of recovery and survival). A nonimmune pathophysiol- cyclosporine but also monoclonal antibodies to -interferon and
ogy has been inferred from a failure to respond to immunosuppres- tumor necrosis factor abrogated hematologic disease, rescuing
sion, but refractoriness to therapy is also consistent with very animals. A powerful innocent bystander effect, in which acti-
severe stem-cell depletion, a spent immune response, or immuno- vated cytotoxic T cells kill genetically identical targets, was present
logic mechanisms not susceptible to current therapies. in secondary transplantation experiments.26 In a minor histocompat-
In early laboratory experiments, removal of lymphocytes ibility antigen-discordant model, marrow destruction resulted from
from aplastic bone marrows improved colony numbers in tissue activity of an expanded H60 antigenspecific T-cell clone.27

Figure 3. Immune destruction of hematopoiesis. Anti-


gens are presented to T lymphocytes by antigen-
presenting cells (APCs), which trigger T cells to activate
and proliferate. T-bet, a transcription factor, binds to the
interferon- (INF-) promoter region and induces gene
expression. SAP binds to Fyn and modulates SLAM
activity on IFN- expression, diminishing gene transcrip-
tion. Patients with aplastic anemia show constitutive
T-bet expression and low SAP levels. IFN- and TNF-
up-regulate other T cells cellular receptors and also the
Fas receptor. Increased production of interleukin-2 leads
to polyclonal expansion of T cells. Activation of Fas
receptor by the Fas ligand leads to apoptosis of target
cells. Some effects of IFN- are mediated through inter-
feron regulatory factor 1 (IRF-1), which inhibits the tran-
scription of cellular genes and entry into the cell cycle.
IFN- is a potent inducer of many cellular genes, includ-
ing inducible nitric oxide synthase (NOS), and production
of the toxic gas nitric oxide (NO) may further diffuse toxic
effects. These events ultimately lead to reduced cell
cycling and cell death by apoptosis.
2512 YOUNG et al BLOOD, 15 OCTOBER 2006 VOLUME 108, NUMBER 8

Why T cells are activated in aplastic anemia is unclear. failure syndrome indicated a genetic basis for telomere deficiency.
HLA-DR2 is overrepresented among patients, suggesting a role for Central to the repair machinery is an RNA template, encoded by
antigen recognition, and its presence is predictive of a better TERC, on which telomerase, a reverse transcriptase encoded by
response to cyclosporine.28,29 Polymorphisms in cytokine genes, TERT, elongates the nucleotide repeat structure; other proteins,
associated with an increased immune response, also are more including the DKC1 gene product dyskerin, are associated with the
prevalent: a nucleotide polymorphism in the tumor necrosis telomere repair complex. Systematic surveys of DNA disclosed
factor- (TNF2) promoter at 308,30,31 homozygosity for a vari- first TERC47,48 and later TERT mutations49 in some patients with
able number of dinucleotide repeats in the gene encoding -inter- apparently acquired aplastic anemia, including older adults. Family
feron,32 and polymorphisms in the interleukin 6 gene.33 Constitu- members who share the mutation, despite normal or near-normal
tive expression of T-bet, a transcriptional regulator that is critical to blood counts, have hypocellular marrows, reduced CD34-cell
Th1 polarization, occurs in a majority of aplastic anemia patients.34 counts and poor hematopoietic colony formation, increased hema-
Mutations in PRF1, the gene for perforin, are responsible for some topoietic growth factor levels, and of course short telomeres;
cases of familial hemophagocytosis; mutations in SAP, a gene however, their clinical presentation is much later than in typical
encoding for a small modulator protein that inhibits -interferon dyskeratosis congenita, and they lack typical physical anoma-
production, underlie X-linked lymphoproliferation, a fatal illness lies.47,49 Chromosomes are also protected by several proteins that
associated with an aberrant immune response to herpesviruses and bind directly to telomeres, and polymorphisms in their genes
aplastic anemia. Perforin is overexpressed in aplastic marrow.35 We (TERF1, TERF2) are also more or less prevalent in aplastic anemia
have detected heterozygous mutations in PRF1 in 5 adults with compared with healthy controls.50 A few of our patients also have
severe aplasia and hemophagocytosis of the marrow, and SAP heterozygous mutations in the Shwachman-Bodian-Diamond syn-
protein levels are markedly diminished in a majority of acquired drome (SBDS) gene. Almost all children with this form of
aplastic anemia cases (E. Solomou, unpublished data, June 2006). constitutional aplastic anemia are compound heterozygotes for
These alterations in nucleotide sequence and in gene regulation mutations in SBDS, and their white cells have extremely short
suggest a genetic basis for aberrant T-cell activation in bone telomeres51; however, the SBDS gene product has not been directly
marrow failure. Genome-wide transcriptional analysis of T cells linked to the telomere repair complex or to telomere binding. A
from aplastic anemia patients has implicated components of innate parsimonious inference from all these data is that inherited mutations in
immunity in aplastic anemia, including Toll-like receptors and genes that repair or protect telomeres are genetic risk factors in acquired
natural killer cells,36 for which there is some preliminary experimen- aplastic anemia, probably because they confer a quantitatively reduced
tal support.37,38 hematopoietic stem-cell compartment that may also be qualitatively
inadequate to sustain immune-mediated damage.52
Hematopoiesis Telomeres are short in one third to one half of aplastic anemia
Immune attack leads to marrow failure. Anhematopoiesis was patients,45,46 but mutations have been identified in only less than
inferred from the empty appearance of the marrow at autopsy by 10% of cases. The most interesting explanation is involvement of
the earliest observers of the disease. The pallor of the modern other genes, including genes for other members of the large repair
biopsy core or empty spicules of an aspirate, few or no CD34 cells complex, telomere binding proteins, still obscure components of
on flow cytometry, and minimal numbers of colonies derived from the alternative repair system, and some DNA helicases. Alterna-
committed progenitors in semisolid media all reflect the severe tively, telomere shortening may be secondary to stem-cell
reduction in hematopoietic cells that defines the disease. Stem-cell replication.
surrogatereally correlativeassays, LTC-ICs,39 or cobblestone- Clonal evolution
forming cells,40 which measure a primitive infrequent and quies-
cent multipotential progenitor cell, also show marked deficiency, Clinically, aplastic anemia may coexist or appear to evolve to other
and from the product of the low percentage of marrow cellularity hematologic diseases that are characterized by proliferation of
and the scant numbers of LTC-ICs per mononuclear cell, suggest distinctive cell clones, as in paroxysmal nocturnal hemoglobinuria
that only a small percentage of residual early hematopoietic cells (PNH) or myelodysplasia (MDS; Figure 2). The mechanisms
remains in severely affected patients at presentation. Qualititative linking immune-mediated and premalignant pathophysiologies are
features of these few cells, as measured, for example, by poor not elucidated in marrow failure or in parallel circumstances
colony formation per CD34 cell or inadequate response to hemato- (chronic hepatitis and hepatocellular carcinoma, ulcerative colitis
poietic growth factors, are harder to interpret, although recent and colon cancer, and many others). The presence of tiny clones at
genetic studies have suggested explanatory mechanisms (see next the time of diagnosis of aplastic anemia, detected using extremely
paragraph). The reduced number and function of the marrow is sensitive assaysphenotypic (flow cytometry for PNH) or cytoge-
secondary to cell destruction, and apoptosis is prevalent among the netic (fluorescent in situ hybridization for MDS)also creates the
few remaining elements.41,42 Microarray of the scant CD34 cells problems of disease classification and patient diagnosis.
from marrow failure patients revealed a transcriptome in which PNH. Fifty percent or more of patients at presentation with
genes involved in apoptosis, cell death, and immune regulation pancytopenia have expanded populations of PNH cells, easily
were up-regulated43; this transcriptional signature was reproduced detected by flow cytometry due to the absence of glycosylphosphati-
in normal CD34 cells exposed to -interferon.44 dylinositol-linked membrane proteins, the result of somatic PIG-A
One peculiar feature of white blood cells in aplastic anemia is gene mutations.53 Most clones are small and do not lead to clinical
short telomeres.45,46 Telomere shortening was initially most easily manifestations of hemolysis or thrombosis, but classic PNH can be
blamed on stem-cell exhaustion. However, the discovery, first by dominated by marrow failure (the aplastic anemia/PNH syn-
linkage analysis in large pedigrees, that the X-linked form of drome), and all PNH patients show evidence of underlying
dyskeratosis congenita was due to mutations in DKC1 and subse- hematopoietic deficiency. The global absence of a large number of
quently purposeful identification of mutations in TERC in some cell-surface proteins in PNH has been hypothesized to allow
autosomal dominant patients with this constitutional marrow escape and survival of a pre-existing mutant clone. Association
BLOOD, 15 OCTOBER 2006 VOLUME 108, NUMBER 8 CURRENT CONCEPTS IN APLASTIC ANEMIA 2513

of an expanded PNH clone with HLA-DR229 and with autoantibod- remain below normal but adequate to avoid transfusion and to
ies,18 and as a predictor of responsiveness to immunosuppressive prevent infection. Most specialists use an ATG-based regimen in
therapies,54 suggests that the escape is from immune attack. combination with cyclosporine, based on the outcomes of relatively
However, there is little concrete experimental evidence of reproduc- large studies performed in the 1990s67 (Table 1). The larger
ible differences in either differential immune responsiveness or experience is with ATG produced in horses, although a rabbit ATG,
susceptibility of PNH clones compared with phenotypically normal recently approved for use in the United States, is more potent by
target-cell populations.55,56 In contrast to cells of normal pheno- weight and in the treatment of graft rejection after solid organ
type, the marrow PNH clone retains its proliferative capacity in transplantations (a current NIH trial is directly comparing these 2
tissue culture and does not overexpress Fas57; comparison by ATGs as first therapy in severe aplastic anemia). ATG is cytolytic:
microarray shows that residual cells of normal phenotype in the lymphocyte numbers consistently decline during the first few days
PNH bone marrow up-regulate the same apoptosis and cell-death of infusion and then return to pretreatment levels in a week or 2.
genes as do CD34 cells in aplastic marrow, while the PIG-A clone ATGs are produced by immunizing animals against human thymo-
appears transcriptionally similar to CD34 cells from healthy cytes, not lymphocytes, and the mix of antibody specificities plus
donors.58 Despite a few provocative studies,37,55,59 no satisfying direct experimentation suggests that ATG may be immunomodula-
mechanism to explain clonal escape has convincing empiric tory as well as lymphocytotoxic, perhaps producing a state of
support (see Young53 for review). tolerance by preferential depletion of activated T cells. The toxicity
MDS. Stereotypical patterns of aneuploidy develop in a minor- of ATG is allergic, related to administration of a heterologous
ity of patients over time: monosomy 7 or trisomy 8 is most protein, and there is little added infection risk beyond the neutrope-
characteristic.60 Trisomy 8 is MDS with many immune abnormali- nia intrinsic to the disease. ATG doses and regimens are empiric
ties that resemble aplastic anemia. Patients respond to immunosup- and traditional. By administration of a larger dose over fewer days,
pressive therapies, and oligoclonal T-cell expansions are usually immune complex formation and consequent serum sickness are
present.61 T-cell oligoclones appear to recognize the aneuploid minimized, as patients usually do not produce their own antibodies
cells, and specifically WT1 antigen that they express at high to the foreign protein until a week or 10 days after exposure.
levels,62 but the target cells are not killed due to up-regulation of Cyclosporines selective effect on T-cell function is due to direct
antiapoptosis genes, including c-myc, survivin, and CDK1.63 For inhibition on the expression of nuclear regulatory proteins, result-
trisomy 8, abnormal cells targeted by the immune system appear to be ing in reduced T-cell proliferation and activation. While severe
selected for their capacity to survive cytotoxic lymphocyte attack. aplastic anemia can respond to cyclosporine alone, it is less
Monosomy 7 is also a frequent cytogenetic abnormality in effective than either ATG alone or ATG plus cyclosporine.73,74 As
aplastic anemia but has a poorer prognosis, with patients usually with ATG, doses and length of treatment have not been formally
succumbing to refractory cytopenias or evolving to acute leuke- established. Cyclosporine has many side effects, but most are
mia.64 Emergence of monosomy 7 has been linked to exogenous manageable by dose reduction; permanent kidney damage is
use of G-CSF in aplastic anemia,65 as occurs in treated severe unusual with monitoring (to maintain blood levels at nadir of about
congenital neutropenia. Laboratory studies of marrow from aplas- 200 ng/mL). Maintenance of blood counts may be achieved with
tic anemia and myelodysplasia patients suggest that monosomy 7 very low doses of cyclosporine, such that drug levels in blood are
clones expand in an abnormal cytokine milieu: high G-CSF undetectable and toxicity is minimal, even with years of treatment.
concentrations lead to selection of cells that bear a short isoform of Outcomes of combined immunosuppressive therapy. Re-
the G-CSF receptor that signals proliferation but not differentiation.66 ported hematologic response rates vary, at least in part due to lack
of consensus on parameters (transfusion independence, absolute or
relative improvement in blood counts) and defined landmarks. In
Treatment our experience, improvement of blood counts so that the criteria for
severity are no longer met highly correlates with termination of
Immunosuppression
transfusions, freedom from neutropenic infection, and better sur-
Antithymocyte globulin (ATG) and cyclosporine (combined or vival. By this standard, about 60% of patients are responders at 3 or
intensive immunosuppression). For aplastic anemia that is severe, 6 months after initiation of horse ATG.70 Comparable figures for
as defined by peripheral-blood counts, definitive therapies are hematologic response rates have come from Europe69 and Japan.71
immunosuppression or stem-cell transplantation; immunosuppres- Responders have much better survival prospects than do nonre-
sive therapies are most widely used because of lack of histocompat- sponders. Long-term prognosis is predicted by the robustness of the
ible sibling donors, patient age, and the immediate cost of early blood count response (defined as either platelets or reticulo-
transplantation. Even in responding patients, blood counts often cytes 50 109/L [50 000/L] 3 months after treatment): about

Table 1. Intensive immunosuppression (ATG plus cyclosporine) for severe aplastic anemia
Study N Median age, y Response, % Relapse, % Clonal evolution, % Survival, %

German68 84 32 65 19 8 58 at 11 y
EGMBT69 100 16 77 12 11 87 at 5 y
NIH70 122 35 61 35 11 55 at 7 y
Japan*71 119 9 68 22 6 88 at 3 y
NIH72 104 30 62 37 9 80 at 4 y

Only studies of more than 20 enrolled patients are tabulated. Responses to immunosuppressive therapy are usually partial; blood counts may not become normal but
transfusions are no longer required and the neutrophil count is adequate to prevent infection. Relapse is usually responsive to further immunosuppressive therapies. Clonal
evolution is to dysplastic bone marrow changes and/or cytogenetic abnormalities. For details, see Immunosuppression.
*With androgens and G-CSF.
With mycophenolate mofetil.
2514 YOUNG et al BLOOD, 15 OCTOBER 2006 VOLUME 108, NUMBER 8

50% of patients who are treated with horse ATG have a robust in aplastic anemia, results were not superior to these agents only
response and almost all of them will survive long term. Outcomes (P.S., unpublished data, January 2006).
of immunosuppressive therapy are related to patient age: 5-year Cyclophosphamide. As with ATG, recovery of blood counts
survival of more than 90% of children has been reported in recent can occur after a failed bone marrow transplantation preceded by
German,75 Japanese,71 and Chinese76 trials, compared with about conditioning with cyclophosphamide. High-dose cyclophospha-
50% survival for adults older than 60 years in the collective mide was used intermittently by investigators at Johns Hopkins
European experience.77 University (Baltimore, MD) in the 1980s during periods in which
Relapse, defined conservatively as a requirement for additional ATG apparently was not available to them; in their most recent
immunosuppression and not necessarily recurrent pancytopenia, is update, of 38 previously untreated patients, the response rate was
not uncommon, occurring in 30% to 40% of responding patients. 74% and survival estimated at 86%.83,84 In contrast, an NIH
Relapse defined by renewed need for transfusion was estimated at randomized study was halted early due to the development of
12% of European patients at 3 years, but prolonged cyclosporine fungal infections and a much higher death rate in the cyclophospha-
dependency among all patients was common.69 Reinstitution of mide arm,85 and both relapse and cytogenetic evolution were
cyclosporine usually reverses declining blood counts, but when observed.86 The major toxicity of high-dose cyclophosphamide,
required, a second round of horse78 or rabbit79 ATG is usually prolonged neutropenia with concomitant susceptibility to infection,
effective. In our experience, relapse does not confer a poor is now addressed by the Baltimore investigators by routine
prognosis, but it is obviously inconvenient and may not always be antimicrobial prophylaxis and prolonged G-CSF administration.
remediable. Molecular analysis of the T-cell response in aplastic Cyclophosphamide therapy does not eradicate PNH clones, and
anemia, discussed in Pathophysiology, suggests that the major relapses now have been observed in Baltimore.84 In the absence of
reason for relapse is incomplete eradication of pathogenic clones another randomized trial, comparison of data from a small,
by ATG. single-center pilot with historical and more general results is
More serious than relapse is evolution of aplastic anemia to problematic; it is especially difficult to exclude biased patient
another clonal hematologic disease, PNH, myelodysplasia, and selection, both explicit (such as exclusion of those unlikely to
leukemia. Small PNH clones present at diagnosis usually remain respond or with a generally poor prognosis or older patients) and
stable over time but may expand sufficiently to produce symptom- implicit (inability to treat uninsured individuals or foreign citizens).
atic hemolysis. For myelodysplasia and leukemia, the cumulative Management of refractory aplastic anemia. There is no
long-term rate of clonal evolution is about 15%70,80; evolution is established algorithm for the management of patients who have
not inevitable in aplastic anemia, and some cytogenetic abnormali- failed to respond to ATG.67 Transplantation from an alternative
ties may be transient or, as with trisomy 8, responsive to immuno- donor is offered by many centers to children who have failed a
suppressive treatments. As discussed in Clonal evolution, emer- single course of immunosuppression and to adults after 2 rounds of
gence of monosomy 7 may be favored in severely neutropenic ATG therapy (see Other stem-cell sources). Response rates to
patients who require chronic G-CSF therapy.66 second ATG have ranged from 22% to 64%.78 In an Italian study of
Improving on ATG and cyclosporine. Growth factors. Histori- rabbit ATG as second therapy, 23 (77%) of 30 improved87; in the
cally, intensification of immunosuppression has increased response NIH experience, the proportion responding was closer to 30%.79
rates. However, attempts to improve on ATG plus cyclosporine Cyclophosphamide also has been administered in this setting, with
have been frustratingly disappointing. Megadoses of methylpred- a response rate of about 50% reported.83 A third course of
nisolone only added toxicities. Small pilots of GM-CSF81 and immunosuppression may benefit only patients who showed some
much larger, randomized studies of G-CSF71,82 as routine additions response to a previous treatment.88 Response to retreatment
to ATG and cyclosporine have been negative to date; improved correlates to a better survival compared with refractory patients.79
neutrophil counts did not translate into a higher rate of recovery or The improved survival of patients who are refractory to immuno-
even less infection. A very large ongoing European study of G-CSF suppression, due to better supportive care, complicates the decision
should definitively answer efficacy and safety concerns. to undertake high-risk transplantation (see Other stem-cell
Other immunosuppressive drugs. As the addition of cyclospor- sources). We have tested alemtuzumab, a humanized monoclonal
ine clearly improved outcomes compared with the use of ATG antibody specific for CD52, an antigen present on all lymphocytes;
alone, other immunosuppressive drugs might be predicted, based alemtuzumab induces profound immunosuppression by lymphocy-
on their mode of action, animal studies, and experience in other totoxicity and has been effective in lymphoproliferative diseases,
human diseases and with organ transplantation, to be effective. graft-versus-host disease (GVHD), and autoimmune disorders. To
Mycophenolate mofetil is a tolerizing agent, as it selectively date, 4 of 8 patients who were refractory to treatment with horse
depletes activated cells by inhibition of inosine monophosphate ATG have responded to alemtuzumab, and toxicity has been
dehydrogenase, a critical enzyme of the purine salvage pathway, modest (P.S., unpublished data, January 2006); we are now testing
therefore blocking activated lymphocyte proliferation. Nonethe- alemtuzumab in a randomized comparison with both horse and
less, its addition to ATG and cyclosporine in an NIH trial of 104 rabbit ATG in severe aplastic anemia at presentation.
patients did not change hematologic response (about 62%), relapse Treatment of moderate pancytopenia. Clinically, the course of
(37%), or evolution rates; at best, there was a modest sparing of moderate aplastic anemia is variable: some patients progress to
cyclosporine usage.72 The 4-year survival rate for all treated severe disease, others remain stable and may not require interven-
patients was 80%almost certainly due to better supportive care tion; regular transfusions may not be required.89 Very few clinical
rather than any new drug effect. trials have specifically addressed moderate disease. Immunosuppres-
Sirolimus, which blocks the serine-threonine kinase known as sion can reverse moderate pancytopenia and alleviate transfusion
mammalian target of rapamycin is synergistic with cyclosporine in requirements; ATG and cyclosporine are more effective in combina-
tissue culture and in clinical transplantation. Again, when tested in tion,73 but in practice are often used sequentially. Daclizumab, a
a randomized protocol in combination with ATG and cyclosporine humanized monoclonal antibody to the interleukin-2 receptor,
BLOOD, 15 OCTOBER 2006 VOLUME 108, NUMBER 8 CURRENT CONCEPTS IN APLASTIC ANEMIA 2515

improved blood counts and relieved transfusion requirements in 6 Graft-versus-host disease remains a serious problem for older
of 16 evaluable patients; the outpatient regimen had little toxicity.90 patients, even with routine cyclosporine prophylaxis. In the IBMTR,
When there is residual hematopoietic function, androgens may rates of severe GVHD doubled in adults compared with children
be effective (although male hormones have failed most rigorous (15%-20% for recipients 20 years of age to 40%-45% for 20
trials in severe aplastic anemia). Some moderate aplastic anemia years of age).107 In Seattle, chronic graft-versus-host disease
likely results from telomere gene mutations and stem-cell exhaus- developed in 41% of patients who had survived more than 2 years
tion. In vitro, androgens increase telomerase activity in human after transplantation, tripling the risk of death and often requiring
lymphocytes and CD34 cells, acting through the estradiol recep- years of immunosuppressive therapy.109 Even with resolution,
tor,91 and this activity may provide a mechanism of action for their chronic GVHD remains a risk factor for late complications such as
effects on marrow function. growth and endocrine system effects, pulmonary disease, cataracts,
neurologic dysfunction, and secondary malignancy. Addition of
Hematopoietic stem-cell transplantation ATG105 and more recently its substitution by alemtuzumab110 may
reduce the frequency and severity of acute GVHD, a predictor of
Allogeneic HLA-matched sibling donor transplantation. Hemato- chronic GVHD.
poietic and immune system cells are replaced by stem-cell transplan- Matched unrelated donor transplant. A matched sibling donor
tation; conditioning with cyclophosphamide is not myeloablative is available in only 20% to 30% of cases. As the outcome in aplastic
but is sufficiently immunosuppressive to prevent and to eliminate
patients who have failed a single round of ATG has been poor,
residual host marrow by a graft-versus-marrow effect.
alternative sources of hematopoietic stem cells have been sought,
Allogeneic transplant from a matched sibling donor cures the
usually from now very large donor registries (Table 3). Outcomes
great majority of patients (Table 2): the most recent cohort reported
of 318 alternative donor transplants performed from 1988 to 1998
to the IBMTR showed 77% 5-year survival,107 and in children, and
recently have been summarized for the European registry92: for
patients undergoing transplantation who are minimally transfused,
matched unrelated donors, the rejection rate was 15% and for
survival of 80% to 90% may be routinely achieved. Graft rejection,
grades II to IV GVHD, 48%, and 5-year survival was estimated at
a historic problem in the application of transplantation to aplastic
anemia (most dramatically manifest in rejection of unprepared 39%. From diverse registry data (collected through EBMT, IBMTR,
syngeneic stem cells), is now not frequent in patients who undergo and the National Marrow Donor Program),92,107,121 the mortality
transplantation early and with a modest transfusion burden, likely a rate is about twice that observed in matched sibling transplants;
benefit of less immunogenic blood products (leukocyte-depleted even with predominantly younger patients as recipients, age is
erythrocytes, for example) from fewer donors (platelets collected probably the most powerful influence on survival, but also
by cytopheresis). Conditioning regimens that do not include important are the closeness of the class I HLA match and the length
irradiation now regularly achieve engraftment and avoid many of of time from diagnosis. A retrospective analysis from the Japan
irradiations long-term complications, especially late cancers. In a Marrow Donor Program suggested that patients with the most
recent series of 81 patients who were prepared by cyclophospha- favorable characteristics and conditioned with a minimal dose of
mide plus ATG, sustained engraftment was achieved by 96%, and 3 radiation might anticipate survival comparable with matched
of the 4 patients who initially rejected the transplant successfully sibling transplants.113
underwent a retransplantation; 88% of the patients survived long Prospective trials have enrolled fewer patients but have
term.104 The combination of cyclophosphamide plus fludarabine, better results (perhaps due to superior protocols, but both careful
with or without ATG, has achieved high rates of graft acceptance patient selection and publication bias are likely important). In
and survival even in heavily transfused patients who received a contrast to allogeneic sibling transplants, transplants from
transplant of mobilized peripheral-blood stem cells, months after unrelated donors still require irradiation to ensure engraftment,
proving refractory to immunosuppressive drugs.106,108 due both to source of the donor cells and the transfusion status of

Table 2. Allogeneic sibling transplantation for severe aplastic anemia


Years of Age, y Acute Actuarial
Institution/study study N (median in y) Graft rejection/failure, % GVHD,* % Chronic GVHD, % survival, %

IBMTR92 1988-1992 471 20 (1-51) 16 19 32 66 at 5 y


Vienna93 1982-1996 20 25 (17-37) 0 26 53 95 at 15 y
EBMT94 1991-1998 71 19 (4-46) 3 30 35 86 at 5 y
Seoul95 1990-1999 22 22 (14-43) 5 10 33 95 at 5 y
Seoul96 1990-2001 64 28 (14-43) 18 31 19 79 at 6 y
Hamburg97 1990-2001 21 25 (7-43) 5 5 5 86 at 5 y
Paris98 1994-2001 33 20 (8-42) 6 0 42 94 at 5 y
Sao Paulo99 1993-2001 81 24 (3-53) 22 37 39 56 at 6 y
Taipei100 1985-2001 79 22 (4-43) 8 7 35 74 at 5 y
Tunis101 1998-2001 31 19 (4-39) 16 11 3 86 at 2 y
Seoul102 1995-2001 113 28 (16-50) 15 11 12 89 at 6 y
London103 1989-2003 33 17 (4-46) 24 14 4 81 at 5 y
Seattle104,105 1988-2004 94 26 (2-59) 4 24 26 88 at 6 y
Mexico City106 2000-2005 23 25 (4-65) 26 17 26 88 at 4 y

In contrast to Table 1, response rates are not provided because, in surviving patients who do not experience primary graft rejection or secondary graft failure, full
hematologic recovery with donor hematopoiesis is anticipated. Only studies reporting at least 20 patients are tabulated.
GVHD indicates graft-versus-host disease; IBMTR, International Blood and Marrow Transplant Registry; and EBMT, European Group for Bone Marrow Transplant.
*Results are generally for grades II to IV and patients at risk.
2516 YOUNG et al BLOOD, 15 OCTOBER 2006 VOLUME 108, NUMBER 8

Table 3. Alternative donor stem-cell transplantation for severe aplastic anemia


Age, y,
Year of median Acute Chronic
Study publication N Donor source, N Conditioning (range) GVHD,* % GVHD, % Survival, %

Nagoya111 2001 15 MUD: 11; MMUD: 4 Cy/ATG/TBI 11 (3-19) 33 13 100 at 4 y


Great Britain112 2001 8 MUD: 7; MMUD: 1 Cy/CP/TBI 7 (0-10) 25 0 100 at 3 y
Japan Marrow 2002 154 MUD: 79; MMUD: 75 Cy TBI or LFI; 17 (1-46) 29 30 56 at 5 y
Donor Cy/ATG TBI or LFI
Program113
Memphis114 2004 9 MUD: 4; MMUD: 5 High CD34 cell dose, TCD, 11 (6-16) 0 0 89 at 4 y
Cy/ATG/TLI or
TBI thiotepa
Gyeonggi-do115 2004 5 MUD Cy/Flu/ATG 13 (7-18) 0 0 80 at 2 y
Guangzhou116 2004 6 UCB Cy/ATG 26 (22-37) 0 33 66 at 2 y
Genoa117 2005 38 MUD: 33; MMRD: 5 Cy/Flu/ATG 14 (3-37) 11 24 73 at 2 y
Philadelphia118 2005 12 MUD: 4; MMUD: 8 Partial TCD, TBI Cy/ 9 (1-20) 33 25 75 at 4 y
Ara-C or Cy/TT or ATG
Seoul119 2005 13 MUD: 12; MMUD: 1 Cy/ATG 22 (15-34) 31 62 75 at 3 y
IBMTR92 2006 318 MUD: 181; MMRD: Various 16 (1-55) 48 for MUD 29 for MUD 39 at 5 y for MUD
86; MMUD: 51
Seattle120 2006 87 MUD: 62; MMUD: 25 Cy/ATG/TBI 19 (1-53) 70 for MUD 52 for MUD 61 at 5 y for MUD

Only studies reporting at least 5 patients are tabulated. GVHD, graft-versus-host disease; IBMTR, International Blood and Marrow Transplant Registry; MUD, matched
unrelated donor; MMUD, mismatched unrelated donor; Cy, cyclophosphamide; ATG, antithymocyte globulin; TBI, total body irradiation; CP, alemtuzumab; LFI, limited field
irradiation; TCD, T-cell depletion; TLI, total lymphoid irradiation; Flu, fludarabine; UCB, umbilical cord blood; MMRD, mismatched related donor; and TT, thiotepa.
*GVHD results are generally for grades II to IV and patients at risk.

the recipient. In a recent multicenter study, 62 patients with the small numbers of stem cells contained in a single cord-blood
severe aplastic anemia who were refractory to immunosuppres- sample. In a report from the National Marrow Donor Program,
sive therapy underwent matched unrelated stem-cell transplanta- engraftment occurred in less than half of 19 recipients, and almost
tion following conditioning with cyclophosphamide, ATG, and all the patients died from transplant-related causes or survived due
total body irradiation; graft failure occurred in 2%, acute grades to autologous reconstitution or a second transplantation.123 Surpris-
II to IV GVHD was observed in 70%, chronic GVHD was ingly, 5 of 6 Chinese adult aplastic anemia patients successfully
observed in 52%, and overall survival was 61%. Twenty five engrafted, and 4 survived.116 Advocates of this approach are using
patients who lacked an HLA-identical donor received an HLA- pooled donations to increase stem-cell numbers.
nonidentical stem-cell graft: 88% showed sustained engraftment, Family members are almost always available to the patient as a
and overall survival was 44%.120 The interval from diagnosis to stem-cell source, but survival after haploidentical transplantation
transplantation in this study did not impact survival. has been poor. More recently, engraftment was achieved in 3
A European protocol substituted irradiation with fludarabine for children using the St Jude protocol, but one ultimately rejected and
unrelated and mismatched family donors: 73% were estimated to mixed chimerism in the others required further immunosuppression
survive 2 years; while GVHD rates were relatively low, perhaps and donor lymphocyte infusions.124
due to absence of radiation damage, graft rejection occurred in
about one third of the older children and younger adults.117
Childrens Hospital of Milwaukee pioneered a rigorous condition- Conclusions and prospects
ing regimen of cytosine arabinoside, cyclophosphamide, and total
body irradiation, which produced long-term survival of about 50% The treatment of severe aplastic anemia, whether by allogeneic
with very little GVHD.122 Other single-institution protocols have stem-cell transplantation or immunosuppression, has improved
used a diversity of strategies to improve graft acceptance and dramatically over the last 25 years, and long-term survival of
reduce GVHD: T-cell depletion, CD34-cell purification, alemtu- more than 75% of patients can be anticipated with either
zumab, chemotherapy and monoclonal antibodies in combination; therapy.125 For transplantation, the immediate challenge is the
while almost exclusively enrolling small numbers of children and extension of stem-cell replacement to all patients, regardless of
still preliminary, survival and morbidity may rival results of age, with a histocompatible sibling, and to others who lack a
conventional sibling transplants. In current practice, unrelated family donor using alternative stem-cell sources. The ability to
transplant is offered for children who have failed a single course of achieve engraftment under these difficult circumstances may
immunosuppression and to adults who are refractory to multiple require conditioning regimens in which complications, particu-
courses of ATG and alternative therapies such as androgens. larly second malignancies, may not be apparent for many years.
Studies with longer follow-up of larger numbers of patients are More optimistically, donor selection based on high-resolution
crucial to establish the optimal conditioning regimen and to define histocompatibility typing may improve outcomes. The success
which patients will benefit and especially how early unrelated of umbilical cord-blood transplantations, with their low risk of
transplantation should be performed. GVHD, may be enhanced by larger pools and histocompatibility
Other stem-cell sources. HLA mismatching is better tolerated matching. For immunosuppression, many new drugs and biolog-
for umbilical cord transplantations, making them in theory widely ics have yet to be tested in aplastic anemia. Again, the costs of
applicable. Published data for this procedure in marrow failure is intensification need to be balanced against the benefits of higher
limited, and almost all recipients have been small children, due to hematologic response rates and lower rates of relapse and
BLOOD, 15 OCTOBER 2006 VOLUME 108, NUMBER 8 CURRENT CONCEPTS IN APLASTIC ANEMIA 2517

evolution. Repeated courses of immunosuppression offer the environment initiate and perpetuate the marrow destruction of
possibility of blood-count restitution to 75% to more than 90% aplastic anemia.
of patients, based on the range of published hematologic
recovery rates with initial horse ATG followed by rabbit ATG,
alemtuzumab, or cyclophosphamide. If residual stem-cell num-
Acknowledgments
bers are limiting, ex vivo expansion of hematopoiesis may be
possible, as for example using Hox box proteins. Quantitative We are thankful to colleagues from the Hematology Branch, Drs A.
and practical measurements of oligoclonal T-cell activity and of John Barrett, Cynthia Dunbar, Richard Childs, and Elaine Sloand,
hematopoietic stem-cell number and function would allow and in Europe, Professors Judith Marsh, Gerard Socie, and Andre
laboratory testing to guide treatment decisions. Ultimately, Tichelli for their careful reading of the paper and helpful criticisms.
definition of genetic risk factors, affecting hematopoietic-cell The authors apologize to their colleagues whose papers were
function and the immune response, will clarify how agents in the not cited in the bibliography due to space constraints.

References
1. Young NS. Aplastic anemia. In: Young NS, ed. 18. Feng X, Chuhjo T, Sugimori C, et al. Diazepam- relevance of the TNF-alpha promoter/enhancer
The Bone Marrow Failure Syndromes. Philadel- binding inhibitor-related protein 1: a candidate polymorphism in patients with aplastic anemia.
phia, PA: W. B. Saunders; 2000:1-46. autoantigen in acquired aplastic anemia patients Ann Hematol. 2002;81:566-569.
2. Kojima S, Nakao S, Tomonaga M, et al. Consen- harboring a minor populatoin of paroxysmal noc- 32. Dufour C, Capasso M, Svahn J, et al. Homozygo-
sus conference on the treatment of aplastic ane- turnal hemoglobinuria-type cells. Blood. 2004; sis for (12) CA repeats in the first intron of the hu-
mia. Int J Hematol. 2000;72:118-123. 104:2425-2431. man IFN-gamma gene is significantly associated
3. Marsh JCW. Management of acquired aplastic 19. Hinterberger W, Rowlings PA, Hinterberger- with the risk of aplastic anaemia in Caucasian
anaemia. Blood Rev. 2005;19:143-151. Fischer M, et al. Results of transplanting bone population. Br J Haematol. 2004;126:682-685.
marrow from genetically identical twins into pa- 33. Gidvani V, Ramkissoon S, Wong E, et al. Tumor
4. Nakao S, Feng X, Sugimori C. Immune patho- tients with aplastic anemia. Ann Intern Med.
physiology of aplastic anemia. Intl J Hematol. necrosis factor-alpha and interleukin-6 promoter
1997;126:116-122. gene polymorphisms in acquired bone marrow
2005;82:196-200.
20. Young NS. Hematopoietic cell destruction by im- failure syndromes [abstract]. Proceedings of the
5. Young NS. Acquired aplastic anemia. In: Young mune mechanisms in aquired aplastic anemia. 46th ASH Annual Meeting. 2004;104:12b. Ab-
NS, Gerson SL, High K, eds. Clinical Hematology. Semin Hematol. 2000;37:3-14. stract 3707.
Philadelphia, PA: Elsevier; 2006:136-157.
21. Sloand EM, Kim S, Maciejewski JP, et al. Intracel- 34. Solomou EE, Keyvanfar K, Young NS. T-bet, a
6. Schrezenmeier H, Bacigalupo A. Aplastic Ane- lular interferon- in circulating and marrow T cells Th1 transcription factor, is up-regulated in T cells
mia: Pathophysiology and Treatment. Cambridge, detected by flow cytometry and the response to from patients with aplastic anemia. Blood. 2006;
United Kingdom: Cambridge University Press; imunosuppressive therapy in patients with aplas- 107:3983-3991.
2000. tic anemia. Blood. 2002;100:1185-1191. 35. Xu JL, Nagasaka T, Nakashima N. Involvement of
7. Choudhry VP, Gupta S, Gupta M, Kashyap R, 22. Risitano AM, Maciejewski JP, Green S, et al. In- cytotoxic granules in the apoptosis of aplastic
Saxena R. Pregnancy associated aplastic ane- vivo dominant immune responses in aplastic anaemia. Br J Haematol. 2003;120:850-852.
mia: a series of 10 cases with review of literature. anaemia: molecular tracking of putatively patho-
Hematolgy. 2002;7:233-238. 36. Zeng W, Kajigaya S, Chen G, et al. Transcript
genetic T-cell clones by TCR beta-CDR3 se- profile of CD4 and CD8 cells from the bone
8. Lu J, Basu A, Melenhorst J, Young NS, Brown quencing. Lancet. 2004;364:355-364. marrow of acquired aplastic anemia patients. Exp
KE. Analysis of T-cell repertoire in hepatitis- 23. Maciejewski JP, Selleri C, Anderson S, Young NS. Hematol. 2004;32:806-814.
associated aplastic anemia. Blood. 2004;103: Fas antigen expression on CD34 human mar-
4588-4593. 37. Poggi A, Negrini S, Zocchi MR, et al. Patients
row cells is induced by interferon-gamma and with paroxysmal nocturnal hemoglobinuria have a
9. Morgan GJ, Alvares CL. Benzene and the hemo- tumor necrosis factor-alpha and potentiates cyto- high frequency of peripheral-blood T cells ex-
poietic stem cell. Chem Biol Interact. 2005;153- kine-mediated hematopoietic suppression in vitro. pressing activating isoforms of inhibiting super-
154:217-222. Blood. 1995;85:3183-3190. family receptors. Blood. 2005;106:2399-2408.
10. Young NS. Drugs and chemicals. In: Young NS, 24. Selleri C, Maciejewski JP, Sato T, Young NS. In- 38. Howe EC, Wlodarski M, Ball EJ, Rybicki L, Ma-
Alter BP, eds. Aplastic Anemia, Acquired and In- terferon- constitutively expressed in the stromal ciejewski JP. Killer immunoglobulin-like receptor
herited. Philadelphia, PA: W. B. Saunders; 1994: microenviroment of human marrow cultures medi- genotype in immune-mediated bone marrow
100-132. ates potent hematopoietic inhibition. Blood. 1996; failure syndromes. Exp Hematol. 2005;33:
11. Gerson WT, Fine DG, Spielberg SP, Sensenbren- 87:4149-4157. 1357-1362.
ner LL. Anticonvulsant-induced aplastic anemia: 25. Bloom ML, Wolk A, Simon-Stoos K, et al. A 39. Maciejewski JP, Selleri C, Tadatsugu S, Anderson
increased susceptibility to toxic drug metabolites mouse model of human immune mediated bone S, Young NS. A severe and consistent deficit in
in vitro. Blood. 1983;61:889-893. marrow failure. Exp Hematol. 2004;32:1163-1172. marrow and circulating primitive hematopoietic
12. Sutton JF, Stacey M, Kearsey SE, et al. In- 26. Chen J, Lipovsky K, Ellison FM, Calado RT, cells (long-term culture-initiating cells) in acquired
creased risk for aplastic anemia and myelodys- Young NS. Bystander destruction of hematopoi- aplastic anemia. Blood. 1996;88:1983-1991.
plastic syndrome in individuals lacking GSTT1 etic progenitor and stem cells in a mouse model 40. Schrezenmeier H, Jenal M, Herrmann F, Heimpel
gene. Pediatric Blood Cancer. 2004;42:122-126. of infusion-induced bone marrow failure. Blood. H, Raghavachar A. Quantitative analysis of cob-
13. Dufour C, Syahn J, Bacigalupo A, et al. Genetic 2004;104:1671-1678. blestone area-forming cells in bone marrow of
polymorphisms of CYP3A4, GSTT1, GSTM1, 27. Chen J, Young NS. A murine model of bone mar- patients with aplastic anemia by limiting dilution
GSTP1 and NQO1 and the risk of acquired idio- row failure mediated by disparity in minor histo- assay. Blood. 1996;88:4474-4480.
pathic aplastic anemia in Caucasian patients. compatibility antigens [abstract]. Blood. 2004; 41. Philpott NJ, Scopes J, Marsh JCW, Gordon-Smith
Haematologica. 2005;90:1027-1031. 106:12a. Abstract 132. EC, Gibson FM. Increased apoptosis in aplastic
14. Bacigalupo A. Aetiology of severe aplastic anae- 28. Nakao S, Takamatsu H, Chuhjo T, et al. Identifica- anemia bone marrow progenitor cells: possible
mia and outcome after allogeneic bone marrow tion of a specific HLA class II haplotype strongly pathophysiologic significance. Exp Hematol.
transplantation or immunosuppression. Eur J associated with susceptibility to cyclosporine- 1995;23:1642-1648.
Haematol. 1996;57(suppl 60):16-19. dependent aplastic anemia. Blood. 1994;84: 42. Kakagianni T, Giannakoulas NC, Thanopoulou E,
15. Kaufman DW, Kelly JP, Levy M, Shapiro S. The 4257-4261. et al. A probable role for trail-induced apoptosis in
Drug Etiology of Agranulocytosis and Aplastic 29. Maciejewski JP, Follmann D, Rivera CE, et al. the pathogenesis of marrow failure: implications
Anemia. New York, NY: Oxford; 1991. Increased frequency of HLA-DR2 in patients with from an in vitro model and from marrow of aplas-
16. Issaragrisil S, Kaufman D, Anderson T, et al. The paroxysmal nocturnal hemoglobinuria and PNH/ tic anemia patients. Leuk Res. 2006;30:713-721.
epidemiology of aplastic anemia in Thailand. aplastic anemia syndrome. Blood. 2001;98: 43. Zeng W, Chen G, Kajigaya S, et al. Gene expres-
Blood. 2006;107:1299-1307. 3513-3519. sion profiling in CD34 cells to identify differences
17. Hirano N, Butler MO, von Bergwelt-Baildon MS, 30. Peng J, Liu C, Zhu K, et al. The TNF2 allele is a between aplastic anemia patients and healthy
et al. Autoantibodies frequently detected in pa- risk factor to severe aplastic anemia independent volunteers. Blood. 2004;103:325-332.
tients with aplastic anemia. Blood. 2003;102: of HLA-DR. Hum Immunol. 2003;64:896-901. 44. Zeng W, Miyazato A, Chen G, Kajigaya S, Young
4567-4575. 31. Demeter J, Messer G, Schrezenmeier H. Clinical NS. Interferon--induced gene expression in
2518 YOUNG et al BLOOD, 15 OCTOBER 2006 VOLUME 108, NUMBER 8

CD34 cells: identification of pathologic cytokine- 62. Sloand EM, Rezvani K, Mainwaring L, et al. My- 77. Tichelli A, Socie G, Henry-Amar M, et al. Effec-
specific signature profiles. Blood. 2006;107: elodysplasia with trisomy 8 is associated with a tiveness of immunosuppressive therapy in older
167-175. cytotoxic CD8 T cell immune response to Wilms patients with aplastic anemia: The European
45. Ball SE, Gibson FM, Rizzo S, et al. Progressive tumor-1 protein (WT1) [abstract]. Blood. 2004; Group for Blood and Marrow Transplantation
telomere shortening in aplastic anemia. Blood. 104:138a. Abstract 474. Several Aplastic Anaemia Working Party. Ann In-
1998;91:3582-3592. 63. Sloand EM, Mainwaring L, Tarnowka MK, Barrett tern Med. 1999;130:193-201.
46. Brummendorf TH, Maciejewski JP, Young NS, AJ, Young NS. CD34 cells from patients with tri- 78. Tichelli A, Passweg J, Nissen C, et al. Repeated
Lansdorp PL. Telomere length in leukocyte sub- somy 8 express early apoptotic markers but do treatment with horse antilymphocyte globulin for
populations of patients with aplastic anemia. not undergo programmed cell death because of severe aplastic anaemia. Br J Haematol. 1998;
Blood. 2001;97:895-900. upregulation of anti-apoptotic proteins [abstract]. 100:393-400.
Blood. 2003;102:145a. Abstract 497.
47. Fogarty PF, Yamaguchi H, Wiestner A, et al. Late 79. Scheinberg P, Nunez O, Young NS. Retreatment
presentation of dyskeratosis congenita as appar- 64. Maciejewski JP, Risitano AM, Nunez O, Young with rabbit anti-thymocyte globulin and cyclospor-
ently acquired aplastic anaemia due to mutations NS. Distinct clinical outcomes for cytogenetic ab- ine for patients with relapsed or refractory severe
in telomerase RNA. Lancet. 2003;362: normalities evolving from aplastic anemia. Blood. aplastic anemia. Br J Haematol. 2006;133:
1628-1630. 2002;99:3129-3135. 622-627.
48. Yamaguchi H, Baerlocher GM, Lansdorp PL, et 65. Kojima S, Ohara A, Tsuchida M, et al. Risk factors 80. Socie G, Rosenfeld S, Frickhofen N, Gluckman
al. Mutations of the human telomerase RNA gene for evolution of acquired aplastic anemia into my- E, Tichelli A. Late clonal diseases of treated
(TERC) in aplastic anemia and myelodysplastic elodysplastic syndrome and acute myeloid leuke- aplastic anemia. Semin Hematol. 2000;37:
syndrome. Blood. 2003;102:916-918. mia after immunosuppressive therapy in children. 91-101.
49. Yamaguchi H, Calado RT, Ly H, et al. Mutations in Blood. 2002;100:786-790.
81. Jeng MR, Naidu PE, Rieman MD, et al. Granulo-
TERT, the gene for telomerase reverse transcrip- 66. Sloand EM, Yong A, Solomou EE, et al. Granulo-
cyte-macrophage colony stimulating factor and
tase, in aplastic anemia. N Engl J Med. 2005;352: cyte colony-stimulating factor preferentially stimu-
immunosuppression in the treatment of pediatric
1413-1424. lates proliferation of monosomy 7 cells bearing
acquired severe aplastic anemia. Pediatr Blood
50. Savage SA, Calado RT, Lansdorp PM, Chanock the isoform IV receptor. Proc Natl Acad Sci U S A.
Cancer. 2005;45:170-175.
SJ, Young NS. Genetic variation in telomeric re- In press.
82. Gluckman E, Rokicka-Milewska R, Hann I, et al.
peat binding factor 1 but not telomeric repeat 67. Frickhofen N, Rosenfeld SJ. Immunosuppressive
Results and follow-up of a phase III randomized
binding factor 2 are associated with aplastic ane- treatment of aplastic anemia with antithymocyte
study of recombinant human-granulocyte stimu-
mia. Exp Hematol. 2006;34:664-671. globuilin and cyclosporine. Semin Hematol. 2000;
lating factor as support for immunosuppressive
51. Thornley I, Dror Y, Sung L, Wynn RF, Freedman 37:56-68.
therapy in patients with severe aplastic anaemia.
MH. Abnormal telomere shortening in leucocytes 68. Frickhofen N, Heimpel H, Kaltwasser JP, Br J Haematol. 2002;119:1075-1082.
of children with Shwachman-Diamond syndrome. Schrezenmaier H. Antithymocyte globulin with
Br J Haematol. 2002;117:189-192. or without cyclosporin A: 11-year follow-up of a ran- 83. Brodsky RA, Sensenbrenner LL, Smith BD, et al.
domized trial comparing treatments of aplastic ane- Durable treatment-free remission after high dose
52. Calado RT, Bruno T, Wilkerson KL, Young NS.
mia. Blood. 2003;101:1236-1242. cyclophosphamide therapy for previously severe
Evidence for T-cell oligoclonal expansion in
aplastic anemia. Ann Intern Med. 2001;135:
aplastic anemia associated with telomerase com- 69. Bacigalupo A, Bruno B, Saracco P, et al. Antilym- 477-483.
plex mutations: pathophysiological and clinical phocyte globuilin, cyclosporine, prednisolone,
implications [abstract]. Blood. 2005;106:307a. and granulocyte colony-stimulating factor for se- 84. Brodsky RA, Chen A, Dorr DM, Brodsky I, Jones
Abstract 1052. vere aplastic anemia: an update of the GITMO/ RJ. High dose cyclophosphamide (CY) for severe
53. Young NS. Paroxysmal nocturnal hemoglobin- EBMT study on 100 patients: European Group for aplastic anemia (SAA): safety and long term fol-
uria: current issues in pathophysiology and treat- Blood and Marrow Transplantation (EBMT) Work- low-up [abstract]. Blood. 2005;106:41a-42a. Ab-
ment. Curr Hematol Rep. 2005;4:103-109. ing Party on Severe Aplastic Anemia and the stract 129.
Gruppo Italiano Trapianti di Midolio Osseo 85. Tisdale JF, Dunn DE, Geller NL, et al. High-dose
54. Sugimori C, Chuhjo T, Feng X, et al. Minor popu-
(GITMO). Blood. 2000;95:1931-1934. cyclophosphamide in severe aplastic anemia: a
lation of CD55-CD59 blood cells predicts re-
sponse to immunosuppressive therapy and prog- 70. Rosenfeld S, Follman D, Nunez O, Young NS. randomized trial. Lancet. 2000;356:1554-1559.
nosis in patients with aplastic anemia. Blood. Antithymocyte globulin and cyclosporine for se- 86. Tisdale JF, Maciejewski JP, Nunez O, Rosenfeld
2006;107:1308-1314. vere aplastic anemia: association between hema- SJ, Young NS. Late complications following treat-
55. Takami A, Zeng W, Wang H, Matsuda T, Nakao S. tologic response and long-term outcome. JAMA. ment for severe aplastic anemia (SAA) with high-
Cytotoxicity against lymphoblastoid cells medi- 2003;289:1130-1135. dose cyclophosphamide (Cy): follow-up of a ran-
ated by a T-cell clone from an aplastic anaemia 71. Kojima S, Hibi S, Kosaka Y, et al. Immunosup- domized trial. Blood. 2002;100:4668-4670.
patient: role of CD59 on target cells. Br J Haema- pressive therapy using antithymocyte globulin, 87. Di Bona E, Rodeghiero F, Bruno B, et al. Rabbit
tol. 1999;107:791-796. cyclosporine, and danazol with or without human antithymocyte globulin (r-ATG) plus cyclosporine
56. Karadimitris A, Notaro R, Koehne G, Roberts granulocyte colony-stimulating factor in children
and granulocyte colony stimulating factor is an
IAG, Luzzatto L. PNH cells are as sensitive to with acquired aplastic anemia. Blood. 2000;96:
effective treatment for aplastic anaemia patients
T-cell-mediated lysis as their normal counter- 2049-2054.
unresponsive to a first course of intensive immu-
parts: implications for the pathogenesis of parox- 72. Scheinberg P, Nunez O, Wu C, Young NS. Treat- nosuppressive therapy. Gruppo Italiano Trapianto
ysmal nocturnal haemoglobinuria. Brit J Haema- ment of severe aplastic anaemia with combined di Midollo Osseo (GITMO). Br J Haematol. 1999;
tol. 2000;111:1158-1163. immunosuppression: antithymocyte globulin, 107:330-334. (Erratum in: Br J Haematol. 2000;
57. Chen G, Kirby M, Zeng W, Young NS, Maciejew- ciclosporin, and mycophenolate mofetil. Br J 108:461.)
ski JP. Superior growth of glycophosphatidy li- Haematol. 2006;133:606-611.
88. Gupta V, Gordon-Smith EC, Cook G, et al. A third
nositol-anchored protein-deficient progenitor cells 73. Marsh J, Schrezenmeier H, Marin P, et al. Pro- course of anti-thymocyte globulin in aplastic
in vitro is due to the higher apoptotic rate of pro- spective randomized multicenter study comparing anaemia is only beneficial in previous respond-
genitors with normal phenotype in vivo. Exp He- cyclosporin alone versus the combination of anti- ers. Br J Haematol. 2005;129:110-117.
matol. 2002;30:774-782. thymocyte globulin and cyclosporin for treatment
of patients with nonsevere aplastic anemia: a re- 89. Howard SC, Naidu PE, Hu XJ, et al. Natural his-
58. Chen G, Zeng W, Maciejewski JP, et al. Differen-
port from the European Blood and Marrow Trans- tory of moderate aplastic anemia in children. Pe-
tial gene expression profile in hematopoietic pro-
plant (EBMT) Severe Aplastic Anemia Working diatr Blood Cancer. 2004;43:545-551.
genitors form paroxysmal nocturnal hemoglobin-
uria patients reveals an apoptosis/immune Party. Blood. 1999;93:2191-2195. 90. Maciejewski JP, Sloand EM, Nunez O, Boss C,
response in normal phenotype cells. Leukemia. 74. Raghavachar A, Kolbe K, Hoffken K, et al. A ran- Young NS. Recombinant humanized anti-IL-2
2005;19:862-868. domized trial of standard immunosuppression receptor antibody (Daclizumab) produces re-
59. Hanaoka N, Kawaguchi T, Horikawa K, et al. Im- versus cyclosporine and filgastrim in severe sponses in patients with moderate aplastic ane-
munoselection by natural killer cells of PIGA mu- aplastic anemia [abstract]. Blood. 1997;90:439a. mia. Blood. 2003;102:3584-3586.
tant cells missing stress-inducible ULBP. Blood. Abstract 1951. 91. Calado RT, Yewdell WT, Wilkerson KL, Kajigaya
2006;107:1184-1191. 75. Fuhrer M, Rampf U, Baumann I, et al. Immuno- S, Young NS. Sex hormones up-regulate telomer-
60. Maciejewski JP, Selleri C. Evolution of clonal cy- suppressive therapy for aplastic anemia in chil- ase activity of normal human hematopoietic cells
togenetic abnormalities in aplastic anemia. Leuk dren: a more severe disease predicts better sur- and restore telomerase activity in carriers of te-
Lymphoma. 2004;45:433-440. vival. Blood. 2005;106:2102-2104. lomerase complex mutations [abstract]. Blood.
2005;106:641a. Abstract 2276.
61. Sloand EM, Fuhrer M, Risitano A, et al. Preferen- 76. Fang JP, Xu HG, Huang SL, Chen C, Huang K.
tial suppression of trisomy 8 versus normal hema- Immunosuppressive treatment of aplastic anemia 92. Passweg JR, Perez WS, Eapen M, et al. Bone
topoietic cell growth by autologous lymphocytes in Chinese children with antithymocyte globulin marrow transplants from mismatched related and
in patients with trisomy 8 myelodysplastic syn- and cyclosporine. Pediatr Hematol Oncol. 2006; unrelated donors for severe aplastic anemia.
drome. Blood. 2005;106:841-851. 23:45-50. Bone Marrow Transplant. 2006;37:641-649.
BLOOD, 15 OCTOBER 2006 VOLUME 108, NUMBER 8 CURRENT CONCEPTS IN APLASTIC ANEMIA 2519

93. Reiter E, Keil F, Brugger S, et al. Excellent long- mide and antithymocyte globulin as a conditioning ditioning regimen for unrelated bone marrow
term survival after allogeneic marrow transplanta- regimen for allogeneic marrow transplantation in pa- transplantation in severe aplastic anemia. Bone
tion in patients with severe aplastic anemia. Bone tients with aplastic anaemia: a long-term follow-up. Marrow Transplant. 2004;34:939-943.
Marrow Transplant. 1997;19:1191-1196. Br J Haematol. 2005;130:747-751. 116. Mao P, Wang S, Wang S, et al. Umbilical cord
94. Locatelli F, Bruno B, Zecca M, et al. Cyclosporin 105. Storb R, Blume KG, ODonnell MR, et al. Cyclo- blood transplant for adult patients with severe
A and short-term methotrexate versus cyclo- phosphamide and antithymocyte globulin to con- aplastic anemia using anti-lymphocyte globulin
sporin A as graft versus host disease prophylaxis dition patients with aplastic anemia for allogeneic and cyclophosphamide as conditioning therapy.
in patients with severe aplastic anemia given allo- marrow transplantations: the experience in four Bone Marrow Transplant. 2004;33:33-38.
geneic bone marrow transplantation from an centers. Biol Blood Marrow Transplant. 2001;7: 117. Bacigalupo A, Locatelli F, Lanino E, et al. Fludara-
HLA-identical sibling: results of a GITMO/EBMT 39-44. bine, cyclophosphamide and anti-thymocyte
randomized trial. Blood. 2000;96:1690-1697. 106. Gomez-Almaguer D, Vela-Ojeda J, Jaime-Perez globulin for alternative donor transplants in ac-
95. Kim I, Yoon SS, Park S, Kim BK, Kim NK. The JC, et al. Allografting in patients with severe, re- quired severe aplastic anemia: a report from the
treatment of severe aplastic anemia: outcomes of fractory aplastic anemia using peripheral blood EBMT-SAA Working Party. Bone Marrow Trans-
bone marrow transplantation and immunosup- stem cells and a fludarabine-based conditioning plant. 2005;36:947-950.
pressive therapy in a single institution of Korea. regimen: the Mexican experience. Am J Hematol. 118. Bunin N, Aplenc R, Iannone R, et al. Unrelated
J Korean Med Sci. 2003;18:365-371. 2006;81:157-161. donor bone marrow transplantation for children
96. Ahn MJ, Choi JH, Lee YY, et al. Outcome of adult 107. Horowitz MM. Current status of allogeneic bone with severe aplastic anemia: minimal GVHD and
severe or very severe aplastic anemia treated marrow transplantation in acquired aplastic ane- durable engraftment with partial T-cell depletion.
with immunosuppressive therapy compared with mia. Semin Hematol. 2000;37:30-42. Bone Marrow Transplant. 2006;37:143-149.
bone marrow transplantation: multicenter trial. Intl 108. Srinivasan R, Takahashi Y, McCoy JP, et al. Over- 119. Lee J-H, Choi S-J, Lee J-H, et al. Non-total body
J Hematol. 2003;78:133-138. coming graft rejection in heavily transfused and irradiation containing preparative regimen in alter-
97. Kroger N, Zabelina T, Renges H, et al. Long-term allo-immunised patients with bone marrow failure native donor bone marrow transplantation for se-
follow-up of allogeneic stem cell transplantation in syndromes using fludarabine-based haematopoi- vere aplastic anemia. Bone Marrow Transplant.
patients with severe aplastic anemia after condi- etic cell transplantation. Br J Haematol. 2006; 2005;35:755-761.
tioning with cyclophosphamide plus antithymo- 133:305-314. 120. Deeg HJ, ODonnell M, Tolar J, et al. Optimization
cyte globulin. Ann Hematol. 2002;81:627-631. 109. Deeg HJ, Leisenring W, Storb R, et al. Long-term of conditioning for marrow transplantation from
98. Ades L, Mary JY, Robin M, et al. Long-term out- outcome after marrow transplantation for severe unrelated donors for patients with aplastic anemia
come after bone marrow transplantation for se- aplastic anemia. Blood. 1998;91:3637-3645. after failure of immunosuppressive therapy.
vere aplastic anemia. Blood. 2004;103: 110. Gupta V, Ball SE, Sage D, et al. Marrow trans- Blood. 2006;108:1485-1491.
2490-2497. plants from matched unrelated donors for aplastic 121. Deeg HJ, Seidel K, Casper J, et al. Marrow trans-
99. Dulley FL, Vigorito AC, Aranha FJP, et al. Addition anaemia using alemtuzumab, fludarabine and plantation from unrelated donors for patients with
of low-dose busulfan to cyclophosphamide in cyclophosphamide based conditioning. Bone severe aplastic anemia who have failed immuno-
aplastic anemia patients prior to allogeneic bone Marrow Transplant. 2005;35:467-471. suppressive therapy. Biol Blood Marrow Trans-
marrow transplantation to reduce rejection. Bone 111. Kojima S, Inaba J, Yoshimi A, et al. Unrelated do- plant. 1999;5:243-252.
Marrow Transplant. 2004;33:9-13. nor marrow transplantation in children with se- 122. Margolis DA, Casper JT. Alternative-donor hema-
100. Bai LY, Chiou TJ, Liu JH, et al. Hematopoietic vere aplastic anaemia using cyclophosphamide, topoietic stem-cell transplantation for severe
stem cell transplantation for severe aplastic ane- anti-thymocyte globulin and total body irradiation. aplastic anemia. Semin Hematol. 2000;37:43-55.
mia: experience of an institute in Taiwan. Ann He- Br J Haematol. 2001;114:706-711. 123. Rubinstein P, Carrier C, Scaradavou A, et al. Out-
matol. 2004;83:38-43. 112. Vassiliou GS, Webb DKH, Pamphilon D, Knapper comes among 562 recipients of placental-blood
101. Abdelkefi A, Ben Othman T, Ladeb S, et al. Bone S, Veys PA. Improved outcome of alternative do- transplants from unrelated donors. N Engl J Med.
marrow transplantation for patients with acquired nor bone marrow transplantation in children with 1997;339:1565-1576.
severe aplastic anemia using cyclophosphamide severe aplastic anaemia using a conditioning 124. Woodard P, Cunningham JM, Benaim E, et al.
and antithymocyte globulin: the experience from regimen containing low-dose total body irradia- Effective donor lymphohematopoietic reconstitu-
a single center. Hematol J. 2003;4:208-213. tion, cyclophosphamide and Campath. Br J tion after haploidentical CD34-selected hemato-
102. Kim HJ, Park CY, Park YH, et al. Successful allo- Haematol. 2001;114:701-705. poietic stem cell transplantation in children with
geneic hematopoietic stem cell transplantation 113. Kojima S, Matsuyama T, Kato S, et al. Outcome refractory severe aplastic anemia. Bone Marrow
using triple agent immunosuppression in severe of 154 patients with severe aplastic anemia who Transplant. 2004;33:411-418.
aplastic anemia patients. Bone Marrow Trans- received transplants from unrelated donors: the 125. Bacigalupo A, Brand R, Oneto R, et al. Treatment
plant. 2003;31:79-86. Japan Marrow Donor Program. Blood. 2002;100: of acquired severe aplastic anemia: bone marrow
103. Gupta V, Ball SE, Yi Q, et al. Favorable effect on 799-805. transplantation compared with immunosuppres-
acute and chronic graft-versus-host disease with 114. Benesch M, Urban C, Sykora KW, et al. Trans- sive therapy: The European Group for Blood and
cyclophosphamide and in vivo anti-CD52 mono- plantation of highly purified CD34 progenitor Marrow Transplantation experience. Semin He-
clonal antibodies for marrow transplantation from cells from alternative donors in children with re- matol. 2000;37:69-80.
HLA-identical sibling donors for acquired aplastic fractory severe aplastic anaemia. Br J Haematol. 126. Ehrlich P. Ueber einem Fall von Anamie mit Be-
anemia. Biol Blood Bone Marrow Transplant. 2004;125:58-63. merkungen uber regenerative Veranderungen
2004;10:867-876. 115. Kang HJ, Shin HY, Choi HS, Ahn HS. Fludara- des Knochenmarks. Charite-Annalen. 1888;13:
104. Kahl C, Leisenring W, Deeg HJ, et al. Cyclophospha- bine, cyclophosphamide plus thymoglobulin con- 300-309.
Science in medicine

Sickle cell disease: old discoveries,


new concepts, and future promise
Paul S. Frenette and George F. Atweh

Division of Hematology/Oncology, Mount Sinai School of Medicine, New York, New York, USA.

The discovery of the molecular basis of sickle cell disease was an important landmark in
molecular medicine. The modern tools of molecular and cellular biology have refined our
understanding of its pathophysiology and facilitated the development of new therapies. In
this review, we discuss some of the important advances in this field and the impediments that
limit the impact of these advances.

Historical perspective Natural history of SCD


Sickle cell disease (SCD) was first described in 1910, in a den- During the time of these pioneering laboratory investigations,
tal student who presented with pulmonary symptoms (1). Her- further clinical observations brought to light the wide-ranging
rick coined the term sickle-shaped to describe the peculiar manifestations of SCD. In retrospect, it is clear that no amount
appearance of the rbc of this patient (Figure 1). However, given of anecdotal reports would have substituted for the tremendous
the patients symptoms, he was not sure at the time whether the contributions of the study of the natural history of this disease
blood condition was a disease sui generis or a manifestation of known as the Cooperative Study of Sickle Cell Disease (CSSCD).
another disease (2). In the next 15 years, several similar cases The CSSCD was commissioned in 1978 by the National Heart,
were described, supporting the idea that this was a new disease Lung, and Blood Institute to characterize prospectively the clini-
entity and providing enough evidence for a preliminary clinical cal course of SCD in a cohort of more than 4,000 patients from 23
and pathological description (3). Shortly thereafter, Hahn and centers across the United States (12). The CSSCD cohort defined
Gillespie suggested that anoxia caused rbc sickling by demon- the incidence and characteristics of virtually every known compli-
strating that shape changes could be induced by saturating a cell cation of SCD. One of the earliest contributions of the CSSCD was
suspension with carbon dioxide (4). Scriver and Waugh, in exper- to identify the persistent high mortality rate of severe pneumococ-
iments that would undoubtedly not receive institutional review cal infections in children with SCD, despite the widespread use of
board approval today, proved this concept in vivo by inducing pneumococcal vaccination. This led to the development of the land-
venous stasis in a finger using a rubber band. They showed that mark Prophylactic Penicillin Study (PROPS), which highlighted the
stasis-induced hypoxia dramatically increased the proportion of importance of neonatal screening for SCD followed by prophylactic
sickle-shaped cells from approximately 15% to more than 95% penicillin therapy in children between the ages of 4 months and 5
(5). These seminal studies were noted by Linus Pauling, who was years (13). Other CSSCD studies that evaluated the mortality of the
the first to hypothesize in 1945 that the disease might origi- disease found that 50% of patients died before the fifth decade, and
nate from an abnormality in the hemoglobin molecule (6). This most of those who died did not have overt chronic organ failure but
hypothesis was validated in 1949 by the demonstration of the succumbed during an episode of acute pain, acute chest syndrome,
differential migration of sickle versus normal hemoglobin as or stroke (14). Prospective follow-up made it possible to deter-
assessed by gel electrophoresis (7). That same year, the autosomal mine the incidence (approximately 13 per 100 patient-years), risk
recessive inheritance of the disease was elucidated (8). Around factors, presentation, and prognosis of the acute chest syndrome
the same time, Watson et al. predicted the importance of fetal (15, 16). Parallel studies established that approximately 11% of
hemoglobin (Hb F) by suggesting that its presence could explain patients with SCD will go on to develop a clinically apparent stroke
the longer period necessary for sickling of newborn rbc com- by the age of 20 years, and 24% by the age of 45 years (17). This high
pared with those from mothers who had sicklemia (9). Ingram risk of a life-threatening complication generated the momentum
and colleagues demonstrated shortly thereafter that the mutant for the Stroke Prevention Trial in Sickle Cell Anemia (STOP), which
sickle hemoglobin (Hb S) differed from normal hemoglobin A demonstrated the benefit of prophylactic transfusions in prevent-
by a single amino acid (10). This was followed by studies that ing a first stroke in patients with an elevated flow rate by transcra-
analyzed the structure and physical properties of Hb S, which nial Doppler ultrasonography (18). Moreover, the observation that
formed intracellular polymers upon deoxygenation (11). These more than 50% of patients with SCD have at least one crisis per
studies placed SCD at the leading edge of investigations to eluci- year and the association between multiple pain episodes and early
date the molecular basis of human diseases. death in young adults (14, 19) provided the impetus for the Mul-
ticenter Study of Hydroxyurea (MSH), which is discussed below.
The CSSCD also shed some important light on the incidence and
Nonstandard abbreviations used: BMT, bone marrow transplantation; CSSCD,
Cooperative Study of Sickle Cell Disease; Hb F, fetal hemoglobin; Hb S, sickle hemo- risk of other complicating conditions, including alloimmunization
globin; MSH, Multicenter Study of Hydroxyurea; SCD, sickle cell disease. (20), pregnancy (21), and surgery (22). Improvements in survival of
Conflict of interest: The authors have declared that no conflict of interest exists. patients with SCD have been associated with a marked increase in
Citation for this article: J. Clin. Invest. 117:850858 (2007). doi:10.1172/JCI30920. the incidence of chronic organ dysfunction, especially pulmonary

850 The Journal of Clinical Investigation http://www.jci.org Volume 117 Number 4 April 2007
science in medicine

Homozygosity for the sickle mutation (i.e., HbSS disease) is respon-


sible for the most common and most severe variant of SCD. Several
other genetic variants of SCD result from the interaction of different
mutations of the human b-globin genes (Table 1). When the bS gene
interacts with the bC gene, the resulting sickling disorder known as
HbSC disease is typically very mild (14). When a bS gene interacts
with a b-thalassemia gene (a mutant b-globin gene that either
fails to produce normal b-globin mRNA or produces it at mark-
edly decreased levels), the severity of the resulting sickling disorder
depends on the severity of the coinherited b-thalassemia mutation.
When the coinherited b-thalassemia gene is completely inactive (i.e.,
b0-thalassemia), the resulting sickling disorder known as Sb0-thalas-
Figure 1 semia tends to be of severity similar to that of homozygous HbSS
Sickle erythrocytes. Peripheral blood smear from a patient with SCD disease (27). In contrast, when the coinherited b-thalassemia gene is
obtained during a routine clinic visit. The smear shows classical partially active (i.e., b+-thalassemia), the resulting sickling disorder
sickle-shaped (arrows) and various other misshaped erythrocytes
known as Sb+-thalassemia can have a spectrum of clinical severity.
(arrowheads). The image was obtained from an air-dried smear using
differential interference contrast (DIC) microscopy with an Olympus If the b+-thalassemia mutation is mild, as is commonly the case in
BX61WI work station equipped with a LUMPlanFI 60 numerical aper- people of African descent, the resulting Sb+-thalassemia tends to be
ture 0.90 objective (Olympus) and a CoolSnap HQ camera (6.6 mm2 clinically mild. In contrast, if the b+-thalassemia mutation is severe,
pixel, 1,392 1,040 pixel format) (Roper Scientific). Scale bar: 10 mm. as is commonly the case in the Mediterranean populations, the clini-
cal sickling disorder tends to be moderate (28).

hypertension (23). The lessons learned from the study of the natu- Advances in the pathophysiology of SCD
ral history of SCD underscored the fact that this disease, which is In addition to the obvious shape changes that result from the
caused by a single missense mutation in a gene whose expression formation of intracellular hemoglobin polymers, the polymers
is restricted to the hematopoietic system, can have wide-ranging can have a direct impact on the rbc plasma membrane, leading
manifestations and complications that affect every aspect of the to the extracellular exposure of protein epitopes and glycolipids
life of afflicted patients. that are normally found inside the cell (Figure 3). These chang-
es and the aberrant expression of adhesion molecules on stress
Genetics of SCD reticulocytes likely explain the increased adherence of sickle rbc
As mentioned above, pioneering studies by Pauling et al. estab- to vascular endothelium. Although the increased propensity of
lished that SCD results from a defect in the hemoglobin molecule sickle rbc to stick to one another was noted many years before
(7). During the same year, the mode of inheritance of the disease the field of cell adhesion was even conceptualized at the molecu-
was shown to be autosomal recessive (8). The sickle mutation was lar level, pioneering independent studies by Hebbel, Hoover, and
characterized several years later by Ingram et al. as a glutamine-to- their colleagues demonstrated that sickle rbc were more apt than
valine substitution at the sixth residue of the b-globin polypeptide normal rbc to adhere to endothelial cells in vitro (29, 30). Dur-
(24). Several decades later, the human globin genes were cloned, ing the two decades that followed, multiple studies implicated
their DNA sequence was determined, the organization of the glo- virtually all major adhesion pathways in the interactions between
bin gene clusters was characterized, and a great deal of insight was sickle cells and endothelial cells. These pathways include those
provided into the mechanisms of their regulated expression (25). involving the integrins (a4b1, aVb3) (3133) and their receptors;
Human hemoglobin is a tetrameric molecule that consists of two immunoglobulin family members (VCAM-1, ICAM-4) (34, 35);
pairs of identical polypeptide subunits, each encoded by a different the endothelial selectins (36, 37); soluble adhesion proteins such
family of genes. The human a-like globin genes (, a1, and a2) are as thrombospondin (38), fibrinogen (39), fibronectin (40), von
located on chromosome 16, and the b-like globin genes (, Gg, Ag, Willebrand factor (41, 42); and other exposed membrane compo-
d, and b) are located on chromosome 11. Interestingly, the genes nents such as Band3 and sulfated glycolipids (43, 44). Thus, inas-
are present on both chromosomes in the same order in which they much as sickle adhesion to the endothelium plays a role in sickle
are expressed during development (Figure 2). During fetal life, the cell vasoocclusion, the presence of such diverse mechanisms of
predominant type of hemoglobin is Hb F (a2g2). During the postna- adhesion presents an enormous challenge for delineating physi-
tal period, Hb F is gradually replaced by Hb A (a2b2). Hb A2 (a2d2) ologically relevant therapeutic targets. Interestingly, recent stud-
is a minor adult-type hemoglobin that accounts for less than 2.5% ies have suggested that targeting a specific adhesion pathway may
of the circulating hemoglobin in normal individuals in adult life. be sufficient to reduce vasoocclusion (33, 37, 45).
Upon completion of the switch from Hb F to Hb A, patients with The studies of the pathophysiology of SCD have been facilitat-
disorders of the b-globin genes start manifesting the clinical fea- ed by the development of a number of mouse models that express
tures of their diseases. The prospect of therapeutic reactivation of either a mixture of mouse globins with Hb S, a super-sickling
Hb F production in adult life (see Advances in the therapy of SCD) hemoglobin (e.g., SAD, NY1, S-Antilles mice), or human globin
has been in large part responsible for the tremendous interest in the chains exclusively (e.g., Berkeley, NY1KO mice) (46). The severity
elucidation of the molecular mechanisms of the switch from fetal to of the phenotype of these transgenic mice depends on the pres-
adult hemoglobin production. This field of investigation, which has ence of mouse globin chains, the mean corpuscular hemoglo-
recently been reviewed (26), led to approval by the FDA of the use of bin concentration (MCHC), and the presence of human Hb F.
hydroxyurea for the treatment of patients with SCD. Although mice that express the human globin chains exclusively

The Journal of Clinical Investigation http://www.jci.org Volume 117 Number 4 April 2007 851
science in medicine

Figure 2
Chromosomal organization of the a- and b-
globin gene clusters. (A) The genes of the
b-globin gene cluster (, Gg, Ag, d, and b) are
present on chromosome 11 in the same order
in which they are expressed during develop-
ment. The blocus control region (bLCR) is a
major regulatory element located far upstream
of the genes of the cluster that is necessary
for the high level of expression of those genes.
(B) The genes of the a-globin gene cluster
(, a1, and a2) are present on chromosome
16, also in the same order in which they are
expressed during development. HS-40 is a
major regulatory element located far upstream
of the genes of the cluster that is necessary
for their high level of expression. (C) During
fetal life, Hb F (a2g2) is the predominant type
of hemoglobin. Hemoglobin switching refers
to the developmental process that leads to
the silencing of g-globin gene expression and
the reciprocal activation of adult b-globin gene
expression. This results in the replacement of
Hb F by Hb A (a2b2) as the predominant type
of hemoglobin in adult life. Figure modified
from ref. 128.

have a much more severe phenotype that closely mimics the NF-kB and activator protein1. These transcription factors in turn
major features of the human disease, all transgenic sickle mice induce the expression of E-selectin, VCAM-1, and ICAM-1 and the
exhibit pathological features of the disease, either spontaneously recruitment of adherent leukocytes in venules (51).
or in an inducible manner (46, 47). The presence of adherent leukocytes in small postcapillary venules
As in the human disease, there is a clear role for inflammatory is emerging as a key factor that contributes to vasoocclusion during
mediators in the pathogenesis of disease in murine models of SCD SCD (57, 58) (Figure 4). Leukocytes are large cells that are rigid and
(4852). Inflammation is likely triggered by the abnormal eryth- not easily deformed as a result of a high viscoelastic coefficient (59).
rocyte membrane and the presence of chronic hemolysis. Dense These physical properties endow leukocytes with a greater potential
sickle cells that become dehydrated after several rounds of sickling than discoid or sickle-shaped rbc (which would lie flat along the
expose their annexin Vbinding phosphatidyl serine on the outer endothelium) to promote vascular obstruction. It has been known
layer of the plasma membrane (53, 54). These negatively charged for many years, as a result of microdynamic measurements, that leu-
glycolipids can activate the coagulation cascade (55), leading to kocyte recruitment in cat mesenteric venules can lead to a decrease
the generation of tissue factor and thrombin, which in turn pro- in the effective diameter of the blood vessels and an increase in blood
mote the inflammatory response. Chronic hemolysis, on the other flow resistance due to obstruction of the lumen by wbc (60). In addi-
hand, leads to the release of plasma-free hemoglobin, which can tion, sickle rbc have been shown to interact directly with adherent
scavenge NO and result in endothelial dysfunction (56). Moreover, wbc in a mouse model of vasoocclusion induced by surgical trauma
the release of heme iron from lysed rbc is a major cause of oxidative and TNF-a, leading to reduced blood flow and death of the mouse
stress that can induce redox-sensitive transcription factors such as (49). Leukocyte adhesion was shown to be critical in this process,

852 The Journal of Clinical Investigation http://www.jci.org Volume 117 Number 4 April 2007
science in medicine

Table 1
Genotypes and phenotypes of different sickling disorders

Genotype Interacting genes Typical clinical severityA % of Hb type/total Hb in a typical patientA,B


Hb S Hb A Hb F Hb C Hb A2
HbAA (normal) b and b None 96% 2% 2%
HbSS bS and bS Severe 95% 3% 2%
HbSC bS and bC Mild 48% 3% 47% 2%
HbSb0 bS and b0-thalassemia Severe 93% 2% 5%
HbSb+ bS and b+-thalassemia Moderate 85% 6% 5% 4%
(severe thalassemia mutation)
HbSb+ bS and b+-thalassemia Mild 70% 23% 3% 4%
(mild thalassemia mutation)
ATypical refers to the most common presentation of a particular sickling genotype. It should be noted that in many patients, the genotype does not accu-
rately predict the clinical phenotype. BAssessed by gel electrophoresis. Hb A2, minor adult hemoglobin.

since flow reductions and death were prevented in mice deficient in become possible to make definitive diagnoses of the different sick-
both P- and E-selectins, key adhesion molecules mediating leuko- ling disorders during the first trimester by analyzing fetal DNA
cyte rolling and adhesion (49, 61). Moreover, in vitro studies have obtained by chorionic villous biopsy (67). The molecular diagnos-
revealed that human sickle cells can directly bind to activated neu- tic technology is being pushed further to allow the diagnosis to be
trophils (62). Recent studies using high-speed fluorescence digital made from a small number of fetal cells that can be harvested from
videomicroscopy suggest that most interactions between rbc and the maternal circulation (68).
wbc in vivo are mediated by adherent neutrophils (63). As leukocyte
adhesion plays a key role in vasoocclusion, it thus offers an attrac- Advances in the therapy of SCD
tive therapeutic target for this disease and is consistent with several Induction of Hb F. A large number of epidemiological, clinical, and
clinical studies derived from the CSSCD cohort in which high leu- laboratory observations have converged to support the notion that
kocyte counts correlated with mortality (14), acute chest syndrome Hb F administration can ameliorate the clinical severity of SCD.
(16), stroke (17, 64, 65), and poor prognosis later in life when identi- Patients with SCD from the eastern provinces of Saudi Arabia (69)
fied in infants with SCD (66). and from India (70) typically have a very mild sickling disorder
associated with high levels of Hb F. Furthermore, the CSSCD iden-
Advances in the diagnosis of SCD tified Hb F as a prognostic factor for several sickle cell complica-
The diagnosis of SCD is usually simple and rarely poses a major tions, including painful events (19), acute chest syndrome (16),
challenge. Although the sickle-shaped rbc that gave the disease its and death (14). Elegant laboratory studies conducted many years
name are not always present in the patients blood film, the char- earlier had demonstrated that Hb F interferes with the polym-
acteristic migration of Hb S by gel electrophoresis is sufficient to erization of deoxygenated Hb S in vitro (71). Based on all these
make a diagnosis of a sickling disorder. Gel electrophoresis allows observations, it was proposed that pharmacological induction
a definitive diagnosis of some but not all sickling genotypes. For of Hb F production may be an effective therapeutic strategy for
example, when Hb A is present in the blood of a patient with SCD ameliorating the severity of SCD. When the different globin genes
at a lower level than Hb S, this is indicative of HbSb+-thalassemia were cloned in the late 1970s (72) and the mechanisms responsible
(Table 1). However, the distinction between HbSb0-thalassemia for their regulation were elucidated during the 1980s (73, 74), it
and homozygous HbSS can be much more challenging to make became clear that epigenetic factors such as DNA methylation and
when no Hb A is detected by gel electrophoresis. In such situations, histone acetylation played important roles in the developmental
the diagnosis of HbSb0-thalassemia is suggested by an elevated Hb regulation of globin gene expression (7577). Thus, it was pro-
A2 level and a low mean corpuscular volume. Fortunately, there posed that pharmacological agents that alter the epigenetic con-
are a number of excellent reference laboratories in the US where figuration of the g-globin genes may provide a viable therapeutic
a definitive molecular diagnosis can be made in essentially every approach to the induction of Hb F.
patient with a hemoglobin disorder. 5-Azacytidine. 5-Azacytidine was the first agent to be used to
The major challenge in the diagnosis of sickling disorders is to induce Hb F expression via epigenetic silencing of the g-globin
identify the disease during the prenatal period, at a time when such genes in adult life. The rationale for this approach was based on
information would be critically important in enabling a couple at the discovery that the actively transcribed adult b-globin genes are
risk to make an informed decision about potential termination of hypomethylated and the nontranscribed fetal g-globin genes are
pregnancy. The marked differences in the expected clinical sever- hypermethylated in adult life. In contrast, the adult b-globin genes
ity of the different sickling disorders discussed above should be are hypermethylated and the g-globin genes are hypomethylated
taken into consideration when making such decisions. Before the in fetal life (75, 76). 5-Azacytidine was shown to induce very high
advent of molecular diagnostics, the only way to make a diagnosis levels of Hb F in anemic baboons (78). Its ability to stimulate Hb F
prenatally was to obtain a fetal blood sample for analysis, which production was also demonstrated in a small number of patients
could only be performed after the 20th week of pregnancy. By that with SCD and b-thalassemia (79, 80). In spite of these promis-
time, the pregnancy is already too advanced to make it possible ing results, this drug was never tested in large-scale clinical trials
to terminate safely. With the advent of DNA diagnostics, it has because of concerns about potential carcinogenicity.

The Journal of Clinical Investigation http://www.jci.org Volume 117 Number 4 April 2007 853
science in medicine

Figure 3
Alteration of the rbc membrane by polymers of sickle hemoglobin. Deoxygenation of Hb S induces a change in conformation in which the
mutant b chain binds to a complementary hydrophobic site resulting from a valine replacement, leading to the formation of a hemoglobin
polymer (Hb polymer; lower right, inset). The hemoglobin polymers disrupt the rbc cytoskeleton and form protrusions, giving rise to the char-
acteristic sickle appearance. Interruption of the attachment of the membrane to the protein cytoskeleton results in exposure of transmembrane
protein epitopes and lipid exchanges, notably of phosphatidylserine (PS), between the inside and the outside of the cell (upper right, inset).
Exposure of negatively charged glycolipids contributes to the proinflammatory and prothrombotic state of sickle cell blood. Adapted with
permission from Blackwell Publishing (129).

Hydroxyurea. The clinical development of hydroxyurea as an with hydroxyurea were shown to have improved survival (86).
agent that induces Hb F production in SCD provides a vivid illus- Other studies demonstrated the clinical efficacy and short-term
tration of how scientific discoveries can be translated into thera- safety of hydroxyurea in children with SCD (8789).
pies that improve the outlook for patients afflicted with a debili- Although hydroxyurea was shown to have Hb Finducing
tating disease. Interestingly, controversy over the mechanism of activity similar to that of 5-azacytidine in anemic baboons and
induction of Hb F by 5-azacytidine provided the motivation for patients with SCD, its molecular targets and mechanism(s) of
a study in which hydroxyurea, an S phasespecific chemothera- action are still not fully elucidated. It was originally proposed
peutic agent that does not inhibit DNA methyltransferase, was that hydroxyurea may elevate Hb F levels by accelerating ery-
shown to result in a marked increase in Hb F levels in baboons throid differentiation in the bone marrow, leading to the
(81). Hydroxyurea is an inhibitor of ribonucleotide reductase appearance of fetal-like cells in the peripheral blood (25).
that had been in use for many years in the treatment of myelo- More recent studies have shown that hydroxyurea generates NO
proliferative disorders. It is an orally available drug that is rela- in vivo, which results in the activation of the NO/cGMP signal-
tively well tolerated and simple to use. After the demonstration ing pathway and the upregulation of g-globin gene expression
of its ability to induce Hb F production in baboons, hydroxy- in patients with SCD (90). Hydroxyurea has other effects that
urea was tested in a number of small clinical trials in adults with may also benefit patients with SCD. For example, hydroxyurea
SCD (8284). A larger MSH study showed a marked decrease in was shown to decrease the adhesion of sickle cells to endothe-
the frequency of painful crises and acute chest syndrome and lium and to decrease the expression level of soluble VCAM-1
a reduction in transfusion requirements and hospitalizations (91, 92). Owing to its myelosuppressive activity, hydroxyurea
in adults with moderate to severe SCD after hydroxyurea treat- reduces circulating wbc counts and likely the number of adher-
ment (85). After 9 years of follow-up, patients with SCD treated ent leukocytes recruited to the wall of small venules. The reduc-

854 The Journal of Clinical Investigation http://www.jci.org Volume 117 Number 4 April 2007
science in medicine

Figure 4
Sickle cell vasoocclusion. Abnormal, sickle
rbc induce the expression of inflammatory and
coagulation mediators, leading to the activa-
tion of the vascular endothelium. Sickle rbc
themselves may also stimulate endothelial
cells directly by adhesion. The stimulated
endothelial cells are poised to recruit roll-
ing and adherent leukocytes in venules by
expressing chemokines and cell adhesion
molecules such as the selectins and immu-
noglobulin family members. Activated, firmly
adherent neutrophils capture circulating dis-
coid and sickle-shaped rbc, leading to tran-
sient episodes of vascular occlusions that are
initiated in the smallest postcapillary venules.
Interactions between rbc and leukocytes tend
to occur at vessel junctions, where leukocyte
recruitment is the most active. In sickle mice,
vasoocclusion can be prevented by the inhibi-
tion of leukocyte adhesion or the inflammatory
response. The large arrow indicates the direc-
tion of blood flow.

tion of wbc counts was correlated with the clinical benefit from Bone marrow transplantation
hydroxyurea (93). It is still not entirely clear how much of the The idea of replacing the bone marrow, which is the source of the
clinical benefit from hydroxyurea could be attributed to its defective sickle cells, with bone marrow that produces normal rbc
effect on Hb F levels compared with its other activities. is an intuitive therapeutic approach in SCD. However, for many
Butyrate. Concerns over the potential for serious side effects of years, such an approach was considered too risky for a nonma-
chemotherapeutic agents such as 5-azacytidine and hydroxyurea lignant disorder such as SCD, since the mortality of the proce-
has stimulated the continuation of the search for safe and effec- dure itself was around 20%. The reduction in mortality follow-
tive inducers of Hb F production. Butyrate, a short-chain fatty ing bone marrow transplantation (BMT), resulting from recent
acid that inhibits histone deacetylase (HDAC), was shown to advances in immunosuppressive therapy and supportive care and
stimulate embryonic or fetal globin gene expression in chicken, the fact that long-term survival of patients with b-thalassemia
mice, and baboons (9496). When arginine butyrate was admin- after BMT was shown to be greater than 90% (101), resulted in
istered to patients with SCD intermittently (four days every four renewed interest in this therapy for SCD. Clinical trials that were
weeks), it resulted in sustained induction of Hb F production in conducted in children with SCD in Europe and the US showed
a majority of patients (97). In spite of the considerable promise greater than 90% long-term survival (102104). A major limita-
of this agent in the treatment of SCD, the difficulty of admin- tion in the use of BMT for the treatment of SCD is the fact that
istrating large volumes of this drug through central venous a matched sibling donor is available to less than 15% of patients
catheters poses a major therapeutic challenge. It is unlikely that who are suitable candidates for transplantation (105). In an effort
the full potential of butyrate and other HDAC inhibitors will be to increase the availability of sources of hematopoietic stem cells
realized until an oral compound is identified that has the same for transplantation, clinical trials are being conducted to evalu-
efficacy as butyrate. ate cord blood transplantation in the treatment of SCD (106). To
Decitabine. The recent introduction of decitabine (5-aza-2- date, very few transplantation procedures have been performed
deoxycytidine), a new analog of 5-azacytidine that does not in adults with SCD because of concerns that the morbidity and
incorporate into RNA, has resulted in renewed interest in the mortality of BMT is higher in adults than in children. The use of
use of DNA hypomethylation therapy for the induction of Hb nonmyeloablative BMT to reduce peritransplant morbidity and
F production in SCD. In recent small-scale clinical trials in mortality has been associated with a very high graft rejection rate
patients with SCD, treatment with decitabine resulted in sig- (107). BMT is the only curative therapy for SCD, and the major
nificant increases in mean g-globin synthesis, Hb F levels, and challenge is to make it more widely available to patients with a
the number of F cells (rbc that contain Hb F) (98100). Interest- severe disease phenotype.
ingly, increased Hb F levels were observed in 100% of patients
with SCD who received decitabine, including patients who had Impediments that limit the impact of the
previously failed to respond to hydroxyurea. The increase in the advances in SCD
levels of Hb F was associated with significant improvement in In spite of the fact that hydroxyurea has been shown to improve
several factors that are important in the pathophysiology of both survival and the quality of life in patients with SCD, only
vasoocclusion, including rbc adhesion, endothelial damage, and a small fraction of eligible patients with SCD in the US are cur-
activation of the coagulation pathway (100). Larger and longer- rently receiving hydroxyurea (108, 109). Although the reasons
term studies are needed to confirm the efficacy and safety of for the reluctance to use hydroxyurea are not entirely clear, there
decitabine in the treatment of SCD. are many potential contributing factors. These include patient

The Journal of Clinical Investigation http://www.jci.org Volume 117 Number 4 April 2007 855
science in medicine

concerns about a drug that is used primarily to treat cancer, phy- ease (e.g., methylenetetrahydrofolate reductase and the pathogenesis
sician concerns about potential long-term mutagenic effects, of avascular necrosis [ref. 115], factor V R485K and the risk of
lack of familiarity of primary care providers with the use of a venous thrombosis [ref. 116], and UDP glucuronosyltransferase-1
chemotherapeutic agent, and resistance among patients with polymorphism and serum bilirubin levels [ref. 117]). Other inves-
SCD to use therapies that are perceived to be experimental in tigators are using an unbiased genetic approach that consists of
nature. Careful investigation into the impediments to the use of the analysis of hundreds of SNPs in a large number of patients to
hydroxyurea is necessary in order to realize the full potential of identify genes that increase the risk of a particular complication.
this important therapeutic advance. Using such an approach, Sebastiani and colleagues recently iden-
Prenatal diagnosis is another area in which the development tified 31 SNPs in 12 genes that interact with Hb F to modulate
of important new technology has had a very limited impact in the risk of stroke (118). In the future, such biased and unbiased
SCD. In spite of the fact that DNA diagnostics have made it pos- approaches may make it possible to identify a genetic blueprint
sible to identify an affected fetus much earlier during pregnancy, in a particular patient that defines his/her risk of developing any
the impact of these advances on the number of new births with of the known complications of SCD. It might even be possible
SCD in the US has been extremely small. In contrast, the same at some point to use this information to implement therapeutic
technology has had a very large impact on the number of new interventions before the complications develop.
births with b-thalassemia in Mediterranean regions including Future of therapeutics in SCD. Gene therapy offers enormous promise
Greece, Cyprus, and Sardinia (110). Although the reasons for the as a potential curative therapy for SCD, but concerns over the safety
differences in the impact of the same technology in these closely of random genomic insertion must first be resolved (119). Preclinical
related disorders have not been investigated, it is conceivable that studies in mice have provided the proof of principle that transduction
they are a reflection of the fact that a majority of patients with of bone marrow stem cells with lentiviral vectors that express a
b-thalassemia die from iron overload before the third decade of b-globin gene can prevent Hb S polymerization in vivo (120, 121).
life, while survival of patients with SCD into the fifth, sixth, and The wide range of abnormalities engendered by the sickle cell muta-
even seventh decades of life is not unusual. The reluctance to ter- tion offers several other opportunities for therapeutic interventions.
minate an affected pregnancy may also be motivated by cultural For example, the NIH Road Map is supporting ongoing investiga-
and ethnic factors that have not received adequate attention. It is tions in which high-throughput screening approaches are used to
sobering to keep in mind that the one intervention that has had discover novel low-molecular-weight compounds that can alter
the largest impact on the natural history of SCD during the last key aspects of the disease, including hemoglobin polymerization,
few decades is the introduction of penicillin prophylaxis during expression of Hb F, and leukocyte adhesion. Current clinical trials
childhood (13). This should serve as a reminder that important are evaluating the efficacy of Ca2+-sensitive Gardos channel inhibi-
therapeutic advances are bound to have a very limited impact on tors (e.g., ICA-17043), with or without hydroxyurea, in preventing
the natural history of any human disease unless they are widely dehydration of erythrocytes (122). Vasoactive drugs (e.g., NO, silde-
accepted by the patients they are intended to help. nafil, endothelin antagonists) are being evaluated for the treatment
of pulmonary hypertension. Statins are of potentially great interest
Future directions since they can increase NO production and reduce leukocyte adhe-
Future of diagnostics in SCD. As discussed above, the molecular sion (123, 124). Antiinflammatory drugs that inhibit NF-kB and the
methods of identifying the sickle mutation in utero and after upregulation of adhesion molecules have shown promise in pilot
birth are well established and widely available. However, although clinical studies (125). Intravenous gammaglobulins are currently
the same sickle mutation in the b-globin gene is responsible for under clinical evaluation following a study demonstrating a dose-
the spectrum of the pathophysiology of the sickling disorder, dependent reduction in leukocyte adhesion and in the number of
the clinical manifestations of the disease are extremely heteroge- interactions between rbc and wbc, accompanied by improvements in
neous. Many factors that contribute to this heterogeneity, such microcirculatory blood flow and survival of sickle transgenic mice
as an interaction of the bC gene and the b-thalassemia gene, are (126). Furthermore, there is growing interest in the prevention and
well known. Other factors such as the Hb F levels and the coin- treatment of vasoocclusion by novel selectin antagonists since they
heritance of a-thalassemia have also been known to modulate the appear to participate in multiple pathways involved in sickle vaso-
clinical severity of sickling disorders for many years (69, 70, 111). occlusion, including the adhesion of leukocytes, rbc, and platelets
Other genetic determinants that contribute to the variability of to the endothelium and to each other (127). Almost a century after
Hb F levels were identified outside the b-globin gene cluster and SCD was first described, we may be at the dawn of a new era in which
mapped to two different chromosomes (112, 113). More recent a physician might be able to use genetic information to select one or
studies have demonstrated that the gene responsible for the vari- more drugs that target specific aspects of disease pathophysiology
ability in Hb F levels that was previously mapped to chromosome that are relevant to a particular patient with SCD.
6p23 (112) is cMYB (114). As the understanding of the pathophys-
iology of the disease evolves, the number of potential epistatic Acknowledgments
genes (i.e., modifying genes) increases. Thus, mutations or poly- The authors would like to acknowledge the support of the NIH for
morphisms that have an impact on cell adhesion, thrombosis, rbc their research on SCD (HL69438 to P.S. Frenette and HL073438
dehydration, and inflammation are likely in candidate epistatic to G.F. Atweh).
genes in SCD. As is the case with the coinheritance of a-thalasse-
mia, some of these genetic determinants might increase the risk of Address correspondence to: George F. Atweh, Division of Hematolo-
some complications and decrease the risks of others. A number of gy/Oncology Box 1079, Department of Medicine, One Gustave L.
studies have investigated the potential effects of candidate modi- Levy Place, New York, New York 10029, USA. Phone: (212) 241-8109;
fying genes that are implicated in the pathophysiology of the dis- Fax: (212) 369-8375; E-mail: george.atweh@mssm.edu.

856 The Journal of Clinical Investigation http://www.jci.org Volume 117 Number 4 April 2007
science in medicine

1. Herrick, J.B. 1910. Peculiar elongated and sickle- 27. Serjeant, G.R., Sommereux, A.M., Stevenson, M., 47. Manci, E.A., et al. 2006. Pathology of Berkeley sickle
shaped red blood corpuscules in a case of severe Mason, K., and Serjeant, B.E. 1979. Comparison cell mice: similarities and differences with human
anemia. Arch. Int. Med. 6:517521. of sickle cell-beta0 thalassaemia with homozygous sickle cell disease. Blood. 107:16511658.
2. Herrick, J.B. 1924. Abstract of discussion. JAMA. sickle cell disease. Br. J. Haematol. 41:8393. 48. Kaul, D.K., and Hebbel, R.P. 2000. Hypoxia/reoxy-
83:16. 28. Serjeant, G.R., Ashcroft, M.T., Serjeant, B.E., and genation causes inflammatory response in trans-
3. Sydenstricker, V.P. 1924. Further observations on Milner, P.F. 1973. The clinical features of sickle-cell- genic sickle mice but not in normal mice. J. Clin.
sickle cell anemia. JAMA. 83:1215. thalassaemia in Jamaica. Br. J. Haematol. 24:1930. Invest. 106:411420.
4. Hanh, E.V., and Gillespie, E.B. 1927. Sickle cell ane- 29. Hebbel, R.P., et al. 1980. Abnormal adherence of 49. Turhan, A., Weiss, L.A., Mohandas, N., Coller, B.S.,
mia. Arch. Int. Med. 39:233. sickle erythrocytes to cultured vascular endothe- and Frenette, P.S. 2002. Primary role for adherent
5. Scriver, J.R., and Waugh, T.R. 1930. Studies on a case lium: possible mechanism for microvascular occlu- leukocytes in sickle cell vascular occlusion: a new
of sickle cell anemia. Can. Med. Assoc. J. 23:375380. sion in sickle cell disease. J. Clin. Invest. 65:154160. paradigm. Proc. Natl. Acad. Sci. U. S. A. 99:30473051.
6. Pauling, L. 1964. Molecular disease and evolution. 30. Hoover, R., Rubin, R., Wise, G., and Warren, R. 1979. 50. Belcher, J.D., et al. 2003. Transgenic sickle mice have
Bull. N. Y. Acad. Med. 40:334342. Adhesion of normal and sickle erythrocytes to vascular inflammation. Blood. 101:39533959.
7. Pauling, L., Itano, H.A., Singer, S.J., and Wells, endothelial monolayer cultures. Blood. 54:872876. 51. Belcher, J.D., et al. 2006. Heme oxygenase-1 is a
I.C. 1949. Sickle cell anemia, a molecular disease. 31. Joneckis, C.C., Ackley, R.L., Orringer, E.P., Wayner, modulator of inflammation and vaso-occlusion in
Science. 110:543548. E.A., and Parise, L.V. 1993. Integrin alpha 4 beta 1 transgenic sickle mice. J. Clin. Invest. 116:808816.
8. Neel, J.V. 1949. The inheritance of sickle cell anemia. and glycoprotein IV (CD36) are expressed on cir- doi:10.1172/JCI26857.
Science. 110:6466. culating reticulocytes in sickle cell anemia. Blood. 52. Kaul, D.K., et al. 2004. Anti-inflammatory therapy
9. Watson, J., Stahman, A.W., and Bilello, F.P. 1948. 82:35483555. ameliorates leukocyte adhesion and microvascular
The significance of the paucity of sickle cells in new- 32. Swerlick, R.A., Eckman, J.R., Kumar, A., Jeitler, flow abnormalities in transgenic sickle mice. Am. J.
born Negro infants. Am. J. Med. Sci. 215:419423. M., and Wick, T.M. 1993. Alpha 4 beta 1-integrin Physiol. Heart Circ. Physiol. 287:H293H301.
10. Ingram, V.M. 1958. Abnormal human haemoglo- expression on sickle reticulocytes: vascular cell 53. de Jong, K., Larkin, S.K., Styles, L.A., Bookchin,
bins. I. The comparison of normal human and adhesion molecule-1-dependent binding to endo- R.M., and Kuypers, F.A. 2001. Characterization of
sickle-cell haemoglobins by fingerprinting. Biochim. thelium. Blood. 82:18911899. the phosphatidylserine-exposing subpopulation of
Biophys. Acta. 28:539545. 33. Kaul, D.K., et al. 2000. Monoclonal antibodies to sickle cells. Blood. 98:860867.
11. Ferrone, F.A. 2004. Polymerization and sickle cell dis- alphaVbeta3 (7E3 and LM609) inhibit sickle red 54. Yasin, Z., et al. 2003. Phosphatidylserine externaliza-
ease: a molecular view. Microcirculation. 11:115128. blood cell-endothelium interactions induced by tion in sickle red blood cells: associations with cell
12. Gaston, M., and Rosse, W.F. 1982. The cooperative platelet-activating factor. Blood. 95:368374. age, density, and hemoglobin F. Blood. 102:365370.
study of sickle cell disease: review of study design and 34. Spring, F.A., et al. 2001. Intercellular adhesion 55. Chiu, D., Lubin, B., Roelofsen, B., and van Deenen,
objectives. Am. J. Pediatr. Hematol. Oncol. 4:197201. molecule-4 binds alpha(4)beta(1) and alpha(V)- L.L. 1981. Sickled erythrocytes accelerate clotting
13. Gaston, M.H., et al. 1986. Prophylaxis with oral family integrins through novel integrin-binding in vitro: an effect of abnormal membrane lipid
penicillin in children with sickle cell anemia. A mechanisms. Blood. 98:458466. asymmetry. Blood. 58:398401.
randomized trial. N. Engl. J. Med. 314:15931599. 35. Gee, B.E., and Platt, O.S. 1995. Sickle reticulocytes 56. Reiter, C.D., et al. 2002. Cell-free hemoglobin lim-
14. Platt, O.S., et al. 1994. Mortality in sickle cell adhere to VCAM-1. Blood. 85:268274. its nitric oxide bioavailability in sickle-cell disease.
disease. Life expectancy and risk factors for early 36. Natarajan, M., Udden, M.M., and McIntire, L.V. Nat. Med. 8:13831389.
death. N. Engl. J. Med. 330:16391644. 1996. Adhesion of sickle red blood cells and dam- 57. Okpala, I. 2006. Leukocyte adhesion and the patho-
15. Vichinsky, E.P., et al. 1997. Acute chest syndrome in age to interleukin-1 beta stimulated endothelial physiology of sickle cell disease. Curr. Opin. Hematol.
sickle cell disease: clinical presentation and course. cells under flow in vitro. Blood. 87:48454852. 13:4044.
Cooperative Study of Sickle Cell Disease. Blood. 37. Embury, S.H., et al. 2004. The contribution of 58. Frenette, P.S. 2004. Sickle cell casoocclusion: het-
89:17871792. endothelial cell P-selectin to the microvascular erotypic, multicellular aggregations driven by leu-
16. Castro, O., et al. 1994. The acute chest syndrome in flow of mouse sickle erythrocytes in vivo. Blood. kocyte adhesion. Microcirculation. 11:167177.
sickle cell disease: incidence and risk factors. The 104:33783385. 59. Schmid-Schonbein, G.W. 1993. The damaging poten-
Cooperative Study of Sickle Cell Disease. Blood. 38. Hillery, C.A., Scott, J.P., and Du, M.C. 1999. The car- tial of leukocyte activation in the microcirculation.
84:643649. boxy-terminal cell-binding domain of thrombos- Angiology. 44:4556.
17. Ohene-Frempong, K., et al. 1998. Cerebrovascular pondin is essential for sickle red blood cell adhesion. 60. House, S.D., and Lipowsky, H.H. 1987. Leukocyte-
accidents in sickle cell disease: rates and risk factors. Blood. 94:302309. endothelium adhesion: microhemodynamics in
Blood. 91:288294. 39. Wautier, J.L., et al. 1983. Fibrinogen, a modulator mesentery of the cat. Microvasc. Res. 34:363379.
18. Adams, R.J., et al. 1998. Prevention of a first stroke of erythrocyte adhesion to vascular endothelium. 61. Frenette, P.S., Mayadas, T.N., Rayburn, H., Hynes,
by transfusions in children with sickle cell anemia J. Lab. Clin. Med. 101:911920. R.O., and Wagner, D.D. 1996. Susceptibility to
and abnormal results on transcranial Doppler 40. Kasschau, M.R., Barabino, G.A., Bridges, K.R., and infection and altered hematopoiesis in mice defi-
ultrasonography. N. Engl. J. Med. 339:511. Golan, D.E. 1996. Adhesion of sickle neutrophils cient in both P- and E-selectins. Cell. 84:563574.
19. Platt, O.S., et al. 1991. Pain in sickle cell disease. and erythrocytes to fibronectin. Blood. 87:771780. 62. Hofstra, T.C., Kalra, V.K., Meiselman, H.J., and
Rates and risk factors. N. Engl. J. Med. 325:1116. 41. Kaul, D.K., Nagel, R.L., Chen, D., and Tsai, H.M. Coates, T.D. 1996. Sickle erythrocytes adhere to
20. Rosse, W.F., et al. 1990. Transfusion and alloim- 1993. Sickle erythrocyte-endothelial interactions polymorphonuclear neutrophils and activate the
munization in sickle cell disease. The Cooperative in microcirculation: the role of von Willebrand neutrophil respiratory burst. Blood. 87:44404447.
Study of Sickle Cell Disease. Blood. 76:14311437. factor and implications for vasoocclusion. Blood. 63. Chiang, E.Y., Hidalgo, A., Chang, J., and Fren-
21. Koshy, M., Burd, L., Wallace, D., Moawad, A., and 81:24292438. ette, P.S. 2007. Imaging receptor microdomains
Baron, J. 1988. Prophylactic red-cell transfusions 42. Wick, T.M., et al. 1987. Unusually large von Wille- on leukocyte subsets in live mice. Nat. Methods.
in pregnant patients with sickle cell disease. A brand factor multimers increase adhesion of sickle 4:219222.
randomized cooperative study. N. Engl. J. Med. erythrocytes to human endothelial cells under con- 64. Kinney, T.R., et al. 1999. Silent cerebral infarcts
319:14471452. trolled flow. J. Clin. Invest. 80:905910. in sickle cell anemia: a risk factor analysis. The
22. Koshy, M., et al. 1995. Surgery and anesthesia in 43. Thevenin, B.J.M., Crandall, I., Ballas, S.K., Sherman, Cooperative Study of Sickle Cell Disease. Pediatrics.
sickle cell disease. Cooperative Study of Sickle Cell I.W., and Shohet, S.B. 1997. Band 3 peptides block 103:640645.
Diseases. Blood. 86:36763684. the adherence of sickle cells to endothelial cells in 65. Balkaran, B., et al. 1992. Stroke in a cohort of
23. Castro, O., and Gladwin, M.T. 2005. Pulmonary vitro. Blood. 90:41724179. patients with homozygous sickle cell disease.
hypertension in sickle cell disease: mechanisms, 44. Hillery, C.A., Du, M.C., Montgomery, R.R., and J. Pediatr. 120:360366.
diagnosis, and management. Hematol. Oncol. Clin. Scott, J.P. 1996. Increased adhesion of erythrocytes 66. Miller, S.T., et al. 2000. Prediction of adverse out-
North Am. 19:881896, vii. to components of the extracellular matrix: isola- comes in children with sickle cell disease. N. Engl. J.
24. Ingram, V.M. 1959. Abnormal human haemoglo- tion and characterization of a red blood cell lipid Med. 342:8389.
bins. III. The chemical difference between normal that binds thrombospondin and laminin. Blood. 67. Orkin, S.H., Little, P.F., Kazazian, H.H., Jr., and
and sickle cell haemoglobins. Biochim. Biophys. Acta. 87:48794886. Boehm, C.D. 1982. Improved detection of the sickle
36:402411. 45. Belcher, J.D., et al. 2005. Critical role of endothelial mutation by DNA analysis: application to prenatal
25. Stamatoyannopoulos, G. 2005. Control of globin cell activation in hypoxia-induced vasoocclusion diagnosis. N. Engl. J. Med. 307:3236.
gene expression during development and erythroid in transgenic sickle mice. Am. J. Physiol. Heart Circ. 68. Cheung, M.C., Goldberg, J.D., and Kan, Y.W. 1996.
differentiation. Exp. Hematol. 33:259271. Physiol. 288:H2715H2725. Prenatal diagnosis of sickle cell anaemia and thal-
26. Bank, A. 2006. Regulation of human fetal hemo- 46. Atweh, G.F., et al. 2003. Hemoglobinopathies. assaemia by analysis of fetal cells in maternal blood.
globin: new players, new complexities. Blood. Hematology Am. Soc. Hematol. Educ. Program. Nat. Genet. 14:264268.
107:435443. 2003:1439. 69. Perrine, R.P., Pembrey, M.E., John, P., Perrine, S.,

The Journal of Clinical Investigation http://www.jci.org Volume 117 Number 4 April 2007 857
science in medicine

and Shoup, F. 1978. Natural history of sickle cell hemoglobin by the nitric oxidedependent acti- tologists/oncologists. Am. J. Hematol. 79:107113.
anemia in Saudi Arabs. A study of 270 subjects. vation of soluble guanylyl cyclase. J. Clin. Invest. 110. Cao, A. 1987. Results of programmes for antenatal
Ann. Intern. Med. 88:16. 111:231239. doi:10.1172/JCI200316672. detection of thalassemia in reducing the incidence
70. Kar, B.C., et al. 1986. Sickle cell disease in Orissa 91. Saleh, A.W., Hillen, H.F., and Duits, A.J. 1999. of the disorder. Blood Rev. 1:169176.
State, India. Lancet. 2:11981201. Levels of endothelial, neutrophil and platelet-spe- 111. Higgs, D.R., et al. 1982. The interaction of alpha-
71. Nagel, R.L., et al. 1979. Structural bases of the cific factors in sickle cell anemia patients during thalassemia and homozygous sickle-cell disease.
inhibitory effects of hemoglobin F and hemoglo- hydroxyurea therapy. Acta Haematol. 102:3137. N. Engl. J. Med. 306:14411446.
bin A2 on the polymerization of hemoglobin S. 92. Bridges, K.R., et al. 1996. A multiparameter analy- 112. Garner, C., et al. 1998. Haplotype mapping of a
Proc. Natl. Acad. Sci. U. S. A. 76:670672. sis of sickle erythrocytes in patients undergoing major quantitative-trait locus for fetal hemoglo-
72. Lawn, R.M., Fritsch, E.F., Parker, R.C., Blake, G., and hydroxyurea therapy. Blood. 88:47014710. bin production, on chromosome 6q23. Am. J. Hum.
Maniatis, T. 1978. The isolation and characteriza- 93. Charache, S., et al. 1996. Hydroxyurea and sickle cell Genet. 62:14681474.
tion of linked delta- and beta-globin genes from a anemia. Clinical utility of a myelosuppressive switch- 113. Dover, G.J., et al. 1992. Fetal hemoglobin levels
cloned library of human DNA. Cell. 15:11571174. ing agent. The Multicenter Study of Hydroxy- in sickle cell disease and normal individuals are
73. Collins, F.S., and Weissman, S.M. 1984. The molec- urea in Sickle Cell Anemia. Medicine (Baltimore). partially controlled by an X-linked gene located at
ular genetics of human hemoglobin. Prog. Nucleic 75:300326. Xp22.2. Blood. 80:816824.
Acid Res. Mol. Biol. 31:315462. 94. Ginder, G.D., Whitters, M.J., and Pohlman, J.K. 114. Jiang, J., et al. 2006. cMYB is involved in the regu-
74. Orkin, S.H. 1990. Globin gene regulation and 1984. Activation of a chicken embryonic globin lation of fetal hemoglobin production in adults.
switching: circa 1990. Cell. 63:665672. gene in adult erythroid cells by 5-azacytidine Blood. 108:10771083.
75. van der Ploeg, L.H., and Flavell, R.A. 1980. DNA and sodium butyrate. Proc. Natl. Acad. Sci. U. S. A. 115. Kutlar, A., Kutlar, F., Turker, I., and Tural, C.
methylation in the human gamma delta beta-glo- 81:39543958. 2001. The methylene tetrahydrofolate reductase
bin locus in erythroid and nonerythroid tissues. 95. Constantoulakis, P., et al. 1991. Locus control (C677T) mutation as a potential risk factor for
Cell. 19:947958. region-A gamma transgenic mice: a new model for avascular necrosis in sickle cell disease. Hemoglobin.
76. Mavilio, F., et al. 1983. Molecular mechanisms of studying the induction of fetal hemoglobin in the 25:213217.
human hemoglobin switching: selective under- adult. Blood. 77:13261333. 116. Helley, D., et al. 1997. Polymorphism in exon 10 of
methylation and expression of globin genes in 96. Constantoulakis, P., Knitter, G., and Stamatoy- the human coagulation factor V gene in a popu-
embryonic, fetal, and adult erythroblasts. Proc. annopoulos, G. 1989. On the induction of fetal lation at risk for sickle cell disease. Hum. Genet.
Natl. Acad. Sci. U. S. A. 80:69076911. hemoglobin by butyrates: in vivo and in vitro stud- 100:245248.
77. Groudine, M., and Weintraub, H. 1981. Activation ies with sodium butyrate and comparison of com- 117. Kutlar, A. 2005. Sickle cell disease: a multigenic
of globin genes during chicken development. Cell. bination treatments with 5-AzaC and AraC. Blood. perspective of a single gene disorder. Hematology.
24:393401. 74:19631971. 10(Suppl. 1):9299.
78. DeSimone, J., Heller, P., Hall, L., and Zwiers, D. 97. Atweh, G.F., et al. 1999. Sustained induction of 118. Sebastiani, P., Ramoni, M.F., Nolan, V., Baldwin,
1982. 5-Azacytidine stimulates fetal hemoglobin fetal hemoglobin by pulse butyrate therapy in C.T., and Steinberg, M.H. 2005. Genetic dissection
synthesis in anemic baboons. Proc. Natl. Acad. Sci. sickle cell disease. Blood. 93:17901797. and prognostic modeling of overt stroke in sickle
U. S. A. 79:44284431. 98. Koshy, M., et al. 2000. 2-Deoxy 5-azacytidine and cell anemia. Nat. Genet. 37:435440.
79. Ley, T.J., et al. 1982. 5-Azacytidine selectively fetal hemoglobin induction in sickle cell anemia. 119. Sadelain, M. 2006. Recent advances in globin gene
increases gamma-globin synthesis in a patient with Blood. 96:23792384. transfer for the treatment of beta-thalassemia and
beta+ thalassemia. N. Engl. J. Med. 307:14691475. 99. DeSimone, J., et al. 2002. Maintenance of elevated sickle cell anemia. Curr. Opin. Hematol. 13:142148.
80. Ley, T.J., et al. 1983. 5-Azacytidine increases gamma- fetal hemoglobin levels by decitabine during dose 120. Pawliuk, R., et al. 2001. Correction of sickle cell dis-
globin synthesis and reduces the proportion of interval treatment of sickle cell anemia. Blood. ease in transgenic mouse models by gene therapy.
dense cells in patients with sickle cell anemia. 99:39053908. Science. 294:23682371.
Blood. 62:370380. 100. Saunthararajah, Y., et al. 2003. Effects of 5-aza-2-deox- 121. Levasseur, D.N., Ryan, T.M., Pawlik, K.M., and
81. Letvin, N.L., Linch, D.C., Beardsley, G.P., McIntyre, ycytidine on fetal hemoglobin levels, red cell adhesion, Townes, T.M. 2003. Correction of a mouse model
K.W., and Nathan, D.G. 1984. Augmentation of and hematopoietic differentiation in patients with of sickle cell disease: lentiviral/antisickling beta-
fetal-hemoglobin production in anemic monkeys sickle cell disease. Blood. 102:38653870. globin gene transduction of unmobilized, purified
by hydroxyurea. N. Engl. J. Med. 310:869873. 101. Lucarelli, G., et al. 1991. Bone marrow transplanta- hematopoietic stem cells. Blood. 102:43124319.
82. Platt, O.S., et al. 1984. Hydroxyurea enhances fetal tion in thalassemia. Hematol. Oncol. Clin. North Am. 122. Mueller, B.U., and Brugnara, C. 2001. Prevention of
hemoglobin production in sickle cell anemia. 5:549556. red cell dehydration: a possible new treatment for
J. Clin. Invest. 74:652656. 102. Vermylen, C., Cornu, G., Ferster, A., Ninane, J., and sickle cell disease. Pediatr. Pathol. Mol. Med. 20:1525.
83. Charache, S., et al. 1992. Hydroxyurea: effects on Sariban, E. 1993. Bone marrow transplantation in 123. Solovey, A., et al. 2004. Endothelial cell expres-
hemoglobin F production in patients with sickle sickle cell disease: the Belgian experience. Bone Mar- sion of tissue factor in sickle mice is augmented by
cell anemia. Blood. 79:25552565. row Transplant. 12(Suppl. 1):116117. hypoxia/reoxygenation and inhibited by lovastatin.
84. Rodgers, G.P., Dover, G.J., Noguchi, C.T., Schech- 103. Bernaudin, F., et al. 1993. Bone marrow transplan- Blood. 104:840846.
ter, A.N., and Nienhuis, A.W. 1990. Hematologic tation (BMT) in 14 children with severe sickle cell 124. Takemoto, M., and Liao, J.K. 2001. Pleiotropic
responses of patients with sickle cell disease to disease (SCD): the French experience. GEGMO. effects of 3-hydroxy-3-methylglutaryl coenzyme a
treatment with hydroxyurea. N. Engl. J. Med. Bone Marrow Transplant. 12(Suppl. 1):118121. reductase inhibitors. Arterioscler. Thromb. Vasc. Biol.
322:10371045. 104. Walters, M.C., et al. 1996. Bone marrow trans- 21:17121719.
85. Charache, S., et al. 1995. Effect of hydroxyurea plantation for sickle cell disease. N. Engl. J. Med. 125. S olovey, A.A., Solovey, A.N., Harkness, J., and
on the frequency of painful crises in sickle cell 335:369376. Hebbel, R.P. 2001. Modulation of endothelial cell
anemia. Investigators of the Multicenter Study of 105. Walters, M.C., et al. 1996. Barriers to bone marrow activation in sickle cell disease: a pilot study. Blood.
Hydroxyurea in Sickle Cell Anemia. N. Engl. J. Med. transplantation for sickle cell anemia. Biol. Blood 97:19371941.
332:13171322. Marrow Transplant. 2:100104. 126. Turhan, A., et al. 2004. Intravenous immune
86. Steinberg, M.H., et al. 2003. Effect of hydroxyurea 106. Locatelli, F., et al. 2003. Related umbilical cord globulin prevents venular vaso-occlusion in sickle
on mortality and morbidity in adult sickle cell ane- blood transplantation in patients with thalassemia cell mice by inhibiting leukocyte adhesion and the
mia: risks and benefits up to 9 years of treatment. and sickle cell disease. Blood. 101:21372143. interactions between sickle erythrocytes and adher-
JAMA. 289:16451651. 107. Iannone, R., et al. 2003. Results of minimally toxic ent leukocytes. Blood. 103:23972400.
87. Hankins, J.S., et al. 2005. Long-term hydroxyurea nonmyeloablative transplantation in patients with 127. Chiang, E.Y., and Frenette, P.S. 2005. Sickle cell
therapy for infants with sickle cell anemia: the sickle cell anemia and beta-thalassemia. Biol. Blood vaso-occlusion. Hematol. Oncol. Clin. North Am.
Husoft extension study. Blood. 106:22692275. Marrow Transplant. 9:519528. 19:771784.
88. Kinney, T.R., et al. 1999. Safety of hydroxyurea 108. Lanzkron, S., Haywood, C., Jr., Segal, J.B., and Dover, 128. Bank, A. 2005. Understanding globin regulation in
in children with sickle cell anemia: results of the G.J. 2006. Hospitalization rates and costs of care b-thalassemia: its as simple as a, b, g, and d. J. Clin.
HUG-KIDS study, a phase I/II trial. Pediatric of patients with sickle-cell anemia in the state of Invest. 115:14701473. doi:10.1172/JCI25398.
Hydroxyurea Group. Blood. 94:15501554. Maryland in the era of hydroxyurea. Am. J. Hematol. 129. Statius van Eps, L.W. 1999. Sickle cell disease. In
89. Wang, W.C., et al. 2001. A two-year pilot trial of 81:927932. Atlas of diseases of the kidney. Volume 4, chapter 4.
hydroxyurea in very young children with sickle-cell 109. Zumberg, M.S., et al. 2005. Hydroxyurea therapy Saulo Klahr, editor. Current Medicine LLC. Phila-
anemia. J. Pediatr. 139:790796. for sickle cell disease in community-based practic- delphia, Pennsylvania, USA. http://www.kidneyat-
90. Cokic, V.P., et al. 2003. Hydroxyurea induces fetal es: a survey of Florida and North Carolina hema- las.org/toc.htm.

858 The Journal of Clinical Investigation http://www.jci.org Volume 117 Number 4 April 2007
Pediatr Clin N Am 55 (2008) 447460

Update on Thalassemia: Clinical Care


and Complications
Melody J. Cunningham, MD
Thalassemia Research Program, Division of Hematology/Oncology, Childrens Hospital
Boston, 300 Longwood Avenue, Fegan 7, Boston, MA 02115, USA

b-Thalassemia, originally named Cooley anemia, initially was described


by Dr. Cooley in 1925 in Detroit as an inherited blood disease [1]. It is spec-
ulated that thalassemia was rst recognized in the United States and not in
its area of highest prevalence (the Mediterranean) because its presentation
as a distinct clinical entity was masked by the fact that malaria, with its sim-
ilar clinical picture of hemolysis, anemia, and splenomegaly, was ubiquitous
in that region [1]. Thus, patients who had this clinical triad were assumed to
have malaria, not thalassemia [1]. Now it is recognized that various types of
thalassemia are inherited anemias caused by mutations at the globin gene
loci on chromosomes 16 and 11, aecting the production of a- or b-globin
protein, respectively [2,3].
The thalassemia syndromes are named according to the globin chain
aected or the abnormal hemoglobin produced. Thus, b-globin gene muta-
tions give rise to b-thalassemia and a-globin mutations cause a-thalassemia.
In addition, the thalassemias are characterized by their clinical severity (phe-
notype). Thalassemia major (TM) refers to disease requiring more than
eight red blood cell (RBC) transfusions per year and thalassemia intermedia
(TI) to disease that requires no or infrequent transfusions [4]. Thalassemia
trait refers to carriers of mutations; such individuals have microcytosis
and hypochromia but no or only mild anemia [5,6]. Untreated TM uni-
formly is fatal in the rst few years of life [1]. In addition, TM and severe
TI can lead to considerable morbidity aecting nearly all organ systems
[79]. The combination of early diagnosis, improvements in monitoring
for organ complications, and advances in supportive care, however, have en-
abled many patients who have severe thalassemia syndromes to live produc-
tive, active lives well into adulthood [911].

E-mail address: melody.cunningham@childrens.harvard.edu

0031-3955/08/$ - see front matter 2008 Elsevier Inc. All rights reserved.
doi:10.1016/j.pcl.2008.02.002 pediatric.theclinics.com
448 CUNNINGHAM

Epidemiology
Similar to sickle cell disease and G6PD deciency, the high prevalence of
a- and b-thalassemia genotypes is believed a consequence of an evolutionary
protection of heterozygotes against death from Plasmodium falciparum ma-
laria [11,12]. Before the twentieth century, thalassemia tracked with areas of
malarial prevalence. b-Thalassemia arose in the Mediterranean, Middle
East, South and Southeast Asia, and southern China. a-Thalassemia origi-
nated in Africa, the Middle East, China, India, and Southeast Asia [1315].
Immigration and emigration, however, have led to changing demographics,
and patients who have thalassemia syndromes and heterozygote carriers
now reside in all parts of the world [16,17]. Thus, it is important for pedia-
tricians, obstetrician, and hematologists to be aware of a possible diagnosis
of thalassemia wherever they practice and for any patients they evaluate
who have anemia. Consideration of the diagnosis allows proper diagnosis
and management of individual patients and identication of carriers and en-
sures necessary testing and counseling to the population at risk for having
children who have thalassemia.

Diagnosis
Understanding how thalassemia can be diagnosed requires a review of
the structure of hemoglobin and the genetics of the thalassemia syndromes.
Normal human hemoglobin is comprised of two a-like and two b-like globin
chains. Adult hemoglobin consists of hemoglobin A (a2b2) plus small
amounts of hemoglobin A2 (a2d2) and hemoglobin F (a2g2). Genetic muta-
tions in one of the globin genes (a or b) result in decreased or absent
production of that globin chain and a relative excess of the other. These mu-
tations can result in no globin production (b or a ) or decreased globin pro-
duction (b or a).
The a-globin gene is duplicated on chromosome 16; thus, each diploid
cell carries four copies. The clinical syndromes of a-thalassemia reect the
number of inherited genes that are mutated. The a-thalassemia syndromes
are silent carrier, a-thalassemia trait, hemoglobin H disease, and hydrops fe-
talis and reect inheritance of 1, 2, 3, or 4 a-globin gene mutations, respec-
tively. In contrast, a single b-globin gene resides on each chromosome 11.
The four clinical syndromes of b-thalassemia, namely silent carrier, thalas-
semia trait, TI, and TM, correspond to the degree of expression of the two
b-globin genes that encode b-globin and not the number of mutated genes.
a-Thalassemia has a wide spectrum of syndromes due to the possibility of
one, two, three, or four allelic mutations. Mutation in one of the four alleles
results in the silent carrier, with no clinical symptoms, normal complete
blood cell count, and hemoglobin electrophoresis results past infancy.
If two of the four a-globin alleles are mutated, aected individuals have
a-thalassemia trait, with no clinical symptoms, but microcytosis and
THALASSEMIA UPDATE 449

hypochromia and only mild anemia. The newborn screen often reports he-
moglobin Bart, a fast-migrating hemoglobin that appears only in cord and
neonatal blood when there is a deletion of one or more of the four a-globin
alleles. Hemoglobin Bart is a g4 homotetramer that disappears rapidly in the
neonatal period; its amount at birth corresponds to the number of aected
alleles [18]. Three a-gene mutations cause hemoglobin H disease with anemia
characterized by microcytosis and hypochromia. Complete absence of a-glo-
bin chain production (all four alleles aected) leads to hydrops fetalis, which
usually results in death in utero if intrauterine transfusions are not available
[6,18].
b-Thalassemia has a similar spectrum of clinical phenotypes that reect
the underlying allelic mutations in the b-globin genes. If only a single b-glo-
bin gene is aected, then the resulting b-thalassemia silent carrier or trait re-
sults from partial (b) or absent (b ) gene expression, respectively. Similar
to a-thalassemia trait, patients who have b-thalassemia trait typically have
mild anemia, microcytosis, and hypochromia. When both b-globin genes
are aected, then the resulting phenotype is more severe, depending on
the degree of gene expression and relative imbalance of globin chains. For
example, b/b genotypes typically are associated with an intermediate phe-
notype (TI), whereas the b /b genotype leads to the more severe TM.
Specic mutations in the a or b genes may lead to production of unique
hemoglobins on electrophoresis, two of which have unusual features worth
discussing in the context of thalassemia. Hemoglobin Constant Spring (Hb
CS) is an a-globin gene variant caused by a mutation in the normal stop co-
don. The resulting elongated a-globin chain forms an unstable hemoglobin
tetramer. Hb CS often occurs in conjunction with a-thalassemia so is asso-
ciated with the more severe a-thalassemia phenotypes. Hemoglobin E (HbE)
is caused by a nucleotide change in the b-globin gene, which leads to a single
amino acid substitution (Glu26Lys) and diminished expression with
a b phenotype. HbE thus is an unusual thalassemic hemoglobinopathy
that can lead to clinically severe phenotypes when paired with other forms
of b-thalassemia.

Pathophysiology
The thalassemia syndromes were among the rst genetic diseases to be
understood at the molecular level. More than 200 b-globin and 30 a-globin
mutations deletions have been identied; these mutations result in decreased
or absent production of one globin chain (a or b) and a relative excess of the
other. The resulting imbalance leads to unpaired globin chains, which pre-
cipitate and cause premature death (apoptosis) of the red cell precursors
within the marrow, termed ineective erythropoiesis. Of the damaged but
viable RBCs that are released from the bone marrow, many are removed
by the spleen or hemolyzed directly in the circulation due to the hemoglobin
precipitants. Combined RBC destruction in the bone marrow, spleen, and
450 CUNNINGHAM

periphery causes anemia and, ultimately, an escalating cycle of pathology re-


sulting in the clinical syndrome of severe thalassemia.
Damaged erythrocytes enter the spleen and are trapped in this low pH
and low oxygen environment; subsequent splenomegaly exacerbates the
trapping of cells and worsens the anemia. Anemia and poor tissue oxygen-
ation stimulate increased kidney erythropoietin production that further
drives marrow erythropoiesis, resulting in increased ineective marrow ac-
tivity and the classic bony deformities associated with poorly managed
TM and severe TI [19]. Anemia in the severe thalassemia phenotypes neces-
sitates multiple RBC transfusions and, over time, without proper chelation,
results in transfusion-associated iron overload. In addition, ineective eryth-
ropoiesis enhances gastrointestinal iron absorption and can result in iron
overload, even in untransfused patients who have TI [2022]. It has long
been recognized that the severity of ineective erythropoiesis aects the de-
gree of iron loading, but until the recent discovery of hepcidin and under-
standing, its role in iron metabolism the link was not understood.
Hepcidin, an antimicrobial hormone, is recognized as playing a major
role in iron deciency and overload [23]. Hepcidin initially was discovered
due to its role in the etiology of anemia of chronic inammation or chronic
disease [24]. Elevated levels, associated with increased inammatory
markers, maintain low levels of circulating bioavailable iron in two impor-
tant ways: (1) by preventing iron absorption and transport from the gut and
(2) by preventing release and recycling of iron from macrophages and the
reticuloendothelial system [23]. Conversely, inadequate hepcidin allows in-
creased gastrointestinal absorption of iron and ultimately may lead to excess
iron sucient to result in organ toxicity [22,25,26].
Iron not bound to transferrin, also referred to as nontransferrin-bound
iron, damages the endocrine organs, liver, and heart [27]. Nontransferrin-
bound iron can result in myocyte damage leading to arrhythmias and
congestive heart failure, the primary causes of death in patients who have
thalassemia [9,10,28]. Appropriate chelation therapy and close monitoring
of cardiac siderosis can avoid this devastating complication (see the article
by Kwiatkowski elsewhere in this issue for discussion of iron chelators).

Changing demographics and carrier screening


The clinical spectrum of thalassemia in the developed world has changed
dramatically in the 3 decades since the introduction of deferoxamine chela-
tion [9,10]. In addition, new treatment options and prevention strategies to
avoid complications of the disease and its compulsory treatments, coupled
with immigration changes, have altered the demography of thalassemia in
North America [16,17]. Younger patients are now predominantly of Asian
descent, whereas the aging population of patients who have thalassemia
are of Mediterranean descent (Fig. 1). According to the United States Cen-
sus Bureau, the number of Asians increased signicantly from 1980 to a total
THALASSEMIA UPDATE 451

Fig. 1. Changing demographics of patients who have thalassemia in the North American Thal-
assemia Clinical Research Network. Patients of Asian descent predominate in the younger pop-
ulation. (From Vichinsky EP, Macklin EA, Waye JS, et al. Changes in the epidemiology of
thalassemia in North America: a new minority disease. Pediatrics 2005;116(6):e81825; with
permission.)

of 6.9 million in the census count in 1990. Coupled with other changes that
have occurred over the past few decades, it is estimated that up to 100 mil-
lion people of African, Hispanic, Southern and Eastern European, Middle
Eastern, and Asian ethnic backgrounds reside in the United States. Simi-
larly, it is estimated that approximately one sixth of the Canadian popula-
tion is foreign born. This includes a considerable inux of Asians: in the
1990s more than 2 million people immigrated to Canada, approximately
half from Asia. Many of the ethnic immigrants who relocated to North
America are carriers of globin gene mutations, which have important impli-
cations for carrier screening. Recent reports reveal births of children who
have severe a- or b-thalassemia in which appropriate screening and counsel-
ing was not oered to the parents [18].
Screening is inexpensive and simple but requires clinicians to be astutely
attentive and aware of potential carriers. A complete blood cell count iden-
ties microcytosis and hypochromia, which are present in nearly all thalas-
semia carriers at risk for having babies who have a severe thalassemia
syndrome. In an adult, a mean corpuscular volume of less than 80 fL and
mean corpuscular hemoglobin of less than 27 pg should alert a clinician
to perform further screening, specically hemoglobin electrophoresis and,
in some cases, globin genotype testing. In addition, b-thalassemia carriers
have elevated hemoglobin A2 on adult electrophoresis unless there is
concomitant iron deciency, which may falsely normalize the hemoglobin
452 CUNNINGHAM

A2 (HbA2) level [29]. Once conrmed, genetic counseling should be oered


to parents or potential parents [18].
A newborn hemoglobinopathy screen can be helpful in the recognition
and diagnosis of potential thalassemia syndromes. Severe b-thalassemia
with absent b production due to two b mutations has only fetal hemoglobin
(HbF) as there is no hemoglobin A (a2b2) produced. This abnormal HbF
only result should prompt a clinician to follow the babys hemoglobin and
growth to determine at what age the baby will need to initiate transfusions
to prevent the complications of severe anemia. The anticipated severity of
the a-thalassemia syndromes not always is determined easily by the results
of a newborn screen. These babies all have an elevated percentage of hemo-
globin Bart (g4) at birth but the level does not always correspond directly to
the number of aected genes. This should alert pediatricians, however, to
counsel parents to receive personal testing to determine the number of ab-
normal a-globin genes in each parent. If each parent has only one mutated
a-globin gene, the most clinically signicant outcome results in the birth of
a baby who has a two-gene a-thalassemia (a-thalassemia trait), which has
no clinical consequences. Conversely, if each parent has a two-gene muta-
tion on the same chromosome, termed a cis mutation, there exists a risk
for the most severe form of a-thalassemia, four-gene deletion, leading to
hydrops fetalis, which often results in fetal demise without in utero
transfusion [30,31].

Clinical complications
Transfusion-associated issues
Iron overload
The primary long-term complication of chronic RBC transfusions for
thalassemia is iron loading and the resultant parenchymal organ toxicity.
Cardiac iron-overload leading to cardiac failure or arrhythmias is the
most common fatal complication seen in chronically transfused patients
who have thalassemia [9]. Adherence to chelation can prevent cardiac dam-
age and death from cardiac injury. Patients who present with cardiac arryth-
mias or failure due to iron injury often can be rescued by continuous
infusion deferoxamine although adherence to the regimen of subcutaneous
deferoxamine delivered 24 hours per day, 7 days per week, is dicult [32].
In addition, the endocrine organs are exquisitely sensitive to the toxic
eects of iron and this may result in hypogonadotropic hypogonadism, pi-
tuitary damage, diabetes [4,8,33], osteopenia, and osteoporosis. Hypogona-
dotrophic hypogonadism is common in young adults who have TM and
is believed to contribute to low fertility in this population [34,35]. Addi-
tionally, cardiac complications of iron overload may exacerbate pregnancy
and delivery complications in women who have thalassemia. Case reports
and small published series reveal, however, that successful pregnancy and
THALASSEMIA UPDATE 453

delivery of healthy babies is possible in women who have TM [33,35,36].


Spontaneous pregnancy without hormonal assistance is reported [33,35].
Recent literature on hypogonadotrophic hypogonadism suggests that early
intervention with hormonal therapy and aggressive iron chelation therapy
to prevent permanent damage may help preserve innate fertility. The eect
of chelation on preservation of gonadal function currently is being investi-
gated. For example, deferasirox, which provides extended blood chelator
levels, may have a protective eect against toxicity to the endocrinologic
organs, but further research remains necessary.

Alloimmunization
Chronic transfusions may result in the development of anti-RBC anti-
bodies, alloantibodies and autoantibodies, in a variety of diseases [37]. Al-
though several studies have investigated the rates of alloimmunization in
patients who had sickle cell disease [37,38], the thalassemia population is
less well studied. Small retrospective analyses have suggested alloimmuniza-
tion rates of 2.7% to 37% in patients who had thalassemia [39,40]. Rates of
alloimmunization are suggested as higher for transfusions with donor/recip-
ient ethnic disparity [41] and in splenectomized patients [42]. Because of the
risk for alloimmunization and autoimmunization, it is recommended that
extended RBC antigen phenotyping be performed before initiation of
RBC transfusions, so that patients can be transfused safely in the event of
anti-RBC autoantibody or alloantibody formation. If a patient then de-
velops autoantibodies directed against ubiquitous RBC antigens or multiple
alloantibodies, blood that is matched more fully can be transfused more
safely. For sickle cell patients requiring chronic transfusions, Rh and Kell
antigen matching is considered standard of care and performed by many,
but not all, care centers. For patients who have thalassemia, this matching
strategy is not performed routinely. One recent study suggests, however,
that matching for Rh and Kell in this population can decrease the alloanti-
body rate by 53% [42]. Unlike most patients who have sickle cell disease,
patients who have thalassemia usually initiate chronic transfusions at 6
months to 2 years of age. Many clinicians believe that chronic transfusions
early in life may allow development of tolerance to foreign RBC antigens
and prevent development of alloimmunization. Further prospective studies
in this area are required to determine the appropriate transfusion strategy in
patients who have thalassemia.

Viral infection
The transmission of infections, in particular HIV, hepatitis B, and hepa-
titis C, remains a serious complication and a signicant problem in some de-
veloping countries [43]. For now the sole use of volunteer donors who have
no nancial incentive to donate and thus are likely to answer a detailed do-
nor questionnaire honestly provides the greatest protection against transfu-
sion transmission of infections. Additionally, serologic and nucleic acid
454 CUNNINGHAM

testing, used in the developed world, augment the safety of blood products
[44]. When these safety measures are in place, the risk for transfusion-trans-
mission of known infections is extraordinarily low [45].
The development of the hepatitis B vaccine, identication of the hepatitis
C virus (HCV), and a serologic test to screen donors has greatly minimized
the risk for transfusion-transmitted hepatitis B virus and HCV. The preva-
lence of HCV in patients who have thalassemia is disparate and depends
on the screening procedures and donor pool. In the developing world, the
prevalence is 20% to 64% [46], with recent data demonstrating continued ex-
posure and infection to patients who have thalassemia receiving transfusions,
including many pediatric patients [46]. In the North American population
studied in the National Heart, Lung, and Blood Institutesponsored Thalas-
semia Clinical Research Network, the prevalence of exposure was 70% in pa-
tients over 25 years of age but only 5% in patients under 15 years of age [4].
Chronic active hepatitis can lead to brosis, cirrhosis, and hepatocellular
carcinoma (HCC) if untreated [9]. Treatment with interferon-a and
ribavirin is the standard of care for patients who have chronic hepatitis C.
Because ribavirin causes hemolysis and thus increases transfusion require-
ments and concomitant iron exposure, the package guidelines for ribavirin
still recommend that it not be used to treat chronic hepatitis in patients who
are chronically transfused. Small studies have demonstrated, however, that
ribavirin can be given safely and eectively to patients who have thalasse-
mia [4750]. Because hepatic cirrhosis, liver failure, and HCC all are
potential consequences of chronic active hepatitis, the majority of clinicians
who care for these patients recommend treatment with interferon and
ribavirin.
HCV-infected patients who have thalassemia are living long enough to
develop prolonged chronic active hepatitis and be at risk for developing
HCC. A multicenter retrospective review by Borgna-Pignatti and colleagues
reported 22 patients who had HCC from a cohort of approximately 5000
patients who had thalassemia followed in 52 Italian centers [51]. These num-
bers likely underestimate, however, the true risk for cirrhosis and HCC to
patients who have thalassemia and are infected with hepatitis C, because
many succumbed to cardiac complications before living long enough to de-
velop frank cirrhosis or HCC [9]. Patients who do not have evidence of hep-
atitis C exposure should have annual screening for HCV. Patients who have
chronic hepatitis and are at risk for HCC should undergo routine screening
with serum a-fetoprotein and liver ultrasound because survival in patients
who have HCC is inversely proportional to the size of the tumor [52].

Thrombosis and hypercoagulable state


Data compiled from many series present compelling clinical evidence for
increased risk for thrombosis in patients who have b-TI, b-TM, a-thalasse-
mia syndromes, or hemoglobin E/b-thalassemia [5355]. The risk for
THALASSEMIA UPDATE 455

thromboses is increased in patients who are nontransfused or infrequently


transfused and in patients who are splenectomized. This is believed to result
in part from increased proportion of defective, innate RBCs [56]. Defective
RBCs have disrupted membranes resulting in exposure of negatively
charged lipids, including phosphatidylserine, on the external cell surface,
which are believed thrombogenic. In the absence of the splenic removal of
senescent cells, more of these defective cells are circulating, which increases
the risk for thrombosis. In addition, markers studied show increased platelet
activation, which also is believed to increase thrombotic risk [57]. The di-
culty in ascertaining absolute clinical risk and determining appropriate pre-
ventive strategies is that many patients reported in the literature do not have
exact transfusion regimens known [55]. More frequent transfusions result in
an increased ratio of normal transfused RBCs to disrupted innate RBCs,
thus decreasing the clinical risk for thrombosis based on current knowledge
of risk factors.
Data in hemolytic states, including sickle cell disease and thalassemia,
suggest depletion of nitric oxide resulting from chronic hemolysis and in-
creased plasma levels of free hemoglobin [58]. Nitric oxide is a smooth mus-
cle relaxant and decreased levels are believed to cause increased peripheral
vascular resistance and ultimately pulmonary hypertension. Nitric oxide
scavenging by free hemoglobin is implicated in the pulmonary arterial dis-
ease of sickle cell anemia and data suggest that this is a possible factor in
thalassemia. Studies aimed at increasing the levels of nitric oxide and de-
creasing pulmonary hypertension [59] are ongoing and will be critical in
determining clinical approaches to this problem in the aging thalassemia
population.

Improvements in prognosis and survival


Survival
Historically, patients who had thalassemia had a poor prognosis. In
a United States cohort born between 1960 and 1976, the median survival
was 17 years [32]. In an Italian cohort born in the mid-1960s, the median
survival was 12 years [9]. Remarkable and promising improvement in sur-
vival of patients who have thalassemia has been made, however, as demon-
strated in two reports by Pearson and collagues [60,61]. In the rst 1973
manuscript, before the era of deferoxamine chelation, they reported the
ages of 243 patients who had thalassemia in 12 North American centers.
In this cohort, 22% were younger than 5 years of age and 2.1% older
than 25. The precipitous decrease in number of living patients who had thal-
assemia began at 15 years of age. Just over a decade later, in 1985, the same
centers were surveyed and of the 303 patients, 11% were younger than 5
years and the population of patients older than 25 years had increased to
11% (P!.01%).
456 CUNNINGHAM

More recently, Borgna-Pignatti and colleagues [28] published cohort sur-


vival data for nearly 1100 patients who had TM from seven centers in Italy.
Kaplan-Meier survival curves were evaluated for 5-year birth cohorts
(Fig. 2). The curves demonstrated a statistically signicant dierence in sur-
vival in patients in the later birth cohorts. Patients born between 1960 and
1964 had a greater than 60% mortality rate at 30 years of age as compared
with the rate of 10% at 25 years of age in the cohort born from 1975 to 1979.
The majority of the deaths resulted from cardiac hemosiderosis.

Curative therapies
Bone marrow transplantation
Successful cure of b-thalassemia by bone marrow transplantation rst
was reported by Thomas and associates in 1982 [62]. Subsequently, several
centers have explored the use of this modality as denitive therapy [6365].
The most extensive published experience with bone marrow transplantation
in b-thalassemia is that of Lucarelli and coworkers in Italy [65]. Early on
they reported thalassemia-free survival of only 53% in the older patients
who had thalassemia with hepatomegaly, liver brosis, and inadequate pre-
transplant chelation [65]. More recent data, however, even in patients con-
sidered at high-risk for transplant, demonstrate signicant improvements
[66]. Survival for the most recently transplanted 33 pediatric patients was
93% and the rate of graft rejection decreased from 30% to 8% [66]. Adults
treated with this protocol demonstrated improved thalassemia-free survival,
from 62% to 67%, and transplant-related mortality decreased from 37%
but still was signicant at 27%.

Fig. 2. Kaplan-Meier survival curves after the rst decade of life by birth cohort (A) and gender
(B) of 977 patients who had thalassemia in Italy. This demonstrates the dramatic improvement
in the younger cohort born between 1985 and 1997. Because they have not had sucient time to
age into the fourth and fth decades, it makes it dicult to determine the life expectancy of pa-
tients treated in developed countries who have access to appropriate chelation therapy and
adequate medical care. (From Borgna-Pignatti C, Rugolotto S, De SP, et al. Survival and
complications in patients with thalassemia major treated with transfusion and deferoxamine.
Haematologica 2004;89(10):118793; with permission.)
THALASSEMIA UPDATE 457

On the basis of available data, bone marrow transplantation may be rec-


ommended to patients receiving adequate chelation without evidence of liver
disease who have an HLA-matched sibling donor. Many of these patients
can be cured [6769]. Chronic graft-versus-host disease still is a potential
long-term complication of successful allogeneic transplantation. A current
limitation to the general applicability of this therapy is the availability of
a related HLA-matched donor. Only one in four siblings on average is
HLA identical. Improved management of graft-versus-host disease and
the development of technologies for bone marrow transplantation from un-
related donors may expand the pool of potential donors in the near future.
The use of cord blood stem cells and unrelated donors is extending the do-
nor pool and number of patients who may receive bone marrow transplan-
tation [7072].

Gene therapy
Treatment of hematologic and other diseases through gene therapy is
actively studied in murine and primate models [73,74]. The obstacles to
success of this therapeutic modality and the availability of this therapy
for humans include the need for improved eciency of gene delivery, reg-
ulated and sustained expression of introduced genes, and insertion of the
gene into non-oncogenic sites. Although gene therapy is an area of active
clinical investigation, the aforementioned obstacles currently preclude its
use in the management of thalassemia or sickle cell anemia. Nonetheless,
the successful transfer of globin genes into hematopoietic cells of primates
and humans has been demonstrated and is encouraging [75]. A phase I
human gene therapy trial for thalassemia and sickle cell disease has
been initiated in France but clinical data are not yet available.

References
[1] Weatherall DJ, Clegg JB. Historical perspectives: the many and diverse routes to our current
understanding of the thalassaemias. In: Weatherall DJ, Clegg JB, editors. The thalassaemia
syndromes. 4th edition. Oxford (England): Blackwell Science; 2001. p. 362.
[2] Deisseroth A, Nienhuis A, Turner P, et al. Localization of the human alpha-globin structural
gene to chromosome 16 in somatic cell hybrids by molecular hybridization assay. Cell 1977;
12(1):20518.
[3] Deisseroth A, Nienhuis A, Lawrence J, et al. Chromosomal localization of human beta glo-
bin gene on human chromosome 11 in somatic cell hybrids. Proc Natl Acad Sci U S A 1978;
75(3):145660.
[4] Cunningham MJ, Macklin EA, Neufeld EJ, et al. Complications of beta-thalassemia major
in North America. Blood 2004;104(1):349.
[5] Cao A. Carrier screening and genetic counselling in beta-thalassemia. Int J Hematol 2002;
76(Suppl 2):10513.
[6] Galanello R, Sanna MA, Maccioni L, et al. Fetal hydrops in Sardinia: implications for ge-
netic counselling. Clin Genet 1990;38(5):32731.
458 CUNNINGHAM

[7] Calleja EM, Shen JY, Lesser M, et al. Survival and morbidity in transfusion-dependent thal-
assemic patients on subcutaneous desferrioxamine chelation. Nearly two decades of experi-
ence. Ann N Y Acad Sci 1998;850:46970.
[8] Mohammadian S, Bazrafshan HR, Sadeghi-Nejad A. Endocrine gland abnormalities in thal-
assemia major: a brief review. J Pediatr Endocrinol Metab 2003;16(7):95764.
[9] Borgna-Pignatti C, Cappellini MD, De SP, et al. Survival and complications in thalassemia.
Ann N Y Acad Sci 2005;1054:407.
[10] Olivieri NF, Nathan DG, MacMillan JH, et al. Survival in medically treated patients with
homozygous beta-thalassemia. N Engl J Med 1994;331(9):5748.
[11] Clegg JB, Weatherall DJ. Thalassemia and malaria: new insights into an old problem. Proc
Assoc Am Physicians 1999;111(4):27882.
[12] Weatherall DJ. Thalassaemia and malaria, revisited. Ann Trop Med Parasitol 1997;91(7):
88590.
[13] Flint J, Hill AV, Bowden DK, et al. High frequencies of alpha-thalassaemia are the result of
natural selection by malaria. Nature 1986;321(6072):74450.
[14] Kanavakis E, Tzotzos S, Liapaki K, et al. Molecular basis and prevalence of alpha-thalas-
semia in Greece. Birth Defects Orig Artic Ser 1988;23(5B):37780.
[15] Falusi AG, Esan GJ, Ayyub H, et al. Alpha-thalassaemia in Nigeria: its interaction with
sickle-cell disease. Eur J Haematol 1987;38(4):3705.
[16] Vichinsky EP. Changing patterns of thalassemia worldwide. Ann N Y Acad Sci 2005;1054:
1824.
[17] Vichinsky EP, Macklin EA, Waye JS, et al. Changes in the epidemiology of thalassemia in
North America: a new minority disease. Pediatrics 2005;116(6):e81825.
[18] Lorey F, Cunningham G, Vichinsky EP, et al. Universal newborn screening for Hb H disease
in California. Genet Test 2001;5(2):93100.
[19] Logothetis J, Economidou J, Constantoulakis M, et al. Cephalofacial deformities in thalas-
semia major (Cooleys anemia). A correlative study among 138 cases. Am J Dis Child 1971;
121(4):3006.
[20] Kearney SL, Nemeth E, Neufeld EJ, et al. Urinary hepcidin in congenital chronic anemias.
Pediatr Blood Cancer 2007;48(1):5763.
[21] Detivaud L, Nemeth E, Boudjema K, et al. Hepcidin levels in humans are correlated with
hepatic iron stores, hemoglobin levels, and hepatic function. Blood 2005;106(2):7468.
[22] Gardenghi S, Marongiu MF, Ramos P, et al. Ineective erythropoiesis in beta-thalassemia is
characterized by increased iron absorption mediated by down-regulation of hepcidin and up-
regulation of ferroportin. Blood 2007;109(11):502735.
[23] Ganz T. Hepcidin, a key regulator of iron metabolism and mediator of anemia of inamma-
tion. Blood 2003;102(3):7838.
[24] Weinstein DA, Roy CN, Fleming MD, et al. Inappropriate expression of hepcidin is associ-
ated with iron refractory anemia: implications for the anemia of chronic disease. Blood 2002;
100(10):377681.
[25] Breda L, Gardenghi S, Guy E, et al. Exploring the role of hepcidin, an antimicrobial and iron
regulatory peptide, in increased iron absorption in beta-thalassemia. Ann N Y Acad Sci
2005;1054:41722.
[26] De FL, Daraio F, Filippini A, et al. Liver expression of hepcidin and other iron genes in two
mouse models of beta-thalassemia. Haematologica 2006;91(10):133642.
[27] Porter JB, Abeysinghe RD, Marshall L, et al. Kinetics of removal and reappearance of non-
transferrin-bound plasma iron with deferoxamine therapy. Blood 1996;88(2):70513.
[28] Borgna-Pignatti C, Rugolotto S, De SP, et al. Survival and complications in patients with
thalassemia major treated with transfusion and deferoxamine. Haematologica 2004;
89(10):118793.
[29] Aghai E, Shabbad E, Quitt M, et al. Discrimination between iron deciency and heterozy-
gous beta-thalassemia in children. Am J Clin Pathol 1986;85(6):7102.
[30] Lie-Injo LE, Jo BH. A fast-moving haemoglobin in hydrops foetalis. Nature 1960;185:698.
THALASSEMIA UPDATE 459

[31] Kan YW, Allen A, Lowenstein L. Hydrops fetalis with alpha thalassemia. N Engl J Med
1967;276(1):1823.
[32] Ehlers KH, Giardina PJ, Lesser ML, et al. Prolonged survival in patients with beta-thalas-
semia major treated with deferoxamine. J Pediatr 1991;118(4 Pt 1):5405.
[33] Karagiorga-Lagana M. Fertility in thalassemia: the Greek experience. J Pediatr Endocrinol
Metab 1998;11(Suppl 3):94551.
[34] Skordis N, Petrikkos L, Toumba M, et al. Update on fertility in thalassaemia major. Pediatr
Endocrinol Rev 2004;2(Suppl 2):296302.
[35] Skordis N, Christou S, Koliou M, et al. Fertility in female patients with thalassemia.
J Pediatr Endocrinol Metab 1998;11(Suppl 3):93543.
[36] Pafumi C, Farina M, Pernicone G, et al. At term pregnancies in transfusion-dependent beta-
thalassemic women. Clin Exp Obstet Gynecol 2000;27(34):1857.
[37] Hmida S, Mojaat N, Maamar M, et al. Red cell alloantibodies in patients with haemoglobi-
nopathies. Nouv Rev Fr Hematol 1994;36(5):3636.
[38] Olujohungbe A, Hambleton I, Stephens L, et al. Red cell antibodies in patients with homo-
zygous sickle cell disease: a comparison of patients in Jamaica and the United Kingdom.
Br J Haematol 2001;113(3):6615.
[39] Economidou J, Constantoulakis M, Augoustaki O, et al. Frequency of antibodies to various
antigenic determinants in polytransfused patients with homozygous thalassaemia in Greece.
Vox Sang 1971;20(3):2528.
[40] Wang LY, Liang DC, Liu HC, et al. Alloimmunization among patients with transfusion-de-
pendent thalassemia in Taiwan. Transfus Med 2006;16(3):2003.
[41] Vichinsky EP, Earles A, Johnson RA, et al. Alloimmunization in sickle cell anemia and
transfusion of racially unmatched blood. N Engl J Med 1990;322(23):161721.
[42] Singer ST, Wu V, Mignacca R, et al. Alloimmunization and erythrocyte autoimmunization
in transfusion-dependent thalassemia patients of predominantly asian descent. Blood 2000;
96(10):336973.
[43] Moroni GA, Piacentini G, Terzoli S, et al. Hepatitis B or non-A, non-B virus infection in
multitransfused thalassaemic patients. Arch Dis Child 1984;59(12):112730.
[44] Allain JP, Thomas I, Sauleda S. Nucleic acid testing for emerging viral infections. Transfus
Med 2002;12(4):27583.
[45] OBrien SF, Yi QL, Fan W, et al. Current incidence and estimated residual risk of transfu-
sion-transmitted infections in donations made to Canadian Blood Services. Transfusion
2007;47(2):31625.
[46] Ansar MM, Kooloobandi A. Prevalence of hepatitis C virus infection in thalassemia and
haemodialysis patients in north Iran-Rasht. J Viral Hepat 2002;9(5):3902.
[47] Butensky E, Pakbaz Z, Foote D, et al. Treatment of hepatitis C virus infection in thalasse-
mia. Ann N Y Acad Sci 2005;1054:2909.
[48] Inati A, Taher A, Ghorra S, et al. Ecacy and tolerability of peginterferon alpha-2a with
or without ribavirin in thalassaemia major patients with chronic hepatitis C virus infection.
Br J Haematol 2005;130(4):6446.
[49] Telfer PT, Garson JA, Whitby K, et al. Combination therapy with interferon alpha and ri-
bavirin for chronic hepatitis C virus infection in thalassaemic patients. Br J Haematol 1997;
98(4):8505.
[50] Wonke B, Hobrand AV, Bouloux P, et al. New approaches to the management of
hepatitis and endocrine disorders in Cooleys anemia. Ann N Y Acad Sci 1998;850:
23241.
[51] Borgna-Pignatti C, Vergine G, Lombardo T, et al. Hepatocellular carcinoma in the thalas-
saemia syndromes. Br J Haematol 2004;124(1):1147.
[52] Ren FY, Piao XX, Jin AL. Ecacy of ultrasonography and alpha-fetoprotein on early de-
tection of hepatocellular carcinoma. World J Gastroenterol 2006;12(29):46569.
[53] Borgna PC, Carnelli V, Caruso V, et al. Thromboembolic events in beta thalassemia major:
an Italian multicenter study. Acta Haematol 1998;99(2):769.
460 CUNNINGHAM

[54] Cappellini MD. Coagulation in the pathophysiology of hemolytic anemias. Hematology Am


Soc Hematol Educ Program 2007;2007:748.
[55] Eldor A, Rachmilewitz EA. The hypercoagulable state in thalassemia. Blood 2002;99(1):
3643.
[56] Borenstain-Ben YV, Barenholz Y, Hy-Am E, et al. Phosphatidylserine in the outer leaet of
red blood cells from beta-thalassemia patients may explain the chronic hypercoagulable state
and thrombotic episodes. Am J Hematol 1993;44(1):635.
[57] Eldor A, Krausz Y, Atlan H, et al. Platelet survival in patients with beta-thalassemia. Am
J Hematol 1989;32(2):949.
[58] Reiter CD, Gladwin MT. An emerging role for nitric oxide in sickle cell disease vascular ho-
meostasis and therapy. Curr Opin Hematol 2003;10(2):99107.
[59] Machado RF, Martyr S, Kato GJ, et al. Sildenal therapy in patients with sickle cell disease
and pulmonary hypertension. Br J Haematol 2005;130(3):44553.
[60] Pearson HA, Rink L, Guiliotis DK. Thalassemia major in Connecticut: a 20-year study of
changing age distribution and survival. Conn Med 1994;85(5):25960.
[61] Pearson HA, Guiliotis DK, Rink L, et al. Patient age distribution in thalassemia major:
changes from 1973 to 1985. Pediatrics 1987;80(1):537.
[62] Thomas ED, Buckner CD, Sanders JE, et al. Marrow transplantation for thalassaemia.
Lancet 1982;2(8292):2279.
[63] Hongeng S, Pakakasama S, Chuansumrit A, et al. Reduced intensity stem cell transplanta-
tion for treatment of class 3 Lucarelli severe thalassemia patients. Am J Hematol 2007;
82(12):10958.
[64] La NG, Argiolu F, Giardini C, et al. Unrelated bone marrow transplantation for beta-thal-
assemia patients: the experience of the Italian Bone Marrow Transplant Group. Ann N Y
Acad Sci 2005;1054:18695.
[65] Lucarelli G, Galimberti M, Polchi P, et al. Bone marrow transplantation in patients with
thalassemia. N Engl J Med 1990;322(7):41721.
[66] Sodani P, Gaziev D, Polchi P, et al. New approach for bone marrow transplantation in pa-
tients with class 3 thalassemia aged younger than 17 years. Blood 2004;104(4):12013.
[67] Lucarelli G, Andreani M, Angelucci E. The cure of the thalassemia with bone marrow trans-
plantation. Bone Marrow Transplant 2001;28(Suppl 1):S113.
[68] Walters MC, Quirolo L, Trachtenberg ET, et al. Sibling donor cord blood transplantation
for thalassemia major: experience of the Sibling Donor Cord Blood Program. Ann N Y
Acad Sci 2005;1054:20613.
[69] Bhatia M, Walters MC. Hematopoietic cell transplantation for thalassemia and sickle cell
disease: past, present and future. Bone Marrow Transplant 2008;41(2):10917.
[70] Adamkiewicz TV, Szabolcs P, Haight A, et al. Unrelated cord blood transplantation in chil-
dren with sickle cell disease: review of four-center experience. Pediatr Transplant 2007;11(6):
6414.
[71] Adamkiewicz TV, Boyer MW, Bray R, et al. Identication of unrelated cord blood units for
hematopoietic stem cell transplantation in children with sickle cell disease. J Pediatr Hematol
Oncol 2006;28(1):2932.
[72] Walters MC. Cord blood transplantation for sickle cell anemia: bust or boom? Pediatr
Transplant 2007;11(6):5823.
[73] Nishino T, Tubb J, Emery DW. Partial correction of murine beta-thalassemia with a gam-
maretrovirus vector for human gamma-globin. Blood Cells Mol Dis 2006;37(1):17.
[74] Rivella S, May C, Chadburn A, et al. A novel murine model of Cooley anemia and its rescue
by lentiviral-mediated human beta-globin gene transfer. Blood 2003;101(8):29329.
[75] Sadelain M, Lisowski L, Samakoglu S, et al. Progress toward the genetic treatment of the
beta-thalassemias. Ann N Y Acad Sci 2005;1054:7891.
Pediatric Hematologic Epidemiology of childhood cancer
Malignancies 1% of new cancer cases in the U.S. occurs age
19 yr or younger (12,400 cases/yr)
Lakshmanan Krishnamurti, MD Second leading cause of death <14 yrs(10.6% of
Associate Professor Pediatrics all deaths)
Division of Hematology/Oncology/BMT Children dying of cancer lose an avg. of 69.5 yr
Childrens Hospital of Pittsburgh 5-yr survival for children with cancer, improved
1) objectives of lecture from 56% in 1974 to 75% in 2000
To recognize clinical presentation and diagnosis of common pediatric
hematological malignancies Cancer survivors <19 yrs. 90,000174,000
To recognize the treatment, outcomes and long term effects of treatment of
common pediatric hematological malignancies Late effects of cancer and treatment are an
2) key words : Childhood cancer, Leukemia, acute lymphoid; Leukemia, Acute important focus of research
Myeloid; Leukemia Chronic Myeloid; Lymphoma, Hodgkins; Lymphoma,
Non-Hodgkins; stem cell transplantation; GVHD; graft versus leukemia)

Pediatric Cancer Distribution Top Ten Pediatric Cancers


Leukemia Histology Incidence per million (under 15)
CNS
Lymphoma
ALL 32.8
Neuroblastoma Astrocytoma/glioma 19.2
Other Neuroblastoma 10.2
ST Sarcoma Wilms tumor 8.4
Wilms NHL 7.6
Bone
Medullo/CNS PNET 6.5
Retinoblastoma
Liver
AML 6.3
Hodgkin lymphoma 6.1
Rhabdomyosarcoma 4.8
Retinoblastoma 4.0
SEER (1973-1999)

Unique challenges in treating


Age specific Incidence of childhood cancer
childhood cancer
Radiation, surgery, and chemotherapy may adversely
affect growth and development
May cause serious long-term medical and psychosocial
effects.
Relative rarity of specific types of childhood cancer
Sophisticated technology and expertise required for
diagnosis, treatment, and monitoring of late effects
ASll children with cancer should be treated on
standardized clinical protocols in clinical research
settings whenever possible.

1
Genetic conditions with
Children's Oncology Group
increased cancer susceptibility
Facilitates clinical, biologic, and epidemiologic Genetic conditions account for less than 5% of cases with
research in 238 affiliated institutions in North cancer include:
America. neurofibromatosis types 1 and 2
Down syndrome,
Centers have the required facilities and
expertise and are committed to learning more Beckwith-Wiedemann syndrome
about and defining the optimal treatment of Tuberous sclerosis
pediatric malignancy through participation in Von Hippel-Lindau disease
national clinical trials. Xeroderma pigmentosum
Coordinated treatment efforts have substantially Ataxia-telangiectasia
increased survival for many children with cancer. Nevus basal cell carcinoma syndrome
Li-Fraumeni (P53) syndrome.

Survival in childhood cancer Childhood Leukemia


Cancer 5-year survival Leukemia is the most common childhood cancer and is
usually acute.
All malignancies 75% The most common subtype is acute lymphoblastic
leukemia (ALL), ( 75% to 80% cases)
combined
acute myeloid leukemia (AML) accounts for
Acute lymphoblastic 78% approximately 20%.
leukemia Males are affected by leukemia slightly more often than
females in all but infants' age groups.
Acute Myeloid leukemia 40%
Incidence of ALL is highest between ages 2 and 5
years.
Non Hodgkins 92% Incidence of AML constant during childhood, with slight
peaks in the first 2 years of life and in late adolescence.
Hodgkins 73%
Dario Campana Ching-Hon Pui. CHILDHOOD LEUKEMIA. In Abeloff:
Clinical Oncology, 3rd ed., Copyright 2004 Churchill Livingstone

Childhood Leukemia :Clinical Findings Childhood Leukemia : Extramedullary involvement

Physical signs and symptoms of Common sites of extramedullary


thrombocytopenia and anemia are common. involvement in ALL include liver, spleen,
Neutropenia can lead to severe infection. thymus, and lymph nodes.
Bone pain and arthralgia caused by leukemic Skin, gums, and the head and neck area
infiltration more common in ALL are typical sites of extramedullary disease
Bone Pain can be especially severe in young in AML.
children.
Infiltration of the central nervous system
can be found in both ALL and AML.

2
Differential Diagnosis
Differential Diagnosis of Childhood ALL

Nonmalignant
Malignancies
Conditions NBL
Infectious Lymphoma
Mononucleosis Neuroblastoma
JRA Retinoblastoma
ITP Rhabdomyosarcoma
Pertussis;
Parapertussis Leukemoid Reaction
Aplastic Anemia
ALL

FAB L1 Morphology

M1: < 5% blasts


M2: 5-
5-25% blasts
M3: > 25% blasts

FAB L3 Morphology
FAB L2 Morphology

Only L3 (1%) morphology dictates


immunophenotype, the presence of
surface Ig and t(8;14)

3
Acute Myelogenous Leukemia
Childhood Leukemia : Differential Diagnosis
The acute onset of petechiae, ecchymoses, and
bleeding could suggest idiopathic thrombocytopenic
purpura.

Both acute leukemia and aplastic anemia can present


with pancytopenia and complications associated with
bone marrow failure.

Infectious mononucleosis and other viral infections can


be confused with ALL.

Bone pain, arthralgia, and occasionally arthritis can


mimic juvenile rheumatoid arthritis, rheumatic fever,
other collagen diseases, or osteomyelitis.
These cells should be myeloperoxidase
positive; PAS negative Childhood ALL should also be distinguished from
pediatric small round cell tumors that involve the bone
marrow.

Flow Cytometry for diagnosis of


Factors Impacting Prognosis in ALL acute leukemia
B-lineage ( precursor B)
NCI Risk Grouping CD10, 19, 20, 22 & Tdt
Std Risk: Age < 10 and WBC <50,000/l CD 10- correlates with 11q23
High Risk: Age > 10 and/or WBC > 50k
T-ALL
Immunophenotype
CD10, 2, 5, 7, intracellular CD3,Tdt
Rapidity of response
Cytogenetics
AML
CD 38, 33, 13, 15, 4+, Tdt-
Treatment protocols
Compliance B-ALL
Drug sensitivity, pharmacogenetics CD 20, 22, 38 & SIg+, CD10-

ALL: Rate Of Initial Response Is Strong Day 29 Flow MRD Correlates with Day 8
Predictor Of Event-
Event-free-
free-survival Morphologic Assessment of Response
Marrow % patients 6 year EFS
Morphology Day 14 M3
50

40
M3
M1 (< 5%) 84% 72% %
30
p < 0.001 MRD + M2
(n=1016) 20
M2 (5-25%) 8% 32% M1 M2
10 M1

0
M3 (> 25%) 8% 40% >0.01% >0.1 p<.001

MRD level
Steinherz, et al: JCO 14: 389-398, 1996 Borowitz, 2002

4
Clinical Significance of Minimal Residual Effect of end-induction MRD on early and late
Disease in patients with ALL during Induction relapse

Borowitz, M. J. et al. Blood


Borowitz, M. J. et al. Blood
2008;111:5477-5485
2008;111:5477-5485

Copyright 2008 American Society of Hematology. Copyright restrictions may apply. Copyright 2008 American Society of Hematology. Copyright restrictions may apply.

Genotype Correlates with Outcome


Childrens Oncology Group CNS Classification
100
CNS 1: cytospin negative
CNS 2: cytospin positive, < 5 WBC
TEL (n =176) Trisomies 4,10,17 (n = 746)
80
TLP+: cytospin positive, < 5 WBC, > 10
t(1;19) (n = 139) RBC or > 5 WBC, corrected with
Probability

60
t(4;11) (n = 44) algorithm
40
CNS 3: cytospin positive, > 5 WBC, +
RBC, not corrected by algorithm or
t(9;22) (n=132)
20 Tris 4,10,17
4 Yr EFS (%)
92.1
SE (%)
1.1 Cranial nerve palsy, brain/eye involvement or
TEL 89.0 3.1
t(1;19) 68.9 4.1
B-precursor ALL hypothalamic syndrome
t(4;11) 49.9 11.2
t(9;22) 27.5 4.4
0
0 1 2 3 4 5 6 7 8 9 10 11 12 13 14 15 16
Years Followed 10/2001

Emergencies
Components of Therapy
Hyperleukocytosis
Respiratory distress/abnormal mental Induction
status-if the child would rather not lay
down, its ok! CNS Prophylaxis/Consolidation
Renal dysfunction- K, PO4, uric acid- Delayed Intensification
adequate access and hydration,
Rasburicase w/o alkalinzation or allopurinol Maintenance Therapy
with alkalinzation (what goes in must come
out)
Leukocytopheresis
Infectious complications

5
Induction Therapy
Induction Three or Four Drugs with IT Rx
Dexamethasone Improves EFS for Std Risk Pts
Three drugs ( Vincristine, steroid and Dexamethasone
Asparaginase) for standard risk patients 1
0.95
Better CNS entry;
0.9 PRED
(N=530)

Fourth drug ( Anthracycline) added for 0.85


more potent

PROBABILITY
0.8
0.75

high risk group 0.7 DEX Superior EFS


0.65 (N=530)

0.6
6 YEAR EFS (p=.003):
PRED
DEX
77.3%
84.8%
Increased risk of
0.55
0.5
0 1 2 3 4 5 6 7 8
bone disease
Increased mortality
with anthracycline
YEARS FOLLOWED

Bostrom et al, 2003

CNS Prophylaxis Delayed Intensification


Standard risk patients receive intrathecal
chemotherapy as the mainstay of CNS Combination chemotherapy similar to
prophylaxis induction and consolidation
Radiotherapy Targets residual leukemic cells post-
Vasculopathies induction which are relatively/absolutely
Oncogenesis resistant
Growth/Thyroid Hormone deficiency
Reserved for high risk patients for T-ALL, CNS3
Induction, consolidation, delayed intennsification
totals approximately 6 months
Duration of therapy after completion of
consolidation two years for girls, 3 years for boys

Maintenance Therapy T-ALL- 20% of Childhood ALL

Designed to provide ongoing leukemia Teenage boys, bulk disease, respiratory


distress, higher incidence CNS relapse
cell kill with minimal host toxicity
Better outcome with more aggressive therapy
Backbone includes 6-MP/MTX
NCI Standard Risk
Steroid/vincristine pulses are often 1952/1962 71.0%
included 9404 87.4%
Ongoing asparagine depletion may be NCI High Risk
advantageous 1961 (Aug. BFM) 76.9%
9404 (HDMTX) 76.4%
Antiangiogenic thru methyl-MP riboside

6
Infant Leukemia Despite the success of primary
Overall EFS 25 to 53% at 4 yrs therapy, failure is common in
based on 6 published trials childhood ALL
Poor outcome associated with:
Age < 6 months, < 91 days 2000 new cases yearly
CD10 neg
EFS 73% @ 7 years
MLL gene rearrangements
>400 1st relapses
Prednisone poor response
WBC > 50, 100 or 300,000 >200 2nd relapses
Transplant is controversial > 200 3rd/subsequent relapse

Outcome after Relapse: B-lineage


Introduction of New Agents in Patients with
Site and Time- MRC UKALLR1 Very High Risk of Treatment Failure

Site Time from Dx EFS at 5yrs


Reduction Re- Re- Re-
EMR > 2.5 yrs 77% Phase induction induction induction
Block 1 Block 2 Block 3
BMR > 2.5 yrs 57% Pilot I VCR, Pred, CTX, VP-16, MTX ARA-C, ASP
PEG, Adr
EMR 2 - 2.5 yrs 44%

MRD
EMR < 2 yrs 39% MRD

BMR 2 2.5 yrs 35% New Agent Pilot


VCR, Pred, CTX, VP-16, MTX ARA-C, ASP
BMR < 2 yrs 7% PEG, Adr Time

MRD Correlate with 4


Note: T-ALL carries a > 2x increased risk of 2nd relapse month EFS
Epratuzumab
Lawson et al Br J Haem, 2000

POG 9412 for CNS Relapse


MRD vs Time of First Relapse EFS by Duration of CR1
p = 0.0009
< 18 months (early relapse)
42.0 13.1
Timepoint MRD+ MRD+ p value 18 months (late relapse)
77.3 7.7%
ER LR POG 9412 Event-Free Survival
by CR1 Duration
1 31/41 14/29 .018
100
(76%) (48%)
80
2 18/22 9/21 .009
Probability (%)

60 <18 mos >=18 mos

(82%) (43%)
40

3 9/15 3/19 .01 20


(60%) (16%)
0
0 1 2 3 4 5 6 7
Raetz, ASH 2004 Years

7
Indications for SCT in ALL PROBABILITY OF SURVIVAL AFTER
ALLOGENEIC TRANSPLANTS FOR ALL, AGE <20 YEARS BY
DONOR TYPE AND REMISSION STATUS, 1994-1999

ALL in first remission for very high risk


100
disease (Ph+, severe hypodiploidy,
induction failure) 80

PROBABILITY, %
HLA-identical sibling, CR1 (N = 561)

ALL in second CR 60

Early relapse (<36 mo from diagnosis) or HLA-identical sibling, CR2+ (N = 962)

40 Unrelated, CR1 (N = 280)


any recurrent T cell Unrelated, CR2+ (N = 805)

If late relapse data supports SCT only with 20


MSD P = 0.0001

0
0 1 2 3 4 5 6
YEARS
SUM02_6.
ppt

Unrelated donor umbilical cord blood


Improved Survival in Childhood ALL
transplantation for hematological by Study Era
malignancies
cumulative incidence of neutrophil engraftment 100 1996-2000
by day 42 was 79.9%
Estimated Survival Percentage

(n=3421)
1989-1995
acute grades III/IV GVHD by day 100 19.5% 80 (n=5121)
1983-1988
chronic GVHD at 2 years was 20.8% (n=3711)
60 1978-1983
HR matching decreased the probability of (n=2984)
1975-1977
severe acute GVHD. 40 (n=1313)
1972-1975
cumulative incidence of relapse at 2 years19.9% (n=936)
20 1970-1972
probabilities of 6-month and 2-year survivals (n=499)

were 67.4% and 49.5%. 0


1968-1970
(n=402)
0 2 4 6 8 10 12
Kurtzberg et al. Blood, 15 November 2008, Vol. 112, No. 10, pp. 4318-4327.
Years From Study Entry

Long-term toxicities may include neurocognitive AML risk groups


skills, fertility, second malignancies, bone disease
Good Risk: Any patient with favorable genetic abnormalities
However, (t(8;21); inv(16)/t(16;16) including those molecularly detected,
Children not radiated, surviving, event-free > 10 irrespective of marrow status after Course 1 or the presence of
yrs, can expect normal long-term survival other genetic abnormalities (About 30% of patients)
Standard Risk: Any patient not in either good risk of poor risk
groups, i.e., neither favorable nor adverse genetic abnormalities
and not more than 15% blasts in the bone marrow after Course 1
(About 50% of patients)
Poor Risk: Any patient with more than 15% blasts in bone
marrow after Course 1 or with adverse genetic abnormalities (-5, -
7, del(5q), abn(3q), complex) and without favorable genetic
abnormalities (About 20% of patients)

Reproduced with permission,


Pui et al, NEJM 2004

8
Results of Different Approaches to
Treatment of childhood AML Intensification
EFS: European Trials
Most protocols for AML include remission
induction and consolidation therapy, although
other postremission therapy differs widely
between studies.
Autologous hematopoietic stem cell
transplantation is not recommended. POG 9421 EFS:

Allogeneic transplantation appears to improve EFS: CCG-2891 100


STD. DAT/NO CSA VS. H.D. DAT/CSA
P=0.028

overall survival in AML, although the indications 80 STD DAT/NO CSA H.D. DAT/CSA

Probability (%)
for this procedure during first remission are 60 43 +/- 5.5%

debated.
40

20
31 +/- 5.9%
0
0 1 2 3 4 5 6 7
Years

Arceci, R.J. and Golub, T., Chapter 20, Pizzo and Poplack, Principles and Practice of Pediatric Oncology, 2006

Conclusions from CCG-2891 Conclusions from CCG-2891


Not all pediatric patients with AML need intensive timing 1.00

induction therapy or SCT (e.g.: Down Syndrome). 0-2 years (n=94), 86%

Post-Rem. Therapy *RFS from EOI 0.75


Overall Survival Probability

BMT 67% >2-4 years (n=56), 70%

HD-AraC/Asp 91% 0.50

p<0.05

Induction Therapy EFS* 0.25


*> 4 years (n=9), 28%

*p = 0.0001
Standard Timing 79%**
0.00
0.00
Intensive Timing 50% 0 1 2 3 4 5 6 7 8

*3 years p<0.05 **Excludes BMT Years from Study Entry

From CCG 2891 Trial Courtesy of W. Woods Gamis et al., JCO, 21: 3415-3422, 2003.

AML with Monosomy 7


Very high risk patients need alternative treatments
EFS

Years

From CCG 2891 Trial Courtesy of W. Woods From CCG 2891 Trial Courtesy of W. Woods From CCG 2

9
Prognostic Factors: AML Newly Diagnosed AML/MDS
BFM Low and High Risk Categories Risk Group Stratification
POG Cytogenetic Analysis
Low Risk are about 37% of patients
N EFS
Achieve remission, M1 with Auer rods,
M2 with WBC < 20,000/ul, M3, M4 with Inv 16 28 ~59%
eosinophilia t(8;21) 56 ~45%
Achieve about a 90% 3 year RFS All Patients 478 ~35%
compared to 42% for High Risk Group 11q23 88 ~20%
Results not completely confirmed by others
Raimondi, S. C. et al., Blood, 94:3707-3716; 1999.

Prognostic Factors and


Risk Group Stratification
Risk Groups and Outcome from AML 12
MRC 12 Survival (5 year)
Low Risk
t(8;21), t(15;17), inv(16) 73%
Risk Group Survival Relapse Rate
Intermediate Risk 44%
Normal with neither UF or F Good Risk 76% 25%
cytogenetics
Standard Risk 48% 52%
High Risk 17%
Monosomy 5 or 7, del(5q) Poor Risk 21% 73%
abn(3q) or complex karyotype
(> 5 unrelated cytogenetic abns)

Occult leukemia predicts relapse Occult leukemia related to survival

San Miguel et al, Blood 2001 San Miguel et al, Blood 2001

10
Conclusions From San Miguel et al
Study Pediatric CCG 2961 Study
Median number of residual leukemia cells during
remission was higher in patients 1. Studied 178 of 252 patients with
requiring an additional cycle of chemotherapy for CR responsive disease (71%)
with higher WBC at diagnosis
2. This group of patients comparable to
with leukemias characterized by adverse
other patients on study
cytogenetics
Multivariate analysis showed level of residual leukemia 3. 23 patients (13%) had MRD by flow
had greatest independent prognostic impact (=0.002), cytometry
followed by cytogenetic abnormalities (p=0.03).

Survival With and Without Occult Leukemia at Detection of leukemic blast cells in patients
End of Consolidation* by multidimensional flow cytometry was
predictive of a more rapid relapse

69%
Time to Relapse (from achievement of CR)

MRD + 168.5 days


MRD - 293 days
41%

(p=0.008)

*Allogeneic Transplants Censored Sievers et al., Blood, 9:3398; 2003. Sievers et al., Blood, 9:3398; 2003.

Acute Promyelocytic Leukemia


Time Dependent Univariate and Multivariate Cox
Models Demonstrating Relative Risk (RR) of Distinctive Features Morphologic Variants
Leukemic Relapse or Death by Risk Factors
APML APML - Microgranular

Sievers et al., Blood, 9:3398; 2003. Images from http://www.ashimagebank.org

11
Acute Promyelocytic Leukemia Acute Promyelocytic Leukemia
Treatment Treatment
- Optimal treatment includes induction and -Hyperleukocytosis
consolidation with anthracycline +/- cytarabine
and etoposide based chemotherapy plus ATRA - Leukopheresis contraindicated
- Maintenance therapy required and best using - Start treatment with chemotherapy plus
ATRA plus low dose 6-mercaptopurine plus ATRA and possibly hydroxyurea and
methotrexate consider dexamethasone. Do not use
ATRA alone which may result in further
- Allogeneic or autologous bone marrow differentiation and often an increase in
transplantation not recommended in CR1 WBC.

Acute Promyelocytic Leukemia Acute Promyelocytic Leukemia


Treatment Outcomes: Maintenance with ATRA plus low dose
chemotherapy improves overall relapse rate and EFS
- ATRA Syndrome (Better termed Differentiation Syndrome)
- Characterized by shortness of breath, pulmonary rales and/or
infiltrates during first three weeks of ATRA treatment
Combined ATRA + CT
- Treatment includes dexamethasone (10 mg IV q 12 hours and
continue for 14 days) and discontinuing ATRA until pulmonary 2 yr relapse: 7.4%
Relapse

symptoms are stable. 2 yr EFS: 93%+3%


- Patients can usually be re-treated with ATRA once symptoms 2 yr Survival: 93%
resolve
Particularly important in
- Always consider other etiologies such as infection, fluid overload, patients with high WBC
pulmonary embolus, conjestive heart failure)

- Incidence similar in children and adults


Fenaux et al., Blood, 94:1192; 1999

Chronic Myelogenous Leukemia Chronic Myelogenous Leukemia

Accounts for about 5% of all childhood Treatment


leukemia Can CML be cured without HSCT?
80% of pediatric cases occur after age Non-Hematopoietic Stem Cell Transplant Approaches
STI571 (Gleevac, Imatinib mesylate) first line
4 years but CML as been reported in
Interferon Alpha (Regular and Pegylated forms)
the younger patients
Hydroxyurea, Busulphan
Ionizing radiation is known etiologic Chemotherapy alone or in combination
factor
Hematopoietic Stem Cell Transplantation

Fu & Altman, Chapter 21, Pizzo and Poplack, Principles and Practice of Pediatric Oncology, 2006

12
CML: Proposed Schema for Pediatric Patients
Chronic Myelogenous Leukemia
Imatinib Dosing in Children
- 260 to 340 mg/m2 provides drug exposures similar to
the 400 or 600 mg adult dosage levels

Imatinib Resistance
- Mutations in bcr-abl
- Amplification of bcr-abl
- Increased expression of alternative survival pathways
- Noncompliance
- Alternative bcr-abl inhibitors (e.g., Dasatinib) Pulsipher, M., Pediatric
Blood & Cancer 43:1;2004

Juvenile Myelomonocytic Juvenile Myelomonocytic


Leukemia/Juvenile Chronic Myeloid Leukemia/Juvenile Chronic Myeloid
Leukemia Leukemia
Hypersensitivity of JMML cells to GM-CSF results in failure to thrive, malaise, bleeding, and fever .
spontaneous growth factor independent production of splenomegaly, pallor, hepatomegaly, and
MCFU lymphadenopathy.
increased frequency in neurofibromatosis (NF) patients cough, tachypnea, and wheezing, accompanied
Philadelphia chromosome is not present by an interstitial pulmonary pattern
Other chromosome abnormalities have been noted in An eczematoid rash may occur, particularly in
30% of patients. patients with neurofibromatosis
presence of monosomy 7 in patients with JMML thrombocytopenia, anemia, and an elevated
suggests more aggressive course.
leukocyte count (usually <100,000/mm3) with a
Most are younger than 2 years of age, and 95% are prominent monocytosis.
younger than 4 years of age.
Occasional blasts.

Juvenile Myelomonocytic Juvenile Myelomonocytic


Leukemia/Juvenile Chronic Myeloid Leukemia/Juvenile Chronic Myeloid
Leukemia Leukemia
Bone marrow shows myeloid and erythroid hyperplasia 30% have true myeloid blast crisis.
with myeloid/erythroid ratios of 2:1 to 5:1 (lower than in
ACML). Dysplastic features are not present. intensive chemotherapy, with or without
increased fetal hemoglobin levels and glucose-6- allogeneic BMT, are disappointing.
phosphate dehydrogenase activity levels, and decreased
I antigen, carbonic anhydrase activity, and hemoglobin 13-cis retinoic acid farnesyltransferase
A2, all of which are consistent with an increased number
of fetal RBCs. inhibitors (which interfere with RAS
MCV is not typically elevated even in the presence of function and an inhibitor of GM-CS, are in
these fetal-type RBCs. clinical trial
Leukocyte alkaline phosphatase activity is decreased.

13
Hodgkin Disease Hodgkin Disease
Epidemiology (Reed-Sternberg cell)

Bimodal Mid-20s and after 50 yr..


Rarely < 5 yr..
Three distinct forms
Childhood (< 14 yr.) M>F
Mixed cellularity more common
1-10% of cells in tumor
Associated with lower socioeconomic status
Hodgkin Reed-Sternberg (HRS) Variants mononuclear,
Young Adult (15-34 yr.) M~F lacunar, or mummified
Nodular sclerosis more common Molecular studies have demonstrated the majority to be of B-
Associated with higher socioeconomic status cell origin
Classical Hodgkin Lymphoma (cHL) CD45 (-), CD15 (+), CD30
Older Adult (55-74 yr.) (+), EMA (-), ALK (-)
Only 10-20% will express B-cell markers (CD19, CD20, CD79a)
About 50% will be positive for EBV gene expression

Hodgkin Disease Hodgkin Disease


Classification Clinical Presentation
A no associated symptoms
Ann Arbor Staging B
Fever (>38o C) usually > 3 consecutive days
Stage I single site/nodal region of involvement Unexplained weight loss of 10% - preceding 6 months
Stage II 2 or more sites/nodal regions on same side of Drenching night sweats
diaphragm X bulky disease
Stage III Sites/nodal regions involved on both sides of mediastinum > 1/3 intrathoracic diameter
diaphragm > 10 cm other nodal site
Stage IV diffuse or disseminated involvement in one or E extranodal involvement
more extralymphatic organs Pruritus
More frequent with advanced disease
(E) designation for extralymphatic involvement in any
Resolves with treatment HD
stage
Alcohol-induced pain
(S) - splenic involvement Occurs within minutes of EtOH consumption
in enlarged nodal areas
Resolves with treatment of HD

Hodgkin Disease Hodgkin Disease


Treatment Treatment
Low-risk (Stage I-IIA, no bulk, no B Sx)
2-4 cycles of chemotherapy +/- low-dose involved field radiation
90-95% of all children/adolescence with HD can
be cured Intermediate-risk (Stage I-II with bulk or B Sx,
Stage IIIA & IVA)
Multi-agent chemotherapy given in short, pulsed 4-6 cycles + involved field radiation
cycles of chemotherapy. High-Risk (Stage IIIB and IVB)
Radiation involved field (15 25 Gy) 4-8 cycles (?intensive chemotherapy) + involved field radiation
Therapy directed to minimize late effects Refractory or Recurrent disease
Intensified chemotherapy
Autologous BMT
?Allogeneic BMT

14
Therapy Related Late Effects
Burkitt Lymphoma

Childrens Oncology Group


http://www.survivorshipguidelines.org
Therapeutic agent related
Cytoxan infertility (Males), 2nd malignancy
Ara-C, Methotrexate and CNS therapy neurocognitive and
growth development
40% of NHL in children/adolescents
Steroids and Methotrexate bone growth and development,
cataracts Mature B-cell (TdT-), CD10(+/-), CD19(+), CD20(+), sIg (+)
Doxorubicin heart dysfunction C-MYC (+) - t(8;14), t(2;8), t(8;22)
Vincristine peripheral neuropathies Burkitt leukemia (B-ALL) (20% of cases)
Etoposide 2nd malignancy (MDS/AML) treated same as lymphoma
Radiation 2nd malignancy (Breast, skin and soft tissue Abdominal disease most common presentation
cancers), Infertility, growth, endocrinopathies, heart Head & neck second most common site
dysfunction Extranodal disease very common
Very rapidly growing (t1/2 = 18-24 hr)

Diffuse Large B-cell Lymphoma Lymphoblastic Lymphoma

10% of NHL in children/adolescents 30% of NHL in children/adolescents


Mature B-cell - CD20(+), CD19 (+) CD10(+/-),sIg (+/-), CD30(+/-),EMA Precursor T- or B- cell
(+/-), CD45(+), CD15 (-) TdT (+) in all cases,
Can be difficult to distinguish between: T-cell (CD2, CD5, CD7), B-cell (CD10, CD19, CD20)
HD (R-S cells can be seen in DLBCL)
?Same disease as ALL treated the same
ALCL
Much overlap with Burkitts-like T-cell LL
Many variants: 85-90% of LL
Primary mediastinal B-cell lymphoma (genetically like HD), 90% Stage III/IV mediastinal mass common
T-cel/histiocytic rich B-cell lymphoma B-cell LL
Anaplastic DLBCL (CD30+, but not ALCL) 10-15% of LL
>90% Stage I/II (skin and bone common)

Lymphoproliferative disease in Adult NHL Occurring in


Immunodeficient Patient Children/Adolescents
Immunodeficient patients (HIV, inherited
immunodeficiency and iatrogenic post-transplant) at Unclear if same disease
10-100 fold increased risk for NHL Follicular Lymphoma rarely bcl-2 (+)
Associated with T-cell deficiency not B-cell or PMN DLBCL often c-myc (+)
deficiencies Types seen in children/adolescents
Extranodal disease very common primary CNS Cutaneous - Mycosis fungoides, SQ panniculitic T-cell very
indolent
lymphoma
Primary CNS lymphoma if diagnosed a immunodeficiency
Majority associated with EBV evaluation is warranted
Most common histology is DLBCL < BL <<< HD, ALCL Low-grade/intermediate grade B-NHL - follicular lymphoma,
and mature T-NHL MALT, mantle cell NHL
Mature T-cell NHL hepatosplenic T-cell, NK/T-cell lymphoma
Tolerate standard therapy less well (very aggressive and poor prognosis)

15
Non-Hodgkin Lymphoma Non-Hodgkin Lymphoma
Staging Staging
Murphy Staging System for Pediatric NHL
As opposed to Hodgkin Disease, NHL does not Stage I a single extranodal tumor or single nodal site excluding
spread in a predictable pattern through any abdominal or mediastinal disease.
lymphatics
Stage II a) single extranodal tumor with regional node
Ann Arbor (used for adult NHL) has not been as involvement, b) > 2 extranodal tumor without nodal involvement or > 2
useful for pediatric NHL because: nodal regions on same side of diaphragm, c) primary GI tumor +/-
Does not accurate reflect prognosis nodes that is grossly (>95%) resected.
Extranodal involvement for common in pediatric NHL
Pediatric NHL is felt to be a disseminated process Stage III a) > 2 extranodal tumors or nodal regions on opposite sides
even with clinically localized disease of diaphragm, b) all primary thoracic disease (mediastinum, pleura,
Microscopic disease is often present in marrow or thymus) c) GI disease not grossly resected, d) paraspinal and
peripheral blood at diagnosis epidural tumors.

Stage IV CNS and/or marrow involvement

Non-Hodgkin Lymphoma Medical Emergencies with NHL


Treatment Treatment
Lymphoblastic lymphoma Superior mediastinal syndrome (SMS)
Treated like ALL Presentation - Symptoms and Physical Findings
Induction, consolidation, maintenance Cough/dyspnea 68%
Total duration of therapy > 2 yr. Dysphagia/orthopnea 63%
Wheezing 31%
All other NHL Hoarseness 9%
Treated with short pulsed cycles of multi-agent Facial edema 12%
chemotherapy Chest pain 6%
Duration 3 52 weeks Treatment
CNS prophylaxis Attempt to make tumor diagnosis
Standard is all NHL pediatric NHL patients receive Often do not tolerate anesthesia
CNS prophylaxis Marrow or pleural effusions
? Role of prophylactic cranial XRT Emergency XRT +/- steroids

Non-Hodgkin Lymphoma
Outcome
Localized (Stage I/II)
> 95% EFS
LL needs to be treated like leukemia
Burkitt/DLBCL (Stage III/IV)
90% EFS
Poor prognostics factors
Primary mediastinal disease
CNS and marrow involvement
Lymphoblastic Lymphoma (Stage III/IV)
85-90% EFS
Anaplastic Large Cell Lymphoma (Stage III/IV)
75% EFS
Refractory/Recurrent NHL
Very poor outcome difficult to get BL and LL back into good CR
BMT (autologous vs allogeneic)

16
Pediatr Clin N Am 55 (2008) 2151

Acute Myeloid Leukemia


Jerey E. Rubnitz, MD, PhDa,*, Brenda Gibson, MDb,
Franklin O. Smith, MDc
a
Department of Oncology, St. Jude Childrens Research Hospital,
MS 260, 332 N. Lauderdale, Memphis, TN 38105, USA
b
Department of Paediatric Haematology, Royal Hospital for Sick Children,
Yorkhill, G3 8SJ, Glasgow, Scotland, UK
c
Division of Hematology/Oncology, University of Cincinnati College of Medicine,
Cincinnati Childrens Hospital Medical Center

Acute myeloid leukemia (AML) is a heterogeneous group of leukemias


that arise in precursors of myeloid, erythroid, megakaryocytic, and mono-
cytic cell lineages. These leukemias result from clonal transformation of
hematopoietic precursors through the acquisition of chromosomal rear-
rangements and multiple gene mutations. New molecular technologies
have allowed a better understanding of these molecular events, improved
classication of AML according to risk, and the development of molecularly
targeted therapies. As a result of highly collaborative clinical research by
pediatric cooperative cancer groups worldwide, disease-free survival
(DFS) has improved signicantly during the past 3 decades [115]. Further
improvements in the outcome of children who have AML probably will
reect continued progress in understanding the biology of AML and the
concomitant development of new molecularly targeted agents for use in
combination with conventional chemotherapy drugs.

Epidemiology and risk factors


Approximately 6500 children and adolescents in the United States
develop acute leukemia each year [16]. AML comprises only 15% to 20%
of these cases but accounts for a disproportionate 30% of deaths from acute
leukemia. The incidence of pediatric AML is estimated to be between ve
and seven cases per million people per year, with a peak incidence of 11 cases

JER was supported, in part, by the American Lebanese Syrian Associated Charities
(ALSAC).
* Corresponding author.
E-mail address: jeffrey.rubnitz@stjude.org (J.E. Rubnitz).

0031-3955/08/$ - see front matter 2008 Elsevier Inc. All rights reserved.
doi:10.1016/j.pcl.2007.11.003 pediatric.theclinics.com
22 RUBNITZ et al

per million at 2 years of age [1719]. Incidence reaches a low point at age
approximately 9 years, then increases to nine cases per million during adoles-
cence and remains relatively stable until age 55 years. There is no dierence in
incidence between male and female or black and white populations [16]. There
is, however, evidence suggesting that incidence is highest in Hispanic children,
intermediate in black children (5.8 cases per million), and slightly lower in
white children (4.8 cases per million) [2023]. The French-American-British
(FAB) classication subtypes of AML are equally represented across ethnic
and racial groups with the exception of acute promyelocytic leukemia
(APL), which has a higher incidence among children of Latin and Hispanic
ancestry.
During the years between 1977 and 1995, the overall incidence of AML
remained stable, but there was a disturbing increase in the incidence of
secondary AML as the result of prior exposure to chemotherapy and radi-
ation [2430]. This risk remains particularly high among individuals exposed
to alkylating agents (cyclophosphamide, nitrogen mustard, ifosfamide,
melphalan, and chlorambucil) and intercalating topoisomerase II inhibitors,
including the epipodophyllotoxins (etoposide).
Most children who have de novo AML have no identiable predisposing
environmental exposure or inherited condition, although a number of envi-
ronmental exposures, inherited conditions, and acquired disorders are asso-
ciated with the development of AML. Myelodysplastic syndrome and AML
reportedly are associated with exposure to chemotherapy and ionizing
radiation and also to chemicals that include petroleum products and organic
solvents (benzene), herbicides, and pesticides (organophosphates) [3136].
A large number of inherited conditions predispose children to the develop-
ment of AML. Among these are Down syndrome, Fanconi anemia, severe
congenital neutropenia (Kostmann syndrome), Shwachman-Diamond
syndrome, Diamond-Blackfan syndrome, neurobromatosis type 1, Noonan
syndrome, dyskeratosis congenita, familial platelet disorder with a predispo-
sition to AML (FDP/AML), congenital amegakaryocytic thrombocytope-
nia, ataxia-telangiectasia, Klinefelters syndrome, Li-Fraumeni syndrome,
and Bloom syndrome [3740].
Finally, AML has been associated with several acquired conditions
including aplastic anemia [41,42], myelodysplastic syndrome, acquired
amegakaryocytic thrombocytopenia [43,44], and paroxysmal nocturnal
hemoglobinuria.

Pathogenesis
AML is the result of distinct but cooperating genetic mutations that
confer a proliferative and survival advantage and that impair dierentiation
and apoptosis [4547]. This multistep mechanism for the pathogenesis of
AML is supported by murine models [48,49], the analysis of leukemia in
twins [5053], and the analysis of patients who have FDP/AML syndrome
ACUTE MYELOID LEUKEMIA 23

[54]. Mutations in a number of genes that confer a proliferative and/or


survival advantage to cells but do not aect dierentiation (Class I muta-
tions) have been identied in AML, including mutations of FLT3, ALM,
oncogenic Ras and PTPN11, and the BCR/ABL and TEL/PDGFbR gene
fusions. Similarly, gene mutations and translocation-associated fusions that
impair dierentiation and apoptosis (Class II mutations) in AML include
the AML/ETO and PML/RARa fusions, MLL rearrangements, and muta-
tions in CEBPA, CBF, HOX family members, CBP/P300, and co-activators
of TIF1. AML results when hematopoietic precursor cells acquire both Class
I and Class II genetic abnormalities. Although only one cytogenetic or molec-
ular abnormality has been reported in many cases of AML, new molecular
tools now are identifying multiple genetic mutations in such cases.
Accumulating data suggest that the leukemic stem cell arises at dierent
stages of dierentiation and involves heterogeneous, complex patterns of
abnormality in myeloid precursor cells [5560]. The leukemic stem cell, also
called the self-renewing leukemia-initiating cell, is located within both the
CD34 and CD34 cell compartments and is rare (0.2200 per 106 mononu-
clear cells) [6164]. A recent study of pediatric AML suggested that patients
who have FLT3 abnormalities in less mature CD34 CD38 precursor cells
are less likely to survive than patients who have FLT3 mutations in more
mature CD34 CD38 cells (11% versus 100% at 4 years; P .002) [65].
Although sample sizes in this study were small, this result demonstrates the
heterogeneity of genetic abnormalities in various stem cell compartments
and suggests a worse outcome when less mature precursor cells harbor these
abnormalities.

Clinical presentation and diagnosis


The presentation of childhood AML reects signs and symptoms that
result from leukemic inltration of the bone marrow and extramedullary
sites. Replacement of normal bone marrow hematopoietic cells results in
neutropenia, anemia, and thrombocytopenia. Children commonly present
with signs and symptoms of pancytopenia, including fever, fatigue, pallor,
bleeding, bone pain, and infections. Disseminated intravascular coagulation
may be observed at presentation of all AML subtypes but is much more
frequent in childhood APL. Inltration of extramedullary sites can result
in lymphadenopathy, hepatosplenomegaly, chloromatous tumors (myelo-
blastomas and granulocytic sarcomas), disease in the skin (leukemia cutis),
orbit, and epidural space, and, rarely, testicular involvement. The central
nervous system is involved at diagnosis in approximately 15% of cases
[66]. Patients who have high white blood cells counts may present with signs
or symptoms of leukostasis, most often aecting the lung and brain.
A diagnosis is suggested by a complete blood cell count showing pancy-
topenia and blast cells and is conrmed by examination of the bone marrow.
The diagnosis and subtype classication of AML is based on morphologic,
24 RUBNITZ et al

cytochemical, cytogenetic, and uorescent in situ hybridization analyses,


ow cytometric immunophenotyping, and molecular testing (eg, FLT3
mutation analysis).

Treatment of childhood acute myeloid leukemia


The prognosis of children who have AML has improved greatly during
the past 3 decades (Fig. 1). Rates of complete remission (CR) as high
as 80% to 90% and overall survival (OS) rates of 60% now are reported
(Table 1) [1]. This success reects the use of increasingly intensive induction
chemotherapy followed by postremission treatment with additional anthra-
cyclines and high-dose cytarabine or myeloablative regimens followed by
stem cell transplantation (SCT). The drugs used in the treatment of AML
have changed little, but renement of their delivery and striking advances
in supportive care have allowed administration of optimally intensive ther-
apy with less morbidity and mortality. Better postrelapse salvage therapy
also has contributed to the improvement in OS.
Treatment of AML in children generally is based on an anthracycline,
cytarabine, and etoposide regimen given as a minimum of four cycles of
chemotherapy. A recent report compared the results of anthracycline, cytar-
abine, and etoposide regimens used by 13 national study groups [1]. The
regimens diered in many ways, including the cumulative doses of drugs,
the choice of anthracycline, the number and intensity of blocks of treatment,
and the intrathecal chemotherapy used for central nervous system (CNS)
prophylaxis. Treatment generally was risk stratied, although the denition
of risk groups varied, as did the indications for SCT. Despite the varying
strategies, results are relatively similar (see Table 1) [2]. Many groups now

Fig. 1. Overall survival of children younger than 15 years of age who had acute myeloid leuke-
mia treated in MRC trials during the past 3 decades.
ACUTE MYELOID LEUKEMIA 25

achieve CR rates of 80% to 90%, relapse rates of 30% to 40%, event-free


survival (EFS) rates of 50%, and OS rates of 60% [315].
Because of the small number of pediatric patients who have AML, many
important questions have not been addressed in the context of randomized
trials. The unresolved issues include the optimal intensity of chemotherapy,
the optimal anthracycline, the optimal dose of cytarabine, the cumulative
dose of anthracycline that minimizes cardiotoxicity without compromising
outcome, the role of allogeneic SCT in rst CR, and the use of risk-directed
therapy.

Induction and consolidation therapy


The most favorable outcomes are achieved by the use of a relatively high
cumulative dose of either anthracycline or cytarabine (see Table 1) [1,2]. The
schedule and timing of intensication also are important. The Childrens
Cancer Group (CCG) reported that intensively timed induction therapy
(the second cycle delivered 10 days after the rst cycle) was more advanta-
geous than standard therapy (the second cycle delivered 14 or more days af-
ter the rst cycle, dependent on bone marrow status and cell-count recovery)
[4,67]. Both the CR and EFS rates were signicantly higher with intensively
timed dosing, regardless of postremission therapy, suggesting that the depth
of remission may profoundly aect survival. The benet derived from early
intensication, whether achieved by time sequencing or by adjusting cytar-
abine and etoposide doses to achieve a targeted plasma level, may be lost,
however, if prolonged neutropenia and thrombocytopenia cause unaccept-
able delays in subsequent treatment [9,13]. The intensication of early ther-
apy beyond a certain threshold therefore is unlikely to improve outcome and
may even be detrimental to OS [13].
In a Medical Research Council (MRC) study, an additional course of
postremission chemotherapy (four versus ve courses in total) provided
no advantage to patients already receiving intensive treatment [5], suggest-
ing a plateau in the benet of conventional postremission chemotherapy.
If such a plateau is conrmed, it is likely that any additional antileukemic
eect will have to come from alternative approaches, such as targeted or cel-
lular therapies.
Certain anthracyclines are favored for their perceived greater antileuke-
mic eect and/or their lower cardiotoxicity, but no anthracycline agent
has been demonstrated to be superior. The MRC found daunorubicin and
mitoxantrone to be equally ecacious but mitoxantrone to be more myelo-
suppressive [5]. Idarubicin is used commonly because in vitro and preclinical
studies suggest that it oers a greater clinical benet because of its faster cel-
lular uptake, increased retention, and lower susceptibility to multidrug resis-
tant glycoprotein [68,69]. In addition, its main metabolite, idarubicinol, has
a prolonged plasma half-life (54 hours) and has antileukemic activity in the
cerebrospinal uid [70]. In the Berlin-Frankfurt-Munster (BFM) AML 93
26
Table 1
Outcome data from 13 national groups for patients younger than 15 years of age who had acute myeloid leukemia
% of total
number of
patients who
Early % 5-Year % 5-Year Death Cumulative underwent
Study Number Non- death Complete event-free overall rate in doses of ara-C, allogeneic
(years of of patients responders rate response survival survival complete etoposide, and stem cell
enrollment) enrolled (%) (%) (%) (SE) (SE) response (%) anthracyclinesa transplantation
AIEOP92 160 5 6 89 54 (4) 60 (4) 7 No strict 29
(19922001) protocol
guidelines

RUBNITZ
AML-BFM93 427 10 7 83 51 (3) 58 (2) 4 41.1 g/m2 7
(19931998) 950 mg/m2
300400 mg/m2

et al
CCG2891 (19891995) 750 18 4 78 34 (3) 47 (4) 15 14.6 g/m2 25
1100 mg/m2
180 mg/m2
DCOG-ANLL 92/94 78 8 10 82 42 (6) 42 (6) 16 33.2 g/m2 27
(19921998) 950 mg/m2
400 mg/m2
EORTC-CLG 58,921 166 13 2 84 48 (4) 62 (4) 6 23.3229.32 g/m2 20
(19932000) 1350 mg/m2
380 mg/m2
GATLA-AML90 179 11 20 70 31 (4) 41 (4) 7 41.1 g/m2 3
(19901997) 1450 mg/m2
300 mg/m2
LAME91 (19911998) 247 5 4 91 48 (4) 62 (4) 6 9.813.4 g/m2 30
400 mg/m2
460 mg/m2
NOPHO-AML93 223 5 2 92 50 (3) 66 (3) 2 49.661.3 g/m2 25
(19932001) 1600 mg/m2
300375 mg/m2
PINDA-92 151 5 26 68 36 36 4 7.64 g/m2 d
(19921998) 450 mg/m2
350 mg/m2
POG8821 (19881993) 511 19 4 77 31 (2) 42 (2) 8 55.7 g/m2 13
2250 mg/m2
360 mg/m2
PPLLSG98 104 13 8 80 47 (5) 50 (5) 10 7.015.1 g/m2 Not reported
(19982002) 450950 mg/m2
420600 mg/m2

ACUTE MYELOID LEUKEMIA


St. Jude-AML91 62 16 3 79 44 (15) 57 (11) ? 3.8 g/m2 Not given
(19911997) 1200 g/m2
270 mg/m2
UK MRC AML10 303 3 4 93 49 58 10 10.6 g/m2 20
(19881995) 5001500 mg/m2
550 mg/m2
UK MRC AML12 455 4 4 92 56 66 6 4.634.6 g/m2 8
(19952002) 1500 mg/m2
300610 mg/m2
Abbreviations: AIEOP, Associazione Italiana Ematologia Oncologia Pediatrica; BFM, Berlin-Frankfurt-Munster; CCG, Childrens Cancer Group;
DCOG, Dutch Childhood Oncology Group; EORTC-CLG, European Organization for the Research and Treatment of CancerChildren Leukemia Group;
GATLA, The Argentine Group for the Treatment of Acute Leukemia; LAME, Leucemie Aigue Myeloblastique Enfant); NOPHO, Nordic Society of Pedi-
atric Haematology and Oncology; PINDA, the National Program for Antineoplastic Drugs for Children; POG, Pediatric Oncology Group; PPLLSG, Polish
Pediatric Leukemia/Lymphoma Study Group; UK MRC, United Kingdom Medical Research Council.
a
Cumulative dose of anthracyclines was calculated by applying the following arbitrary conversion factors to obtain daunorubicin equivalents: idarubicin,
5; mitoxantrone, 5; doxorubicin, 1. Some groups (Leucemie Aique Myeloide Enfant and the Medical Research Council in the United Kingdom) also
administered amsacrine, which is not included in calculated total anthracycline exposure.

27
28 RUBNITZ et al

trial, induction therapy with idarubicin, cytarabine, and etoposide (AIE)


resulted in signicantly greater blast-cell clearance at day 15 than induction
with daunorubicin, cytarabine, and etoposide (ADE) (P .01) but did not
improve 5-year OS (51% with AIE versus 50% with ADE; P .72) or EFS
(60% for AIE versus 57% for ADE; P .55) [71]. Similarly, the Australian
and New Zealand Childrens Cancer Study Group reported that idarubicin
and daunorubicin were equally ecacious, but idarubicin was more toxic
[72]. The addition of cyclosporin A to induction chemotherapy to inhibit
P-glycoproteinmediated anthracycline eux did not prolong the duration
of remission or improve OS in children [73].
Another important question is whether the cumulative dose of anthracy-
clines can be reduced safely without compromising survival. Although
cumulative doses above 375 mg/m2 increase the risk of cardiotoxicity,
EFS is lower in protocols that use lower doses of anthracycline [1,2]. Opti-
mal results may be achievable with a cumulative dose of approximately 375
to 550 mg/m2 if high-dose cytarabine is used in postremission therapy [1,2].
The full impact of cardiotoxicity, particularly late cardiotoxicity, also is
poorly dened. In the MRC AML10 protocol, which delivered a high cumu-
lative anthracycline dose (550 mg/m2), 9 of 341 registered patients died of
acute cardiotoxicity (all after a cumulative dose of 300 mg/m2); 7 of the 9
deaths occurred during an episode of sepsis. Subclinical decits in cardiac
function would have gone undetected in the absence of cardiac monitoring
[74]. Minimizing cardiotoxicity is important, however, and cardioprotectant
agents and liposomal anthracyclines with reduced cardiotoxicity are being
tested.
The use of high-dose cytarabine in postremission therapy seems to be im-
portant in improving survival, but the optimal dose has not been deter-
mined. Core binding factor (CBF) leukemias may respond particularly
well to multiple courses of high-dose cytarabine [75].

Central nervous systemdirected therapy


The impact of CNS involvement on EFS is not well dened
[8,9,11,13,76,77]. Most pediatric clinical trial groups use intrathecal chemo-
therapy for CNS prophylaxis, employing either one or three agents and
various doses. Not all pediatric groups routinely use intrathecal CNS
prophylaxis [9], however, and few adult groups do. The correlation between
the type of CNS treatment given and the incidence of CNS relapse is
not clear. The CNS relapse rate seems to be around 2% for isolated CNS
relapse and between 2% and 9% for combined CNS and bone marrow
relapse [2,410]. The low rate of CNS relapse may reect both the use of
intrathecal chemotherapy and the CNS protection aorded by high-dose
cytarabine and by idarubicin, both of which can penetrate the CNS [70].
Cranial irradiation, because of its sequelae, is not widely used as prophy-
laxis. It is used currently only by the BFM Study Group, which observed
ACUTE MYELOID LEUKEMIA 29

an increase in CNS and systemic relapse in patients who did not receive
cranial irradiation in the AML BFM 87 trial [78]. The current AML
BFM 98 trial is exploring reduction of the dose of cranial irradiation to limit
late sequelae. The necessity of cranial irradiation for patients who have CNS
involvement at presentation or CNS relapse is unproven. Many groups
reserve cranial irradiation for patients whose CNS is not cleared of leukemic
cells by intrathecal and intensive systemic chemotherapy [4,11,13].

Maintenance therapy
Maintenance therapy is no longer used in the treatment of AML, having
failed to demonstrate benet except in BFM studies. Patients who have
APL, however, do seem to benet from antimetabolite maintenance treat-
ment given with all-trans retinoic acid (ATRA). In patients who have
non-APL AML, maintenance treatment showed no benet in two random-
ized studies (Leucemie Aigue Myeloblastique Enfant 91 and CCG 213);
these studies even suggested that maintenance therapy may be deleterious
when intensive chemotherapy is used and may contribute to clinical drug
resistance and treatment failure after relapse [9,79].

Stem cell transplantation


SCT is the most successful curative treatment for AML; it produces a strong
graft-versus-leukemia eect and can cure even relapsed AML. Its potential
benet, however, must be weighed against the risk of transplantation-related
mortality and the late sequelae of transplantation. SCT has become a less
attractive option as the outcomes of increasingly intensive chemotherapy
and postrelapse salvage therapy have improved. Furthermore, although
SCT is reported to provide a survival advantage for patients in rst CR, stud-
ies so far have used matched sibling donors, who are available to only about
one in four patients. Although experienced groups have reported comparable
outcomes with alternative donors, it is too early to determine whether their
wider use will result in greater transplantation-related mortality.
The role of allogeneic SCT, particularly whether it should be done during
rst CR or reserved for second remission, remains the most controversial issue
in pediatric AML. Competing factors, particularly risk group, may tip the bal-
ance in favor of SCT or intensive chemotherapy. Most groups agree that chil-
dren who have APL, AML and Down syndrome or AML and the t(8;21) or
inv(16) are not candidates for SCT in rst CR, but opinions dier about
patients in the standard-risk and high-risk categories. The trend in Europe
[79] is to reduce the use of SCT in rst CR, but in the United States [80]
SCT in rst CR is supported. Both views have been reported recently [8082].
In the absence of randomized, controlled trials comparing allogeneic SCT
with postremission intensive chemotherapy, biologic randomization or
donor versus no donor studies are accepted as the least biased compari-
son methods, but even these are open to criticism. Much of the trial data
30 RUBNITZ et al

used to support the benets of SCT and intensive chemotherapy are old and
do not reect current improvements in SCT and intensive chemotherapy.
A meta-analysis [83] of studies enrolling patients younger than 21 years of
age between 1985 and 2000 that recommended SCT if a histocompatible
family donor were available found that SCT from a matched sibling donor
reduced the risk of relapse signicantly and improved DFS and OS.
The MRC AML10 (included in the meta-analysis) and AML12 studies
combined (relapse risk did not dier between the trials; P .3) showed a sig-
nicant reduction in relapse risk (2P 0.02) but no signicant improvement
in DFS (2P 0.06) or OS (2P 0.1) [5]. MRC AML10 is typical of a num-
ber of trials in which SCT signicantly reduced the risk of relapse, but the
resulting improvement in survival was not statistically signicant (68% ver-
sus 59%; P .3). The small number of pediatric patients in AML10 hinders
meaningful interpretation, but at 7 years follow-up SCT recipients (children
and adults) who had a suitable donor showed a signicant reduction in
relapse risk (36%, versus 52% in patients who did not have a suitable donor;
P .0001) and a signicant improvement in DFS (50%, versus 42% in
patients who did not have a suitable donor; P .001) but no signicant
improvement in OS (55% versus 50%; P .1) [84]. The reduction in relapse
risk was seen in all risk and age groups, but the signicant benet in DFS
was seen only in the cytogenetic intermediate-risk group (50% versus
39%; P .004). The 86 children who had a donor, 61 of whom (71%)
underwent SCT, had no survival advantage, and children who did not un-
dergo SCT were salvaged more easily [5].
The lack of benet found for pediatric SCT in the MRC trials mirrors the
experience of the BFM [3,85]. CCG trial 2891, however, showed a signicant
survival advantage for patients who underwent allogeneic SCT versus autol-
ogous SCT (60% versus 53%; P .002) or chemotherapy (60% versus 48%;
P .05) as postremission treatment, although autologous SCT provided no
advantage over intensive chemotherapy [86]. The benet was most marked in
patients who had received intensively timed induction chemotherapy. The
CCG analysis was not a true intent-to-treat comparison, however. Although
it included patients whether or not they received SCT, it did not include all
patients who lacked a donor; instead, it included only patients who lacked
a donor and who were randomly assigned to autologous SCT instead of
chemotherapy [86], and favorable cytogenetics were overrepresented among
patients who had a donor (38% versus 23%). The MRC AML10 (5-year
OS, 58%) and CCG 2891 (5-year OS, 47%; 49% for the intensive arm) studies
enrolled patients during approximately the same time period, although the
patient populations may not have been comparable. It is possible that the im-
proved outcomes achieved by intensive chemotherapy may diminish the role
of SCT in rst CR of AML and that SCT provides a benet only when com-
pared with relatively less intensive treatment.
Randomized studies analyzed according to intent to treat have failed to
show that autologous SCT provides a survival advantage over intensive
ACUTE MYELOID LEUKEMIA 31

chemotherapy [8789], and a meta-analysis concluded that data were insuf-


cient to determine whether autologous SCT is superior to nonmyeloabla-
tive chemotherapy [83].
The controversy continues. In some groups, all patients who have
a matched sibling donor proceed to SCT, whereas in others SCT is reserved
for patients at high risk, although high risk is not dened consistently. In the
MRC, SCT has not been demonstrated to reduce the risk of relapse even in
children at high risk [90]. Unless it is demonstrated to reduce the risk of
relapse, transplantation can oer no benet. SCT may have a role in the
treatment of pediatric AML in rst CR if the graft-versus-leukemia eect
can be expanded by pre- and posttransplantation graft manipulation, which
may include the use of killer-cell immunoglobulin receptorincompatible
donors and donor lymphocyte infusions.
There is also a need to improve risk-group stratication and to identify
better the children who may benet from SCT. This goal may be achieved
by identifying better prognostic indicators and by using minimal residual
disease (MRD) monitoring, both of which are discussed in later sections.

Special subgroups
Acute myeloid leukemia in children who have Down syndrome
Children who have Down syndrome who develop AML generally do so
between 1 and 4 years of age. This subset of cases of AML is very responsive
to therapy but carries a signicant risk of early mortality. Children treated
during the past decade have had a reported EFS estimate of 83% [91], with
relapse rates as low as 3% [92]. The recommendation is to limit the cumu-
lative anthracycline dose to 240 to 250 mg/m2 [93] or to reduce overall dose
intensity rather than the absolute dose [94].

Acute promyelocytic leukemia


Children who have APL are treated with special APL protocols that
combine ATRA with intensive chemotherapy. Although ATRA can cause
considerable (but manageable) toxicity in some children, this approach
induces a stable and continuous remission without the early hemorrhagic
deaths that previously characterized this type of leukemia. APL is the
only subtype of AML in which maintenance chemotherapy is believed to
be of benet [95]. SCT in rst CR is not indicated for a disease that responds
so well to chemotherapy. Regimens increasingly based on alternatives to
traditional chemotherapy, including ATRA and arsenic trioxide, are being
tested [96].

Relapsed acute myeloid leukemia


After relapse, chemotherapy alone is unlikely to be curative, and the
survival rate is only 21% to 33% in recent reports [77,97101]. In these
reports, the length of rst remission was the best predictor of survival
32 RUBNITZ et al

[97100]. Various remission induction regimens, including udarabine plus


cytarabine and mitoxantrone plus cytarabine, seem to give similar results.
The addition of liposomal daunorubicin to udarabine plus cytarabine is
being tested currently to try to improve CR rates while minimizing cardio-
toxicity. It is important to reduce the toxicity of reinduction to a level that
allows SCT to proceed, because children who receive SCT can have a 5-year
survival probability of 60% (56% after early relapse; 65% after late relapse)
[102].
The targeted immunotherapy agents gemtuzumab ozogamicin and clofar-
abine have shown activity against relapsed AML. Gemtuzumab ozogamicin
has been shown to be safe and well tolerated in children and, as a single
agent, has induced responses in 30% of patients who have recurrent
CD33 AML [103]. Clofarabine has demonstrated activity against refrac-
tory and relapsed AML [104]. Both of these drugs may be more useful
when given in combination with other chemotherapeutic agents.
A second allograft seems to oer a benet to patients who experience
relapse after SCT during rst CR. Despite a high rate of transplantation-
related mortality and second relapse, more than one third of patients are
reported to be long-term survivors. Patients who undergo SCT during
remission may have an even better outcome [105]. Therefore every eort
should be made to induce remission before the second SCT.

Prognostic factors
Although clinical measures of tumor burden, such as leukocyte count and
hepatosplenomegaly, largely have been replaced by genetic factors in the
risk-classication schemes of contemporary treatment protocols, several
clinical features are still prognostically important. In both adult and pediat-
ric patients who have AML, age at diagnosis is associated inversely with the
probability of survival [106,107]. In an analysis of 424 patients less than
21 years of age, an age greater than 10 years at diagnosis was signicantly
associated with a worse outcome, even after controlling for cytogenetics,
leukocyte count, and FAB subtype [107]. The eect of age was important
only among patients treated in contemporary trials, reinforcing the view
that the eect of any prognostic factor ultimately depends on the therapy
given. Two recent studies suggest that another clinically apparent featured
ethnicitydmay be an important predictor of outcome [108,109]. Among
more than 1600 children who had AML treated on the CCG 2891 and 2961
trials, black children treated with chemotherapy had a signicantly worse out-
come than white children treated with chemotherapy, a disparity that the
authors suggest may reect pharmacogenetic dierences [109]. Body mass
index, another easily measured clinical feature, also may aect the outcome
of children who have AML [110]. In the CCG 2961 trial, underweight and over-
weight patients were less likely to survive than normoweight patients because
of a greater risk of treatment-related death [110].
ACUTE MYELOID LEUKEMIA 33

In addition to clinical features, certain pathologic features, such as M0


and M7 subtypes, seem to carry prognostic importance in AML [111,112].
The present authors and others have demonstrated that nonDown syn-
drome patients who have megakaryoblastic leukemia have signicantly
worse outcomes than patients who have other subtypes of AML
[111,113,114]. The EFS estimates for patients who have megakaryoblastic
leukemia treated in the CCG 2891 trial or in the St. Jude trial were only
22% and 14%, respectively [111,113]. In the St. Jude study [111] and in a re-
port from the European Group for Blood and Marrow Transplantation
[115], patients who underwent SCT during rst remission had a better out-
come than those who received chemotherapy, suggesting that SCT should be
recommended for these patients. A study by French investigators, however,
suggested that children who had megakaryoblastic leukemia with the
t(1;22), but without Down syndrome, had a better outcome than similar
children who did not have the t(1;22), indicating that this subgroup may
not need transplantation [114]. In addition, the BFM study group reported
an improved outcome for patients who had megakaryoblastic leukemia
treated in recent, more intensive trials [116]. SCT did not provide a benet
to patients treated in these trials. Thus, the role of SCT for patients who
have megakaryoblastic leukemia remains controversial.
Conventional cytogenetic studies have demonstrated that the karyotype
of leukemic blast cells is one of the best predictors of outcome [117,118].
An analysis of more than 1600 patients enrolled in the MRC AML 10 trial
revealed that t(8;21) and inv(16) were associated with a favorable outcome
(5-year OS estimates, 69% and 61%, respectively), whereas a complex kar-
yotype, -5, del(5q), -7, and abnormalities of 3q predicted a poor outcome
[117]. On the basis of these observations, the MRC investigators proposed
a cytogenetics-based risk classication system that is used by many cooper-
ative groups today [117]. Among the 340 patients in the MRC study who
were less than 15 years old, those with a favorable karyotype had a 3-year
survival estimate of 78%, compared with 55% for the intermediate-risk
group and 42% for the high-risk group. Other cooperative groups have con-
rmed the MRC ndings, with slightly dierent results for some subgroups
that probably reect dierences in therapy. For example, in the Pediatric
Oncology Group 8821 trial, patients who had t(8;21) had a 4-year OS esti-
mate of 52% and those who had inv(16) had an estimate of 75% [118]. Sim-
ilarly, among adults who had AML treated in Cancer and Leukemia Group
B trials, patients who had these karyotypes had a better outcome than
others and had a particularly good outcome when treated with multiple
courses of high-dose cytarabine [75,119,120].
Because both t(8;21) and inv(16) disrupt the CBF, they are often referred
to as CBF leukemias and are grouped together in risk-classication
systems. Several studies, however, have demonstrated that CBF leukemia
is a heterogeneous group of diseases in adults and therefore probably is
heterogeneous in children as well [121,122]. An analysis of 312 adults who
34 RUBNITZ et al

had CBF AML demonstrated that, although CR and relapse rates were
similar for patients who had t(8;21) and inv(16), OS was signicantly worse
for those who had t(8;21), primarily because of a lower salvage rate after
relapse [121]. In addition, race was prognostically important among patients
who had t(8;21), whereas sex and secondary cytogenetic changes were pre-
dictive of outcome among patients who had inv(16). A similar analysis of
370 adults who had CBF AML conrmed the heterogeneity of this type
of AML and conrmed the poor outcome after relapse among patients
who had t(8;21) [122]. Not surprisingly, in both studies, outcome depended
on treatment intensity.
Other prognostically important cytogenetic abnormalities include rear-
rangements of the MLL gene, located at chromosome band 11q23. The
abnormality is usually a reciprocal translocation between MLL and one
of more than 30 other genes in distinct chromosomal loci [123]. MLL rear-
rangements are seen in as many as 20% of cases of AML, although the
reported frequency varies among studies [124,125]. In general, children
and adults whose leukemic cells contain 11q23 abnormalities are considered
at intermediate risk, and their outcome does not dier signicantly from
that of patients without these translocations (3-year OS estimate, 50% in
the MRC AML 10 trial) [117]. Some studies, however, suggest that t(9;11)
confers a favorable outcome [124]. Among patients treated for AML at
St. Jude, those who had t(9;11) had a better outcome (5-year EFS estimate,
65%) than did patients in all other cytogenetic or molecular subgroups. This
nding may be attributable to the use of epipodophyllotoxins and cladri-
bine, both of which are eective against monoblastic leukemia.
In the MRC AML 10 study mentioned previously, monosomy 7 was
associated with a particularly poor outcome (5-year OS, 10%) but was
detected in only 4% of cases [117].
Because of the rarity of this abnormality, an international collaborative
study was undertaken to characterize further the impact of -7 and del(7q)
in children and adolescents who have AML [126]. In this study, which
included 172 patients who had -7 (with or without other abnormalities)
and 86 patients who had del(7q) (also with or without other changes),
patients who had -7 had lower CR rates (61% versus 89%) and worse
outcome (5-year survival, 30% versus 51%) than those who had del(7q).
Patients who had del(7q) and a favorable genetic abnormality had a good
outcome (5-year survival, 75%), suggesting that the del(7q) does not alter
the impact of the favorable feature. By contrast, patients who had -7 and
inv(3), -5/del(5q), or 21 had a dismal outcome (5-year survival, 5%) that
was not improved by SCT [126].
During the past 10 years, molecular studies have demonstrated heteroge-
neity within cytogenetically dened subgroups of AML and have identied
new, prognostically important subgroups. Mutations of c-kit, ras, and FLT3
have been detected in cases of childhood and adult AML; c-kit mutations
may be particularly important in cases of CBF leukemia [127131]. Several
ACUTE MYELOID LEUKEMIA 35

studies demonstrated that among adult patients who had t(8;21), those who
had mutations at c-kit codon 816 had a signicantly higher relapse rate and
worse outcome than those who had wild-type c-kit [127129]. In some stud-
ies, mutations of c-kit also seem to confer a worse outcome among patients
who have inv(16) [132]. Although c-kit mutations have been detected in 3%
to 11% of pediatric AML cases, their prognostic impact is uncertain
[130,133]. One study found c-kit mutations in 37% of cases of CBF leuke-
mia, but these cases did not dier from others in outcome [130]. In contrast,
the Japanese Childhood AML Cooperative Study Group found that c-kit
mutations, in 8 of 46 patients who had t(8;21), were associated with signif-
icantly worse OS, DFS, and relapse rates [131].
The impact of FLT3 mutations in childhood and adult AML has been
established by dozens of studies, only a few of which are summarized
here. In one of the rst studies reported, the estimated 5-year OS rate was
only 14% for adult patients who had internal tandem duplications (ITD)
of FLT3, and the presence of these mutations was the strongest prognostic
factor in multivariate analysis [134]. Similarly, in an analysis of 106 adults
who had AML treated in MRC trials, 13 of the 14 patients who had
FLT3 ITD died within 18 months of diagnosis [135]. A subsequent study
of 854 patients treated in the MRC AML trials demonstrated a FLT3
ITD, present in 27% of cases, was associated with an increased risk of
relapse and a lower probability of DFS, EFS, and OS [136]. Other reports
have conrmed the presence of FLT ITD in 20% to 30% of adult AML
cases, but some studies suggest that its negative prognostic impact may
depend on the absence of the wild-type allele or the ratio of the mutant to
the wild-type allele [137139].
Studies of childhood AML identify FLT3 ITD in only 10% to 15% of
cases, but still it is associated with a poor outcome [140143]. Among 91
pediatric patients who had AML treated in CCG trials, the 8-year EFS
estimate was only 7% for patients who had FLT3 ITD, whereas among
234 patients treated on Dutch AML protocols, the 5-year EFS for these
patients estimate was only 29% [140,141]. In both studies, multivariate anal-
ysis demonstrated that FLT3-ITD was the strongest predictor of relapse.
A more recent study of 630 patients treated in contemporary CCG trials
conrmed the poor outcome associated with FLT3 ITD and demonstrated
that survival decreased with an increasing allelic ratio of FLT ITD to
FLT3 wild-type [143]. The estimated progression-free survival was consider-
ably lower with a ratio greater than 0.4 than with a lower ratio (16% versus
72%). CCG investigators also compared the outcome of patients who had
FLT3 ITD in CD34/CD33 precursors with that of patients who had
the mutated gene in only the more mature CD34/CD33 progenitors
[65]. Patients who had the mutation in the less mature precursors had dra-
matically worse outcomes, conrming the heterogeneity within FLT3 ITD
positive cases of AML and suggesting that only a subset of these patients
have a poor prognosis. Data from studies by the Pediatric Oncology Group
36 RUBNITZ et al

suggest that gene expression proles also may be used to identify patients
who have a good prognosis despite FLT3 mutations [144].
Other molecular alterations reported to be prognostic factors in AML
include expression of ATP-binding cassette transporters [145147], CEBPA
mutations [148,149], DCC expression [150], secretion of vascular endothelial
growth factor [151], expression of apoptosis-related genes [152154], expres-
sion of BAALC [155], expression of ERG [156,157], NPM1 mutations
[158160], partial tandem duplications (PTD) of the MLL gene [161,162],
and global gene expression patterns [163167]. The clinical relevance of
these alterations has been reviewed comprehensively [168] and is discussed
only briey here. Mutations of the nucleophosmin member 1 (NPM1)
gene have been detected in about 50% of cases of adult AML with a normal
karyotype [159] but occur much less commonly in childhood AML [160]. In
both populations, NPM1 mutations are associated with FLT3 ITD; how-
ever, in patients who have wild-type FLT3, NPM1 mutations are associated
with a favorable outcome [168]. MLL PTD occur in about 5% to 10% of
adult AML cases and, like NPM1 mutations, commonly are associated
with FLT3 ITD [168]. MLL PTD seem to be an adverse prognostic factor,
but it is not clear whether the negative impact is related to the association
with FLT3 ITD. High expression of the BAALC gene and the ERG gene
are additional factors that have independent negative prognostic signi-
cance among adult patients who have a normal karyotype, whereas muta-
tions of the CEBPA gene are associated with a favorable outcome [168].
A risk-classication scheme for adults who have a normal AML karyotype
that incorporates the status of FLT3, NPM1, BAALC, MLL, and CEBPA
has been proposed and may be used in future clinical trials [168]. MLL PTD,
BAALC, and CEBPA have not been studied extensively in childhood AML.
Nevertheless, it is likely that forthcoming pediatric clinical trials will use
gene-expression proling to identify important prognostic subgroups that
may benet from more intensive or novel therapies [144,169].

Minimal residual disease


The heterogeneity within cytogenetically and even molecularly dened
subgroups indicates that other methods are needed to optimize risk classi-
cation. Many studies of ALL and AML have demonstrated the prognostic
importance of early response to therapy (ie, reduction or elimination of
leukemic cells in the bone marrow), which may be a more powerful predictor
of outcome than genetic features [170]. Response to therapy can be mea-
sured by morphologic [171,172] or cytogenetic [173] examination of bone
marrow, but these methods cannot detect levels of residual leukemia below
1% (1 leukemic cell in 100 mononuclear bone marrow cells). In contrast,
MRD assays provide objective and sensitive measurement of low levels of
leukemic cells [170,174] in childhood [175178] and adult [179183] AML.
Methods of assessing MRD include DNA-based polymerase chain reaction
ACUTE MYELOID LEUKEMIA 37

(PCR) analysis of clonal antigen-receptor gene rearrangements (applicable


to less than 10% of AML cases), RNA-based PCR analysis of leukemia-
specic gene fusions (applicable to less than 50% of AML cases), and
ow cytometric detection of aberrant immunophenotype (applicable to
more than 90% of AML cases). Among 252 children evaluated for MRD
in the CCG-2961 trial, occult leukemia (dened as more than 0.5% bone
marrow blast cells with an aberrant phenotype) was detected in 16% of
the children considered to be in remission [176]. Multivariate analysis
demonstrated that patients who had detectable MRD were 4.8 times more
likely to experience relapse (P ! .0001) and 3.1 times more likely to die
(P ! .0001) than patients who were MRD negative. A study at St. Jude
Childrens Research Hospital yielded similar ndings: the 2-year survival
estimate for patients who had detectable MRD at the end of induction ther-
apy was 33%, compared with 72% for MRD-negative patients (P .022)
[177]. Recent studies in adults have conrmed that the level of residual leu-
kemia cells detected immunophenotypically after induction or consolidation
therapy is associated strongly with the risk of relapse [181183].
Quantitative reverse transcription PCR assays of leukemia-specic fusion
transcripts is an alternative method of MRD detection that can be used in
AML cases that harbor these gene fusions [113,184190]. Several studies
have indicated that quantication of AML1-ETO and CBFb-MYH11 fusion
transcripts at the time of diagnosis and during therapy is a useful predictor
of outcome. Similarly, there is emerging evidence that quantitative PCR
assessment of WT1 transcripts also may prove useful for monitoring
MRD and predicting outcome in patients who have AML [191193].

Pharmacogenomics of therapy for acute myeloid leukemia


Patient factors, such as pharmacodynamics and pharmacogenomics,
signicantly aect the outcome of treatment in many types of cancer, includ-
ing AML [194,195]. The eect of such factors is demonstrated clearly by the
chemosensitivity and excellent outcome of AML in children who have
Down syndrome, who have cure rates of 80% to 100% [196]. Increased
levels of cystathionine-b-synthetase (CBS), a high frequency of CBS genetic
polymorphisms, low levels of cytidine deaminase, and altered expression of
other GATA1 target genes in these patients leukemic blast cells contribute
to the high cure rates [197200]. Polymorphisms or altered expression of
other proteins involved in cytarabine metabolism, such as deoxycytidine
kinase, DNA polymerase, and es nucleoside transporter, also may play
a role in leukemic blast cell sensitivity to this agent [201203]. In addition,
polymorphisms may inuence toxicity. For example, homozygous deletions
of the glutathione S-transferase theta (GSTT1) gene have been reported to
be associated with a higher frequency of early toxic death and a lower like-
lihood of survival [204,205]. Recently, polymorphisms of the XPD gene
(XPD751), which is involved in DNA repair, were shown to be associated
38 RUBNITZ et al

with a lower likelihood of survival and a higher risk of therapy-related


leukemia in elderly patients who had AML [206]. XPD751 does not seem
to inuence outcome in children who have AML, however [207].

Complications and supportive care


At the time of diagnosis, patients who have AML may have life-threaten-
ing complications, including bleeding, leukostasis, tumor lysis syndrome,
and infection. The rst three are managed through the use of platelet trans-
fusions, leukapheresis or exchange transfusion, aggressive hydration, oral
phosphate binders and recombinant urate oxidase, and the prompt initia-
tion of chemotherapy. Infectious complications at the time of diagnosis
and during therapy remain a major cause of morbidity and mortality
[74,208211]. Viridans streptococci, which commonly colonize the oral, gas-
trointestinal, and vaginal mucosa, are particularly troublesome in patients
undergoing therapy for AML [208,210,212,213]. Because of the high risk
of sepsis, most clinicians agree that all patients who have AML and who
have febrile neutropenia should be hospitalized and treated with broad-
spectrum intravenous antibiotics, such as a third- or fourth-generation ceph-
alosporin, as well as vancomycin. Patients who have evidence of sepsis or
infection with Pseudomonas aeruginosa should receive an aminoglycoside,
and patients who have severe abdominal pain, evidence of typhlitis, or
known infection with Bacillus cereus should be treated with a carbapenem
(imipenem or meropenem) rather than a cephalosporin. In addition, patients
who have AML are at high risk of fungal infection [213] and therefore
should receive empiric antifungal therapy with traditional amphotericin B,
lipid formulations of amphotericin B, an azole (voriconazole or posacona-
zole), or an echinocandin (caspofungin or micafungin). Cytokines such as
granulocyte-macrophage colony stimulating factor and granulocyte col-
ony-stimulating factor also should be considered in cases of proven sepsis
or fungal infection, but there is little evidence that their prophylactic use sig-
nicantly reduces morbidity [214216].
Because of the high incidence of bacterial and fungal infections, the pres-
ent authors recently tested several prophylactic antimicrobial regimens in
78 children receiving chemotherapy for AML at St. Jude Childrens
Research Hospital. Oral cephalosporins were ineective, but intravenous
cefepime completely prevented viridans streptococcal sepsis and reduced
the odds of bacterial sepsis by 91%. Similarly, intravenous vancomycin
given with oral ciprooxacin reduced the odds of viridans streptococcal
sepsis by 98% and the odds of any bacterial sepsis by 94%. All patients
received antifungal prophylaxis with oral voriconazole, which contributed
to a low rate of disseminated fungal infection (1.0/000 patient-days). Most
important, there were no deaths from bacterial or fungal infection among
patients who received prophylactic antibiotics and voriconazole. Because
of the relatively small number of patients studied, these prophylactic
ACUTE MYELOID LEUKEMIA 39

antibiotic regimens must be evaluated in a multi-institutional setting before


recommendations can be made.

Future directions
As a result of highly collaborative clinical trials, the outcome for children
who have AML has improved continuously over the past several decades,
but approximately half of all children diagnosed as having AML still die
of the disease or of complications of treatment. Further advances will
require a greater understanding of the biology of AML, improved risk
stratication and risk-directed therapies, improved treatment of high-risk
disease, and the development of molecularly targeted agents or better cellu-
lar therapies. Targeted therapies may cause less toxicity, but they may be
clinically applicable only to well-dened molecular subgroups, as with the
use of ATRA and arsenic trioxide for APL [95,217]. Agents under investiga-
tion include gemtuzumab ozogamicin [218], proteasome inhibitors [219,220],
histone deacetylase inhibitors [221,222], and tyrosine kinases inhibitors
[223225]. Clofarabine, a purine nucleoside analogue that was designed to
integrate the qualities of udarabine and cladribine, also has activity against
AML [226228]. Recently, cellular therapy with haploidentical natural killer
cells has been shown to exert antitumor activity with minimal toxicity in
patients who have relapsed AML [229]. Timely evaluation of these and other
therapies will require novel clinical trial designs with new statistical models
that allow the testing of new treatment approaches in increasingly small sub-
groups of patients. In addition, future clinical trials will require interna-
tional collaboration among the pediatric cooperative oncology groups.

Acknowledgments
The authors thank Sharon Naron for expert editorial review.

References
[1] Kaspers G, Creutzig U. Pediatric AML: long term results of clinical trials from 13 study
groups worldwide. Leukemia 2005;19:2025146.
[2] Kaspers GJ, Creutzig U. Pediatric acute myeloid leukemia: international progress and
future directions. Leukemia 2005;19(12):20259.
[3] Creutzig U, Zimmermann M, Ritter J, et al. Treatment strategies and long-term results in pae-
diatric patients treated in four consecutive AML-BFM trials. Leukemia 2005;19(12):203042.
[4] Smith FO, Alonzo TA, Gerbing RB, et al. Long-term results of children with acute myeloid
leukemia: a report of three consecutive phase III trials by the Childrens Cancer Group:
CCG 251, CCG 213 and CCG 2891. Leukemia 2005;19(12):205462.
[5] Gibson BE, Wheatley K, Hann IM, et al. Treatment strategy and long-term results in
paediatric patients treated in consecutive UK AML trials. Leukemia 2005;19(12):21308.
[6] Pession A, Rondelli R, Basso G, et al. Treatment and long-term results in children with
acute myeloid leukaemia treated according to the AIEOP AML protocols. Leukemia
2005;19(12):204353.
40 RUBNITZ et al

[7] Kardos G, Zwaan CM, Kaspers GJ, et al. Treatment strategy and results in children treated
on three Dutch Childhood Oncology Group acute myeloid leukemia trials. Leukemia 2005;
19(12):206371.
[8] Entz-Werle N, Suciu S, van der Wer ten Bosch J, et al. Results of 58872 and 58921 trials in
acute myeloblastic leukemia and relative value of chemotherapy vs allogeneic bone marrow
transplantation in rst complete remission: the EORTC Children Leukemia Group report.
Leukemia 2005;19(12):207281.
[9] Perel Y, Auvrignon A, Leblanc T, et al. Treatment of childhood acute myeloblastic leuke-
mia: dose intensication improves outcome and maintenance therapy is of no benetd
multicenter studies of the French LAME (Leucemie Aigue Myeloblastique Enfant)
Cooperative Group. Leukemia 2005;19(12):20829.
[10] Lie SO, Abrahamsson J, Clausen N, et al. Long-term results in children with AML: NOPHO-
AML Study Groupreport of three consecutive trials. Leukemia 2005;19(12):2090100.
[11] Ravindranath Y, Chang M, Steuber CP, et al. Pediatric Oncology Group (POG) studies of
acute myeloid leukemia (AML): a review of four consecutive childhood AML trials
conducted between 1981 and 2000. Leukemia 2005;19(12):210116.
[12] Dluzniewska A, Balwierz W, Armata J, et al. Twenty years of Polish experience with three
consecutive protocols for treatment of childhood acute myelogenous leukemia. Leukemia
2005;19(12):211724.
[13] Ribeiro RC, Razzouk BI, Pounds S, et al. Successive clinical trials for childhood acute
myeloid leukemia at St Jude Childrens Research Hospital, from 1980 to 2000. Leukemia
2005;19(12):21259.
[14] Armendariz H, Barbieri MA, Freigeiro D, et al. Treatment strategy and long-term results in
pediatric patients treated in two consecutive AML-GATLA trials. Leukemia 2005;19(12):
213942.
[15] Quintana J, Advis P, Becker A, et al. Acute myelogenous leukemia in Chile PINDA proto-
cols 87 and 92 results. Leukemia 2005;19(12):21436.
[16] Smith MA, Ries LAG, Gurney JG, et al. Leukemia. In: Ries LAG, Smith MA, Gurney JG,
et al, editors. Cancer incidence and survival among children and adolescents: United States
SEER Progam 19751995. NIH Pub. No. 994649. Bethesda (MD): National Cancer Insti-
tute, SEER Program; 1999. p. 1734.
[17] Glavel J, Goubin A, Auclerc MF, et al. Incidence of childhood leukaemia and non-
Hodgkins lymphoma in France: National Registry of Childhood Leukaemia and Lym-
phoma, 19901999. Eur J Cancer Prev 2004;13:97103.
[18] Hjalgrim LL, Rostgaard K, Schmiegelow K, et al. Age- and sex-specic incidence of child-
hood leukemia by immunophenotype in the Nordic countries. J Natl Cancer Inst 2003;
95(20):153944.
[19] Xie Y, Davies SM, Xiang Y, et al. Trends in leukemia incidence and survival in the United
States (19731998). Cancer 2003;97(9):222935.
[20] Gurney JG, Severson RK, Davis S, et al. Incidence of cancer in children in the United States.
Sex-, race-, and 1-year age-specic rates by histologic type. Cancer 1995;75(8):218695.
[21] Bhatia S, Neglia JP. Epidemiology of childhood acute myelogenous leukemia [see
comments]. J Pediatr Hematol Oncol 1995;17(2):94100.
[22] Ross JA, Davies SM, Potter JD, et al. Epidemiology of childhood leukemia, with a focus on
infants. Epidemiol Rev 1994;16(2):24372.
[23] Sandler DP, Ross JA. Epidemiology of acute leukemia in children and adults. Semin Oncol
1997;24(1):316.
[24] Linassier C, Barin C, Calais G, et al. Early secondary acute myelogenous leukemia in breast
cancer patients after treatment with mitoxantrone, cyclophosphamide, uorouracil and
radiation therapy. Ann Oncol 2000;11(10):128994.
[25] Micallef IN, Lillington DM, Apostolidis J, et al. Therapy-related myelodysplasia and
secondary acute myelogenous leukemia after high-dose therapy with autologous hemato-
poietic progenitor-cell support for lymphoid malignancies. J Clin Oncol 2000;18(5):94755.
ACUTE MYELOID LEUKEMIA 41

[26] Smith MA, McCarey RP, Karp JE. The secondary leukemias: challenges and research
directions. J Natl Cancer Inst 1996;88(7):40718.
[27] Sandoval C, Pui CH, Bowman LC, et al. Secondary acute myeloid leukemia in children pre-
viously treated with alkylating agents, intercalating topoisomerase II inhibitors, and irradi-
ation. J Clin Oncol 1993;11(6):103945.
[28] Relling MV, Yanishevski Y, Nemec J, et al. Etoposide and antimetabolite pharmacology in
patients who develop secondary acute myeloid leukemia. Leukemia 1998;12(3):34652.
[29] Pui CH, Ribeiro RC, Hancock ML, et al. Acute myeloid leukemia in children treated with
epipodophyllotoxins for acute lymphoblastic leukemia. N Engl J Med 1991;325(24):16827.
[30] Le Deley MC, Leblanc T, Shamsaldin A, et al. Risk of secondary leukemia after a solid
tumor in childhood according to the dose of epipodophyllotoxins and anthracyclines:
a case-control study by the Societe Francaise dOncologie Pediatrique. J Clin Oncol
2003;21(6):107481.
[31] Korte JE, Hertz-Picciotto I, Schulz MR, et al. The contribution of benzene to smoking-
induced leukemia. Environ Health Perspect 2000;108(4):3339.
[32] McBride ML. Childhood cancer and environmental contaminants. Can J Public Health
1998;89(Suppl 1):S5368.
[33] Yin SN, Hayes RB, Linet MS, et al. An expanded cohort study of cancer among benzene-
exposed workers in China. Benzene Study Group. Environ Health Perspect 1996;104(Suppl
6):133941.
[34] Yin SN, Hayes RB, Linet MS, et al. A cohort study of cancer among benzene-exposed
workers in China: overall results. Am J Ind Med 1996;29(3):22735.
[35] Linet MS, Bailey PE. Benzene, leukemia, and the Supreme Court. J Public Health Policy
1981;2(2):11635.
[36] Mills PK, Zahm SH. Organophosphate pesticide residues in urine of farmworkers and their
children in Fresno County, California. Am J Ind Med 2001;40(5):5717.
[37] Rosenberg PS, Greene MH, Alter BP. Cancer incidence in persons with Fanconi anemia.
Blood 2003;101(3):8226.
[38] German J. Blooms syndrome. XX. The rst 100 cancers. Cancer Genet Cytogenet 1997;
93(1):1006.
[39] Bader JL, Miller RW. Neurobromatosis and childhood leukemia. J Pediatr 1978;92(6):
9259.
[40] Bader-Meunier B, Tchernia G, Mielot F, et al. Occurrence of myeloproliferative disorder
in patients with Noonan syndrome. J Pediatr 1997;130(6):8859.
[41] Socie G, Henry-Amar M, Bacigalupo A, et al. Malignant tumors occurring after treatment
of aplastic anemia. European Bone Marrow Transplantation-Severe Aplastic Anaemia
Working Party. N Engl J Med 1993;329(16):11527.
[42] Imashuku S, Hibi S, Nakajima F, et al. A review of 125 cases to determine the risk of
myelodysplasia and leukemia in pediatric neutropenic patients after treatment with
recombinant human granulocyte colony-stimulating factor. Blood 1994;84(7):23801.
[43] Xue Y, Zhang R, Guo Y, et al. Acquired amegakaryocytic thrombocytopenic purpura with
a Philadelphia chromosome. Cancer Genet Cytogenet 1993;69(1):516.
[44] Geissler D, Thaler J, Konwalinka G, et al. Progressive preleukemia presenting amegakar-
yocytic thrombocytopenic purpura: association of the 5q- syndrome with a decreased
megakaryocytic colony formation and a defective production of Meg-CSF. Leuk Res
1987;11(8):7317.
[45] Gilliland DG. Molecular genetics of human leukemia. Leukemia 1998;12(Suppl 1):S712.
[46] Dash A, Gilliland DG. Molecular genetics of acute myeloid leukaemia. Best Pract Res Clin
Haematol 2001;14(1):4964.
[47] Gilliland DG, Tallman MS. Focus on acute leukemias. Cancer Cell 2002;1(5):41720.
[48] Castilla LH, Garrett L, Adya N, et al. The fusion gene Cbfb-MYH11 blocks myeloid
dierentiation and predisposes mice to acute myelomonocytic leukaemia. Nat Genet
1999;23:1446.
42 RUBNITZ et al

[49] Higuchi M, OBrien D, Kumaravelu P, et al. Expression of a conditional AML1-ETO


oncogene bypasses embryonic lethality and establishes a murine model of human t(8;21)
acute myeloid leukemia. Cancer Cell 2002;1(1):6374.
[50] Ford AM, Ridge SA, Cabrera ME, et al. In utero rearrangements in the trithorax-related
oncogene in infant leukaemias. Nature 1993;363:35860.
[51] Gill Super HJ, Rothberg PG, Kobayashi H, et al. Clonal, nonconstitutional rearrange-
ments of the MLL gene in infant twins with acute lymphoblastic leukemia: in utero chro-
mosome rearrangement of 11q23. Blood 1994;83(3):6414.
[52] Megonigal MD, Rappaport EF, Jones DH, et al. t(11;22)(q23;q11.2) In acute myeloid
leukemia of infant twins fuses MLL with hCDCrel, a cell division cycle gene in the genomic
region of deletion in DiGeorge and velocardiofacial syndromes. Proc Natl Acad Sci U S A
1998;95(11):64138.
[53] Wiemels JL, Ford AM, Van Wering ER, et al. Protracted and variable latency of acute
lymphoblastic leukemia after TEL-AML1 gene fusion in utero. Blood 1999;94(3):
105762.
[54] Song WJ, Sullivan MG, Legare RD, et al. Haploinsuciency of CBFA2 causes familial
thrombocytopenia with propensity to develop acute myelogenous leukaemia. Nat Genet
1999;23(2):16675.
[55] Bonnet D, Dick JE. Human acute myeloid leukemia is organized as a hierarchy that
originates from a primitive hematopoietic cell. Nat Med 1997;3(7):7307.
[56] Caligiuri MA, Strout MP, Gilliland DG. Molecular biology of acute myleloid leukemia.
Semin Oncol 1997;24:3244.
[57] Sabbath KD, Ball ED, Larcom P, et al. Heterogeneity of clonogenic cells in acute myelo-
blastic leukemia. J Clin Invest 1985;75:74653.
[58] Lapidot T, Sirard C, Vormoor J, et al. A cell initiating human acute myeloid leukaemia after
transplantation into SCID mice. Nature 1994;367(6464):6458.
[59] Mehrotra B, George TI, Kavanau K, et al. Cytogenetically aberrant cells in the stem cell
compartment (CD34lin-) in acute myeloid leukemia. Blood 1995;86(3):113947.
[60] Sirard C, Lapidot T, Vormoor J, et al. Normal and leukemic SCID-repopulating cells
(SRC) coexist in the bone marrow and peripheral blood from CML patients in chronic
phase, whereas leukemic SRC are detected in blast crisis. Blood 1996;87(4):153948.
[61] Hope KJ, Jin L, Dick JE. Human acute myeloid leukemia stem cells. Arch Med Res 2003;
34(6):50714.
[62] Hope KJ, Jin L, Dick JE. Acute myeloid leukemia originates from a hierarchy of leukemic
stem cell classes that dier in self-renewal capacity. Nat Immunol 2004;5(7):73843.
[63] Warner JK, Wang JC, Hope KJ, et al. Concepts of human leukemic development.
Oncogene 2004;23(43):716477.
[64] Terpstra W, Prins A, Ploemacher RE, et al. Long-term leukemia-initiating capacity of
a CD34-subpopulation of acute myeloid leukemia. Blood 1996;87(6):218794.
[65] Pollard JA, Alonzo TA, Gerbing RB, et al. FLT3 internal tandem duplication in
CD34/. Blood 2006;108(8):27649.
[66] Pui CH, Dahl GV, Kalwinsky DK, et al. Central nervous system leukemia in children with
acute nonlymphoblastic leukemia. Blood 1985;66(5):10627.
[67] Woods WG, Kobrinsky N, Buckley JD, et al. Time-sequential induction therapy improves
postremission outcome in acute myeloid leukemia: a report from the Childrens Cancer
Group. Blood 1996;87(12):497989.
[68] Carella AM, Berman E, Maraone MP, et al. Idarubicin in the treatment of acute leukemias.
An overview of preclinical and clinical studies. Haematologica 1990;75(2):15969.
[69] Berman E, McBride M. Comparative cellular pharmacology of daunorubicin and idarubi-
cin in human multidrug-resistant leukemia cells. Blood 1992;79(12):326773.
[70] Reid JM, Pendergrass TW, Krailo MD, et al. Plasma pharmacokinetics and cerebrospinal
uid concentrations of idarubicin and idarubicinol in pediatric leukemia patients: a Chil-
drens Cancer Study Group report. Cancer Res 1990;50(20):65258.
ACUTE MYELOID LEUKEMIA 43

[71] Creutzig U, Ritter J, Zimmermann M, et al. Idarubicin improves blast cell clearance during
induction therapy in children with AML: results of study AML-BFM 93. AML-BFM
Study Group. Leukemia 2001;15(3):34854.
[72] OBrien TA, Russell SJ, Vowels MR, et al. Results of consecutive trials for children newly
diagnosed with acute myeloid leukemia from the Australian and New Zealand Childrens
Cancer Study Group. Blood 2002;100(8):270816.
[73] Becton D, Dahl GV, Ravindranath Y, et al. Randomized use of cyclosporin A (CsA) to
modulate P-glycoprotein in children with AML in remission: Pediatric Oncology Group
Study 9421. Blood 2006;107(4):131524.
[74] Riley LC, Hann IM, Wheatley K, et al. Treatment-related deaths during induction and rst
remission of acute myeloid leukaemia in children treated on the Tenth Medical Research
Council acute myeloid leukaemia trial (MRC AML10). The MCR Childhood Leukaemia
Working Party. Br J Haematol 1999;106(2):43644.
[75] Byrd JC, Dodge RK, Carroll A, et al. Patients with t(8;21)(q22;q22) and acute myeloid
leukemia have superior failure-free and overall survival when repetitive cycles of high-
dose cytarabine are administered. J Clin Oncol 1999;17(12):376775.
[76] Abbott BL, Rubnitz JE, Tong X, et al. Clinical signicance of central nervous system
involvement at diagnosis of pediatric acute myeloid leukemia: a single institutions experi-
ence. Leukemia 2003;17(11):20906.
[77] Johnston DL, Alonzo TA, Gerbing RB, et al. Risk factors and therapy for isolated central
nervous system relapse of pediatric acute myeloid leukemia. J Clin Oncol 2005;23(36):
91728.
[78] Creutzig U, Ritter J, Zimmermann M, et al. Does cranial irradiation reduce the risk for
bone marrow relapse in acute myelogenous leukemia? Unexpected results of the Childhood
Acute Myelogenous Leukemia Study BFM-87. J Clin Oncol 1993;11(2):27986.
[79] Wells RJ, Woods WG, Buckley JD, et al. Treatment of newly diagnosed children and
adolescents with acute myeloid leukemia: a Childrens Cancer Group study. J Clin Oncol
1994;12(11):236777.
[80] Creutzig U, Reinhardt D. Current controversies: which patients with acute myeloid leukae-
mia should receive a bone marrow transplantation? A European view. Br J Haematol 2002;
118(2):36577.
[81] Chen AR, Alonzo TA, Woods WG, et al. Current controversies: which patients with acute
myeloid leukaemia should receive a bone marrow transplantation? An American view.
Br J Haematol 2002;118(2):37884.
[82] Wheatley K. Current controversies: which patients with acute myeloid leukaemia should
receive a bone marrow transplantation? A statisticians view. Br J Haematol 2002;118(2):
3516.
[83] Bleakley M, Lau L, Shaw PJ, et al. Bone marrow transplantation for paediatric AML in
rst remission: a systematic review and meta-analysis. Bone Marrow Transplant 2002;
29(10):84352.
[84] Burnett AK, Wheatley K, Goldstone AH, et al. The value of allogeneic bone marrow trans-
plant in patients with acute myeloid leukaemia at diering risk of relapse: results of the UK
MRC AML 10 trial. Br J Haematol 2002;118(2):385400.
[85] Creutzig U, Reinhardt D, Zimmermann M, et al. Intensive chemotherapy versus bone
marrow transplantation in pediatric acute myeloid leukemia: a matter of controversies.
Blood 2001;97(11):36712.
[86] Woods WG, Neudorf S, Gold S, et al. A comparison of allogeneic bone marrow transplan-
tation, autologous bone marrow transplantation, and aggressive chemotherapy in children
with acute myeloid leukemia in remission. Blood 2001;97(1):5662.
[87] Stevens RF, Hann IM, Wheatley K, et al. Marked improvements in outcome with chemo-
therapy alone in paediatric acute myeloid leukemia: results of the United Kingdom Medical
Research Councils 10th AML trial. MRC Childhood Leukaemia Working Party. Br J
Haematol 1998;101(1):13040.
44 RUBNITZ et al

[88] Ravindranath Y, Yeager AM, Chang MN, et al. Autologous bone marrow transplantation
versus intensive consolidation chemotherapy for acute myeloid leukemia in childhood.
Pediatric Oncology Group. N Engl J Med 1996;334(22):142834.
[89] Amadori S, Testi AM, Arico M, et al. Prospective comparative study of bone marrow trans-
plantation and postremission chemotherapy for childhood acute myelogenous leukemia.
The Associazione Italiana Ematologia ed Oncologia Pediatrica Cooperative Group.
J Clin Oncol 1993;11(6):104654.
[90] Gibson B, Hann I, Webb I, et al. Should stem cell transplantation (SCT) be recommended
for a child with AML in 1st CR. Blood 2007;106:171.
[91] Zeller B, Gustafsson G, Forestier E, et al. Acute leukaemia in children with Down
syndrome: a population-based Nordic study. Br J Haematol 2005;128(6):797804.
[92] Ao A, Hills R, Stiller C, et al. Treatment for myeloid leukaemia of Down syndrome:
population-based experience in the UK and results from the Medical Research Council
AML10 and AML 12 trials. Br J Haematol 2005;132:57683.
[93] Creutzig U, Ritter J, Vormoor J, et al. Myelodysplasia and acute myelogenous leukemia in
Downs syndrome. A report of 40 children of the AML-BFM Study Group. Leukemia
1996;10(11):167786.
[94] Gamis AS, Woods WG, Alonzo TA, et al. Increased age at diagnosis has a signicantly
negative eect on outcome in children with Down syndrome and acute myeloid leukemia:
a report from the Childrens Cancer Group Study 2891. J Clin Oncol 2003;21(18):
341522.
[95] Testi AM, Biondi A, Lo CF, et al. GIMEMA-AIEOP AIDA protocol for the treatment of
newly diagnosed acute promyelocytic leukemia (APL) in children. Blood 2005;106:44753.
[96] Ravindranath Y, Gregory J, Feusner J. Treatment of acute promyelocytic leukemia in
children: arsenic or ATRA. Leukemia 2004;18(10):15767.
[97] Webb DK, Wheatley K, Harrison G, et al. Outcome for children with relapsed acute
myeloid leukaemia following initial therapy in the Medical Research Council (MRC)
AML 10 trial. MRC Childhood Leukaemia Working Party. Leukemia 1999;13(1):2531.
[98] Stahnke K, Boos J, Bender-Gotze C, et al. Duration of rst remission predicts remission
rates and long-term survival in children with relapsed acute myelogenous leukemia. Leuke-
mia 1998;12(10):15348.
[99] Aladjidi N, Auvrignon A, Leblanc T, et al. Outcome in children with relapsed acute myeloid
leukemia after initial treatment with the French Leucemie Aique Myeloide Enfant (LAME)
89/91 protocol of the French Society of Pediatric Hematology and Immunology. J Clin
Oncol 2003;21(23):437785.
[100] Wells RJ, Adams MT, Alonzo TA, et al. Mitoxantrone and cytarabine induction, high-dose
cytarabine, and etoposide intensication for pediatric patients with relapsed or refractory
acute myeloid leukemia: Childrens Cancer Group Study 2951. J Clin Oncol 2003;21(15):
29407.
[101] Rubnitz JE, Razzouk BI, Lensing S, et al. Prognostic factors and outcome of recurrence in
childhood acute myeloid leukemia. Cancer 2007;109(1):15763.
[102] Abrahamsson J, Clausen N, Gustafsson G, et al. Improved outcome after relapse in
children with acute myeloid leukaemia. Br J Haematol 2007;136(2):22236.
[103] Zwaan CM, Reinhardt D, Corbacioglu S, et al. Gemtuzumab ozogamicin: rst clinical
experiences in children with relapsed/refractory acute myeloid leukemia treated on compas-
sionate-use basis. Blood 2003;101(10):386871.
[104] Jeha S, Gandhi V, Chan KW, et al. Clofarabine, a novel nucleoside analog, is active in
pediatric patients with advanced leukemia. Blood 2004;103(3):7849.
[105] Meshinchi S, Leisenring WM, Carpenter PA, et al. Survival after second hematopoietic
stem cell transplantation for recurrent pediatric acute myeloid leukemia. Biol Blood
Marrow Transplant 2003;9(11):70613.
[106] Appelbaum FR, Gundacker H, Head DR, et al. Age and acute myeloid leukemia. Blood
2006;107(9):34815.
ACUTE MYELOID LEUKEMIA 45

[107] Razzouk BI, Estey E, Pounds S, et al. Impact of age on outcome of pediatric acute myeloid
leukemia: a report from 2 institutions. Cancer 2006;106(11):2495502.
[108] Rubnitz JE, Lensing S, Razzouk BI, et al. Eect of race on outcome of white and black
children with acute myeloid leukemia: the St. Jude experience. Pediatr Blood Cancer
2007;48(1):105.
[109] Aplenc R, Alonzo TA, Gerbing RB, et al. Ethnicity and survival in childhood acute myeloid
leukemia: a report from the Childrens Oncology Group. Blood 2006;108(1):7480.
[110] Lange BJ, Gerbing RB, Feusner J, et al. Mortality in overweight and underweight children
with acute myeloid leukemia. J Am Med Assoc 2005;293(2):20311.
[111] Athale UH, Razzouk BI, Raimondi SC, et al. Biology and outcome of childhood acute
megakaryoblastic leukemia: a single institutions experience. Blood 2001;97(12):372732.
[112] Barbaric D, Alonzo TA, Gerbing R, et al. Minimally dierentiated acute myeloid leukemia
(FAB AML-10) is associated with an adverse outcome in children: a report from the Chil-
drens Oncology Group, studies CCG-2891 and CCG-2961. Blood 2007;109(6):231421.
[113] Barnard D, Alonzo TA, Gerbing R, et al. Comparison of childhood myelodysplastic
syndrome, AML FAB M6 or M7, CCG 2891: report from the Childrens Oncology Group.
Pediatr Blood Cancer 2007;49:1722.
[114] Dastugue N, Lafage-Pochitalo M, Pages MP, et al. Cytogenetic prole of childhood and
adult megakaryoblastic leukemia (M7): a study of the Groupe Francais de Cytogenetique
Hematologique (GFCH). Blood 2002;100(2):61826.
[115] Garderet L, Labopin M, Gorin NC, et al. Hematopoietic stem cell transplantation for de
novo acute megakaryocytic leukemia in rst complete remission: a retrospective study of
the European Group for Blood and Marrow Transplantation (EBMT). Blood 2005;
105(1):4059.
[116] Reinhardt D, Diekamp S, Langebrake C, et al. Acute megakaryoblastic leukemia in
children and adolescents, excluding Downs syndrome: improved outcome with intensied
induction treatment. Leukemia 2005;19(8):14956.
[117] Grimwade D, Walker H, Oliver F, et al. The importance of diagnostic cytogenetics on
outcome in AML: analysis of 1,612 patients entered into the MRC AML 10 trial. The
Medical Research Council Adult and Childrens Leukaemia Working Parties. Blood
1998;92(7):232233.
[118] Raimondi SC, Chang MN, Ravindranath Y, et al. Chromosomal abnormalities in 478
children with acute myeloid leukemia: clinical characteristics and treatment outcome in
a cooperative Pediatric Oncology Group study-POG 8821. Blood 1999;94(11):370716.
[119] Bloomeld CD, Lawrence D, Byrd JC, et al. Frequency of prolonged remission duration
after high-dose cytarabine intensication in acute myeloid leukemia varies by cytogenetic
subtype. Cancer Res 1998;58(18):41739.
[120] Byrd JC, Mrozek K, Dodge RK, et al. Pretreatment cytogenetic abnormalities are predic-
tive of induction success, cumulative incidence of relapse, and overall survival in adult
patients with de novo acute myeloid leukemia: results from Cancer and Leukemia Group
B (CALGB 8461). Blood 2002;100(13):432536.
[121] Marcucci G, Mrozek K, Ruppert AS, et al. Prognostic factors and outcome of core binding
factor acute myeloid leukemia patients with t(8;21) dier from those of patients with
inv(16): a Cancer and Leukemia Group B Study. J Clin Oncol 2006;24:570517.
[122] Appelbaum F, Kopecky KJ, Tallman M, et al. The clinical spectrum of adult acute myeloid
leukaemia associated with core binding factor translocations. Br J Haematol 2006;135:
16573.
[123] Dimartino JF, Cleary ML. Mll rearrangements in haematological malignancies: lessons
from clinical and biological studies. Br J Haematol 1999;106(3):61426.
[124] Rubnitz JE, Raimondi SC, Tong X, et al. Favorable impact of the t(9;11) in childhood acute
myeloid leukemia. J Clin Oncol 2002;20(9):23029.
[125] Schoch C, Schnittger S, Klaus M, et al. AML with 11q23/MLL abnormalities as dened by
the WHO classication: incidence, partner chromosomes, FAB subtype, age distribution,
46 RUBNITZ et al

and prognostic impact in an unselected series of 1897 cytogenetically analyzed AML cases.
Blood 2003;102(7):2395402.
[126] Hasle H, Alonzo TA, Auvrignon A, et al. Monosomy 7 and deletion 7q in children and
adolescents with acute myeloid leukemia: an international retrospective study. Blood
2007;109(11):46417.
[127] Cairoli R, Beghini A, Grillo G, et al. Prognostic impact of c-KIT mutations in core binding
factor leukemias: an Italian retrospective study. Blood 2007;107:34638.
[128] Schnittger S, Kohl T, Haferlach T, et al. KIT-D816 mutations in AML1-ETO-positive
AML are associated with impaired event-free and overall survival. Blood 2006;107:17919.
[129] Paschka P, Marcucci G, Ruppert AS, et al. Adverse prognostic signicance of KIT muta-
tions in adult acute myeloid leukemia with inv(16) and t(8;21): a Cancer and Leukemia
Group B Study. J Clin Oncol 2006;24(24):390411.
[130] Goemans BF, Zwaan CM, Miller M, et al. Mutations in KIT and RAS are frequent events
in pediatric core-binding factor acute myeloid leukemia. Leukemia 2005;19(9):153642.
[131] Shimada A, Taki T, Tabuchi K, et al. KIT mutations, and not FLT3 internal tandem
duplication, are strongly associated with a poor prognosis in pediatric acute myeloid leuke-
mia with t(8;21); a study of the Japanese Childhood AML Cooperative Study Group. Blood
2006;107:18069.
[132] Boissel N, Leroy H, Brethon B, et al. Incidence and prognostic impact of c-Kit, FLT3, and
Ras gene mutations in core binding factor acute myeloid leukemia (CBF-AML). Leukemia
2006;20(6):96570.
[133] Meshinchi S, Stirewalt DL, Alonzo TA, et al. Activating mutations of RTK/ras signal
transduction pathway in pediatric acute myeloid leukemia. Blood 2003;102(4):14749.
[134] Kiyoi H, Naoe T, Nakano Y, et al. Prognostic implication of FLT3 and N-RAS gene
mutations in acute myeloid leukemia. Blood 1999;93(9):307480.
[135] Abu-Duhier FM, Goodeve AC, Wilson GA, et al. FLT3 internal tandem duplication
mutations in adult acute myeloid leukaemia dene a high-risk group. Br J Haematol
2000;111(1):1905.
[136] Kottaridis PD, Gale RE, Frew ME, et al. The presence of a FLT3 internal tandem
duplication in patients with acute myeloid leukemia (AML) adds important prognostic
information to cytogenetic risk group and response to the rst cycle of chemotherapy:
analysis of 854 patients from the United Kingdom Medical Research Council AML 10
and 12 trials. Blood 2001;98(6):17529.
[137] Whitman SP, Archer KJ, Feng L, et al. Absence of the wild-type allele predicts poor prog-
nosis in adult de novo acute myeloid leukemia with normal cytogenetics and the internal
tandem duplication of FLT3: a cancer and leukemia group B study. Cancer Res 2001;
61(19):72339.
[138] Schnittger S, Schoch C, Dugas M, et al. Analysis of FLT3 length mutations in 1003 patients
with acute myeloid leukemia: correlation to cytogenetics, FAB subtype, and prognosis in
the AMLCG study and usefulness as a marker for the detection of minimal residual disease.
Blood 2002;100(1):5966.
[139] Thiede C, Steudel C, Mohr B, et al. Analysis of FLT3-activating mutations in 979 patients
with acute myelogenous leukemia: association with FAB subtypes and identication of
subgroups with poor prognosis. Blood 2002;99(12):432635.
[140] Zwaan CM, Meshinchi S, Radich JP, et al. FLT3 internal tandem duplication in 234
children with acute myeloid leukemia: prognostic signicance and relation to cellular
drug resistance. Blood 2003;102(7):238794.
[141] Meshinchi S, Woods WG, Stirewalt DL, et al. Prevalence and prognostic signicance of
Flt3 internal tandem duplication in pediatric acute myeloid leukemia. Blood 2001;97(1):
8994.
[142] Iwai T, Yokota S, Nakao M, et al. Internal tandem duplication of the FLT3 gene and
clinical evaluation in childhood acute myeloid leukemia. The Childrens Cancer and
Leukemia Study Group, Japan. Leukemia 1999;13(1):3843.
ACUTE MYELOID LEUKEMIA 47

[143] Meshinchi S, Alonzo TA, Stirewalt DL, et al. Clinical implications of FLT3 mutations in
pediatric AML. Blood 2006;108(12):365461.
[144] Lacayo NJ, Meshinchi S, Kinnunen P, et al. Gene expression proles at diagnosis in de
novo childhood AML patients identify FLT3 mutations with good clinical outcomes.
Blood 2004;104(9):264654.
[145] Leith CP, Kopecky KJ, Chen IM, et al. Frequency and clinical signicance of the expression
of the multidrug resistance proteins MDR1/P-glycoprotein, MRP1, and LRP in acute
myeloid leukemia: a Southwest Oncology Group Study. Blood 1999;94(3):108699.
[146] Legrand O, Simonin G, Beauchamp-Nicoud A, et al. Simultaneous activity of MRP1 and
Pgp is correlated with in vitro resistance to daunorubicin and with in vivo resistance in adult
acute myeloid leukemia. Blood 1999;94(3):104656.
[147] den Boer ML, Pieters R, Kazemier KM, et al. Relationship between major vault protein/
lung resistance protein, multidrug resistance-associated protein, P-glycoprotein expression,
and drug resistance in childhood leukemia. Blood 1998;91(6):20928.
[148] Preudhomme C, Sagot C, Boissel N, et al. Favorable prognostic signicance of CEBPA
mutations in patients with de novo acute myeloid leukemia: a study from the Acute Leuke-
mia French Association (ALFA). Blood 2002;100(8):271723.
[149] Frohling S, Schlenk RF, Stolze I, et al. CEBPA mutations in younger adults with acute
myeloid leukemia and normal cytogenetics: prognostic relevance and analysis of cooperat-
ing mutations. J Clin Oncol 2004;22(4):62433.
[150] Inokuchi K, Yamaguchi H, Hanawa H, et al. Loss of DCC gene expression is of prognostic
importance in acute myelogenous leukemia. Clin Cancer Res 2002;8(6):18828.
[151] De Bont ES, Fidler V, Meeuwsen T, et al. Vascular endothelial growth factor secretion is an
independent prognostic factor for relapse-free survival in pediatric acute myeloid leukemia
patients. Clin Cancer Res 2002;8(9):285661.
[152] Kohler T, Schill C, Deininger MW, et al. High Bad and Bax mRNA expression correlate
with negative outcome in acute myeloid leukemia (AML). Leukemia 2002;16(1):229.
[153] Karakas T, Miething CC, Maurer U, et al. The coexpression of the apoptosis-related genes
Bcl-2 and Wt1 in predicting survival in adult acute myeloid leukemia. Leukemia 2002;16(5):
84654.
[154] Del Poeta G, Venditti A, Del Principe MI, et al. Amount of spontaneous apoptosis detected
by Bax/Bcl-2 ratio predicts outcome in acute myeloid leukemia (AML). Blood 2003;101(6):
212531.
[155] Baldus CD, Thiede C, Soucek S, et al. BAALC expression and FLT3 internal tandem
duplication mutations in acute myeloid leukemia patients with normal cytogenetics:
prognostic implications. J Clin Oncol 2006;24(5):7907.
[156] Marcucci G, Maharry K, Whitman SP, et al. High expression levels of the ETS-related
gene, ERG, predict adverse outcome and improve molecular risk-based classication of
cytogenetically normal acute myeloid leukemia: a Cancer and Leukemia Group B Study.
J Clin Oncol 2007;25(22):333743.
[157] Marcucci G, Baldus CD, Ruppert AS, et al. Overexpression of the ETS-related gene, ERG,
predicts a worse outcome in acute myeloid leukemia with normal karyotype: a Cancer and
Leukemia Group B study. J Clin Oncol 2005;23(36):923442.
[158] Boissel N, Renneville A, Biggio V, et al. Prevalence, clinical prole, and prognosis of NPM
mutations in AML with normal karyotype. Blood 2005;106(10):361820.
[159] Thiede C, Koch S, Creutzig E, et al. Prevalence and prognostic impact of NPM1 muta-
tions in 1485 adult patients with acute myeloid leukemia (AML). Blood 2006;107(10):
401120.
[160] Brown P, McIntyre E, Rau R, et al. The incidence and clinical signicance of nucleophos-
min mutations in childhood AML. Blood 2007;110(3):97985.
[161] Schnittger S, Kinkelin U, Schoch C, et al. Screening for MLL tandem duplication in 387
unselected patients with AML identify a prognostically unfavorable subset of AML.
Leukemia 2000;14(5):796804.
48 RUBNITZ et al

[162] Dohner K, Tobis K, Ulrich R, et al. Prognostic signicance of partial tandem duplications
of the MLL gene in adult patients 16 to 60 years old with acute myeloid leukemia and
normal cytogenetics: a study of the Acute Myeloid Leukemia Study Group Ulm. J Clin
Oncol 2002;20(15):325461.
[163] Yagi T, Morimoto A, Eguchi M, et al. Identication of a gene expression signature associ-
ated with pediatric AML prognosis. Blood 2003;102(5):184956.
[164] Valk PJ, Verhaak RG, Beijen MA, et al. Prognostically useful gene-expression proles in
acute myeloid leukemia. N Engl J Med 2004;350(16):161728.
[165] Bullinger L, Dohner K, Bair E, et al. Use of gene-expression proling to identify prognostic
subclasses in adult acute myeloid leukemia. N Engl J Med 2004;350(16):160516.
[166] Radmacher MD, Marcucci G, Ruppert AS, et al. Independent conrmation of a prognostic
gene-expression signature in adult acute myeloid leukemia with a normal karyotype: a Can-
cer and Leukemia Group B study. Blood 2006;108(5):167783.
[167] Wilson CS, Davidson GS, Martin SB, et al. Gene expression proling of adult acute
myeloid leukemia identies novel biologic clusters for risk classication and outcome
prediction. Blood 2006;108(2):68596.
[168] Mrozek K, Marcucci G, Paschka P, et al. Clinical relevance of mutations and gene-
expression changes in adult acute myeloid leukemia with normal cytogenetics: are we ready
for a prognostically prioritized molecular classication? Blood 2007;109(2):43148.
[169] Ross ME, Mahfouz R, Onciu M, et al. Gene expression proling of pediatric acute myelog-
enous leukemia. Blood 2004;104(12):367987.
[170] Campana D. Determination of minimal residual disease in leukaemia patients. Br J
Haematol 2003;121(6):82338.
[171] Creutzig U, Zimmermann M, Ritter J, et al. Denition of a standard-risk group in children
with AML. Br J Haematol 1999;104(3):6309.
[172] Kern W, Haferlach T, Schoch C, et al. Early blast clearance by remission induction therapy
is a major independent prognostic factor for both achievement of complete remission and
long-term outcome in acute myeloid leukemia: data from the German AML Cooperative
Group (AMLCG) 1992 Trial. Blood 2003;101(1):6470.
[173] Marcucci G, Mrozek K, Ruppert AS, et al. Abnormal cytogenetics at date of morphologic
complete remission predicts short overall and disease-free survival, and higher relapse rate
in adult acute myeloid leukemia: results from Cancer and Leukemia Group B study 8461.
J Clin Oncol 2004;22(12):24108.
[174] Szczepanski T, Orfao A, van DV, et al. Minimal residual disease in leukaemia patients.
Lancet Oncol 2001;2(7):40917.
[175] Sievers EL, Lange BJ, Buckley JD, et al. Prediction of relapse of pediatric acute myeloid
leukemia by use of multidimensional ow cytometry. J Natl Cancer Inst 1996;88(20):
14838.
[176] Sievers EL, Lange BJ, Alonzo TA, et al. Immunophenotypic evidence of leukemia after
induction therapy predicts relapse: results from a prospective Childrens Cancer Group
study of 252 patients with acute myeloid leukemia. Blood 2003;101(9):3398406.
[177] Coustan-Smith E, Ribeiro RC, Rubnitz JE, et al. Clinical signicance of residual disease
during treatment in childhood acute myeloid leukaemia. Br J Haematol 2003;123(2):24352.
[178] Langebrake C, Creutzig U, Dworzak M, et al. Residual disease monitoring in childhood
acute myeloid leukemia by multiparameter ow cytometry: the MRD-AML-BFM Study
Group. J Clin Oncol 2006;24(22):368692.
[179] San Miguel JF, Martinez A, Macedo A, et al. Immunophenotyping investigation of
minimal residual disease is a useful approach for predicting relapse in acute myeloid
leukemia patients. Blood 1997;90(6):246570.
[180] San Miguel JF, Vidriales MB, Lopez-Berges C, et al. Early immunophenotypical evaluation
of minimal residual disease in acute myeloid leukemia identies dierent patient risk groups
and may contribute to postinduction treatment stratication. Blood 2001;98(6):174651.
ACUTE MYELOID LEUKEMIA 49

[181] Buccisano F, Maurillo L, Gattei V, et al. The kinetics of reduction of minimal residual
disease impacts on duration of response and survival of patients with acute myeloid
leukemia. Leukemia 2006;20(10):17839.
[182] Feller N, van der Pol MA, van Stijn A, et al. MRD parameters using immunophenotypic
detection methods are highly reliable in predicting survival in acute myeloid leukaemia.
Leukemia 2004;18(8):138090.
[183] Kern W, Voskova D, Schoch C, et al. Determination of relapse risk based on assessment of
minimal residual disease during complete remission by multiparameter ow cytometry in
unselected patients with acute myeloid leukemia. Blood 2004;104(10):307885.
[184] Tobal K, Newton J, Macheta M, et al. Molecular quantitation of minimal residual disease
in acute myeloid leukemia with t(8;21) can identify patients in durable remission and predict
clinical relapse. Blood 2000;95(3):8159.
[185] Schnittger S, Weisser M, Schoch C, et al. New score predicting for prognosis in PML-
RARA, AML1-ETO, or CBFBMYH11 acute myeloid leukemia based on quantica-
tion of fusion transcripts. Blood 2003;102(8):274655.
[186] Buonamici S, Ottaviani E, Testoni N, et al. Real-time quantitation of minimal residual dis-
ease in inv(16)-positive acute myeloid leukemia may indicate risk for clinical relapse and
may identify patients in a curable state. Blood 2002;99(2):4439.
[187] Guerrasio A, Pilatrino C, De Micheli D, et al. Assessment of minimal residual disease
(MRD) in CBFbeta/MYH11-positive acute myeloid leukemias by qualitative and quanti-
tative RT-PCR amplication of fusion transcripts. Leukemia 2002;16(6):117681.
[188] Viehmann S, Teigler-Schlegel A, Bruch J, et al. Monitoring of minimal residual disease
(MRD) by real-time quantitative reverse transcription PCR (RQ-RT-PCR) in childhood
acute myeloid leukemia with AML1/ETO rearrangement. Leukemia 2003;17(6):11306.
[189] Krauter J, Gorlich K, Ottmann O, et al. Prognostic value of minimal residual disease quan-
tication by real-time reverse transcriptase polymerase chain reaction in patients with core
binding factor leukemias. J Clin Oncol 2003;21(23):441322.
[190] Perea G, Lasa A, Aventin A, et al. Prognostic value of minimal residual disease (MRD) in
acute myeloid leukemia (AML) with favorable cytogenetics [t(8;21) and inv(16)]. Leukemia
2006;20:8794.
[191] Trka J, Kalinova M, Hrusak O, et al. Real-time quantitative PCR detection of WT1 gene
expression in children with AML: prognostic signicance, correlation with disease status
and residual disease detection by ow cytometry. Leukemia 2002;16(7):13819.
[192] Cilloni D, Gottardi E, De Micheli D, et al. Quantitative assessment of WT1 expression by
real time quantitative PCR may be a useful tool for monitoring minimal residual disease in
acute leukemia patients. Leukemia 2002;16(10):211521.
[193] Lapillonne H, Renneville A, Auvrignon A, et al. High WT1 expression after induction
therapy predicts high risk of relapse and death in pediatric acute myeloid leukemia.
J Clin Oncol 2006;24(10):150715.
[194] Evans WE, Relling MV. Moving towards individualized medicine with pharmacogenom-
ics. Nature 2004;429(6990):4648.
[195] Monzo M, Brunet S, Urbano-Ispizua A, et al. Genomic polymorphisms provide prognos-
tic information in intermediate-risk acute myeloblastic leukemia. Blood 2006;107(12):
48719.
[196] Gamis AS. Acute myeloid leukemia and Down syndrome evolution of modern therapyd
state of the art review. Pediatr Blood Cancer 2005;44(1):1320.
[197] Ge Y, Jensen T, James SJ, et al. High frequency of the 844ins68 cystathionine-beta-synthase
gene variant in Down syndrome children with acute myeloid leukemia. Leukemia 2002;
16(11):233941.
[198] Ge Y, Stout ML, Tatman DA, et al. GATA1, cytidine deaminase, and the high cure rate of
Down syndrome children with acute megakaryocytic leukemia. J Natl Cancer Inst 2005;
97(3):22631.
50 RUBNITZ et al

[199] Ge Y, Dombkowski AA, Laura KM, et al. Dierential gene expression, GATA1 target
genes, and the chemotherapy sensitivity of Down syndrome megakaryocytic leukemia.
Blood 2006;107(4):157081.
[200] Taub JW, Ge Y. Down syndrome, drug metabolism and chromosome 21. Pediatr Blood
Cancer 2005;44(1):339.
[201] Galmarini CM, Thomas X, Calvo F, et al. In vivo mechanisms of resistance to cytarabine in
acute myeloid leukaemia. Br J Haematol 2002;117(4):8608.
[202] Gati WP, Paterson AR, Belch AR, et al. Es nucleoside transporter content of acute
leukemia cells: role in cell sensitivity to cytarabine (AraC). Leuk Lymphoma 1998;
32(12):4554.
[203] Galmarini CM, Thomas X, Graham K, et al. Deoxycytidine kinase and cN-II nucleotidase
expression in blast cells predict survival in acute myeloid leukaemia patients treated with
cytarabine. Br J Haematol 2003;122(1):5360.
[204] Davies SM, Robison LL, Buckley JD, et al. Glutathione S-transferase polymorphisms and
outcome of chemotherapy in childhood acute myeloid leukemia. J Clin Oncol 2001;19(5):
127987.
[205] Naoe T, Tagawa Y, Kiyoi H, et al. Prognostic signicance of the null genotype of glutathi-
one S- transferase-T1 in patients with acute myeloid leukemia: increased early death after
chemotherapy. Leukemia 2002;16(2):2038.
[206] Allan JM, Smith AG, Wheatley K, et al. Genetic variation in XPD predicts treatment out-
come and risk of acute myeloid leukemia following chemotherapy. Blood 2004;104(13):
38727.
[207] Mehta PA, Alonzo TA, Gerbing RB, et al. XPD Lys751Gln polymorphism in the etiology
and outcome of childhood acute myeloid leukemia: a Childrens Oncology Group report.
Blood 2006;107(1):3945.
[208] Okamoto Y, Ribeiro RC, Srivastava DK, et al. Viridans streptococcal sepsis: clinical fea-
tures and complications in childhood acute myeloid leukemia. J Pediatr Hematol Oncol
2003;25(9):696703.
[209] Creutzig U, Zimmermann M, Reinhardt D, et al. Early deaths and treatment-related
mortality in children undergoing therapy for acute myeloid leukemia: analysis of the
multicenter clinical trials AML-BFM 93 and AML-BFM 98. J Clin Oncol 2004;22(21):
438493.
[210] Lehrnbecher T, Varwig D, Kaiser J, et al. Infectious complications in pediatric acute
myeloid leukemia: analysis of the prospective multi-institutional clinical trial AML-BFM
93. Leukemia 2004;18(1):727.
[211] Rubnitz JE, Lensing S, Zhou Y, et al. Death during induction therapy and rst remission of
acute leukemia in childhood: the St. Jude experience. Cancer 2004;101(7):167784.
[212] Gamis AS, Howells WB, DeSwarte-Wallace J, et al. Alpha hemolytic Streptococcal infec-
tion during intensive treatment for acute myeloid leukemia: a report from the Childrens
Cancer Group Study CCG-2891. J Clin Oncol 2000;18(9):184555.
[213] Sung L, Lange BJ, Gerbing RB, et al. Microbiologically documented infections and
infection-related mortality in children with acute myeloid leukemia. Blood 2007;110:
35329.
[214] Godwin JE, Kopecky KJ, Head DR, et al. A double-blind placebo-controlled trial of
granulocyte colony-stimulating factor in elderly patients with previously untreated acute
myeloid leukemia: a Southwest Oncology Group study (9031). Blood 1998;91(10):360715.
[215] Heil G, Hoelzer D, Sanz MA, et al. A randomized, double-blind, placebo-controlled, phase
III study of lgrastim in remission induction and consolidation therapy for adults with de
novo acute myeloid leukemia. The International Acute Myeloid Leukemia Study Group.
Blood 1997;90(12):47108.
[216] Amadori S, Suciu S, Jehn U, et al. Use of glycosylated recombinant human G-CSF
(lenograstim) during and/or after induction chemotherapy in patients 61 years of age and
older with acute myeloid leukemia: nal results of AML-13, a randomized phase 3 study
ACUTE MYELOID LEUKEMIA 51

of the European Organisation for Research and Treatment of Cancer and Gruppo Italiano
Malattie Ematologiche dellAdulto (EORTC/GIMEMA) Leukemia Groups. Blood 2005;
106:2734.
[217] George B, Mathews V, Poonkuzhali B, et al. Treatment of children with newly diagnosed
acute promyelocytic leukemia with arsenic trioxide: a single center experience. Leukemia
2004;18(10):158790.
[218] Burnett A, Kell WJ, Goldstone A, et al. The addition of gemtuzumab ozogamicin to
induction chemotherapy for AML improves disease free survival without extra toxicity:
preliminary analysis of 1115 patients in the MRC AML15 trial. Blood 2006;108:13.
[219] Guzman ML, Swiderski CF, Howard DS, et al. Preferential induction of apoptosis for
primary human leukemic stem cells. Proc Natl Acad Sci U S A 2002;99(25):162205.
[220] Adams J. The proteasome: a suitable antineoplastic target. Nat Rev Cancer 2004;4(5):
34960.
[221] Insinga A, Monestiroli S, Ronzoni S, et al. Inhibitors of histone deacetylases induce tumor-
selective apoptosis through activation of the death receptor pathway. Nat Med 2005;11(1):
716.
[222] Nebbioso A, Clarke N, Voltz E, et al. Tumor-selective action of HDAC inhibitors involves
TRAIL induction in acute myeloid leukemia cells. Nat Med 2005;11(1):7784.
[223] Levis M, Allebach J, Tse KF, et al. A FLT3-targeted tyrosine kinase inhibitor is cytotoxic to
leukemia cells in vitro and in vivo. Blood 2002;99(11):388591.
[224] Brown P, Meshinchi S, Levis M, et al. Pediatric AML primary samples with FLT3/ITD
mutations are preferentially killed by FLT3 inhibition. Blood 2004;104(6):18419.
[225] Smith BD, Levis M, Beran M, et al. Single-agent CEP-701, a novel FLT3 inhibitor, shows
biologic and clinical activity in patients with relapsed or refractory acute myeloid leukemia.
Blood 2004;103(10):366976.
[226] Parker WB, Shaddix SC, Chang CH, et al. Eects of 2-chloro-9-(2-deoxy-2-uoro-beta-
D-arabinofuranosyl)adenine on K562 cellular metabolism and the inhibition of human
ribonucleotide reductase and DNA polymerases by its 50 -triphosphate. Cancer Res 1991;
51(9):238694.
[227] Xie KC, Plunkett W. Deoxynucleotide pool depletion and sustained inhibition of ribonu-
cleotide reductase and DNA synthesis after treatment of human lymphoblastoid cells with
2-chloro-9-(2-deoxy-2-uoro-beta-D-arabinofuranosyl) adenine. Cancer Res 1996;56(13):
30307.
[228] Gandhi V, Kantarjian H, Faderl S, et al. Pharmacokinetics and pharmacodynamics of
plasma clofarabine and cellular clofarabine triphosphate in patients with acute leukemias.
Clin Cancer Res 2003;9(17):633542.
[229] Miller JS, Soignier Y, Panoskaltsis-Mortari A, et al. Successful adoptive transfer and in
vivo expansion of human haploidentical NK cells in patients with cancer. Blood 2005;
105(8):30517.
Seminar

Acute lymphoblastic leukaemia


Ching-Hon Pui, Leslie L Robison, A Thomas Look

Lancet 2008; 371: 103043 Acute lymphoblastic leukaemia, a malignant disorder of lymphoid progenitor cells, aects both children and adults,
Department of Oncology with peak prevalence between the ages of 2 and 5 years. Steady progress in development of eective treatments has
(Prof C-H Pui MD) and led to a cure rate of more than 80% in children, creating opportunities for innovative approaches that would preserve
Department of Epidemiology
past gains in leukaemia-free survival while reducing the toxic side-eects of current intensive regimens. Advances in
and Cancer Control
(Prof L L Robison PhD), St Jude our understanding of the pathobiology of acute lymphoblastic leukaemia, fuelled by emerging molecular technologies,
Childrens Research Hospital suggest that drugs specically targeting the genetic defects of leukaemic cells could revolutionise management of this
and University of Tennessee disease. Meanwhile, studies are underway to ascertain the precise events that take place in the genesis of acute
Health Science Center,
lymphoblastic leukaemia, to enhance the clinical application of known risk factors and antileukaemic agents, and to
Memphis, TN, USA; and
Department of Pediatric identify treatment regimens that might boost the generally low cure rates in adults and subgroups of children with
Oncology, Dana-Farber Cancer high-risk leukaemia.
Institute and Harvard Medical
School, Boston, MA, USA
(Prof A T Look MD)
Introduction extensive list of conicting or isolated reports of factors
Addition of acute lymphoblastic leukaemia to the growing purported to confer an increased risk for this disease,
Correspondence to:
Prof Ching-Hon Pui, St Jude list of cancers that have succumbed to eective treatment including parental occupation, maternal reproductive
Childrens Research Hospital, is tempting. The decision would be easy to justify in view history, parental tobacco or alcohol use, maternal diet,
332 N Lauderdale, Memphis, of data showing cure rates higher than 80% for children prenatal vitamin use, exposure to pesticides or solvents,
TN 38105, USA
treated in modern centres, most of whom will lead and exposure to the highest levels (>03 or 04 T) of
ching-hon.pui@stjude.org
healthy productive lives as long-term cancer survivors.13 residential, power-frequency magnetic elds.9,10
Thus, the future management of acute lymphoblastic Observations of a peak age of development of childhood
leukaemia might be viewed as simply tweaking existing acute lymphoblastic leukaemia of 25 years, an association
protocols and devising alternative regimens for the fth of industrialisation and modern or auent societies with
of patients who respond poorly to available agents. This increased prevalence of the disease, and the occasional
scenario, however attractive, must be rejected on several clustering of childhood leukaemia cases (especially in new
grounds. It does not accommodate the poor prognosis towns) have fuelled two parallel infection-based hypotheses
for adults with acute lymphoblastic leukaemia or the by British investigators: Kinlens population-mixing
complexity, expense, and toxic eects of contemporary hypothesis and Greaves delayed-infection hypothesis
multiagent treatments.1,4 Most importantly, it overlooks (gure 1).11,12 Kinlens hypothesis predicts that clusters of
our rapidly increasing ability to analyse the genetic and childhood cases of acute lymphoblastic leukaemia result
epigenetic abnormalities of leukaemic cells and to from exposure of susceptible (non-immune) individuals
translate them into enhanced diagnostic methods and to common but fairly non-pathological infections after
molecularly targeted therapy.5,6 Although the molecular population-mixing with carriers. The delayed-infection
medicine approach is still in its investigative stage, with hypothesis of Greaves is based on a minimal two-hits
many new obstacles to overcome, it holds enormous model and suggests that some susceptible individuals
promise. Put simply, we are about to enter an era in with a prenatally acquired preleukaemic clone had low or
which leukaemia patients will probably receive no exposure to common infections early in life because
individualised treatment based on the genetic features of they lived in an auent hygienic environment. Such
their malignant cells and their own unique genetic infectious insulation predisposes the immune system of
make-up (so-called pharmacogenomics).7 Our intent in these individuals to aberrant or pathological responses
this Seminar is to review advances in both the after subsequent or delayed exposure to common
fundamental understanding and clinical management of infections at an age commensurate with increased
acute lymphoblastic leukaemia in children and adults. lymphoid-cell proliferation.
Retrospective identication of leukaemia-specic
Epidemiology and cause fusion genes, hyperdiploidy, or clonotypic rearrange-
The precise pathogenetic events leading to development ments of immunoglobulin or T-cell-receptor loci in
of acute lymphoblastic leukaemia are unknown. Only a
few cases (<5%) are associated with inherited,
predisposing genetic syndromes, such as Downs Search strategy and selection criteria
syndrome, Blooms syndrome, ataxia-telangiectasia, and We searched Medline and PubMed for articles published in
Nijmegen breakage syndrome, or with ionising radiation English dating from 2002, with the keywords acute
or exposure to specic chemotherapeutic drugs. Although lymphoblastic leukemia, acute lymphocytic leukemia, and
accumulating published work on high birthweight as a acute lymphoid leukemia. In some instances, review articles
risk factor for childhood acute lymphoblastic leukaemia were selected over original articles because of space constraints.
is becoming increasingly convincing,8 there exists an

1030 www.thelancet.com Vol 371 March 22, 2008


Seminar

archived neonatal blood spots (Guthrie cards) and


studies of leukaemia in monozygotic twins indicate Low exposure
Exposure to
to pathogens
clearly a prenatal origin for some childhood Rapid cell resulting in
pathogens during
period of
leukaemias.1215 Screening of neonatal cord-blood samples proliferation decreased
increased lymphoid
proliferative
has revealed a putative leukaemic clone with the stress
proliferation
TEL-AML1 fusion gene (also known as ETV6-RUNX1) in
1% of newborn babies, a frequency 100 times higher Fetal development Birth Infancy early childhood

than the prevalence of acute lymphoblastic leukaemia


dened by this fusion gene later in childhood.16 The
variable incubation period and clinical outcome of such Live in an environment

hypothesis
Greaves
Random with decreased
cases, and the 10% concordance rate of leukaemia in mutations exposure to common
Delayed exposure to common pathogens
identical twins with this genotype, support the notion pathogens
that additional postnatal events are needed for full
leukaemic transformation.17 A recent study further
established the presence of a preleukaemic clone with
hypothesis
Low or no
Kinlens

the TEL-AML1 fusion.15 exposure to Exposure to the new pathogens as a result of


non-endemic increased population mixing
Investigations have also focused on the genetic pathogens
variability in xenobiotic metabolism, DNA repair
pathways, and cell-cycle checkpoint functions that might Figure 1: Infection-based models of leukaemia development
interact with environmental, dietary, maternal, and other
external factors to aect development of acute dierentiation-linked cell-surface glycoproteins that
lymphoblastic leukaemia. Although the number of largely recapitulate those of immature lymphoid
investigations and sample sizes are limited, data exist to progenitor cells within the early developmental stages of
support a possible causal role for polymorphisms in normal T and B lymphocytes.6,28,30 The dominant theme
genes encoding cytochrome P450, NAD(P)H quinone of contemporary research in pathobiology of acute
oxidoreductase, glutathione S-transferases, methyl- lymphoblastic leukaemia is to understand the outcomes
enetetahydrofolate reductase, thymidylate synthase, of frequently arising genetic lesions, in terms of their
serine hyroxymethyltransferase, and cell-cycle inhi- eects on cell proliferation, dierentiation, and survival,
bitors.1823 To date, however, no direct gene-environment and then to devise selectively targeted treatments against
interactions have been established convincingly. the altered gene products to which the leukaemic clones
In view of the scarcity of causal insights from large-scale have become addicted.31
epidemiological studies, some investigators have adopted
a strategy that focuses on distinct subtypes of childhood Chromosomal translocations
acute lymphoblastic leukaemia. An important example is Chromosomal translocations that activate specic genes
the study of infant acute lymphoblastic leukaemia with are a dening characteristic of human leukaemias and
MLL rearrangement,24 a genetic abnormality that has also of acute lymphoblastic leukaemia in particular.6,28
been associated with secondary leukaemia after exposure Gene-expression patterns studied in large series of newly
to a topoisomerase II inhibitor.25 Thus, dietary, medical, diagnosed leukaemias have substantiated the idea that
and environmental exposures to substances that inhibit specic chromosomal translocations identify unique
topoisomerases, and the reduced ability of fetuses or subtypes of the disease.3235 Usually, translocations activate
their mothers to detoxify such agents, could lead to transcription-factor genes, which in many cases can
development of infant leukaemia.26,27 control cell dierentiation (rather than cell division per
se), are developmentally regulated, and frequently encode
Pathobiology proteins at the apex of important transcriptional
Acute lymphoblastic leukaemia is thought to originate cascades.28 These so-called master oncogenic transcription
from various important genetic lesions in blood-progenitor factors, which can exert either positive or negative control
cells that are committed to dierentiate in the T-cell or over downstream responder genes, are expressed
B-cell pathway, including mutations that impart the aberrantly in leukaemic cells as one gene product or as a
capacity for unlimited self-renewal and those that lead to unique fusion protein combining elements from two
precise stage-specic developmental arrest.6,28 In some dierent transcription factors.6,28
cases, the rst mutation along the multistep pathway to About 25% of cases of B-cell precursor acute
overt acute lymphoblastic leukaemia might arise in a lymphoblastic leukaemia, the most frequent form of
haemopoietic stem cell possessing multilineage develop- acute leukaemia in children, harbour the TEL-AML1
mental capacity.29 The cells implicated in acute fusion genegenerated by the t(12;21)(p13;q22)
lymphoblastic leukaemia have clonal rearrangements in chromosomal translocation.6 Although the molecular
their immunoglobulin or T-cell receptor genes and pathogenesis of TEL-AML1-positive leukaemia remains
express antigen-receptor molecules and other unclear, ndings in mice establish the Tel gene as an

www.thelancet.com Vol 371 March 22, 2008 1031


Seminar

Sending cell Receiving cell

Golgi
pre-Notch ICN
Fucosylation and
endoplasmic reticulum exit
S1
Glycosylation Endoplasmic
Cleavage
B reticulum
Fringe Furin

NEURL
OFUT 1
C Heterodimerisation A
pre-Notch ICN
MIB
Notch
ICN
Jagged Nucleus
Fringe dnMAML NRARP MINT

DLL
Deltex? E HES 1
Ligand ICN ICN MAML
PRE T
DELTEX
endocytosis Notch CSL n300 NRARP
and CoA CD25
degradation S3 Numb MYC
S2
cleavage F
cleavage
CoR
-secretase CSL
complex
ADAM
metalloprotease

GSI ICN SEL 10


Ubiquitination and
G proteosome degradation

Figure 2: NOTCH signalling in normal thymocytes


The NOTCH signalling pathway is complex and involves the coordinated activities of many dierent molecules. Briey, NOTCH is synthesised in the endoplasmic
reticulum (A) as one protein consisting of an extracellular domain (pre-Notch) and an intracellular domain (ICN), which are transported in tandem to the Golgi (B),
where several post-translational modications take place, including a proteolytic cleavage (S1) that separates the two domains from each other. The resultant
heterodimer is then transported to the cell membrane, where NOTCH interacts with ligands (C) and is cleaved twice (D) by the ADAM protease (S2) and a -secretase
complex (S3), enabling the liberated ICN domain to translocate to the nucleus (E). Nuclear ICN forms a binding/activator complex with a group of cooperating
proteins (F), resulting in transcriptional activation of several functionally important genes, including MYC and pre-T. Hyperphosphorylation of ICN via interaction of
its PEST domain (polypeptide enriched in proline, glutamate, serine, and threonine) with CDK8, MAML, and p300 facilitates ubiquitylation (G) by SEL1 family
members, targeting ICN to the proteosome. ADAM=a disintegrin and metalloproteinase domain. DLL=Delta ligand. hes1=hairy/enhancer of split. GSI=-secretase
inhibition. MAML=mastermind-like proteins. MIB=mindbomb. NEURL=neuralised-like. Nrarp=gene encoding NOTCH-regulated ankyrin-repeat protein.
OFUT1=O-fucosyltransferase1. Pre-T=pre-TCR. Deltex=positive regulator of Notch signalling pathway. CS1=DNA binding component. CoA=co-activators.
CoR=co-repressors. Fringe=regulator of Notch ligand. SEL10=positive regulator of Notch. Numb=negative receptor of Notch.

important regulator of haemopoietic-cell development, such as the signalling pathway for RAS (GTP-binding
essential for denitive haemopoiesis.36 Similarly, Aml1 protein that activates target genes involved in cell
gene is essential for denitive embryonic haemo- dierentiation, proliferation, and survival).39 As an
poiesis.37,38 Thus, the presence of the TEL-AML1 fusion activated kinase, BCR-ABL oers an attractive therapeutic
protein in B-cell progenitors seems to lead to disordered target, and imatinib mesilate, a small-molecule inhibitor
early B-lineage lymphocyte development, a hallmark of of the ABL kinase, has proven eective against leukaemias
leukaemic lymphoblasts. Analysis of TEL-AML1-induced that express BCR-ABL.40
cord blood cells suggests that the fusion gene serves as a More than 50% of cases of T-cell acute lymphoblastic
rst-hit mutation by endowing the preleukemic cell with leukaemia have activating mutations that involve
altered self-renewal and survival properties.15 NOTCH1,41 a gene encoding a transmembrane receptor
In adults, the most frequent chromosomal translocation that regulates normal T-cell development.42 NOTCH
is t(9;22), or the Philadelphia chromosome, which causes receptors become activated when ligands of the
fusion of the BCR signalling protein to the ABL Delta-Serrate-Lag2 family of proteins bind to the
non-receptor tyrosine kinase, resulting in constitutive extracellular portion of the transmembrane molecule.
tyrosine kinase activity and complex interactions of this This interaction initiates a cascade of proteolytic cleavages,
fusion protein with many other transforming elements, terminating in -secretase generation of intracellular

1032 www.thelancet.com Vol 371 March 22, 2008


Seminar

NOTCH1, which translocates to the nucleus and regulates LEF1, IKZF1 (Ikaros), and IKZF3 (Aiolos). Finally, in
by transcription a diverse set of responder genes, T-cell acute lymphoblastic leukaemia, at least ve
including the MYC oncogene (gure 2).43,44 The precise multistep mutational pathways leading to frank leukaemia
mechanisms by which aberrant NOTCH signalling (due have been identied, and in some cases these pathways
to mutational activation) causes T-cell acute lymphoblastic entail ve or more documented genetic lesions.28,30,42
leukaemia are still unclear but probably entail constitutive Ongoing research to dene the oncogenic contributions
expression of oncogenic responder genes, such as MYC, of various classes of genetic lesions relies heavily on
and cooperation with other signalling pathways (pre-TCR animal models that accurately recapitulate the molecular
[T-cell receptor for antigen] and RAS, for example). pathogenesis of B-cell precursor or T-cell acute
Interference with NOTCH signalling by small-molecule lymphoblastic leukaemia.49 Most studies undertaken to
inhibition of -secretase activity has the potential to date have used genetically engineered mice to elucidate
induce remission of T-cell acute lymphoblastic the multistep transformation pathways leading to T-cell
leukaemia. acute lymphoblastic leukaemia.50 Such models depend
Evidence suggests that the MYC oncoprotein is an on breeding strategies to combine one or more genetic
important downstream mediator of the pro-growth lesions and show synergy in transformation,51 whereas
eects of NOTCH1 signalling in developing thym- some investigators have also capitalised on the use of
ocytes.28,45 However, results of retroviral insertional retroviral insertional mutagenesis screens to uncover
mutagenesis in murine models of transgenic T-cell acute collaborating oncogenes.46,52 New models of T-cell acute
lymphoblastic leukaemia show that Notch1 mutations, lymphoblastic leukaemia in the zebrash oer a powerful
with outcomes similar to those in primary human T-cell alternative vertebrate system for leukaemia research,
acute lymphoblastic leukaemias, can potentiate the whose unique advantages complement those of extant
eects of pre-existing MYC overexpression,42,46 suggesting murine models.53 Currently available zebrash models of
that NOTCH1 must have important transformational acute lymphoblastic leukaemia include a myc
targets other than MYC. Activating mutations in transgene-driven system, in which lymphoblasts
NOTCH1 sucient to produce constitutive NOTCH1 faithfully reproduce the multistep oncogenic pathway
signalling can induce T-cell acute lymphoblastic noted in up to 60% of human T-cell acute lymphoblastic
leukaemia in experimental models and could be the leukaemias,54,55 and a transgenic zebrash model, in
instigating event in most human T-cell leukaemias.28,42 which the TEL-AML1 oncoprotein induces B-cell
-secretase, a multicomponent membrane-associated precursor leukaemia.56
enzyme, is needed for NOTCH1 signalling through The challenge now is to understand how these
mutant NOTCH receptors in T-cell acute lymphoblastic cooperative genetic lesions and their aected pathways
leukaemia, providing an attractive target for therapeutic interact to alter the proliferation, dierentiation, and
intervention with newly developed -secretase survival of lymphocyte progenitors leading to their
inhibitors.28,47 leukaemic conversion. This research will undoubtedly
provide the molecular rationales needed to select new
Cooperating mutations therapeutic targets and to develop interfering small
Although chromosomal abnormalities are a hallmark of molecules or antibodies with high levels of antileukaemic
pathogenesis of acute lymphoblastic leukaemia, evidence specicity and activity.57 The table provides a partial list of
suggests that they must act in concert with several other molecularly targeted drugs now in clinical testing.
genetic lesions to induce overt leukaemia. A prime
example is the biallelic deletion or epigenetic silencing of Diagnosis
the cyclin-dependent kinase inhibitor 2A gene (CDKN2A), Phenotype
which encodes both the tumour suppressors p16INK4A and Immunophenotyping of leukaemic lymphoblasts by ow
p14ARF and whose inactivation neutralises both the TP53 cytometry is essential to establish the correct diagnosis
and retinoblastoma pathways in most cases of T-cell and and dene cell lineage. Although acute lymphoblastic
many cases of B-cell precursor acute lymphoblastic leukaemia can be readily subclassied according to the
leukaemia.6 In a genome-wide analysis of 242 cases of many steps of normal B-cell and T-cell dierentiation,
paediatric acute lymphoblastic leukaemia using the only ndings with therapeutic importance are T-cell,
high-resolution single nucleotide polymorphism arrays, mature B-cell, and B-cell precursor phenotypes.26,58
deletions, amplications, point mutations, and other Myeloid-associated antigen expression can be detected in
structural rearrangements were identied in genes as many as half the cases of acute lymphoblastic
encoding regulators of B-lymphocyte development in leukaemia. However, with contemporary treatment, this
40% of cases of B-cell precursor acute lymphoblastic so-called aberrant antigen expression has no prognostic
leukaemia.48 The PAX5 gene was the most frequent target implications but can be used to distinguish leukaemic
of somatic mutation, being altered in almost a third of cells from normal progenitor cells, thereby enabling
cases. Deletions were also detected in other B-cell detection of minimal (ie, submicroscopic) residual
developmental genes, such as TCF3 (E2A), EBF1 (EBF), leukaemia.6,26

www.thelancet.com Vol 371 March 22, 2008 1033


Seminar

Traditionally, pharmacogenetic studies have focused on


Mechanism of action Subtype of
leukaemia targeted single genes identied on the basis of their inuence
on the pharmacokinetics and pharmacological eects
Clofarabine Inhibits DNA polymerase and ribonucleotide All
reductase; disrupts mitochrondria of anticancer drugs. Findings of global gene-expression
membrane proling studies have identied a growing number of
Nelarabine Inhibits ribonucleotide reductase and DNA T-cell genomic determinants of treatment responses that
synthesis could allow development of polygenic models for
Forodesine Inhibits purine nucleoside phosphorylase T-cell optimisation of treatment for acute lymphoblastic
-secretase inhibitors Inhibit -secretase, an enzyme required for T-cell leukaemia.6,7,63
NOTCH1 signalling
Despite the promise of pharmacogenetic studies to
Rituximab Anti-CD20 chimeric murine-human CD20-positive
enhance treatment outcome in acute lymphoblastic
monoclonal antibody
leukaemia, only polymorphisms and the activity of
Epratuzumab Anti-CD22 humanised monoclonal antibody CD22-positive
thiopurine methyltransferasean enzyme that catalyses
Alemtuzumab Anti-CD52 humanised monoclonal antibody CD52-positive
S-methylation (inactivation) of thiopurines such as
Gemtuzumab ozogamicin Anti-CD33 monoclonal antibody conjugated CD33-positive
with calicheamicin mercaptopurine and thioguaninehave been useful in
Imatinib mesilate ABL kinase inhibition BCR-ABL-positive
clinical practice.7,64,65 About 10% of the total population
Nilotinib ABL kinase inhibition BCR-ABL-positive
inherit one wild-type gene encoding thiopurine
Dasatinib BCR-ABL kinase inhibition BCR-ABL-positive
methyltransferase and one non-functional variant allele,
MK-0457 Aurora kinase inhibition BCR-ABL-positive
resulting in intermediate enzyme activity, whereas 1 in
300 people inherit two non-functional variant alleles with
Lestaurtinib; midostaurin; tandutinib; FMS-like tyrosine kinase 3 inhibition MLL-rearranged;
sunitinib malate; IMC-EB10 hyperdiploid no enzyme activity. When treated with conventional
Tipifarnib; lonafarnib Farnesyltransferase inhibition All doses of thiopurines, up to half of patients with the
Azacytidine; decitabine; DNA methyltransferase inhibition All heterozygous deciency and all homozygous-decient
temozolomide patients develop haemopoietic toxic eects, which can be
Romidepsin; vorinostat; valproic acid; Histone deacetylase inhibition All fatal in the homozygous group.7 The enzyme deciency
MD-27-275; AN-9 also confers a high risk of developing therapy-related
Sirolimus; temsirolimus; everolimus; Mammalian target-of-rapamycin inhibition All acute myeloid leukemia7 and radiation-induced brain
AP-23573
tumours, in the context of intensive thiopurine
Bortzezomib Inhibition of ubiquitin proteasome pathway All
treatment.66 Conversely, patients with high levels of
Flavopiridol Serine-threonine cyclin-dependent kinase All
enzyme activity might be at greater risk of relapse owing
inhibition
to decreased exposure of leukaemic cells to active drug
Oblimersen Downregulation of BCL2 All
metabolites.65 In most centres, studies of thiopurine
17-AAG Heat shock protein-90 inhibitor BCR-ABL-positive;
ZAP-70-positive methyltransferase activity are undertaken only in people
with poor tolerance to antimetabolite-based therapy, and
Table: Selected antileukaemic drugs being tested in clinical trials
the result is used to guide reductions in drug dosage.67
We use a fairly high dose of mercaptopurine and, thus,
Genotype study this enzyme prospectively in all patients, lowering
Although chromosomal analysis is still an integral the dose of mercaptopurine in individuals with enzyme
component of initial work-up of acute lymphoblastic deciency.64
leukaemia, other highly specic and sensitive
techniquessuch as RT-PCR, uorescence in-situ Risk assessment
hybridisation, and ow cytometryare increasingly Careful assessment of the risk of relapse in individual
used to detect specic fusion transcripts, gain or loss of patients ensures that very intensive treatment is given
cellular DNA content, or specic chromosomes with only to high-risk cases, thus sparing people at lower risk
prognostic or therapeutic relevance.26,58 Although still a from undue toxic eects. Although enhanced treatment
research technique, gene-expression proling can not has abolished the prognostic strength of many clinical
only identify accurately the major subtypes of acute and biological risk factors identied in the past, we
lymphoblastic leukaemia but also implicate single genes would stress that even so-called low-risk patients need a
or signalling pathways as important determinants of certain degree of treatment intensication to avoid
clinical outcome.3234,5961 Once this method has been unacceptable rates of relapse. Findings have shown that
rened and made cost eective, it will undoubtedly adolescents and young adults who were treated on adult
replace many current diagnostic techniques. protocols fared signicantly worse than the same
age-groups treated on paediatric protocols.6870 The
Pharmacogenetics superior outcome achieved with paediatric regimens
Genetic polymorphisms of drug transporters, receptors, has been attributed to more eective treatment and to
targets, and drug-metabolising enzymes can aect the better adherence by patients, parents, and doctors.6872
eectiveness and toxic eects of antineoplastic drugs.7,62 To understand the actual basis for this dierence in

1034 www.thelancet.com Vol 371 March 22, 2008


Seminar

outcome, several combined adult and paediatric 1


consortia are using common regimens to treat patients
aged 150 years. 09

08
Clinical factors
07
Age at diagnosis has a strong prognostic eect (gure 3).
In work done at St Jude Childrens Research Hospital, 06

Probability
847 children with acute lymphoblastic leukaemia were 05
enrolled in four consecutive treatment protocols from 1991
04
to 2006. Children aged 19 years had a better outcome than
either infants or adolescents.26,58 5-year event-free survival 03 19 years (n=598)
estimates were 88% (SE 2) for children aged 19 years, >15 years (n=78)
02 1015 years (n=153)
73% (4) for adolescents aged 1015 years, 69% (7) for those <12 months (n=18)
older than 15 years, and 44% (11) for babies younger than 01

12 months. Babies younger than 6 months have an 0


especially poor outcome.73,74 The outcome of treatment in 0 2 4 6 8 10 12 14 16
adults worsens with increasing age. Indeed, in the past, Years from diagnosis
Number at risk
patients older than 60 years were not even included in <12 months 18 11 8 8 6 1 1 0 0
19 years 598 528 378 295 213 149 85 31 4
clinical trials owing to their many coexisting health 1015 years 153 130 100 77 50 26 11 6 1
problems, their heightened susceptibility to >15 years 78 60 40 29 20 7 1 0 0
treatment-related toxic eects, and their high frequency of
Philadelphia chromosome-positive acute lymphoblastic Figure 3: Kaplan-Meier estimates of event-free survival according to age at diagnosis of acute
leukaemia.1,4,75 This practice has begun to change, partly lymphoblastic leukaemia
because of enhanced supportive care now available for
older adults and development of specic tyrosine kinase black children had the same high cure rates as did white
inhibitors for Philadelphia chromosome-positive acute children when given equal access to eective treatment,80
lymphoblastic leukaemia (table).75,76 underscoring the over-riding prognostic importance of
Leucocyte count is a continuous prognostic variable, treatment. The adverse prognosis previously ascribed to
with increasing counts conferring a poorer outcome, male sex has also been abolished with enhanced treatment
especially in patients with B-cell precursor disease.58,76 regimens.2,81
In T-cell acute lymphoblastic leukaemia, a leucocyte Likewise, the eect of obesity on outcome of acute
count greater than 100x109/L is associated with an lymphoblastic leukaemia is also treatment dependent. A
increased risk of relapse in the CNS.77 Patients with report by the Childrens Oncology Group showed that
extreme hyperleucocytosis (>400x109/L) are at high risk overweight children with acute lymphoblastic leukaemia,
for early complications such as CNS haemorrhage and aged 10 years or older, have a poor treatment outcome.82
pulmonary and neurological events due to leucostasis.78 By contrast, we noted no association between the
A uniform risk-classication system, based on both age body-mass index of patients with acute lymphoblastic
and leucocyte count, was devised to facilitate leukaemia and clinical outcome, toxic eects, or the
comparisons of treatment results in childhood acute pharmacokinetics of several drugs tested.83
lymphoblastic leukaemia.26 Two-thirds of patients aged
19 years with a leucocyte count less than 50x109/L were Biological factors
judged to have a standard (or low) risk of relapse, T-cell and mature B-cell immunophenotypes, once
whereas the remaining third were classied as high associated with a poor outcome, have little prognostic
risk. This system by itself has limited value because up importance in childhood acute lymphoblastic leukaemia
to a third of the so-called standard-risk patients could and are actually favourable features in adult disease in the
relapse, and individuals at very high riskwho need context of contemporary treatment.2,76,81,84 Although genetic
allogeneic haemopoietic stem-cell transplantation abnormalities do not account entirely for treatment
cannot be distinguished reliably from high-risk cases.26 outcome they still provide indispensable prognostic
Moreover, risk criteria apply only to B-cell precursor information (gure 4). 841 children with acute
acute lymphoblastic leukaemia and have little prognostic lymphoblastic leukaemia and successful cytogenetic and
value in T-cell disease. immunophenotypic studies were enrolled in four
In US cooperative group studies, black and Hispanic consecutive treatment protocols at St Jude Childrens
patients fared worse than similarly treated white Research Hospital from 1991 to 2006. Patients with
individuals.79 The poor prognosis for black people could be hyperdiploidy (>50 chromosomes), TEL-AML1 fusion, and
related to their high frequency of T-cell acute lymphoblastic t(1;19)/E2A-PBX1 fusion had the most favourable outcome,
leukaemia and the t(1;19) chromosomal abnormality with whereas those with the t(9;22)/BCR-ABL fusion or t(4;11)/
E2A-PBX1 fusion.80 However, in single-institution studies, MLL-AF4 fusion had a dismal prognosis. 5-year event-free

www.thelancet.com Vol 371 March 22, 2008 1035


Seminar

survival estimates were 91% (SE 3) for hyperdiploidy, 89% favourable prognosis in childhood T-cell acute
(3) for TEL-AML1 fusion, 86% (7) for E2A-PBX1 fusion, lymphoblastic leukaemia94 but an unfavourable outcome
82% (3) for other B-lineage disease, 73% (5) for T-cell acute in adults.95
lymphoblastic leukaemia, 37% (12) for BCR-ABL fusion, Data of microarray analyses of leukaemic cells identied
and 32% (12) for MLL-AF4 fusion. genes that aect the intracellular disposition of
In general, the Philadelphia chromosome, t(4;11) with antileukaemic drugs and have shown distinct sets of
MLL-AF4 fusion, and hypodiploidy (<44 chromosomes genes that are associated with resistance to dierent
per leukaemic cell) all confer a poor outcome, whereas classes of antileukaemic agents.5961,9699 Aberrant
hyperdiploidy (>50 chromosomes), TEL-AML1 fusion, and expression of some genes also seemed to have
trisomy 4, 10, and 17 are associated with favourable prognostic relevance.59,60 It is also noteworthy that
prognosis.6,8587 About 2% of childhood cases were noted to numerical chromosomal abnormalities, depending on
have intrachromosomal amplication of chromosome 21, whether the aected chromosomes contain the wild-type
which is associated with a B-cell precursor imm- or variant allele of the genes, can greatly aect the
unophenotype, older age, low white-cell counts, and, more pharmacogenomics of cancer treatment and, thus,
importantly, a threefold increase in risk of relapse.88 The clinical outcome.100
high frequency of unfavourable genetic features and low
rate of favourable genetic abnormalities in adults with Response to treatment
acute lymphoblastic leukaemia partly explain their inferior Response to treatment is determined by the entire con-
outcome compared with childhood cases.76 stellation of leukaemic-cell biological features (intrinsic
Age aects the prognostic importance of genetic drug sensitivity) in concert with the pharmacodynamics
abnormalities for unknown reasons. In children with and pharmacogenomics of the host, the regimens
Philadelphia chromosome-positive acute lymphoblastic administered, and treatment adherence. Not surprisingly,
leukaemia, those aged 19 years fared better than did the degree of reduction of the leukaemic cell clone early
adolescents,89 who in turn had a better prognosis than during remission induction therapy has independent
adults.90,91 In patients with MLL-AF4 fusion, infants and prognostic importance1,26,101104 even in low-risk cases dened
adults have a worse prognosis than children.9193 The by clinical and biological features.105 However,
t(1;19) with E2A-PBX1 fusion has no prognostic morphology-based methods traditionally used to assess
implications in childhood acute lymphoblastic leukaemia treatment response are neither precise enough nor
but is still associated with a poor prognosis in some adult sensitive enough to measure this cytoreduction reliably.26,106
cases.76 Findings are scarce to suggest that activating Molecular and ow-cytometric methods, which are at least
mutations of the NOTCH1 gene are associated with a 100-fold more sensitive than morphological determinations,
now allow minimal residual leukaemia to be detected at
very low levels (<001%), providing an useful means to
1
identify patients at very low or high risk of relapse.1,26,101106
09 Indeed, patients with 1% or more leukaemic cells at the
08
end of 46 weeks of remission induction therapy fare
almost as poorly as those who fail to achieve clinical
07
remission by the accepted morphological standard
06 (5% leukaemic cells), whereas people who achieve
Probability

molecular or immunological remission (<001% leukaemic


05
cells) have an excellent outcome.1,26 Findings of one study
04 in adults showed that monitoring of minimal residual
03 leukaemia can be used for early detection of impending
Hyperdiploidy (n=205) T cell (n=138)
relapse and hence for early treatment intervention.107
02
E2APBX1 (n=40) BCRABL (n=22) Residual leukaemic T cells can be detected easily by
01 TELAML1 (n=163) MLLAF4 (n=15) their positivity for terminal deoxynucleotidyl transferase
Other B-lineage (n=261)
0
and cytoplasmic CD3. A simple and inexpensive assay
0 2 4 6 8 10 12 14 16 for minimal residual leukaemia has been developed for
Years from diagnosis B-cell precursor acute lymphoblastic leukaemia.108 It is
Number at risk based on the rationale that normal, immature, CD19+
Hyperdiploidy 205 190 144 108 80 52 25 10 1
E2APBX1 40 36 27 19 14 9 6 0 0
B-cell progenitors (those expressing CD10, CD34, or
TELAML1 163 144 105 83 60 46 30 10 0 both) are exquisitely sensitive to corticosteroids and other
Other B-lineage 261 221 161 130 92 50 28 13 3 antileukaemic drugs and are undetectable consistently in
T cell 138 112 75 60 36 22 8 3 1 bone marrow after 2 weeks of remission induction
BCRABL 22 15 7 5 3 2 0 0 0
MLLAF4
treatment. Hence, any cells with this immunophenotype
15 9 6 4 4 2 1 1 0
after 2 weeks of remission induction probably represent
Figure 4: Kaplan-Meier analysis of event-free survival according to biological subtype of leukaemia minimal residual leukaemia.108

1036 www.thelancet.com Vol 371 March 22, 2008


Seminar

Treatment asparaginase treatment (ie, the amount of asparagine


With the exception of patients with mature B-cell acute depletion) are far more important than the type of
lymphoblastic leukaemia, who are treated with short- asparaginase used.1,2 Compared with Escherichia coli
term intensive chemotherapy (including high-dose asparaginase, Erwinia asparaginase was associated with
methotrexate, cytarabine, and cyclophosphamide),109111 inferior antileukaemic response but fewer toxic
treatment for acute lymphoblastic leukaemia typically eects,121,122 a nding now attributed to use of inadequate
consists of a remission-induction phase, an intensication doses of the Erwinia drug. In some current protocols,
(or consolidation) phase, and continuation therapy to polyethylene glycol-conjugated asparaginasea long-
eliminate residual disease. Treatment is also directed to acting and less allergenic formhas replaced the native
the CNS early in the clinical course to prevent relapse product in initial treatment.123126 Many complications
attributable to leukaemic cells sequestered in this site.1 recorded during remission induction are attributable to
The drugs currently in use for these phases were the synergistic eects of corticosteroid and asparaginase.
developed and tested between the 1950s and 1970s, but In the context of multiagent treatment, a fairly small
eorts to identify new antileukaemic agents have begun increase in dose of dexamethasone or asparaginase can
to intensify (table). result in excessive toxic eects and death, especially in
older children and adults.
Remission-induction phase
The goal of remission-induction treatment is to eradicate Consolidation (intensication) treatment
more than 99% of the initial leukaemic cell burden and With the restoration of normal haemopoiesis and body
to restore normal haemopoiesis and healthy performance function, intensication treatment is generally used to
status. This approach typically includes administration of eradicate drug-resistant residual leukaemic cells, thus
a glucocorticoid (prednisone or dexamethasone), reducing the risk of relapse. For example, patients with
vincristine, and at least a third drug (asparaginase, TEL-AML1-positive disease have an especially good
anthracycline, or both). A three-drug induction regimen outcome in clinical trials of intensive post-remission
seems sucient for most standard-risk cases provided therapy with corticosteroids, vincristine, and aspara-
they receive intensied post-remission treatment. ginase.81,127 Although the importance of this treatment
Children with high-risk or very high-risk acute lymph- phase is rarely disputed, consensus is scarce on the best
oblastic leukaemia, and virtually all adult cases of the regimens and duration of treatment. Frequently used
disease, are treated with four or more drugs for remission strategies include high-dose methotrexate plus
induction.1 We measure levels of minimal residual mercaptopurine, reinduction treatment with the same
leukaemia after 2 weeks of remission induction and we agent that was given initially, frequent pulses of
intensify treatment in patients with high amounts of vincristine and corticosteroid plus high-dose asparaginase
residual blasts (>1%). Clinical remission can now be for 2030 weeks, and an augmented regimen consisting
induced in 9699% of children and 7893% of adults.1 of reinduction treatment and additional doses of
Although no induction regimen is clearly superior to any vincristine, asparaginase, and intravenous methotrexate
others, addition of cyclophosphamide and intensive during periods of myelosuppression.13,128 For patients
treatment with asparaginase are widely considered with high-risk or very high-risk acute lymphoblastic
benecial to patients with T-cell acute lymphoblastic leukaemia, incorporation of high-dose methotrexate plus
leukaemia,2,76 and imatinib mesilate has greatly enhanced mercaptopurine into a regimen based on intensive
the remission-induction rate, duration of disease-free asparaginase treatment could be desirable. Findings of
survival, and quality of life of patients with Philadelphia ongoing studies will establish if these approaches in
chromosome-positive acute lymphoblastic leukaemia.112114 children are eective and tolerable in adults.
Whether the cure rate of this subtype of leukaemia can Reinduction treatment has become an integral
be raised with imatinib or the newly developed, more component of contemporary protocols. In one randomised
potent, tyrosine kinase inhibitors nilotinib and dasatinib study of intermediate-risk acute lymphoblastic leukaemia,
remains unknown.115,116 double reinduction further enhanced treatment outcome,
Presumably because of its longer half-life and whereas additional pulses of vincristine and prednisone
increased penetration into the CNS, dexamethasone after one reinduction course were not benecial, suggesting
has been deemed more eective than either prednisone that the increased dose-intensity of other drugssuch as
or prednisolone for treatment of acute lymphoblastic asparaginaseled to the noted improvement.129 Although
leukaemia.117,118 However, ndings of a small randomised a standard intensication regimen for adult acute
study showed that an augmented dose of prednisolone lymphoblastic leukaemia is absent, post-remission
produced results comparable with those achieved with treatment with cytarabine, cyclophosphamide, anthra-
dexamethasone in the context of other intensive cyclines, and methotrexate has improved outcome in some
treatment.119 Similarly, the pharmacodynamics of non-randomised studies.130132
asparaginase dier by formulation,120 and in terms of The best dose of methotrexate depends on the
leukaemia control, the dose intensity and duration of leukaemic-cell genotype and phenotype and host pharma-

www.thelancet.com Vol 371 March 22, 2008 1037


Seminar

cogenetic and pharmacokinetic variables. Methotrexate between 05 and 1510/L to ensure adequate dose
at 12 g/m is adequate for most patients with intensity during the continuation phase.1
standard-risk acute lymphoblastic leukaemia, but a Since thioguanine is more potent than mercaptopurine
higher dose (eg, 5 g/m) might benet individuals with in model systems and leads to higher concentrations of
T-cell or high-risk B-cell precursor disease.133,134 The fairly thioguanine nucleotides in cells and cytotoxic concentra-
low accumulation of methotrexate polyglutamates in tions in cerebrospinal uid,149 several randomised trials
blast cells with either TEL-AML1 or E2A-PBX1 fusion have been done to compare the eectiveness of these two
suggests that patients with these genotypes could also drugs.150152 Thioguanine, given at a daily dose of 40 mg/m
benet from an increased dose of methotrexate.98 or more, produced superior antileukaemic responses to
However, mega doses of methotrexate (eg, 336 g/m) do mercaptopurine but was associated with profound
not seem necessary for patients with acute lymphoblastic thrombocytopenia, an increased risk of death in
leukaemia.135 Finally, leucovorin rescue, although remission, and an unacceptably high rate (1020%) of
necessary after treatment with high-dose methotrexate, hepatic veno-occlusive disease.150152 Although the lower
must not be given too early or at too high a dosage activity of thiopurine methyltransferase is associated with
because it might counteract the antileukaemic eects of thioguanine-related liver damage, this measure cannot
methotrexate.134136 identify reliably patients at risk.153 Mercaptopurine,
therefore, remains the drug of choice for acute lymph-
Allogeneic haemopoietic stem-cell oblastic leukaemia, although thioguanine could still be
transplantation given in short-term courses during the intensication
Allogeneic haemopoietic stem-cell transplantation is the phase of treatment.
most intensive form of treatment for acute lymphoblastic In a multicentre randomised trial, addition of six pulses
leukaemia. Comparisons between this modality and of vincristine and dexamethasone during early continu-
intensive chemotherapy have yielded inconsistent results ation treatment failed to improve outcome of children
owing to the few patients studied and dierences in with intermediate-risk acute lymphoblastic leukaemia.154
case-selection criteria.137140 Nonetheless, allogeneic trans- Whether more intensive pulse therapy would enhance
plantation clearly benets several subgroups of patients outcome in the context of contemporary therapy remains
with high-risk acute lymphoblastic leukaemia, such as to be studied.
individuals with Philadelphia chromosome-positive
disease (even when treated with a tyrosine kinase CNS-directed treatment
inhibitor) and those with a poor initial response to CNS relapse is a major obstacle to cure, accounting for
treatment.89,113,137,140142 It also improves the outcome of 3040% of initial relapses in some studies.117,129,155 Factors
adults with the t(4;11) subtype of acute lymphoblastic associated with an increased risk of CNS relapse include
leukaemia, but its benets in infants with this genotype a T-cell immunophenotype, hyperleucocytosis, high-risk
are controversial.74,92,143,144 Findings of studies suggest that genetic abnormalities, and presence of leukaemic cells in
matched unrelated-donor or cord-blood transplantation cerebrospinal uid (even from iatrogenic introduction
could produce results comparable with those obtained due to a traumatic lumbar puncture).3,77,156158 Poly-
with matched related-donor transplantation.145,146 In view morphisms in genes that code for proteins implicated in
of the substantial morbidity and mortality associated the pharmacodynamics of antileukaemic drugs have also
with this procedure and the growing prospects for been associated with risk of CNS relapse.63
eective targeted therapy, the need for allogeneic Because of its many associated acute and late
transplantation should be reassessed continuously. complications,159,160 cranial irradiation is now administered
Autologous transplantation, despite several practical to only 520% of patients at high risk for CNS relapse.77
advantages, has failed to enhance outcome in either adult With eective systemic treatment, the radiation dose can
or paediatric acute lymphoblastic leukaemia.138,147 be lowered to 12 Gy for most patients and to 18 Gy for
those with CNS leukaemia at diagnosis.133 In fact, 18 Gy
Continuation treatment irradiation was shown to be eective even in patients
For reasons that (currently) remain elusive, patients with with late isolated CNS relapse, in the context of intensive
acute lymphoblastic leukaemia need continuation systemic chemotherapy.161 We are testing the feasibility
treatment to prevent or forestall relapse. Although about of omitting radiation for all patients with acute
two-thirds of childhood cases can be treated successfully lymphoblastic leukaemia, reserving its use exclusively
with only 12 months of therapy, they cannot be identied for remission retrieval therapy. Whether or not cranial
prospectively with any degree of certainty.148 Hence, all radiation is used, the best regimen of intrathecal therapy
patients receive chemotherapy for 2025 years. Daily should be administered.162 To avoid traumatic lumbar
mercaptopurine and methotrexate every week constitute puncture from the repeated procedure and potential
the backbone of continuation regimens. Many investigators CNS seeding, we give intrathecal therapy with the very
advocate that drug dosages be adjusted to maintain rst diagnostic lumbar puncture, after the diagnosis of
leucocyte counts below 310/L and neutrophil counts leukaemia has been established. Some investigators

1038 www.thelancet.com Vol 371 March 22, 2008


Seminar

recommend an Ommaya reservoir for this treatment in the case of -secretase inhibitors, alleviation of adverse
adults with acute lymphoblastic leukaemia.76 In one eects on gastrointestinal stem cells seems to be possible
randomised trial, triple intrathecal therapy with through an intermittent schedule that is still eective
methotrexate, cytarabine, and hydrocortisone was more against leukaemic cells. This pitfall, and possible
eective than intrathecal methotrexate in preventing avoidance strategies based on drug scheduling, will loom
CNS relapse, but it was associated with an increased especially large in leukaemia subtypes in which malignant
frequency of bone marrow or testicular relapse.163 One cells have become addicted to signalling pathways that
explanation for this seemingly paradoxical nding is that are also essential for maintenance and renewal of healthy
an isolated CNS relapse is, in fact, an early manifestation tissues. As daunting as these challenges can seem, the
of systemic relapse, and the better CNS control secured payo in terms of understanding the pathobiology of
with triple intrathecal therapy does not obviate overt acute lymphoblastic leukaemia and devising novel
leukaemic relapse in other sites. If so, more eective eective treatments with few or no toxic eects could be
systemic chemotherapy is needed before the full benet enormous, making it our charge to bring this promise to
of triple intrathecal therapy can be realised. Indeed, fruition.
systemic treatment has a substantial role in prevention Are cancer stem cells likely to aect development of
of CNS relapse.117,118,122,152 future targeted treatments for acute lymphoblastic
leukaemia? Current evidence suggests that the stem-cell
Remaining questions and the future properties of certain human cancers could cause a
What are the major causative factors in the development resurgence of tumour unless the malignant stem cells are
of acute lymphoblastic leukaemia? Apart from isolated specically targeted by treatment.167 Findings show that
cases that can be attributed to inherited genetic syn- transformation of committed haemopoietic progenitors
dromes or exposures to known leukaemogenic agents, by the MLL-AF9 oncoprotein can impart stem-cell
identication of causal factors with a predictable eect properties, especially a self-renewal-associated genetic
on substantial numbers of children or adults has not programme.168 Whether important subpopulations of
been possible, impeding eorts to develop eective prev- leukaemic cells with stem-cell properties underlie some
entive measures against acute lymphoblastic leukaemia. cases of relapsed acute lymphoblastic leukaemia remains
In view of the failure of large-scale epidemiological to be determined and, therefore, they must be considered
studies to nd such associations, future research in this in the design of molecular therapeutics. Finally, increasing
area will probably restrict its focus to patients with a evidence suggests that the homing and engraftment
common primary genetic lesion, such as those with properties of leukaemic stem cells dier from those of
either BCR-ABL, MLL-AF4, or TEL-AML1 fusion, or normal haemopoietic stem cells169,170 and that bone-marrow
hyperdiploidy. mesenchymal cells can protect leukaemic cells from the
Assuming that molecular therapeutics will eventually cytotoxic eects of chemotherapy.171 Possibly, enhanced
replace standard combination chemotherapy and understanding of the molecular interactions between
haemopoietic stem-cell transplantation in the manage- leukaemic cells and the bone-marrow microenvironment
ment of patients with acute lymphoblastic leukaemia, will lead to treatment strategies that enhance the
which molecules implicated in disease pathogenesis are antileukaemic eects of chemotherapy.
most likely to yield substantial clinical benets? Conict of interest statement
Experience to date shows that transient responses can be C-HP received honoraria from Enzon, Sano Aventis, and Mundipharm
obtained by inhibition of certain key enzymes, such as for lectures. ATL received research support from Merck.
tyrosine kinases, DNA methyltransferase, histone Acknowledgments
deacetylase, secretase, serine-threonine kinases, and Supported in part by grants CA21765, CA51001, CA60419, CA36401,
CA78224, GM61393, NR07610, CA90246, CA52259, CA68484, CA06516
proteosomes (table).164 However, rapid development of from the National Institutes of Health, and by the Amercian Lebanese
drug resistance suggests that curative treatment will Syrian Associated Charities. These sponsors had no role in writing of the
need alternative strategies. For example, short-lived Seminar.
remissions induced in BCR-ABL-positive acute lymph- References
oblastic leukaemia by imatinib mesilate suggest a need 1 Pui CH, Evans WE. Treatment of acute lymphoblastic leukemia.
N Engl J Med 2006; 354: 16678.
to combine this drug with newly developed ABL-kinase
2 Silverman LB, Gelber RD, Dalton VK, et al. Improved outcome for
inhibitors, agents whose mechanism of action diers children with acute lymphoblastic leukemia: results of Dana-Farber
from that of imatinib, or with specic inhibitors of Consortium Protocol 91-01. Blood 2001; 97: 121118.
pathways downstream of, or parallel, to the BCR-ABL 3 Brger B, Zimmermann M, Mann G, et al. Diagnostic
cerebrospinal uid examination in children with acute
pathway.39,115,116,165 A dierent situation arises when lymphoblastic leukemia: signicance of low leukocyte counts with
NOTCH signalling is interrupted. That is, proliferative blasts or traumatic lumbar puncture. J Clin Oncol 2003; 21: 17981.
intestinal crypt cells are destined to become post-mitotic 4 Vitale A, Guarini A, Chiaretti S, Fo R. The changing scene of adult
acute lymphoblastic leukemia. Curr Opin Oncol 2006; 18: 65259.
goblet cells in the absence of NOTCH signals,47,166 raising 5 Romn-Gmez J, Cordeu L, Agirre X, et al. Epigenetic regulation of
the spectre of on-target toxic eects in human trials of Wnt-signaling pathway in acute lymphoblastic leukemia. Blood
-secretase inhibitors and other targeted therapeutics. In 2007; 109: 346269.

www.thelancet.com Vol 371 March 22, 2008 1039


Seminar

6 Pui CH, Relling MV, Downing JR. Acute lymphoblastic leukemia. 33 Armstrong SA, Staunton JE, Silverman LB, et al. MLL
N Engl J Med 2004; 350: 153548. translocations specify a distinct gene expression prole that
7 Evans WE, Relling MV. Moving towards individualized medicine distinguishes a unique leukemia. Nat Genet 2002; 30: 4147.
with pharmacogenomics. Nature 2004; 429: 46468. 34 Ferrando AA, Armstrong SA, Neuberg DS, et al. Gene expression
8 Hjalgrim LL, Westergaard T, Rostgaard K, et al. Birth weight as a signatures in MLL-rearranged T-lineage and B-precursor acute
risk factor for childhood leukemia: a meta-analysis of leukemias: dominance of HOX dysregulation. Blood 2003;
18 epidemiologic studies. Am J Epidemiol 2003; 158: 72435. 102: 26268.
9 Ahlbom A, Day N, Feychting M, et al. A pooled analysis of magnetic 35 Lu J, Getz G, Miska EA, et al. MicroRNA expression proles
elds and childhood leukaemia. Br J Cancer 2000; 83: 69298. classify human cancers. Nature 2005; 435: 83438.
10 Buer PA, Kwan ML, Reynolds P, Urayama KY. Environmental and 36 Hock H, Meade E, Medeiros S, et al. Tel/Etv6 is an essential and
genetic risk factors for childhood leukemia: appraising the selective regulator of adult hematopoietic stem cell survival.
evidence. Cancer Invest 2005; 23: 6075. Genes Dev 2004; 18: 233641.
11 Kinlen L. Infections and immune factors in cancer: the role of 37 Okuda T, van Deursen J, Hiebert SW, Grosveld G, Downing JR.
epidemiology. Oncogene 2004; 23: 634148. AML1, the target of multiple chromosomal translocations in
human leukemia, is essential for normal fetal liver hematopoiesis.
12 Greaves M. Infection, immune responses and the aetiology of
Cell 1996; 84: 32130.
childhood leukaemia. Nat Rev Cancer 2006; 6: 193203.
38 Wang Q, Stacy T, Binder M, et al. Disruption of the Cbfa2 gene
13 Wiemels JL, Cazzaniga G, Daniotti M, et al. Prenatal origin of acute
causes necrosis and hemorrhaging in the central nervous system and
lymphoblastic leukaemia in children. Lancet 1999; 354: 1499503.
blocks denitive hematopoiesis. Proc Natl Acad Sci USA 1996;
14 Maia AT, van der Velden VH, Harrison CJ, et al. Prenatal origin of 93: 344449.
hyperdiploid acute lymphoblastic leukemia in identical twins.
39 Ren R. Mechanisms of BCR-ABL in the pathogenesis of chronic
Leukemia 2003; 17: 200206.
myelogenous leukaemia. Nat Rev Cancer 2005; 5: 17283.
15 Hong D, Gupta R, Ancli O, et al. Initiating and cancer-propagating
40 Druker BJ. Imatinib as a paradigm of targeted therapies.
cells in TEL-AML1-associated childhood leukemia. Science 2008;
Adv Cancer Res 2004; 91: 130.
319: 33639.
41 Weng AP, Ferrando AA, Lee W, et al. Activating mutations of
16 Mori H, Colman SM, Xiao Z, et al. Chromosome translocations and
NOTCH1 in human T cell acute lymphoblastic leukemia. Science
covert leukemic clones are generated during normal fetal
2004; 306: 26971.
development. Proc Natl Acad Sci USA 2002; 99: 824247.
42 Grabher C, von Boehmer H, Look AT. Notch 1 activation in the
17 Greaves MF, Wiemels J. Origins of chromosome translocations in
molecular pathogenesis of T-cell acute lymphoblastic leukaemia.
childhood leukaemia. Nat Rev Cancer 2003; 3: 63949.
Nat Rev Cancer 2006; 6: 34759.
18 Lanciotti M, Dufour C, Corral L, et al. Genetic polymorphism of
43 Weng AP, Millholland JM, Yashiro-Ohtani Y, et al. c-Myc is an
NAD(P)H: quinone oxidoreductase is associated with an increased
important direct target of Notch1 in T-cell acute lymphoblastic
risk of infant acute lymphoblastic leukemia without MLL gene
leukemia/lymphoma. Genes Dev 2006; 20: 2096109.
rearrangements. Leukemia 2005; 19: 21416.
44 Palomero T, Lim WK, Odom DT, et al. NOTCH1 directly regulates
19 Krajinovic M, Labuda D, Sinnett D. Glutathione S-transferase P1
c-MYC and activates a feed-forward-loop transcriptional network
genetic polymorphisms and susceptibility to childhood acute
promoting leukemic cell growth. Proc Natl Acad Sci USA 2006;
lymphoblastic leukaemia. Pharmacogenetics 2002; 12: 65558.
103: 18 26166.
20 Skibola CF, Smith MT, Kane E, et al. Polymorphisms in the methylen
45 Sharma VM, Calvo JA, Draheim KM, et al. Notch1 contributes to
etetrahydrofolate reductase gene are associated with susceptibility to
mouse T-cell leukemia by directly inducing the expression of
acute leukemia in adults. Proc Natl Acad Sci USA 1999; 96: 1281015.
c-myc. Mol Cell Biol 2006; 26: 802231.
21 Skibola CF, Smith MT, Hubbard A, et al. Polymorphisms in the
46 Hoemann CD, Beaulieu N, Girard L, Rebai N, Jolicoeur P. Two
thymidylate synthase and serine hydroxymethyltransferase genes and
distinct Notch1 mutant alleles are involved in the induction of
risk of adult acute lymphocytic leukemia. Blood 2002; 99: 378691.
T-cell leukemia in c-myc transgenic mice. Mol Cell Biol 2000;
22 Healy J, Blanger H, Beaulieu P, et al. Promoter SNPs in G1/S 20: 383142.
checkpoint regulators and their impact on the susceptibility to
47 Wong GT, Manfra D, Poulet FM, et al. Chronic treatment with the
childhood leukemia. Blood 2007; 109: 68392.
gamma-secretase inhibitor LY-411,575 inhibits beta-amyloid peptide
23 Gast A, Bermejo JL, Flohr T, et al. Folate metabolic gene production and alters lymphopoiesis and intestinal cell
polymorphisms and childhood acute lymphoblastic leukemia: a dierentiation. J Biol Chem 2004; 279: 12 87682.
case-control study. Leukemia 2007; 21: 32025.
48 Mullighan CG, Goorha S, Radtke I, et al. Genome-wide analysis of
24 Spector LG, Xie Y, Robison LL, et al. Maternal diet and infant genetic alterations in acute lymphoblastic leukaemia. Nature 2007;
leukemia: the DNA topoisomerase II inhibitor hypothesis: a report 446: 75864.
from the Childrens Oncology Group.
49 Van Dyke T, Jacks T. Cancer modeling in the modern era: progress
Cancer Epidemiol Biomarkers Prev 2005; 14: 65155.
and challenges. Cell 2002; 108: 13544.
25 Pui CH, Relling MV. Topoisomerase II inhibitor-related acute
50 ONeil J, Shank J, Cusson N, Murre C, Kelliher M. TAL1/SCL
myeloid leukaemia. Br J Haematol 2000; 109: 1323.
induces leukemia by inhibiting the transcriptional activity of
26 Pui CH, Campana D, Evans WE. Childhood acute lymphoblastic E47/HEB. Cancer Cell 2004; 5: 58796.
leukaemia: current status and future perspectives. Lancet Oncol
51 Eischen CM, Weber JD, Roussel MF, Sherr CJ, Cleveland JL.
2001; 2: 597607.
Disruption of the ARF-Mdm2-p53 tumor suppressor pathway in
27 Biondi A, Cimino G, Pieters R, Pui CH. Biological and therapeutic Myc-induced lymphomagenesis. Genes Dev 1999; 13: 265869.
aspects of infant leukemia. Blood 2000; 96: 2433.
52 Dupuy AJ, Akagi K, Largaespada DA, Copeland NG, Jenkins NA.
28 Armstrong SA, Look AT. Molecular genetics of acute lymphoblastic Mammalian mutagenesis using a highly mobile somatic Sleeping
leukemia. J Clin Oncol 2005; 23: 630615. Beauty transposon system. Nature 2005: 436: 22126.
29 Wang JC, Dick JE. Cancer stem cells: lessons from leukemia. 53 Amatruda JF, Shepard JL, Stern HM, Zon LI. Zebrash as a cancer
Trends Cell Biol 2005; 15: 494501. model system. Cancer Cell 2002; 1: 22931.
30 Ferrando AA, Neuberg DS, Staunton J, et al. Gene expression 54 Langenau DM, Traver D, Ferrando AA, et al. Myc-induced T cell
dene novel oncogenic pathways in T-cell acute lymphoblastic leukemia in transgenic zebrash. Science 2003; 299: 88790.
leukemia. Cancer Cell 2002; 1: 7587.
55 Langenau DM, Feng H, Berghmans S, et al. Cre/lox-regulated
31 Weinstein IB, Joe AK. Mechanisms of disease: oncogene transgenic zebrash model with conditional myc-induced T cell
addictiona rationale for molecular targeting in cancer therapy. acute lymphoblastic leukemia. Proc Natl Acad Sci USA 2005;
Nat Clin Pract Oncol 2006; 3: 44857. 102: 606873.
32 Yeoh EJ, Ross ME, Shurtle SA, et al. Classication, subtype 56 Sabaawy HE, Azuma M, Embree LJ, et al. TEL-AML1 transgenic
discovery, and prediction of outcome in pediatric acute zebrash model of precursor B cell acute lymphoblastic leukemia.
lymphoblastic leukemia by gene expression proling. Cancer Cell Proc Natl Acad Sci USA 2006; 103: 15 16671.
2002; 1: 13343.
57 Croce CM. Oncogenes and cancer. N Engl J Med 2008; 358: 50211.

1040 www.thelancet.com Vol 371 March 22, 2008


Seminar

58 Pui CH, Evans WE. Acute lymphoblastic leukemia. N Engl J Med 81 Pui CH, Sandlund JT, Pei D, et al. Improved outcome for
1998; 339: 60515. children with acute lymphoblastic leukemia: results of Total
59 Mi S, Lu J, Sun M. MicroRNA expression signatures accurately Therapy Study XIIIB at St Jude Childrens Research Hospital.
discriminate acute lymphoblastic leukemia from acute myeloid Blood 2004; 104: 269096.
leukemia. Proc Natl Acad Sci USA; 2007; 104: 1997176. 82 Butturini AM, Dorey FJ, Lange BJ, et al. Obesity and outcome in
60 Kawamata N, Ogawa S, Zimmermann M, et al. Molecular pediatric acute lymphoblastic leukemia. J Clin Oncol 2007;
allelokaryotyping of pediatric acute lymphoblastic leukemias by 25: 206369.
high-resolution single nucleotide polymorphism oligonucleotide 83 Hijiya N, Panetta JC, Zhou Y, et al. Body mass index does not
genomic microarray. Blood 2008; 111: 77684. inuence pharmacokinetics or outcome of treatment in children
61 Tissing WJ, den Boer ML, Meijerink JP, et al. Genomewide with acute lymphoblastic leukemia. Blood 2006; 108: 39974002.
identication of prednisolone-responsive gene in acute 84 Rowe JM, Buck G, Burnett AK, et al. Induction therapy for adults
lymphoblastic leukemia cells. Blood 2007; 109: 392935. with acute lymphoblastic leukemia: results of more than 1500
62 Kishi S, Cheng C, French D, et al. Ancestry and pharmacogenetics patients from the international ALL trialMRC UKALL XII/
of antileukemic drug toxicity. Blood 2007; 109: 415157. ECOG E2993. Blood 2005; 106: 376067.
63 Rocha JC, Cheng C, Liu W, et al. Pharmacogenetics of outcome in 85 Moorman AV, Harrison CJ, Buck GA, et al. Karyotype is an
children with acute lymphoblastic leukemia. Blood 2005; 105: 475258. independent prognostic factor in adult acute lymphoblastic
64 Relling MV, Pui CH, Cheng C, Evans WE. Thiopurine leukemia (ALL): analysis of cytogenetic data from patients treated
methyltransferase in acute lymphoblastic leukemia. Blood 2006; on the Medical Research Council (MRC) UKALLXII/Eastern
107: 84344. Cooperative Oncology Group (ECOG) 2993 trial. Blood 2007;
109: 318997.
65 Stanulla M, Schaeeler E, Flohr T, et al. Thiopurine
methyltransferase (TPMT) genotype and early treatment response 86 Nachman JB, Heerema NA, Sather H, et al. Outcome of
to mercaptopurine in childhood acute lymphoblastic leukemia. treatment in children with hypodiploid acute lymphoblastic
JAMA 2005; 293: 148589. leukemia. Blood 2007; 110: 111215.
66 Relling MV, Rubnitz JE, Rivera GK, et al. High incidence of 87 Schultz KR, Pullen DJ, Sather HN, et al. Risk- and response-based
secondary brain tumours after radiotherapy and antimetabolites. classication of childhood B-precursor acute lymphoblastic
Lancet 1999; 354: 3439. leukemia: a combined analysis of prognostic markers from the
Pediatric Oncology Group (POG) and Childrens Cancer Group
67 Evans WE, Hon YY, Bongaars L, et al. Preponderance of
(CCG). Blood 2007; 109: 92635.
thiopurine S-methyltransferase deciency and heterozygosity
among patients intolerant to mercaptopurine or azathioprine. 88 Moorman AV, Richards SM, Robinson HM, et al. Prognosis of
J Clin Oncol 2001; 19: 2293301. children with acute lymphoblastic leukemia (ALL) and
intrachromosomal amplication of chromosome 21 (iAMP21).
68 Boissel N, Auclerc MF, Lhritier V, et al. Should adolescents with
Blood 2007; 109: 232730.
acute lymphoblastic leukemia be treated as old children or young
adults? Comparison of the French FRALLE-93 and LALA-94 trials. 89 Arico M, Valsecchi MG, Camitta B, et al. Outcome of treatment
J Clin Oncol 2003; 21: 77480. in children with Philadelphia chromosome-positive acute
lymphoblastic leukemia. N Engl J Med 2000; 342: 9981006.
69 de Bont JM, Holt B, Dekker AW, et al. Signicant dierence in
outcome for adolescents with acute lymphoblastic leukemia 90 Gleissner B, Gokbuget N, Bartram CR, et al. Leading prognostic
treated on pediatric vs adult protocols in the Netherlands. relevance of the BCR-ABL translocation in adult acute B-lineage
Leukemia 2004; 18: 203235. lymphoblastic leukemia: a prospective study of the German
Multicenter Trial group and conrmed polymerase chain reaction
70 Ramanujachar R, Richards S, Hann I, et al. Adolescents with
analysis. Blood 2002; 99: 153643.
acute lymphoblastic leukaemia: outcome on UK national
paediatric (ALL97) and adult (UKALLXII/E2993) trials. 91 Mancini M, Scappaticci D, Cimino G, et al. A comprehensive
Pediatr Blood Cancer 2007; 48: 25461. genetic classication of adult acute lymphoblastic leukemia
(ALL): analysis of the GIMEMA 0496 protocol. Blood 2005;
71 Hallbk H, Gustafsson G, Smedmyr B, et al. Treatment outcome
105: 343441.
in young adults and children >10 years of age with acute
lymphoblastic leukemia in Sweden: a comparison between a 92 Pui CH, Gaynon PS, Boyett JM, et al. Outcome of treatment in
pediatric protocol and an adult protocol. Cancer 2006; 107: 155161. childhood acute lymphoblastic leukaemia with rearrangements of
the 11q23 chromosomal region. Lancet 2002; 359: 190915.
72 Barry E, DeAngelo DJ, Neuberg D, et al. Favorable outcome for
adolescents with acute lymphoblastic leukemia treated on 93 Gleissner B, Goekbuget N, Rieder H, et al. CD10- pre-B acute
Dana-Farber Cancer Institute Acute Lymphoblastic Leukemia lymphoblastic leukemia (ALL) is a distinct high-risk subgroup of
Consortium Protocols. J Clin Oncol 2007; 25: 81319. adult ALL associated with a high frequency of MLL aberrations:
results of the German Multicenter Trials for Adult ALL (GMALL).
73 Hilden JM, Dinndorf PA, Meerbaum SO, et al. Analysis of prognostic
Blood 2005; 106: 405456.
factors of acute lymphoblastic leukemia in infants: report on CCG
1953 from the Childrens Oncology Group. Blood 2006; 108: 44151. 94 Breit S, Stanulla M, Flohr T, et al. Activating NOTCH1 mutations
predict favorable early treatment response and long-term outcome
74 Pieters R, Schrappe M, De Lorenzo P, et al. A treatment protocol
in childhood precursor T-cell lymphoblastic leukemia. Blood 2006;
for infants younger than 1 year with acute lymphoblastic
108: 115157.
leukaemia (Interfant-99): an observational study and a multicentre
randomised trial.l. Lancet 2007; 370: 24050. 95 Zhu YM, Zhao WL, Fu JF, et al. NOTCH1 mutations in T-cell
acute lymphoblastic leukemia: prognostic signicance and
75 Larson RA. Management of acute lymphoblastic leukemia in older
implication in multifactorial leukemogenesis. Clin Cancer Res
patients. Semin Hematol 2006; 43: 12633.
2006; 12: 304349.
76 Landau H, Lamanna N. Clinical manifestations and treatment of
96 Cheok MH, Yang W, Pui CH, et al. Treatment-specic changes in
newly diagnosed acute lymphoblastic leukemia in adults.
gene expression discriminate in vivo drug response in human
Curr Hematol Malig Rep 2006; 1: 17179.
leukemia cells. Nat Genet 2003; 34: 8590.
77 Pui C-H. Central nervous system disease in acute lymphoblastic
97 Zaza G, Cheok M, Yang M, et al. Gene expression and
leukemia: prophylaxis and treatment. Hematology Am Soc Hematol
thioguanine nucleotide disposition in acute lymphoblastic
Educ Program 2006: 14246.
leukemia after in vivo mercaptopurine treatment. Blood 2005;
78 Lowe EJ, Pui CH, Hancock ML, et al. Early complications in 106: 177885.
children with acute lymphoblastic leukemia presenting with
98 Kager L, Cheok M, Yang W, et al. Folate pathway gene expression
hyperleukocytosis. Pediatr Blood Cancer 2005; 45: 1015.
diers in subtypes of acute lymphoblastic leukemia and
79 Kadan-Lottick NS, Ness KK, Bhata S, Gurney JE. Survival inuences methotrexate pharmacodynamics. J Clin Invest 2005;
variability by race and ethnicity in childhood acute lymphoblastic 115: 11017.
leukemia. JAMA 2003; 290: 200814.
99 Holleman A, Cheok MH, den Boer ML, et al. Gene-expression
80 Pui CH, Sandlund JT, Pei D, et al. Results of therapy for acute patterns in drug-resistant acute lymphoblastic leukemia cells and
lymphoblastic leukemia in black and white children. JAMA 2003; response to treatment. N Engl J Med 2004; 351: 53342.
290: 200107.

www.thelancet.com Vol 371 March 22, 2008 1041


Seminar

100 Cheng Q, Yang W, Raimondi SC, et al. Karyotypic abnormalities 119 Igarashi S, Manabe A, Ohara A, et al. No advantage of
create discordance of germline genotype and cancer cell dexamethasone over prednisolone for the outcome of standard- and
phenotypes. Nat Genet 2005; 37: 87882. intermediate-risk childhood acute lymphoblastic leukemia in the
101 Panzer-Grmayer ER, Schneider M, Panzer S, et al. Rapid Tokyo Childrens Cancer Study Group L95-14 protocol. J Clin Oncol
molecular response during early induction chemotherapy 2005; 23: 648998.
predicts a good outcome in childhood acute lymphoblastic 120 Pinheiro JP, Boos J. The best way to use asparaginase in childhood
leukemia. Blood 2000; 95: 79094. acute lymphoblastic leukemia: still to be dened? Br J Haematol
102 Dworzak MN, Frschl G, Printz D, et al. Prognostic signicance and 2004; 125: 11727.
modalities of ow cytometric minimal residual disease detection in 121 Duval M, Suciu S, Ferster A, et al. Comparison of Escherichia
childhood acute lymphoblastic leukemia. Blood 2002; 99: 195258. coli-asparaginase with Erwinia-asparaginase in the treatment of
103 Zhou J, Goldwasser MA, Li A, et al. Quantitative analysis of childhood lymphoid malignancies: results of a randomized European
minimal residual disease predicts relapse in children with Organisation for Research and Treatment of Cancer-Childrens
B-lineage acute lymphoblastic leukemia in DFCI ALL Consortium Leukemia Group phase 3 trial. Blood 2002; 99: 273439.
Protocol 95-01. Blood 2007; 110: 160711. 122 Moghrabi A, Levy DE, Asselin B, et al. Results of the Dana-Farber
104 Bruggemann M, Ra T, Flohr T, et al. Clinical signicance of minimal Cancer Institute ALL Consortium Protocol 95-01 for children with
residual disease quantication in adult patients with standard-risk acute lymphoblastic leukemia. Blood 2007; 109: 896904.
acute lymphoblastic leukemia. Blood 2006; 107: 111623. 123 Avramis VI, Sencer S, Periclou AP, et al. A randomized comparison
105 Chauvenet AR, Martin PL, Devidas M, et al. Antimetabolite of native Escherichia coli asparaginase and polyethylene glycol
therapy for lesser-risk B-lineage acute lymphoblastic leukemia of conjugated asparaginase for treatment of children with newly
childhood: a report from Childrens Oncology Group Study P9201. diagnosed standard-risk acute lymphoblastic leukemia: a Childrens
Blood 2007; 110: 110511. Cancer Group study. Blood 2002; 99: 198694.
106 Coustan-Smith E, Sancho J, Behm FG, et al. Prognostic importance 124 Rizzari C, Citterio M, Zucchetti M, et al. A pharmacological study
of measuring early clearance of leukemic cells by ow cytometry in on pegylated asparaginase used in front-line treatment of children
childhood acute lymphoblastic leukemia. Blood 2002; 100: 5258. with acute lymphoblastic leukemia. Haematologica 2006; 91: 2431.
107 Ra T, Gkbget N, Luschen S, et al. Molecular relapse in adult 125 Douer D, Yampolsky H, Cohen LJ, et al. Pharmacodynamics and
standard-risk ALL patients detected by prospective MRD safety of intravenous pegaspargase during remission induction in
monitoring during and after maintenance treatment: data from adults aged 55 years or younger with newly diagnosed acute
the GMALL 06/99 and 07/03 trials. Blood 2007; 109: 91015. lymphoblastic leukemia. Blood 2007; 109: 274450.
108 Coustan-Smith E, Ribeiro RC, Stow P, et al. A simplied ow 126 Wetzler M, Sanford BL, Kurtzberg J, et al. Eective asparagine
cytometric assay identies children with acute lymphoblastic leukemia depletion with pegylated asparaginase results in improved
who have a superior clinical outcome. Blood 2006; 108: 97102. outcomes in adult acute lymphoblastic leukemia: Cancer and
109 Patte C, Auperin A, Michon J, Behrendt H. The Socit Franaise Leukemia Group B Study 9511. Blood 2007; 109: 416467.
dOncologie Pdiatrique LMB89 protocol: highly eective 127 Loh ML, Goldwasser MA, Silverman LB, et al. Prospective analysis
multiagent chemotherapy tailored to the tumor burden and initial of TEL/AML1-positive patients treated on Dana-Farber Cancer
response in 561 unselected children with B-cell lymphomas and Institute Consortium Protocol 95-01. Blood 2006; 107: 450813.
L3 leukemia. Blood 2001; 97: 337079. 128 Nachman JB, Sather HN, Sensel MG, et al. Augmented
110 Woessmann W, Seidemann K, Mann G, et al. The impact of the post-induction therapy for children with high-risk acute
methotrexate administration schedule and dose in the treatment lymphoblastic leukemia and a slow response to initial therapy.
of children and adolescents with B-cell neoplasms: a report of the N Engl J Med 1998; 338: 166371.
BFM Group Study NHL-BFM95. Blood 2005; 105: 94858. 129 Lange BJ, Bostrom BC, Cherlow JM, et al. Double-delayed
111 Lee EJ, Petroni GR, Schier CA, et al. Brief-duration high-intensity intensication improves event-free survival for children with
chemotherapy for patients with small noncleaved-cell lymphoma intermediate-risk acute lymphoblastic leukemia: a report from the
or FAB L3 acute lymphocytic leukemia: results of cancer and Childrens Cancer Group. Blood 2002; 99: 82533.
leukemia group B study 9251. J Clin Oncol 2001; 19: 401422. 130 Hoelzer D, Gokbuget N. New approaches to lymphoblastic
112 Yanada M, Takeuchi J, Sugiura I, et al. High complete remission leukemia in adults: where do we go? Semin Oncol 2000; 27: 54059.
rate and promising outcome by combination of imatinib and 131 Kantarjian H, Thomas D, OBrien S, et al. Long-term follow-up
chemotherapy for newly diagnosed BCR-ABL-positive acute results of hyperfractionated cyclophosphamide, vincristine,
lymphoblastic leukemia: a phase II study by the Japan Adult doxorubicin, and dexamethasone (Hyper-CVAD), a dose-intensive
Leukemia Study Group. J Clin Oncol 2006; 24: 46066. regimen, in adult acute lymphocytic leukemia. Cancer 2004;
113 de Labarthe A, Rousselot P, Huguet-Rigal F, et al. Imatinib 101: 2788801.
combined with induction or consolidation chemotherapy in 132 Linker C, Damon L, Ries C, Navarro W. Intensied and shortened
patients with de novo Philadelphia chromosome-positive acute cyclical chemotherapy for adult acute lymphoblastic leukemia.
lymphoblastic leukemia: results of the GRAAPH-2003 study. Blood J Clin Oncol 2002; 20: 246471.
2007; 109: 140813. 133 Schrappe M, Reiter A, Ludwig WD, et al. Improved outcome in
114 Delannoy A, Delabesse E, Lheritier V, et al. Imatinib and childhood acute lymphoblastic leukemia despite reduced use of
methylprednisolone alternated with chemotherapy improve the anthracyclines and cranial radiotherapy: results of trial
outcome of elderly patients with Philadelphia-positive acute ALL-BFM 90. Blood 2000; 95: 331022.
lymphoblastic leukemia: results of the GRAALL AFR09 study. 134 Pui CH, Relling MV, Evans WE. Is mega dose of methotrexate
Leukemia 2006; 20: 152632. benecial to patients with acute lymphoblastic leukemia?
115 Kantarjian H, Giles F, Wunderle L, et al. Nilotinib in Leuk Lymphoma 2006; 47: 243132.
imatinib-resistant CML and Philadelphia chromosome-positive 135 Nathan PC, Whitcomb T, Wolters PL, et al. Very high-dose
ALL. N Engl J Med 2006; 354: 254251. methotrexate (336 g/m2) as central nervous system preventive
116 Talpaz M, Shah NP, Kantarjian H, et al. Dasatinib in therapy for childhood acute lymphoblastic leukemia: results of
imatinib-resistant Philadelphia chromosome-positive leukemias. National Cancer Institute/Childrens Cancer Group trials
N Engl J Med 2006; 354: 253141. CCG-191P, CCG-134P, and CCG-144P. Leuk Lymphoma 2006,
117 Bostrom BC, Sensel MR, Sather HN, et al. Dexamethasone versus 47: 2488504.
prednisone and daily oral versus weekly intravenous 136 Skrby TV, Anderson H, Heldrup J, et al. High leucovorin doses
mercaptopurine for patients with standard-risk acute lymphoblastic during high-dose methotrexate treatment may reduce the cure rate
leukemia: a report from the Childrens Cancer Group. Blood 2003; in childhood acute lymphoblastic leukemia. Leukemia 2006;
101: 380917. 20: 195562.
118 Mitchell CD, Richards SM, Kinsey SE, et al. Benet of 137 Hunault M, Harousseau JL, Delain M, et al. Better outcome of adult
dexamethasone compared with prednisolone for childhood acute acute lymphoblastic leukemia after early genoidentical allogeneic
lymphoblastic leukaemia: results of the UK Medical Research bone marrow transplantation (BMT) than after late high-dose therapy
Council ALL97 randomized trial. Br J Haematol 2005; 129: 73445. and autologous BMT: a GOELAMS trial. Blood 2004; 104: 302837.

1042 www.thelancet.com Vol 371 March 22, 2008


Seminar

138 Ribera JM, Ortega JJ, Oriol A et al. Comparison of intensive 154 Conter V, Valsecchi MG, Silvestri D, et al. Pulses of vincristine and
chemotherapy, allogeneic or autologous stem cell transplantation as dexamethasone in addition to intensive chemotherapy for children
postremission treatment, for children with very high-risk acute with intermediate-risk acute lymphoblastic leukaemia: a
lymphoblastic leukemia: PETHEMA ALL-93 Trial. J Clin Oncol multicentre randomised trial. Lancet 2007; 369: 12331.
2007; 25: 1624. 155 Hutchinson RJ, Gaynon PS, Sather H, et al. Intensication of
139 Balduzzi A, Valsecchi MG, Uderzo C, et al. Chemotherapy versus therapy for children with lower-risk acute lymphoblastic leukemia:
allogeneic transplantation for very-high-risk childhood acute long-term follow-up of patients treated on Childrens Cancer Group
lymphoblastic leukaemia in rst complete remission: comparison Trial 1881. J Clin Oncol 2003; 21: 179097.
by genetic randomisation in an international prospective study. 156 Gajjar A, Harrison PL, Sandlund JT, et al. Traumatic lumbar
Lancet 2005; 366: 63542. puncture at diagnosis adversely aects outcome in childhood acute
140 Thomas X, Boiron JM, Huguet F, et al. Outcome of treatment in lymphoblastic leukemia. Blood 2000; 96: 338184.
adults with acute lymphoblastic leukemia: analysis of the LALA-94 157 te Loo DM, Kamps WA, Van der Does-van den Berg AV, Van Wering
trial. J Clin Oncol 2004; 22: 407586. ER, de Graaf SS. Prognostic signicance of blasts in the
141 Schrauder A, Reiter A, Gadner H, et al. Superiority of allogeneic cerebrospinal uid without pleiocytosis or a traumatic lumbar
hematopoietic stem cell transplantation compared with puncture in children with acute lymphoblastic leukemia: the
chemotherapy alone in high-risk childhood T-cell acute experience of the Dutch Childhood Oncology Group. J Clin Oncol
lymphoblastic leukemia: results from ALL-BFM 90 and 95. 2006; 24: 233236.
J Clin Oncol 2006; 24: 574249. 158 Lazarus HM, Richards SM, Chopra R, et al. Central nervous system
142 Vey N, Thomas X, Picard C, et al. Allogeneic stem cell involvement in adult acute lymphoblastic leukemia at diagnosis:
transplantation improves the outcome of adults with results from the international ALL trial MRC UKALL XII/ECOG
t(1;19)/E2A-PBX1 and t(4;11)/MLL-AF4 positive B-cell acute E2993. Blood 2006; 108: 46572.
lymphoblastic leukemia: results of the prospective multicenter 159 Pui CH, Cheng C, Leung W, et al. Extended follow-up of long-term
LALA-94 study. Leukemia 2006; 20: 215561. survivors of childhood acute lymphoblastic leukemia. N Engl J Med
143 Kosaka Y, Koh K, Kinukawa N, et al. Infant acute lymphoblastic 2003; 349: 64049.
leukemia with MLL gene rearrangements: outcome following 160 Hijiya N, Hudson MM, Lensing S, et al. Cumulative incidence of
intensive chemotherapy and hematopoietic stem cell secondary neoplasms as a rst event after childhood acute
transplantation. Blood 2004; 104: 352734. lymphoblastic leukemia. JAMA 2007; 297: 120715.
144 Sanders JE, Im HJ, Homeister PA, et al. Allogeneic hematopoietic 161 Barredo JC, Devidas M, Lauer SJ, et al. Isolated CNS relapse of
cell transplantation for infants with acute lymphoblastic leukemia. acute lymphoblastic leukemia treated with intensive systemic
Blood 2005; 105: 374956. chemotherapy and delayed CNS radiation: a pediatric oncology
145 Rocha V, Labopin M, Sanz G, et al. Transplants of umbilical-cord group study. J Clin Oncol 2006; 24: 314249.
blood or bone marrow from unrelated donors in adults with acute 162 Pui CH. Toward optimal central nervous system-directed treatment
leukemia. N Engl J Med 2004; 351: 227685. in childhood acute lymphoblastic leukemia. J Clin Oncol 2003;
146 Eapen M, Rubinstein P, Zhang MJ, et al. Outcomes of 21: 17981.
transplantation of unrelated donor umbilical cord blood and bone 163 Matloub Y, Lindemulder S, Gaynon PS, et al. Intrathecal triple
marrow in children with acute leukaemia: a comparison study. therapy decreases central nervous system relapse but fails to
Lancet 2007; 369: 194754. improve event-free survival when compared with intrathecal
147 Dhedin N, Dombret H, Thomas X, et al. Autologous stem cell methotrexate: results of the Childrens Cancer Group (CCG) 1952
transplantation in adults with acute lymphoblastic leukemia in rst study for standard-risk acute lymphoblastic leukemia, reported by
complete remission: analysis of the LALA-85, -87 and -94 trials. the Childrens Oncology Group. Blood 2006; 108: 116573.
Leukemia 2006; 20: 33644. 164 Pui CH, Jeha S. New therapeutic strategies for the treatment
148 Toyoda Y, Manabe A, Tsuchida M, et al. Six months of maintenance of acute lymphoblastic leukaemia. Nat Rev Drug Discov 2007;
chemotherapy after intensied treatment for acute lymphoblastic 6: 14965.
leukemia of childhood. J Clin Oncol 2000; 18: 150816. 165 Burgess MR, Skaggs BJ, Shah NP, Lee FY, Sawyers CL. Comparative
149 Jacobs SS, Stork LC, Bostrom BC, et al. Substitution of oral and analysis of two clinically active BCR-ABL kinase inhibitors reveals
intravenous thioguanine for mercaptopurine in a treatment the role of conformation-specic binding in resistance.
regimen for children with standard risk acute lymphoblastic Proc Natl Acad Sci U S A 2005; 102: 3395400.
leukemia: a collaborative Childrens Oncology Group/National 166 van Es JH, van Gijn ME, Riccio O, et al. Notch/gamma-secretase
Cancer Institute pilot trial (CCG-1942). Pediatr Blood Cancer 2007; inhibition turns proliferative cells in intestinal crypts and adenomas
49: 25055. into goblet cells. Nature 2005; 435: 95963.
150 Harms DO, Gobel U, Spaar HJ, et al. Thioguanine oers no 167 Jordan CT, Guzman ML, Noble M. Cancer stem cells. N Engl J Med
advantage over mercaptopurine in maintenance treatment of 2006; 355: 125361.
childhood ALL: results of the randomized trial COALL-92. Blood 168 Krivtsov AV, Twomey D, Feng Z, et al. Transformation from
2003; 102: 273640. committed progenitor to leukaemia stem cell initiated by MLL-AF9.
151 Stork LC, Sather H, Hutchinson RJ, et al. Comparison of Nature 2006; 442: 81822.
mercaptopurine (MP) with thioguanine (TG) and IT methotrexate 169 Krause DS, Lazarides K, von Andrian UH, van Etten RA.
(ITM) with IT triples (ITT) in children with SR-ALL: results of Requirement for CD44 in homing and engraftment of
CCG-1952. Blood 2002; 100: 156a. BCR-ABL-expressing leukemic stem cells. Nat Med 2006;
152 Vora A, Mitchell CD, Lennard L, et al. Toxicity and ecacy of 12: 117580.
6-thioguanine versus 6-mercaptopurine in childhood lymphoblastic 170 Jin L, Hope KJ, Zhai Q, Smadja-Joe F, Dick JE. Targeting of CD44
leukaemia: a randomised trial. Lancet 2006; 368: 133948. eradicates human acute myeloid leukemic stem cells. Nat Med
153 Lennard L, Richards S, Cartwright CS, et al. The thiopurine 2006; 12: 116774.
methyltransferase genetic polymorphism is associated with 171 Iwamoto S, Mihara K, Downing JR, Pui CH, Campana D.
thioguanine-related veno-occlusive disease of the liver in children Mesenchymal cells regulate the response of acute lymphoblastic
with acute lymphoblastic leukemia. Clin Pharmacol Ther 2006; leukemia cells to asparaginase. J Clin Invest 2007; 117: 104957.
80: 37583.

www.thelancet.com Vol 371 March 22, 2008 1043


Hematopathology Workshop III: Objectives
Lymphomas Better understand what malignant
lymphomas are and how they behave by
Reviewing and applying lessons exploring how hematopathologists
learned from the lectures related to the diagnose and classify them.
diagnosis, clinical evaluation, treatment Learn the role of hematopathologists as
& understanding of the lymphomas part of a team of physicians taking care of
Understanding the role of the hematopathologist patients with possible lymphomas.
Be prepared for the future the details of
Steven H. Swerdlow, M.D. today will be replaced by new ones
tomorrow (hopefully at least after the
examination)

Plan for workshop session What we arent doing


Trying to turn everyone into an expert
Review evaluation of lymphoid hematopathologist
proliferations especially at nodal sites Examining the role of fine needle aspiration of
lymphoid proliferations
What questions need to be asked when Reviewing 1001 facts object of cases is not to
a lymph node biopsy is performed? memorize every feature or differential diagnosis
shown but to understand the concepts of what
What tools can we use to help answer lymphomas are and how we diagnose and classify
these questions? them.
Interactive case evaluations of four Concentrating on the distinction of lymphomas from
patients with diagnostic biopsies other malignant neoplasms
performed as part of an evaluation for Talking about the latest but experimental molecular
techniques such as gene expression profiling
suspected malignancies

A 60 year old woman comes in to


your office with an enlarged lymph Questions to be answered
node in her neck that has not Lymph node biopsy
(extranodal proliferation)
responded to a course of antibiotics
& has no apparent explanation. Benign Malignant

Specific/non-specific Lymphoma Other (eg metastatic


A 3 cm lymph node is excised. reactive proliferation (B/T/NK) carcinoma)
What decisions need to be rendered?
Hodgkin lymphoma Non-Hodgkin lymphoma
What should be done with the lymph
node? Precise type Precise type

1
What precise type of lymphoma? your options in 2001 Precursor lymphoid neoplasms (WHO 2008)
Precursor B-cell neoplasm Precursor T-cell neoplasms
Precursor B-lymphoblastic leukemia/lymphoma (precursor Precursor T-lymphoblastic leukemia/lymphoma (precursor
B-cell ALL) T-cell ALL)
Mature B-cell neoplasms
Chronic lymphocytic leukemia/small lymphocytic lymphoma
Blastic NK cell lymphoma
Mature T-cell and NK-cell neoplasms
B lymphoblastic leukaemia/lymphoma
B-cell prolymphocytic leukemia
Lymphoplasmacytic lymphoma
T-cell prolymphocytic leukemia
T-cell large granular lymphocytic leukemia
B lymphoblastic leukaemia/lymphoma, NOS
Splenic marginal zone B-cell lymphoma
Hairy cell leukemia
Aggressive NK-cell leukemia
Adult T-cell leukemia/lymphoma
B lymphoblastic leukaemia/lymphoma with recurrent
Plasma cell myeloma
Solitary plasmacytoma of bone
Extranodal NK/T-cell lymphoma, nasal-type cytogenetic abnormalities
Enteropathy-type T-cell lymphoma
? B lymphoblastic leukaemia/lymphoma with t(9:22)
2008
Extraosseous plasmacytoma Hepatosplenic T-cell lymphoma
Extranodal marginal zone B-cell lymphoma of mucosa-
(q34;q11.2); BCR/ABL
WHO
Subcutaneous panniculitis-like T-cell lymphoma
associated lymphoid tissue (MALT-lymphoma) Mycosis fungoides
Nodal marginal zone B-cell lymphoma Sezary syndrome B lymphoblastic leukaemia/lymphoma with t(v;11q23); MLL
Follicular lymphoma
Mantle cell lymphoma
Primary cutaneous anaplastic large cell lymphoma rearranged
Peripheral T-cell lymphoma, unspecified
Diffuse large B-cell lymphoma Angioimmunoblastic T-cell lymphoma B lymphoblastic leukaemia/lymphoma with t(12;21)
Mediastinal (thymic) large B-cell lymphoma
Intravascular large B-cell lymphoma
Anaplastic large cell lymphoma (p13;q22); TEL/AML1(ETV6-RUNX1)
T-cell proliferation of uncertain malignant potential
Primary effusion lymphoma Lymphomatoid papulosis B lymphoblastic leukaemia/lymphoma with hyperdiploidy
Burkitt lymphoma/leukemia
B-cell proliferations of uncertain malignant potential B lymphoblastic leukaemia/lymphoma with hypodiploidy
Lymphomatoid granulomatosis (Hypodiploid ALL)
Post-transplant lymphoproliferative disorder, polymorphic
B lymphoblastic leukaemia/lymphoma with
Hodgkin lymphoma t(5;14)(q31;q32)(IL3-IGH)
Nodular lymphocyte predominant Hodgkin lymphoma
Classical Hodgkin lymphoma B lymphoblastic leukaemia/lymphoma with
Nodular sclerosis classical Hodgkin lymphoma t(1;19)(q23;p13.3); E2A-PBX1;TCF3/PBX1)
Lymphocyte-rich classical Hodgkin lymphoma
Mixed cellularity classical Hodgkin lymphoma T lymphoblastic leukaemia/lymphoma
Lymphocyte depleted classical Hodgkin lymphoma

Mature B-cell neoplasms (WHO 2008) Mature T-


T-cell & NK-
NK-cell neoplasms (WHO 2008)
Chronic lymphocytic leukaemia/small Mantle cell lymphoma
lymphocytic lymphoma Diffuse large B-cell lymphoma (DLBCL), T-cell prolymphocytic leukaemia Mycosis fungoides
B-cell prolymphocytic leukaemia NOS T-cell large granular lymphocytic Szary syndrome
Splenic marginal zone lymphoma T-cell/histiocyte rich large B-cell lymphoma
leukaemia
Primary DLBCL of the CNS Primary cutaneous CD30
Hairy cell leukaemia Primary cutaneous DLBCL, leg type Chronic lymphoproliferative positive T-cell
Splenic lymphoma/leukaemia,
unclassifiable
EBV+ DLBCL of the elderly disorder of NK-cells lymphoproliferative disorders
Splenic diffuse red pulp small B-cell lymphoma DLBCL associated with chronic Aggressive NK cell leukaemia Lymphomatoid papulosis
Hairy cell leukaemia-variant inflammation
Lymphomatoid granulomatosis Systemic EBV positive T-cell Primary cutaneous anaplastic
Lymphoplasmacytic lymphoma large cell lymphoma
Waldenstrm macroglobulinemia Primary mediastinal (thymic) large B-cell lymphoproliferative disease of
Heavy chain diseases lymphoma childhood Primary cutaneous gamma-delta
Alpha heavy chain disease Intravascular large B-cell lymphoma Hydroa vacciniforme-like T-cell lymphoma
Gamma heavy chain disease ALK positive large B-cell lymphoma
Mu heavy chain disease lymphoma Primary cutaneous CD8 positive
Plasmablastic lymphoma aggressive epidermotropic
Plasma cell myeloma
Large B-cell lymphoma arising in HHV8-
Adult T-cell
Solitary plasmacytoma of bone leukaemia/lymphoma cytotoxic T-cell lymphoma
associated multicentric Castleman
Extraosseous plasmacytoma disease Extranodal NK/T cell lymphoma, Primary cutaneous CD4 positive
Extranodal marginal zone lymphoma of Primary effusion lymphoma nasal type small/medium T-cell lymphoma
mucosa-associated lymphoid tissue Burkitt lymphoma Peripheral T-cell lymphoma,
(MALT lymphoma) Enteropathy-associated T-cell
Nodal marginal zone lymphoma
B-cell lymphoma, unclassifiable, with
lymphoma NOS
features intermediate between diffuse
Paediatric nodal marginal zone lymphoma large B-cell lymphoma and Burkitt Hepatosplenic T-cell lymphoma Angioimmunoblastic T-cell
Follicular lymphoma lymphoma lymphoma
Paediatric follicular lymphoma B-cell lymphoma, unclassifiable, with
Subcutaneous panniculitis-like T-
cell lymphoma Anaplastic large cell lymphoma,
Primary cutaneous follicle centre features intermediate between diffuse ALK positive
lymphoma large B-cell lymphoma and classical
Hodgkin lymphoma Anaplastic large cell lymphoma,
ALK negative

Why so many questions? Remember


We dont want to confuse lymphomas with
either benign/reactive proliferations or other Lymphomas are malignant neoplasms of cells of
tumors the immune system.
Different types of lymphomas must be B cell tumors, T cell tumors, NK cell tumors
distinguished because Lymphomas
Very different behavior (death in a year, Are generally destructive proliferations that
versus survivals for over a decade partially or totally destroy the normal tissues
sometimes with little therapy) where they are growing.
Very different therapies (watch and wait, Show varying degrees of recapitulation of the
antibiotics, targeted therapies like antibody normal cells they most closely resemble
to specific B-cell antigens, non-aggressive useful in thinking about recognizing and
chemotherapies, very aggressive classifying lymphomas as well as understanding
chemotherapies, stem cell transplantation) how they might grow.

2
How do we assess whether a Is this lymph node proliferation
lymphoid proliferation is destructive destructive?
Sinuses Mantle zones
and how it relates to normal
lymphoid tissue compartments?
GC
Look at glass slides GC
Destruction of normal architectural features
of the lymph node or other tissues
GC
Look at the growth pattern and cytologic
appearance of the cells NO, we can see the normal nodal
Use additional studies especially structures.
immunophenotypic studies

Is this lymph node proliferation Other features of lymphomas in general that


(1) help us understand what lymphomas are
destructive? and (2) we may need to call upon to make a
precise diagnosis
Yes, this is a lymphoma. Lymphomas are more homogeneous
proliferations than those present in reactive
lymphoid proliferations
What does homogeneous mean to you (and the
patient)?
Lots of cells that are all the same not what
you expect in a normal/reactive immune
response
How do we assess if the cells are all
the same ie, homogeneous?

Which biopsy is from the patient


LOOK at the glass slides with lymphoma?
Are the cells all of one kind
or are there lots of different
kinds?
BUT some lymphomas include
many reactive lymphoid cells
(recapitulating normal immune
reactions)

Drug reaction

3
Use phenotypic studies (flow Flow cytometric immunophenotypic studies
cytometry or tissue sections) were performed on a cell suspension of a
Is there a more homogeneous population of cells lymph node biopsy. Is this proliferation from
marking in the same fashion than we see in a reactive hyperplasia or from a malignant
reactive settings? lymphoma? And the answer is.
lots of cells with same antigenic expression,
including sometimes combinations of antigens >>>>>>
>>>>>>
that are either not normally found or found on + we
Therefore infer

cells in small numbers OR a more mixed
population
that there is a
Findings that infer monoclonality one clone B-cell
monoclonal
(homogeneous) versus numerous clones .This
population.
With B-cells look for whether the B-cells
supports
the
include mix of kappa positive and lambda +
positive cells or a markedly skewed ratio of
presence a
.
lymphoma.

Other assessments of clonality Lymphomas, but not reactive


to demonstrate abnormal proliferations, may show acquired
homogeneity of the proliferating somatic genetic abnormalities
cells How will these be documented?
Molecular analyses (Southern blot
Molecular studies analysis or PCR)
Southern blot analysis or Cytogenetic studies
PCR studies Classical G-banded karyotypes
Cytogenetic studies FISH studies using probes to specific
portions of chromosomes

Is this karyotype from a reactive lymph node or a


lymphoma? Why might it be useful to document
Same finding in 20/20 cells these abnormalities?
To help establish a neoplastic diagnosis
Clonal cytogenetic Our emphasis up to this point
abnormality typical for To help determine the type of malignant
a translocation lymphoma.
Critical to classify the malignant lymphomas
between c-myc
Lets look back at some of the abnormalities
oncogene and weve seen in the lymphomas and learn how
immunoglobulin heavy much we can squeeze out of them and what
chain gene. NOT they can teach us about malignant lymphomas.
expected in a reactive These are just examples!
hyperplasia.

4
Histopathology Sheet of large transformed
lymphoid cells growing diffusely
Limits differential diagnosis to the
lymphomas composed of a homogenous
population of transformed cells
What are transformed cells?
Lymphoid cells that are analagous to
lymphocytes that have been
stimulated by a mitogen.
Dividing (vs resting/dormant)
See example of transformed vs non-
transformed cells in a normal germinal
center

Germinal (follicular) center Neoplastic diffuse proliferation of


transformed cells
Lymphomas like this, as you would predict, are
Transformed cell among the aggressive/non-indolent malignant
(centroblast) lymphomas.
Need to be further classified for therapeutic &
prognostic purposes will require other studies.
Major lymphomas in the differential diagnosis
Diffuse large B-cell lymphoma
eg, if CD20+ would probably get anti-CD20 antibody therapy
together with combination chemotherapy
Non-transformed cells (cleaved Some of the T-cell lymphomas
cells/centrocytes) Anti-CD20 therapy would be inappropriate here.
On average, would be considered more aggressive than
DLBCL.

How does this help in beginning to


Therefore, the differential diagnosis
classify a lymphoma? is narrowed to the B-cell lymphomas
Light chain class restriction infers that express surface
monoclonality (although doesnt prove
+ it).
immunoglobulin Still a long list but
Large monoclonal B-cell populations as shorter than a list
you see here are strong support for a of ALL the
B-cell lymphoma (although not absolute lymphomas (or
proof) ALL the B-cell
lymphomas)
+

5
Mature B-cell neoplasms (WHO 2008)
Chronic lymphocytic leukaemia/small
lymphocytic lymphoma


Mantle cell lymphoma
Diffuse large B-cell lymphoma (DLBCL),
How does this help to classify a
B-cell prolymphocytic leukaemia NOS


Splenic marginal zone lymphoma
Hairy cell leukaemia


T-cell/histiocyte rich large B-cell lymphoma
Primary DLBCL of the CNS
lymphoma?
Primary cutaneous DLBCL, leg type
Splenic lymphoma/leukaemia,
unclassifiable
EBV+ DLBCL of the elderly This translocation is consistent with a
DLBCL associated with chronic


Splenic diffuse red pulp small B-cell lymphoma
Hairy cell leukaemia-variant inflammation c-myc/IgH translocation if necessary
Lymphoplasmacytic lymphoma Lymphomatoid granulomatosis
Waldenstrm macroglobulinemia Primary mediastinal (thymic) large B-cell can further prove with cytogenetic
lymphoma
Heavy chain diseases
Alpha heavy chain disease Intravascular large B-cell lymphoma FISH analysis or molecular studies.
Gamma heavy chain disease ALK positive large B-cell lymphoma

Mu heavy chain disease
Plasma cell myeloma
Plasmablastic lymphoma So, once again
C-myc/IgH will need to
translocations integrate
are a feature
Large B-cell lymphoma arising in HHV8-


Solitary plasmacytoma of bone
Extraosseous plasmacytoma
associated multicentric Castleman thisBurkitt
of important ancillary
lymphoma study
(very with other
rapidly
disease
Extranodal marginal zone lymphoma of Primary effusion lymphoma findings extremely
dividing to arrive ataggressive
a precise diagnosis.
type of
mucosa-associated lymphoid tissue Burkitt lymphoma

(MALT lymphoma)
Nodal marginal zone lymphoma
B-cell lymphoma, unclassifiable, with lymphoma) but also seen in some
features intermediate between diffuse
Paediatric nodal marginal zone lymphoma large B-cell lymphoma and Burkitt other aggressive B-cell lymphomas
Follicular lymphoma lymphoma
Paediatric follicular lymphoma B-cell lymphoma, unclassifiable, with that might get treated differently.
Primary cutaneous follicle centre features intermediate between diffuse
lymphoma large B-cell lymphoma and classical
Hodgkin lymphoma

Mystery case #1

Questions ? 35 year old female presents with a neck


mass, thought to be a salivary gland
tumor.
The mass is excised.

Sinuses Mantle zones

GC
GC

What tissue is this? GC


What features are present to help decide if this is
benign or malignant? (ie, does this illustrate features Monocytoid B-cell areas
that are those of a lymphoma?) another B-cell compartment

6
So, looks benign, what about the See heterogeneous population of cells,
supporting reactive hyperplasia what else
flow cytometric immunophenotypic
do you notice here?
studies
15% kappa positive B-cells & 8% lambda
positive B-cells
Infers polyclonal B-cells, is a strike against a
B-cell lymphoma (but not an absolute strike).
75% heterogeneous T-cells
The expected finding in a reactive lymph
node CMV inclusion
Are you then done? If uncertain, how might you prove what it
is?
No, try to make a more precise diagnosis

Immunohistochemistry (stain with


an antibody to CMV brown is Diagnosis
positive) Reactive lymphoid hyperplasia associated
with CMV infection.
What does this mean for the patient?
CMV mononucleosis
Benign disorder that is a lot like EBV-
associated infectious mononucleosis;
however, tends to occur in older individuals,
less pharyngitis.

Mystery case #2

Questions ? A 55 year old man noted a lump in his


neck while shaving. He felt perfectly well
otherwise. The mass was excised and
sent fresh to Hematopathology.
The grossly worrisome biopsy was
processed so that flow cytometric
immunophenotypic and cytogenetic
studies could be performed. Material was
also saved for potential molecular studies.

7
Are we concerned about a malignant See homogeneous follicle also worrisome
lymphoma here, or is this another type of
reactive hyperplasia? Why?

Yes crowded follicles throughout


architectural destruction.

See homogeneity of centrocytes (cleaved Do these flow cytometric studies


cells) in the follicles with few centroblasts confirm your histologic impression,
(transformed germinal center cells) & if so, why?
supports that this is a follicular LYMPHOMA
not hyperplasia. 75% kappa positive B-cells & 2% lambda positive
B-cells
Infers monoclonal B-cells, supports the
diagnosis of a B-cell lymphoma
B-cells are CD5-, CD10+.
Helpful since this is the phenotype of normal
germinal center cells and also of most follicular
lymphomas
20% heterogeneous T-cells
B-cell lymphomas can have variable numbers of
admixed reactive T-cells

NEXT decision to make after deciding that


What if you still werent sure or
you are dealing with a follicular lymphoma
there were problems with these (not being stressed today to avoid stress)
studies? what grade is the follicular lymphoma since
Try to demonstrate the cytogenetic those with numerous centrocytes behave
abnormality found in most follicular differently than those with numerous
lymphomas IgH/BCL2 translocation centroblasts (as you might expect).
(many techniques could be used) Based on number of centrocytes in the
FISH study on right Normal t(14;18) neoplastic follicles (grade 1-2 & 3).
shows that there is a So diagnosis in this case is:
reciprocal
translocation Follicular lymphoma, grade 1-2 of
between one chr. 14
and one chr. 18
3 with a follicular growth pattern

8
Features of grade 1-2 follicular
Staging non-Hodgkin lymphomas
lymphomas
Typically indolent behavior but widely Good physical examination.
disseminated in most patients and Bone marrow examination with aspirates
traditionally considered to be incurable. and biopsies (because the lymphoma
What procedures would be typically done might not aspirate well)
to stage a patient with a non-Hodgkin CT scans looking for adenopathy and
lymphoma and rule out the possibility that other masses. PET/CT scans are also
the disease is more localized than in most used to look for evidence of lymphoma.
patients?

Mystery case #2b Need to be concerned about the


possibility of transformation
Our patient had bone marrow involvement (progression of the lymphoma from
by his follicular lymphoma and was one with many centrocytes (the
watched and then received some
chemotherapy when he became more
non-dividing cells in the germinal
symptomatic. centers) to one with many
4 years later, however, he developed a centroblasts. The latter cases also
rapidly enlarging lymph node in his groin. often lose their follicular
What should you suspect? architecture.
The inguinal lymph node in this patient was biopsied.

DIFFUSE ARCHITECTURAL
EFFACEMENT supports Same or different?
malignant and is already VERY
worrisome for transformation.

So, what do you think?


Benign? Follicular lymphoma
(again)? Transformation?
How will you decide?

9
Still need to document B-cell origin & if
considering transformation can look for
similarities to original lymphoma
Flow cytometric immunophenotypic
studies showed a CD20+ CD10+ kappa
monoclonal B-cell population. (as before)
If this is transformation of the prior indolent
lymphoma, will the t(14;18), ie the
IgH/BCL2 translocation still be present?
YES a clonal abnormality wouldnt be
expected to go away. There may now be
additional cytogenetic abnormalities.
Different numerous transformed cells

Conclusion transformation of the If you had a small specimen & the flow
patients grade 1-2 follicular cytometric studies were ambiguous or if
you were dealing with a T-cell
lymphoma to a diffuse large B-cell
proliferation, what technique could you
lymphoma turn to in order to help establish the
What is the expected behavior? presence of a monoclonal lymphoid
Transformed indolent lymphomas tend to population and assess its lineage?
behave in an aggressive fashion and
would be considered worse than a de Genotypic studies (Southern blot analysis
novo diffuse large B-cell lymphoma. or PCR studies of the IgH or T-cell
receptor genes)

Mystery case #3
18 year old male presented with chest
discomfort. A large anterior mediastinal

Questions ? mass was identified on CT scan as well as


a pleural effusion. For uncertain reasons
a mediastinal biopsy was performed.
What type of lymphoid neoplasm would
you suspect in these circumstances?
T-lymphoblastic leukemia/lymphoma
Hodgkin lymphoma
Mediastinal large B-cell lymphoma
Other

10
What do you suspect now? Monotonous sheet of cells
Certainly doesnt look like a normal lymph certainly looks neoplastic.
node/thymus or other normal tissue.

Does it look slightly follicular/nodular??? Need a closer look at the cells.

Medium sized, dispersed


Blastic vs transformed cells
chromatin (very
We are using blastic to refer to precursor
fine/delicate, like a fine (primitive) cells including lymphoblasts and
haze), nucleoli not myeloid blasts (the types of cells seen in
prominent, many mitotic acute leukemias).
figures. FEATURES we Transformed cells are mature cells that
call blastic. have been stimulated in some fashion
analagous to what happens with mitogen
stimulation of normal lymphocytes.

How do you characterize these cells?

Limited differential diagnosis for


What is the next step in this case?
blastic-appearing neoplasms
Decide what type of blastic appearing Some non-hematopoietic/lymphoid tumors
neoplasm this represents. such as small cell carcinomas
What is the first (and probably only B-lymphoblastic leukemia/lymphoma
required tool) that we will turn to? T-lymphoblastic leukemia/lymphoma
Immunophenotypic studies flow Acute myeloid leukemias/myeloid sarcomas
cytometry if we received the specimen
Blastic-appearing more mature B-cell and T-
fresh, otherwise paraffin section
cell neoplasms
immunostains.

11
Flow cytometric immunophenotypic studies
3% kappa positive B-cells & 2% lambda positive B- Remainder of FCIPS results
cells
Infers small polyclonal mature B-cell population Surface CD3 negative but cytoplasmic
but doesnt exclude precursor B-cells and certainly CD3 positive, CD2 positive, CD5 positive,
doesnt make this benign
CD7 positive
95% T-cells
Given the appearance of this biopsy (a very CD1a positive
monotonous cell population) suggests that this is a TdT positive
T-cell neoplasm.
What should we now conclude?
What else do you want to know about the phenotype
Are there any phenotypic aberrancies (to further
support that the phenotype reflects a neoplastic Diagnosis: T-lymphoblastic leukemia/
population) and, given the appearance of the lymphoma (T-ALL)
neoplasm, are there features to support that these
are T-lymphoblasts?

Leukemia vs lymphoma? T-lymphoblastic leukemia/lymphoma


Although the distinction gets very blurry (T-ALL)
sometimes, leukemias are
hematopoietic/lymphoid neoplasms that tend to
be marrow-based whereas lymphomas tend to Most commonly occurs in teen-aged
originate in lymph nodes and at other males who often present with a
extramedullary sites. mediastinal mass. There may be an
Some lymphoid neoplasms in particular, associated neoplastic pleural effusion
however, can have either a leukemic OR a
lymphomatous presentation. Some of these (another way to attempt to make the
have hybrid names (like in this case). diagnosis).
By convention, a patient as presented here Aggressive disorder that will get treated
would have a bone marrow examination. If it
has >25% blasts, the patient would be
like an acute leukemia (even if a
considered to have T-lymphoblastic leukemia. lymphoma).

Mystery case #4
24 year old female presents with a very
large mediastinal mass, fever and night
sweats. She is also found to have
Questions ? supraclavicular lymphadenopathy on
physical examination.
A supraclavicular lymph node biopsy is
performed. What are you suspecting?
Major differential diagnosis includes nodular
sclerosis Hodgkin lymphoma & mediastinal
large B-cell lymphoma (both of which
typically occur in young females). Presence
of other adenopathy more like HL. Much
less likely to be another T-lymphoblastic
neoplasm. Other disorders are also
possible.

12
Gross lymph node specimen what do you observe? Microscopic section what do you observe?

Architectural effacement
Fibrous thickened capsule
Large nodules surrounded by
dense fibrous bands

Are you thinking benign or malignant?


Very abnormal lymph node with large
fleshy nodules surrounded by white
fibrous bands in some areas.

The collagen bands are birefingent under polarized Higher magnification what do you observe?
light. This is one of the features of nodular sclerosis
Hodgkin lymphoma (and some other lymphomas as
well as other neoplasms). Many small lymphocytes
Eosinophils

What appears to be a
Reed-Sternberg cell

Lacunar type cells


binucleate/multinucleate
cells sitting in artifactually
cleared out spaces
(lacunes) another
feature of NSHL

13
So, what are we thinking?
And in addition
Looks like classical Hodgkin lymphoma of
nodular sclerosis type, why? Morphologic features of the nodular
Looks like a lymphoma architectural sclerosis subtype of classical Hodgkin
destruction, apparent lymphoid lymphoma.
proliferation We worry about HL-mimics non-Hodgkin
Morphologically has the features we lymphomas, other neoplasms or even
associate with classical Hodgkin sometimes reactive proliferations.
lymphoma What can we do to help convince
ourselves further that we are dealing with
Mixed inflammatory background classical Hodgkin lymphoma?
Variable numbers of neoplastic Reed-
Sternberg cells & R-S variants

Ask Dr. Cant. Immunohistochemistry


Hodgkin?
Use antibodies on the histologic sections
to look for some antigens to be expressed
and others not to be expressed.
Flow cytometric studies usually dont
characterize the neoplastic cells but may
help in cases of non-Hodgkin lymphomas
that might mimic Hodgkin lymphomas by
demonstrating aberrant B- or T-cell
populations.

Classical Reed-Sternberg cells


Not our object today to teach what Usually lack surface T-cell and B-cell
the phenotype of classic RS cells markers as well as the common leukocyte
antigen (a somewhat oversimplified
is, but to recognize how we will statement)
further establish the diagnosis (and Usually do express the B-cell marker PAX-5
how it teaches us more about the Usually do not express certain B-cell
disease). transcription factors (and do not express
immunoglobulin)
Usually express CD15 and CD30 antigens.
This constellation of markers is unusual in
other lymphomas (but still requires
judgements to be made).

14
Classical Reed-Sternberg cells
Usually lack surface T-cell and B-cell CD15+ CD30+
markers as well as the common leukocyte
antigen (a somewhat oversimplified
statement)
Usually do express the B-cell marker PAX-5
Usually do not express certain B-cell
Attranscription factorsof(and
least most cases do not
Hodgkin express
lymphoma
immunoglobulin)
are B-cell neoplasms composed of very
abnormal
Usually express CD15
B-cells that andmake
dont CD30 antigens.
immunoglobulin butof
This constellation can still survive.
markers is unusual in
other lymphomas (but still requires
judgements to be made).

Diagnosis
Nodular sclerosis classical Hodgkin lymphoma
What needs to be done for the patient?
The patient must be staged how
disseminated is the disease and does the
patient have B symptoms (fever, night sweats,
Questions ?
weight loss)? More important than precise
subtype of Hodgkin lymphoma.
Why do we emphasize Hodgkin lymphoma
here?
Considered distinct from NHL in terms of how it
spreads (contiguous vs. non-contiguous
spread) and how it is treated. Good survivals
with proper therapy.

Parting messages
Malignant lymphomas are a diverse group of
neoplasms that must be distinguished from
reactive proliferations, other neoplasms and
from each other.
These decisions, made by hematopathologists,
can be very difficult.
We rely not only on conventional histopathology
but a wide variety of ancillary tools.
The information we obtain is useful not only
diagnostically, but also because it continues to
teach us about the disease entities that we are
diagnosing and provides clues sometimes
about novel therapies.

15
TOPIC: CASE CONFERENCE III- Hematologic Malignancies
(SMALL GROUP)

DATE: January 16, 2008

TIME: 10:00 AM-12:00

Student Groups Small Group Rooms Facilitators

Group 65&66 Rooms 502 & 503 Patricia Kropf , MD

Group 67&68 Rooms 504 & 505 Rajesh Sehgal, MD

Groups 69&70 Rooms 506 & 507 Ahmad Tarhini, MD,Msc

Groups 71&72 Rooms 508 & 509 Enrico Novelli, MD

Groups 73&74 Rooms 510 & 511 Robert Redner, MD

Groups 75&76 Rooms 512 & 513 Wassim McHayleh, MD

Groups 77&78 Rooms 514 & 515 Roy Smith, MD

Groups 79&80 Rooms 516 & 517 Peter Shaw, MD

1
Case # 1
A 15 year old male is seen by you in the Emergency Department at Childrens where he
was referred due to excessive bruising.

The patient had been in excellent health one week previously when he began to notice
bruising on his legs without associated trauma. The bruising extended to his arms over
the ensuing days. On the day of evaluation he noted both gingival bleeding and a severe
nosebleed that required 3 hours to stop.

On taking his history he denies any recent infections, fevers or fatigue. He has been in
good health and runs cross country on the school track team. He denies use of alcohol or
tobacco. His family history is unremarkable. On review of systems, he denied any
headaches or neurologic symptoms.

Vital signs at the time of evaluation were normal. Remarkable findings noted on exam
include pale conjunctivae and petechiae on his soft palate. In addition, bilateral cervical
and axillary adenopathy are present; these nodes are 1-2 cm in diameter, firm and non-
tender. A spleen tip is palpable 3cm below the left costal margin. Scrotal and testicular
exam was normal. His stool is positive for occult blood and there are numerous petechial
and purpuric areas on his arms and legs. Neurologic examination have intact cranial
nerves. No papilledema is noted.

Initial laboratory test results obtained in the ED include:

Result [Normal]
Hemoglobin (Hb) 10 g/dL [13-15 g/dL]
Hematocrit (HCT) 30% [38-42%]
Mean Corpuscular Volume (MCV) 89fL [83-97fL]
RDW 12% [12-15%]
White blood cells 80 x 109/L [4-11 x 109/L]
Neutrophils 6% [44-77%]
Lymphocytes 10% [13-44%]
Monocytes 4% [4-12%]
Blasts 80% [0%]
Platelets 8 [150-350 X 109/L]

He is admitted to the hospital and a bone marrow aspirate and biopsy is performed.
Samples of the aspirate are sent for flow cytometry, cytogenetics and molecular
diagnostics. The studies reveal the following:

Aspirate: blasts cells represent 80% of all of the mononuclear cells present; these
cells are positive on immunostains for terminal deoxynucleotidyltransferase (TDT)
Biopsy: the marrow is hypercellular (90%) with decreased megakaryocytes
Flow cytometry: there is a population of cells that are CD19+, CD10+ and CD33-
CD13-

2
Cytogenetics: 20 metaphases are scored with 80% of the cells demonstrating a
t(9,22).
Molecular testing: PCR detects the bcr/abl oncoprotein in 60% of 10 x 109/L cells
tested

Therapy is instituted within the next 48 hours.

Questions for discussion:

Question # 1:What are the remarkable findings on physical examination?


Why were the eye, testicular and neurologic examinations
mentioned? What are sanctuary sites?

Question # 2: Based on the history and laboratory results what is your


differential diagnosis?

Question # 3 : What are the principles of treatment for your patient?

3
Case # 2
A 55 yo male is admitted to your service from the Emergency Department where he
presented with severe, sudden back pain and numbness in his legs.

The patient has been in relatively good health all of his life until one month ago. At that
time he noted persistent mid-thoracic back pain. There was no report of any prior
trauma. The initial pain was characterized as a mild discomfort but became suddenly
worse today. Associated with this change was the development of numbness in his
legs. He denies any problems with walking or urination.

Past medical history is positive for ischemic heart disease and diabetes. He has had a
coronary artery bypass two years previously. Current medications include insulin,
metoprolol, lovastatin and aspirin. He has no drug allergies. Social history is remarkable
for smoking 2 packs of cigarettes daily for 30 years and drinking six beers daily. He
works as a mechanic. He is divorced and has no children.
Family history: father died at age 50 of an MI. Mother is alive at age 75 with diabetes
and a stroke. Two brothers have died of lung cancer. Two additional siblings are alive,
one of whom has diabetes. Review of systems is otherwise negative.

On physical examination his vital signs are unremarkable. There is no adenopathy or


hepatosplenomegaly present. There is exquisite tenderness to percussion of his thoracic
vertebrae at T5-7. Rectal examination reveals diminished rectal tone. The neurologic
examination demonstrates the following: oriented X3; cerebellar and cranial nerve
testing are normal; motor strengths are symmetrically 5/5 in all extremities; diminished
sensation to touch is noted in both lower extremities and a sensory level is present at the
umbilicus; DTR's are +3 in the upper extremities and +4 in the lower extremities.

Laboratory data obtained in the Emergency Department include:

Result [Normal]
Hemoglobin (Hb) 6 g/dL [13-17 g/dL]
Hematocrit (HCT) 18% [38-49%]
Mean Corpuscular Volume (MCV) 89fL [83-97fL]
RDW 12% [12-15%]
White blood cells 3 x 109/L [4-11 x 109/L]
Neutrophils 66% [44-77%]
Lymphocytes 27% [13-44%]
Monocytes 7% [4-12%]
Platelets 100 x 109/L [150-350 X 109/L]

Creatinine 4 mg/dL 0.5-1.4


Calcium 12 mg/dL 8.5-10.5
Total protein 10 g/dL 6.3-7.7

4
Albumin 2.5 g/dL 3.5-5
Uric acid 15 mg/dL 2.5-8.5

After completing the history and physical several additional laboratory and radiographic
tests are requested and a procedure is scheduled.

Questions for discussion:

Question # 1: What additional laboratory, radiographic tests and


procedures are requested?

Question # 2: What is your differential diagnosis?

Question # 3: What is monoclonal gammopathy of unknown significance?

Question # 4: What are the signs and symptoms of multiple myeloma?

Question # 5: What are the principles of treatment for your patient?

S-ar putea să vă placă și